From pete at mollysrevenge.com Fri Jul 1 00:05:13 2011 From: pete at mollysrevenge.com (Pete) Date: Thu, 30 Jun 2011 21:05:13 -0700 Subject: Inserting text into a field Message-ID: - combobox with text choices - field with text in it and the cursor somewhere in the middle of the text - select an entry from the combobox - how do I insert the combobox entry into the text field, starting where the cursor is positioned? Pete Molly's Revenge From jacque at hyperactivesw.com Fri Jul 1 00:29:52 2011 From: jacque at hyperactivesw.com (J. Landman Gay) Date: Thu, 30 Jun 2011 23:29:52 -0500 Subject: Inserting text into a field In-Reply-To: References: Message-ID: <4E0D4D40.5030302@hyperactivesw.com> On 6/30/11 11:05 PM, Pete wrote: > - combobox with text choices > - field with text in it and the cursor somewhere in the middle of the text > - select an entry from the combobox > - how do I insert the combobox entry into the text field, starting where the > cursor is positioned? put the label of me into the selection -- Jacqueline Landman Gay | jacque at hyperactivesw.com HyperActive Software | http://www.hyperactivesw.com From kray at sonsothunder.com Fri Jul 1 00:32:44 2011 From: kray at sonsothunder.com (Ken Ray) Date: Thu, 30 Jun 2011 23:32:44 -0500 Subject: Combobox In-Reply-To: Message-ID: > Just to clarify, are you saying that option menus on Macs are different than > on Windows in general, or just LC option menus? It's Macs vs. Windows... LC is just adopting the look and feel of the host platform. Ken Ray Sons of Thunder Software, Inc. Email: kray at sonsothunder.com Web Site: http://www.sonsothunder.com/ From pete at mollysrevenge.com Fri Jul 1 01:20:28 2011 From: pete at mollysrevenge.com (Pete) Date: Thu, 30 Jun 2011 22:20:28 -0700 Subject: Combobox In-Reply-To: References: Message-ID: Got it, thanks. Pete Molly's Revenge On Thu, Jun 30, 2011 at 9:32 PM, Ken Ray wrote: > > > Just to clarify, are you saying that option menus on Macs are different > than > > on Windows in general, or just LC option menus? > > It's Macs vs. Windows... LC is just adopting the look and feel of the host > platform. > > > Ken Ray > Sons of Thunder Software, Inc. > Email: kray at sonsothunder.com > Web Site: http://www.sonsothunder.com/ > > > > _______________________________________________ > use-livecode mailing list > use-livecode at lists.runrev.com > Please visit this url to subscribe, unsubscribe and manage your > subscription preferences: > http://lists.runrev.com/mailman/listinfo/use-livecode > > From pete at mollysrevenge.com Fri Jul 1 02:25:07 2011 From: pete at mollysrevenge.com (Pete) Date: Thu, 30 Jun 2011 23:25:07 -0700 Subject: Inserting text into a field In-Reply-To: <4E0D4D40.5030302@hyperactivesw.com> References: <4E0D4D40.5030302@hyperactivesw.com> Message-ID: Thanks. Pete Molly's Revenge On Thu, Jun 30, 2011 at 9:29 PM, J. Landman Gay wrote: > On 6/30/11 11:05 PM, Pete wrote: > >> - combobox with text choices >> - field with text in it and the cursor somewhere in the middle of the text >> - select an entry from the combobox >> - how do I insert the combobox entry into the text field, starting where >> the >> cursor is positioned? >> > > put the label of me into the selection > > -- > Jacqueline Landman Gay | jacque at hyperactivesw.com > HyperActive Software | http://www.hyperactivesw.com > > ______________________________**_________________ > use-livecode mailing list > use-livecode at lists.runrev.com > Please visit this url to subscribe, unsubscribe and manage your > subscription preferences: > http://lists.runrev.com/**mailman/listinfo/use-livecode > > From richmondmathewson at gmail.com Fri Jul 1 02:32:01 2011 From: richmondmathewson at gmail.com (Richmond Mathewson) Date: Fri, 01 Jul 2011 09:32:01 +0300 Subject: Inserting text into a field In-Reply-To: References: <4E0D4D40.5030302@hyperactivesw.com> Message-ID: <4E0D69E1.7080908@gmail.com> On 07/01/2011 09:25 AM, Pete wrote: > Thanks. > Pete > Molly's Revenge > > > > > On Thu, Jun 30, 2011 at 9:29 PM, J. Landman Gaywrote: > >> On 6/30/11 11:05 PM, Pete wrote: >> >>> - combobox with text choices >>> - field with text in it and the cursor somewhere in the middle of the text >>> - select an entry from the combobox >>> - how do I insert the combobox entry into the text field, starting where >>> the >>> cursor is positioned? >>> >> put the label of me into the selection >> You can put 'just about anything' into a text field; personally I find this a useful sort of way to see all the properties attached to an object: Whoops: perhaps I should have said "all the properties inherent in an object", or 'grounded'; wow: wandering off into John Locke land . . . :) From palcibiades-first at yahoo.co.uk Fri Jul 1 03:27:48 2011 From: palcibiades-first at yahoo.co.uk (Peter Alcibiades) Date: Fri, 1 Jul 2011 00:27:48 -0700 (PDT) Subject: [OT] Text analysis and author, anyone done it? In-Reply-To: References: <21FFE6C9-9A54-43F7-94CB-73E7A21D6FAA@tamu.edu> Message-ID: <1309505268322-3637660.post@n4.nabble.com> I do think its possible, and has actually been done successfully. The Bible is a difficult case since we don't have value free assessments of authorship. Consequently it is reasonable to argue that what the programs do is successfully implement the prejudices of their authors. However, when we apply this to Dickens, and then ask whether the various completions of Edwin Drood were completed by him, and we apply it to Jane Austen and ask whether the software shows the same person to have written the works of Austen and Fanny Burney, we are dealing with definitely known authorship, so we can assume that if the algorithms discriminate correctly in these cases they will probably work on other material where authorship is unknown. The case which I'm looking to apply this to is a bit more like the literary case. There a number of texts of which the authorship is definitely known and not subject to dispute. There is then one text whose authorship is unknown. The question is whether it is probably by one of the known authors. We do also have a case like the Biblical case - where there are texts under one signature that we suspect to have come from more than one author, and perhaps from the author of the text of primary interest. It would be nice to be able to discriminate between authors in this body of work as well. Thanks for the very helpful references. Its early days yet, but there is no reason why statistical analysis should not illuminate this question, and there are some promising leads. Certainty is not to be expected, but statistical text analysis is definitely one weapon in the armory. I've no views on Paul and Hebrews. Whether an author really does write in statistically different ways depending on audience, well, its an empirical question, haven't come across any studies. Yes, the working assumption in the stats is that they don't. One could probably tell by looking at the work of some prolific authors who have published in different segments under different names. There are such cases. -- View this message in context: http://runtime-revolution.278305.n4.nabble.com/Re-Text-analysis-and-author-anyone-done-it-tp3636729p3637660.html Sent from the Revolution - User mailing list archive at Nabble.com. From toolbook at kestner.de Fri Jul 1 04:14:51 2011 From: toolbook at kestner.de (Tiemo Hollmann TB) Date: Fri, 1 Jul 2011 10:14:51 +0200 Subject: why does lock screen doesn't locks the screen? Message-ID: <000d01cc37c6$f9286c50$eb7944f0$@de> Hello, In a function I set the screenMouseLoc to four different locations on an image to get the mousecolor of these four locations. I have set the screen lock at the beginning of the function, but the cursor is flickering at all four locations, so obviously the screen is redrawn everytime I set the mouse at a new location though the screenloc is true. What can I do to not see the mouse (and perhaps speed up the function), and why does the lock screen doesn't work in this case? Thanks Tiemo From richmondmathewson at gmail.com Fri Jul 1 04:11:47 2011 From: richmondmathewson at gmail.com (Richmond Mathewson) Date: Fri, 01 Jul 2011 11:11:47 +0300 Subject: [OT] Text analysis and author, anyone done it? In-Reply-To: <1309505268322-3637660.post@n4.nabble.com> References: <21FFE6C9-9A54-43F7-94CB-73E7A21D6FAA@tamu.edu> <1309505268322-3637660.post@n4.nabble.com> Message-ID: <4E0D8143.5010603@gmail.com> On 07/01/2011 10:27 AM, Peter Alcibiades wrote: > I do think its possible, and has actually been done successfully. The Bible > is a difficult case since we don't have value free assessments of > authorship. Consequently it is reasonable to argue that what the programs > do is successfully implement the prejudices of their authors. > > However, when we apply this to Dickens, and then ask whether the various > completions of Edwin Drood were completed by him, and we apply it to Jane > Austen and ask whether the software shows the same person to have written > the works of Austen and Fanny Burney, we are dealing with definitely known > authorship, so we can assume that if the algorithms discriminate correctly > in these cases they will probably work on other material where authorship is > unknown. > > The case which I'm looking to apply this to is a bit more like the literary > case. There a number of texts of which the authorship is definitely known > and not subject to dispute. There is then one text whose authorship is > unknown. The question is whether it is probably by one of the known > authors. > > We do also have a case like the Biblical case - where there are texts under > one signature that we suspect to have come from more than one author, and > perhaps from the author of the text of primary interest. It would be nice > to be able to discriminate between authors in this body of work as well. I would leave that to Burton Mack: http://en.wikipedia.org/wiki/Burton_L._Mack "Quelle" is NOT for computer programmers . . . :) > Thanks for the very helpful references. Its early days yet, but there is no > reason why statistical analysis should not illuminate this question, and > there are some promising leads. Certainty is not to be expected, but > statistical text analysis is definitely one weapon in the armory. > > I've no views on Paul and Hebrews. Whether an author really does write in > statistically different ways depending on audience, well, its an empirical > question, haven't come across any studies. Yes, the working assumption in > the stats is that they don't. One could probably tell by looking at the > work of some prolific authors who have published in different segments under > different names. There are such cases. > > -- > View this message in context: http://runtime-revolution.278305.n4.nabble.com/Re-Text-analysis-and-author-anyone-done-it-tp3636729p3637660.html > Sent from the Revolution - User mailing list archive at Nabble.com. > > _______________________________________________ > use-livecode mailing list > use-livecode at lists.runrev.com > Please visit this url to subscribe, unsubscribe and manage your subscription preferences: > http://lists.runrev.com/mailman/listinfo/use-livecode From shaosean at wehostmacs.com Fri Jul 1 04:36:08 2011 From: shaosean at wehostmacs.com (Shao Sean) Date: Fri, 1 Jul 2011 04:36:08 -0400 Subject: why does lock screen doesn't locks the screen? Message-ID: <2A4EE588-1E3D-4D00-9142-5B3A4CF79BBE@wehostmacs.com> hide the cursor ? From toolbook at kestner.de Fri Jul 1 04:56:04 2011 From: toolbook at kestner.de (Tiemo Hollmann TB) Date: Fri, 1 Jul 2011 10:56:04 +0200 Subject: AW: why does lock screen doesn't locks the screen? In-Reply-To: <2A4EE588-1E3D-4D00-9142-5B3A4CF79BBE@wehostmacs.com> References: <2A4EE588-1E3D-4D00-9142-5B3A4CF79BBE@wehostmacs.com> Message-ID: <001201cc37cc$ba0c8320$2e258960$@de> Hi Shao, I've tried to set the cursor to none (hide it), but it seems to have no effect. Everytime I set the mouseloc, the cursor is shown. Thanks Tiemo > -----Urspr?ngliche Nachricht----- > Von: use-livecode-bounces at lists.runrev.com [mailto:use-livecode- > bounces at lists.runrev.com] Im Auftrag von Shao Sean > Gesendet: Freitag, 1. Juli 2011 10:36 > An: use-livecode at lists.runrev.com > Betreff: Re: why does lock screen doesn't locks the screen? > > hide the cursor ? > > _______________________________________________ > use-livecode mailing list > use-livecode at lists.runrev.com > Please visit this url to subscribe, unsubscribe and manage your subscription > preferences: > http://lists.runrev.com/mailman/listinfo/use-livecode From dick.kriesel at mail.com Fri Jul 1 05:02:38 2011 From: dick.kriesel at mail.com (Dick Kriesel) Date: Fri, 1 Jul 2011 02:02:38 -0700 Subject: why does lock screen doesn't locks the screen? In-Reply-To: <000d01cc37c6$f9286c50$eb7944f0$@de> References: <000d01cc37c6$f9286c50$eb7944f0$@de> Message-ID: Hi, Tiemo. Does it happen in a standalone? from the dictionary: Note: When using script debug mode, the screen cannot be locked and the lock screen command has no effect. -- Dick On Jul 1, 2011, at 1:14 AM, Tiemo Hollmann TB wrote: > Hello, > > In a function I set the screenMouseLoc to four different locations on an > image to get the mousecolor of these four locations. > > I have set the screen lock at the beginning of the function, but the cursor > is flickering at all four locations, so obviously the screen is redrawn > everytime I set the mouse at a new location though the screenloc is true. > > What can I do to not see the mouse (and perhaps speed up the function), and > why does the lock screen doesn't work in this case? > > Thanks > > Tiemo From toolbook at kestner.de Fri Jul 1 05:20:11 2011 From: toolbook at kestner.de (Tiemo Hollmann TB) Date: Fri, 1 Jul 2011 11:20:11 +0200 Subject: AW: why does lock screen doesn't locks the screen? In-Reply-To: References: <000d01cc37c6$f9286c50$eb7944f0$@de> Message-ID: <001301cc37d0$184608f0$48d21ad0$@de> Hi Dick, yes it's the same in a standalone using both set the cursor to none and lock screen Thanks Tiemo > -----Urspr?ngliche Nachricht----- > Von: use-livecode-bounces at lists.runrev.com [mailto:use-livecode- > bounces at lists.runrev.com] Im Auftrag von Dick Kriesel > Gesendet: Freitag, 1. Juli 2011 11:03 > An: How to use LiveCode > Betreff: Re: why does lock screen doesn't locks the screen? > > Hi, Tiemo. Does it happen in a standalone? > > from the dictionary: > Note: When using script debug mode, the screen cannot be locked and the lock > screen command has no effect. > > -- Dick > > > On Jul 1, 2011, at 1:14 AM, Tiemo Hollmann TB wrote: > > > Hello, > > > > In a function I set the screenMouseLoc to four different locations on an > > image to get the mousecolor of these four locations. > > > > I have set the screen lock at the beginning of the function, but the cursor > > is flickering at all four locations, so obviously the screen is redrawn > > everytime I set the mouse at a new location though the screenloc is true. > > > > What can I do to not see the mouse (and perhaps speed up the function), and > > why does the lock screen doesn't work in this case? > > > > Thanks > > > > Tiemo > > > _______________________________________________ > use-livecode mailing list > use-livecode at lists.runrev.com > Please visit this url to subscribe, unsubscribe and manage your subscription > preferences: > http://lists.runrev.com/mailman/listinfo/use-livecode From toolbook at kestner.de Fri Jul 1 05:39:43 2011 From: toolbook at kestner.de (Tiemo Hollmann TB) Date: Fri, 1 Jul 2011 11:39:43 +0200 Subject: what RGB is blue? Message-ID: <001401cc37d2$d3257910$79706b30$@de> Hello, I am taking the mousecolor at different points from an image by script (not by clicking). I would like to analyse if the color I've taken is a "kind of blue", or another color. I want to change the backgroundcolor of an image. The background is always blue, but different blues and changing over the background. So what I want to do is to verify, what is background and what is foreground of my image. 100% pure blue would be 0,0,255. But for a human being 25,75,130 (greyblue) is also still blue, but 240,20,180 is pink, though the third RGB value is higher as in my greyblue. So I can't just check only the third RGB value, neither the sum or cross total. Has anybody ever heard, if you can define at all by math "what is blue"? Any color specialist around here? Tiemo From tsj at unimelb.edu.au Fri Jul 1 05:40:57 2011 From: tsj at unimelb.edu.au (Terry Judd) Date: Fri, 1 Jul 2011 19:40:57 +1000 Subject: [OT] Text analysis and author, anyone done it? In-Reply-To: <1309505268322-3637660.post@n4.nabble.com> Message-ID: On 01/07/2011 05:27 PM, "Peter Alcibiades" wrote: > > The case which I'm looking to apply this to is a bit more like the literary > case. There a number of texts of which the authorship is definitely known > and not subject to dispute. There is then one text whose authorship is > unknown. The question is whether it is probably by one of the known > authors. > > We do also have a case like the Biblical case - where there are texts under > one signature that we suspect to have come from more than one author, and > perhaps from the author of the text of primary interest. It would be nice > to be able to discriminate between authors in this body of work as well. > One fairly simple approach that you could certainly implement in LiveCode involve compressing (zipping) chunks of text separately and combined and comparing their lengths. If two chunks of text have a relatively high degree of similarity then their combined compressed length will be less than for two equivalent but dissimilar chunks. So, in the case of authorship, if you have text from 3 known authors and one unknown author you combine the unknown one with each of the known ones and compare the zipped length of these combined text to the zipped length of the 3 individual texts. The combined text that has the smallest increase in length relative to the individual length of its know text is then most likely to have both texts authored by the same person (I hope that makes sense). Terry... -- Dr Terry Judd | Senior Lecturer in Medical Education Medical Education Unit Melbourne Medical School The University of Melbourne From tsj at unimelb.edu.au Fri Jul 1 05:44:09 2011 From: tsj at unimelb.edu.au (Terry Judd) Date: Fri, 1 Jul 2011 19:44:09 +1000 Subject: what RGB is blue? In-Reply-To: <001401cc37d2$d3257910$79706b30$@de> Message-ID: I think if the blue value is the largest of the 3 values then the colour will always be a blue of sorts. Terry... On 01/07/2011 07:39 PM, "Tiemo Hollmann TB" wrote: > Hello, > > I am taking the mousecolor at different points from an image by script (not > by clicking). I would like to analyse if the color I've taken is a "kind of > blue", or another color. I want to change the backgroundcolor of an image. > The background is always blue, but different blues and changing over the > background. So what I want to do is to verify, what is background and what > is foreground of my image. > > 100% pure blue would be 0,0,255. But for a human being 25,75,130 (greyblue) > is also still blue, but 240,20,180 is pink, though the third RGB value is > higher as in my greyblue. > > So I can't just check only the third RGB value, neither the sum or cross > total. Has anybody ever heard, if you can define at all by math "what is > blue"? > > Any color specialist around here? > > Tiemo > > > > > > _______________________________________________ > use-livecode mailing list > use-livecode at lists.runrev.com > Please visit this url to subscribe, unsubscribe and manage your subscription > preferences: > http://lists.runrev.com/mailman/listinfo/use-livecode > -- Dr Terry Judd | Senior Lecturer in Medical Education Medical Education Unit Melbourne Medical School The University of Melbourne From m.schonewille at economy-x-talk.com Fri Jul 1 05:52:45 2011 From: m.schonewille at economy-x-talk.com (Mark Schonewille) Date: Fri, 1 Jul 2011 11:52:45 +0200 Subject: what RGB is blue? In-Reply-To: <001401cc37d2$d3257910$79706b30$@de> References: <001401cc37d2$d3257910$79706b30$@de> Message-ID: Tiemo, The distance between any colour (1) and a colour chosen by you (2; e.g. r=0, g=0, b=255) can be calculated as sqrt((r1-r2)^2+(g1-g2)^2+(b1-b2)^2) <= T where T is a threshold between 0 and 442 set by you. Change the colour of the background pixel whenever the distance between the colour of that pixel and your selected colour is less than the threshold. I use this technique in Color Converter http://www.color-converter.com -- Best regards, Mark Schonewille Economy-x-Talk Consulting and Software Engineering Homepage: http://economy-x-talk.com Twitter: http://twitter.com/xtalkprogrammer KvK: 50277553 New: Download the Installer Maker Plugin 1.6 for LiveCode here http://qery.us/ce On 1 jul 2011, at 11:39, Tiemo Hollmann TB wrote: > Hello, > > I am taking the mousecolor at different points from an image by script (not > by clicking). I would like to analyse if the color I've taken is a "kind of > blue", or another color. I want to change the backgroundcolor of an image. > The background is always blue, but different blues and changing over the > background. So what I want to do is to verify, what is background and what > is foreground of my image. > > 100% pure blue would be 0,0,255. But for a human being 25,75,130 (greyblue) > is also still blue, but 240,20,180 is pink, though the third RGB value is > higher as in my greyblue. > > So I can't just check only the third RGB value, neither the sum or cross > total. Has anybody ever heard, if you can define at all by math "what is > blue"? > > Any color specialist around here? > > Tiemo > From keith.clarke at clarkeandclarke.co.uk Fri Jul 1 05:55:29 2011 From: keith.clarke at clarkeandclarke.co.uk (Keith Clarke) Date: Fri, 1 Jul 2011 10:55:29 +0100 Subject: what RGB is blue? In-Reply-To: <001401cc37d2$d3257910$79706b30$@de> References: <001401cc37d2$d3257910$79706b30$@de> Message-ID: Hi Tiemo, Not a colour expert but it's an interesting problem - especially as 'blueness' may be subjective. I visualise RGB values correlating with x, y, and z axis values of a colour 'sphere' where the origin is black and the surface, at cube-root(r-cubed + g-cubed + b-cubed) where r, g and b all = 255, is white. Maybe you can define a section of that sphere's volume which is consistently blue in nature, given your target hues? Maybe the surface area of this section volume is defined by a slice (what is a 3D 'chord'?) by specific colours on the blue-red, blue-green spectra? That's probably all rubbish but you never know? :-) Best, Keith.. On 1 Jul 2011, at 10:39, Tiemo Hollmann TB wrote: > Hello, > > I am taking the mousecolor at different points from an image by script (not > by clicking). I would like to analyse if the color I've taken is a "kind of > blue", or another color. I want to change the backgroundcolor of an image. > The background is always blue, but different blues and changing over the > background. So what I want to do is to verify, what is background and what > is foreground of my image. > > 100% pure blue would be 0,0,255. But for a human being 25,75,130 (greyblue) > is also still blue, but 240,20,180 is pink, though the third RGB value is > higher as in my greyblue. > > So I can't just check only the third RGB value, neither the sum or cross > total. Has anybody ever heard, if you can define at all by math "what is > blue"? > > Any color specialist around here? > > Tiemo From toolbook at kestner.de Fri Jul 1 06:41:20 2011 From: toolbook at kestner.de (Tiemo Hollmann TB) Date: Fri, 1 Jul 2011 12:41:20 +0200 Subject: AW: what RGB is blue? In-Reply-To: References: <001401cc37d2$d3257910$79706b30$@de> Message-ID: <001901cc37db$6f8b2fe0$4ea18fa0$@de> Thank you Mark, for your profound advice. It sounds just as what I was looking for. I will give it a try Thanks Tiemo > -----Urspr?ngliche Nachricht----- > Von: use-livecode-bounces at lists.runrev.com [mailto:use-livecode- > bounces at lists.runrev.com] Im Auftrag von Mark Schonewille > Gesendet: Freitag, 1. Juli 2011 11:53 > An: How to use LiveCode > Betreff: Re: what RGB is blue? > > Tiemo, > > The distance between any colour (1) and a colour chosen by you (2; e.g. r=0, > g=0, b=255) can be calculated as > > sqrt((r1-r2)^2+(g1-g2)^2+(b1-b2)^2) <= T > > where T is a threshold between 0 and 442 set by you. Change the colour of the > background pixel whenever the distance between the colour of that pixel and > your selected colour is less than the threshold. > > I use this technique in Color Converter http://www.color-converter.com > > -- > Best regards, > > Mark Schonewille > > Economy-x-Talk Consulting and Software Engineering > Homepage: http://economy-x-talk.com > Twitter: http://twitter.com/xtalkprogrammer > KvK: 50277553 > > New: Download the Installer Maker Plugin 1.6 for LiveCode here > http://qery.us/ce > > On 1 jul 2011, at 11:39, Tiemo Hollmann TB wrote: > > > Hello, > > > > I am taking the mousecolor at different points from an image by script (not > > by clicking). I would like to analyse if the color I've taken is a "kind of > > blue", or another color. I want to change the backgroundcolor of an image. > > The background is always blue, but different blues and changing over the > > background. So what I want to do is to verify, what is background and what > > is foreground of my image. > > > > 100% pure blue would be 0,0,255. But for a human being 25,75,130 (greyblue) > > is also still blue, but 240,20,180 is pink, though the third RGB value is > > higher as in my greyblue. > > > > So I can't just check only the third RGB value, neither the sum or cross > > total. Has anybody ever heard, if you can define at all by math "what is > > blue"? > > > > Any color specialist around here? > > > > Tiemo > > > > _______________________________________________ > use-livecode mailing list > use-livecode at lists.runrev.com > Please visit this url to subscribe, unsubscribe and manage your subscription > preferences: > http://lists.runrev.com/mailman/listinfo/use-livecode From toolbook at kestner.de Fri Jul 1 07:58:15 2011 From: toolbook at kestner.de (Tiemo Hollmann TB) Date: Fri, 1 Jul 2011 13:58:15 +0200 Subject: AW: what RGB is blue? In-Reply-To: <001901cc37db$6f8b2fe0$4ea18fa0$@de> References: <001401cc37d2$d3257910$79706b30$@de> <001901cc37db$6f8b2fe0$4ea18fa0$@de> Message-ID: <001d01cc37e6$2e181e00$8a485a00$@de> Mark, you made my day! With this approach I even don't need to take the mousecolor anymore. I can just go with my blue and the threshold. Thanks, Tiemo > -----Urspr?ngliche Nachricht----- > Von: use-livecode-bounces at lists.runrev.com [mailto:use-livecode- > bounces at lists.runrev.com] Im Auftrag von Tiemo Hollmann TB > Gesendet: Freitag, 1. Juli 2011 12:41 > An: 'How to use LiveCode' > Betreff: AW: what RGB is blue? > > Thank you Mark, for your profound advice. > It sounds just as what I was looking for. I will give it a try > Thanks > Tiemo > > > -----Urspr?ngliche Nachricht----- > > Von: use-livecode-bounces at lists.runrev.com [mailto:use-livecode- > > bounces at lists.runrev.com] Im Auftrag von Mark Schonewille > > Gesendet: Freitag, 1. Juli 2011 11:53 > > An: How to use LiveCode > > Betreff: Re: what RGB is blue? > > > > Tiemo, > > > > The distance between any colour (1) and a colour chosen by you (2; e.g. > r=0, > > g=0, b=255) can be calculated as > > > > sqrt((r1-r2)^2+(g1-g2)^2+(b1-b2)^2) <= T > > > > where T is a threshold between 0 and 442 set by you. Change the colour of > the > > background pixel whenever the distance between the colour of that pixel > and > > your selected colour is less than the threshold. > > > > I use this technique in Color Converter http://www.color-converter.com > > > > -- > > Best regards, > > > > Mark Schonewille > > > > Economy-x-Talk Consulting and Software Engineering > > Homepage: http://economy-x-talk.com > > Twitter: http://twitter.com/xtalkprogrammer > > KvK: 50277553 > > > > New: Download the Installer Maker Plugin 1.6 for LiveCode here > > http://qery.us/ce > > > > On 1 jul 2011, at 11:39, Tiemo Hollmann TB wrote: > > > > > Hello, > > > > > > I am taking the mousecolor at different points from an image by script > (not > > > by clicking). I would like to analyse if the color I've taken is a "kind > of > > > blue", or another color. I want to change the backgroundcolor of an > image. > > > The background is always blue, but different blues and changing over the > > > background. So what I want to do is to verify, what is background and > what > > > is foreground of my image. > > > > > > 100% pure blue would be 0,0,255. But for a human being 25,75,130 > (greyblue) > > > is also still blue, but 240,20,180 is pink, though the third RGB value > is > > > higher as in my greyblue. > > > > > > So I can't just check only the third RGB value, neither the sum or cross > > > total. Has anybody ever heard, if you can define at all by math "what is > > > blue"? > > > > > > Any color specialist around here? > > > > > > Tiemo > > > > > > > _______________________________________________ > > use-livecode mailing list > > use-livecode at lists.runrev.com > > Please visit this url to subscribe, unsubscribe and manage your > subscription > > preferences: > > http://lists.runrev.com/mailman/listinfo/use-livecode > > > _______________________________________________ > use-livecode mailing list > use-livecode at lists.runrev.com > Please visit this url to subscribe, unsubscribe and manage your subscription > preferences: > http://lists.runrev.com/mailman/listinfo/use-livecode From dave.cragg at lacscentre.co.uk Fri Jul 1 09:14:33 2011 From: dave.cragg at lacscentre.co.uk (Dave Cragg) Date: Fri, 1 Jul 2011 14:14:33 +0100 Subject: [OT] Text analysis and author, anyone done it? In-Reply-To: References: <21FFE6C9-9A54-43F7-94CB-73E7A21D6FAA@tamu.edu> Message-ID: <6A6F7964-116F-4F98-AFF2-98F8372500F7@lacscentre.co.uk> On 30 Jun 2011, at 23:42, Bob Sneidar wrote: > Since the subject was broached using textual analysis > Software cannot fix that. Well, I just put your text through my analogue software text analyzer, at it says there's a 96.2% chance it was written by Elvis. That sounds pretty convincing to me. :-) Dave From m.schonewille at economy-x-talk.com Fri Jul 1 09:23:08 2011 From: m.schonewille at economy-x-talk.com (Mark Schonewille) Date: Fri, 1 Jul 2011 15:23:08 +0200 Subject: LiveCode.tv Event - Who are the presenters Message-ID: <9DAFA3BD-E4C4-4A92-86C6-6B7A888F5CCB@economy-x-talk.com> Hi, This is an important message. I haven't been in touch with Bj?rnke the past few days and thus I don't know who are going to present. I know of one person that he will do a presentation, but now I don't know who is the other person. If you are presenting tomorrow, please send me a message and tell me what the subject of your presentation is. -- Best regards, Mark Schonewille Economy-x-Talk Consulting and Software Engineering Homepage: http://economy-x-talk.com Twitter: http://twitter.com/xtalkprogrammer KvK: 50277553 New: Download the Installer Maker Plugin 1.6 for LiveCode here http://qery.us/ce From keith.clarke at clarkeandclarke.co.uk Fri Jul 1 09:58:03 2011 From: keith.clarke at clarkeandclarke.co.uk (Keith Clarke) Date: Fri, 1 Jul 2011 14:58:03 +0100 Subject: How to set-up Chatrev server on Linux / on-rev? Message-ID: <00A02220-E0EF-40CD-8F21-68117AB9B486@clarkeandclarke.co.uk> Hi folks, I think the title says it all - I want to attempt to configure a Chatrev server on Linux - either on my Linux VPS or my on-rev account. Are there any set-up or admin docs or posts around, as I can't find anything in the list archives or via Google? I've downloaded the server stack from http://bjoernke.com/index.irev?target=chatrev but what next? Save as a Linux stand-alone? upload to the server, if so, where? How does one start the server with no UI access - and what about enabling server ports, etc? Best, Keith.. From gcanyon+rev at gmail.com Fri Jul 1 11:24:50 2011 From: gcanyon+rev at gmail.com (Geoff Canyon Rev) Date: Fri, 1 Jul 2011 10:24:50 -0500 Subject: what RGB is blue? In-Reply-To: References: <001401cc37d2$d3257910$79706b30$@de> Message-ID: I guess I'm playing devil's advocate, but if you were testing for "blue"-ness, wouldn't you convert to HSV and compare the H, regardless of S or V? gc On Fri, Jul 1, 2011 at 4:52 AM, Mark Schonewille < m.schonewille at economy-x-talk.com> wrote: > Tiemo, > > The distance between any colour (1) and a colour chosen by you (2; e.g. > r=0, g=0, b=255) can be calculated as > > sqrt((r1-r2)^2+(g1-g2)^2+(b1-b2)^2) <= T > > where T is a threshold between 0 and 442 set by you. Change the colour of > the background pixel whenever the distance between the colour of that pixel > and your selected colour is less than the threshold. > > I use this technique in Color Converter http://www.color-converter.com > > -- > Best regards, > > Mark Schonewille > > Economy-x-Talk Consulting and Software Engineering > Homepage: http://economy-x-talk.com > Twitter: http://twitter.com/xtalkprogrammer > KvK: 50277553 > > New: Download the Installer Maker Plugin 1.6 for LiveCode here > http://qery.us/ce > > On 1 jul 2011, at 11:39, Tiemo Hollmann TB wrote: > > > Hello, > > > > I am taking the mousecolor at different points from an image by script > (not > > by clicking). I would like to analyse if the color I've taken is a "kind > of > > blue", or another color. I want to change the backgroundcolor of an > image. > > The background is always blue, but different blues and changing over the > > background. So what I want to do is to verify, what is background and > what > > is foreground of my image. > > > > 100% pure blue would be 0,0,255. But for a human being 25,75,130 > (greyblue) > > is also still blue, but 240,20,180 is pink, though the third RGB value is > > higher as in my greyblue. > > > > So I can't just check only the third RGB value, neither the sum or cross > > total. Has anybody ever heard, if you can define at all by math "what is > > blue"? > > > > Any color specialist around here? > > > > Tiemo > > > > _______________________________________________ > use-livecode mailing list > use-livecode at lists.runrev.com > Please visit this url to subscribe, unsubscribe and manage your > subscription preferences: > http://lists.runrev.com/mailman/listinfo/use-livecode > From form at nonsanity.com Fri Jul 1 11:29:04 2011 From: form at nonsanity.com (Nonsanity) Date: Fri, 1 Jul 2011 11:29:04 -0400 Subject: why does lock screen doesn't locks the screen? In-Reply-To: <000d01cc37c6$f9286c50$eb7944f0$@de> References: <000d01cc37c6$f9286c50$eb7944f0$@de> Message-ID: "lock screen" only locks the window contents, not the whole screen (despite the name) and never the cursor. The cursor belongs to the user and lives above the screen. But you can "lock cursor" to keep in in the shape that you want. I can't get "set cursor to none" to hide the cursor here, so there may be something wrong with that feature. What you can do instead is make a 16x16 all-alpha png file and import that into the stack. Then you can set the cursor to that image's id and then lock the cursor and do your movements. When the handler exits, the lock will be released, but it's always better to put an "unlock cursor" (or screen) because these commands stack. (Five locks takes five unlocks.) I made such an image and tested it, and it worked. I'd give you a link to the image, but Dropbox is being flaky right now. Wait... I'll put it on another server: http://nonsanity.com/x/invisible16x16.png And here's my script. I made sure the points I picked were over different objects, like a field, to make sure the cursor wouldn't change as it would normally. on mouseUp set the cursor to 1007 lock cursor set the cursor to 1007 wait 50 repeat with a = 1 to 4 set the screenmouseloc to word a of "202,398 290,270 168,222 363,594" wait 50 end repeat end mouseUp Hope that helps. ~ Chris Innanen ~ Nonsanity On Fri, Jul 1, 2011 at 4:14 AM, Tiemo Hollmann TB wrote: > Hello, > > In a function I set the screenMouseLoc to four different locations on an > image to get the mousecolor of these four locations. > > I have set the screen lock at the beginning of the function, but the cursor > is flickering at all four locations, so obviously the screen is redrawn > everytime I set the mouse at a new location though the screenloc is true. > > What can I do to not see the mouse (and perhaps speed up the function), and > why does the lock screen doesn't work in this case? > > Thanks > > Tiemo > > > > > > _______________________________________________ > use-livecode mailing list > use-livecode at lists.runrev.com > Please visit this url to subscribe, unsubscribe and manage your > subscription preferences: > http://lists.runrev.com/mailman/listinfo/use-livecode > From bobs at twft.com Fri Jul 1 11:42:17 2011 From: bobs at twft.com (Bob Sneidar) Date: Fri, 1 Jul 2011 08:42:17 -0700 Subject: [OT] Text analysis and author, anyone done it? In-Reply-To: <6A6F7964-116F-4F98-AFF2-98F8372500F7@lacscentre.co.uk> References: <21FFE6C9-9A54-43F7-94CB-73E7A21D6FAA@tamu.edu> <6A6F7964-116F-4F98-AFF2-98F8372500F7@lacscentre.co.uk> Message-ID: I'm busted! Does this mean I have to go back to Las Vegas? Bob On Jul 1, 2011, at 6:14 AM, Dave Cragg wrote: > > On 30 Jun 2011, at 23:42, Bob Sneidar wrote: > >> Since the subject was broached using textual analysis > >> Software cannot fix that. > > Well, I just put your text through my analogue software text analyzer, at it says there's a 96.2% chance it was written by Elvis. That sounds pretty convincing to me. :-) > > Dave > _______________________________________________ > use-livecode mailing list > use-livecode at lists.runrev.com > Please visit this url to subscribe, unsubscribe and manage your subscription preferences: > http://lists.runrev.com/mailman/listinfo/use-livecode From bobs at twft.com Fri Jul 1 11:45:48 2011 From: bobs at twft.com (Bob Sneidar) Date: Fri, 1 Jul 2011 08:45:48 -0700 Subject: what RGB is blue? In-Reply-To: References: Message-ID: <39D19E24-76DE-48C1-A6ED-3502A94F9699@twft.com> It would have to be higher that both the first two by a significant margin. Arbitrarily I would say if higher by a factor of 3, but if there is any standard about this I don't know it. Bob On Jul 1, 2011, at 2:44 AM, Terry Judd wrote: > I think if the blue value is the largest of the 3 values then the colour > will always be a blue of sorts. > > Terry... > > > On 01/07/2011 07:39 PM, "Tiemo Hollmann TB" wrote: > >> Hello, >> >> I am taking the mousecolor at different points from an image by script (not >> by clicking). I would like to analyse if the color I've taken is a "kind of >> blue", or another color. I want to change the backgroundcolor of an image. >> The background is always blue, but different blues and changing over the >> background. So what I want to do is to verify, what is background and what >> is foreground of my image. >> >> 100% pure blue would be 0,0,255. But for a human being 25,75,130 (greyblue) >> is also still blue, but 240,20,180 is pink, though the third RGB value is >> higher as in my greyblue. >> >> So I can't just check only the third RGB value, neither the sum or cross >> total. Has anybody ever heard, if you can define at all by math "what is >> blue"? >> >> Any color specialist around here? >> >> Tiemo >> >> >> >> >> >> _______________________________________________ >> use-livecode mailing list >> use-livecode at lists.runrev.com >> Please visit this url to subscribe, unsubscribe and manage your subscription >> preferences: >> http://lists.runrev.com/mailman/listinfo/use-livecode >> > > -- > Dr Terry Judd | Senior Lecturer in Medical Education > Medical Education Unit > Melbourne Medical School > The University of Melbourne > > > > > _______________________________________________ > use-livecode mailing list > use-livecode at lists.runrev.com > Please visit this url to subscribe, unsubscribe and manage your subscription preferences: > http://lists.runrev.com/mailman/listinfo/use-livecode From bobs at twft.com Fri Jul 1 11:47:15 2011 From: bobs at twft.com (Bob Sneidar) Date: Fri, 1 Jul 2011 08:47:15 -0700 Subject: what RGB is blue? In-Reply-To: References: <001401cc37d2$d3257910$79706b30$@de> Message-ID: <2518FE84-E00E-4B67-B02E-F96F05B756B2@twft.com> Well remember that we perceive color in reference to the colors around it. What he might pick on a color wheel might not seem all that blue when it is by itself or nearby other colors. Bob On Jul 1, 2011, at 2:55 AM, Keith Clarke wrote: > Hi Tiemo, > Not a colour expert but it's an interesting problem - especially as 'blueness' may be subjective. > > I visualise RGB values correlating with x, y, and z axis values of a colour 'sphere' where the origin is black and the surface, at cube-root(r-cubed + g-cubed + b-cubed) where r, g and b all = 255, is white. > > Maybe you can define a section of that sphere's volume which is consistently blue in nature, given your target hues? Maybe the surface area of this section volume is defined by a slice (what is a 3D 'chord'?) by specific colours on the blue-red, blue-green spectra? > > That's probably all rubbish but you never know? :-) > Best, > Keith.. > > On 1 Jul 2011, at 10:39, Tiemo Hollmann TB wrote: > >> Hello, >> >> I am taking the mousecolor at different points from an image by script (not >> by clicking). I would like to analyse if the color I've taken is a "kind of >> blue", or another color. I want to change the backgroundcolor of an image. >> The background is always blue, but different blues and changing over the >> background. So what I want to do is to verify, what is background and what >> is foreground of my image. >> >> 100% pure blue would be 0,0,255. But for a human being 25,75,130 (greyblue) >> is also still blue, but 240,20,180 is pink, though the third RGB value is >> higher as in my greyblue. >> >> So I can't just check only the third RGB value, neither the sum or cross >> total. Has anybody ever heard, if you can define at all by math "what is >> blue"? >> >> Any color specialist around here? >> >> Tiemo > > > _______________________________________________ > use-livecode mailing list > use-livecode at lists.runrev.com > Please visit this url to subscribe, unsubscribe and manage your subscription preferences: > http://lists.runrev.com/mailman/listinfo/use-livecode From m.schonewille at economy-x-talk.com Fri Jul 1 11:50:56 2011 From: m.schonewille at economy-x-talk.com (Mark Schonewille) Date: Fri, 1 Jul 2011 17:50:56 +0200 Subject: what RGB is blue? In-Reply-To: References: <001401cc37d2$d3257910$79706b30$@de> Message-ID: <71CCE3E9-BB48-4B96-86A4-F633580EB3AA@economy-x-talk.com> Geoff, No, because HSV is just a different represenation of RGB and in RGB there's only one single white point and only one single black point while in HSV there's a completely white line and a completely black plain (a little bit of white is still white and very bright black is still black). For this reason, you might expect RGB to be more accurate at the extremes. Still, because it is just a matter of math, it shouldn't really matter much. I'm not a true expert, this is just what I understand from it. -- Best regards, Mark Schonewille Economy-x-Talk Consulting and Software Engineering Homepage: http://economy-x-talk.com Twitter: http://twitter.com/xtalkprogrammer KvK: 50277553 New: Download the Installer Maker Plugin 1.6 for LiveCode here http://qery.us/ce On 1 jul 2011, at 17:24, Geoff Canyon Rev wrote: > I guess I'm playing devil's advocate, but if you were testing for > "blue"-ness, wouldn't you convert to HSV and compare the H, regardless of S > or V? > > gc > From martyknapp at comcast.net Fri Jul 1 11:58:21 2011 From: martyknapp at comcast.net (Marty Knapp) Date: Fri, 01 Jul 2011 08:58:21 -0700 Subject: Font Problem Message-ID: <4E0DEE9D.1090903@comcast.net> I have a Mac only LiveCode standalone that requires a custom symbol font (yes, it's licensed). I use revFontLoad to put it into use. This works great on my 3 Macs and 90% of my users. The other 10% report that that it does not load. After I run revFontLoad I check the fontnames to see if the font is present and then alert the user if it hasn't loaded. If it hasn't, I then have these people manually install the font in Library>Fonts and restart. A percentage of these report that my app still doesn't find the font. In a few cases the user had a 3rd party font management program in use. 2 users reported that they had to uninstall the font program before my app would recognize the font. One was using FontExplorer and the other FontAgent, I believe. I tried FontExplorer and it worked fine for me - no conflict with revFontLoad. On a couple of occasions the people emailed me and used the font in the email to show that the font was working everywhere but in my app. I usually have people use OnyX or FontNuke to clear the font caches, though that hasn't seemed to help I've tried using graphics instead of a font, but the resolution isn't high enough for my use, so the font is required. Any suggestions for me? Marty Knapp From bobs at twft.com Fri Jul 1 12:08:04 2011 From: bobs at twft.com (Bob Sneidar) Date: Fri, 1 Jul 2011 09:08:04 -0700 Subject: [OT] what RGB is blue? In-Reply-To: References: <001401cc37d2$d3257910$79706b30$@de> Message-ID: This brings up an interesting point about color that has long interested me. Given that we see all the frequencies of light in the visible spectrum, why would we see distinct colors? For instance, why in a rainbow is there a blue band and then a green band? While we can in fact see colors in between, our eyes do not seem to be "attuned" as well to them as to what we would call the pure colors. Red, Yellow, and Blue always seem predominant to us, but green, orange and purple also seem strong. Beyond that we don't notice much. What is in between green and blue? Something that some call more blue, some more green. I remember having arguments as a child about whether something was aqua or green. Whenever we look at a color, don't we catch ourselves comparing it to how close to a primary or secondary color it is? Could it be that our eyes (meaning the entire optical system) resonate well with certain colors, the way someone with perfect tone sense knows a C is what it is and not 3 cents off just by listening to it? The way almost all of us know that 1,3,5,8 is harmonic, but 3,4,7,8 is not? We say someone who cannot tell is tone deaf. We say someone who cannot discern the colors is color blind. We are saying something is wrong with their senses, that they are lacking something we recognize that everyone else has. Some will argue that not all civilizations perceive music this way. I would say that it is possible to train the human mind to accept dissonance, the same way it is possible to train an artists eye to see and work with colors that are not primary or secondary. But it is not the way we see and hear things by nature, the way we first perceived them as children. Or so it seems to me. I wonder then if colors have octaves? Maybe we perceive yellow as an octave of blue? Of red as an octave of both? But I am out of my reckoning here. Still, if this were really the case, then maybe all the host of heaven (the stars) really are singing. Bob On Jul 1, 2011, at 8:24 AM, Geoff Canyon Rev wrote: > I guess I'm playing devil's advocate, but if you were testing for > "blue"-ness, wouldn't you convert to HSV and compare the H, regardless of S > or V? > > gc > > On Fri, Jul 1, 2011 at 4:52 AM, Mark Schonewille < > m.schonewille at economy-x-talk.com> wrote: > >> Tiemo, >> >> The distance between any colour (1) and a colour chosen by you (2; e.g. >> r=0, g=0, b=255) can be calculated as >> >> sqrt((r1-r2)^2+(g1-g2)^2+(b1-b2)^2) <= T >> >> where T is a threshold between 0 and 442 set by you. Change the colour of >> the background pixel whenever the distance between the colour of that pixel >> and your selected colour is less than the threshold. >> >> I use this technique in Color Converter http://www.color-converter.com >> >> -- >> Best regards, >> >> Mark Schonewille >> >> Economy-x-Talk Consulting and Software Engineering >> Homepage: http://economy-x-talk.com >> Twitter: http://twitter.com/xtalkprogrammer >> KvK: 50277553 >> >> New: Download the Installer Maker Plugin 1.6 for LiveCode here >> http://qery.us/ce >> >> On 1 jul 2011, at 11:39, Tiemo Hollmann TB wrote: >> >>> Hello, >>> >>> I am taking the mousecolor at different points from an image by script >> (not >>> by clicking). I would like to analyse if the color I've taken is a "kind >> of >>> blue", or another color. I want to change the backgroundcolor of an >> image. >>> The background is always blue, but different blues and changing over the >>> background. So what I want to do is to verify, what is background and >> what >>> is foreground of my image. >>> >>> 100% pure blue would be 0,0,255. But for a human being 25,75,130 >> (greyblue) >>> is also still blue, but 240,20,180 is pink, though the third RGB value is >>> higher as in my greyblue. >>> >>> So I can't just check only the third RGB value, neither the sum or cross >>> total. Has anybody ever heard, if you can define at all by math "what is >>> blue"? >>> >>> Any color specialist around here? >>> >>> Tiemo >>> >> >> _______________________________________________ >> use-livecode mailing list >> use-livecode at lists.runrev.com >> Please visit this url to subscribe, unsubscribe and manage your >> subscription preferences: >> http://lists.runrev.com/mailman/listinfo/use-livecode >> > _______________________________________________ > use-livecode mailing list > use-livecode at lists.runrev.com > Please visit this url to subscribe, unsubscribe and manage your subscription preferences: > http://lists.runrev.com/mailman/listinfo/use-livecode From bobs at twft.com Fri Jul 1 12:11:52 2011 From: bobs at twft.com (Bob Sneidar) Date: Fri, 1 Jul 2011 09:11:52 -0700 Subject: Font Problem In-Reply-To: <4E0DEE9D.1090903@comcast.net> References: <4E0DEE9D.1090903@comcast.net> Message-ID: <45080A9C-C06C-48B8-8D87-1D0690DB3FC7@twft.com> I am going to say that there is a conflict with another font id. A lot of Mac users have damaged fonts and don't know it. Do they get another font besides the one they installer, or do they get nothing at all? What is their software or os reporting? Bob On Jul 1, 2011, at 8:58 AM, Marty Knapp wrote: > I have a Mac only LiveCode standalone that requires a custom symbol font (yes, it's licensed). I use revFontLoad to put it into use. This works great on my 3 Macs and 90% of my users. The other 10% report that that it does not load. > > After I run revFontLoad I check the fontnames to see if the font is present and then alert the user if it hasn't loaded. If it hasn't, I then have these people manually install the font in Library>Fonts and restart. A percentage of these report that my app still doesn't find the font. In a few cases the user had a 3rd party font management program in use. 2 users reported that they had to uninstall the font program before my app would recognize the font. One was using FontExplorer and the other FontAgent, I believe. I tried FontExplorer and it worked fine for me - no conflict with revFontLoad. > > On a couple of occasions the people emailed me and used the font in the email to show that the font was working everywhere but in my app. > > I usually have people use OnyX or FontNuke to clear the font caches, though that hasn't seemed to help > > I've tried using graphics instead of a font, but the resolution isn't high enough for my use, so the font is required. > > Any suggestions for me? > > Marty Knapp > > _______________________________________________ > use-livecode mailing list > use-livecode at lists.runrev.com > Please visit this url to subscribe, unsubscribe and manage your subscription preferences: > http://lists.runrev.com/mailman/listinfo/use-livecode From zellner at tamu.edu Fri Jul 1 12:29:00 2011 From: zellner at tamu.edu (Ronald Zellner) Date: Fri, 1 Jul 2011 11:29:00 -0500 Subject: Inserting text into a field Message-ID: This may be over-kill, but it works. Script on the text field: on selectionChanged --set variable for insertion point global insertPoint put word 2 of the selectedChunk into insertPoint end selectionChanged Script on Combo box: on menuPick pItemName global insertPoint switch pItemName end switch if character insertPoint of field "NewText" = space then -- check for relative location related to spaces put space & pItemName before character insertPoint of field "NewText" else put pItemName & space before character insertPoint of field "NewText" end if end menuPick Same script works in an Option Menu button. Ron From martyknapp at comcast.net Fri Jul 1 12:40:52 2011 From: martyknapp at comcast.net (Marty Knapp) Date: Fri, 01 Jul 2011 09:40:52 -0700 Subject: Font Problem In-Reply-To: <45080A9C-C06C-48B8-8D87-1D0690DB3FC7@twft.com> References: <4E0DEE9D.1090903@comcast.net> <45080A9C-C06C-48B8-8D87-1D0690DB3FC7@twft.com> Message-ID: <4E0DF894.4050700@comcast.net> But if there was an id conflict why would the font work in their email program, word processor, etc, but not in my app? They do get a font - it's not blank. If anyone is getting an error report (other than the font is missing) they have not reported it to me. Marty > I am going to say that there is a conflict with another font id. A lot of Mac users have damaged fonts and don't know it. Do they get another font besides the one they installer, or do they get nothing at all? What is their software or os reporting? > > Bob > > > On Jul 1, 2011, at 8:58 AM, Marty Knapp wrote: > >> I have a Mac only LiveCode standalone that requires a custom symbol font (yes, it's licensed). I use revFontLoad to put it into use. This works great on my 3 Macs and 90% of my users. The other 10% report that that it does not load. >> >> After I run revFontLoad I check the fontnames to see if the font is present and then alert the user if it hasn't loaded. If it hasn't, I then have these people manually install the font in Library>Fonts and restart. A percentage of these report that my app still doesn't find the font. In a few cases the user had a 3rd party font management program in use. 2 users reported that they had to uninstall the font program before my app would recognize the font. One was using FontExplorer and the other FontAgent, I believe. I tried FontExplorer and it worked fine for me - no conflict with revFontLoad. >> >> On a couple of occasions the people emailed me and used the font in the email to show that the font was working everywhere but in my app. >> >> I usually have people use OnyX or FontNuke to clear the font caches, though that hasn't seemed to help >> >> I've tried using graphics instead of a font, but the resolution isn't high enough for my use, so the font is required. >> >> Any suggestions for me? >> >> Marty Knapp >> >> _______________________________________________ >> use-livecode mailing list >> use-livecode at lists.runrev.com >> Please visit this url to subscribe, unsubscribe and manage your subscription preferences: >> http://lists.runrev.com/mailman/listinfo/use-livecode > > _______________________________________________ > use-livecode mailing list > use-livecode at lists.runrev.com > Please visit this url to subscribe, unsubscribe and manage your subscription preferences: > http://lists.runrev.com/mailman/listinfo/use-livecode > From mstuart at adaptcrm.com Fri Jul 1 12:43:32 2011 From: mstuart at adaptcrm.com (Mark Stuart) Date: Fri, 1 Jul 2011 09:43:32 -0700 Subject: LiveCode development for Android - compatible controls Message-ID: Hi All, I'm having a go at developing for the Android platform, and doing this on WinXP, using LC v4.6.2. I've gone thru the "Hello World" lesson on the runrev site - it worked fine in the IDE and emulator. Now as I continue the discovery period, I'm trying to find which controls work on Android. As in the lesson above the button and answer dialog work. I've created a new stack with an option button and a scrolling field. The idea is to load data into the field based on a selection from the option button. In the option button, I created a menuPick script and tested it on the Android emulator. The script wouldn't fire. I then changed to a mouseUp script in the button and that didn't fire either. The button script is simple: on menuPick pSelection answer pSelection end menuPick I put the same script into a "on mouseUp" in the option button. All this works in the IDE, but not on the Android emulator. So my question is: how many are finding these stumble blocks and where can I find more lessons/examples of how all this works. Run Rev lessons are lacking in this area. Regards, Mark Stuart BTW: check out http://www.basic4ppc.com A very complete package for the Windows and Android development environment. From bobs at twft.com Fri Jul 1 12:53:24 2011 From: bobs at twft.com (Bob Sneidar) Date: Fri, 1 Jul 2011 09:53:24 -0700 Subject: Font Problem In-Reply-To: <4E0DF894.4050700@comcast.net> References: <4E0DEE9D.1090903@comcast.net> <45080A9C-C06C-48B8-8D87-1D0690DB3FC7@twft.com> <4E0DF894.4050700@comcast.net> Message-ID: <7091E513-E347-4CF9-B193-A855D7DBEFB2@twft.com> Good point. Missed that in the original email. Bob On Jul 1, 2011, at 9:40 AM, Marty Knapp wrote: > But if there was an id conflict why would the font work in their email program, word processor, etc, but not in my app? They do get a font - it's not blank. If anyone is getting an error report (other than the font is missing) they have not reported it to me. > > Marty >> I am going to say that there is a conflict with another font id. A lot of Mac users have damaged fonts and don't know it. Do they get another font besides the one they installer, or do they get nothing at all? What is their software or os reporting? >> >> Bob From jacque at hyperactivesw.com Fri Jul 1 13:20:34 2011 From: jacque at hyperactivesw.com (J. Landman Gay) Date: Fri, 01 Jul 2011 12:20:34 -0500 Subject: LiveCode development for Android - compatible controls In-Reply-To: References: Message-ID: <4E0E01E2.6060905@hyperactivesw.com> On 7/1/11 11:43 AM, Mark Stuart wrote: > In the option button, I created a menuPick script and tested it on the > Android emulator. The script wouldn't fire. > I then changed to a mouseUp script in the button and that didn't fire > either. Option buttons don't exist on Android. The usual interface for settings and such is to provide a regular button at the bottom and switch to a list view. For screen-integrated options, use a regular button and when touched, show the Android list dialog. That isn't implemented natively yet, but you can fake it with a custom control that uses a field and some circle-icon buttons. The Android developer site has a good overview of the OS HIG. > So my question is: how many are finding these stumble blocks and where > can I find more lessons/examples of how all this works. > Run Rev lessons are lacking in this area. The best source of info is the Android release notes, which you can access from LiveCode's Help menu. That explains what is implemented so far and what will be implemented later. -- Jacqueline Landman Gay | jacque at hyperactivesw.com HyperActive Software | http://www.hyperactivesw.com From pete at mollysrevenge.com Fri Jul 1 13:59:22 2011 From: pete at mollysrevenge.com (Pete) Date: Fri, 1 Jul 2011 10:59:22 -0700 Subject: System Date/Time Format Problems Message-ID: I'm running into some major differences in the way LC displays system date and time versus what I have set up in my OSX control panel and wondering if anyone else has run into this. This is in the context of providing my users with a preference setting for how they want dates/times displayed. First problem is that OSX has 4 date and time formats (Short, Medium, Long, and Full) whereas LC only has three (Short, Abbrev, Long) so there are some inevitable discrepancies. For dates: LC Short = OSX Short LC Long = OSX Full LC Abbrev = no OSX format no LC format = OSX medium or long For times: LC Short = OSX Short LC Abbrev = OSX Short LC Long = OSX Medium no LC format = OSX Long or full Any thoughts? Pete Molly's Revenge From david at dvglasgow.wanadoo.co.uk Fri Jul 1 14:02:31 2011 From: david at dvglasgow.wanadoo.co.uk (David Glasgow) Date: Fri, 1 Jul 2011 19:02:31 +0100 Subject: [OT] Text analysis and author, anyone done it? In-Reply-To: References: Message-ID: On 1 Jul 2011, at 5:29 pm, Peter Alcibiades wrote: > I do think its possible, and has actually been done successfully. The Bible > is a difficult case since we don't have value free assessments of > authorship. Consequently it is reasonable to argue that what the programs > do is successfully implement the prejudices of their authors. > > However, when we apply this to Dickens, and then ask whether the various > completions of Edwin Drood were completed by him, and we apply it to Jane > Austen and ask whether the software shows the same person to have written > the works of Austen and Fanny Burney, we are dealing with definitely known > authorship, so we can assume that if the algorithms discriminate correctly > in these cases they will probably work on other material where authorship is > unknown. > > The case which I'm looking to apply this to is a bit more like the literary > case. There a number of texts of which the authorship is definitely known > and not subject to dispute. There is then one text whose authorship is > unknown. The question is whether it is probably by one of the known > authors The International Association of Forensic Linguistics do stuff on disputed authorship. I recall there was a summer school a couple of years back that had someone on computer assisted authorship analysis. Best Wishes, David Glasgow Carlton Glasgow Partnership i-psych.co.uk From form at nonsanity.com Fri Jul 1 14:27:48 2011 From: form at nonsanity.com (Nonsanity) Date: Fri, 1 Jul 2011 14:27:48 -0400 Subject: [OT] what RGB is blue? In-Reply-To: References: <001401cc37d2$d3257910$79706b30$@de> Message-ID: Go with the math. You can't trust your eyes. http://boingboing.net/2008/02/08/color-tile-optical-i.html Well, your eyes aren't really the culprit, if not color blind. We can see three color ranges at the cellular level. http://en.wikipedia.org/wiki/Cone_cell Your brain is the thing that's broken in that regard, as the illusion demonstrates. It gets in the way of direct interpretation of reality, as it always does. Not that our eyes are all that well put together... The retina is inside-out, so that light has to pass through tissue before it reaches the photo-receptive rods and cones on the back. (Hence the blind spot, where the wiring on the front - which the light hits first - has to punch through the retina to connect with the brain.) But no surprise there since you can see mammal embryos going through the earlier, simpler evolutionary forms of eye as each more advanced layer of instructions turns on and reshapes it further. That's evolution for you... Perfection it ignores in favor of "just good enough to breed". :) ~ Chris Innanen ~ Nonsanity >> On 1 jul 2011, at 11:39, Tiemo Hollmann TB wrote: > >> > >>> Hello, > >>> > >>> I am taking the mousecolor at different points from an image by script > >> (not > >>> by clicking). I would like to analyse if the color I've taken is a > "kind > >> of > >>> blue", or another color. I want to change the backgroundcolor of an > >> image. > >>> The background is always blue, but different blues and changing over > the > >>> background. So what I want to do is to verify, what is background and > >> what > >>> is foreground of my image. > >>> > >>> 100% pure blue would be 0,0,255. But for a human being 25,75,130 > >> (greyblue) > >>> is also still blue, but 240,20,180 is pink, though the third RGB value > is > >>> higher as in my greyblue. > >>> > >>> So I can't just check only the third RGB value, neither the sum or > cross > >>> total. Has anybody ever heard, if you can define at all by math "what > is > >>> blue"? > >>> > >>> Any color specialist around here? > >>> > >>> Tiemo > From dan at clearvisiontech.com Fri Jul 1 15:01:08 2011 From: dan at clearvisiontech.com (Dan Friedman) Date: Fri, 1 Jul 2011 12:01:08 -0700 Subject: Approved! In-Reply-To: References: Message-ID: <220D66C7-5D35-4F03-A045-D53AE31EC7CE@clearvisiontech.com> Hey all... Apple just approved my second app, "Custom Blind Timer". It's a timer to run your poker tournament. Like the last app, this one was also created 100% in LiveCode. http://itunes.apple.com/us/app/custom-blind-timer/id446493355?mt=8&ls=1 -Dan From bobs at twft.com Fri Jul 1 16:24:42 2011 From: bobs at twft.com (Bob Sneidar) Date: Fri, 1 Jul 2011 13:24:42 -0700 Subject: [OT] what RGB is blue? In-Reply-To: References: <001401cc37d2$d3257910$79706b30$@de> Message-ID: <46EA8E9B-CA91-49AC-A10D-330F10C4F9A1@twft.com> Well that is the point I was making, but when I see how badly my mind has to misinterpret color in order to make sense of an image, I begin to doubt whether anything I see is valid or not. Bob On Jul 1, 2011, at 11:27 AM, Nonsanity wrote: > Go with the math. You can't trust your eyes. > > http://boingboing.net/2008/02/08/color-tile-optical-i.html From m.schonewille at economy-x-talk.com Fri Jul 1 16:29:40 2011 From: m.schonewille at economy-x-talk.com (Mark Schonewille) Date: Fri, 1 Jul 2011 22:29:40 +0200 Subject: LiveCode.tv Event - Who are the presenters In-Reply-To: <9DAFA3BD-E4C4-4A92-86C6-6B7A888F5CCB@economy-x-talk.com> References: <9DAFA3BD-E4C4-4A92-86C6-6B7A888F5CCB@economy-x-talk.com> Message-ID: <4746D475-FA61-4E7D-A93B-E7B7D96440AD@economy-x-talk.com> I haven't got a reply yet. If you are presenting tomorrow and I don't know about it, please send me a message immediately. -- Mark > Hi, > > This is an important message. > > I haven't been in touch with Bj?rnke the past few days and thus I don't know who are going to present. I know of one person that he will do a presentation, but now I don't know who is the other person. > > If you are presenting tomorrow, please send me a message and tell me what the subject of your presentation is. > > -- > Best regards, > > Mark Schonewille > > Economy-x-Talk Consulting and Software Engineering > Homepage: http://economy-x-talk.com > Twitter: http://twitter.com/xtalkprogrammer > KvK: 50277553 > > New: Download the Installer Maker Plugin 1.6 for LiveCode here http://qery.us/ce From jrioux at benedictine.edu Fri Jul 1 16:48:49 2011 From: jrioux at benedictine.edu (Jean Rioux) Date: Fri, 1 Jul 2011 15:48:49 -0500 Subject: Enhanced QT External: Intel-Only? Message-ID: Does anyone know whether this external is "Intel-only"? I just read one LiveCode programmer's guidelines for submission to the Mac App Store (revUp issue 100), which mentioned that the app, including all its externals, must be "Intel-only". "All Mac Builds Require to be Intel Only. This means no universal or PPC builds at all, this also includes all externals and helper applications located within your application bundle. I found out the hard way about this! All binaries have to be Intel Only." I'm not sure whether the question even applies to the Enhanced QT External, but my prospective Mac App submission does use it. Any help would be greatly appreciated. I'd rather not "find out the hard way" that I cannot use it. Jean Rioux From jperryl at ecs.fullerton.edu Fri Jul 1 16:50:13 2011 From: jperryl at ecs.fullerton.edu (Judy Perry) Date: Fri, 1 Jul 2011 13:50:13 -0700 (PDT) Subject: [OT] Amiga OS in 2011 In-Reply-To: References: Message-ID: I would hardly consider graffiti an improvement... Judy On Mon, 27 Jun 2011, Bob Sneidar wrote: > Let me guess, you dropped it into a toilet... OH WAIT! It wouldn't fit, now would it? LOL! Actually, I was sad to see the newton go. Apple did all the R&D, and then others came along and improved upon it. Way of the world I guess. > > Bob From stephenREVOLUTION2 at barncard.com Fri Jul 1 17:25:40 2011 From: stephenREVOLUTION2 at barncard.com (stephen barncard) Date: Fri, 1 Jul 2011 14:25:40 -0700 Subject: Enhanced QT External: Intel-Only? In-Reply-To: References: Message-ID: Everything Apple Desktop is 'Intel Only' today. PPC support is fading very fast with most other vendors. Livecode keeps it available for compiling now, but the App store is very strict about this. Runrev offers Mac PPC support for legacy, education and Richmond. :) On 1 July 2011 13:48, Jean Rioux wrote: > Does anyone know whether this external is "Intel-only"? I just read one > LiveCode programmer's guidelines for submission to the Mac App Store (revUp > issue 100), which mentioned that the app, including all its externals, must > be "Intel-only". > > "All Mac Builds Require to be Intel Only. This means no universal or PPC > builds at all, this also includes all externals and helper applications > located within your application bundle. I found out the hard way about this! > All binaries have to be Intel Only." > > I'm not sure whether the question even applies to the Enhanced QT External, > but my prospective Mac App submission does use it. > > Any help would be greatly appreciated. I'd rather not "find out the hard > way" that I cannot use it. > > Jean Rioux > _______________________________________________ > use-livecode mailing list > use-livecode at lists.runrev.com > Please visit this url to subscribe, unsubscribe and manage your > subscription preferences: > http://lists.runrev.com/mailman/listinfo/use-livecode > -- Stephen Barncard San Francisco Ca. USA more about sqb From shaosean at wehostmacs.com Fri Jul 1 18:17:44 2011 From: shaosean at wehostmacs.com (Shao Sean) Date: Fri, 1 Jul 2011 18:17:44 -0400 Subject: Enhanced QT External: Intel-Only? Message-ID: <2A51AE50-7DF6-4E2D-8D9D-DBF9E9A5974F@wehostmacs.com> hey.. i am still on ppc.. richard, we need to ban together against the intel heathens :P you can always re-compile the external to intel-only to make doubly sure that it is, but i think the released version is universal as it works on my G4.. From devin_asay at byu.edu Fri Jul 1 18:33:57 2011 From: devin_asay at byu.edu (Devin Asay) Date: Fri, 1 Jul 2011 16:33:57 -0600 Subject: PNGs and screenGamma Message-ID: <8F420390-07FD-4317-9ADF-94BBCE36FF27@byu.edu> Hi all, About a year and a half ago Scott Rossi posted this useful tip: > This might be useful for folks delivering image-heavy apps on Mac OS X 10.6 > and later... > > Apparently with Snow Leopard, Apple has changed the default gamma of the > system to 2.2 (used to be 1.8). > > html> > > I was recently throwing some PNG images on a card on Rev that looked > significantly darker than where they were created (in Photoshop), and for > the life of me could not figure out why. I'm still searching for a reliable > way to strip the gamma information from PNG images, but after finding the > above article, I tried changing the screenGamma in Rev to 2.2 and amazingly > (or maybe expectedly) the image seems to display properly. > > set the screenGamma to 2.2 > > I need to do some more testing, but it appears that when delivering apps on > Snow Leopard that make heavy use of PNGs, one will need to do something to > the effect of: > > if "mac" is in platform() and systemVersion() > 10.5 then ... > > Also used the following method to force refresh the display of the image: > > set text of img 1 to text of img 1 > > Hope this saves some head banging for other folks using images. > > Regards, > > Scott Rossi > Creative Director > Tactile Media, UX Design Just now, I've run into this exact same problem. I tried applying Scott's workaround, only to discover that it only works with Revolution 4.0 and earlier. Any time I try to display these same PNGs in LiveCode 4.5 or higher they come out significantly darker, regardless of whether I reset the screenGamma. Oddly, PNGs that were originally produced in OS X 10.5 do not come out darker, so it definitely has something to do with 10.6. Only the latest versions of LC don't allow me to correct the problem any more. Has anyone run into this, and has anyone found a solution? Regards, Devin Devin Asay Humanities Technology and Research Support Center Brigham Young University From warren at warrensweb.us Fri Jul 1 18:37:09 2011 From: warren at warrensweb.us (Warren Samples) Date: Fri, 1 Jul 2011 17:37:09 -0500 Subject: Enhanced QT External: Intel-Only? In-Reply-To: <2A51AE50-7DF6-4E2D-8D9D-DBF9E9A5974F@wehostmacs.com> References: <2A51AE50-7DF6-4E2D-8D9D-DBF9E9A5974F@wehostmacs.com> Message-ID: <201107011737.09765.warren@warrensweb.us> On Friday, July 01, 2011 05:17:44 PM Shao Sean wrote: > hey.. i am still on ppc.. richard, we need to ban together against the > intel heathens :P > > you can always re-compile the external to intel-only to make doubly > sure that it is, but i think the released version is universal as it > works on my G4.. Or, you could just ignore the AppStore. Warren From psahores at free.fr Fri Jul 1 18:56:58 2011 From: psahores at free.fr (Pierre Sahores) Date: Sat, 2 Jul 2011 00:56:58 +0200 Subject: Approved! In-Reply-To: <220D66C7-5D35-4F03-A045-D53AE31EC7CE@clearvisiontech.com> References: <220D66C7-5D35-4F03-A045-D53AE31EC7CE@clearvisiontech.com> Message-ID: <41D3F7B1-7E4A-47E5-AB54-CF1654890B6C@free.fr> Bravo :D Le 1 juil. 2011 ? 21:01, Dan Friedman a ?crit : > Hey all... Apple just approved my second app, "Custom Blind Timer". It's a timer to run your poker tournament. Like the last app, this one was also created 100% in LiveCode. > > http://itunes.apple.com/us/app/custom-blind-timer/id446493355?mt=8&ls=1 > > -Dan > _______________________________________________ > use-livecode mailing list > use-livecode at lists.runrev.com > Please visit this url to subscribe, unsubscribe and manage your subscription preferences: > http://lists.runrev.com/mailman/listinfo/use-livecode > -- Pierre Sahores mobile : (33) 6 03 95 77 70 www.woooooooords.com www.sahores-conseil.com From pete at mollysrevenge.com Fri Jul 1 19:00:25 2011 From: pete at mollysrevenge.com (Pete) Date: Fri, 1 Jul 2011 16:00:25 -0700 Subject: Upper/Lower case issue Message-ID: I have two radio buttons whose names are identical except one is in lowercase and the other is in uppercase. They are in a group which should prevent more than one of them being set. The following statement ends up setting the hilite of both buttons: set the hilitedbuttonname of *group* "xyz" to tVar I tried setting the casesensitive property but the dictionary clearly states that it doesn't affect object names. There's a valid reason for the buttons being named that way and I'd rather not change them unless I have to - any ideas on how to avoid this problem? Pete Molly's Revenge From devin_asay at byu.edu Fri Jul 1 19:04:58 2011 From: devin_asay at byu.edu (Devin Asay) Date: Fri, 1 Jul 2011 17:04:58 -0600 Subject: Upper/Lower case issue In-Reply-To: References: Message-ID: Pete, Does it work to set the hilitedButton instead of the hilitedButtonName? The former uses the number of the button, I believe. Devin On Jul 1, 2011, at 5:00 PM, Pete wrote: > I have two radio buttons whose names are identical except one is in > lowercase and the other is in uppercase. They are in a group which should > prevent more than one of them being set. > > The following statement ends up setting the hilite of both buttons: > > set the hilitedbuttonname of *group* "xyz" to tVar > > I tried setting the casesensitive property but the dictionary clearly states > that it doesn't affect object names. There's a valid reason for the buttons > being named that way and I'd rather not change them unless I have to - any > ideas on how to avoid this problem? > > Pete > Molly's Revenge > _______________________________________________ > use-livecode mailing list > use-livecode at lists.runrev.com > Please visit this url to subscribe, unsubscribe and manage your subscription preferences: > http://lists.runrev.com/mailman/listinfo/use-livecode Devin Asay Humanities Technology and Research Support Center Brigham Young University From m.schonewille at economy-x-talk.com Fri Jul 1 19:06:31 2011 From: m.schonewille at economy-x-talk.com (Mark Schonewille) Date: Sat, 2 Jul 2011 01:06:31 +0200 Subject: Upper/Lower case issue In-Reply-To: References: Message-ID: Pete, Use different names and equal labels. There is NEVER a valid reason to give objects the same name. -- Best regards, Mark Schonewille Economy-x-Talk Consulting and Software Engineering Homepage: http://economy-x-talk.com Twitter: http://twitter.com/xtalkprogrammer KvK: 50277553 New: Download the Installer Maker Plugin 1.6 for LiveCode here http://qery.us/ce On 2 jul 2011, at 01:00, Pete wrote: > I have two radio buttons whose names are identical except one is in > lowercase and the other is in uppercase. They are in a group which should > prevent more than one of them being set. > > The following statement ends up setting the hilite of both buttons: > > set the hilitedbuttonname of *group* "xyz" to tVar > > I tried setting the casesensitive property but the dictionary clearly states > that it doesn't affect object names. There's a valid reason for the buttons > being named that way and I'd rather not change them unless I have to - any > ideas on how to avoid this problem? > > Pete > Molly's Revenge From pete at mollysrevenge.com Fri Jul 1 19:18:08 2011 From: pete at mollysrevenge.com (Pete) Date: Fri, 1 Jul 2011 16:18:08 -0700 Subject: Upper/Lower case issue In-Reply-To: References: Message-ID: Sorry Mark, I don't agree with you. Too complicated to explain by email but I can assure there is a good reason. Pete Molly's Revenge On Fri, Jul 1, 2011 at 4:06 PM, Mark Schonewille < m.schonewille at economy-x-talk.com> wrote: > Pete, > > Use different names and equal labels. There is NEVER a valid reason to give > objects the same name. > > -- > Best regards, > > Mark Schonewille > > Economy-x-Talk Consulting and Software Engineering > Homepage: http://economy-x-talk.com > Twitter: http://twitter.com/xtalkprogrammer > KvK: 50277553 > > New: Download the Installer Maker Plugin 1.6 for LiveCode here > http://qery.us/ce > > On 2 jul 2011, at 01:00, Pete wrote: > > > I have two radio buttons whose names are identical except one is in > > lowercase and the other is in uppercase. They are in a group which > should > > prevent more than one of them being set. > > > > The following statement ends up setting the hilite of both buttons: > > > > set the hilitedbuttonname of *group* "xyz" to tVar > > > > I tried setting the casesensitive property but the dictionary clearly > states > > that it doesn't affect object names. There's a valid reason for the > buttons > > being named that way and I'd rather not change them unless I have to - > any > > ideas on how to avoid this problem? > > > > Pete > > Molly's Revenge > > > > _______________________________________________ > use-livecode mailing list > use-livecode at lists.runrev.com > Please visit this url to subscribe, unsubscribe and manage your > subscription preferences: > http://lists.runrev.com/mailman/listinfo/use-livecode > > From m.schonewille at economy-x-talk.com Fri Jul 1 19:27:21 2011 From: m.schonewille at economy-x-talk.com (Mark Schonewille) Date: Sat, 2 Jul 2011 01:27:21 +0200 Subject: Upper/Lower case issue In-Reply-To: References: Message-ID: <64050804-7291-4338-A5FF-DA56C4CE734C@economy-x-talk.com> Pete, I tried to make something clear something important, but you're free to disagree of course. -- Best regards, Mark Schonewille Economy-x-Talk Consulting and Software Engineering Homepage: http://economy-x-talk.com Twitter: http://twitter.com/xtalkprogrammer KvK: 50277553 New: Download the Installer Maker Plugin 1.6 for LiveCode here http://qery.us/ce On 2 jul 2011, at 01:18, Pete wrote: > Sorry Mark, I don't agree with you. Too complicated to explain by email but > I can assure there is a good reason. > Pete > Molly's Revenge From pete at mollysrevenge.com Fri Jul 1 19:40:02 2011 From: pete at mollysrevenge.com (Pete) Date: Fri, 1 Jul 2011 16:40:02 -0700 Subject: Upper/Lower case issue In-Reply-To: References: Message-ID: Good call Devin, I found a way to do it using hilitedButton, coupled with the fact that comparing variables obeys the casesensitive setting. I have a repeat loop that cycles through all the buttons in the group, puts their short name into a variable, compares the short name variable with the name I'm looking for and sets the hilitedButton of the group if the names match. Thanks for the idea! Pete Molly's Revenge On Fri, Jul 1, 2011 at 4:04 PM, Devin Asay wrote: > Pete, > > Does it work to set the hilitedButton instead of the hilitedButtonName? The > former uses the number of the button, I believe. > > Devin > > On Jul 1, 2011, at 5:00 PM, Pete wrote: > > > I have two radio buttons whose names are identical except one is in > > lowercase and the other is in uppercase. They are in a group which > should > > prevent more than one of them being set. > > > > The following statement ends up setting the hilite of both buttons: > > > > set the hilitedbuttonname of *group* "xyz" to tVar > > > > I tried setting the casesensitive property but the dictionary clearly > states > > that it doesn't affect object names. There's a valid reason for the > buttons > > being named that way and I'd rather not change them unless I have to - > any > > ideas on how to avoid this problem? > > > > Pete > > Molly's Revenge > > _______________________________________________ > > use-livecode mailing list > > use-livecode at lists.runrev.com > > Please visit this url to subscribe, unsubscribe and manage your > subscription preferences: > > http://lists.runrev.com/mailman/listinfo/use-livecode > > Devin Asay > Humanities Technology and Research Support Center > Brigham Young University > > > _______________________________________________ > use-livecode mailing list > use-livecode at lists.runrev.com > Please visit this url to subscribe, unsubscribe and manage your > subscription preferences: > http://lists.runrev.com/mailman/listinfo/use-livecode > > From pete at mollysrevenge.com Fri Jul 1 19:41:16 2011 From: pete at mollysrevenge.com (Pete) Date: Fri, 1 Jul 2011 16:41:16 -0700 Subject: Upper/Lower case issue In-Reply-To: <64050804-7291-4338-A5FF-DA56C4CE734C@economy-x-talk.com> References: <64050804-7291-4338-A5FF-DA56C4CE734C@economy-x-talk.com> Message-ID: That's OK Mark, I agree with what you said 99.99999% of the time, it's just that there is a valid reason for what I'm doing in this particular instance. Pete Molly's Revenge On Fri, Jul 1, 2011 at 4:27 PM, Mark Schonewille < m.schonewille at economy-x-talk.com> wrote: > Pete, > > I tried to make something clear something important, but you're free to > disagree of course. > > -- > Best regards, > > Mark Schonewille > > Economy-x-Talk Consulting and Software Engineering > Homepage: http://economy-x-talk.com > Twitter: http://twitter.com/xtalkprogrammer > KvK: 50277553 > > New: Download the Installer Maker Plugin 1.6 for LiveCode here > http://qery.us/ce > > On 2 jul 2011, at 01:18, Pete wrote: > > > Sorry Mark, I don't agree with you. Too complicated to explain by email > but > > I can assure there is a good reason. > > Pete > > Molly's Revenge > > > _______________________________________________ > use-livecode mailing list > use-livecode at lists.runrev.com > Please visit this url to subscribe, unsubscribe and manage your > subscription preferences: > http://lists.runrev.com/mailman/listinfo/use-livecode > > From m.schonewille at economy-x-talk.com Fri Jul 1 19:56:02 2011 From: m.schonewille at economy-x-talk.com (Mark Schonewille) Date: Sat, 2 Jul 2011 01:56:02 +0200 Subject: Enhanced QT External: Intel-Only? In-Reply-To: References: Message-ID: Jean, You can use the lipo command line to make an Intel bundle. Enter the following in the terminal lipo /path/to/EnhancedQT.bundle/Contents/MacOS/EnhancedQT -thin i386 -output ~/desktop/enhqt and press return. Make sure that you use the path to the actual executable rather than the bundle. Now copy the original bundle, remove the EnhancedQT executable from it and replace it with enhqt (which is on your desktop). Rename enhqt to EnhancedQT rename EnhancedQT copy.bundle into EnhancedQTx86.bundle. After executing this line, execute touch /path/to/EnhancedQTx86.bundle This time you can use the path to the bundle instead of the executable. Now select the package EnhancedQTx86.bundle and press command-I. The info window should now state that you have an intel-only package. I haven't tested the result but I expect it to work. -- Best regards, Mark Schonewille Economy-x-Talk Consulting and Software Engineering Homepage: http://economy-x-talk.com Twitter: http://twitter.com/xtalkprogrammer KvK: 50277553 New: Download the Installer Maker Plugin 1.6 for LiveCode here http://qery.us/ce On 1 jul 2011, at 22:48, Jean Rioux wrote: > Does anyone know whether this external is "Intel-only"? I just read one LiveCode programmer's guidelines for submission to the Mac App Store (revUp issue 100), which mentioned that the app, including all its externals, must be "Intel-only". > > "All Mac Builds Require to be Intel Only. This means no universal or PPC builds at all, this also includes all externals and helper applications located within your application bundle. I found out the hard way about this! All binaries have to be Intel Only." > > I'm not sure whether the question even applies to the Enhanced QT External, but my prospective Mac App submission does use it. > > Any help would be greatly appreciated. I'd rather not "find out the hard way" that I cannot use it. > > Jean Rioux From mwieder at ahsoftware.net Fri Jul 1 20:56:40 2011 From: mwieder at ahsoftware.net (Mark Wieder) Date: Fri, 1 Jul 2011 17:56:40 -0700 Subject: Upper/Lower case issue In-Reply-To: References: <64050804-7291-4338-A5FF-DA56C4CE734C@economy-x-talk.com> Message-ID: <15121444796.20110701175640@ahsoftware.net> Pete- Friday, July 1, 2011, 4:41:16 PM, you wrote: > That's OK Mark, I agree with what you said 99.99999% of the time, it's just > that there is a valid reason for what I'm doing in this particular instance. It seems like you found a way out of your predicament, but I'm still with Mark on this. If you think you need to give the same name to two buttons there's another way of getting the result you're looking for. -- -Mark Wieder mwieder at ahsoftware.net From m.schonewille at economy-x-talk.com Fri Jul 1 21:24:54 2011 From: m.schonewille at economy-x-talk.com (Mark Schonewille) Date: Sat, 2 Jul 2011 03:24:54 +0200 Subject: LiveCode.tv Event #30 Message-ID: Hi guys 'n gals, This time, the organisation of the event is somewhat chaotic. First, one presenter decided to cancel, then I simply lost another presenter --I have no idea who he or she might have been-- and Bj?rnke is currently unavailable. Fortunately, Chipp and Judy have been willing to help out in the very last moment. This weekend's event is scheduled for 2 July 2011 at 19:00h GMT (Sat. 22:00 in Moscow, Sat. 20:00 in Paris, Sat. 14:00 in New York, Sat. 11:00 in Los Angeles, Sun. 04:00 in Sydney, Sun. 03:00 in Tokyo, Sun. 2:00 in Beijing; check the correct time in your own time zone). Judy will start with a beginners session. She will discuss the creation and use of tab buttons and menu buttons. If you have just started using LiveCode, then this is a must-see. Chipp is going to show his new altButton droptool, InterfaceDesigner and, if time permits, the MagicCarpet AutoUpdate Architecture. There is now an open-source version of MagicCarpet, maintained by Richard Gaskin. The European HyperCard User Group (eHUG, http://www.ehug.info) will raffle off the e-book Take Control of Upgrading to Lion courtesy of TidBITS. You can fine more info on the TC series at http://www.takecontrolbooks.com . During the event, you will also be able to get a 25% discount on a license for Installer Maker Plugin for LiveCode. You can find more info about this plug-in at http://qery.us/ce and go to http://livetalk.us/im for the discounted license. Make sure to use ChatRev during the happening, otherwise you won't know where to watch the streams, which will be announced in ChatRev. Download ChatRev here: http://bjoernke.com?target=chatrev or enter in the message box: go stack URL "http://bjoernke.com/chatrev/chatrev1.3b3.rev" We are looking for new presenters! If you would like to participate in the event, please head over to http://blog.livecode.tv/participate/ for ideas and leave a comment and/or contact Mark or Bj?rnke. An easy way to contact us is through this web form http://qery.us/du See also the following page for the same information: http://livecode.tv -- Best regards, Mark Schonewille Economy-x-Talk Consulting and Software Engineering Homepage: http://economy-x-talk.com Twitter: http://twitter.com/xtalkprogrammer KvK: 50277553 New: Download the Installer Maker Plugin 1.6 for LiveCode here http://qery.us/ce From chipp at chipp.com Fri Jul 1 23:49:59 2011 From: chipp at chipp.com (Chipp Walters) Date: Fri, 1 Jul 2011 22:49:59 -0500 Subject: LiveCode.tv Event #30 In-Reply-To: References: Message-ID: Also, I'll be giving away a free copy of Interface Designer to all who attend! Look forward to seeing you all tomorrow! On Fri, Jul 1, 2011 at 8:24 PM, Mark Schonewille < m.schonewille at economy-x-talk.com> wrote: > Hi guys 'n gals, > > This time, the organisation of the event is somewhat chaotic. First, one > presenter decided to cancel, then I simply lost another presenter --I have > no idea who he or she might have been-- and Bj?rnke is currently > unavailable. Fortunately, Chipp and Judy have been willing to help out in > the very last moment. > > This weekend's event is scheduled for 2 July 2011 at 19:00h GMT (Sat. 22:00 > in Moscow, Sat. 20:00 in Paris, Sat. 14:00 in New York, Sat. 11:00 in Los > Angeles, Sun. 04:00 in Sydney, Sun. 03:00 in Tokyo, Sun. 2:00 in Beijing; > check the correct time in your own time zone). > > Judy will start with a beginners session. She will discuss the creation and > use of tab buttons and menu buttons. If you have just started using > LiveCode, then this is a must-see. > > Chipp is going to show his new altButton droptool, InterfaceDesigner and, > if time permits, the MagicCarpet AutoUpdate Architecture. There is now an > open-source version of MagicCarpet, maintained by Richard Gaskin. > > The European HyperCard User Group (eHUG, http://www.ehug.info) will raffle > off the e-book Take Control of Upgrading to Lion courtesy of TidBITS. You > can fine more info on the TC series at http://www.takecontrolbooks.com . > > During the event, you will also be able to get a 25% discount on a license > for Installer Maker Plugin for LiveCode. You can find more info about this > plug-in at http://qery.us/ce and go to http://livetalk.us/im for the > discounted license. > > Make sure to use ChatRev during the happening, otherwise you won't know > where to watch the streams, which will be announced in ChatRev. Download > ChatRev here: > http://bjoernke.com?target=chatrev > or enter in the message box: > go stack URL "http://bjoernke.com/chatrev/chatrev1.3b3.rev" > > We are looking for new presenters! If you would like to participate in the > event, please head over to http://blog.livecode.tv/participate/ for ideas > and leave a comment and/or contact Mark or Bj?rnke. An easy way to contact > us is through this web form http://qery.us/du > > See also the following page for the same information: > http://livecode.tv > > -- > Best regards, > > Mark Schonewille > > Economy-x-Talk Consulting and Software Engineering > Homepage: http://economy-x-talk.com > Twitter: http://twitter.com/xtalkprogrammer > KvK: 50277553 > > New: Download the Installer Maker Plugin 1.6 for LiveCode here > http://qery.us/ce > > > _______________________________________________ > use-livecode mailing list > use-livecode at lists.runrev.com > Please visit this url to subscribe, unsubscribe and manage your > subscription preferences: > http://lists.runrev.com/mailman/listinfo/use-livecode > -- Chipp Walters CEO, Shafer Walters Group, Inc. From DunbarX at aol.com Sat Jul 2 01:01:47 2011 From: DunbarX at aol.com (Craig Newman) Date: Sat, 2 Jul 2011 05:01:47 +0000 (UTC) Subject: Invitation to connect on LinkedIn Message-ID: <909850851.5111933.1309582907660.JavaMail.app@ela4-bed80.prod> LinkedIn ------------ I'd like to add you to my professional network on LinkedIn. - Craig Craig Newman Owner at Starfire Lighting, Inc. Greater New York City Area Confirm that you know Craig Newman https://www.linkedin.com/e/vqip3p-gpm3ummh-5b/isd/3408965419/n7g-k5mp/ -- (c) 2011, LinkedIn Corporation From DunbarX at aol.com Sat Jul 2 01:02:03 2011 From: DunbarX at aol.com (Craig Newman) Date: Sat, 2 Jul 2011 05:02:03 +0000 (UTC) Subject: Invitation to connect on LinkedIn Message-ID: <1806583153.5178085.1309582923541.JavaMail.app@ela4-bed34.prod> LinkedIn ------------ I'd like to add you to my professional network on LinkedIn. - Craig Craig Newman Owner at Starfire Lighting, Inc. Greater New York City Area Confirm that you know Craig Newman https://www.linkedin.com/e/-2aaum3-gpm3uyvf-12/isd/3408966244/4XZ9Ljdo/ -- (c) 2011, LinkedIn Corporation From pete at mollysrevenge.com Sat Jul 2 01:27:39 2011 From: pete at mollysrevenge.com (Pete) Date: Fri, 1 Jul 2011 22:27:39 -0700 Subject: Upper/Lower case issue In-Reply-To: <15121444796.20110701175640@ahsoftware.net> References: <64050804-7291-4338-A5FF-DA56C4CE734C@economy-x-talk.com> <15121444796.20110701175640@ahsoftware.net> Message-ID: But they're not the same name - one is upper case and one is lower case. LC made a decision to treat them the same since casesensitive doesn't apply to object names, but they are different by any other definition. In summary, the names of the buttons are strings that define elements of date formats. For example, %m means zero suppressed month number and %M means 2-digit month numbers with leading zero, %d means zero suppressed day number, %D means 2 digit day number, etc. This is a very common convention for defining date strings as I'm sure you know. I use the button labels to show a more meaningful description to the user and then store the names of the labels in a preferences file. That way, I can pass the date definition string directly to my date formatting handler. Next time the card is opened, I need to get the settings from the prefs file and set the appropriate buttons. I'm sure there are other ways to do this - with custom properties for example. But I can't think of any practical downside to using the button names, although I'm ready to hear any. Pete Molly's Revenge On Fri, Jul 1, 2011 at 5:56 PM, Mark Wieder wrote: > Pete- > > Friday, July 1, 2011, 4:41:16 PM, you wrote: > > > That's OK Mark, I agree with what you said 99.99999% of the time, it's > just > > that there is a valid reason for what I'm doing in this particular > instance. > > It seems like you found a way out of your predicament, but I'm still > with Mark on this. If you think you need to give the same name to two > buttons there's another way of getting the result you're looking for. > > -- > -Mark Wieder > mwieder at ahsoftware.net > > > _______________________________________________ > use-livecode mailing list > use-livecode at lists.runrev.com > Please visit this url to subscribe, unsubscribe and manage your > subscription preferences: > http://lists.runrev.com/mailman/listinfo/use-livecode > > From dunbarx at aol.com Sat Jul 2 01:38:47 2011 From: dunbarx at aol.com (dunbarx at aol.com) Date: Sat, 02 Jul 2011 01:38:47 -0400 Subject: Upper/Lower case issue In-Reply-To: <15121444796.20110701175640@ahsoftware.net> References: <64050804-7291-4338-A5FF-DA56C4CE734C@economy-x-talk.com> <15121444796.20110701175640@ahsoftware.net> Message-ID: <8CE06917330DD6C-1D78-1FBF3@webmail-m070.sysops.aol.com> Pete. In the old HC days, before there was a name AND a label, I might have understood that you might have a valid reason to use identical names. You are aware that these two properties exist, and still have your reasons. Can you say briefly what they are? I know you said they were too complicated, but just a one or two liner? Craig Newman -----Original Message----- From: Mark Wieder To: How to use LiveCode Sent: Fri, Jul 1, 2011 8:56 pm Subject: Re: Upper/Lower case issue Pete- Friday, July 1, 2011, 4:41:16 PM, you wrote: > That's OK Mark, I agree with what you said 99.99999% of the time, it's just > that there is a valid reason for what I'm doing in this particular instance. It seems like you found a way out of your predicament, but I'm still with Mark on this. If you think you need to give the same name to two buttons there's another way of getting the result you're looking for. -- -Mark Wieder mwieder at ahsoftware.net _______________________________________________ use-livecode mailing list use-livecode at lists.runrev.com Please visit this url to subscribe, unsubscribe and manage your subscription preferences: http://lists.runrev.com/mailman/listinfo/use-livecode From henshaw at me.com Sat Jul 2 07:09:41 2011 From: henshaw at me.com (Andy Henshaw) Date: Sat, 02 Jul 2011 12:09:41 +0100 Subject: Approved! In-Reply-To: <220D66C7-5D35-4F03-A045-D53AE31EC7CE@clearvisiontech.com> References: <220D66C7-5D35-4F03-A045-D53AE31EC7CE@clearvisiontech.com> Message-ID: <0A148280-BC23-4587-B3F3-E70701FEECB1@me.com> Well done, your app looks very slick! Andy On 1 Jul 2011, at 20:01, Dan Friedman wrote: > Hey all... Apple just approved my second app, "Custom Blind Timer". It's a timer to run your poker tournament. Like the last app, this one was also created 100% in LiveCode. > > http://itunes.apple.com/us/app/custom-blind-timer/id446493355?mt=8&ls=1 > > -Dan > _______________________________________________ > use-livecode mailing list > use-livecode at lists.runrev.com > Please visit this url to subscribe, unsubscribe and manage your subscription preferences: > http://lists.runrev.com/mailman/listinfo/use-livecode From pmbrig at gmail.com Sat Jul 2 08:43:24 2011 From: pmbrig at gmail.com (Peter Brigham MD) Date: Sat, 2 Jul 2011 08:43:24 -0400 Subject: System Date/Time Format Problems In-Reply-To: References: Message-ID: <7C561176-82BD-45A5-9E68-A470C13DB8D4@gmail.com> There may be a better way, but you might have to use a shell call. Execute the following in the message box: put shell("man date | col -b") and look at the options. You can get any format you want, and this way you would have full control over date display format in LC. -- Peter Peter M. Brigham pmbrig at gmail.com http://home.comcast.net/~pmbrig On Jul 1, 2011, at 1:59 PM, Pete wrote: > I'm running into some major differences in the way LC displays system date > and time versus what I have set up in my OSX control panel and wondering if > anyone else has run into this. This is in the context of providing my users > with a preference setting for how they want dates/times displayed. > > First problem is that OSX has 4 date and time formats (Short, Medium, Long, > and Full) whereas LC only has three (Short, Abbrev, Long) so there are some > inevitable discrepancies. > > For dates: > LC Short = OSX Short > LC Long = OSX Full > LC Abbrev = no OSX format > no LC format = OSX medium or long > > > For times: > LC Short = OSX Short > LC Abbrev = OSX Short > LC Long = OSX Medium > no LC format = OSX Long or full > > Any thoughts? > > Pete > Molly's Revenge > _______________________________________________ > use-livecode mailing list > use-livecode at lists.runrev.com > Please visit this url to subscribe, unsubscribe and manage your subscription preferences: > http://lists.runrev.com/mailman/listinfo/use-livecode From richmondmathewson at gmail.com Sat Jul 2 09:17:11 2011 From: richmondmathewson at gmail.com (Richmond Mathewson) Date: Sat, 02 Jul 2011 16:17:11 +0300 Subject: PNGs and screenGamma In-Reply-To: <8F420390-07FD-4317-9ADF-94BBCE36FF27@byu.edu> References: <8F420390-07FD-4317-9ADF-94BBCE36FF27@byu.edu> Message-ID: <4E0F1A57.3030006@gmail.com> Interestingly enough . . . my Pentium 4 running Ubuntu 11.04 has a screen gamma of 2.2 . . . So, I can only conclude, that while Apple's bye-line may be "Think Different", as Lion begins to look like something halfway between GNOME 3 and UNITY, and Snow Leopard sets a screen gamma to 2.2, the real thinking is not so much "different" as conformist. On 07/02/2011 01:33 AM, Devin Asay wrote: > Hi all, > > About a year and a half ago Scott Rossi posted this useful tip: > >> This might be useful for folks delivering image-heavy apps on Mac OS X 10.6 >> and later... >> >> Apparently with Snow Leopard, Apple has changed the default gamma of the >> system to 2.2 (used to be 1.8). >> >> > html> >> >> I was recently throwing some PNG images on a card on Rev that looked >> significantly darker than where they were created (in Photoshop), and for >> the life of me could not figure out why. I'm still searching for a reliable >> way to strip the gamma information from PNG images, but after finding the >> above article, I tried changing the screenGamma in Rev to 2.2 and amazingly >> (or maybe expectedly) the image seems to display properly. >> >> set the screenGamma to 2.2 I have just set up a merry little stack that can set the screen gamma from 2.2 to 1.8 and back again [rocket science it ain't]. It sports 4 open source PNG images; now, reading this posting I would expect the appearance of those PNGs to change a spot when I set the screenGamma; but they don't seem to [at least on Linux]: LC 4.5. Download it and play with it: http://andregarzia.on-rev.com/richmond/STUFF/gamma.rev My PPC macMini returns a screen gamma of 1.7 (mainly because I've been bu**ering around with the Mac to suit my needs); agin, setting the SG to 2.2 results in nothing: LC 4.5. Ar-har-har! When I open the same stack under LC/RR 4.0, having, supposedly, set the SG to 2.2 when it was open in LC 4.5 it reports the SG as 1.7. So the screen gamma Dinge-an-sich is NOT reset. Doing the "put the image into itself" THANG [click on button 'THANG'] crashes 4.0 (Mac PPC), on 4.5 (Mac PPC) one of the images vanishes, and on Linux all the images vanish. Joy, joy, oh, complete, unmitigated joy . . . :( >> I need to do some more testing, but it appears that when delivering apps on >> Snow Leopard that make heavy use of PNGs, one will need to do something to >> the effect of: >> >> if "mac" is in platform() and systemVersion()> 10.5 then ... >> >> Also used the following method to force refresh the display of the image: >> >> set text of img 1 to text of img 1 >> >> Hope this saves some head banging for other folks using images. >> >> Regards, >> >> Scott Rossi >> Creative Director >> Tactile Media, UX Design > Just now, I've run into this exact same problem. I tried applying Scott's workaround, only to discover that it only works with Revolution 4.0 and earlier. Any time I try to display these same PNGs in LiveCode 4.5 or higher they come out significantly darker, regardless of whether I reset the screenGamma. Oddly, PNGs that were originally produced in OS X 10.5 do not come out darker, so it definitely has something to do with 10.6. Only the latest versions of LC don't allow me to correct the problem any more. > > Has anyone run into this, and has anyone found a solution? > > Regards, > > Devin > > > > Devin Asay > Humanities Technology and Research Support Center > Brigham Young University > > _______________________________________________ > use-livecode mailing list > use-livecode at lists.runrev.com > Please visit this url to subscribe, unsubscribe and manage your subscription preferences: > http://lists.runrev.com/mailman/listinfo/use-livecode From davidocoker at gmail.com Sat Jul 2 09:17:36 2011 From: davidocoker at gmail.com (David C.) Date: Sat, 2 Jul 2011 08:17:36 -0500 Subject: [OT] Text analysis and author, anyone done it? In-Reply-To: References: <21FFE6C9-9A54-43F7-94CB-73E7A21D6FAA@tamu.edu> Message-ID: On Thu, Jun 30, 2011 at 5:42 PM, Bob Sneidar wrote: > Since the subject was broached using textual analysis of Biblical passages as an example, I will respond in like kind. If anyone will be offended at this, please, stop reading, close the email, and ignore any future posts to this thread by me. I've given fair warning. Please no flames. > > This linked article is a perfect case in point for why software based textual analysis does NOT work well for the purposes that some may intend. Part of the problem is that it relies upon a number of assumptions, human assumptions no less (for all software is really a reflection of the developer), not the least of which is that a single author can have only one style of writing. Paul for example, was one of the most learned Hebrew scholars of his time, as well but was raised in the Grecian society and well schooled in their ways and traditions. When writing to the Gentiles, you will find that his style was distinctly different from the book of Hebrews, where he wrote to his fellow countrymen about what Christianity means to the Jew, to the Torah (the books of the law) and to those Hebrews who had embraced Christianity. The differences have cause no end of disagreement among the learned about who really wrote the book of Hebrews. Some say it was Peter or John, but then the same difficulty arises when comparing Hebrews to other Biblical writings of those authors. > > In response to the specific example in the article about the book of Isaiah, Jesus quoted from the two sections of Isaiah, commonly believed by "progressive" critics to be written by two different "Isaiahs", saying in the second quote, "That same Isaiah...". There are some who believe there were 5 or 6 Isaiahs, although this is considered "fringe" even by the majority of the progressive critics. > > Even in the mainstream, many critics believe that Isaiah was written AFTER the time of Christ, mainly due to the very specific prophecies about the Christ which are too many to name here. (That is a pretty nice trick, seeing that the Dead Sea Scrolls contain parts that the scholars say came after Christ, but no archaeologist in his right mind would contend the Dead Sea Scrolls are post Christ!) > > Now I don't think it takes a genius to see that if Jesus was wrong about anything, then his whole claim to be the Son of God lies in ruins, and we can only pity him as a self deluded fool, or worse yet, a vile deceiver. But if His claims are true, then it is the critics who are to be pitied. Because they are simply and completely wrong. > > I can tell you that when I am typing an email of this nature, my grammar and tone morphs into something much more formal and precise. When I email my friends about a funny joke, my writing style is very much different. Software analysis of all I have ever written from my first essay in grade school to now would probably conclude there are 20 of me. > > All that to say this: Software may be able to turn up some interesting facets of writing styles, and we may be able to learn something from it, but it is not the software that fails us here. It is humans who put way too much value in the apparent results than are warranted. I guess people cannot help but grab ahold of what they can, so they can believe what they want to believe, on either side of the issue. Software cannot fix that. > > Bob A very insightful reply... thank you Bob! Best regards, David C. From barryb at libero.it Sat Jul 2 09:18:45 2011 From: barryb at libero.it (barryb at libero.it) Date: Sat, 2 Jul 2011 15:18:45 +0200 (CEST) Subject: Why does lockScreen not work here? Message-ID: <24695931.6051621309612725320.JavaMail.defaultUser@defaultHost> This is a similar question to that of Tiemo (why does lock screen doesn't locks the screen? (Tiemo Hollmann TB)) about finding colours under other objects (controls) but that thread evolved so fast in a single Digest I may appear a bit late. Thus the hijacked thread title. The following handler detects the colour onto which a dragged object is dropped by moving it and getting the mouseColor but. Although it could be interpreted as a settling wobble, I would prefer not to see the little movement . This works correctly, as is, but if I uncomment the lines 'lockScreen' and 'unlockScreen' then the whole handler is stopped. I've tried moving the commented lines around and using 'set lock screen' but the result is always the same, suggestions please. ----------------------- Global gCurrBall on Rollit --lockScreen move gCurrBall relative 20,-20 --These movements should not be seen by user- get the mouseColor put it into tColour move gCurrBall relative -20,20 --unlockScreen if tColour is 0,0,0 then Dropit else exit to top end Rollit on Dropit.......... --------------------------- Using Windows XPHome, Rev vs 3.5 / 4.0 Best to all Barry Barber From m.schonewille at economy-x-talk.com Sat Jul 2 09:32:47 2011 From: m.schonewille at economy-x-talk.com (Mark Schonewille) Date: Sat, 2 Jul 2011 15:32:47 +0200 Subject: Why does lockScreen not work here? In-Reply-To: <24695931.6051621309612725320.JavaMail.defaultUser@defaultHost> References: <24695931.6051621309612725320.JavaMail.defaultUser@defaultHost> Message-ID: Barry, You made a mistake. The correct syntax is: lock screen or set the lockScreen to true and unlock screen or set the lockScreen to false -- Best regards, Mark Schonewille Economy-x-Talk Consulting and Software Engineering Homepage: http://economy-x-talk.com Twitter: http://twitter.com/xtalkprogrammer KvK: 50277553 New: Download the Installer Maker Plugin 1.6 for LiveCode here http://qery.us/ce On 2 jul 2011, at 15:18, barryb at libero.it wrote: > > > This is a similar question to that of Tiemo (why does lock screen doesn't locks the screen? (Tiemo Hollmann TB)) about finding colours under other objects (controls) but that thread evolved so fast in a single Digest I may appear a bit late. Thus the hijacked thread title. > > The following handler detects the colour onto which a dragged object is dropped by moving it and getting the mouseColor but. Although it could be interpreted as a settling wobble, I would prefer not to see the little movement . > > This works correctly, as is, but if I uncomment the lines 'lockScreen' and 'unlockScreen' then the whole handler is stopped. > > I've tried moving the commented lines around and using 'set lock screen' but the result is always the same, suggestions please. > > ----------------------- > Global gCurrBall > > on Rollit > --lockScreen > move gCurrBall relative 20,-20 --These movements should not be seen by user- > get the mouseColor > put it into tColour > move gCurrBall relative -20,20 > --unlockScreen > if tColour is 0,0,0 then > Dropit > else exit to top > end Rollit > > on Dropit.......... > --------------------------- > Using Windows XPHome, Rev vs 3.5 / 4.0 > > Best to all > Barry Barber From richmondmathewson at gmail.com Sat Jul 2 09:53:23 2011 From: richmondmathewson at gmail.com (Richmond Mathewson) Date: Sat, 02 Jul 2011 16:53:23 +0300 Subject: [OT] Text analysis and author, anyone done it? In-Reply-To: References: <21FFE6C9-9A54-43F7-94CB-73E7A21D6FAA@tamu.edu> Message-ID: <4E0F22D3.7040000@gmail.com> On 07/02/2011 04:17 PM, David C. wrote: > On Thu, Jun 30, 2011 at 5:42 PM, Bob Sneidar wrote: >> Since the subject was broached using textual analysis of Biblical passages as an example, I will respond in like kind. If anyone will be offended at this, please, stop reading, close the email, and ignore any future posts to this thread by me. I've given fair warning. Please no flames. >> >> This linked article is a perfect case in point for why software based textual analysis does NOT work well for the purposes that some may intend. Part of the problem is that it relies upon a number of assumptions, human assumptions no less (for all software is really a reflection of the developer), not the least of which is that a single author can have only one style of writing. Paul for example, was one of the most learned Hebrew scholars of his time, as well but was raised in the Grecian society and well schooled in their ways and traditions. When writing to the Gentiles, you will find that his style was distinctly different from the book of Hebrews, where he wrote to his fellow countrymen about what Christianity means to the Jew, to the Torah (the books of the law) and to those Hebrews who had embraced Christianity. The differences have cause no end of disagreement among the learned about who really wrote the book of Hebrews. Some say it was Peter or John, but then the same difficulty arises when comparing Hebrews to other Biblical writings of those authors. >> >> In response to the specific example in the article about the book of Isaiah, Jesus quoted from the two sections of Isaiah, commonly believed by "progressive" critics to be written by two different "Isaiahs", saying in the second quote, "That same Isaiah...". There are some who believe there were 5 or 6 Isaiahs, although this is considered "fringe" even by the majority of the progressive critics. >> >> Even in the mainstream, many critics believe that Isaiah was written AFTER the time of Christ, mainly due to the very specific prophecies about the Christ which are too many to name here. (That is a pretty nice trick, seeing that the Dead Sea Scrolls contain parts that the scholars say came after Christ, but no archaeologist in his right mind would contend the Dead Sea Scrolls are post Christ!) >> >> Now I don't think it takes a genius to see that if Jesus was wrong about anything, then his whole claim to be the Son of God lies in ruins, and we can only pity him as a self deluded fool, or worse yet, a vile deceiver. But if His claims are true, then it is the critics who are to be pitied. Because they are simply and completely wrong. >> >> I can tell you that when I am typing an email of this nature, my grammar and tone morphs into something much more formal and precise. When I email my friends about a funny joke, my writing style is very much different. Software analysis of all I have ever written from my first essay in grade school to now would probably conclude there are 20 of me. >> >> All that to say this: Software may be able to turn up some interesting facets of writing styles, and we may be able to learn something from it, but it is not the software that fails us here. It is humans who put way too much value in the apparent results than are warranted. I guess people cannot help but grab ahold of what they can, so they can believe what they want to believe, on either side of the issue. Software cannot fix that. >> >> Bob > A very insightful reply... thank you Bob! > > Best regards, > David C. > > _______________________________________________ Yes; very well put. Richmond. From pmbrig at gmail.com Sat Jul 2 09:56:11 2011 From: pmbrig at gmail.com (Peter Brigham MD) Date: Sat, 2 Jul 2011 09:56:11 -0400 Subject: [OT] what RGB is blue? In-Reply-To: <46EA8E9B-CA91-49AC-A10D-330F10C4F9A1@twft.com> References: <001401cc37d2$d3257910$79706b30$@de> <46EA8E9B-CA91-49AC-A10D-330F10C4F9A1@twft.com> Message-ID: <2FDC8C44-2A4A-495C-9E80-1A076ECC0F22@gmail.com> I posted a stack to RevOnline demonstrating one of the illusions from the boingboing page. See "optical illusion" -- Peter Peter M. Brigham pmbrig at gmail.com http://home.comcast.net/~pmbrig On Jul 1, 2011, at 4:24 PM, Bob Sneidar wrote: > Well that is the point I was making, but when I see how badly my mind has to misinterpret color in order to make sense of an image, I begin to doubt whether anything I see is valid or not. > > Bob > > > On Jul 1, 2011, at 11:27 AM, Nonsanity wrote: > >> Go with the math. You can't trust your eyes. >> >> http://boingboing.net/2008/02/08/color-tile-optical-i.html > > > _______________________________________________ > use-livecode mailing list > use-livecode at lists.runrev.com > Please visit this url to subscribe, unsubscribe and manage your subscription preferences: > http://lists.runrev.com/mailman/listinfo/use-livecode From barryb at libero.it Sat Jul 2 09:59:11 2011 From: barryb at libero.it (barryb at libero.it) Date: Sat, 2 Jul 2011 15:59:11 +0200 (CEST) Subject: Why does lockScreen not work here? Message-ID: <11465002.6059511309615151220.JavaMail.defaultUser@defaultHost> Thank you Mark!Now you all know why I don't ask questions very often.But I do enjoy reading all my Digests every evening (can't understand how you people keep it up in real time) and that takes so long I never get around to working on my stacks (or reading the fm)! Barry From richmondmathewson at gmail.com Sat Jul 2 10:16:57 2011 From: richmondmathewson at gmail.com (Richmond Mathewson) Date: Sat, 02 Jul 2011 17:16:57 +0300 Subject: [OT] Text analysis and author, anyone done it? In-Reply-To: References: <21FFE6C9-9A54-43F7-94CB-73E7A21D6FAA@tamu.edu> Message-ID: <4E0F2859.6060602@gmail.com> Analysis of texts by machines is fairly suspect because it rests on a bogus premise: Humans are like machines. They are not, and machines will never be like humans. The mechanistic view of the human brain that has developed over the last 100 or so years has served to block our understanding of important aspects of what "being human" means. There are 2 main views of how humans came about: 1. Some sort of evolution from less complicated life forms. 2. Some sort of creation from life-less matter. Whichever way you cut it, the Human=Computer just doesn't work: 1. Computers have not evolved in the way that the term 'evolution' is understood in Darwinian theory and its developments. Computers have not randomly developed features that has meant that some of them have been picked off by predators, and some have survivded for the next round. 2. Computers of the sort we have now, were not created "crash-bang-wallop" a week ago. Computers are examples of intelligent design (pace Paley), conscious modification and improvement. ------------------------------------------------------------------------------------------------------------- What computer analysis of texts CAN do is present us with patterns: "Fambles goes into the bar, where he sees Sonny with his black eye and those torn jeans; so they sit drinking together and having a crack about old times on the state farm." might, at a pinch, show some pattern similar to Damon Runyon's writings [i.e. persistent use of the historical present]. What it won't show is who wrote it. Nor will analysis of texts written about 2000 years ago in Koine tell us very much, as we have NO text we know 100% was written by the chap [Paul] whose writings we are interested in. I am presently having a look at some Sanskrit texts supposedly written about 1500 years ago, claimed to have been written 4000 years ago, and there is also a big argument about where they were written. The tradition in which they were written in would have us believe that they have been handed down from God, so are written 'transparently' (i.e. unaffected by any personal foibles of the scribe). The only sensible conclusion I can come to is "this is a can of worms"; and worms that will always be worms, or, at best, matters of belief and prejudice. It is a very Western obsession to worry about who wrote religious documents, and as soon as one starts to apply rational thought processes to religious texts one ends up badly mired indeed. The biggest "weapon" of the rationalist is computer analysis; on matters of faith it matters not one wit, and the big mistake is thinking it does. --------------------------------------------------------------- So; the writings in the New Testament are "what they are", and non-machine driven textual and toponymic analysis have got a long way to unravelling various things in a way computers will not manage. Religious people will worry much more, about the spirit in which the writings of "St. Paul" were written, and the message(s) they contain; and that is probably a better bet than feeding them into machines for number-crunching. Well, that's a quite sufficent rant; I'm off to have an hour with the Gospel of Thomas: always calms me down. From williamdesmet at gmail.com Sat Jul 2 11:10:41 2011 From: williamdesmet at gmail.com (William de Smet) Date: Sat, 2 Jul 2011 17:10:41 +0200 Subject: FlushEvents on iOS? Message-ID: Hi there, I am 'converting' one of my apps to iOS (iPad) and I am looking for an alternative for: put flushEvents("all") into temp. Is there something like this on iOS? And another strange thing is that I don't have sound on the iPad simulator 4.3 It does work fine when I use the iPhone simulator 4.3. Anyone else having the same thing? greetings, William From pete at mollysrevenge.com Sat Jul 2 12:16:36 2011 From: pete at mollysrevenge.com (Pete) Date: Sat, 2 Jul 2011 09:16:36 -0700 Subject: System Date/Time Format Problems In-Reply-To: <7C561176-82BD-45A5-9E68-A470C13DB8D4@gmail.com> References: <7C561176-82BD-45A5-9E68-A470C13DB8D4@gmail.com> Message-ID: Thanks Peter. This and the strftime shell command will save me a lot of time in formatting dates in LC! But Im still faced with the problem of figuring out what the user's System Preference settings are for the short, medium, long, and full date and time formats. I want to give them a preference that will allow them to set one of those options for how they want dates/times displayed in my app, or have a custom option that allows them to specify another format using the date/strftime format strings conventions. Incidentally, I added a problem report to the QC regarding the issues with the various forms of the LC system date compared with the OS X system date formats. Pete Molly's Revenge On Sat, Jul 2, 2011 at 5:43 AM, Peter Brigham MD wrote: > There may be a better way, but you might have to use a shell call. Execute > the following in the message box: > > put shell("man date | col -b") > > and look at the options. You can get any format you want, and this way you > would have full control over date display format in LC. > > -- Peter > > Peter M. Brigham > pmbrig at gmail.com > http://home.comcast.net/~pmbrig > > On Jul 1, 2011, at 1:59 PM, Pete wrote: > > > I'm running into some major differences in the way LC displays system > date > > and time versus what I have set up in my OSX control panel and wondering > if > > anyone else has run into this. This is in the context of providing my > users > > with a preference setting for how they want dates/times displayed. > > > > First problem is that OSX has 4 date and time formats (Short, Medium, > Long, > > and Full) whereas LC only has three (Short, Abbrev, Long) so there are > some > > inevitable discrepancies. > > > > For dates: > > LC Short = OSX Short > > LC Long = OSX Full > > LC Abbrev = no OSX format > > no LC format = OSX medium or long > > > > > > For times: > > LC Short = OSX Short > > LC Abbrev = OSX Short > > LC Long = OSX Medium > > no LC format = OSX Long or full > > > > Any thoughts? > > > > Pete > > Molly's Revenge > > _______________________________________________ > > use-livecode mailing list > > use-livecode at lists.runrev.com > > Please visit this url to subscribe, unsubscribe and manage your > subscription preferences: > > http://lists.runrev.com/mailman/listinfo/use-livecode > > > _______________________________________________ > use-livecode mailing list > use-livecode at lists.runrev.com > Please visit this url to subscribe, unsubscribe and manage your > subscription preferences: > http://lists.runrev.com/mailman/listinfo/use-livecode > > From jacque at hyperactivesw.com Sat Jul 2 12:37:03 2011 From: jacque at hyperactivesw.com (J. Landman Gay) Date: Sat, 02 Jul 2011 11:37:03 -0500 Subject: System Date/Time Format Problems In-Reply-To: References: <7C561176-82BD-45A5-9E68-A470C13DB8D4@gmail.com> Message-ID: <4E0F492F.5040701@hyperactivesw.com> On 7/2/11 11:16 AM, Pete wrote: > But Im still faced with the problem of figuring out what the user's System > Preference settings are for the short, medium, long, and full date and time > formats. You can get their preferred format by either setting the usesystemdate to true and then getting a date or time, or just by getting "the system date". Then all the LiveCode date formats will reflect the user preference. For the abbrev date, where the month is not fully expanded, a quick lookup in the monthnames would fix that. Just replace the abbreviated month name with the expanded one. -- Jacqueline Landman Gay | jacque at hyperactivesw.com HyperActive Software | http://www.hyperactivesw.com From jacque at hyperactivesw.com Sat Jul 2 12:42:45 2011 From: jacque at hyperactivesw.com (J. Landman Gay) Date: Sat, 02 Jul 2011 11:42:45 -0500 Subject: Upper/Lower case issue In-Reply-To: References: <64050804-7291-4338-A5FF-DA56C4CE734C@economy-x-talk.com> <15121444796.20110701175640@ahsoftware.net> Message-ID: <4E0F4A85.6030504@hyperactivesw.com> On 7/2/11 12:27 AM, Pete wrote: > But they're not the same name - one is upper case and one is lower case. LC > made a decision to treat them the same since casesensitive doesn't apply to > object names, but they are different by any other definition. The entire engine is case insensitive, not just object names. That's why we have a way to force case sensitivity for strings. > I'm sure there are other ways to do this - with custom properties for > example. But I can't think of any practical downside to using the button > names, although I'm ready to hear any. I'm of the school that also avoids duplicate object names. In this case I'd probably name the buttons with a space and a number after the duplicate part of the name, and then when processing them, refer to "word 1 of the short name of btn x" to get the necessary info. -- Jacqueline Landman Gay | jacque at hyperactivesw.com HyperActive Software | http://www.hyperactivesw.com From pete at mollysrevenge.com Sat Jul 2 12:43:17 2011 From: pete at mollysrevenge.com (Pete) Date: Sat, 2 Jul 2011 09:43:17 -0700 Subject: Upper/Lower case issue In-Reply-To: <8CE06917330DD6C-1D78-1FBF3@webmail-m070.sysops.aol.com> References: <64050804-7291-4338-A5FF-DA56C4CE734C@economy-x-talk.com> <15121444796.20110701175640@ahsoftware.net> <8CE06917330DD6C-1D78-1FBF3@webmail-m070.sysops.aol.com> Message-ID: Hi Craig, Just to be clear, the button names are not identical - one is upper case and one is lower case. In its wisdom, LC treats them as identical because they are object names but if those string were not object names, they would not be treated as identical if casesensitive was true. Here's what I'm trying to do. The names of the buttons are strings that define elements of date formats. For example, %m means zero suppressed month number and %M means 2-digit month numbers with leading zero, %d means zero suppressed day number, %D means 2 digit day number, etc. This is a very common convention for defining date strings. Since those are pretty cryptic strings, I use the button labels to show a more meaningful description to the user and then store the names of the buttons (%m, %D, etc) in a date format string in my preferences file. That way, I can pass the date definition string directly to my date formatting handler. Next time the preferences card is opened, I need to get the settings from the prefs file and set the appropriate buttons, and that's where the problem arose. Here's the code I used to get round the problem. * * *private command setButtonHilite pgroup, pbutton* * set the casesensitive to true repeat with count=1 to the number of buttons in group pgroup put the short name of button count of group pgroup into myName if myName is pbutton then set the hilitedButton of group pgroup to count exit repeat end if end repeat end setButtonHilite There are only 2-4 buttons in each group I need to process this way so there's no performance hit. I suppose I could have used a custom property to do this but I'd already gone down the path of using the button name before I discovered the casesensitive issue so opted to stick with that approach. Plus I would have needed a similar command to locate the button with the correct custom property. Hope that explains the situation * Pete Molly's Revenge On Fri, Jul 1, 2011 at 10:38 PM, wrote: > Pete. > > > In the old HC days, before there was a name AND a label, I might have > understood that you might have a valid reason to use identical names. You > are aware that these two properties exist, and still have your reasons. Can > you say briefly what they are? > > I know you said they were too complicated, but just a one or two liner? > > > Craig Newman > > > > > -----Original Message----- > From: Mark Wieder > To: How to use LiveCode > Sent: Fri, Jul 1, 2011 8:56 pm > Subject: Re: Upper/Lower case issue > > > Pete- > > Friday, July 1, 2011, 4:41:16 PM, you wrote: > > > That's OK Mark, I agree with what you said 99.99999% of the time, it's > just > > that there is a valid reason for what I'm doing in this particular > instance. > > It seems like you found a way out of your predicament, but I'm still > with Mark on this. If you think you need to give the same name to two > buttons there's another way of getting the result you're looking for. > > -- > -Mark Wieder > mwieder at ahsoftware.net > > > _______________________________________________ > use-livecode mailing list > use-livecode at lists.runrev.com > Please visit this url to subscribe, unsubscribe and manage your > subscription > preferences: > http://lists.runrev.com/mailman/listinfo/use-livecode > > > _______________________________________________ > use-livecode mailing list > use-livecode at lists.runrev.com > Please visit this url to subscribe, unsubscribe and manage your > subscription preferences: > http://lists.runrev.com/mailman/listinfo/use-livecode > > From m.schonewille at economy-x-talk.com Sat Jul 2 12:54:12 2011 From: m.schonewille at economy-x-talk.com (Mark Schonewille) Date: Sat, 2 Jul 2011 18:54:12 +0200 Subject: Upper/Lower case issue In-Reply-To: References: <64050804-7291-4338-A5FF-DA56C4CE734C@economy-x-talk.com> <15121444796.20110701175640@ahsoftware.net> <8CE06917330DD6C-1D78-1FBF3@webmail-m070.sysops.aol.com> Message-ID: Pete, That's why we have custom properties. -- Best regards, Mark Schonewille Economy-x-Talk Consulting and Software Engineering Homepage: http://economy-x-talk.com Twitter: http://twitter.com/xtalkprogrammer KvK: 50277553 New: Download the Installer Maker Plugin 1.6 for LiveCode here http://qery.us/ce On 2 jul 2011, at 18:43, Pete wrote: > Since those are pretty cryptic strings, I use the button labels to show a > more meaningful description to the user and then store the names of the > buttons (%m, %D, etc) in a date format string in my preferences file. That > way, I can pass the date definition string directly to my date formatting > handler. Next time the preferences card is opened, I need to get the > settings from the prefs file and set the appropriate buttons, and that's > where the problem arose. From mwieder at ahsoftware.net Sat Jul 2 12:57:15 2011 From: mwieder at ahsoftware.net (Mark Wieder) Date: Sat, 2 Jul 2011 09:57:15 -0700 Subject: Upper/Lower case issue In-Reply-To: References: <64050804-7291-4338-A5FF-DA56C4CE734C@economy-x-talk.com> <15121444796.20110701175640@ahsoftware.net> <8CE06917330DD6C-1D78-1FBF3@webmail-m070.sysops.aol.com> Message-ID: <454598359.20110702095715@ahsoftware.net> Pete- Instead of storing the button name in the preferences file, why not just save the button number? private command setButtonHilite pgroup, pbuttonNumber set the hilitedButton of group pgroup to pbuttonNumber end setButtonHilite -- -Mark Wieder mwieder at ahsoftware.net From m.schonewille at economy-x-talk.com Sat Jul 2 13:01:01 2011 From: m.schonewille at economy-x-talk.com (Mark Schonewille) Date: Sat, 2 Jul 2011 19:01:01 +0200 Subject: LiveCode.tv Event #30 In-Reply-To: References: Message-ID: Hi, The LiveCode.tv event is starting in an hour. We have two presentations for you, lots of expertise, a complimentary copy of InterfaceDesigner, and a chance to win an ebook! Go to http://livecode.tv for more info. -- Best regards, Mark Schonewille Economy-x-Talk Consulting and Software Engineering Homepage: http://economy-x-talk.com Twitter: http://twitter.com/xtalkprogrammer KvK: 50277553 New: Download the Installer Maker Plugin 1.6 for LiveCode here http://qery.us/ce From jacque at hyperactivesw.com Sat Jul 2 13:05:01 2011 From: jacque at hyperactivesw.com (J. Landman Gay) Date: Sat, 02 Jul 2011 12:05:01 -0500 Subject: Upper/Lower case issue In-Reply-To: References: <64050804-7291-4338-A5FF-DA56C4CE734C@economy-x-talk.com> <15121444796.20110701175640@ahsoftware.net> <8CE06917330DD6C-1D78-1FBF3@webmail-m070.sysops.aol.com> Message-ID: <4E0F4FBD.3080700@hyperactivesw.com> On 7/2/11 11:43 AM, Pete wrote: > > Here's the code I used to get round the problem. You're doing way too much work. Name your buttons: %M 1 %m 2 %d 1 %D 2 Store the full button name in your prefs so that you'll have the correct name of the button to hilite when you need to set them. You won't need to process anything, just use "repeat for each line L in tPrefs" and set the hilite of each button. There will be no ambiguity. The only time you need to do any parsing at all is when you actually want to format a date for display. In that case: put word 1 of the short name of btn x into tFormat Then work with that. -- Jacqueline Landman Gay | jacque at hyperactivesw.com HyperActive Software | http://www.hyperactivesw.com From pete at mollysrevenge.com Sat Jul 2 15:26:30 2011 From: pete at mollysrevenge.com (Pete) Date: Sat, 2 Jul 2011 12:26:30 -0700 Subject: Upper/Lower case issue In-Reply-To: <4E0F4A85.6030504@hyperactivesw.com> References: <64050804-7291-4338-A5FF-DA56C4CE734C@economy-x-talk.com> <15121444796.20110701175640@ahsoftware.net> <4E0F4A85.6030504@hyperactivesw.com> Message-ID: Thanks for all the good suggestions, they all have merit but I don't feel any of them are any better or worse than what I'm already doing. I understand and agree with the need to avoid duplicate object names but, once again, these are not duplicate names in any generally accepted sense of the word no matter what LC thinks of them - their case makes them unique - so I don't feel like I'm breaking any golden programming rules. Pete Molly's Revenge On Sat, Jul 2, 2011 at 9:42 AM, J. Landman Gay wrote: > On 7/2/11 12:27 AM, Pete wrote: > >> But they're not the same name - one is upper case and one is lower case. >> LC >> made a decision to treat them the same since casesensitive doesn't apply >> to >> object names, but they are different by any other definition. >> > > The entire engine is case insensitive, not just object names. That's why we > have a way to force case sensitivity for strings. > > > I'm sure there are other ways to do this - with custom properties for >> example. But I can't think of any practical downside to using the button >> names, although I'm ready to hear any. >> > > I'm of the school that also avoids duplicate object names. In this case I'd > probably name the buttons with a space and a number after the duplicate part > of the name, and then when processing them, refer to "word 1 of the short > name of btn x" to get the necessary info. > > -- > Jacqueline Landman Gay | jacque at hyperactivesw.com > HyperActive Software | http://www.hyperactivesw.com > > > ______________________________**_________________ > use-livecode mailing list > use-livecode at lists.runrev.com > Please visit this url to subscribe, unsubscribe and manage your > subscription preferences: > http://lists.runrev.com/**mailman/listinfo/use-livecode > > From scott at elementarysoftware.com Sat Jul 2 15:27:38 2011 From: scott at elementarysoftware.com (Scott Morrow) Date: Sat, 2 Jul 2011 12:27:38 -0700 Subject: FlushEvents on iOS? In-Reply-To: References: Message-ID: <8CC31A26-C042-4DE7-9704-CCC1B011CACE@elementarysoftware.com> Hello William, Perhaps < iphoneClearTouches > Scott Morrow Elementary Software (Now with 20% less chalk dust!) web http://elementarysoftware.com/ email scott at elementarysoftware.com ------------------------------------------------------- from iOS Release Notes: Clearing pending interactions As interaction events (touch and mouse messages) are queued, it is possible for such messages to accumulate when they aren't needed. In particular, when executing 'waits', 'moves' or during card transitions. To handle this case, the iphoneClearTouches command has been added. At the point of calling, this will collect all pending touch interactions and remove them from the event queue. Note that this also cancels any existing mouse or touch sequences, meaning that you (nor the engine) will not receive a mouseUp, mouseRelease, touchEnd or touchCancel message for any current interactions. A good example of when this command might be useful is when playing an instructional sound: on tellUserInstructions play specialFolderPath("engine") & slash & "Instruction_1.mp3" wait until the sound is "done" iphoneClearTouches end tellUserInstructions Here, if the iphoneClearTouches call was not made, any tough events the user created while the sound was playing would be queued and then be delivered immediately afterwards potentially causing unwanted effects. On Jul 2, 2011, at 8:10 AM, William de Smet wrote: > Hi there, > > I am 'converting' one of my apps to iOS (iPad) and I am looking for an > alternative for: put flushEvents("all") into temp. > Is there something like this on iOS? > > And another strange thing is that I don't have sound on the iPad simulator 4.3 > It does work fine when I use the iPhone simulator 4.3. > Anyone else having the same thing? > > greetings, > > William > > _______________________________________________ > use-livecode mailing list > use-livecode at lists.runrev.com > Please visit this url to subscribe, unsubscribe and manage your subscription preferences: > http://lists.runrev.com/mailman/listinfo/use-livecode From pete at mollysrevenge.com Sat Jul 2 15:46:46 2011 From: pete at mollysrevenge.com (Pete) Date: Sat, 2 Jul 2011 12:46:46 -0700 Subject: System Date/Time Format Problems In-Reply-To: <4E0F492F.5040701@hyperactivesw.com> References: <7C561176-82BD-45A5-9E68-A470C13DB8D4@gmail.com> <4E0F492F.5040701@hyperactivesw.com> Message-ID: Thanks Jacque. You may have missed the original post in this thread, so here's a quick recap. I want the user to be able to set a preference that he/she wants his dates/times/displayed in any of the OS X formats of short, medium, long, and full that are set in the OS X System Prefernces. The problem is that the Livecode system date formats don't fully reflect the system preference settings on OS X (don't know about Windows). Some of that is because OS X has four date/time formats and LC only has three, but there's at least one instance where LC returns a date that is not in any of the OS X date formats. Here's a summary of what happens on my Mac running OS X 10.6.8: For dates: LC Short = OSX Short LC Long = OSX Full LC Abbrev = returns a date that is not in any of the OS X formats no LC format for OSX medium or long For times: LC Short = OSX Short LC Abbrev = OSX Short LC Long = OSX Medium no LC format for OSX Long or full So if a user tells me he wants his dates in OS X medium or long format, I can't find out what that is with LC code, same for an OS X long time. I'm hoping I will be able get hold of all the date/time formats with a shell command or with Applescript but no luck yet. I may have to ask the user to define the formats again with my ap ( that goes back to the other thread about duplicate object names!) Pete Molly's Revenge On Sat, Jul 2, 2011 at 9:37 AM, J. Landman Gay wrote: > On 7/2/11 11:16 AM, Pete wrote: > > But Im still faced with the problem of figuring out what the user's System >> Preference settings are for the short, medium, long, and full date and >> time >> formats. >> > > You can get their preferred format by either setting the usesystemdate to > true and then getting a date or time, or just by getting "the system date". > Then all the LiveCode date formats will reflect the user preference. For the > abbrev date, where the month is not fully expanded, a quick lookup in the > monthnames would fix that. Just replace the abbreviated month name with the > expanded one. > > -- > Jacqueline Landman Gay | jacque at hyperactivesw.com > HyperActive Software | http://www.hyperactivesw.com > > > ______________________________**_________________ > use-livecode mailing list > use-livecode at lists.runrev.com > Please visit this url to subscribe, unsubscribe and manage your > subscription preferences: > http://lists.runrev.com/**mailman/listinfo/use-livecode > > From dick.kriesel at mail.com Sat Jul 2 18:51:28 2011 From: dick.kriesel at mail.com (Dick Kriesel) Date: Sat, 2 Jul 2011 15:51:28 -0700 Subject: Upper/Lower case issue In-Reply-To: <4E0F4A85.6030504@hyperactivesw.com> References: <64050804-7291-4338-A5FF-DA56C4CE734C@economy-x-talk.com> <15121444796.20110701175640@ahsoftware.net> <4E0F4A85.6030504@hyperactivesw.com> Message-ID: <46B21DE4-5D3B-4EEC-8BB4-8C0846ED1BFD@mail.com> On Jul 2, 2011, at 9:42 AM, J. Landman Gay wrote: > The entire engine is case insensitive, not just object names. Hi, Jacque. I guess that's why union and intersect can produce wrong results for mixed-case strings. Is that right? -- Dick From niconiko at gmail.com Sat Jul 2 18:57:36 2011 From: niconiko at gmail.com (Nicolas Cueto) Date: Sun, 3 Jul 2011 07:57:36 +0900 Subject: command "group" fails when objectList is a string Message-ID: Hi. Here's an example of what does work: group group id 342201 and group id 342165 and group id 342129 And what doesn't: put "group id 342201 and group id 342165 and group id 342129" into tString group tString But shouldn't it work? Which leads me to a second question. My workaround is: put "group group id 342201 and group id 342165 and group id 342129" into tString" do tString The LC docs say "do" can take time. And it does. But, any reasonable guesses as to how long is safe? For now, I've experimented with: wait 0 milliseconds wait 50 milliseconds wait 500 milliseconds Some too slow. Some work at times, and other times not. But perhaps there's some unbeknownst to me LC command or property that can help get around all this mess? Thanks. -- Nicolas Cueto From dglass at graymattercomputing.com Sat Jul 2 19:52:49 2011 From: dglass at graymattercomputing.com (David Glass) Date: Sat, 02 Jul 2011 16:52:49 -0700 Subject: Combobox In-Reply-To: References: Message-ID: <4E0FAF51.5000003@graymattercomputing.com> I don't suppose this will make its appearance as a DropTool? On 06/30/2011 7:30 PM, Ken Ray wrote: > I don't usually use LC's combo boxes - I made a custom version of my own > with a field and a truncated option button and I've used that in a handful > of projects. > -- David Glass - Gray Matter Computing graymattercomputing.com Help Desk: http://www.graymattercomputing.com/helpdesk 559-303-4915 From klaus at major.on-rev.com Sat Jul 2 20:04:28 2011 From: klaus at major.on-rev.com (Klaus on-rev) Date: Sun, 3 Jul 2011 02:04:28 +0200 Subject: command "group" fails when objectList is a string In-Reply-To: References: Message-ID: Hi Nicolas, Am 03.07.2011 um 00:57 schrieb Nicolas Cueto: > Hi. > > Here's an example of what does work: > > group group id 342201 and group id 342165 and group id 342129 > > And what doesn't: > > put "group id 342201 and group id 342165 and group id 342129" into tString > group tString > > But shouldn't it work? > > Which leads me to a second question. My workaround is: > > put "group group id 342201 and group id 342165 and group id 342129" > into tString" > do tString > > The LC docs say "do" can take time. And it does. But, any reasonable > guesses as to how long is safe? For now, I've experimented with: > > wait 0 milliseconds > wait 50 milliseconds > wait 500 milliseconds > > Some too slow. Some work at times, and other times not. > > But perhaps there's some unbeknownst to me LC command or property that > can help get around all this mess? try this: ... repeat for each item i in "342201, 342129, 342165" ## or use the name or whatever set the selected of grp ID i to true end repeat group select empty ## Now you can do what you want with "last grp" ... :-) > Thanks. > > -- > Nicolas Cueto Best Klaus -- Klaus Major http://www.major-k.de klaus at major.on-rev.com From pete at mollysrevenge.com Sat Jul 2 20:15:58 2011 From: pete at mollysrevenge.com (Pete) Date: Sat, 2 Jul 2011 17:15:58 -0700 Subject: Combobox In-Reply-To: <4E0FAF51.5000003@graymattercomputing.com> References: <4E0FAF51.5000003@graymattercomputing.com> Message-ID: I hope so! Pete Molly's Revenge On Sat, Jul 2, 2011 at 4:52 PM, David Glass wrote: > I don't suppose this will make its appearance as a DropTool? > > On 06/30/2011 7:30 PM, Ken Ray wrote: > >> I don't usually use LC's combo boxes - I made a custom version of my own >> with a field and a truncated option button and I've used that in a handful >> of projects. >> >> > -- > David Glass - Gray Matter Computing > graymattercomputing.com > Help Desk: http://www.**graymattercomputing.com/**helpdesk > 559-303-4915 > > ______________________________**_________________ > use-livecode mailing list > use-livecode at lists.runrev.com > Please visit this url to subscribe, unsubscribe and manage your > subscription preferences: > http://lists.runrev.com/**mailman/listinfo/use-livecode > > From chipp at altuit.com Sat Jul 2 20:33:15 2011 From: chipp at altuit.com (Chipp Walters) Date: Sat, 2 Jul 2011 19:33:15 -0500 Subject: Upper/Lower case issue In-Reply-To: References: <64050804-7291-4338-A5FF-DA56C4CE734C@economy-x-talk.com> <15121444796.20110701175640@ahsoftware.net> <4E0F4A85.6030504@hyperactivesw.com> Message-ID: <28E6F7F4-DB3A-4D75-918D-809881A9A133@altuit.com> Pete, I'm coming a bit late to this but... You do understand that a button's name is different from it's label. You can have a button named "fred1" with it's label set to: fred And another buttoned named "fred2" with it's label set to: FRED So the end user sees the difference as only upper and lower case, whereas your program differentiates them correctly by their unique names. I wasn't sure this was clear to you. Chipp Walters CEO Shafer Walters Group, Inc On Jul 2, 2011, at 2:26 PM, Pete wrote: > Thanks for all the good suggestions, they all have merit but I don't feel > any of them are any better or worse than what I'm already doing. I > understand and agree with the need to avoid duplicate object names but, once > again, these are not duplicate names in any generally accepted sense of the > word no matter what LC thinks of them - their case makes them unique - so I > don't feel like I'm breaking any golden programming rules. > > > Pete > Molly's Revenge > > > > > On Sat, Jul 2, 2011 at 9:42 AM, J. Landman Gay wrote: > >> On 7/2/11 12:27 AM, Pete wrote: >> >>> But they're not the same name - one is upper case and one is lower case. >>> LC >>> made a decision to treat them the same since casesensitive doesn't apply >>> to >>> object names, but they are different by any other definition. >>> >> >> The entire engine is case insensitive, not just object names. That's why we >> have a way to force case sensitivity for strings. >> >> >> I'm sure there are other ways to do this - with custom properties for >>> example. But I can't think of any practical downside to using the button >>> names, although I'm ready to hear any. >>> >> >> I'm of the school that also avoids duplicate object names. In this case I'd >> probably name the buttons with a space and a number after the duplicate part >> of the name, and then when processing them, refer to "word 1 of the short >> name of btn x" to get the necessary info. >> >> -- >> Jacqueline Landman Gay | jacque at hyperactivesw.com >> HyperActive Software | http://www.hyperactivesw.com >> >> >> ______________________________**_________________ >> use-livecode mailing list >> use-livecode at lists.runrev.com >> Please visit this url to subscribe, unsubscribe and manage your >> subscription preferences: >> http://lists.runrev.com/**mailman/listinfo/use-livecode >> >> > _______________________________________________ > use-livecode mailing list > use-livecode at lists.runrev.com > Please visit this url to subscribe, unsubscribe and manage your subscription preferences: > http://lists.runrev.com/mailman/listinfo/use-livecode From pete at mollysrevenge.com Sat Jul 2 20:38:57 2011 From: pete at mollysrevenge.com (Pete) Date: Sat, 2 Jul 2011 17:38:57 -0700 Subject: Combobox In-Reply-To: <4E0FAF51.5000003@graymattercomputing.com> References: <4E0FAF51.5000003@graymattercomputing.com> Message-ID: Is it possible to attach filesto messages to this list? I have a combobox I'd like to share... Pete Molly's Revenge On Sat, Jul 2, 2011 at 4:52 PM, David Glass wrote: > I don't suppose this will make its appearance as a DropTool? > > On 06/30/2011 7:30 PM, Ken Ray wrote: > >> I don't usually use LC's combo boxes - I made a custom version of my own >> with a field and a truncated option button and I've used that in a handful >> of projects. >> >> > -- > David Glass - Gray Matter Computing > graymattercomputing.com > Help Desk: http://www.**graymattercomputing.com/**helpdesk > 559-303-4915 > > ______________________________**_________________ > use-livecode mailing list > use-livecode at lists.runrev.com > Please visit this url to subscribe, unsubscribe and manage your > subscription preferences: > http://lists.runrev.com/**mailman/listinfo/use-livecode > > From mwieder at ahsoftware.net Sat Jul 2 20:50:33 2011 From: mwieder at ahsoftware.net (Mark Wieder) Date: Sat, 2 Jul 2011 17:50:33 -0700 Subject: [OT] what RGB is blue? In-Reply-To: <2FDC8C44-2A4A-495C-9E80-1A076ECC0F22@gmail.com> References: <001401cc37d2$d3257910$79706b30$@de> <46EA8E9B-CA91-49AC-A10D-330F10C4F9A1@twft.com> <2FDC8C44-2A4A-495C-9E80-1A076ECC0F22@gmail.com> Message-ID: <14032996125.20110702175033@ahsoftware.net> Peter- Saturday, July 2, 2011, 6:56:11 AM, you wrote: > I posted a stack to RevOnline demonstrating one of the illusions > from the boingboing page. See "optical illusion" Nice. I've been meaning to do some of these ever since I followed the boingboing link. http://illusioncontest.neuralcorrelate.com/ -- -Mark Wieder mwieder at ahsoftware.net From pete at mollysrevenge.com Sat Jul 2 21:06:43 2011 From: pete at mollysrevenge.com (Pete) Date: Sat, 2 Jul 2011 18:06:43 -0700 Subject: Upper/Lower case issue In-Reply-To: <28E6F7F4-DB3A-4D75-918D-809881A9A133@altuit.com> References: <64050804-7291-4338-A5FF-DA56C4CE734C@economy-x-talk.com> <15121444796.20110701175640@ahsoftware.net> <4E0F4A85.6030504@hyperactivesw.com> <28E6F7F4-DB3A-4D75-918D-809881A9A133@altuit.com> Message-ID: Hi Chip, Thanks for chiming in and yes I know there is a label as well as a name. Further back in the thread, there's a explanation of what I'm trying to do. The whole problem arises because the casesensitive setting doesn't apply to object names so LC can't distinguish between two objects with the same name but in different cases. But I'm sure you already know that. Pete Molly's Revenge On Sat, Jul 2, 2011 at 5:33 PM, Chipp Walters wrote: > Pete, > > I'm coming a bit late to this but... > > You do understand that a button's name is different from it's label. > > You can have a button named "fred1" with it's label set to: > > fred > > And another buttoned named "fred2" with it's label set to: > > FRED > > So the end user sees the difference as only upper and lower case, whereas > your program differentiates them correctly by their unique names. > > I wasn't sure this was clear to you. > > > > Chipp Walters > CEO Shafer Walters Group, Inc > > On Jul 2, 2011, at 2:26 PM, Pete wrote: > > > Thanks for all the good suggestions, they all have merit but I don't feel > > any of them are any better or worse than what I'm already doing. I > > understand and agree with the need to avoid duplicate object names but, > once > > again, these are not duplicate names in any generally accepted sense of > the > > word no matter what LC thinks of them - their case makes them unique - so > I > > don't feel like I'm breaking any golden programming rules. > > > > > > Pete > > Molly's Revenge > > > > > > > > > > On Sat, Jul 2, 2011 at 9:42 AM, J. Landman Gay >wrote: > > > >> On 7/2/11 12:27 AM, Pete wrote: > >> > >>> But they're not the same name - one is upper case and one is lower > case. > >>> LC > >>> made a decision to treat them the same since casesensitive doesn't > apply > >>> to > >>> object names, but they are different by any other definition. > >>> > >> > >> The entire engine is case insensitive, not just object names. That's why > we > >> have a way to force case sensitivity for strings. > >> > >> > >> I'm sure there are other ways to do this - with custom properties for > >>> example. But I can't think of any practical downside to using the > button > >>> names, although I'm ready to hear any. > >>> > >> > >> I'm of the school that also avoids duplicate object names. In this case > I'd > >> probably name the buttons with a space and a number after the duplicate > part > >> of the name, and then when processing them, refer to "word 1 of the > short > >> name of btn x" to get the necessary info. > >> > >> -- > >> Jacqueline Landman Gay | jacque at hyperactivesw.com > >> HyperActive Software | http://www.hyperactivesw.com > >> > >> > >> ______________________________**_________________ > >> use-livecode mailing list > >> use-livecode at lists.runrev.com > >> Please visit this url to subscribe, unsubscribe and manage your > >> subscription preferences: > >> http://lists.runrev.com/**mailman/listinfo/use-livecode< > http://lists.runrev.com/mailman/listinfo/use-livecode> > >> > >> > > _______________________________________________ > > use-livecode mailing list > > use-livecode at lists.runrev.com > > Please visit this url to subscribe, unsubscribe and manage your > subscription preferences: > > http://lists.runrev.com/mailman/listinfo/use-livecode > > _______________________________________________ > use-livecode mailing list > use-livecode at lists.runrev.com > Please visit this url to subscribe, unsubscribe and manage your > subscription preferences: > http://lists.runrev.com/mailman/listinfo/use-livecode > > From jacque at hyperactivesw.com Sat Jul 2 22:08:41 2011 From: jacque at hyperactivesw.com (J. Landman Gay) Date: Sat, 02 Jul 2011 21:08:41 -0500 Subject: System Date/Time Format Problems In-Reply-To: References: <7C561176-82BD-45A5-9E68-A470C13DB8D4@gmail.com> <4E0F492F.5040701@hyperactivesw.com> Message-ID: <4E0FCF29.9000501@hyperactivesw.com> On 7/2/11 2:46 PM, Pete wrote: > > For dates: > LC Short = OSX Short > LC Long = OSX Full > LC Abbrev = returns a date that is not in any of the OS X formats > no LC format for OSX medium or long > > > For times: > LC Short = OSX Short > LC Abbrev = OSX Short > LC Long = OSX Medium > no LC format for OSX Long or full > > So if a user tells me he wants his dates in OS X medium or long format, I > can't find out what that is with LC code, same for an OS X long time. I'm > hoping I will be able get hold of all the date/time formats with a shell > command or with Applescript but no luck yet. I may have to ask the user to > define the formats again with my ap ( that goes back to the other thread > about duplicate object names!) All I meant was that you already have most of them, and the ones that are missing are easy to construct. LiveCode provides all the data. For the LC formats that have no OS X equivalent, just don't use those. A quick function to construct the ones you do need would be pretty easy. -- Jacqueline Landman Gay | jacque at hyperactivesw.com HyperActive Software | http://www.hyperactivesw.com From jacque at hyperactivesw.com Sat Jul 2 22:11:36 2011 From: jacque at hyperactivesw.com (J. Landman Gay) Date: Sat, 02 Jul 2011 21:11:36 -0500 Subject: Upper/Lower case issue In-Reply-To: <46B21DE4-5D3B-4EEC-8BB4-8C0846ED1BFD@mail.com> References: <64050804-7291-4338-A5FF-DA56C4CE734C@economy-x-talk.com> <15121444796.20110701175640@ahsoftware.net> <4E0F4A85.6030504@hyperactivesw.com> <46B21DE4-5D3B-4EEC-8BB4-8C0846ED1BFD@mail.com> Message-ID: <4E0FCFD8.5000100@hyperactivesw.com> On 7/2/11 5:51 PM, Dick Kriesel wrote: > On Jul 2, 2011, at 9:42 AM, J. Landman Gay wrote: > >> The entire engine is case insensitive, not just object names. > > Hi, Jacque. I guess that's why union and intersect can produce wrong > results for mixed-case strings. Is that right? I haven't tried it, but if that's what you're seeing I'd assume so. The engine is pretty consistent about not noticing case. -- Jacqueline Landman Gay | jacque at hyperactivesw.com HyperActive Software | http://www.hyperactivesw.com From chipp at chipp.com Sat Jul 2 22:15:32 2011 From: chipp at chipp.com (Chipp Walters) Date: Sat, 2 Jul 2011 21:15:32 -0500 Subject: Anyone using MobGUI? Message-ID: If you are, do you mind a few questions? 1. Does the exact same MobGUI plugin work on both Mac and PC? IOW, if you have your plugins synchronized in Dropbox, will you be able to use it? 2. Are the scripts protected? Can you dive in and see what's going on below the surface? This would be great as a learning aid. I pretty much have a rule NOT to use protected scripts in commercial products as it's difficult to support, not to mention to debug things when they start to go wrong. Besides, this community is just too small to worry about other developers ripping you off and trying to sell to others here, so why the need to protect? 3. I didn't see a tab control- is there one on the way? 4. What are the other iOS libraries out there? 5. What is currently the best version of OSX iOS which to work with? 6. Anyone know how to pinch zoom a picture? Is there already a control or library for that? That's it for now! Thanks for any help. From roger.e.eller at sealedair.com Sat Jul 2 22:31:10 2011 From: roger.e.eller at sealedair.com (Roger Eller) Date: Sat, 2 Jul 2011 22:31:10 -0400 Subject: Anyone using MobGUI? In-Reply-To: References: Message-ID: On Sat, Jul 2, 2011 at 10:15 PM, Chipp Walters wrote: > If you are, do you mind a few questions? > > 1. Does the exact same MobGUI plugin work on both Mac and PC? IOW, if you > have your plugins synchronized in Dropbox, will you be able to use it? > 2. Are the scripts protected? Can you dive in and see what's going on > below the surface? This would be great as a learning aid. I pretty much > have > a rule NOT to use protected scripts in commercial products as it's > difficult > to support, not to mention to debug things when they start to go wrong. > Besides, this community is just too small to worry about other developers > ripping you off and trying to sell to others here, so why the need to > protect? > 3. I didn't see a tab control- is there one on the way? > 4. What are the other iOS libraries out there? > 5. What is currently the best version of OSX iOS which to work with? > 6. Anyone know how to pinch zoom a picture? Is there already a control or > library for that? > > That's it for now! Thanks for any help. #3 Is that not considered a tab control in the center? http://mobgui.com/screenshot.html #7 Android theme may be in the works (fingers crossed). ?Roger From chipp at chipp.com Sat Jul 2 22:36:54 2011 From: chipp at chipp.com (Chipp Walters) Date: Sat, 2 Jul 2011 21:36:54 -0500 Subject: Anyone using MobGUI? In-Reply-To: References: Message-ID: I believe this is called a tab bar in iOS: http://www.glyfx.com/grfx/products/previews/iphone_common_preview.png On Sat, Jul 2, 2011 at 9:31 PM, Roger Eller wrote: > On Sat, Jul 2, 2011 at 10:15 PM, Chipp Walters wrote: > > > If you are, do you mind a few questions? > > > > 1. Does the exact same MobGUI plugin work on both Mac and PC? IOW, if > you > > have your plugins synchronized in Dropbox, will you be able to use it? > > 2. Are the scripts protected? Can you dive in and see what's going on > > below the surface? This would be great as a learning aid. I pretty much > > have > > a rule NOT to use protected scripts in commercial products as it's > > difficult > > to support, not to mention to debug things when they start to go wrong. > > Besides, this community is just too small to worry about other > developers > > ripping you off and trying to sell to others here, so why the need to > > protect? > > 3. I didn't see a tab control- is there one on the way? > > 4. What are the other iOS libraries out there? > > 5. What is currently the best version of OSX iOS which to work with? > > 6. Anyone know how to pinch zoom a picture? Is there already a control > or > > library for that? > > > > That's it for now! Thanks for any help. > > > #3 Is that not considered a tab control in the center? > http://mobgui.com/screenshot.html > > #7 Android theme may be in the works (fingers crossed). > > ?Roger > _______________________________________________ > use-livecode mailing list > use-livecode at lists.runrev.com > Please visit this url to subscribe, unsubscribe and manage your > subscription preferences: > http://lists.runrev.com/mailman/listinfo/use-livecode > -- Chipp Walters CEO, Shafer Walters Group, Inc. From jacque at hyperactivesw.com Sat Jul 2 23:15:46 2011 From: jacque at hyperactivesw.com (J. Landman Gay) Date: Sat, 02 Jul 2011 22:15:46 -0500 Subject: System Date/Time Format Problems In-Reply-To: References: <7C561176-82BD-45A5-9E68-A470C13DB8D4@gmail.com> <4E0F492F.5040701@hyperactivesw.com> Message-ID: <4E0FDEE2.7090507@hyperactivesw.com> On 7/2/11 2:46 PM, Pete wrote: > So if a user tells me he wants his dates in OS X medium or long format, I > can't find out what that is with LC code, same for an OS X long time. Start with these. The functions below will only work on US systems, or in countries that use US date/time formats. If you want to accomodate other countries you'll have to do more work. The "lookupZone" function is for the long and full time formats. I have exactly one entry in there for central time. What you need to do for that is make a list of all the time zones, one per line, in the same order my example uses. Store it in a custom property or somewhere. Retrieve the list for the lookup function. There may be a shell call to do that, or a list stored somewhere in OS X. I don't know. Anyway, maybe this will get you started: function formatDate pDate,pFormat -- short, medium, long, full switch pFormat case "short" convert pDate to short date put pDate into tFormattedDate break case "medium" convert pDate to abbrev date put item 2 to -1 of pDate into tFormattedDate break case "long" convert pDate to long date put item 2 to -1 of pDate into tFormattedDate break case "full" convert pDate to long date put pDate into tFormattedDate break end switch return tFormattedDate end formatDate function formatTime pTime,pFormat switch pFormat case "short" convert pTime to short time put pTime into tFormattedTime break case "medium" convert pTime to long time put pTime into tFormattedTime break case "long" case "full" convert pTime to long time put pTime into tFormattedTime convert pTime to internet date put word 6 of pTime into tTimeZone put lookupZone(tTimeZone) into tZoneInfo if pFormat = "long" then put item 2 of tZoneInfo into tTimeZone else -- full put item 3 of tZoneInfo into tTimeZone end if put space & tTimeZone after tFormattedTime break end switch return tFormattedTime end formatTime function lookupZone pHourOffset -- need a list in this format, one line per zone: put "-0500,CST,CT" into tZones return line lineoffset(pHourOffset,tZones) of tZones end lookupZone -- Jacqueline Landman Gay | jacque at hyperactivesw.com HyperActive Software | http://www.hyperactivesw.com From jimaultwins at yahoo.com Sun Jul 3 00:09:50 2011 From: jimaultwins at yahoo.com (Jim Ault) Date: Sat, 2 Jul 2011 21:09:50 -0700 Subject: Combobox -- attachments not allowed, but... In-Reply-To: References: <4E0FAF51.5000003@graymattercomputing.com> Message-ID: <37DE2D7E-C52E-47BC-A76F-268DBD3164AD@yahoo.com> On Jul 2, 2011, at 5:38 PM, Pete wrote: > Is it possible to attach filesto messages to this list? I have a > combobox > I'd like to share... > Pete Haworth You can post to the forum with attachments, and put a link to that post here. http://forums.runrev.com/ Jim Ault From kray at sonsothunder.com Sun Jul 3 00:46:56 2011 From: kray at sonsothunder.com (Ken Ray) Date: Sat, 02 Jul 2011 23:46:56 -0500 Subject: Combobox In-Reply-To: <4E0FAF51.5000003@graymattercomputing.com> Message-ID: > I don't suppose this will make its appearance as a DropTool? Yup! But it may take a bit as I have a couple of other DropTools in the works right now... :-) Ken Ray Sons of Thunder Software, Inc. Email: kray at sonsothunder.com Web Site: http://www.sonsothunder.com/ From warrenkuhl at gmail.com Sun Jul 3 02:04:39 2011 From: warrenkuhl at gmail.com (Warren Kuhl) Date: Sun, 3 Jul 2011 01:04:39 -0500 Subject: Tab Key Message-ID: I have a card that contains a text field and a scrolling field. On the card I have a handler to handle the tab key: on tabKey if the shiftKey is down then tabback else tabforward end tabKey The above is working as expected...unless I select anything in the scrolling field. Once I do this, the card doesn't seem to recognize the tabkey anymore. When the tab key is pressed, the I tab forward to the next field in the text field. The scrolling field provides alternative works to select. When I select any words, the tab key is disabled unless I click into the text field. I am not sure why this is happening. Thanks, Warren From palcibiades-first at yahoo.co.uk Sun Jul 3 04:27:06 2011 From: palcibiades-first at yahoo.co.uk (Peter Alcibiades) Date: Sun, 3 Jul 2011 01:27:06 -0700 (PDT) Subject: [OT] Text analysis and author, anyone done it? In-Reply-To: <4E0F2859.6060602@gmail.com> References: <21FFE6C9-9A54-43F7-94CB-73E7A21D6FAA@tamu.edu> <4E0F2859.6060602@gmail.com> Message-ID: <1309681626447-3641646.post@n4.nabble.com> My own purpose is not to do with religious texts, but it seems like the example is getting in the way of the subject. There's no reason why statistical analysis shouldn't work on textual analysis. Whether it does, and which method is best, and which language is best to do it in, these are all empirical questions to be settled by studies. I haven't had time to do much work on the problem yet, but it seems at first glance as if LC is a reasonable choice. Its going to be a tradeoff between speed and statistics libraries ready made, which you would get in R, and the string manipulation and rapid development that you get with LC. I'm quite skeptical about the idea that when we write for different purposes, we vary the parameters which are used for textual analysis. One of the key characteristics of a native speaker is the ability to create an infinite number of grammatically correct different expressions. This doesn't happen randomly, its rule based, and its a bit like the gait when walking, it is a quite deep feature of our use of language. Its a reasonable hypothesis that even when writing for different purposes and audiences, our writing will be generated in accordance with the same rules. But its not a matter for belief, this is an empirical question. One could take, for instance Bertrand Russell, who wrote for specialist and lay readers. There are novelists - Graham Greene wrote what he called 'entertainments' as well as his serious novels. Some UK ministers have written polemical pieces and also fiction. Academics often write for learned journals and also popularize. So it would be quite easy to test. First however, I shall try to master the basic tools. -- View this message in context: http://runtime-revolution.278305.n4.nabble.com/Re-Text-analysis-and-author-anyone-done-it-tp3636729p3641646.html Sent from the Revolution - User mailing list archive at Nabble.com. From richmondmathewson at gmail.com Sun Jul 3 05:10:07 2011 From: richmondmathewson at gmail.com (Richmond Mathewson) Date: Sun, 03 Jul 2011 12:10:07 +0300 Subject: [OT] Text analysis and author, anyone done it? In-Reply-To: <1309681626447-3641646.post@n4.nabble.com> References: <21FFE6C9-9A54-43F7-94CB-73E7A21D6FAA@tamu.edu> <4E0F2859.6060602@gmail.com> <1309681626447-3641646.post@n4.nabble.com> Message-ID: <4E1031EF.9030301@gmail.com> On 07/03/2011 11:27 AM, Peter Alcibiades wrote: > My own purpose is not to do with religious texts, but it seems like the > example is getting in the way of the subject. There's no reason why > statistical analysis shouldn't work on textual analysis. Whether it does, > and which method is best, and which language is best to do it in, these are > all empirical questions to be settled by studies. I haven't had time to do > much work on the problem yet, but it seems at first glance as if LC is a > reasonable choice. Its going to be a tradeoff between speed and statistics > libraries ready made, which you would get in R, and the string manipulation > and rapid development that you get with LC. > > I'm quite skeptical about the idea that when we write for different > purposes, we vary the parameters which are used for textual analysis. One > of the key characteristics of a native speaker is the ability to create an > infinite number of grammatically correct different expressions. This > doesn't happen randomly, its rule based, and its a bit like the gait when > walking, it is a quite deep feature of our use of language. Its a reasonable > hypothesis that even when writing for different purposes and audiences, our > writing will be generated in accordance with the same rules. Chomskyite attempts to fit language into a mathematical paradigm led me many years ago to Cognitive Linguistics, and end up writing an MA thesis which ended up with one of my supervisors getting all hot under the collar because I took quite a few ideas from Functionalism: http://groups.yahoo.com/group/richmondslinguistics/ > But its not a matter for belief, this is an empirical question. One could > take, for instance Bertrand Russell, who wrote for specialist and lay > readers. There are novelists - Graham Greene wrote what he called > 'entertainments' as well as his serious novels. Some UK ministers have > written polemical pieces and also fiction. Academics often write for learned > journals and also popularize. So it would be quite easy to test. > > First however, I shall try to master the basic tools. > > -- > View this message in context: http://runtime-revolution.278305.n4.nabble.com/Re-Text-analysis-and-author-anyone-done-it-tp3636729p3641646.html > Sent from the Revolution - User mailing list archive at Nabble.com. > > _______________________________________________ > use-livecode mailing list > use-livecode at lists.runrev.com > Please visit this url to subscribe, unsubscribe and manage your subscription preferences: > http://lists.runrev.com/mailman/listinfo/use-livecode From m.schonewille at economy-x-talk.com Sun Jul 3 06:30:15 2011 From: m.schonewille at economy-x-talk.com (Mark Schonewille) Date: Sun, 3 Jul 2011 12:30:15 +0200 Subject: Anyone using MobGUI? In-Reply-To: References: Message-ID: Hi Chipp, I gave it a try. I found a few problems and reported them. I think they have been or are being fixed. As far as I know, there is no tab bar. The library is automatically added to any stack you are working on, if that stack is in the front and you change a setting in the MobGUI palette. You can't view the scripts of the library (which I consider very understandable). -- Best regards, Mark Schonewille Economy-x-Talk Consulting and Software Engineering Homepage: http://economy-x-talk.com Twitter: http://twitter.com/xtalkprogrammer KvK: 50277553 New: Download the Installer Maker Plugin 1.6 for LiveCode here http://qery.us/ce On 3 jul 2011, at 04:36, Chipp Walters wrote: > I believe this is called a tab bar in iOS: > http://www.glyfx.com/grfx/products/previews/iphone_common_preview.png > > On Sat, Jul 2, 2011 at 9:31 PM, Roger Eller wrote: > >> On Sat, Jul 2, 2011 at 10:15 PM, Chipp Walters wrote: >> >>> If you are, do you mind a few questions? >>> >>> 1. Does the exact same MobGUI plugin work on both Mac and PC? IOW, if >> you >>> have your plugins synchronized in Dropbox, will you be able to use it? >>> 2. Are the scripts protected? Can you dive in and see what's going on >>> below the surface? This would be great as a learning aid. I pretty much >>> have >>> a rule NOT to use protected scripts in commercial products as it's >>> difficult >>> to support, not to mention to debug things when they start to go wrong. >>> Besides, this community is just too small to worry about other >> developers >>> ripping you off and trying to sell to others here, so why the need to >>> protect? >>> 3. I didn't see a tab control- is there one on the way? >>> 4. What are the other iOS libraries out there? >>> 5. What is currently the best version of OSX iOS which to work with? >>> 6. Anyone know how to pinch zoom a picture? Is there already a control >> or >>> library for that? >>> >>> That's it for now! Thanks for any help. >> >> >> #3 Is that not considered a tab control in the center? >> http://mobgui.com/screenshot.html >> >> #7 Android theme may be in the works (fingers crossed). >> >> ?Roger From john at splash21.com Sun Jul 3 08:03:25 2011 From: john at splash21.com (John Craig) Date: Sun, 03 Jul 2011 13:03:25 +0100 Subject: Anyone using MobGUI? In-Reply-To: References: Message-ID: <4E105A8D.8070901@splash21.com> Hi, Chipp. I'm (unsurprisingly!) using the plugin. I don't mean that as a joke - I actually started writing it to save me time on my own iOS projects. Other coders had a similar interest / need (feedback after a demo on LiveCode.TV) so it has now developed into it's current form. It works on both Mac and PC - I've got my eye on an Android tablet, and plan to try some android development as well. The scripts are protected - if I can generate some revenue from the plugin, then I can dedicate time to developing it further. I don't have any problems using protected scripts in projects, but I can see your point of view. Then again, if I decided that using LiveCode was too hard to support because I didn't have the source code, and started writing my own cross platform development engine, things would take quite a bit longer ;) The community is small at the moment, but I'm excited about how LiveCode is progressing and the things that are in the pipeline. I'm hoping that the small community is replaced by a huge community. There is a tab control on the way - I'm trying to build a full feature set. The plugin is still in it's early days, and by no means perfect. I've still got a couple of support tickets to sort out just for buttons - the feedback and bug reports have been excellent - really helping to shape things to come. Great presentation last night, BTW - I enjoyed it. There are a couple of techniques that I've discovered during the development of MobGUI that I've thought "hey, that could be a good topic for LiveCode.TV", so although it's not open source so far, there's nothing stopping me sharing ideas and interesting bits and pieces with others! HTH, JC. On 03/07/2011 03:15, Chipp Walters wrote: > If you are, do you mind a few questions? > > 1. Does the exact same MobGUI plugin work on both Mac and PC? IOW, if you > have your plugins synchronized in Dropbox, will you be able to use it? > 2. Are the scripts protected? Can you dive in and see what's going on > below the surface? This would be great as a learning aid. I pretty much have > a rule NOT to use protected scripts in commercial products as it's difficult > to support, not to mention to debug things when they start to go wrong. > Besides, this community is just too small to worry about other developers > ripping you off and trying to sell to others here, so why the need to > protect? > 3. I didn't see a tab control- is there one on the way? > 4. What are the other iOS libraries out there? > 5. What is currently the best version of OSX iOS which to work with? > 6. Anyone know how to pinch zoom a picture? Is there already a control or > library for that? > > That's it for now! Thanks for any help. > _______________________________________________ > use-livecode mailing list > use-livecode at lists.runrev.com > Please visit this url to subscribe, unsubscribe and manage your subscription preferences: > http://lists.runrev.com/mailman/listinfo/use-livecode > From kray at sonsothunder.com Sun Jul 3 08:52:31 2011 From: kray at sonsothunder.com (Ken Ray) Date: Sun, 03 Jul 2011 07:52:31 -0500 Subject: Tab Key In-Reply-To: Message-ID: > When the tab key is pressed, the I tab forward to the next field in > the text field. The scrolling field provides alternative works to > select. When I select any words, the tab key is disabled unless I > click into the text field. > > I am not sure why this is happening. What platform are you on; and is this a scrolling *list* field, or a scrolling (text) field? Ken Ray Sons of Thunder Software, Inc. Email: kray at sonsothunder.com Web Site: http://www.sonsothunder.com/ From alex at harryscollar.com Sun Jul 3 10:39:23 2011 From: alex at harryscollar.com (Alex Shaw) Date: Mon, 04 Jul 2011 00:39:23 +1000 Subject: record sound problem on windows In-Reply-To: <51098212-42E7-4B3F-9D8D-A33609C476DA@swissonline.ch> References: <51098212-42E7-4B3F-9D8D-A33609C476DA@swissonline.ch> Message-ID: <4E107F1B.9050904@harryscollar.com> Hi I'm having a similar problem recording sound on Windows XP with updated quicktime. Using the standard "record sound file "test.wav", works fine on OSX but the resulting file when recorded on XP is high pitched & out of tempo. Is there a special setting needed for XP? Have tried various record rates but to no avail. regards alex On 19/03/11 11:47 PM, Martin Meili wrote: > Hi, > I've got a soud recording problem on Windows XP with LiveCode 4.5.3. > > The command works fine on Mac OSX, both in the stackfile mode and the > standalone mode. > > On Windows it doesn't work neither in the stackfile mode nor the > standalone mode. I doesn't matter whether I work on my Mac or my Dell > windows computer - I've got on both platforms the same problem: > "record sound" doesn't work on Windows. > > If I open the stack with Revolution 4.0 and do the stadalone-build, I > get a working Standalone for Windows which does the "record sound" > command correctly. > > Anybody there who knows about the problem (bug??)? > > Cheers > Martin > > _______________________________________________ > use-livecode mailing list > use-livecode at lists.runrev.com > Please visit this url to subscribe, unsubscribe and manage your > subscription preferences: > http://lists.runrev.com/mailman/listinfo/use-livecode > From warrenkuhl at gmail.com Sun Jul 3 11:19:35 2011 From: warrenkuhl at gmail.com (Warren Kuhl) Date: Sun, 3 Jul 2011 10:19:35 -0500 Subject: Tab Key In-Reply-To: References: Message-ID: Ken, This is a scrolling list field. The platform is Windows. Thanks for any help! Warren On Sun, Jul 3, 2011 at 7:52 AM, Ken Ray wrote: > >> When the tab key is pressed, the I tab forward to the next field in >> the text field. ?The scrolling field provides alternative works to >> select. ?When I select any words, the tab key is disabled unless I >> click into the text field. >> >> I am not sure why this is happening. > > What platform are you on; and is this a scrolling *list* field, or a > scrolling (text) field? > > Ken Ray > Sons of Thunder Software, Inc. > Email: kray at sonsothunder.com > Web Site: http://www.sonsothunder.com/ > > > > _______________________________________________ > use-livecode mailing list > use-livecode at lists.runrev.com > Please visit this url to subscribe, unsubscribe and manage your subscription preferences: > http://lists.runrev.com/mailman/listinfo/use-livecode > From pete at mollysrevenge.com Sun Jul 3 14:14:11 2011 From: pete at mollysrevenge.com (Pete) Date: Sun, 3 Jul 2011 11:14:11 -0700 Subject: System Date/Time Format Problems In-Reply-To: <4E0FDEE2.7090507@hyperactivesw.com> References: <7C561176-82BD-45A5-9E68-A470C13DB8D4@gmail.com> <4E0F492F.5040701@hyperactivesw.com> <4E0FDEE2.7090507@hyperactivesw.com> Message-ID: Thanks Jacque, that's a great start. I also found the dateFormat function which I think may assist in all this. Pete Molly's Revenge On Sat, Jul 2, 2011 at 8:15 PM, J. Landman Gay wrote: > On 7/2/11 2:46 PM, Pete wrote: > > So if a user tells me he wants his dates in OS X medium or long format, I >> can't find out what that is with LC code, same for an OS X long time. >> > > Start with these. The functions below will only work on US systems, or in > countries that use US date/time formats. If you want to accomodate other > countries you'll have to do more work. > > The "lookupZone" function is for the long and full time formats. I have > exactly one entry in there for central time. What you need to do for that is > make a list of all the time zones, one per line, in the same order my > example uses. Store it in a custom property or somewhere. Retrieve the list > for the lookup function. > > There may be a shell call to do that, or a list stored somewhere in OS X. I > don't know. Anyway, maybe this will get you started: > > function formatDate pDate,pFormat -- short, medium, long, full > switch pFormat > case "short" > convert pDate to short date > put pDate into tFormattedDate > break > case "medium" > convert pDate to abbrev date > put item 2 to -1 of pDate into tFormattedDate > break > case "long" > convert pDate to long date > put item 2 to -1 of pDate into tFormattedDate > break > case "full" > convert pDate to long date > put pDate into tFormattedDate > break > end switch > return tFormattedDate > end formatDate > > function formatTime pTime,pFormat > switch pFormat > case "short" > convert pTime to short time > put pTime into tFormattedTime > break > case "medium" > convert pTime to long time > put pTime into tFormattedTime > break > case "long" > case "full" > convert pTime to long time > put pTime into tFormattedTime > convert pTime to internet date > put word 6 of pTime into tTimeZone > put lookupZone(tTimeZone) into tZoneInfo > if pFormat = "long" then > put item 2 of tZoneInfo into tTimeZone > else -- full > put item 3 of tZoneInfo into tTimeZone > end if > put space & tTimeZone after tFormattedTime > break > end switch > return tFormattedTime > end formatTime > > function lookupZone pHourOffset > -- need a list in this format, one line per zone: > put "-0500,CST,CT" into tZones > return line lineoffset(pHourOffset,tZones) of tZones > end lookupZone > > > -- > Jacqueline Landman Gay | jacque at hyperactivesw.com > HyperActive Software | http://www.hyperactivesw.com > > ______________________________**_________________ > use-livecode mailing list > use-livecode at lists.runrev.com > Please visit this url to subscribe, unsubscribe and manage your > subscription preferences: > http://lists.runrev.com/**mailman/listinfo/use-livecode > > From pete at mollysrevenge.com Sun Jul 3 15:39:45 2011 From: pete at mollysrevenge.com (Pete) Date: Sun, 3 Jul 2011 12:39:45 -0700 Subject: Combobox In-Reply-To: References: <4E0FAF51.5000003@graymattercomputing.com> Message-ID: If anyone would like to try my combobox, it's attached to the forum message at http://forums.runrev.com/viewtopic.php?f=9&t=8350 This control has been working fine for me but please let me know of any improvements and I'll do my best to incorporate them - or you can just add them yourself :-). Pete Molly's Revenge On Sat, Jul 2, 2011 at 9:46 PM, Ken Ray wrote: > > > > I don't suppose this will make its appearance as a DropTool? > > Yup! But it may take a bit as I have a couple of other DropTools in the > works right now... > > :-) > > Ken Ray > Sons of Thunder Software, Inc. > Email: kray at sonsothunder.com > Web Site: http://www.sonsothunder.com/ > > > > _______________________________________________ > use-livecode mailing list > use-livecode at lists.runrev.com > Please visit this url to subscribe, unsubscribe and manage your > subscription preferences: > http://lists.runrev.com/mailman/listinfo/use-livecode > > From kray at sonsothunder.com Sun Jul 3 16:35:37 2011 From: kray at sonsothunder.com (Ken Ray) Date: Sun, 03 Jul 2011 15:35:37 -0500 Subject: Tab Key In-Reply-To: Message-ID: > This is a scrolling list field. The platform is Windows. > > Thanks for any help! The problem is that the "traversalOn" property of the list field is "false". This is shown in the properties palette as "Focusable". Make sure that is *checked*, and everything should work... Ken Ray Sons of Thunder Software, Inc. Email: kray at sonsothunder.com Web Site: http://www.sonsothunder.com/ From pete at mollysrevenge.com Sun Jul 3 17:58:14 2011 From: pete at mollysrevenge.com (Pete) Date: Sun, 3 Jul 2011 14:58:14 -0700 Subject: System Date/Time Format Problems In-Reply-To: References: <7C561176-82BD-45A5-9E68-A470C13DB8D4@gmail.com> <4E0F492F.5040701@hyperactivesw.com> <4E0FDEE2.7090507@hyperactivesw.com> Message-ID: I think I already mentioned this but I've entered QC report number 9604 about all this. There's various problems involved: 1) LC needs one more keyword so it can deal with the 4 date/time formats used by OS X. 2) There's a bug in the "abbrev system time" - it returns LC's default abbrev format, not the format in any of the OS X user preferences. 3) There's an inconsistency between the long system date and the long system time - the date returned is in the OS X full format, and the time is in the OS X medium format. These issues occur anywhere you refer to system dates and times, including the convert command. Jacque - thanks for your code - it works pretty well for standard US settings, as you said, but it doesn't help if the user has customised the System Preferences date and time formats or if they are not using US formats. My objective is to allow the user to tell me that dates are to be displayed in any one of the System Preference settings (or set a custom one of their own). I've figured out how to get the user's System Preference settings from a combination of the shell locale command and an Applescript and am busy writing a handler to format dates accordingly. Pete Molly's Revenge On Sun, Jul 3, 2011 at 11:14 AM, Pete wrote: > Thanks Jacque, that's a great start. I also found the dateFormat function > which I think may assist in all this. > Pete > Molly's Revenge > > > > > On Sat, Jul 2, 2011 at 8:15 PM, J. Landman Gay wrote: > >> On 7/2/11 2:46 PM, Pete wrote: >> >> So if a user tells me he wants his dates in OS X medium or long format, I >>> can't find out what that is with LC code, same for an OS X long time. >>> >> >> Start with these. The functions below will only work on US systems, or in >> countries that use US date/time formats. If you want to accomodate other >> countries you'll have to do more work. >> >> The "lookupZone" function is for the long and full time formats. I have >> exactly one entry in there for central time. What you need to do for that is >> make a list of all the time zones, one per line, in the same order my >> example uses. Store it in a custom property or somewhere. Retrieve the list >> for the lookup function. >> >> There may be a shell call to do that, or a list stored somewhere in OS X. >> I don't know. Anyway, maybe this will get you started: >> >> function formatDate pDate,pFormat -- short, medium, long, full >> switch pFormat >> case "short" >> convert pDate to short date >> put pDate into tFormattedDate >> break >> case "medium" >> convert pDate to abbrev date >> put item 2 to -1 of pDate into tFormattedDate >> break >> case "long" >> convert pDate to long date >> put item 2 to -1 of pDate into tFormattedDate >> break >> case "full" >> convert pDate to long date >> put pDate into tFormattedDate >> break >> end switch >> return tFormattedDate >> end formatDate >> >> function formatTime pTime,pFormat >> switch pFormat >> case "short" >> convert pTime to short time >> put pTime into tFormattedTime >> break >> case "medium" >> convert pTime to long time >> put pTime into tFormattedTime >> break >> case "long" >> case "full" >> convert pTime to long time >> put pTime into tFormattedTime >> convert pTime to internet date >> put word 6 of pTime into tTimeZone >> put lookupZone(tTimeZone) into tZoneInfo >> if pFormat = "long" then >> put item 2 of tZoneInfo into tTimeZone >> else -- full >> put item 3 of tZoneInfo into tTimeZone >> end if >> put space & tTimeZone after tFormattedTime >> break >> end switch >> return tFormattedTime >> end formatTime >> >> function lookupZone pHourOffset >> -- need a list in this format, one line per zone: >> put "-0500,CST,CT" into tZones >> return line lineoffset(pHourOffset,tZones) of tZones >> end lookupZone >> >> >> -- >> Jacqueline Landman Gay | jacque at hyperactivesw.com >> HyperActive Software | http://www.hyperactivesw.com >> >> ______________________________**_________________ >> use-livecode mailing list >> use-livecode at lists.runrev.com >> Please visit this url to subscribe, unsubscribe and manage your >> subscription preferences: >> http://lists.runrev.com/**mailman/listinfo/use-livecode >> >> > From capellan2000 at gmail.com Sun Jul 3 19:28:44 2011 From: capellan2000 at gmail.com (Alejandro Tejada) Date: Sun, 3 Jul 2011 19:28:44 -0400 Subject: Second test for Infinite Zoom Message-ID: Hi All, Today, I found time to revisit this code, originally published in 2009. Now the stack could zoom in and zoom out using the mouse scroll wheel. If you want, read that messages thread, started back in 2009: http://runtime-revolution.278305.n4.nabble.com/First-test-for-Infinite-Zoom-td621701.html I created two download links to public files in my Dropbox folder: http://dl.dropbox.com/u/3834621/Zoom_infinite_2011_07_03.zip http://dl.dropbox.com/u/3834621/Zoom_infinite_2011_07_03.rev Tell me if any of them fail. Still available in the web, here you could find the original images: http://public.hbk-bs.de/~baumgarn/zoom/steps/ Have a nice day! Al From jacque at hyperactivesw.com Sun Jul 3 22:01:45 2011 From: jacque at hyperactivesw.com (J. Landman Gay) Date: Sun, 03 Jul 2011 21:01:45 -0500 Subject: System Date/Time Format Problems In-Reply-To: References: <7C561176-82BD-45A5-9E68-A470C13DB8D4@gmail.com> <4E0F492F.5040701@hyperactivesw.com> <4E0FDEE2.7090507@hyperactivesw.com> Message-ID: <4E111F09.7030607@hyperactivesw.com> On 7/3/11 4:58 PM, Pete wrote: > I think I already mentioned this but I've entered QC report number 9604 > about all this. There's various problems involved: > > 1) LC needs one more keyword so it can deal with the 4 date/time formats > used by OS X. > 2) There's a bug in the "abbrev system time" - it returns LC's default > abbrev format, not the format in any of the OS X user preferences. > 3) There's an inconsistency between the long system date and the long system > time - the date returned is in the OS X full format, and the time is in the > OS X medium format. > > These issues occur anywhere you refer to system dates and times, including > the convert command. They aren't bugs, they're legacy formats that go back 20 years, and changing them would break a lot of stacks. When these date/time formats were implemented they matched the ones in Mac OS 6 and 7, and probably still matched in OS 9, though I don't remember. So rather than changing those, we should get new formats that represent more modern system options. This would be a feature request, so it would be good to update your report to reflect that. -- Jacqueline Landman Gay | jacque at hyperactivesw.com HyperActive Software | http://www.hyperactivesw.com From roger.e.eller at sealedair.com Sun Jul 3 22:13:51 2011 From: roger.e.eller at sealedair.com (Roger Eller) Date: Sun, 3 Jul 2011 22:13:51 -0400 Subject: Second test for Infinite Zoom In-Reply-To: References: Message-ID: On Sun, Jul 3, 2011 at 7:28 PM, Alejandro Tejada wrote: > Hi All, > > Today, I found time to revisit this code, originally published > in 2009. Now the stack could zoom in and zoom out using > the mouse scroll wheel. > > If you want, read that messages thread, started back in 2009: > > > http://runtime-revolution.278305.n4.nabble.com/First-test-for-Infinite-Zoom-td621701.html > > I created two download links to public files in my Dropbox folder: > > http://dl.dropbox.com/u/3834621/Zoom_infinite_2011_07_03.zip > http://dl.dropbox.com/u/3834621/Zoom_infinite_2011_07_03.rev > > Tell me if any of them fail. > > Still available in the web, here you could find > the original images: > > http://public.hbk-bs.de/~baumgarn/zoom/steps/ > > Have a nice day! > > Al > An implementation / adaptation of the Pyramid TIFF file format would also allow for fast zooming of bitmaps over the internet or LAN. I wonder how difficult it would be to create a LiveCode native library for PTIF. http://www.digitalpreservation.gov/formats/fdd/fdd000237.shtml ?Roger From chipp at chipp.com Sun Jul 3 23:30:17 2011 From: chipp at chipp.com (Chipp Walters) Date: Sun, 3 Jul 2011 22:30:17 -0500 Subject: Anyone using MobGUI? In-Reply-To: <4E105A8D.8070901@splash21.com> References: <4E105A8D.8070901@splash21.com> Message-ID: On Sun, Jul 3, 2011 at 7:03 AM, John Craig wrote: > > > The scripts are protected - if I can generate some revenue from the plugin, > then I can dedicate time to developing it further. > I'm not sure exactly the correlation between scripts being protected and generating revenue from them. I've been involved with this community since the very start and can't recall anyone ripping off someone else's scripts, thus causing a loss of revenue to the developer. Furthermore, I have seen several tremendous libraries for inclusion into codebases which are non-protected-- and the developer does well (for instance Ken Ray's excellent all Transcript XML library: http://sonsothunder.com/products/xmllib/xmllib.htm ) I suspect if you asked around, you'd find many of the commercial developers aren't fond of using protected libraries at all. They can present a number of problems for a developer and their client if/when things go wrong. So, basically by protecting the scripts, you're effectively locking out some potential customers. I can more understand developers protecting scripts when those scripts are part of tools which don't end up inside the codebase of another developer's project. For instance it might be a good idea to protect a plugin script which acts as a debugger for a project. Even so, I think many are still unlocked because mostly folks here are more eager to share how things are done, rather than hiding how things are done. At Altuit, we've developed for sale add-ons to Rev since before revSelect even existed-- among the more popular altBrowser (which became revBrowser), altSQLite (which became revSQLite), along with the libraries which went with them, and all of the scripts and libraries were unlocked. Also, the source code for the different tools was available in case anyone had an issue. Still, it certainly is your decision. I will support your product, and the fine work they represent, by purchasing a copy anyway. Hopefully in the future you'll consider unlocking the libraries. > Then again, if I decided that using LiveCode was too hard to support > because I didn't have the source code, and started writing my own cross > platform development engine, things would take quite a bit longer ;) > Yes, but they do let you have access to the whole IDE (but the licensing code) from wthin the LiveCode IDE. > The community is small at the moment, but I'm excited about how LiveCode is > progressing and the things that are in the pipeline. I'm hoping that the > small community is replaced by a huge community. > Yes, we've all been hoping so for several years now. > There is a tab control on the way - I'm trying to build a full feature set. > The plugin is still in it's early days, and by no means perfect. I've > still got a couple of support tickets to sort out just for buttons - the > feedback and bug reports have been excellent - really helping to shape > things to come. > Is there a known list of problem areas? Might be helpful if there are more than a few. > Great presentation last night, BTW - I enjoyed it. There are a couple of > techniques that I've discovered during the development of MobGUI that I've > thought "hey, that could be a good topic for LiveCode.TV", so although it's > not open source so far, there's nothing stopping me sharing ideas and > interesting bits and pieces with others! > Just so you know, unprotecting your scripts does not equal making them 'Open Source.' Unprotected scripts still have the protection of copyright law. I'm not the biggest OS advocate, and I do agree you should be able to create revenue from your hard work. From chipp at chipp.com Sun Jul 3 23:35:31 2011 From: chipp at chipp.com (Chipp Walters) Date: Sun, 3 Jul 2011 22:35:31 -0500 Subject: Pinch Zooming an image in iOS Message-ID: These were asked in another thread, but got derailed discussing protecting library stacks. Does anyone have an answer? 1. What is currently the best version of OSX iOS which to work with? 2. Anyone know how to pinch zoom a picture? Is there already a control or library for that? Can Rev even accomplish a decent performance implementation of this via transcript, or does an external need to be written? -- Chipp Walters CEO, Shafer Walters Group, Inc. From jacque at hyperactivesw.com Sun Jul 3 23:51:15 2011 From: jacque at hyperactivesw.com (J. Landman Gay) Date: Sun, 03 Jul 2011 22:51:15 -0500 Subject: Anyone using MobGUI? In-Reply-To: References: <4E105A8D.8070901@splash21.com> Message-ID: <4E1138B3.6000002@hyperactivesw.com> On 7/3/11 10:30 PM, Chipp Walters wrote: > On Sun, Jul 3, 2011 at 7:03 AM, John Craig wrote: >> >> >> The scripts are protected - if I can generate some revenue from the plugin, >> then I can dedicate time to developing it further. >> > > I'm not sure exactly the correlation between scripts being protected and > generating revenue from them. I've been involved with this community since > the very start and can't recall anyone ripping off someone else's scripts, > thus causing a loss of revenue to the developer. I agree we're a pretty good bunch. But I believe most or all the market vendors protect their scripts. Without that, it's too easy for someone to "lend" the script to someone else. I don't begrudge them a bit of protection, and it doesn't seem to have hurt sales. -- Jacqueline Landman Gay | jacque at hyperactivesw.com HyperActive Software | http://www.hyperactivesw.com From chipp at chipp.com Sun Jul 3 23:54:05 2011 From: chipp at chipp.com (Chipp Walters) Date: Sun, 3 Jul 2011 22:54:05 -0500 Subject: interesting custom property behavior... Message-ID: This is one, in all the years of programming Rev and LC, I didn't know about. This weekend, when I was talking about the altButton droptool at LiveCode TV, I mentioned all the properties can be explicitly set outside of the object inspector. For instance to change the width of the object, you can just set it's 'altWidth' property-- and I proceeded to show by going into the LC inspector palette, find the altWidth custom property, and change it. But, nothing happened. I tried again. Nothing happened. I tried with a different custom property. Even though the property value was set, nothing happened. Later I found out I could enter in msg: "set the altWidth of the selObj to 25" and it worked just fine. And the other custom properties worked similarly. It turns out when you set a custom prop from within a LC custom property inspector, it doesn't pass the setProp message. I'm sure this design is on purpose, but I wonder why? -- Chipp Walters CEO, Shafer Walters Group, Inc. From chipp at chipp.com Sun Jul 3 23:57:22 2011 From: chipp at chipp.com (Chipp Walters) Date: Sun, 3 Jul 2011 22:57:22 -0500 Subject: Anyone using MobGUI? In-Reply-To: <4E1138B3.6000002@hyperactivesw.com> References: <4E105A8D.8070901@splash21.com> <4E1138B3.6000002@hyperactivesw.com> Message-ID: Can't someone just lend the whole plugin? I know with altBrowser and altSQLite and altFont, all you needed was a regcode and you were good to go. I guess what I'm saying, is you're never going to stop the pirates. They won't pay no matter what. Most of us in the commercial space know this and we instead try and focus on stopping casual 'lenders' of software- if at all possible. My thinking with regard to copy protection is once you make it too difficult for your honest users, you've lost-- and they will look to other solutions. On Sun, Jul 3, 2011 at 10:51 PM, J. Landman Gay wrote: > On 7/3/11 10:30 PM, Chipp Walters wrote: > >> On Sun, Jul 3, 2011 at 7:03 AM, John Craig wrote: >> >>> >>> >>> The scripts are protected - if I can generate some revenue from the >>> plugin, >>> then I can dedicate time to developing it further. >>> >>> >> I'm not sure exactly the correlation between scripts being protected and >> generating revenue from them. I've been involved with this community since >> the very start and can't recall anyone ripping off someone else's scripts, >> thus causing a loss of revenue to the developer. >> > > I agree we're a pretty good bunch. But I believe most or all the market > vendors protect their scripts. Without that, it's too easy for someone to > "lend" the script to someone else. I don't begrudge them a bit of > protection, and it doesn't seem to have hurt sales. > > -- > Jacqueline Landman Gay | jacque at hyperactivesw.com > HyperActive Software | http://www.hyperactivesw.com > > > ______________________________**_________________ > use-livecode mailing list > use-livecode at lists.runrev.com > Please visit this url to subscribe, unsubscribe and manage your > subscription preferences: > http://lists.runrev.com/**mailman/listinfo/use-livecode > -- Chipp Walters CEO, Shafer Walters Group, Inc. From jacque at hyperactivesw.com Sun Jul 3 23:58:19 2011 From: jacque at hyperactivesw.com (J. Landman Gay) Date: Sun, 03 Jul 2011 22:58:19 -0500 Subject: interesting custom property behavior... In-Reply-To: References: Message-ID: <4E113A5B.5090100@hyperactivesw.com> On 7/3/11 10:54 PM, Chipp Walters wrote: > It turns out when you set a custom prop from within a LC custom property > inspector, it doesn't pass the setProp message. I'm sure this design is on > purpose, but I wonder why? > Recursion? -- Jacqueline Landman Gay | jacque at hyperactivesw.com HyperActive Software | http://www.hyperactivesw.com From chipp at chipp.com Mon Jul 4 00:02:51 2011 From: chipp at chipp.com (Chipp Walters) Date: Sun, 3 Jul 2011 23:02:51 -0500 Subject: Anyone using MobGUI? In-Reply-To: <4E1138B3.6000002@hyperactivesw.com> References: <4E105A8D.8070901@splash21.com> <4E1138B3.6000002@hyperactivesw.com> Message-ID: On Sun, Jul 3, 2011 at 10:51 PM, J. Landman Gay wrote: > > I agree we're a pretty good bunch. But I believe most or all the market > vendors protect their scripts. Without that, it's too easy for someone to > "lend" the script to someone else. I don't begrudge them a bit of > protection, and it doesn't seem to have hurt sales. Also, as I mentioned, I have no problems with folks protecting scripts which are not inserted into my project code. For instance, Mark Schonewille makes an installer which integrates in LC. I don't know if it's protected or not, but it wouldn't bother me to use it unless it inserts scripts directly into my stack which make it impossible for me to debug errors. I don't use it, but I suspect his tool only at most sets some custom props, much like what Jerry Daniels excellent development tools did. Furthermore, with a framework like MobGUI, I suspect you just can't 'lend' a script, but instead one would need the whole framework to make it all work. From chipp at chipp.com Mon Jul 4 00:04:30 2011 From: chipp at chipp.com (Chipp Walters) Date: Sun, 3 Jul 2011 23:04:30 -0500 Subject: interesting custom property behavior... In-Reply-To: <4E113A5B.5090100@hyperactivesw.com> References: <4E113A5B.5090100@hyperactivesw.com> Message-ID: Isn't there the property: the recursionLimit? Seems like an easy fix. On Sun, Jul 3, 2011 at 10:58 PM, J. Landman Gay wrote: > On 7/3/11 10:54 PM, Chipp Walters wrote: > > It turns out when you set a custom prop from within a LC custom property >> inspector, it doesn't pass the setProp message. I'm sure this design is on >> purpose, but I wonder why? >> >> > Recursion? > > -- > Jacqueline Landman Gay | jacque at hyperactivesw.com > HyperActive Software | http://www.hyperactivesw.com > > ______________________________**_________________ > use-livecode mailing list > use-livecode at lists.runrev.com > Please visit this url to subscribe, unsubscribe and manage your > subscription preferences: > http://lists.runrev.com/**mailman/listinfo/use-livecode > -- Chipp Walters CEO, Shafer Walters Group, Inc. From pete at mollysrevenge.com Mon Jul 4 00:13:17 2011 From: pete at mollysrevenge.com (Pete) Date: Sun, 3 Jul 2011 21:13:17 -0700 Subject: System Date/Time Format Problems In-Reply-To: <4E111F09.7030607@hyperactivesw.com> References: <7C561176-82BD-45A5-9E68-A470C13DB8D4@gmail.com> <4E0F492F.5040701@hyperactivesw.com> <4E0FDEE2.7090507@hyperactivesw.com> <4E111F09.7030607@hyperactivesw.com> Message-ID: I think I only described item 2 as a bug (and it definitely is). Item 1 is an enhancement request and item 3, who knows - it's an "inconsistency", which is not good. New formats would be fine with me if they fix the issues. No doubt the mothership will categorise them however they see fit and take the appropriate action. Since they seem to be 99% focused on mobile platforms right now (probably correctly), I'm not holding my breath for any of them to be fixed any time soon. Pete Molly's Revenge On Sun, Jul 3, 2011 at 7:01 PM, J. Landman Gay wrote: > On 7/3/11 4:58 PM, Pete wrote: > >> I think I already mentioned this but I've entered QC report number 9604 >> about all this. There's various problems involved: >> >> 1) LC needs one more keyword so it can deal with the 4 date/time formats >> used by OS X. >> 2) There's a bug in the "abbrev system time" - it returns LC's default >> abbrev format, not the format in any of the OS X user preferences. >> 3) There's an inconsistency between the long system date and the long >> system >> time - the date returned is in the OS X full format, and the time is in >> the >> OS X medium format. >> >> These issues occur anywhere you refer to system dates and times, including >> the convert command. >> > > They aren't bugs, they're legacy formats that go back 20 years, and > changing them would break a lot of stacks. When these date/time formats were > implemented they matched the ones in Mac OS 6 and 7, and probably still > matched in OS 9, though I don't remember. > > So rather than changing those, we should get new formats that represent > more modern system options. This would be a feature request, so it would be > good to update your report to reflect that. > > > -- > Jacqueline Landman Gay | jacque at hyperactivesw.com > HyperActive Software | http://www.hyperactivesw.com > > ______________________________**_________________ > use-livecode mailing list > use-livecode at lists.runrev.com > Please visit this url to subscribe, unsubscribe and manage your > subscription preferences: > http://lists.runrev.com/**mailman/listinfo/use-livecode > > From slava at lexiconbridge.com Mon Jul 4 00:15:11 2011 From: slava at lexiconbridge.com (Slava Paperno) Date: Mon, 4 Jul 2011 00:15:11 -0400 Subject: delete array element Message-ID: <003501cc3a00$fae71140$f0b533c0$@com> Could someone help me translate this entry in the 4.6.2 Dictionary, please? === delete variable Type: command Syntax: delete {local | global | variable} {variableName | arrayIndex} === What is the "arrayIndex" here? The index of what array? It seems to be an alternative to "variableName", but that doesn't make sense. The same entry in the Dictionary also has this example: === delete global myArray[17] -- removes 17th element of that array === When I try to use that command in the Message box, I get an error: global myVariable; put "A" into myVariable[1]; put "B" into myVariable[2]; delete global myVariable[2] "Error description: delete: bad variable expression" I also tried "delete global myVariable[2]" and got this: "Script compile error: Error description: create: no file name supplied" What am I doing wrong? Other than deleting an element, my use of array, including subarrays, is unproblematic. But my attempts to delete the last element of an array (so its length is shortened by one) bomb. (I don't mean to put an empty value into the last element--I do mean to delete the last element so it doesn't exist.) Thanks! Slava From pete at mollysrevenge.com Mon Jul 4 00:22:11 2011 From: pete at mollysrevenge.com (Pete) Date: Sun, 3 Jul 2011 21:22:11 -0700 Subject: Anyone using MobGUI? In-Reply-To: References: <4E105A8D.8070901@splash21.com> <4E1138B3.6000002@hyperactivesw.com> Message-ID: I'm just starting to consider this issue so glad it's being discussed. I think protection of stacks that are incorporated into other products is perhaps more to do with protecting them from end users rather than the developers who incorporate them into their products. I would gladly give the password to my stacks to another developer, but would definitely hesitate to give it to an end user. To me, it's not an issue of piracy, more of someone who doesn't know what they're doing changing my code and then expecting support for it when it crashes. Pete Molly's Revenge On Sun, Jul 3, 2011 at 8:57 PM, Chipp Walters wrote: > Can't someone just lend the whole plugin? I know with altBrowser and > altSQLite and altFont, all you needed was a regcode and you were good to > go. > > I guess what I'm saying, is you're never going to stop the pirates. They > won't pay no matter what. Most of us in the commercial space know this and > we instead try and focus on stopping casual 'lenders' of software- if at > all > possible. > > My thinking with regard to copy protection is once you make it too > difficult > for your honest users, you've lost-- and they will look to other solutions. > > On Sun, Jul 3, 2011 at 10:51 PM, J. Landman Gay >wrote: > > > On 7/3/11 10:30 PM, Chipp Walters wrote: > > > >> On Sun, Jul 3, 2011 at 7:03 AM, John Craig wrote: > >> > >>> > >>> > >>> The scripts are protected - if I can generate some revenue from the > >>> plugin, > >>> then I can dedicate time to developing it further. > >>> > >>> > >> I'm not sure exactly the correlation between scripts being protected and > >> generating revenue from them. I've been involved with this community > since > >> the very start and can't recall anyone ripping off someone else's > scripts, > >> thus causing a loss of revenue to the developer. > >> > > > > I agree we're a pretty good bunch. But I believe most or all the market > > vendors protect their scripts. Without that, it's too easy for someone to > > "lend" the script to someone else. I don't begrudge them a bit of > > protection, and it doesn't seem to have hurt sales. > > > > -- > > Jacqueline Landman Gay | jacque at hyperactivesw.com > > HyperActive Software | http://www.hyperactivesw.com > > > > > > ______________________________**_________________ > > use-livecode mailing list > > use-livecode at lists.runrev.com > > Please visit this url to subscribe, unsubscribe and manage your > > subscription preferences: > > http://lists.runrev.com/**mailman/listinfo/use-livecode< > http://lists.runrev.com/mailman/listinfo/use-livecode> > > > > > > -- > Chipp Walters > CEO, Shafer Walters Group, Inc. > _______________________________________________ > use-livecode mailing list > use-livecode at lists.runrev.com > Please visit this url to subscribe, unsubscribe and manage your > subscription preferences: > http://lists.runrev.com/mailman/listinfo/use-livecode > > From pete at mollysrevenge.com Mon Jul 4 00:23:28 2011 From: pete at mollysrevenge.com (Pete) Date: Sun, 3 Jul 2011 21:23:28 -0700 Subject: record sound problem on windows In-Reply-To: <4E107F1B.9050904@harryscollar.com> References: <51098212-42E7-4B3F-9D8D-A33609C476DA@swissonline.ch> <4E107F1B.9050904@harryscollar.com> Message-ID: > > Using the standard "record sound file "test.wav", works fine on OSX but the > resulting file when recorded on XP is high pitched & out of tempo. > > Are you sure you weren't recording a bagpipe band?!?! From alex at harryscollar.com Mon Jul 4 00:25:22 2011 From: alex at harryscollar.com (Alex Shaw) Date: Mon, 04 Jul 2011 14:25:22 +1000 Subject: record sound problem on windows In-Reply-To: <4E107F1B.9050904@harryscollar.com> References: <51098212-42E7-4B3F-9D8D-A33609C476DA@swissonline.ch> <4E107F1B.9050904@harryscollar.com> Message-ID: <4E1140B2.3040606@harryscollar.com> Hi Just tried on a Windows 7 machine. Works ok. This is definitely a XP-only issue. Anyone recording audio on XP successfully? If not, I will bugzilla it. regards alex On 4/07/11 12:39 AM, Alex Shaw wrote: > Hi > > I'm having a similar problem recording sound on Windows XP with > updated quicktime. > > Using the standard "record sound file "test.wav", works fine on OSX > but the resulting file when recorded on XP is high pitched & out of > tempo. > > Is there a special setting needed for XP? Have tried various record > rates but to no avail. > > regards > alex > > On 19/03/11 11:47 PM, Martin Meili wrote: >> Hi, >> I've got a soud recording problem on Windows XP with LiveCode 4.5.3. >> >> The command works fine on Mac OSX, both in the stackfile mode and the >> standalone mode. >> >> On Windows it doesn't work neither in the stackfile mode nor the >> standalone mode. I doesn't matter whether I work on my Mac or my Dell >> windows computer - I've got on both platforms the same problem: >> "record sound" doesn't work on Windows. >> >> If I open the stack with Revolution 4.0 and do the stadalone-build, >> I get a working Standalone for Windows which does the "record sound" >> command correctly. >> >> Anybody there who knows about the problem (bug??)? >> >> Cheers >> Martin >> >> _______________________________________________ >> use-livecode mailing list >> use-livecode at lists.runrev.com >> Please visit this url to subscribe, unsubscribe and manage your >> subscription preferences: >> http://lists.runrev.com/mailman/listinfo/use-livecode >> > > _______________________________________________ > use-livecode mailing list > use-livecode at lists.runrev.com > Please visit this url to subscribe, unsubscribe and manage your > subscription preferences: > http://lists.runrev.com/mailman/listinfo/use-livecode > From chipp at chipp.com Mon Jul 4 00:39:56 2011 From: chipp at chipp.com (Chipp Walters) Date: Sun, 3 Jul 2011 23:39:56 -0500 Subject: Anyone using MobGUI? In-Reply-To: References: <4E105A8D.8070901@splash21.com> <4E1138B3.6000002@hyperactivesw.com> Message-ID: Aren't all purchasers of a product like this developers by default? Why would end users ever delve into application code? Certainly, I've never tried looking at the source code for a commercial application, even when available. And, mostly code in stacks can't be edited unless you're in an LC IDE. Not sure what your point is. On Sun, Jul 3, 2011 at 11:22 PM, Pete wrote: > I'm just starting to consider this issue so glad it's being discussed. I > think protection of stacks that are incorporated into other products is > perhaps more to do with protecting them from end users rather than the > developers who incorporate them into their products. I would gladly give > the password to my stacks to another developer, but > would definitely hesitate to give it to an end user. To me, it's not an > issue of piracy, more of someone who doesn't know what they're doing > changing my code and then expecting support for it when it crashes. > > Pete > Molly's Revenge > > > > > On Sun, Jul 3, 2011 at 8:57 PM, Chipp Walters wrote: > > > Can't someone just lend the whole plugin? I know with altBrowser and > > altSQLite and altFont, all you needed was a regcode and you were good to > > go. > > > > I guess what I'm saying, is you're never going to stop the pirates. They > > won't pay no matter what. Most of us in the commercial space know this > and > > we instead try and focus on stopping casual 'lenders' of software- if at > > all > > possible. > > > > My thinking with regard to copy protection is once you make it too > > difficult > > for your honest users, you've lost-- and they will look to other > solutions. > > > > On Sun, Jul 3, 2011 at 10:51 PM, J. Landman Gay < > jacque at hyperactivesw.com > > >wrote: > > > > > On 7/3/11 10:30 PM, Chipp Walters wrote: > > > > > >> On Sun, Jul 3, 2011 at 7:03 AM, John Craig wrote: > > >> > > >>> > > >>> > > >>> The scripts are protected - if I can generate some revenue from the > > >>> plugin, > > >>> then I can dedicate time to developing it further. > > >>> > > >>> > > >> I'm not sure exactly the correlation between scripts being protected > and > > >> generating revenue from them. I've been involved with this community > > since > > >> the very start and can't recall anyone ripping off someone else's > > scripts, > > >> thus causing a loss of revenue to the developer. > > >> > > > > > > I agree we're a pretty good bunch. But I believe most or all the market > > > vendors protect their scripts. Without that, it's too easy for someone > to > > > "lend" the script to someone else. I don't begrudge them a bit of > > > protection, and it doesn't seem to have hurt sales. > > > > > > -- > > > Jacqueline Landman Gay | jacque at hyperactivesw.com > > > HyperActive Software | http://www.hyperactivesw.com > > > > > > > > > ______________________________**_________________ > > > use-livecode mailing list > > > use-livecode at lists.runrev.com > > > Please visit this url to subscribe, unsubscribe and manage your > > > subscription preferences: > > > http://lists.runrev.com/**mailman/listinfo/use-livecode< > > http://lists.runrev.com/mailman/listinfo/use-livecode> > > > > > > > > > > > -- > > Chipp Walters > > CEO, Shafer Walters Group, Inc. > > _______________________________________________ > > use-livecode mailing list > > use-livecode at lists.runrev.com > > Please visit this url to subscribe, unsubscribe and manage your > > subscription preferences: > > http://lists.runrev.com/mailman/listinfo/use-livecode > > > > > _______________________________________________ > use-livecode mailing list > use-livecode at lists.runrev.com > Please visit this url to subscribe, unsubscribe and manage your > subscription preferences: > http://lists.runrev.com/mailman/listinfo/use-livecode > -- Chipp Walters CEO, Shafer Walters Group, Inc. From mwieder at ahsoftware.net Mon Jul 4 00:46:03 2011 From: mwieder at ahsoftware.net (Mark Wieder) Date: Sun, 3 Jul 2011 21:46:03 -0700 Subject: interesting custom property behavior... In-Reply-To: References: Message-ID: <114133526828.20110703214603@ahsoftware.net> Chipp- Sunday, July 3, 2011, 8:54:05 PM, you wrote: > It turns out when you set a custom prop from within a LC custom property > inspector, it doesn't pass the setProp message. I'm sure this design is on > purpose, but I wonder why? I believe the property inspector locks messages before setting properties. But that's just dragged from somewhere in my memory - I don't think it's documented and I couldn't tell you why I think that or if it's right. It does seem to fit empirically though. -- -Mark Wieder mwieder at ahsoftware.net From niconiko at gmail.com Mon Jul 4 01:09:35 2011 From: niconiko at gmail.com (Nicolas Cueto) Date: Mon, 4 Jul 2011 14:09:35 +0900 Subject: object beneath mouseLoc? Message-ID: <76D636BD-265D-45C8-852D-1B7FABD17A0F@gmail.com> Hello. Does LC have a built-in way of returning the id of the object(s) at a mouseLoc? For now, I'm relying on mouseMove combined with mouseColor. But on my stack there's a weakness to this method. (It's a grid of round-edged tiles each of which can be deleted if contiguously connected. A diagonal move between tiles, though, enters the clearing between the rounded corners and causes mouseColor to trigger.) As always, thank you for the help. -- Nicolas Cueto (iPhone) From niconiko at gmail.com Mon Jul 4 01:10:24 2011 From: niconiko at gmail.com (Nicolas Cueto) Date: Mon, 4 Jul 2011 14:10:24 +0900 Subject: command "group" fails when objectList is a string In-Reply-To: References: Message-ID: That was perfect, Klaus. Thanks. -- Nicolas Cueto (iPhone) From revdev at pdslabs.net Mon Jul 4 02:23:00 2011 From: revdev at pdslabs.net (Phil Davis) Date: Sun, 03 Jul 2011 23:23:00 -0700 Subject: object beneath mouseLoc? In-Reply-To: <76D636BD-265D-45C8-852D-1B7FABD17A0F@gmail.com> References: <76D636BD-265D-45C8-852D-1B7FABD17A0F@gmail.com> Message-ID: <4E115C44.4050405@pdslabs.net> Try "the mouseControl" - that should work. Phil On 7/3/11 10:09 PM, Nicolas Cueto wrote: > Hello. > > Does LC have a built-in way of returning the id of the object(s) at a mouseLoc? > > For now, I'm relying on mouseMove combined with mouseColor. But on my stack there's a weakness to this method. (It's a grid of round-edged tiles each of which can be deleted if contiguously connected. A diagonal move between tiles, though, enters the clearing between the rounded corners and causes mouseColor to trigger.) > > As always, thank you for the help. > -- > Nicolas Cueto (iPhone) > _______________________________________________ > use-livecode mailing list > use-livecode at lists.runrev.com > Please visit this url to subscribe, unsubscribe and manage your subscription preferences: > http://lists.runrev.com/mailman/listinfo/use-livecode > -- Phil Davis PDS Labs Professional Software Development http://pdslabs.net From toolbook at kestner.de Mon Jul 4 03:13:33 2011 From: toolbook at kestner.de (Tiemo Hollmann TB) Date: Mon, 4 Jul 2011 09:13:33 +0200 Subject: AW: why does lock screen doesn't locks the screen? In-Reply-To: References: <000d01cc37c6$f9286c50$eb7944f0$@de> Message-ID: <006c01cc3a19$e6d98430$b48c8c90$@de> Good idea Chris, thank you Tiemo > -----Urspr?ngliche Nachricht----- > Von: use-livecode-bounces at lists.runrev.com [mailto:use-livecode- > bounces at lists.runrev.com] Im Auftrag von Nonsanity > Gesendet: Freitag, 1. Juli 2011 17:29 > An: How to use LiveCode > Betreff: Re: why does lock screen doesn't locks the screen? > > "lock screen" only locks the window contents, not the whole screen (despite > the name) and never the cursor. The cursor belongs to the user and lives > above the screen. But you can "lock cursor" to keep in in the shape that you > want. > > I can't get "set cursor to none" to hide the cursor here, so there may be > something wrong with that feature. What you can do instead is make a 16x16 > all-alpha png file and import that into the stack. Then you can set the > cursor to that image's id and then lock the cursor and do your movements. > When the handler exits, the lock will be released, but it's always better to > put an "unlock cursor" (or screen) because these commands stack. (Five locks > takes five unlocks.) > > I made such an image and tested it, and it worked. I'd give you a link to > the image, but Dropbox is being flaky right now. Wait... I'll put it on > another server: > > http://nonsanity.com/x/invisible16x16.png > > And here's my script. I made sure the points I picked were over different > objects, like a field, to make sure the cursor wouldn't change as it would > normally. > > on mouseUp > set the cursor to 1007 > lock cursor > set the cursor to 1007 > wait 50 > repeat with a = 1 to 4 > set the screenmouseloc to word a of "202,398 290,270 168,222 363,594" > wait 50 > end repeat > end mouseUp > > Hope that helps. > > ~ Chris Innanen > ~ Nonsanity > > > On Fri, Jul 1, 2011 at 4:14 AM, Tiemo Hollmann TB wrote: > > > Hello, > > > > In a function I set the screenMouseLoc to four different locations on an > > image to get the mousecolor of these four locations. > > > > I have set the screen lock at the beginning of the function, but the cursor > > is flickering at all four locations, so obviously the screen is redrawn > > everytime I set the mouse at a new location though the screenloc is true. > > > > What can I do to not see the mouse (and perhaps speed up the function), and > > why does the lock screen doesn't work in this case? > > > > Thanks > > > > Tiemo > > > > > > > > > > > > _______________________________________________ > > use-livecode mailing list > > use-livecode at lists.runrev.com > > Please visit this url to subscribe, unsubscribe and manage your > > subscription preferences: > > http://lists.runrev.com/mailman/listinfo/use-livecode > > > _______________________________________________ > use-livecode mailing list > use-livecode at lists.runrev.com > Please visit this url to subscribe, unsubscribe and manage your subscription > preferences: > http://lists.runrev.com/mailman/listinfo/use-livecode From niconiko at gmail.com Mon Jul 4 06:45:55 2011 From: niconiko at gmail.com (Nicolas Cueto) Date: Mon, 4 Jul 2011 19:45:55 +0900 Subject: object beneath mouseLoc? In-Reply-To: <4E115C44.4050405@pdslabs.net> References: <76D636BD-265D-45C8-852D-1B7FABD17A0F@gmail.com> <4E115C44.4050405@pdslabs.net> Message-ID: > Try "the mouseControl" - that should work. > Phil Thanks, Phil. Didn't know about that function. But, still a no-go. My script relies on mouseMove and mouseStillDown to let the user select grouped objects by dragging the mouse. I'd also now like the script to "know" when the user has moved out of an object, even if the mouse is down. According to the docs, though, "if the mouse button is down, the mouseControl function returns the control that was clicked, even if the mouse has moved to another control". I tested mouseControl out anyway, and sure enough, only the clicked control gets returned. Perhaps there's other suggestions... -- Nicolas Cueto From niconiko at gmail.com Mon Jul 4 06:52:49 2011 From: niconiko at gmail.com (Nicolas Cueto) Date: Mon, 4 Jul 2011 19:52:49 +0900 Subject: object beneath mouseLoc? In-Reply-To: References: <76D636BD-265D-45C8-852D-1B7FABD17A0F@gmail.com> <4E115C44.4050405@pdslabs.net> Message-ID: I should also note that I'm using AnimationEngine, in case there's an AE-based solution. -- Nicolas Cueto From keith.clarke at clarkeandclarke.co.uk Mon Jul 4 06:55:10 2011 From: keith.clarke at clarkeandclarke.co.uk (Keith Clarke) Date: Mon, 4 Jul 2011 11:55:10 +0100 Subject: object beneath mouseLoc? In-Reply-To: References: <76D636BD-265D-45C8-852D-1B7FABD17A0F@gmail.com> <4E115C44.4050405@pdslabs.net> Message-ID: <11227A64-7FCD-4EF4-A0C7-640B71EFE874@clarkeandclarke.co.uk> Could you track mouseEnter messages to capture the object rect entered (or does that only work on fields - sorry not on my LC machine to check the docs)? Best, Keith.. On 4 Jul 2011, at 11:45, Nicolas Cueto wrote: >> Try "the mouseControl" - that should work. >> Phil > > Thanks, Phil. Didn't know about that function. But, still a no-go. > > My script relies on mouseMove and mouseStillDown to let the user > select grouped objects by dragging the mouse. I'd also now like the > script to "know" when the user has moved out of an object, even if the > mouse is down. According to the docs, though, "if the mouse button is > down, the mouseControl function returns the control that was clicked, > even if the mouse has moved to another control". I tested mouseControl > out anyway, and sure enough, only the clicked control gets returned. > > Perhaps there's other suggestions... > > -- > Nicolas Cueto > > _______________________________________________ > use-livecode mailing list > use-livecode at lists.runrev.com > Please visit this url to subscribe, unsubscribe and manage your subscription preferences: > http://lists.runrev.com/mailman/listinfo/use-livecode From niconiko at gmail.com Mon Jul 4 07:05:15 2011 From: niconiko at gmail.com (Nicolas Cueto) Date: Mon, 4 Jul 2011 20:05:15 +0900 Subject: object beneath mouseLoc? In-Reply-To: <11227A64-7FCD-4EF4-A0C7-640B71EFE874@clarkeandclarke.co.uk> References: <76D636BD-265D-45C8-852D-1B7FABD17A0F@gmail.com> <4E115C44.4050405@pdslabs.net> <11227A64-7FCD-4EF4-A0C7-640B71EFE874@clarkeandclarke.co.uk> Message-ID: > Could you track mouseEnter messages to capture the object rect entered Way back when, I tried both mouseEnter and mouseLeave. And learnt the hard way that these messages only get sent when the mouse is no longer down. Which is when I began experimenting with mouseStillDown and mouseMove. Thanks anyway, Keith. -- Nicolas Cueto From ben at runrev.com Mon Jul 4 07:14:56 2011 From: ben at runrev.com (Benjamin Beaumont) Date: Mon, 4 Jul 2011 12:14:56 +0100 Subject: record sound problem on windows In-Reply-To: <4E1140B2.3040606@harryscollar.com> References: <51098212-42E7-4B3F-9D8D-A33609C476DA@swissonline.ch> <4E107F1B.9050904@harryscollar.com> <4E1140B2.3040606@harryscollar.com> Message-ID: Hi Alex, If you report this in the QCC we'll investigate it. If you could upload the stack that is demonstrating the problem at your end that will help us a great deal. Warm regards, Ben On 4 July 2011 05:25, Alex Shaw wrote: > Hi > > Just tried on a Windows 7 machine. Works ok. > > This is definitely a XP-only issue. > > Anyone recording audio on XP successfully? If not, I will bugzilla it. > > regards > alex > > > On 4/07/11 12:39 AM, Alex Shaw wrote: > >> Hi >> >> I'm having a similar problem recording sound on Windows XP with updated >> quicktime. >> >> Using the standard "record sound file "test.wav", works fine on OSX but >> the resulting file when recorded on XP is high pitched & out of tempo. >> >> Is there a special setting needed for XP? Have tried various record rates >> but to no avail. >> >> regards >> alex >> >> On 19/03/11 11:47 PM, Martin Meili wrote: >> >>> Hi, >>> I've got a soud recording problem on Windows XP with LiveCode 4.5.3. >>> >>> The command works fine on Mac OSX, both in the stackfile mode and the >>> standalone mode. >>> >>> On Windows it doesn't work neither in the stackfile mode nor the >>> standalone mode. I doesn't matter whether I work on my Mac or my Dell >>> windows computer - I've got on both platforms the same problem: "record >>> sound" doesn't work on Windows. >>> >>> If I open the stack with Revolution 4.0 and do the stadalone-build, I >>> get a working Standalone for Windows which does the "record sound" command >>> correctly. >>> >>> Anybody there who knows about the problem (bug??)? >>> >>> Cheers >>> Martin >>> >>> ______________________________**_________________ >>> use-livecode mailing list >>> use-livecode at lists.runrev.com >>> Please visit this url to subscribe, unsubscribe and manage your >>> subscription preferences: >>> http://lists.runrev.com/**mailman/listinfo/use-livecode >>> >>> >> ______________________________**_________________ >> use-livecode mailing list >> use-livecode at lists.runrev.com >> Please visit this url to subscribe, unsubscribe and manage your >> subscription preferences: >> http://lists.runrev.com/**mailman/listinfo/use-livecode >> >> > ______________________________**_________________ > use-livecode mailing list > use-livecode at lists.runrev.com > Please visit this url to subscribe, unsubscribe and manage your > subscription preferences: > http://lists.runrev.com/**mailman/listinfo/use-livecode > -- _____________________________________________ Benjamin Beaumont . RunRev Ltd LiveCode Product Manager mail : 25a Thistle Street Lane South West, Edinburgh, EH2 1EW email : ben at runrev.com company : +44(0) 845 219 89 23 fax : +44(0) 845 458 8487 web : www.runrev.com From richmondmathewson at gmail.com Mon Jul 4 07:18:46 2011 From: richmondmathewson at gmail.com (Richmond Mathewson) Date: Mon, 04 Jul 2011 14:18:46 +0300 Subject: object beneath mouseLoc? In-Reply-To: References: <76D636BD-265D-45C8-852D-1B7FABD17A0F@gmail.com> <4E115C44.4050405@pdslabs.net> Message-ID: <4E11A196.7070904@gmail.com> On 07/04/2011 01:45 PM, Nicolas Cueto wrote: >> Try "the mouseControl" - that should work. >> Phil > Thanks, Phil. Didn't know about that function. But, still a no-go. > > My script relies on mouseMove and mouseStillDown to let the user > select grouped objects by dragging the mouse. I'd also now like the > script to "know" when the user has moved out of an object, even if the > mouse is down. According to the docs, though, "if the mouse button is > down, the mouseControl function returns the control that was clicked, > even if the mouse has moved to another control". I tested mouseControl > out anyway, and sure enough, only the clicked control gets returned. > > Perhaps there's other suggestions... I have a feeling that on mouseLeave works whether the mouse button is up or down. > -- > Nicolas Cueto > > _______________________________________________ > use-livecode mailing list > use-livecode at lists.runrev.com > Please visit this url to subscribe, unsubscribe and manage your subscription preferences: > http://lists.runrev.com/mailman/listinfo/use-livecode From niconiko at gmail.com Mon Jul 4 07:25:01 2011 From: niconiko at gmail.com (Nicolas Cueto) Date: Mon, 4 Jul 2011 20:25:01 +0900 Subject: object beneath mouseLoc? In-Reply-To: <4E11A196.7070904@gmail.com> References: <76D636BD-265D-45C8-852D-1B7FABD17A0F@gmail.com> <4E115C44.4050405@pdslabs.net> <4E11A196.7070904@gmail.com> Message-ID: > I have a feeling that ?on mouseLeave ?works whether the mouse button > is up or down. About mouseLeave from the docs: "If the mouse button is down when the mouse pointer leaves the control, the mouseLeave message is not sent until the mouse button is released." -- Nicolas Cueto From keith.clarke at clarkeandclarke.co.uk Mon Jul 4 07:50:45 2011 From: keith.clarke at clarkeandclarke.co.uk (Keith Clarke) Date: Mon, 4 Jul 2011 12:50:45 +0100 Subject: object beneath mouseLoc? In-Reply-To: References: <76D636BD-265D-45C8-852D-1B7FABD17A0F@gmail.com> <4E115C44.4050405@pdslabs.net> <4E11A196.7070904@gmail.com> Message-ID: What about the on drag... messages - if the mouse is down during movements then these should be active? Best, Keith.. On 4 Jul 2011, at 12:25, Nicolas Cueto wrote: >> I have a feeling that on mouseLeave works whether the mouse button >> is up or down. > > About mouseLeave from the docs: > > "If the mouse button is down when the mouse pointer leaves the > control, the mouseLeave message is not sent until the mouse button is > released." > > -- > Nicolas Cueto > > _______________________________________________ > use-livecode mailing list > use-livecode at lists.runrev.com > Please visit this url to subscribe, unsubscribe and manage your subscription preferences: > http://lists.runrev.com/mailman/listinfo/use-livecode From dixonja at hotmail.co.uk Mon Jul 4 08:02:39 2011 From: dixonja at hotmail.co.uk (John Dixon) Date: Mon, 4 Jul 2011 13:02:39 +0100 Subject: object beneath mouseLoc? In-Reply-To: References: <76D636BD-265D-45C8-852D-1B7FABD17A0F@gmail.com>, <4E115C44.4050405@pdslabs.net>, , <4E11A196.7070904@gmail.com>, , Message-ID: Would 'mouseWithin' help ? on mouseDown repeat until the mouseLoc is not within the rect of me put the mouseLoc wait 0 millisecs with messages end repeat beep end mouseDown > >> I have a feeling that on mouseLeave works whether the mouse button > >> is up or down. > > > > About mouseLeave from the docs: > > > > "If the mouse button is down when the mouse pointer leaves the > > control, the mouseLeave message is not sent until the mouse button is > > released." > > > > -- > > Nicolas Cueto From richmondmathewson at gmail.com Mon Jul 4 08:48:16 2011 From: richmondmathewson at gmail.com (Richmond Mathewson) Date: Mon, 04 Jul 2011 15:48:16 +0300 Subject: object beneath mouseLoc? In-Reply-To: References: <76D636BD-265D-45C8-852D-1B7FABD17A0F@gmail.com> <4E115C44.4050405@pdslabs.net> <4E11A196.7070904@gmail.com> Message-ID: <4E11B690.6000404@gmail.com> On 07/04/2011 02:25 PM, Nicolas Cueto wrote: >> I have a feeling that on mouseLeave works whether the mouse button >> is up or down. > About mouseLeave from the docs: > > "If the mouse button is down when the mouse pointer leaves the > control, the mouseLeave message is not sent until the mouse button is > released." > Blast! Never mind, cheer up! I just set up a stack consisting of one card and an image called "gl", and a field called "POZ". I put this into the card script: on mouseMove if the mouseLoc is not within the rect of img "GL" then put "outwith" into field "POZ" else put "inwith" into field "POZ" end if end mouseMove and this works REGARDLESS as to whether the mouse button is DOWN or UP . . . :) > -- > Nicolas Cueto > > _______________________________________________ > use-livecode mailing list > use-livecode at lists.runrev.com > Please visit this url to subscribe, unsubscribe and manage your subscription preferences: > http://lists.runrev.com/mailman/listinfo/use-livecode From kray at sonsothunder.com Mon Jul 4 09:03:54 2011 From: kray at sonsothunder.com (Ken Ray) Date: Mon, 04 Jul 2011 08:03:54 -0500 Subject: interesting custom property behavior... In-Reply-To: <114133526828.20110703214603@ahsoftware.net> Message-ID: >> It turns out when you set a custom prop from within a LC custom property >> inspector, it doesn't pass the setProp message. I'm sure this design is on >> purpose, but I wonder why? > > I believe the property inspector locks messages before setting > properties. But that's just dragged from somewhere in my memory - I > don't think it's documented and I couldn't tell you why I think that > or if it's right. It does seem to fit empirically though. You're absolutely right, Mark... I ran into this same thing when prepping for my demonstration of DropTools at the LiveCode TV #29 event. I even called Richard to confirm that this wasn't a bug. Sure enough, messages are being locked, and there's a note in the IDE script for the handler related to bug #8345: # OK-2009-10-17 : Bug 8345 # We should lock messages whenver a property is get or set to prevent code # execution caused by custom getProp / setProp handlers. Failing to do # this causes the user to be confused about the actual value of the property # and also could cause stacks to be broken e.g. by running code the user # hadn't finished testing and accidently deleted objects. I've since had to explicitly tell people using the LC IDE to set custom properties via the Message Box (or use the MC IDE, where this doesn't happen). Ken Ray Sons of Thunder Software, Inc. Email: kray at sonsothunder.com Web Site: http://www.sonsothunder.com/ From kray at sonsothunder.com Mon Jul 4 09:05:58 2011 From: kray at sonsothunder.com (Ken Ray) Date: Mon, 04 Jul 2011 08:05:58 -0500 Subject: delete array element In-Reply-To: <003501cc3a00$fae71140$f0b533c0$@com> Message-ID: > When I try to use that command in the Message box, I get an error: > > global myVariable; put "A" into myVariable[1]; put "B" into myVariable[2]; > delete global myVariable[2] > > "Error description: delete: bad variable expression" This is one of those cases where the Message Box can't resolve things properly. If you put the same code in a button an click it, it works without an error. Ken Ray Sons of Thunder Software, Inc. Email: kray at sonsothunder.com Web Site: http://www.sonsothunder.com/ From kray at sonsothunder.com Mon Jul 4 09:11:41 2011 From: kray at sonsothunder.com (Ken Ray) Date: Mon, 04 Jul 2011 08:11:41 -0500 Subject: object beneath mouseLoc? In-Reply-To: <4E11B690.6000404@gmail.com> Message-ID: > Never mind, cheer up! > > I just set up a stack consisting of one card and an image called "gl", and a > field called "POZ". > > I put this into the card script: > > on mouseMove > if the mouseLoc is not within the rect of img "GL" then > put "outwith" into field "POZ" > else > put "inwith" into field "POZ" > end if > end mouseMove > > and this works REGARDLESS as to whether the mouse button is DOWN or UP . > . . :) Yes, this is one of the idiosyncracies of LC, going way back to when it was MetaCard... once you mouseDown on an object, the target remains the originally clicked object even if you ask for things like mouseControl, mouseChar, mouseChunk, etc. The only current way is through polling during mouseMove, which, although ugly, is usually fast enough even if you have hundreds of objects: on mouseMove if the mouse is down then repeat with x = 1 to the number of controls if the mouseLoc is within the rect of control x then -- do what you want to do with it else -- take some other action end if end repeat end if end mouseMove Personally I'd like to kick that concept to the curb and have all mouseXXX function relate to where the actual mouse *is*, regardless of whether it's state is up or down... Ken Ray Sons of Thunder Software, Inc. Email: kray at sonsothunder.com Web Site: http://www.sonsothunder.com/ From alex at harryscollar.com Mon Jul 4 09:15:23 2011 From: alex at harryscollar.com (Alex Shaw) Date: Mon, 04 Jul 2011 23:15:23 +1000 Subject: record sound problem on windows In-Reply-To: References: <51098212-42E7-4B3F-9D8D-A33609C476DA@swissonline.ch> <4E107F1B.9050904@harryscollar.com> <4E1140B2.3040606@harryscollar.com> Message-ID: <4E11BCEB.3060907@harryscollar.com> Hi Benjamin See.. http://quality.runrev.com/show_bug.cgi?id=9606 regards alex On 4/07/11 9:14 PM, Benjamin Beaumont wrote: > Hi Alex, > > If you report this in the QCC we'll investigate it. If you could upload the > stack that is demonstrating the problem at your end that will help us a > great deal. > > Warm regards, > > Ben > > On 4 July 2011 05:25, Alex Shaw wrote: > >> Hi >> >> Just tried on a Windows 7 machine. Works ok. >> >> This is definitely a XP-only issue. >> >> Anyone recording audio on XP successfully? If not, I will bugzilla it. >> >> regards >> alex >> >> >> On 4/07/11 12:39 AM, Alex Shaw wrote: >> >>> Hi >>> >>> I'm having a similar problem recording sound on Windows XP with updated >>> quicktime. >>> >>> Using the standard "record sound file "test.wav", works fine on OSX but >>> the resulting file when recorded on XP is high pitched& out of tempo. >>> >>> Is there a special setting needed for XP? Have tried various record rates >>> but to no avail. >>> >>> regards >>> alex >>> >>> On 19/03/11 11:47 PM, Martin Meili wrote: >>> >>>> Hi, >>>> I've got a soud recording problem on Windows XP with LiveCode 4.5.3. >>>> >>>> The command works fine on Mac OSX, both in the stackfile mode and the >>>> standalone mode. >>>> >>>> On Windows it doesn't work neither in the stackfile mode nor the >>>> standalone mode. I doesn't matter whether I work on my Mac or my Dell >>>> windows computer - I've got on both platforms the same problem: "record >>>> sound" doesn't work on Windows. >>>> >>>> If I open the stack with Revolution 4.0 and do the stadalone-build, I >>>> get a working Standalone for Windows which does the "record sound" command >>>> correctly. >>>> >>>> Anybody there who knows about the problem (bug??)? >>>> >>>> Cheers >>>> Martin >>>> >>>> ______________________________**_________________ >>>> use-livecode mailing list >>>> use-livecode at lists.runrev.com >>>> Please visit this url to subscribe, unsubscribe and manage your >>>> subscription preferences: >>>> http://lists.runrev.com/**mailman/listinfo/use-livecode >>>> >>>> >>> ______________________________**_________________ >>> use-livecode mailing list >>> use-livecode at lists.runrev.com >>> Please visit this url to subscribe, unsubscribe and manage your >>> subscription preferences: >>> http://lists.runrev.com/**mailman/listinfo/use-livecode >>> >>> >> ______________________________**_________________ >> use-livecode mailing list >> use-livecode at lists.runrev.com >> Please visit this url to subscribe, unsubscribe and manage your >> subscription preferences: >> http://lists.runrev.com/**mailman/listinfo/use-livecode >> > > From kray at sonsothunder.com Mon Jul 4 09:24:35 2011 From: kray at sonsothunder.com (Ken Ray) Date: Mon, 04 Jul 2011 08:24:35 -0500 Subject: System Date/Time Format Problems In-Reply-To: Message-ID: I have a handler I've been using for a while that works great with formatting dates. Here it is (sorry for the length, but it's heavily commented - oh and watch for word wraps): ---------------------------------------------------------------------- --| FUNCTION: stsFormattedDate --| --| Author: Ken Ray --| Version: 2.6 --| Created: 8/2/04 --| Last Mod: 2/8/11 --| Requires: -- --| --| Formats an incoming date according to a variety of date --| patterns and outputs the result. --| --| Parameters: --| : The date to be formatted. Must be a date or --| date/time combination, or empty. If empty, uses the --| current date and default 2AM time if time is --| requested in the pattern. Will also check to see if the --| date is a mySQL-formatted --| date if it can't be identified as a "normal" date. --| : The pattern to use to format the incoming date. --| The following patterns are available where "M" is used for --| the month, "D" for the day, "Y" for the year, "W" for --| the weekday name, "H" for hours, "P" for AM/PM, --| "N" for minutes, "S" for seconds, and "G" for GMT --| calculation, as follows: --| M = month number, no leading zeroes (1-12) --| MM = month number, with leading zeroes (01-12) --| MMM = month name, abbreviated (Jan - Dec) --| MMMM = month name, long (January - December) --| D = day number, no leading zeroes (1-31) --| DD = day number, with leading zeroes (01-31) --| W = weekday name, single letter (S/M/T/W/t/F/s) --| WW = weekday name, shortest (Su/M/Tu/W/Th/F/Sa) --| WWW = weekday name, abbreviated (Sun/Mon/Tue/Wed/ --| Thu/Fri/Sat) --| WWWW = weekday name, long (Sunday/Monday/Tuesday/etc.) --| YY = two-digit year (00-99) --| YYYY = four digit year (1970-2004) --| H = hours, no leading zeroes, 12 hour format (1-12) --| HH = hours, leading zeroes, 12 hour format (01-12) --| HHH = hours, no leading zeroes, 24 hour format (1-23) --| HHHH = hours, leading zeroes, 24 hour format (01-23) --| HHHHH = military time format (0000 - 2359) --| P = AM/PM, single character, lower case (a/p) --| PP = AM/PM, single character, upper case (A/P) --| PPP = AM/PM, two characters, lower case (am/pm) --| PPPP = AM/PM, two characters, upper case (AM/PM) --| N = minutes, no leading zeroes (0-59) --| NN = minutes, leading zeroes (00-59) --| S = seconds, no leading zeroes (0-59) --| SS = seconds, leading zeroes (00-59) --| G = GMT Offset (-1100 to +1100) --| GH = GMT Offset hours, including sign (-11 to +11) --| GM = GMT Offset minutes --| : Determines whether or not the format needs to --| have brackets surrounding each part of the pattern. If true, --| it requires that all patterns are surrounded by square --| brackets, and allows the letters used in the pattern --| (MDYWHNSGP) to be used as part of the return string that --| is *not* part of the pattern (for example "GMT" as a string). --| If false, pattern letters are replaced by the corresponding --| date parts in the returned string (so it assumes that --| characters that are not part of the pattern are not letters, --| but symbols). ---------------------------------------------------------------------- function stsFormattedDate pDate,pFormat,pUseBrackets if (pDate = "") or (pDate = "Now") then put the date && the long time into pDate if (pFormat = "") then put "MM/DD/YYYY" into pFormat if isNumber(word -1 of pDate) then if (word -1 of pDate <=2359) and (length(word -1 of pDate)=4) then -- date and military time sent in, just needs a colon between -- hour and minute for it to be converted put ":" before char -2 of pDate else -- simple number, coerce to AM if word -1 of pDate <= 12 then put ((word -1 of pDate) & ":00 AM") into word -1 of pDate else -- a number larger than 12 but not military? Can't do anything -- with that return "invalid time" end if end if end if -- Check for am/pm without preceding space put word -1 of pDate into tTestTime if (tTestTime <> "AM") and (tTestTime <> "PM") then put offset("a",tTestTime) into tLoc if (tLoc <> 0) and (char (tLoc-1) of pDate <> " ") then if char (tLoc+1) of tTestTime <> "m" then put "m" after \ char tLoc of tTestTime put " " before char tLoc of tTestTime put word -2 of tTestTime into tTime if ":"is not in tTime then -- probably something like "2am" put (tTime & ":00") into word -2 of tTestTime end if put tTestTime into word -1 of pDate end if put offset("p",tTestTime) into tLoc if (tLoc <> 0) and (char (tLoc-1) of tTestTime <> " ") then if char (tLoc+1) of tTestTime <> "m" then put "m" after \ char tLoc of tTestTime put " " before char tLoc of tTestTime put word -2 of tTestTime into tTime if ":"is not in tTime then -- probably something like "2pm" put (tTime & ":00") into word -2 of tTestTime end if put tTestTime into word -1 of pDate end if end if put ((pUseBrackets <> "") and (pUseBrackets <> "false")) into pUseBrackets put pDate into tOrigDate -- Check to see if it's mySQL-formatted if matchText(word 1 of pDate,"(?s)(.*)-(.*)-(.*)",tY,tM,tD) and \ length(tY)=4 then if word 2 of pDate <> "" then put tM & "/" & tD & "/" & tY && (word 2 of pDate) into pDate else put tM & "/" & tD & "/" & tY into pDate end if convert pDate to dateItems if (pDate is "invalid date") or (("69" is not in tOrigDate) and \ (item 1 of pDate="1969")) then return "invalid date" end if else convert pDate to dateItems if (pDate is "invalid date") or (("69" is not in tOrigDate) and \ (item 1 of pDate="1969")) then return "invalid date" end if end if put item 1 of pDate into tYear put item 2 of pDate into tMonthNum put item 3 of pDate into tDayNum put item 4 of pDate into tHour put item 5 of pDate into tMinute put item 6 of pDate into tSecond put item 7 of pDate into tWeekdayNum put word -1 of the internet date into G put char -2 to -1 of G into GM put char 1 to -3 of G into GH set the numberFormat to "00" if pUseBrackets then put "[GH],[GM],[G],[MMMM],[MMM],[MM],[M],[WWWW],[WWW]," & \ "[WW],[W],[DD],[D],[YYYY],[YY],[HHHHH],[HHHH],[HHH],[HH]," & \ "[H],[PPPP],[PPP],[PP],[P],[NN],[N],[SS],[S]" into tFormatWords else put "GH,GM,G,MMMM,MMM,MM,M,WWWW,WWW,WW,W," & \ "DD,D,YYYY,YY,HHHHH,HHHH,HHH,HH,H,PPPP,PPP," & \ "PP,P,NN,N,SS,S" into tFormatWords end if put "!@#$%^&*()_+{}|:'<>?~`-=[]" into tReplaceChars repeat with x = 1 to the number of items of tFormatWords replace (item x of tFormatWords) with \ "[[[" & char x of tReplaceChars & "]]]]" in pFormat end repeat put tYear into YYYY put char -2 to -1 of tYear into YY put tMonthNum into M put (tMonthNum+0) into MM put line tMonthNum of the abbreviated monthNames into MMM put line tMonthNum of the long monthNames into MMMM put tDayNum into D put (tDayNum+0) into DD put char tWeekDayNum of "SMTWtFs" into W put item tWeekDayNum of "S,M,Tu,W,Th,F,Sa" into WW put line tWeekDayNum of the abbreviated weekdayNames into WWW put line tWeekDayNum of the long weekdayNames into WWWW put tHour into HHH put (tHour+0) into HHHH if tHour < 12 then if tHour = 0 then put 12 into H else put tHour into H put "a" into P put "A" into PP put "am" into PPP put "AM" into PPPP else put tHour-12 into H if H = 0 then put 12 into H if H < 10 then delete char 1 of H -- remove leading 0 end if put "p" into P put "P" into PP put "pm" into PPP put "PM" into PPPP end if put (H+0) into HH put tMinute into N put (tMinute+0) into NN put HHHH & NN into HHHHH put tSecond into S put (tSecond+0) into SS repeat with x = 1 to the number of items of tFormatWords if pUseBrackets then local tTemp get matchText(item x of tFormatWords,"\[(.*?)\]",tTemp) do "put" && tTemp && "into tVal" else do "put" && (item x of tFormatWords) && "into tVal" end if replace "[[[" & char x of tReplaceChars & "]]]]" with tVal in pFormat end repeat return pFormat end stsFormattedDate Use it like this: put stsFormattedDate(the date && the long time,"M/D/YYYY HH:NN p") --> 7/4/2011 08:23 a Hopes this helps, Ken Ray Sons of Thunder Software, Inc. Email: kray at sonsothunder.com Web Site: http://www.sonsothunder.com/ From mwieder at ahsoftware.net Mon Jul 4 13:21:13 2011 From: mwieder at ahsoftware.net (Mark Wieder) Date: Mon, 4 Jul 2011 10:21:13 -0700 Subject: interesting custom property behavior... In-Reply-To: References: Message-ID: <74178836859.20110704102113@ahsoftware.net> Ken- Monday, July 4, 2011, 6:03:54 AM, you wrote: > Sure enough, messages are being locked, and there's a note in the IDE script > for the handler related to bug #8345: Yeah, I ran into this when I was throwing PowerTools together as well and had to some some arcane scripting to get around the problem when trying to set default properties for objects. -- -Mark Wieder mwieder at ahsoftware.net From jacque at hyperactivesw.com Mon Jul 4 13:22:02 2011 From: jacque at hyperactivesw.com (J. Landman Gay) Date: Mon, 04 Jul 2011 12:22:02 -0500 Subject: object beneath mouseLoc? In-Reply-To: References: Message-ID: <4E11F6BA.2000707@hyperactivesw.com> On 7/4/11 8:11 AM, Ken Ray wrote: > > Personally I'd like to kick that concept to the curb and have all mouseXXX > function relate to where the actual mouse *is*, regardless of whether it's > state is up or down... That would break scripts that need to act on mouseRelease, wouldn't it? For the OP: my Klondike stack polls "is within" on dozens of controls on each mouseMove message, many times per second, and shows no sign of delay. It's very fast. -- Jacqueline Landman Gay | jacque at hyperactivesw.com HyperActive Software | http://www.hyperactivesw.com From mwieder at ahsoftware.net Mon Jul 4 13:28:02 2011 From: mwieder at ahsoftware.net (Mark Wieder) Date: Mon, 4 Jul 2011 10:28:02 -0700 Subject: Anyone using MobGUI? In-Reply-To: References: <4E105A8D.8070901@splash21.com> Message-ID: <166179245750.20110704102802@ahsoftware.net> Chipp- Sunday, July 3, 2011, 8:30:17 PM, you wrote: > I can more understand developers protecting scripts when those scripts are > part of tools which don't end up inside the codebase of another developer's > project. For instance it might be a good idea to protect a plugin script > which acts as a debugger for a project. Even so, I think many are still > unlocked because mostly folks here are more eager to share how things are > done, rather than hiding how things are done. The only time I ever lock scripts is so I can make annoying nag trial versions that are fully functional but otherwise nudge the user in the direction of making a purchase. Everything else is wide open so that maybe someone can learn something, find ugly code to laugh at, and poke around and improve or fix things. -- -Mark Wieder mwieder at ahsoftware.net From andre at andregarzia.com Mon Jul 4 13:36:09 2011 From: andre at andregarzia.com (Andre Garzia) Date: Mon, 4 Jul 2011 14:36:09 -0300 Subject: Anyone using MobGUI? In-Reply-To: <166179245750.20110704102802@ahsoftware.net> References: <4E105A8D.8070901@splash21.com> <166179245750.20110704102802@ahsoftware.net> Message-ID: On Mon, Jul 4, 2011 at 2:28 PM, Mark Wieder wrote: > Chipp- > > Sunday, July 3, 2011, 8:30:17 PM, you wrote: > > > I can more understand developers protecting scripts when those scripts > are > > part of tools which don't end up inside the codebase of another > developer's > > project. For instance it might be a good idea to protect a plugin script > > which acts as a debugger for a project. Even so, I think many are still > > unlocked because mostly folks here are more eager to share how things are > > done, rather than hiding how things are done. > > The only time I ever lock scripts is so I can make annoying nag trial > versions that are fully functional but otherwise nudge the user in the > direction of making a purchase. Everything else is wide open so that > maybe someone can learn something, find ugly code to laugh at, and > poke around and improve or fix things. > > sometimes I am afraid that someone will actually read the code on RevOnRockets, specially the processrequest handler... that code is so full of hacks that will make the reader question my sanity. > -- > -Mark Wieder > mwieder at ahsoftware.net > > > _______________________________________________ > use-livecode mailing list > use-livecode at lists.runrev.com > Please visit this url to subscribe, unsubscribe and manage your > subscription preferences: > http://lists.runrev.com/mailman/listinfo/use-livecode > -- http://www.andregarzia.com All We Do Is Code. From pete at mollysrevenge.com Mon Jul 4 13:56:40 2011 From: pete at mollysrevenge.com (Pete) Date: Mon, 4 Jul 2011 10:56:40 -0700 Subject: System Date/Time Format Problems In-Reply-To: References: Message-ID: Thanks a lot Ken, this looks great and I'll be trying it out very soon. I might add the ability to return the 3-char timezone since I just found out how to get hold of that from OS X (it's in word -2 of the shell date command) Pete Molly's Revenge On Mon, Jul 4, 2011 at 6:24 AM, Ken Ray wrote: > I have a handler I've been using for a while that works great with > formatting dates. Here it is (sorry for the length, but it's heavily > commented - oh and watch for word wraps): > > > > > > _______________________________________________ > use-livecode mailing list > use-livecode at lists.runrev.com > Please visit this url to subscribe, unsubscribe and manage your > subscription preferences: > http://lists.runrev.com/mailman/listinfo/use-livecode > > From chipp at chipp.com Mon Jul 4 14:18:34 2011 From: chipp at chipp.com (Chipp Walters) Date: Mon, 4 Jul 2011 13:18:34 -0500 Subject: Anyone using MobGUI? In-Reply-To: References: <4E105A8D.8070901@splash21.com> <166179245750.20110704102802@ahsoftware.net> Message-ID: And there I thought I was just not able to 'keep up' with your brilliant mind! On Mon, Jul 4, 2011 at 12:36 PM, Andre Garzia wrote: > sometimes I am afraid that someone will actually read the code on > RevOnRockets, specially the processrequest handler... > > that code is so full of hacks that will make the reader question my sanity. > > From kray at sonsothunder.com Mon Jul 4 14:51:28 2011 From: kray at sonsothunder.com (Ken Ray) Date: Mon, 04 Jul 2011 13:51:28 -0500 Subject: object beneath mouseLoc? In-Reply-To: <4E11F6BA.2000707@hyperactivesw.com> Message-ID: >> Personally I'd like to kick that concept to the curb and have all mouseXXX >> function relate to where the actual mouse *is*, regardless of whether it's >> state is up or down... > > That would break scripts that need to act on mouseRelease, wouldn't it? OK, you're right about mouseRelease; that should still be be sent to the original object, but I'd still do it for mouseEnter, mouseLeave, mouseMove, mouseWIthin, mouseChar, mouseCharChunk, mouseLine, mouseControl, mouseColor, and mouseText... :D Ken Ray Sons of Thunder Software, Inc. Email: kray at sonsothunder.com Web Site: http://www.sonsothunder.com/ From kray at sonsothunder.com Mon Jul 4 15:02:24 2011 From: kray at sonsothunder.com (Ken Ray) Date: Mon, 04 Jul 2011 14:02:24 -0500 Subject: interesting custom property behavior... In-Reply-To: <74178836859.20110704102113@ahsoftware.net> Message-ID: >> Sure enough, messages are being locked, and there's a note in the IDE script >> for the handler related to bug #8345: > > Yeah, I ran into this when I was throwing PowerTools together as well > and had to some some arcane scripting to get around the problem when > trying to set default properties for objects. Ah... perhaps messages should be locked when *getting* a custom prop, but unlocked when *setting* it through the Inspector...? Oh well, the point is moot now, I'd think. Ken Ray Sons of Thunder Software, Inc. Email: kray at sonsothunder.com Web Site: http://www.sonsothunder.com/ From richmondmathewson at gmail.com Mon Jul 4 15:03:02 2011 From: richmondmathewson at gmail.com (Richmond Mathewson) Date: Mon, 04 Jul 2011 22:03:02 +0300 Subject: Adventures in Unicode Message-ID: <4E120E66.8080107@gmail.com> possibly subtitled "Wading through treacle". My 'eternal' Devawriter didn't display glyphs properly once they were entered via a slightly mephistophelian process into the OUTPUT field, and at least one of my beta-testers was getting quite crochety about it (mainly as I had had no epiphanies of late). Then, flashes of slightly murky light, and it occurred to me that upping the textSize of the output field, and then setting it down by one again . . . take a deep breath, you know Richmond is daft . . . and then, of course, one has to shift the unicodeText out of the field while it is resized and then return it . . . But; it worketh . . . follow me closely: set the useUnicode to true set the unicodeText of the fld "HOLDER" to the unicodeText of fld "OUTPUT" put the textSize of fld "OUTPUT" into BIGGY put BIGGY + 1 into BIGGGY set the textSize of fld " OUTPUT" to BIGGGY put empty into fld "OUTPUT" set the unicodeText of the fld "OUTPUT" to the unicodeText of fld "HOLDER" put empty into fld "HOLDER" set the unicodeText of the fld "HOLDER" to the unicodeText of fld "OUTPUT" set the textSize of fld " OUTPUT" to BIGGY put empty into fld "OUTPUT" set the unicodeText of the fld "OUTPUT" to the unicodeText of fld "HOLDER" put empty into fld "HOLDER" select after fld "OUTPUT" this is just about as clunky as one gets, but it seems the only way in which one can ensure unicodeText is represented properly. and, before you mention it: http://en.wikipedia.org/wiki/Noddy_Holder I'm off to the funny-farm for a quick frontal lobotomy after that one (which took me about a year to come round to). From richmondmathewson at gmail.com Mon Jul 4 15:04:34 2011 From: richmondmathewson at gmail.com (Richmond Mathewson) Date: Mon, 04 Jul 2011 22:04:34 +0300 Subject: object beneath mouseLoc? In-Reply-To: References: Message-ID: <4E120EC2.7010303@gmail.com> On 07/04/2011 09:51 PM, Ken Ray wrote: >>> Personally I'd like to kick that concept to the curb and have all mouseXXX >>> function relate to where the actual mouse *is*, regardless of whether it's >>> state is up or down... >> That would break scripts that need to act on mouseRelease, wouldn't it? > OK, you're right about mouseRelease; that should still be be sent to the > original object, but I'd still do it for mouseEnter, mouseLeave, mouseMove, > mouseWIthin, mouseChar, mouseCharChunk, mouseLine, mouseControl, mouseColor, > and mouseText... > It threw me somewhat, when I realised that mouseLeave was dependent on the mouse button being up; a bit counter-intuitive. > :D > > Ken Ray > Sons of Thunder Software, Inc. > Email: kray at sonsothunder.com > Web Site: http://www.sonsothunder.com/ > > > > _______________________________________________ > use-livecode mailing list > use-livecode at lists.runrev.com > Please visit this url to subscribe, unsubscribe and manage your subscription preferences: > http://lists.runrev.com/mailman/listinfo/use-livecode From kray at sonsothunder.com Mon Jul 4 15:12:31 2011 From: kray at sonsothunder.com (Ken Ray) Date: Mon, 04 Jul 2011 14:12:31 -0500 Subject: [ANN] aagPasswordField DropTool Message-ID: For his first foray at developing a DropTool, Andre Garzia created aagPasswordField, an ?ber-simple DropTool that provides a simple password field that you can drag and drop into your stacks. You can get the password from the control by getting "the plaintextpassword" of it; pressing backspace or delete clears the field. You can read more about it here: http://droptools.sonsothunder.com/products/aag/aag-pw.irev Enjoy! Ken Ray Sons of Thunder Software, Inc. Email: kray at sonsothunder.com Web Site: http://www.sonsothunder.com/ From dick.kriesel at mail.com Mon Jul 4 15:13:54 2011 From: dick.kriesel at mail.com (Dick Kriesel) Date: Mon, 4 Jul 2011 12:13:54 -0700 Subject: object beneath mouseLoc? In-Reply-To: <76D636BD-265D-45C8-852D-1B7FABD17A0F@gmail.com> References: <76D636BD-265D-45C8-852D-1B7FABD17A0F@gmail.com> Message-ID: Hi, Nicolas. Your problem sounded familiar, so I found a relevant previous post. Does it work for you? On 3/19/07 3:38 AM, "Dick Kriesel" wrote: > Here's another technique for identifying the object at the mouseLoc. This > technique is different because it works with no repeat loop, no checking the > visible or the rect of any object, no formulas based on the mouseLoc, and no > reference to the mouseControl. > > Create a button named "mouseObject" with the following script: > > -- HTH, -- Peter Peter M. Brigham pmbrig at gmail.com http://home.comcast.net/~pmbrig On Jul 9, 2011, at 12:53 AM, Ian McKnight wrote: > Hi John > > I would look at the HTMLtext property in the Manual/Dictionary. > > You would take your text, line by line, and wrap each one with the > appropriate HTML code > eg > put ""&dateStamp&"&cr into myMsg > put ""&firstName&&surName&""&cr after > myMsg > put ""&theMessage&""&cr after myMsg > set the HTMLText of fld "myField" to myMsg > > To get the size 16 point text you may have too set the default text size of > the field to 16 - the HTML should vary it. > To get the ruled line I think it will be necessary to produce a graphic line > and use the tag to reference it. > > > I haven't tried this so you may have to change the colour references to > numeric values but this hopefully will put you on the correct track > > > On 9 July 2011 08:22, John Allijn wrote: > >> Hi, >> >> I have a textfield where I want to present a chunk of text, formatted in a >> few different styles. >> >> This is what the text looks like: >> >> --- >> On >> wrote: >> >> --- >> >> and this is how I like to represent it: >> >> - the text "on timestamp" should be gray and 8 points >> - the firstname and lastname should be 10 points, black and bold >> - the message should be 16 points, blue, regular. >> - put in a divider-line >> >> The problem is that the message-text varies in length and that there may be >> multiple messages in the field. This means (but maybe I'm wrong) that I have >> to set the TextColor and TextSize properties while populating the field and >> that I cannot do this afterwards. >> >> So: >> - How can I set the style of the text, put some text in the field and >> change the style again? >> - how can I put in a horizontal line that separates the messages? >> >> thank you for your thoughts! >> John. >> _______________________________________________ >> use-livecode mailing list >> use-livecode at lists.runrev.com >> Please visit this url to subscribe, unsubscribe and manage your >> subscription preferences: >> http://lists.runrev.com/mailman/listinfo/use-livecode >> > > > > -- > Regards > > > Ian McKnight > > iangmcknight at gmail.com > ======================= > _______________________________________________ > use-livecode mailing list > use-livecode at lists.runrev.com > Please visit this url to subscribe, unsubscribe and manage your subscription preferences: > http://lists.runrev.com/mailman/listinfo/use-livecode From rabit at dimensionB.de Sun Jul 10 12:23:21 2011 From: rabit at dimensionB.de (Ralf Bitter) Date: Sun, 10 Jul 2011 18:23:21 +0200 Subject: [ANN] AndreGarzia.Com new web page. In-Reply-To: References: Message-ID: <76DDCB2B-7FCC-4517-B703-872A471CE0F4@dimensionB.de> Andre, ? nice design, great content. Ralf On 10.07.2011, at 06:05, Andre Garzia wrote: > Hello Folks, > > It is full of joy and pride that I announce my website redesign here. Those > that accessed my home page recently knew that it was not being updated > often, actually, it was not being updated at all. I always wanted to > recreate everything with LiveCode and now I've did it. I started yesterday, > Friday and after a little more than 24h I've got a brand new website. This > is a testimonial of how easy it is to build web stuff with the new LiveCode > Server. > > This new page will serve as a hub for all my new developments and I plan to > release many many many things during the next days and weeks. I welcome > feedback and comments. > > http://andregarzia.com > > =) > > > For those that want to know, this homepage is driven with LiveCode Server. > It uses RevIgniter Framework. It uses no database at all, the pages are all > text files with HTML and special syntax in it. Keeping it simple allowed me > to deliver a full website in less than 24h. The site is easy to extend and > change. Since I am the developer and the user, I don't need an > administration interface, I can create pages by simply dropping plain text > files in the correct place and tweek things by editing special files. > > -- > http://www.andregarzia.com All We Do Is Code. From mwieder at ahsoftware.net Sun Jul 10 12:30:48 2011 From: mwieder at ahsoftware.net (Mark Wieder) Date: Sun, 10 Jul 2011 09:30:48 -0700 Subject: global variable change in substack not available to main stack In-Reply-To: <000f01cc3f10$25cc14a0$71643de0$@lexiconbridge.com> References: <000601cc3ec6$efa35fd0$ceea1f70$@com> <000001cc3ed4$75c3b210$614b1630$@com> <000f01cc3f10$25cc14a0$71643de0$@lexiconbridge.com> Message-ID: <165694213203.20110710093048@ahsoftware.net> Slava- Sunday, July 10, 2011, 7:46:21 AM, you wrote: > handler was forwarding the openCard message which in turn reinitialized the > globals, and thus the new value got lost. My only excuse is that it was 3 AM > on a Sunday. And this is exactly why I think the use of global variables is a Bad Idea. Global variables can be set from anywhere, and trying to track down what happened and why your code isn't working the way you expect it to is a real pain. I think I can count the number of times I've had to resort to global variables on the fingers of one hand. I don't think 3AM Sunday is the issue - it's not particularly anything you did wrong, it's just that the nature of global variables encourages these problem scenarios. -- -Mark Wieder mwieder at ahsoftware.net From mwieder at ahsoftware.net Sun Jul 10 12:32:16 2011 From: mwieder at ahsoftware.net (Mark Wieder) Date: Sun, 10 Jul 2011 09:32:16 -0700 Subject: [ANN] AndreGarzia.Com new web page. In-Reply-To: References: <72F388EE-95B5-4A27-8E7A-394E64164B9E@major.on-rev.com> Message-ID: <184694302015.20110710093216@ahsoftware.net> Andre- Sunday, July 10, 2011, 7:12:21 AM, you wrote: > yes they are from stockexchange... :-) Every April First in San Francisco we have our annual Sock Exchange. -- -Mark Wieder mwieder at ahsoftware.net From jrvalent at wisc.edu Sun Jul 10 13:08:51 2011 From: jrvalent at wisc.edu (rand valentine) Date: Sun, 10 Jul 2011 13:08:51 -0400 Subject: Missing Menu Items Message-ID: Thank you for taking the time to answer my request, Mark, I appreciate it very much. So, yes, I was using a mac to do that scripting. And yes, your comments were right on. There is one more component to my mess, though, namely that I had deleted some of the auto-added menu items, such as Preferences. This I think created chaos at some level. I wish we had better documentation of all of these details in LiveCode, but I'm not holding my breath, having used it and its ancestors for many years. But with your help I was able to at least get something working. rand Message: 10 Date: Sun, 10 Jul 2011 01:13:22 +0200 From: Mark Schonewille To: How to use LiveCode Subject: Re: missing menu items Message-ID: <6E10509D-B732-470B-86EC-7CCAA9F27D50 at economy-x-talk.com> Content-Type: text/plain; charset=us-ascii Hi Rand, I bet you're on a Mac. The Quit menu item is in the application menu as is the preferences menu item. Don't remove these items from the default menus and all will be fine. The "item" right above the Quit and Preferences menu items must be a divider (dash). -- Best regards, Mark Schonewille From jacque at hyperactivesw.com Sun Jul 10 14:02:26 2011 From: jacque at hyperactivesw.com (J. Landman Gay) Date: Sun, 10 Jul 2011 13:02:26 -0500 Subject: global variable change in substack not available to main stack In-Reply-To: <000d01cc3f0d$d3f2ef20$7bd8cd60$@lexiconbridge.com> References: <000601cc3ec6$efa35fd0$ceea1f70$@com> <143663480734.20110710005836@ahsoftware.net> <000d01cc3f0d$d3f2ef20$7bd8cd60$@lexiconbridge.com> Message-ID: <4E19E932.2050903@hyperactivesw.com> On 7/10/11 9:29 AM, Slava Paperno wrote: > I'm curious about globals and the message box. I've been doing both > (omitting the global declaration and including it) and I haven't seen any > difference, but I've seen conflicting opinions in the posts here as well as > some forums. What's the skinny on this? The message box has access to all globals for me, without any declarations. I don't use explicitVariables though; maybe that makes a difference. It's also fine to put your global declarations inside each handler that needs to use it. It's become customary to put it at the top of the script but that isn't required. In fact, to backtrack a bit, with explicitvariables set to false, it is actually possible to have both a global and a local variable with the same name in the same script as long as the global isn't declared at the top. I know we had a discussion about this before where I agreed it couldn't happen, but I've just tried it again and in fact you can, if you declare your global inside the handler: on setup global testVar put 1 into testVar put testVar end setup on test put 2 into testVar put testVar end test Turn off explicitVariables and run "setup" first from the message box to make sure the global is initialized. After that you'll get different numbers depending which you call. The failure I had when I tested before is that, like most people, I was declaring the global outside the handlers. I'd never recommend this kind of naming, of course. -- Jacqueline Landman Gay | jacque at hyperactivesw.com HyperActive Software | http://www.hyperactivesw.com From jacque at hyperactivesw.com Sun Jul 10 14:07:33 2011 From: jacque at hyperactivesw.com (J. Landman Gay) Date: Sun, 10 Jul 2011 13:07:33 -0500 Subject: Missing Menu Items In-Reply-To: References: Message-ID: <4E19EA65.6010900@hyperactivesw.com> On 7/10/11 12:08 PM, rand valentine wrote: > I wish we had better documentation of all of > these details in LiveCode, but I'm not holding my breath, having used it and > its ancestors for many years. But with your help I was able to at least get > something working. Page 259 of the User Guide. ;) As long-time users of xtalk, I think a lot of us haven't read that document. I know I've skipped a lot of it. -- Jacqueline Landman Gay | jacque at hyperactivesw.com HyperActive Software | http://www.hyperactivesw.com From slava at lexiconbridge.com Sun Jul 10 14:57:12 2011 From: slava at lexiconbridge.com (=?utf-8?B?c2xhdmFAbGV4aWNvbmJyaWRnZS5jb20=?=) Date: Sun, 10 Jul 2011 14:57:12 -0400 Subject: =?utf-8?B?UmU6IGdsb2JhbCB2YXJpYWJsZSBjaGFuZ2UgaW4gc3Vic3RhY2sgbm90IGF2YWlsYWJsZSB0byBtYWluIHN0YWNr?= Message-ID: Interesting. Thanks, Jacque. Mark, yes I see your point. You are right. S. Sent from my mobile so please excuse brevity and typos. ----- Reply message ----- From: "J. Landman Gay" To: "How to use LiveCode" Subject: global variable change in substack not available to main stack Date: Sun, Jul 10, 2011 2:02 pm On 7/10/11 9:29 AM, Slava Paperno wrote: > I'm curious about globals and the message box. I've been doing both > (omitting the global declaration and including it) and I haven't seen any > difference, but I've seen conflicting opinions in the posts here as well as > some forums. What's the skinny on this? The message box has access to all globals for me, without any declarations. I don't use explicitVariables though; maybe that makes a difference. It's also fine to put your global declarations inside each handler that needs to use it. It's become customary to put it at the top of the script but that isn't required. In fact, to backtrack a bit, with explicitvariables set to false, it is actually possible to have both a global and a local variable with the same name in the same script as long as the global isn't declared at the top. I know we had a discussion about this before where I agreed it couldn't happen, but I've just tried it again and in fact you can, if you declare your global inside the handler: on setup global testVar put 1 into testVar put testVar end setup on test put 2 into testVar put testVar end test Turn off explicitVariables and run "setup" first from the message box to make sure the global is initialized. After that you'll get different numbers depending which you call. The failure I had when I tested before is that, like most people, I was declaring the global outside the handlers. I'd never recommend this kind of naming, of course. -- Jacqueline Landman Gay | jacque at hyperactivesw.com HyperActive Software | http://www.hyperactivesw.com _______________________________________________ use-livecode mailing list use-livecode at lists.runrev.com Please visit this url to subscribe, unsubscribe and manage your subscription preferences: http://lists.runrev.com/mailman/listinfo/use-livecode From kray at sonsothunder.com Sun Jul 10 15:03:41 2011 From: kray at sonsothunder.com (Ken Ray) Date: Sun, 10 Jul 2011 14:03:41 -0500 Subject: Windows System Folder Script In-Reply-To: <1310311832.28703.YahooMailClassic@web161603.mail.bf1.yahoo.com> References: <1310311832.28703.YahooMailClassic@web161603.mail.bf1.yahoo.com> Message-ID: <1E13AAC0-B320-4249-BA6F-D3E39BA1FCBC@sonsothunder.com> On Jul 10, 2011, at 10:30 AM, Michael Kann wrote: > on mouseUp > set hideConsoleWindows to true > put shell(fld 1) into IGNORE > end mouseUp > > ------------------- > -- gotchas > ------------------- > Without the IGNORE you get the msg box popping up. That's because you're using "put"... if you do: get shell(fld 1) You don't need to worry about the IGNORE. > You need quotes because of the spaces in some folder names. No spaces around the comma. Not sure why "explorer" is needed, but it seems to be required for the folder names with spaces. The reason you need "explorer" is because that is the command you need to execute on the command line in Windows in order to pass along the name of the folder you want to open. When you use "shell" it acts as if you had popped up the Start menu and entered "cmd" in the Run box and then hit enter to bring up a Windows console window and then typed your command and executed it. This is the reason for setting the hideConsoleWindows to true - it prevents the command window from being displayed. When you pass a parameter with spaces to a command line app, you need to enclose it in quotes. You could just as easily done: put shell("explorer" && quote & "shell:Common Start Menu" & quote) into tCommonStartMenuFolder without having to use a field. > Folllowing is a list of the Vista Folders. You could put these in a fld and click on your choice. I've seen a similar list for XP and 7, but I can't seem to find it right now. Perhaps someone can locate it. There's a comparable internal command in LiveCode for geting a 'specialFolderPath'; on Windows you need to provide either a constant or a number that translates to the proper folder path. To see these I have a list on my site: http://www.sonsothunder.com/devres/revolution/tips/file010.htm Ken Ray Sons of Thunder Software, Inc. Email: kray at sonsothunder.com Web Site: http://www.sonsothunder.com/ From jrvalent at wisc.edu Sun Jul 10 15:07:46 2011 From: jrvalent at wisc.edu (rand valentine) Date: Sun, 10 Jul 2011 15:07:46 -0400 Subject: silver menu in Windows, why? Message-ID: <009101cc3f34$a7de6140$f79b23c0$@wisc.edu> Hi, all. I've put a menu in a stack (using the autobuild in the MenuBuilder) I've made which has a light blue background. In Windows, this menu for some reason has a silver opaque background behind it. Of course, with Mac OS the menu's beamed up, so background isn't an issue. I don't see a way to change this background. The menu set seems to be a group, and it seems the silver color is a property of this group. But I can't seem to find a property in the Colors to change to make the menu transparent. Could someone clue me about this? Thanks. rand valentine From john.allijn at alice.nl Sun Jul 10 15:08:48 2011 From: john.allijn at alice.nl (John Allijn) Date: Sun, 10 Jul 2011 21:08:48 +0200 Subject: text formatting In-Reply-To: <0E037C6C-E3E1-402C-AEED-EA28B70849F5@gmail.com> References: <0E037C6C-E3E1-402C-AEED-EA28B70849F5@gmail.com> Message-ID: Hi Peter, Ian, I'm getting closer :) Formatting works quite well this way. Thanks again, John. On Jul 10, 2011, at 6:16 PM, Peter Brigham MD wrote: > You can create the htmltext by hand as suggested, or you can do something like the following (not tested): > > ---------- > > HTH, > -- Peter > > Peter M. Brigham > pmbrig at gmail.com > http://home.comcast.net/~pmbrig > > > On Jul 9, 2011, at 12:53 AM, Ian McKnight wrote: > >> Hi John >> >> I would look at the HTMLtext property in the Manual/Dictionary. >> >> You would take your text, line by line, and wrap each one with the >> appropriate HTML code >> eg >> put ""&dateStamp&"&cr into myMsg >> put ""&firstName&&surName&""&cr after >> myMsg >> put ""&theMessage&""&cr after myMsg >> set the HTMLText of fld "myField" to myMsg >> >> To get the size 16 point text you may have too set the default text size of >> the field to 16 - the HTML should vary it. >> To get the ruled line I think it will be necessary to produce a graphic line >> and use the tag to reference it. >> >> >> I haven't tried this so you may have to change the colour references to >> numeric values but this hopefully will put you on the correct track >> >> >> On 9 July 2011 08:22, John Allijn wrote: >> >>> Hi, >>> >>> I have a textfield where I want to present a chunk of text, formatted in a >>> few different styles. >>> >>> This is what the text looks like: >>> >>> --- >>> On >>> wrote: >>> >>> --- >>> >>> and this is how I like to represent it: >>> >>> - the text "on timestamp" should be gray and 8 points >>> - the firstname and lastname should be 10 points, black and bold >>> - the message should be 16 points, blue, regular. >>> - put in a divider-line >>> >>> The problem is that the message-text varies in length and that there may be >>> multiple messages in the field. This means (but maybe I'm wrong) that I have >>> to set the TextColor and TextSize properties while populating the field and >>> that I cannot do this afterwards. >>> >>> So: >>> - How can I set the style of the text, put some text in the field and >>> change the style again? >>> - how can I put in a horizontal line that separates the messages? >>> >>> thank you for your thoughts! >>> John. >>> _______________________________________________ >>> use-livecode mailing list >>> use-livecode at lists.runrev.com >>> Please visit this url to subscribe, unsubscribe and manage your >>> subscription preferences: >>> http://lists.runrev.com/mailman/listinfo/use-livecode >>> >> >> >> >> -- >> Regards >> >> >> Ian McKnight >> >> iangmcknight at gmail.com >> ======================= >> _______________________________________________ >> use-livecode mailing list >> use-livecode at lists.runrev.com >> Please visit this url to subscribe, unsubscribe and manage your subscription preferences: >> http://lists.runrev.com/mailman/listinfo/use-livecode > > > _______________________________________________ > use-livecode mailing list > use-livecode at lists.runrev.com > Please visit this url to subscribe, unsubscribe and manage your subscription preferences: > http://lists.runrev.com/mailman/listinfo/use-livecode From pete at mollysrevenge.com Sun Jul 10 15:34:44 2011 From: pete at mollysrevenge.com (Pete) Date: Sun, 10 Jul 2011 12:34:44 -0700 Subject: Missing Menu Items In-Reply-To: <4E19EA65.6010900@hyperactivesw.com> References: <4E19EA65.6010900@hyperactivesw.com> Message-ID: There was a change in the way Mac menus are handled in 4.6 - see page 25 of the release notes at http://www.runrev.com/downloads/livecode/4_6_0/LiveCodeNotes-4_6_0.pdf. I don;t think these changes made it to the User Manaul yet. Pete Molly's Revenge On Sun, Jul 10, 2011 at 11:07 AM, J. Landman Gay wrote: > On 7/10/11 12:08 PM, rand valentine wrote: > >> I wish we had better documentation of all of >> these details in LiveCode, but I'm not holding my breath, having used it >> and >> its ancestors for many years. But with your help I was able to at least >> get >> something working. >> > > Page 259 of the User Guide. ;) > > As long-time users of xtalk, I think a lot of us haven't read that > document. I know I've skipped a lot of it. > > -- > Jacqueline Landman Gay | jacque at hyperactivesw.com > HyperActive Software | http://www.hyperactivesw.com > > > ______________________________**_________________ > use-livecode mailing list > use-livecode at lists.runrev.com > Please visit this url to subscribe, unsubscribe and manage your > subscription preferences: > http://lists.runrev.com/**mailman/listinfo/use-livecode > > From alex at tweedly.net Sun Jul 10 15:36:36 2011 From: alex at tweedly.net (Alex Tweedly) Date: Sun, 10 Jul 2011 20:36:36 +0100 Subject: [ANN] AndreGarzia.Com new web page. In-Reply-To: References: Message-ID: <4E19FF44.3080500@tweedly.net> To echo what others have said .... good looking site with great content. One query - normally I can use cmd-< to go back to previous page (Chrome on Mac), but cannot do that on your site. Is that something you are deliberately preventing ? (and if so, why ?) if not, is it something easily reinstated ? (I really miss it) -- Alex. On 10/07/2011 05:05, Andre Garzia wrote: > Hello Folks, > > It is full of joy and pride that I announce my website redesign here. Those > that accessed my home page recently knew that it was not being updated > often, actually, it was not being updated at all. I always wanted to > recreate everything with LiveCode and now I've did it. I started yesterday, > Friday and after a little more than 24h I've got a brand new website. This > is a testimonial of how easy it is to build web stuff with the new LiveCode > Server. > > This new page will serve as a hub for all my new developments and I plan to > release many many many things during the next days and weeks. I welcome > feedback and comments. > > http://andregarzia.com > > =) > > > For those that want to know, this homepage is driven with LiveCode Server. > It uses RevIgniter Framework. It uses no database at all, the pages are all > text files with HTML and special syntax in it. Keeping it simple allowed me > to deliver a full website in less than 24h. The site is easy to extend and > change. Since I am the developer and the user, I don't need an > administration interface, I can create pages by simply dropping plain text > files in the correct place and tweek things by editing special files. > From mikekann at yahoo.com Sun Jul 10 16:06:12 2011 From: mikekann at yahoo.com (Michael Kann) Date: Sun, 10 Jul 2011 13:06:12 -0700 (PDT) Subject: Windows System Folder Script In-Reply-To: <1E13AAC0-B320-4249-BA6F-D3E39BA1FCBC@sonsothunder.com> Message-ID: <1310328372.72692.YahooMailClassic@web161618.mail.bf1.yahoo.com> Ken, Thanks for all the info. Also thanks for the web presentation a couple of weeks ago. Mike --- On Sun, 7/10/11, Ken Ray wrote: From: Ken Ray Subject: Re: Windows System Folder Script To: "How to use LiveCode" Date: Sunday, July 10, 2011, 2:03 PM On Jul 10, 2011, at 10:30 AM, Michael Kann wrote: > on mouseUp > set hideConsoleWindows to true > put shell(fld 1) into IGNORE > end mouseUp > > ------------------- > -- gotchas > ------------------- > Without the IGNORE you get the msg box popping up. That's because you're using "put"... if you do: ???get shell(fld 1) You don't need to worry about the IGNORE. > You need quotes because of the spaces in some folder names. No spaces around the comma. Not sure why "explorer" is needed, but it seems to be required for the folder names with spaces. The reason you need "explorer" is because that is the command you need to execute on the command line in Windows in order to pass along the name of the folder you want to open. When you use "shell" it acts as if you had popped up the Start menu and entered "cmd" in the Run box and then hit enter to bring up a Windows console window and then typed your command and executed it. This is the reason for setting the hideConsoleWindows to true - it prevents the command window from being displayed. When you pass a parameter with spaces to a command line app, you need to enclose it in quotes. You could just as easily done: ? put shell("explorer" && quote & "shell:Common Start Menu" & quote) into tCommonStartMenuFolder without having to use a field. > Folllowing is a list of the Vista Folders. You could put these in a fld and click on your choice. I've seen a similar list for XP and 7, but I can't seem to find it right now. Perhaps someone can locate it. There's a comparable internal command in LiveCode for geting a 'specialFolderPath'; on Windows you need to provide either a constant or a number that translates to the proper folder path. To see these I have a list on my site: http://www.sonsothunder.com/devres/revolution/tips/file010.htm Ken Ray Sons of Thunder Software, Inc. Email: kray at sonsothunder.com Web Site: http://www.sonsothunder.com/??? _______________________________________________ use-livecode mailing list use-livecode at lists.runrev.com Please visit this url to subscribe, unsubscribe and manage your subscription preferences: http://lists.runrev.com/mailman/listinfo/use-livecode From john.allijn at alice.nl Sun Jul 10 16:16:24 2011 From: john.allijn at alice.nl (John Allijn) Date: Sun, 10 Jul 2011 22:16:24 +0200 Subject: [ANN] AndreGarzia.Com new web page. In-Reply-To: References: Message-ID: Wow, Impressive! On Jul 10, 2011, at 6:05, Andre Garzia wrote: > Hello Folks, > > It is full of joy and pride that I announce my website redesign here. Those > that accessed my home page recently knew that it was not being updated > often, actually, it was not being updated at all. I always wanted to > recreate everything with LiveCode and now I've did it. I started yesterday, > Friday and after a little more than 24h I've got a brand new website. This > is a testimonial of how easy it is to build web stuff with the new LiveCode > Server. > > This new page will serve as a hub for all my new developments and I plan to > release many many many things during the next days and weeks. I welcome > feedback and comments. > > http://andregarzia.com > > =) > > > For those that want to know, this homepage is driven with LiveCode Server. > It uses RevIgniter Framework. It uses no database at all, the pages are all > text files with HTML and special syntax in it. Keeping it simple allowed me > to deliver a full website in less than 24h. The site is easy to extend and > change. Since I am the developer and the user, I don't need an > administration interface, I can create pages by simply dropping plain text > files in the correct place and tweek things by editing special files. > > -- > http://www.andregarzia.com All We Do Is Code. > _______________________________________________ > use-livecode mailing list > use-livecode at lists.runrev.com > Please visit this url to subscribe, unsubscribe and manage your subscription preferences: > http://lists.runrev.com/mailman/listinfo/use-livecode From andre at andregarzia.com Sun Jul 10 16:31:26 2011 From: andre at andregarzia.com (Andre Garzia) Date: Sun, 10 Jul 2011 17:31:26 -0300 Subject: [ANN] AndreGarzia.Com new web page. In-Reply-To: <4E19FF44.3080500@tweedly.net> References: <4E19FF44.3080500@tweedly.net> Message-ID: Alex, This might be some javascript bug here, I will try to fix it. I miss it too. Cheers andre On Sun, Jul 10, 2011 at 4:36 PM, Alex Tweedly wrote: > To echo what others have said .... good looking site with great content. > > One query - normally I can use cmd-< to go back to previous page (Chrome on > Mac), but cannot do that on your site. Is that something you are > deliberately preventing ? (and if so, why ?) > if not, is it something easily reinstated ? (I really miss it) > > -- Alex. > > > On 10/07/2011 05:05, Andre Garzia wrote: > >> Hello Folks, >> >> It is full of joy and pride that I announce my website redesign here. >> Those >> that accessed my home page recently knew that it was not being updated >> often, actually, it was not being updated at all. I always wanted to >> recreate everything with LiveCode and now I've did it. I started >> yesterday, >> Friday and after a little more than 24h I've got a brand new website. This >> is a testimonial of how easy it is to build web stuff with the new >> LiveCode >> Server. >> >> This new page will serve as a hub for all my new developments and I plan >> to >> release many many many things during the next days and weeks. I welcome >> feedback and comments. >> >> http://andregarzia.com >> >> =) >> >> >> For those that want to know, this homepage is driven with LiveCode Server. >> It uses RevIgniter Framework. It uses no database at all, the pages are >> all >> text files with HTML and special syntax in it. Keeping it simple allowed >> me >> to deliver a full website in less than 24h. The site is easy to extend and >> change. Since I am the developer and the user, I don't need an >> administration interface, I can create pages by simply dropping plain text >> files in the correct place and tweek things by editing special files. >> >> > > ______________________________**_________________ > use-livecode mailing list > use-livecode at lists.runrev.com > Please visit this url to subscribe, unsubscribe and manage your > subscription preferences: > http://lists.runrev.com/**mailman/listinfo/use-livecode > -- http://www.andregarzia.com All We Do Is Code. From andre at andregarzia.com Sun Jul 10 16:31:41 2011 From: andre at andregarzia.com (Andre Garzia) Date: Sun, 10 Jul 2011 17:31:41 -0300 Subject: [ANN] AndreGarzia.Com new web page. In-Reply-To: References: <4E19FF44.3080500@tweedly.net> Message-ID: Thanks Guys! -- http://www.andregarzia.com All We Do Is Code. From pete at mollysrevenge.com Sun Jul 10 16:34:07 2011 From: pete at mollysrevenge.com (Pete) Date: Sun, 10 Jul 2011 13:34:07 -0700 Subject: global variable change in substack not available to main stack In-Reply-To: <165694213203.20110710093048@ahsoftware.net> References: <000601cc3ec6$efa35fd0$ceea1f70$@com> <000001cc3ed4$75c3b210$614b1630$@com> <000f01cc3f10$25cc14a0$71643de0$@lexiconbridge.com> <165694213203.20110710093048@ahsoftware.net> Message-ID: Here's another twist on this. The Message Box has the ability to create "global variables" - click the 4th icon from the left in the toolbar at the top left of the message bar. If you define a global variable with a name that starts with a $ sign, it seems to be available from any script without declaring it, thus making it truly global. It also shows up in the Variables tab of the script editor window. If you want a variable that is truly global without having to declare it everywhere, this seems to offer that option. You'd have to find a way to create such variables by script rather than through the message box though - maybe they are created as OS environment variables with a shell command, like the other $ variables? Pete Molly's Revenge On Sun, Jul 10, 2011 at 9:30 AM, Mark Wieder wrote: > Slava- > > Sunday, July 10, 2011, 7:46:21 AM, you wrote: > > > handler was forwarding the openCard message which in turn reinitialized > the > > globals, and thus the new value got lost. My only excuse is that it was 3 > AM > > on a Sunday. > > And this is exactly why I think the use of global variables is a Bad > Idea. Global variables can be set from anywhere, and trying to track > down what happened and why your code isn't working the way you expect > it to is a real pain. I think I can count the number of times I've had > to resort to global variables on the fingers of one hand. I don't > think 3AM Sunday is the issue - it's not particularly anything you did > wrong, it's just that the nature of global variables encourages these > problem scenarios. > > -- > -Mark Wieder > mwieder at ahsoftware.net > > > _______________________________________________ > use-livecode mailing list > use-livecode at lists.runrev.com > Please visit this url to subscribe, unsubscribe and manage your > subscription preferences: > http://lists.runrev.com/mailman/listinfo/use-livecode > > From ray at linkit.com Sun Jul 10 19:13:38 2011 From: ray at linkit.com (Ray Horsley) Date: Sun, 10 Jul 2011 18:13:38 -0500 Subject: Launching Applications on a Remote Server Message-ID: <87B7600A-9E42-493B-94C8-4A6AD3B7B360@LinkIt.Com> Does anybody have any experience with the Launch command to launch an app on a remote server? I imagine this could be done, possible via a php script. Thanks, Ray Horsley LinkIt! Software From scott at tactilemedia.com Sun Jul 10 19:21:47 2011 From: scott at tactilemedia.com (Scott Rossi) Date: Sun, 10 Jul 2011 16:21:47 -0700 Subject: Anyone A Guru With RevBrowser ? In-Reply-To: <000b01cc3f0d$3afb6360$b0f22a20$@lexiconbridge.com> Message-ID: More RevBrowser weirdness... While testing the running of a Web app in a RevBrowser stack on Vista, any page that contains Javascript errors causes an IE script error window to appear... WTF? How can this be disabled or at least handled? Thanks for any advice. Regards, Scott Rossi Creative Director Tactile Media, UX Design From bruceap at comcast.net Sun Jul 10 19:29:09 2011 From: bruceap at comcast.net (Bruce Pokras) Date: Sun, 10 Jul 2011 19:29:09 -0400 Subject: [Android] Why so quiet? Who's developing Android apps? In-Reply-To: <4E188107.7070404@hyperactivesw.com> References: <4E188107.7070404@hyperactivesw.com> Message-ID: <70265F34-9EA0-4FF5-9E91-6F6DC68E5AAB@comcast.net> As soon as it gets web access . . . Bruce Pokras Blazing Dawn Software On Jul 9, 2011, at 12:25 PM, J. Landman Gay wrote: > On 7/9/11 10:16 AM, Roger Eller wrote: >> Just wondering if I am alone... echo... echo... ;-) >> >> What are your plans or aspirations regarding the Android platform and >> LiveCode? Those of you who build for iOS, are you also building your apps >> for Android? Are the capabilities you need present in the latest version of >> LiveCode? >> >> I am still happy that RunRev added Android to our toolbelt, but where is the >> community? How about a show of hands if you are using it, and if not, please >> share with us why. > > I'm becoming very fond of the Android OS, I was a little surprised at myself for that, actually. I'll be continuing to develop for it for sure. But at this particular moment I have a client that wants iOS, so I've had to change direction for a few weeks. When that's done, I'll be back. > > I agree that lack of a smooth scrolling mechanism, as mentioned in another post, is an issue. I've tried several different scripts and none were quite responsive enough. > > -- > Jacqueline Landman Gay | jacque at hyperactivesw.com > HyperActive Software | http://www.hyperactivesw.com > > _______________________________________________ > use-livecode mailing list > use-livecode at lists.runrev.com > Please visit this url to subscribe, unsubscribe and manage your subscription preferences: > http://lists.runrev.com/mailman/listinfo/use-livecode From coiin at verizon.net Sun Jul 10 20:18:10 2011 From: coiin at verizon.net (Colin Holgate) Date: Sun, 10 Jul 2011 20:18:10 -0400 Subject: [OT] what RGB is blue? In-Reply-To: <14032996125.20110702175033@ahsoftware.net> References: <001401cc37d2$d3257910$79706b30$@de> <46EA8E9B-CA91-49AC-A10D-330F10C4F9A1@twft.com> <2FDC8C44-2A4A-495C-9E80-1A076ECC0F22@gmail.com> <14032996125.20110702175033@ahsoftware.net> Message-ID: Here's an interesting optical illusion: http://xfiles.funnygarbage.com/~colinholgate/images/diagonals.jpg Notice how the images seems to be slanted slightly, and even appears to go into the browser window chrome. From m.schonewille at economy-x-talk.com Sun Jul 10 20:26:36 2011 From: m.schonewille at economy-x-talk.com (Mark Schonewille) Date: Mon, 11 Jul 2011 02:26:36 +0200 Subject: [OT] what RGB is blue? In-Reply-To: References: <001401cc37d2$d3257910$79706b30$@de> <46EA8E9B-CA91-49AC-A10D-330F10C4F9A1@twft.com> <2FDC8C44-2A4A-495C-9E80-1A076ECC0F22@gmail.com> <14032996125.20110702175033@ahsoftware.net> Message-ID: <57E564DB-52D8-4FE1-8BB9-680A29113002@economy-x-talk.com> Colin, I only see a diagonal lines in different shades of blue and a peanut with three legs and two eyes. How many images, you say? -- Best regards, Mark Schonewille Economy-x-Talk Consulting and Software Engineering Homepage: http://economy-x-talk.com Twitter: http://twitter.com/xtalkprogrammer KvK: 50277553 New: Download the Installer Maker Plugin 1.6 for LiveCode here http://qery.us/ce On 11 jul 2011, at 02:18, Colin Holgate wrote: > Here's an interesting optical illusion: > > http://xfiles.funnygarbage.com/~colinholgate/images/diagonals.jpg > > Notice how the images seems to be slanted slightly, and even appears to go into the browser window chrome. From pete at mollysrevenge.com Sun Jul 10 20:55:02 2011 From: pete at mollysrevenge.com (Pete) Date: Sun, 10 Jul 2011 17:55:02 -0700 Subject: Resizing the objects in a group Message-ID: According to the dictionary: "If the *lockLocation* of a group is false, the group is automatically resized when the objects in it are moved, resized, hidden, or shown." I have a group that has just one control in it - a datagrid - and its lockloc is false. If I resize the datagrid by dragging its handles, the group does not resize and the parts of the datagrid that are outside the group are no longer visible. With some experimentation, this seems to happen with any group that has only one control in it, no matter what type of control it is. I know it's not normal to have a group with just one object in it but any ideas why this might be happening? Pete Molly's Revenge From roger.e.eller at sealedair.com Sun Jul 10 21:03:23 2011 From: roger.e.eller at sealedair.com (Roger Eller) Date: Sun, 10 Jul 2011 21:03:23 -0400 Subject: [Android] Why so quiet? Who's developing Android apps? In-Reply-To: <70265F34-9EA0-4FF5-9E91-6F6DC68E5AAB@comcast.net> References: <4E188107.7070404@hyperactivesw.com> <70265F34-9EA0-4FF5-9E91-6F6DC68E5AAB@comcast.net> Message-ID: Do any of you have a LiveCode stack that simply goes through a list of features available, and validates whether they are present on the current platform or not. I'm thinking of an app that incorporates everything (basic of course) that is a capability on Mac, Lin, or Win, and shows us a list with pass/fail or yes/no. This would be a great tool to test pre-releases of LiveCode without having to find out the hard way that the feature you need isn't there after you've committed yourself to a project. I know we have the release notes, but nothing is better than seeing the feature actually shown by a functional example, and having that basic code handy when the feature actually works would be a great help to using it within your own project. ?Roger From coiin at verizon.net Sun Jul 10 21:49:57 2011 From: coiin at verizon.net (Colin Holgate) Date: Sun, 10 Jul 2011 21:49:57 -0400 Subject: [OT] what RGB is blue? In-Reply-To: <57E564DB-52D8-4FE1-8BB9-680A29113002@economy-x-talk.com> References: <001401cc37d2$d3257910$79706b30$@de> <46EA8E9B-CA91-49AC-A10D-330F10C4F9A1@twft.com> <2FDC8C44-2A4A-495C-9E80-1A076ECC0F22@gmail.com> <14032996125.20110702175033@ahsoftware.net> <57E564DB-52D8-4FE1-8BB9-680A29113002@economy-x-talk.com> Message-ID: <7405B4AB-32CA-4A73-94EA-B89D00EB0A6C@verizon.net> Don't zoom in on the image, just view it at normal size, then you may see what I mean. On Jul 10, 2011, at 8:26 PM, Mark Schonewille wrote: > Colin, > > I only see a diagonal lines in different shades of blue and a peanut with three legs and two eyes. How many images, you say? > > -- > Best regards, > > Mark Schonewille > > Economy-x-Talk Consulting and Software Engineering > Homepage: http://economy-x-talk.com > Twitter: http://twitter.com/xtalkprogrammer > KvK: 50277553 > > New: Download the Installer Maker Plugin 1.6 for LiveCode here http://qery.us/ce > > On 11 jul 2011, at 02:18, Colin Holgate wrote: > >> Here's an interesting optical illusion: >> >> http://xfiles.funnygarbage.com/~colinholgate/images/diagonals.jpg >> >> Notice how the images seems to be slanted slightly, and even appears to go into the browser window chrome. > > > _______________________________________________ > use-livecode mailing list > use-livecode at lists.runrev.com > Please visit this url to subscribe, unsubscribe and manage your subscription preferences: > http://lists.runrev.com/mailman/listinfo/use-livecode From MikeKerner at roadrunner.com Sun Jul 10 21:55:12 2011 From: MikeKerner at roadrunner.com (Mike Kerner) Date: Sun, 10 Jul 2011 21:55:12 -0400 Subject: Pinch Zooming an image in iOS In-Reply-To: References: <8C6388CC-883E-41B7-984E-A59152998BD1@gmail.com> Message-ID: I would not recommend that most people test against iOS 5 yet. It's still only at DP-2. Plus if you aren't an experienced iOS developer there will be a learning curve just figuring out how to install iOS 5, (redacted), and (redacted). -- On the first day, God created the heavens and the Earth On the second day, God created the oceans. On the third day, God put the animals on hold for a few hours, and did a little diving. And God said, "This is good." From david_beck at rotundasoftware.com Sun Jul 10 22:32:16 2011 From: david_beck at rotundasoftware.com (David Beck) Date: Sun, 10 Jul 2011 19:32:16 -0700 Subject: [ANN] AndreGarzia.Com new web page. In-Reply-To: References: Message-ID: <4E1A60B0.5050903@rotundasoftware.com> Looks awesome Andre!! Congrats!! use-livecode-request at lists.runrev.com wrote: > From: Andre Garzia > To: How to use LiveCode, Improvements > to Revolution > Subject: [ANN] AndreGarzia.Com new web page. > Message-ID: > > Content-Type: text/plain; charset=ISO-8859-1 > > Hello Folks, > > It is full of joy and pride that I announce my website redesign here. Those > that accessed my home page recently knew that it was not being updated > often, actually, it was not being updated at all. I always wanted to > recreate everything with LiveCode and now I've did it. I started yesterday, > Friday and after a little more than 24h I've got a brand new website. This > is a testimonial of how easy it is to build web stuff with the new LiveCode > Server. > > This new page will serve as a hub for all my new developments and I plan to > release many many many things during the next days and weeks. I welcome > feedback and comments. > > http://andregarzia.com > > =) > > > For those that want to know, this homepage is driven with LiveCode Server. > It uses RevIgniter Framework. It uses no database at all, the pages are all > text files with HTML and special syntax in it. Keeping it simple allowed me > to deliver a full website in less than 24h. The site is easy to extend and > change. Since I am the developer and the user, I don't need an > administration interface, I can create pages by simply dropping plain text > files in the correct place and tweek things by editing special files. > From andre at andregarzia.com Sun Jul 10 22:47:14 2011 From: andre at andregarzia.com (Andre Garzia) Date: Sun, 10 Jul 2011 23:47:14 -0300 Subject: [ANN] AndreGarzia.Com new web page. In-Reply-To: <4E1A60B0.5050903@rotundasoftware.com> References: <4E1A60B0.5050903@rotundasoftware.com> Message-ID: Thanks!!!! Did you saw your page on the links? =) Hope all is well over there! Cheers andre On Sun, Jul 10, 2011 at 11:32 PM, David Beck wrote: > Looks awesome Andre!! Congrats!! > > use-livecode-request at lists.**runrev.comwrote: > >> From: Andre Garzia >> To: How to use LiveCode>, >> Improvements >> to Revolution >> > >> Subject: [ANN] AndreGarzia.Com new web page. >> Message-ID: >> > k-g at mail.gmail.com> >> Content-Type: text/plain; charset=ISO-8859-1 >> >> >> Hello Folks, >> >> It is full of joy and pride that I announce my website redesign here. >> Those >> that accessed my home page recently knew that it was not being updated >> often, actually, it was not being updated at all. I always wanted to >> recreate everything with LiveCode and now I've did it. I started >> yesterday, >> Friday and after a little more than 24h I've got a brand new website. This >> is a testimonial of how easy it is to build web stuff with the new >> LiveCode >> Server. >> >> This new page will serve as a hub for all my new developments and I plan >> to >> release many many many things during the next days and weeks. I welcome >> feedback and comments. >> >> http://andregarzia.com >> >> =) >> >> >> For those that want to know, this homepage is driven with LiveCode Server. >> It uses RevIgniter Framework. It uses no database at all, the pages are >> all >> text files with HTML and special syntax in it. Keeping it simple allowed >> me >> to deliver a full website in less than 24h. The site is easy to extend and >> change. Since I am the developer and the user, I don't need an >> administration interface, I can create pages by simply dropping plain text >> files in the correct place and tweek things by editing special files. >> >> > ______________________________**_________________ > use-livecode mailing list > use-livecode at lists.runrev.com > Please visit this url to subscribe, unsubscribe and manage your > subscription preferences: > http://lists.runrev.com/**mailman/listinfo/use-livecode > -- http://www.andregarzia.com All We Do Is Code. From chipp at chipp.com Mon Jul 11 00:01:08 2011 From: chipp at chipp.com (Chipp Walters) Date: Sun, 10 Jul 2011 23:01:08 -0500 Subject: Pinch Zooming an image in iOS In-Reply-To: References: <8C6388CC-883E-41B7-984E-A59152998BD1@gmail.com> Message-ID: Thanks Mike for the advice. On Sun, Jul 10, 2011 at 8:55 PM, Mike Kerner wrote: > I would not recommend that most people test against iOS 5 yet. It's still > only at DP-2. Plus if you aren't an experienced iOS developer there will > be > a learning curve just figuring out how to install iOS 5, (redacted), and > (redacted). > > -- > On the first day, God created the heavens and the Earth > On the second day, God created the oceans. > On the third day, God put the animals on hold for a few hours, > and did a little diving. > And God said, "This is good." > _______________________________________________ > use-livecode mailing list > use-livecode at lists.runrev.com > Please visit this url to subscribe, unsubscribe and manage your > subscription preferences: > http://lists.runrev.com/mailman/listinfo/use-livecode > -- Chipp Walters CEO, Shafer Walters Group, Inc. From chipp at chipp.com Mon Jul 11 00:06:00 2011 From: chipp at chipp.com (Chipp Walters) Date: Sun, 10 Jul 2011 23:06:00 -0500 Subject: Anyone A Guru With RevBrowser ? In-Reply-To: References: <000b01cc3f0d$3afb6360$b0f22a20$@lexiconbridge.com> Message-ID: Scott, You might want to make sure you're using an updated version of IE on Windows. On Sun, Jul 10, 2011 at 6:21 PM, Scott Rossi wrote: > More RevBrowser weirdness... > > While testing the running of a Web app in a RevBrowser stack on Vista, any > page that contains Javascript errors causes an IE script error window to > appear... WTF? How can this be disabled or at least handled? > > Thanks for any advice. > > Regards, > > Scott Rossi > Creative Director > Tactile Media, UX Design > > > > _______________________________________________ > use-livecode mailing list > use-livecode at lists.runrev.com > Please visit this url to subscribe, unsubscribe and manage your > subscription preferences: > http://lists.runrev.com/mailman/listinfo/use-livecode > -- Chipp Walters CEO, Shafer Walters Group, Inc. From jacque at hyperactivesw.com Mon Jul 11 00:25:08 2011 From: jacque at hyperactivesw.com (J. Landman Gay) Date: Sun, 10 Jul 2011 23:25:08 -0500 Subject: global variable change in substack not available to main stack In-Reply-To: References: <000601cc3ec6$efa35fd0$ceea1f70$@com> <000001cc3ed4$75c3b210$614b1630$@com> <000f01cc3f10$25cc14a0$71643de0$@lexiconbridge.com> <165694213203.20110710093048@ahsoftware.net> Message-ID: <4E1A7B24.5020809@hyperactivesw.com> On 7/10/11 3:34 PM, Pete wrote: > If you want a variable that is truly global without having to declare it > everywhere, this seems to offer that option. You'd have to find a way to > create such variables by script rather than through the message box though - > maybe they are created as OS environment variables with a shell command, > like the other $ variables? You only have to use it, and it becomes an environment variable. on mouseUp put "testing" into $TESTVAR end mouseUp -- Jacqueline Landman Gay | jacque at hyperactivesw.com HyperActive Software | http://www.hyperactivesw.com From pete at mollysrevenge.com Mon Jul 11 01:30:32 2011 From: pete at mollysrevenge.com (Pete) Date: Sun, 10 Jul 2011 22:30:32 -0700 Subject: global variable change in substack not available to main stack In-Reply-To: <4E1A7B24.5020809@hyperactivesw.com> References: <000601cc3ec6$efa35fd0$ceea1f70$@com> <000001cc3ed4$75c3b210$614b1630$@com> <000f01cc3f10$25cc14a0$71643de0$@lexiconbridge.com> <165694213203.20110710093048@ahsoftware.net> <4E1A7B24.5020809@hyperactivesw.com> Message-ID: Thanks for the info. Pete Molly's Revenge On Sun, Jul 10, 2011 at 9:25 PM, J. Landman Gay wrote: > On 7/10/11 3:34 PM, Pete wrote: > > If you want a variable that is truly global without having to declare it >> everywhere, this seems to offer that option. You'd have to find a way to >> create such variables by script rather than through the message box though >> - >> maybe they are created as OS environment variables with a shell command, >> like the other $ variables? >> > > You only have to use it, and it becomes an environment variable. > > on mouseUp > put "testing" into $TESTVAR > end mouseUp > > -- > Jacqueline Landman Gay | jacque at hyperactivesw.com > HyperActive Software | http://www.hyperactivesw.com > > ______________________________**_________________ > use-livecode mailing list > use-livecode at lists.runrev.com > Please visit this url to subscribe, unsubscribe and manage your > subscription preferences: > http://lists.runrev.com/**mailman/listinfo/use-livecode > > From john.allijn at alice.nl Mon Jul 11 02:04:07 2011 From: john.allijn at alice.nl (John Allijn) Date: Mon, 11 Jul 2011 08:04:07 +0200 Subject: [Android] Why so quiet? Who's developing Android apps? In-Reply-To: References: <4E188107.7070404@hyperactivesw.com> <70265F34-9EA0-4FF5-9E91-6F6DC68E5AAB@comcast.net> Message-ID: <10D8C0E0-5BA2-438F-BA9F-4DC87959B16E@alice.nl> Hi, Last week I released an android version of my iOS app 'READ'. In the summer academy i learned how to handle different screensizes in the code. Knowing that, android was easy. Now one stack runs on iPod, iPhone, Retina, iPad and Android. READ is a very simple app. I'm now working on a messaging application. I design it to be cross platform and only use the functionality that is available on all platforms. To get a more native look and feel I purchased MobGUI. Sure hope that there will be an android-update for that as well :) The messaging app is (for me) quite difficult to write so it will probably cost me a few more months to get it up and running. Here is READ in the android marketplace: https://market.android.com/details?id=com.allijn.read&feature=search_result Developing for android is a nice-to-have. I don't have it myself, nor do i know anyone with an android phone. However, writing the code for your app is the hard part. If you can release it for another platform with only a few minor tweeks and very little cost, you should do it. You double your potential customers... Regards, John Send from my iPad On Jul 11, 2011, at 3:03, Roger Eller wrote: > Do any of you have a LiveCode stack that simply goes through a list of > features available, and validates whether they are present on the current > platform or not. I'm thinking of an app that incorporates everything (basic > of course) that is a capability on Mac, Lin, or Win, and shows us a list > with pass/fail or yes/no. > > This would be a great tool to test pre-releases of LiveCode without having > to find out the hard way that the feature you need isn't there after you've > committed yourself to a project. I know we have the release notes, but > nothing is better than seeing the feature actually shown by a functional > example, and having that basic code handy when the feature actually works > would be a great help to using it within your own project. > > ?Roger > _______________________________________________ > use-livecode mailing list > use-livecode at lists.runrev.com > Please visit this url to subscribe, unsubscribe and manage your subscription preferences: > http://lists.runrev.com/mailman/listinfo/use-livecode From keith.clarke at clarkeandclarke.co.uk Mon Jul 11 02:53:08 2011 From: keith.clarke at clarkeandclarke.co.uk (Keith Clarke) Date: Mon, 11 Jul 2011 07:53:08 +0100 Subject: [OT] what RGB is blue? In-Reply-To: <7405B4AB-32CA-4A73-94EA-B89D00EB0A6C@verizon.net> References: <001401cc37d2$d3257910$79706b30$@de> <46EA8E9B-CA91-49AC-A10D-330F10C4F9A1@twft.com> <2FDC8C44-2A4A-495C-9E80-1A076ECC0F22@gmail.com> <14032996125.20110702175033@ahsoftware.net> <57E564DB-52D8-4FE1-8BB9-680A29113002@economy-x-talk.com> <7405B4AB-32CA-4A73-94EA-B89D00EB0A6C@verizon.net> Message-ID: <74F27E7A-B14A-412B-9D3B-F257436681E9@clarkeandclarke.co.uk> ...I see it Colin - but then, I have astigmatism, where everything viewed is distorted and even with spectacles the brain has an ongoing job to recalibrate the 'seen' against known patterns. Best, Keith.. On 11 Jul 2011, at 02:49, Colin Holgate wrote: > Don't zoom in on the image, just view it at normal size, then you may see what I mean. > On Jul 10, 2011, at 8:26 PM, Mark Schonewille wrote: > >> Colin, >> >> I only see a diagonal lines in different shades of blue and a peanut with three legs and two eyes. How many images, you say? >> >> -- >> Best regards, >> >> Mark Schonewille >> >> Economy-x-Talk Consulting and Software Engineering >> Homepage: http://economy-x-talk.com >> Twitter: http://twitter.com/xtalkprogrammer >> KvK: 50277553 >> >> New: Download the Installer Maker Plugin 1.6 for LiveCode here http://qery.us/ce >> >> On 11 jul 2011, at 02:18, Colin Holgate wrote: >> >>> Here's an interesting optical illusion: >>> >>> http://xfiles.funnygarbage.com/~colinholgate/images/diagonals.jpg >>> >>> Notice how the images seems to be slanted slightly, and even appears to go into the browser window chrome. >> >> >> _______________________________________________ >> use-livecode mailing list >> use-livecode at lists.runrev.com >> Please visit this url to subscribe, unsubscribe and manage your subscription preferences: >> http://lists.runrev.com/mailman/listinfo/use-livecode > > > _______________________________________________ > use-livecode mailing list > use-livecode at lists.runrev.com > Please visit this url to subscribe, unsubscribe and manage your subscription preferences: > http://lists.runrev.com/mailman/listinfo/use-livecode From sundown at pacifier.com Mon Jul 11 04:20:09 2011 From: sundown at pacifier.com (-=>JB<=-) Date: Mon, 11 Jul 2011 01:20:09 -0700 Subject: [ANN] AndreGarzia.Com new web page. In-Reply-To: References: Message-ID: That looks really nice, Andre. Well Done! -=>JB<=- On Jul 9, 2011, at 9:05 PM, Andre Garzia wrote: > Hello Folks, > > It is full of joy and pride that I announce my website redesign here. Those > that accessed my home page recently knew that it was not being updated > often, actually, it was not being updated at all. I always wanted to > recreate everything with LiveCode and now I've did it. I started yesterday, > Friday and after a little more than 24h I've got a brand new website. This > is a testimonial of how easy it is to build web stuff with the new LiveCode > Server. > > This new page will serve as a hub for all my new developments and I plan to > release many many many things during the next days and weeks. I welcome > feedback and comments. > > http://andregarzia.com > > =) > > > For those that want to know, this homepage is driven with LiveCode Server. > It uses RevIgniter Framework. It uses no database at all, the pages are all > text files with HTML and special syntax in it. Keeping it simple allowed me > to deliver a full website in less than 24h. The site is easy to extend and > change. Since I am the developer and the user, I don't need an > administration interface, I can create pages by simply dropping plain text > files in the correct place and tweek things by editing special files. > > -- > http://www.andregarzia.com All We Do Is Code. > _______________________________________________ > use-livecode mailing list > use-livecode at lists.runrev.com > Please visit this url to subscribe, unsubscribe and manage your subscription preferences: > http://lists.runrev.com/mailman/listinfo/use-livecode > From toolbook at kestner.de Mon Jul 11 05:39:09 2011 From: toolbook at kestner.de (Tiemo Hollmann TB) Date: Mon, 11 Jul 2011 11:39:09 +0200 Subject: How to test if an image is empty? Message-ID: <003e01cc3fae$66f78a60$34e69f20$@de> Hello, when I put empty into img "foo" the imagedata of img "foo" is still not empty, there are still any binary data in the image. I can't test if the imagedata of img "foo" is empty" nor can I test if the img "foo" is empty. How can I test if I have put empty into an image? Can I test it binary? Thanks Tiemo From scott at tactilemedia.com Mon Jul 11 05:44:44 2011 From: scott at tactilemedia.com (Scott Rossi) Date: Mon, 11 Jul 2011 02:44:44 -0700 Subject: Anyone A Guru With RevBrowser ? In-Reply-To: Message-ID: Yes, well, that might work for me, but I don't think I can tell the client to force everyone else with a similar setup. The problem has something to do with the way RevBrowser interacts with IE. In IE, Javascript errors are silently indicated via an icon in the status portion of the browser -- perhaps it can't find settings to control this under RevBrowser so it throws up a standard error dialog. Plus, I've tried everything I can think of to exit the browser when clicking a logout link, but it still refuses to shutdown unless clicking a native LiveCode button. Really wish I could find some solutions so I don't have to tell the client to forget it. Oh well. Regards, Scott Rossi Creative Director Tactile Media, UX Design Recently, Chipp Walters wrote: > Scott, > > You might want to make sure you're using an updated version of IE on > Windows. > > On Sun, Jul 10, 2011 at 6:21 PM, Scott Rossi wrote: > >> More RevBrowser weirdness... >> >> While testing the running of a Web app in a RevBrowser stack on Vista, any >> page that contains Javascript errors causes an IE script error window to >> appear... WTF? How can this be disabled or at least handled? >> >> Thanks for any advice. >> >> Regards, >> >> Scott Rossi From m.schonewille at economy-x-talk.com Mon Jul 11 05:47:20 2011 From: m.schonewille at economy-x-talk.com (Mark Schonewille) Date: Mon, 11 Jul 2011 11:47:20 +0200 Subject: How to test if an image is empty? In-Reply-To: <003e01cc3fae$66f78a60$34e69f20$@de> References: <003e01cc3fae$66f78a60$34e69f20$@de> Message-ID: <01CE125B-61DB-48B2-9273-3281A40B1954@economy-x-talk.com> Hi Tiemo, To make an image object really empty, set its text to empty, rather than the imagedata. Now, 'img x is empty' should return true. -- Best regards, Mark Schonewille Economy-x-Talk Consulting and Software Engineering Homepage: http://economy-x-talk.com Twitter: http://twitter.com/xtalkprogrammer KvK: 50277553 New: Download the Installer Maker Plugin 1.6 for LiveCode here http://qery.us/ce On 11 jul 2011, at 11:39, Tiemo Hollmann TB wrote: > Hello, > > when I put empty into img "foo" the imagedata of img "foo" is still not > empty, there are still any binary data in the image. > > I can't test if the imagedata of img "foo" is empty" nor can I test if the > img "foo" is empty. How can I test if I have put empty into an image? Can I > test it binary? > > Thanks > > Tiemo From scott at tactilemedia.com Mon Jul 11 05:48:24 2011 From: scott at tactilemedia.com (Scott Rossi) Date: Mon, 11 Jul 2011 02:48:24 -0700 Subject: How to test if an image is empty? In-Reply-To: <003e01cc3fae$66f78a60$34e69f20$@de> Message-ID: Recently, Tiemo Hollmann TB wrote: > when I put empty into img "foo" the imagedata of img "foo" is still not > empty, there are still any binary data in the image. > > I can't test if the imagedata of img "foo" is empty" nor can I test if the > img "foo" is empty. How can I test if I have put empty into an image? Can I > test it binary? To "truly" empty an image, I set the text property of the image to empty, and also check that property to make sure it's empty. return (the text of img 1 is empty) Regards, Scott Rossi Creative Director Tactile Media, UX Design From dr.alistair at gmail.com Mon Jul 11 05:53:03 2011 From: dr.alistair at gmail.com (planix) Date: Mon, 11 Jul 2011 02:53:03 -0700 (PDT) Subject: getting from exe to Livecode Message-ID: <1310377983813-3659096.post@n4.nabble.com> Hi, I have just discovered, after numerous changes of my operating system (finally settled on Ubuntu) for various reasons, that I have deleted (doh!) my latest version of an application that I was working on. I did do backups and archive and all the rest but these files were worked on in a rush and I obviously forgot to put them where they needed to be for capture (damn and blast). I usually also print out a hard copy of the scripts that I am writing but, wouldn't you know it, I didn't do that this time. I do, however, have a windows exe file of the last version which I (happily enough) emailed to a student to use in his project. I bet you can see where this is going. Can anyone tell me whether there is a straightforward (not necessarily simple) way of reverse engineering the exe file so that I can get at the scripts? cheers Alistair Campbell Townsville -- View this message in context: http://runtime-revolution.278305.n4.nabble.com/getting-from-exe-to-Livecode-tp3659096p3659096.html Sent from the Revolution - User mailing list archive at Nabble.com. From toolbook at kestner.de Mon Jul 11 07:14:23 2011 From: toolbook at kestner.de (Tiemo Hollmann TB) Date: Mon, 11 Jul 2011 13:14:23 +0200 Subject: AW: How to test if an image is empty? In-Reply-To: References: <003e01cc3fae$66f78a60$34e69f20$@de> Message-ID: <004601cc3fbb$b38f50d0$1aadf270$@de> Thanks Mark and Scott, "the text" ... hmm, not the first to think about with an image :) Thanks for your quick response Tiemo > -----Urspr?ngliche Nachricht----- > Von: use-livecode-bounces at lists.runrev.com [mailto:use-livecode- > bounces at lists.runrev.com] Im Auftrag von Scott Rossi > Gesendet: Montag, 11. Juli 2011 11:48 > An: LiveCode Mail List > Betreff: Re: How to test if an image is empty? > > Recently, Tiemo Hollmann TB wrote: > > > when I put empty into img "foo" the imagedata of img "foo" is still not > > empty, there are still any binary data in the image. > > > > I can't test if the imagedata of img "foo" is empty" nor can I test if the > > img "foo" is empty. How can I test if I have put empty into an image? Can I > > test it binary? > > To "truly" empty an image, I set the text property of the image to empty, > and also check that property to make sure it's empty. > > return (the text of img 1 is empty) > > Regards, > > Scott Rossi > Creative Director > Tactile Media, UX Design > > > > _______________________________________________ > use-livecode mailing list > use-livecode at lists.runrev.com > Please visit this url to subscribe, unsubscribe and manage your subscription > preferences: > http://lists.runrev.com/mailman/listinfo/use-livecode From lists at mangomultimedia.com Mon Jul 11 07:14:07 2011 From: lists at mangomultimedia.com (Trevor DeVore) Date: Mon, 11 Jul 2011 07:14:07 -0400 Subject: Question for Trevor DeVore In-Reply-To: References: Message-ID: On Fri, Jul 8, 2011 at 5:40 PM, Pete wrote: > Do you foresee any undesirable side effects from storing other, custom, > column properties within the datagrid in the same way? For example - set > the > mydgColumnProperty["Col1"] of to "xyz" No. Just don't prefix it with "dg". That way there is no chance of a naming conflict going forward. -- Trevor DeVore Blue Mango Learning Systems LiveCode Resources for Developers: http://livecode.bluemangolearning.com From mikekann at yahoo.com Mon Jul 11 08:48:24 2011 From: mikekann at yahoo.com (Michael Kann) Date: Mon, 11 Jul 2011 05:48:24 -0700 (PDT) Subject: getting from exe to Livecode In-Reply-To: <1310377983813-3659096.post@n4.nabble.com> Message-ID: <1310388504.89000.YahooMailClassic@web161614.mail.bf1.yahoo.com> http://www.analogx.com/contents/download/Programming/textscan/Freeware.htm --- On Mon, 7/11/11, planix wrote: From: planix Subject: getting from exe to Livecode To: use-revolution at lists.runrev.com Date: Monday, July 11, 2011, 4:53 AM Hi, I have just discovered, after numerous changes of my operating system (finally settled on Ubuntu) for various reasons, that I have deleted (doh!) my latest version of an application that I was working on. I did do backups and archive and all the rest but these files were worked on in a rush and I obviously forgot to put them where they needed to be for capture (damn and blast). I usually also print out a hard copy of the scripts that I am writing but, wouldn't you know it, I didn't do that this time. I do, however, have a windows exe file of the last version which I (happily enough) emailed to a student to use in his project. I bet you can see where this is going. Can anyone tell me whether there is a straightforward (not necessarily simple) way of reverse engineering the exe file so that I can get at the scripts? cheers Alistair Campbell Townsville -- View this message in context: http://runtime-revolution.278305.n4.nabble.com/getting-from-exe-to-Livecode-tp3659096p3659096.html Sent from the Revolution - User mailing list archive at Nabble.com. _______________________________________________ use-livecode mailing list use-livecode at lists.runrev.com Please visit this url to subscribe, unsubscribe and manage your subscription preferences: http://lists.runrev.com/mailman/listinfo/use-livecode From chipp at altuit.com Mon Jul 11 08:52:47 2011 From: chipp at altuit.com (Chipp Walters) Date: Mon, 11 Jul 2011 07:52:47 -0500 Subject: Anyone A Guru With RevBrowser ? In-Reply-To: References: Message-ID: I take it you've tried "send in time" on your before navigate handler, to give the handler time to finish before trying to shut it down? Chipp Walters CEO, Shafer Walters Group, Inc On Jul 11, 2011, at 4:44 AM, Scott Rossi wrote: > Yes, well, that might work for me, but I don't think I can tell the client > to force everyone else with a similar setup. The problem has something to > do with the way RevBrowser interacts with IE. In IE, Javascript errors are > silently indicated via an icon in the status portion of the browser -- > perhaps it can't find settings to control this under RevBrowser so it throws > up a standard error dialog. > > Plus, I've tried everything I can think of to exit the browser when clicking > a logout link, but it still refuses to shutdown unless clicking a native > LiveCode button. > > Really wish I could find some solutions so I don't have to tell the client > to forget it. Oh well. > > Regards, > > Scott Rossi > Creative Director > Tactile Media, UX Design > > > > > Recently, Chipp Walters wrote: > >> Scott, >> >> You might want to make sure you're using an updated version of IE on >> Windows. >> >> On Sun, Jul 10, 2011 at 6:21 PM, Scott Rossi wrote: >> >>> More RevBrowser weirdness... >>> >>> While testing the running of a Web app in a RevBrowser stack on Vista, any >>> page that contains Javascript errors causes an IE script error window to >>> appear... WTF? How can this be disabled or at least handled? >>> >>> Thanks for any advice. >>> >>> Regards, >>> >>> Scott Rossi > > > > _______________________________________________ > use-livecode mailing list > use-livecode at lists.runrev.com > Please visit this url to subscribe, unsubscribe and manage your subscription preferences: > http://lists.runrev.com/mailman/listinfo/use-livecode From kray at sonsothunder.com Mon Jul 11 09:29:18 2011 From: kray at sonsothunder.com (Ken Ray) Date: Mon, 11 Jul 2011 08:29:18 -0500 Subject: AW: How to test if an image is empty? In-Reply-To: <004601cc3fbb$b38f50d0$1aadf270$@de> References: <003e01cc3fae$66f78a60$34e69f20$@de> <004601cc3fbb$b38f50d0$1aadf270$@de> Message-ID: <71E960B1-1ACD-4600-94F6-4DA4B438B82B@sonsothunder.com> > Thanks Mark and Scott, > "the text" ... hmm, not the first to think about with an image :) If it helps, Tiemo, you can use this: put "" into image 1 and if image 1 is empty then... The "text" property is implied in the same was as for fields ("put field 1 into.." vs. "put the text of field 1 into..."). Here's a good way to think about images: there's a distinction between the actual image binary data and what is used to show the user what the image currently looks like (including scaling, rotation, rendering in various quality levels, the "paintCompression", etc.). The binary data is the image's *contents* (the 'text' of the image in this case), and what is shown to the user is the 'imageData'+'maskData'+'alphaData' (which I'll just call "image data"). If you empty out the image data, the binary data still exists - all you've done is to say (in effect) "don't show this image to the user", or more accurately: "show blank to the user". You can have binary data without image data but not the other way around; once you set the image data of an image, the binary data is *created* to support the image data you set. This can be very useful in keeping a scaled version of an image in a stack without holding on to all the extra "weight" of the original image. So for example, if you imported a 100K image that was 1000 x 1000 pixels and then scaled it down to 100 x 100 (and set the lockLoc to true so it doesn't "pop" back to its original size), the user would see 100 x 100 pixels of image data, but the image would be storing 1000 x 1000 pixels of binary data. However, if you created a new blank 100 x 100 image an then executed: set the imageData of img 2 to the imageData of img 1 set the alphaData of img 2 to the alphaData of img 1 set the maskData of img 2 to the maskData of img 1 the binary data for img 2 would only be what is necessary to support what the user sees (100 x 100 pixels). It would look exactly like image 1, but would be only 1% of the original number of pixels and would only take up 1K instead of 100K. You could then delete image 1 and you'd have exactly what you started with but storing a bunch less space. This is great for working with thumbnails of full-resolution images; of course if you *need* to keep the full-res image around because the image might scale *up* from 100x100 to 1000x1000 (or any size in between) then you want to work with the full-res image and not make a "cheap copy", but you get the idea. I have a very old (but still mostly accurate) primer on imageData, alphaData, and maskData here: http://www.sonsothunder.com/devres/revolution/tips/imag003.htm Hope this helps, Ken Ray Sons of Thunder Software, Inc. Email: kray at sonsothunder.com Web Site: http://www.sonsothunder.com/ > Thanks for your quick response > Tiemo > >> -----Urspr?ngliche Nachricht----- >> Von: use-livecode-bounces at lists.runrev.com [mailto:use-livecode- >> bounces at lists.runrev.com] Im Auftrag von Scott Rossi >> Gesendet: Montag, 11. Juli 2011 11:48 >> An: LiveCode Mail List >> Betreff: Re: How to test if an image is empty? >> >> Recently, Tiemo Hollmann TB wrote: >> >>> when I put empty into img "foo" the imagedata of img "foo" is still not >>> empty, there are still any binary data in the image. >>> >>> I can't test if the imagedata of img "foo" is empty" nor can I test if > the >>> img "foo" is empty. How can I test if I have put empty into an image? > Can I >>> test it binary? >> >> To "truly" empty an image, I set the text property of the image to empty, >> and also check that property to make sure it's empty. >> >> return (the text of img 1 is empty) >> >> Regards, >> >> Scott Rossi >> Creative Director >> Tactile Media, UX Design >> >> >> >> _______________________________________________ >> use-livecode mailing list >> use-livecode at lists.runrev.com >> Please visit this url to subscribe, unsubscribe and manage your > subscription >> preferences: >> http://lists.runrev.com/mailman/listinfo/use-livecode > > > _______________________________________________ > use-livecode mailing list > use-livecode at lists.runrev.com > Please visit this url to subscribe, unsubscribe and manage your subscription preferences: > http://lists.runrev.com/mailman/listinfo/use-livecode From gregory.lypny at videotron.ca Mon Jul 11 09:34:26 2011 From: gregory.lypny at videotron.ca (Gregory Lypny) Date: Mon, 11 Jul 2011 09:34:26 -0400 Subject: Rev Customer Databased Hacked? Message-ID: <1E064547-F17F-4887-92CD-1A73766F0113@videotron.ca> Hello everyone, Have any of you received this message from Heather? Implications? Gregory > Dear Gregory Lypny, > > I need to inform you that over the weekend we experienced an attack on our customer database. Although we caught this very quickly I regret that some information may have been compromised. A small number of accounts were affected, unfortunately yours was one of them. > > The information concerned includes your name, email address, on-rev username and the server you are hosted on. It does not include your password, or any postal address or billing information. This information alone does not represent a security risk. However, if you have any concerns at all that your password for your on-rev account is not secure, you should change it immediately. cPanel offers a secure password generator that includes numbers and punctuation in a random string, we strongly advise you use this service. > > We deeply regret this breach of our security procedures. We felt it important to inform you of it as quickly as possible as a precautionary measure. > > We have already traced and fixed the exploit that made this possible and can assure you that the same error will not happen again in the future. > > > Regards, > > Heather Nagey > Customer Services Manager > http://www.runrev.com/ > LiveCode - Realize fast, compile-free coding From andre.rombauts at gmail.com Mon Jul 11 09:49:45 2011 From: andre.rombauts at gmail.com (Andre Rombauts) Date: Mon, 11 Jul 2011 15:49:45 +0200 Subject: Rev Customer Databased Hacked? In-Reply-To: <1E064547-F17F-4887-92CD-1A73766F0113@videotron.ca> References: <1E064547-F17F-4887-92CD-1A73766F0113@videotron.ca> Message-ID: <1B37C466-3009-4599-9095-1B464480117C@gmail.com> I did too... Le 11 juil. 2011 ? 15:34, Gregory Lypny a ?crit : > Hello everyone, > > Have any of you received this message from Heather? Implications? > > Gregory > > >> Dear Gregory Lypny, >> From rene.micout at numericable.com Mon Jul 11 09:51:18 2011 From: rene.micout at numericable.com (=?iso-8859-1?Q?Ren=E9_Micout?=) Date: Mon, 11 Jul 2011 15:51:18 +0200 Subject: Rev Customer Databased Hacked? In-Reply-To: <1E064547-F17F-4887-92CD-1A73766F0113@videotron.ca> References: <1E064547-F17F-4887-92CD-1A73766F0113@videotron.ca> Message-ID: <202532DA-543B-441C-A4F4-48B1F7D10689@numericable.com> Yes, I changed my password... in case... Le 11 juil. 2011 ? 15:34, Gregory Lypny a ?crit : > Hello everyone, > > Have any of you received this message from Heather? Implications? > > Gregory > > >> Dear Gregory Lypny, >> >> I need to inform you that over the weekend we experienced an attack on our customer database. Although we caught this very quickly I regret that some information may have been compromised. A small number of accounts were affected, unfortunately yours was one of them. >> >> The information concerned includes your name, email address, on-rev username and the server you are hosted on. It does not include your password, or any postal address or billing information. This information alone does not represent a security risk. However, if you have any concerns at all that your password for your on-rev account is not secure, you should change it immediately. cPanel offers a secure password generator that includes numbers and punctuation in a random string, we strongly advise you use this service. >> >> We deeply regret this breach of our security procedures. We felt it important to inform you of it as quickly as possible as a precautionary measure. >> >> We have already traced and fixed the exploit that made this possible and can assure you that the same error will not happen again in the future. >> >> >> Regards, >> >> Heather Nagey >> Customer Services Manager >> http://www.runrev.com/ >> LiveCode - Realize fast, compile-free coding > _______________________________________________ > use-livecode mailing list > use-livecode at lists.runrev.com > Please visit this url to subscribe, unsubscribe and manage your subscription preferences: > http://lists.runrev.com/mailman/listinfo/use-livecode From MikeKerner at roadrunner.com Mon Jul 11 10:08:46 2011 From: MikeKerner at roadrunner.com (Mike Kerner) Date: Mon, 11 Jul 2011 10:08:46 -0400 Subject: Rev Customer Databased Hacked? In-Reply-To: <202532DA-543B-441C-A4F4-48B1F7D10689@numericable.com> References: <1E064547-F17F-4887-92CD-1A73766F0113@videotron.ca> <202532DA-543B-441C-A4F4-48B1F7D10689@numericable.com> Message-ID: Not yet I haven't, which is curious to me. -- On the first day, God created the heavens and the Earth On the second day, God created the oceans. On the third day, God put the animals on hold for a few hours, and did a little diving. And God said, "This is good." From rene.micout at numericable.com Mon Jul 11 10:19:07 2011 From: rene.micout at numericable.com (=?iso-8859-1?Q?Ren=E9_Micout?=) Date: Mon, 11 Jul 2011 16:19:07 +0200 Subject: [ANN] Update 3 custom controls on RevOnLine Message-ID: <82DF1CA7-88E7-4813-A98E-0DEE415CE774@numericable.com> New versions of 3 custom controls on RevOnline : Circular Slider Store: version 1.1 (revision 2) - Replace copy-paste by Drag and Drop. - Remove bug under Windows (0 millisecs) > thank you SparkOut (Simon). Spinner Store: version 1.2 (revision 4) - Changing name: Spinner Store. - Add 12 types of spinner. - Add drag and drop. HUD Panel Factory: version 1.6 (revision 7) simplifying interface and adding drag and drop and direct resizing. Bon souvenir de Paris Ren? From rene.micout at numericable.com Mon Jul 11 10:31:44 2011 From: rene.micout at numericable.com (=?iso-8859-1?Q?Ren=E9_Micout?=) Date: Mon, 11 Jul 2011 16:31:44 +0200 Subject: Mac OS X missing controls in the LiveCode IDE Message-ID: I made an inventory of missing Mac OS X controls, see list below : 1. Round button 2. ? Textured button > http://blog.chipp.com/new-altbutton-control-for-livecode/ (?) 3. ? Recessed button > I have a project on it 4. ? Disclosure button > I have a project on it 5. ? Gradient button > http://www.buttongadget.com (?) 6. ? Rounded rect button > http://blog.chipp.com/new-altbutton-control-for-livecode/ (?) 7. ? Rounded textured button > http://blog.chipp.com/new-altbutton-control-for-livecode/ (?) 8. ? Help button > I have a project on it 9. ? Bevel button > http://blog.chipp.com/new-altbutton-control-for-livecode/ (?) 10. ? Radio button > size mini > I work on it 11. ? Check box > size mini > I work on it 12. ? Stepper > size mini > I work on it 13. Date picker 14. Segmented control 15. ? Search field > http://droptools.sonsothunder.com/all.irev 16. Token field 17. ? Image Well > http://droptools.sonsothunder.com/all.irev 18. Color well 19. ? Slider (vertical and horizontal) > size : small and mini > I work on it 20. ? Circular slider > I have a project on it 21. ? Circular progress indicator > http://revonline2.runrev.com/stack/440/Spinning-wheel 22. Vertical and horizontal split view 23. ? HUD panel > http://revonline2.runrev.com/stack/574/HUD-Panel-Factory 24. Puff of smoke animation 25. Answer font 26. Toolbar and his items (separator toolbar, print toolbar item, show color toolbar item, show font toolbar item, customize toolbar item, search toolbar item). Some have their "custom control"... But not all... Is my list up to date, incomplete or wrong? Thank you by advance... Ren? From mikekann at yahoo.com Mon Jul 11 10:37:55 2011 From: mikekann at yahoo.com (Michael Kann) Date: Mon, 11 Jul 2011 07:37:55 -0700 (PDT) Subject: Rev Customer Databased Hacked? In-Reply-To: Message-ID: <1310395075.84529.YahooMailClassic@web161612.mail.bf1.yahoo.com> Mike, Not everyone's data got hacked. Mike --- On Mon, 7/11/11, Mike Kerner wrote: From: Mike Kerner Subject: Re: Rev Customer Databased Hacked? To: "How to use LiveCode" Date: Monday, July 11, 2011, 9:08 AM Not yet I haven't, which is curious to me. -- On the first day, God created the heavens and the Earth On the second day, God created the oceans. On the third day, God put the animals on hold for a few hours, ???and did a little diving. And God said, "This is good." _______________________________________________ use-livecode mailing list use-livecode at lists.runrev.com Please visit this url to subscribe, unsubscribe and manage your subscription preferences: http://lists.runrev.com/mailman/listinfo/use-livecode From MikeKerner at roadrunner.com Mon Jul 11 10:44:19 2011 From: MikeKerner at roadrunner.com (Mike Kerner) Date: Mon, 11 Jul 2011 10:44:19 -0400 Subject: Rev Customer Databased Hacked? In-Reply-To: <1310395075.84529.YahooMailClassic@web161612.mail.bf1.yahoo.com> References: <1310395075.84529.YahooMailClassic@web161612.mail.bf1.yahoo.com> Message-ID: So that doesn't strike you as curious, Mike? Think about it. It doesn't make you wonder what the structure is if some customers have their records compromised but others do not? -- On the first day, God created the heavens and the Earth On the second day, God created the oceans. On the third day, God put the animals on hold for a few hours, and did a little diving. And God said, "This is good." From mikekann at yahoo.com Mon Jul 11 11:03:44 2011 From: mikekann at yahoo.com (Michael Kann) Date: Mon, 11 Jul 2011 08:03:44 -0700 (PDT) Subject: Rev Customer Databased Hacked? In-Reply-To: Message-ID: <1310396624.47204.YahooMailClassic@web161603.mail.bf1.yahoo.com> Mike, You make a good point. I bought the lifetime membership so perhaps my info is in a different pile than yours. I was more concerned that any data was obtainable from the outside at all. Mike --- On Mon, 7/11/11, Mike Kerner wrote: From: Mike Kerner Subject: Re: Rev Customer Databased Hacked? To: "How to use LiveCode" Date: Monday, July 11, 2011, 9:44 AM So that doesn't strike you as curious, Mike?? Think about it.? It doesn't make you wonder what the structure is if some customers have their records compromised but others do not? -- On the first day, God created the heavens and the Earth On the second day, God created the oceans. On the third day, God put the animals on hold for a few hours, ???and did a little diving. And God said, "This is good." _______________________________________________ use-livecode mailing list use-livecode at lists.runrev.com Please visit this url to subscribe, unsubscribe and manage your subscription preferences: http://lists.runrev.com/mailman/listinfo/use-livecode From richmondmathewson at gmail.com Mon Jul 11 11:07:44 2011 From: richmondmathewson at gmail.com (Richmond Mathewson) Date: Mon, 11 Jul 2011 18:07:44 +0300 Subject: Rev Customer Databased Hacked? In-Reply-To: References: <1310395075.84529.YahooMailClassic@web161612.mail.bf1.yahoo.com> Message-ID: <4E1B11C0.5010701@gmail.com> On 07/11/2011 05:44 PM, Mike Kerner wrote: > So that doesn't strike you as curious, Mike? Think about it. It doesn't > make you wonder what the structure is if some customers have their records > compromised but others do not? > > I wonder if RunRev don't have 2 databases (say, an older one with longer-term customers, and a newer one with newer customers), and 1 got hacked ??? From form at nonsanity.com Mon Jul 11 11:25:38 2011 From: form at nonsanity.com (Nonsanity) Date: Mon, 11 Jul 2011 11:25:38 -0400 Subject: Rev Customer Databased Hacked? In-Reply-To: References: <1310395075.84529.YahooMailClassic@web161612.mail.bf1.yahoo.com> Message-ID: I think it's more likely that someone found a way to query user information from the database, and started doing so, probably with some automation. The culprit was noticed by the unusual activity before they hit on every record in the DB. It's highly unlikely that someone managed to just download the whole DB. ~ Chris Innanen ~ Nonsanity On Mon, Jul 11, 2011 at 10:44 AM, Mike Kerner wrote: > So that doesn't strike you as curious, Mike? Think about it. It doesn't > make you wonder what the structure is if some customers have their records > compromised but others do not? > > From lists at mangomultimedia.com Mon Jul 11 11:28:09 2011 From: lists at mangomultimedia.com (Trevor DeVore) Date: Mon, 11 Jul 2011 11:28:09 -0400 Subject: Rev Customer Databased Hacked? In-Reply-To: References: <1310395075.84529.YahooMailClassic@web161612.mail.bf1.yahoo.com> Message-ID: On Mon, Jul 11, 2011 at 10:44 AM, Mike Kerner wrote: > So that doesn't strike you as curious, Mike? Think about it. It doesn't > make you wonder what the structure is if some customers have their records > compromised but others do not? The original poster did not include the subject of the email that RunRev sent out. "Important information about your on-Rev hosting account" Only user information for on-Rev accounts was obtained. -- Trevor DeVore Blue Mango Learning Systems LiveCode Resources for Developers: http://livecode.bluemangolearning.com From massung at gmail.com Mon Jul 11 11:41:54 2011 From: massung at gmail.com (Jeff Massung) Date: Mon, 11 Jul 2011 09:41:54 -0600 Subject: Rev Customer Databased Hacked? In-Reply-To: <1E064547-F17F-4887-92CD-1A73766F0113@videotron.ca> References: <1E064547-F17F-4887-92CD-1A73766F0113@videotron.ca> Message-ID: I would just like to say that I haven't been an On-Rev customer for over a year now, and I want to thank the Rev team for still including me in this email (so, yes, I got one, too). Jeff M. From jimaultwins at yahoo.com Mon Jul 11 11:42:55 2011 From: jimaultwins at yahoo.com (Jim Ault) Date: Mon, 11 Jul 2011 08:42:55 -0700 Subject: Rev Customer Databased Hacked? In-Reply-To: References: <1310395075.84529.YahooMailClassic@web161612.mail.bf1.yahoo.com> Message-ID: and now my password is 127.5 characters long... OK, I am not that paranoid, but I did make a change for On-Rev (and the On-Rev forums just in case, even though it is a separate entity. No need to have spam hit that server require the moderators to do extra work.) Jim Ault Las Vegas On Jul 11, 2011, at 8:28 AM, Trevor DeVore wrote: > On Mon, Jul 11, 2011 at 10:44 AM, Mike Kerner >wrote: >> So that doesn't strike you as curious, Mike? Think about it. It >> doesn't >> make you wonder what the structure is if some customers have their >> records >> compromised but others do not? > > The original poster did not include the subject of the email that > RunRev > sent out. > "Important information about your on-Rev hosting account" > Only user information for on-Rev accounts was obtained. > From ambassador at fourthworld.com Mon Jul 11 11:48:43 2011 From: ambassador at fourthworld.com (Richard Gaskin) Date: Mon, 11 Jul 2011 08:48:43 -0700 Subject: Rev Customer Databased Hacked? In-Reply-To: <1310396624.47204.YahooMailClassic@web161603.mail.bf1.yahoo.com> References: <1310396624.47204.YahooMailClassic@web161603.mail.bf1.yahoo.com> Message-ID: <4E1B1B5B.8020109@fourthworld.com> Michael Kann wrote: > I was more concerned that any data was obtainable from the outside > at all. A concern, but not a surprise, given the range of software components that comprise modern web apps. I think it speaks well of the RunRev that passwords weren't compromised. Oddly enough I was writing this morning's post to the LiveCode Journal blog about security when this thread showed up here. The post includes a couple helpful links, the best one being to a recent report of the 25 Most Dangerous Software Errors: -- Richard Gaskin Fourth World LiveCode training and consulting: http://www.fourthworld.com Webzine for LiveCode developers: http://www.LiveCodeJournal.com LiveCode Journal blog: http://LiveCodejournal.com/blog.irv From chipp at chipp.com Mon Jul 11 12:05:25 2011 From: chipp at chipp.com (Chipp Walters) Date: Mon, 11 Jul 2011 11:05:25 -0500 Subject: Rev Customer Databased Hacked? In-Reply-To: <1E064547-F17F-4887-92CD-1A73766F0113@videotron.ca> References: <1E064547-F17F-4887-92CD-1A73766F0113@videotron.ca> Message-ID: FYI, I sent an email back to Heather telling her if she receives any request about my account coming from someone claiming to be me, please do not hit the "reply" button, but rather email me directly with requested data. I also mentioned she might want to ask me a personal question only she and I know the answer to. From jacque at hyperactivesw.com Mon Jul 11 12:11:20 2011 From: jacque at hyperactivesw.com (J. Landman Gay) Date: Mon, 11 Jul 2011 11:11:20 -0500 Subject: Rev Customer Databased Hacked? In-Reply-To: References: <1310395075.84529.YahooMailClassic@web161612.mail.bf1.yahoo.com> Message-ID: <4E1B20A8.3000804@hyperactivesw.com> On 7/11/11 9:44 AM, Mike Kerner wrote: > So that doesn't strike you as curious, Mike? Think about it. It doesn't > make you wonder what the structure is if some customers have their records > compromised but others do not? It may be significant that not all accounts are on the same server. -- Jacqueline Landman Gay | jacque at hyperactivesw.com HyperActive Software | http://www.hyperactivesw.com From bobs at twft.com Mon Jul 11 12:25:46 2011 From: bobs at twft.com (Bob Sneidar) Date: Mon, 11 Jul 2011 09:25:46 -0700 Subject: Rev Customer Databased Hacked? In-Reply-To: <1E064547-F17F-4887-92CD-1A73766F0113@videotron.ca> References: <1E064547-F17F-4887-92CD-1A73766F0113@videotron.ca> Message-ID: <2EF08577-CEBA-4EF4-83BC-3E55B42D20C7@twft.com> I have received it. It's probably legit. I don't see any links to a site to authenticate, so what would anyone gain by telling you to change your password? Bob On Jul 11, 2011, at 6:34 AM, Gregory Lypny wrote: > Hello everyone, > > Have any of you received this message from Heather? Implications? > > Gregory > > >> Dear Gregory Lypny, >> >> I need to inform you that over the weekend we experienced an attack on our customer database. Although we caught this very quickly I regret that some information may have been compromised. A small number of accounts were affected, unfortunately yours was one of them. >> >> The information concerned includes your name, email address, on-rev username and the server you are hosted on. It does not include your password, or any postal address or billing information. This information alone does not represent a security risk. However, if you have any concerns at all that your password for your on-rev account is not secure, you should change it immediately. cPanel offers a secure password generator that includes numbers and punctuation in a random string, we strongly advise you use this service. >> >> We deeply regret this breach of our security procedures. We felt it important to inform you of it as quickly as possible as a precautionary measure. >> >> We have already traced and fixed the exploit that made this possible and can assure you that the same error will not happen again in the future. >> >> >> Regards, >> >> Heather Nagey >> Customer Services Manager >> http://www.runrev.com/ >> LiveCode - Realize fast, compile-free coding > _______________________________________________ > use-livecode mailing list > use-livecode at lists.runrev.com > Please visit this url to subscribe, unsubscribe and manage your subscription preferences: > http://lists.runrev.com/mailman/listinfo/use-livecode From mpetrides at earthlink.net Mon Jul 11 12:42:41 2011 From: mpetrides at earthlink.net (Marian Petrides) Date: Mon, 11 Jul 2011 11:42:41 -0500 Subject: Rev Customer Databased Hacked? In-Reply-To: <2EF08577-CEBA-4EF4-83BC-3E55B42D20C7@twft.com> References: <1E064547-F17F-4887-92CD-1A73766F0113@videotron.ca> <2EF08577-CEBA-4EF4-83BC-3E55B42D20C7@twft.com> Message-ID: <80A09283-D8D8-4599-8057-735A02C21137@earthlink.net> I am an On-Rev lifetime subscriber but don't recall getting this message. So it must be something else, I guess. On Jul 11, 2011, at 11:25 AM, Bob Sneidar wrote: > I have received it. It's probably legit. I don't see any links to a site to authenticate, so what would anyone gain by telling you to change your password? > > Bob > > > On Jul 11, 2011, at 6:34 AM, Gregory Lypny wrote: > >> Hello everyone, >> >> Have any of you received this message from Heather? Implications? >> >> Gregory >> >> >>> Dear Gregory Lypny, >>> >>> I need to inform you that over the weekend we experienced an attack on our customer database. Although we caught this very quickly I regret that some information may have been compromised. A small number of accounts were affected, unfortunately yours was one of them. >>> >>> The information concerned includes your name, email address, on-rev username and the server you are hosted on. It does not include your password, or any postal address or billing information. This information alone does not represent a security risk. However, if you have any concerns at all that your password for your on-rev account is not secure, you should change it immediately. cPanel offers a secure password generator that includes numbers and punctuation in a random string, we strongly advise you use this service. >>> >>> We deeply regret this breach of our security procedures. We felt it important to inform you of it as quickly as possible as a precautionary measure. >>> >>> We have already traced and fixed the exploit that made this possible and can assure you that the same error will not happen again in the future. >>> >>> >>> Regards, >>> >>> Heather Nagey >>> Customer Services Manager >>> http://www.runrev.com/ >>> LiveCode - Realize fast, compile-free coding >> _______________________________________________ >> use-livecode mailing list >> use-livecode at lists.runrev.com >> Please visit this url to subscribe, unsubscribe and manage your subscription preferences: >> http://lists.runrev.com/mailman/listinfo/use-livecode > > > _______________________________________________ > use-livecode mailing list > use-livecode at lists.runrev.com > Please visit this url to subscribe, unsubscribe and manage your subscription preferences: > http://lists.runrev.com/mailman/listinfo/use-livecode From stephenREVOLUTION2 at barncard.com Mon Jul 11 12:46:41 2011 From: stephenREVOLUTION2 at barncard.com (stephen barncard) Date: Mon, 11 Jul 2011 09:46:41 -0700 Subject: Mac OS X missing controls in the LiveCode IDE In-Reply-To: References: Message-ID: Perhaps with the new externals interface we can get access to these directly from the OS. I"d like to use the system font and size menu panel like the one that's used almost everywhere like TextEdit and TexEdit. On 11 July 2011 07:31, Ren? Micout wrote: > I made an inventory of missing Mac OS X controls, see list below : > 1. Round button > 2. ? Textured button > > http://blog.chipp.com/new-altbutton-control-for-livecode/ (?) > 3. ? Recessed button > I have a project on it > 4. ? Disclosure button > I have a project on it > 5. ? Gradient button > http://www.buttongadget.com (?) > 6. ? Rounded rect button > > http://blog.chipp.com/new-altbutton-control-for-livecode/ (?) > 7. ? Rounded textured button > > http://blog.chipp.com/new-altbutton-control-for-livecode/ (?) > 8. ? Help button > I have a project on it > 9. ? Bevel button > > http://blog.chipp.com/new-altbutton-control-for-livecode/ (?) > 10. ? Radio button > size mini > I work on it > 11. ? Check box > size mini > I work on it > 12. ? Stepper > size mini > I work on it > 13. Date picker > 14. Segmented control > 15. ? Search field > http://droptools.sonsothunder.com/all.irev > 16. Token field > 17. ? Image Well > http://droptools.sonsothunder.com/all.irev > 18. Color well > 19. ? Slider (vertical and horizontal) > size : small and mini > I work on > it > 20. ? Circular slider > I have a project on it > 21. ? Circular progress indicator > > http://revonline2.runrev.com/stack/440/Spinning-wheel > 22. Vertical and horizontal split view > 23. ? HUD panel > http://revonline2.runrev.com/stack/574/HUD-Panel-Factory > 24. Puff of smoke animation > 25. Answer font > 26. Toolbar and his items (separator toolbar, print toolbar item, show > color toolbar item, show font toolbar item, customize toolbar item, search > toolbar item). > Some have their "custom control"... But not all... Is my list up to date, > incomplete or wrong? > Thank you by advance... > Ren? > _______________________________________________ > use-livecode mailing list > use-livecode at lists.runrev.com > Please visit this url to subscribe, unsubscribe and manage your > subscription preferences: > http://lists.runrev.com/mailman/listinfo/use-livecode -- Stephen Barncard San Francisco Ca. USA more about sqb From richmondmathewson at gmail.com Mon Jul 11 12:50:28 2011 From: richmondmathewson at gmail.com (Richmond Mathewson) Date: Mon, 11 Jul 2011 19:50:28 +0300 Subject: Rev Customer Databased Hacked? In-Reply-To: <80A09283-D8D8-4599-8057-735A02C21137@earthlink.net> References: <1E064547-F17F-4887-92CD-1A73766F0113@videotron.ca> <2EF08577-CEBA-4EF4-83BC-3E55B42D20C7@twft.com> <80A09283-D8D8-4599-8057-735A02C21137@earthlink.net> Message-ID: <4E1B29D4.4010509@gmail.com> I feel deprived, unloved and generally rejected because I didn't receive the "Hacked" e-mail. Har, har, har. Bl**dy glad I didn't get it; but reading this thread it does have a feling of bruised egos who weere not included. Now I remember a lecture on that psychological phemenon at Durham years ago . . . :) From andre at andregarzia.com Mon Jul 11 12:56:43 2011 From: andre at andregarzia.com (Andre Garzia) Date: Mon, 11 Jul 2011 13:56:43 -0300 Subject: Rev Customer Databased Hacked? In-Reply-To: <80A09283-D8D8-4599-8057-735A02C21137@earthlink.net> References: <1E064547-F17F-4887-92CD-1A73766F0113@videotron.ca> <2EF08577-CEBA-4EF4-83BC-3E55B42D20C7@twft.com> <80A09283-D8D8-4599-8057-735A02C21137@earthlink.net> Message-ID: Folks, I am speculating here but if the attacker just go some of the accounts then it is possible that the attacker hacked into some of the on-rev servers but not all of them, then, just the users on those machines were compromissed. I did not change my password, I am still deciding if I will do it or not. Again, if you have a need for utmost security, you should not be on shared hosting, you need to me on your own box on co-location with security experts on payroll. If you are on shared hosts, then, by default, you are subject to such attacks. Cheers andre PS: I have a lifetime on-rev account and am happy with it. I also have a VPS (it is as good as I can pay) for more sensitive stuff and I have one or two linodes. On Mon, Jul 11, 2011 at 1:42 PM, Marian Petrides wrote: > I am an On-Rev lifetime subscriber but don't recall getting this message. > So it must be something else, I guess. > > On Jul 11, 2011, at 11:25 AM, Bob Sneidar wrote: > > > I have received it. It's probably legit. I don't see any links to a site > to authenticate, so what would anyone gain by telling you to change your > password? > > > > Bob > > > > > > On Jul 11, 2011, at 6:34 AM, Gregory Lypny wrote: > > > >> Hello everyone, > >> > >> Have any of you received this message from Heather? Implications? > >> > >> Gregory > >> > >> > >>> Dear Gregory Lypny, > >>> > >>> I need to inform you that over the weekend we experienced an attack on > our customer database. Although we caught this very quickly I regret that > some information may have been compromised. A small number of accounts were > affected, unfortunately yours was one of them. > >>> > >>> The information concerned includes your name, email address, on-rev > username and the server you are hosted on. It does not include your > password, or any postal address or billing information. This information > alone does not represent a security risk. However, if you have any concerns > at all that your password for your on-rev account is not secure, you should > change it immediately. cPanel offers a secure password generator that > includes numbers and punctuation in a random string, we strongly advise you > use this service. > >>> > >>> We deeply regret this breach of our security procedures. We felt it > important to inform you of it as quickly as possible as a precautionary > measure. > >>> > >>> We have already traced and fixed the exploit that made this possible > and can assure you that the same error will not happen again in the future. > >>> > >>> > >>> Regards, > >>> > >>> Heather Nagey > >>> Customer Services Manager > >>> http://www.runrev.com/ > >>> LiveCode - Realize fast, compile-free coding > >> _______________________________________________ > >> use-livecode mailing list > >> use-livecode at lists.runrev.com > >> Please visit this url to subscribe, unsubscribe and manage your > subscription preferences: > >> http://lists.runrev.com/mailman/listinfo/use-livecode > > > > > > _______________________________________________ > > use-livecode mailing list > > use-livecode at lists.runrev.com > > Please visit this url to subscribe, unsubscribe and manage your > subscription preferences: > > http://lists.runrev.com/mailman/listinfo/use-livecode > > > _______________________________________________ > use-livecode mailing list > use-livecode at lists.runrev.com > Please visit this url to subscribe, unsubscribe and manage your > subscription preferences: > http://lists.runrev.com/mailman/listinfo/use-livecode > -- http://www.andregarzia.com All We Do Is Code. From rene.micout at numericable.com Mon Jul 11 13:06:54 2011 From: rene.micout at numericable.com (=?iso-8859-1?Q?Ren=E9_Micout?=) Date: Mon, 11 Jul 2011 19:06:54 +0200 Subject: Mac OS X missing controls in the LiveCode IDE In-Reply-To: References: Message-ID: <32257BFE-617E-4FC9-ABD7-92D81E9A60E5@numericable.com> Le 11 juil. 2011 ? 18:46, stephen barncard a ?crit : > Perhaps with the new externals interface we can get access to these directly > from the OS. Thank you Stephen... I am not informed about "new externals interface"... What is it? and where can I find information about that? > I"d like to use the system font and size menu panel like the one that's used > almost everywhere like TextEdit and TexEdit. YES !!! From jacque at hyperactivesw.com Mon Jul 11 13:13:08 2011 From: jacque at hyperactivesw.com (J. Landman Gay) Date: Mon, 11 Jul 2011 12:13:08 -0500 Subject: getting from exe to Livecode In-Reply-To: <1310377983813-3659096.post@n4.nabble.com> References: <1310377983813-3659096.post@n4.nabble.com> Message-ID: <4E1B2F24.3070306@hyperactivesw.com> On 7/11/11 4:53 AM, planix wrote: > I do, however, have a windows exe file of the last version which I (happily > enough) emailed to a student to use in his project. I bet you can see where > this is going. Can anyone tell me whether there is a straightforward (not > necessarily simple) way of reverse engineering the exe file so that I can > get at the scripts? If you did not password-protect your stack before building the exe, then the scripts will be in plain text in the executable. Open it in a text editor and they will be visible, in between all the binary code. If you did password-protect the stack then you can't get access or extract the stack. To RR's credit, the ability to do that was removed some years ago. But you can write to support to see if they can help you. It may cost a fee. -- Jacqueline Landman Gay | jacque at hyperactivesw.com HyperActive Software | http://www.hyperactivesw.com From mpetrides at earthlink.net Mon Jul 11 13:13:32 2011 From: mpetrides at earthlink.net (Marian Petrides) Date: Mon, 11 Jul 2011 12:13:32 -0500 Subject: Rev Customer Databased Hacked? In-Reply-To: <4E1B29D4.4010509@gmail.com> References: <1E064547-F17F-4887-92CD-1A73766F0113@videotron.ca> <2EF08577-CEBA-4EF4-83BC-3E55B42D20C7@twft.com> <80A09283-D8D8-4599-8057-735A02C21137@earthlink.net> <4E1B29D4.4010509@gmail.com> Message-ID: <95BCF900-4221-40B1-B145-19043F33BDF6@earthlink.net> Not bruised egos, Richmond--at least not in my case. Simply concern about whether I was one of the intended recipients of the email but did not receive it for sometime. I would rather NOT change my password but will if I have to. On Jul 11, 2011, at 11:50 AM, Richmond Mathewson wrote: > I feel deprived, unloved and generally rejected because I didn't receive > the "Hacked" e-mail. > > Har, har, har. > > Bl**dy glad I didn't get it; but reading this thread it does have a feling of > bruised egos who weere not included. > > Now I remember a lecture on that psychological phemenon at Durham > years ago . . . :) > > _______________________________________________ > use-livecode mailing list > use-livecode at lists.runrev.com > Please visit this url to subscribe, unsubscribe and manage your subscription preferences: > http://lists.runrev.com/mailman/listinfo/use-livecode From richmondmathewson at gmail.com Mon Jul 11 13:27:20 2011 From: richmondmathewson at gmail.com (Richmond Mathewson) Date: Mon, 11 Jul 2011 20:27:20 +0300 Subject: Rev Customer Databased Hacked? In-Reply-To: <95BCF900-4221-40B1-B145-19043F33BDF6@earthlink.net> References: <1E064547-F17F-4887-92CD-1A73766F0113@videotron.ca> <2EF08577-CEBA-4EF4-83BC-3E55B42D20C7@twft.com> <80A09283-D8D8-4599-8057-735A02C21137@earthlink.net> <4E1B29D4.4010509@gmail.com> <95BCF900-4221-40B1-B145-19043F33BDF6@earthlink.net> Message-ID: <4E1B3278.4010601@gmail.com> On 07/11/2011 08:13 PM, Marian Petrides wrote: > Not bruised egos, Richmond--at least not in my case. Simply concern about whether I was one of the intended recipients of the email but did not receive it for sometime. I would rather NOT change my password but will if I have to. I wonder what the hackers could want with the information? Maybe just pure malice. > On Jul 11, 2011, at 11:50 AM, Richmond Mathewson wrote: > >> I feel deprived, unloved and generally rejected because I didn't receive >> the "Hacked" e-mail. >> >> Har, har, har. >> >> Bl**dy glad I didn't get it; but reading this thread it does have a feling of >> bruised egos who weere not included. >> >> Now I remember a lecture on that psychological phemenon at Durham >> years ago . . . :) >> >> _______________________________________________ >> use-livecode mailing list >> use-livecode at lists.runrev.com >> Please visit this url to subscribe, unsubscribe and manage your subscription preferences: >> http://lists.runrev.com/mailman/listinfo/use-livecode > > _______________________________________________ > use-livecode mailing list > use-livecode at lists.runrev.com > Please visit this url to subscribe, unsubscribe and manage your subscription preferences: > http://lists.runrev.com/mailman/listinfo/use-livecode From jacque at hyperactivesw.com Mon Jul 11 13:31:28 2011 From: jacque at hyperactivesw.com (J. Landman Gay) Date: Mon, 11 Jul 2011 12:31:28 -0500 Subject: Rev Customer Databased Hacked? In-Reply-To: <95BCF900-4221-40B1-B145-19043F33BDF6@earthlink.net> References: <1E064547-F17F-4887-92CD-1A73766F0113@videotron.ca> <2EF08577-CEBA-4EF4-83BC-3E55B42D20C7@twft.com> <80A09283-D8D8-4599-8057-735A02C21137@earthlink.net> <4E1B29D4.4010509@gmail.com> <95BCF900-4221-40B1-B145-19043F33BDF6@earthlink.net> Message-ID: <4E1B3370.8040708@hyperactivesw.com> On 7/11/11 12:13 PM, Marian Petrides wrote: > Not bruised egos, Richmond--at least not in my case. Simply concern > about whether I was one of the intended recipients of the email but > did not receive it for sometime. I would rather NOT change my > password but will if I have to. The info was sent out about 11 AM UK time. I think if you haven't received it by now, you were not affected. Passwords were not obtained, so if the affected people are using good passwords, I don't see any reason to change them. I'm not changing mine. -- Jacqueline Landman Gay | jacque at hyperactivesw.com HyperActive Software | http://www.hyperactivesw.com From jhj at jhj.com Mon Jul 11 13:48:04 2011 From: jhj at jhj.com (Jerry J) Date: Mon, 11 Jul 2011 10:48:04 -0700 Subject: Rev Customer Databased Hacked? In-Reply-To: <4E1B3370.8040708@hyperactivesw.com> References: <1E064547-F17F-4887-92CD-1A73766F0113@videotron.ca> <2EF08577-CEBA-4EF4-83BC-3E55B42D20C7@twft.com> <80A09283-D8D8-4599-8057-735A02C21137@earthlink.net> <4E1B29D4.4010509@gmail.com> <95BCF900-4221-40B1-B145-19043F33BDF6@earthlink.net> <4E1B3370.8040708@hyperactivesw.com> Message-ID: <270FFB88-A378-4FBC-957D-73AC8686C698@jhj.com> On Jul 11, 2011, at 10:31 AM, J. Landman Gay wrote: > The info was sent out about 11 AM UK time. I think if you haven't received it by now, you were not affected. Passwords were not obtained, so if the affected people are using good passwords, I don't see any reason to change them. I'm not changing mine. I'm not bothering to change my password either. As for the info that was compromised, the only bit thats not been posted here before is my on-rev username, and I bet you can look at my email address and guess it on the first try! --jhj From john at splash21.com Mon Jul 11 13:51:17 2011 From: john at splash21.com (John Craig) Date: Mon, 11 Jul 2011 18:51:17 +0100 Subject: [ANN] MobGUI V0.12 : tab bar and buttons Message-ID: <4E1B3815.20708@splash21.com> Hi, all. I've just updated the plugin and uploaded a new video to http://mobgui.com The test stack created in the video is available in the RunRev MobGUI forum. I created a few (basic) icons with GIMP (gimp.org) for the demo, but was thinking of compiling a list of the most useful general purpose icons. I have the glyphish icons, but can't give them away with the plugin and they don't come with inactive and active states. I know a graphic designer over here who'll give me a good rate for spending a few hours creating a small icon set that I can give away with the plugin, so gather your thoughts. It means you'd have a set of icons that you knew were definitely OK for use in your apps without royalties or affiliation. Don't send me lists of icons just yet - let me organize something online where we can try to determine what the most popular would be and I'll try and move it forward. That is unless everyone already knows of good resources, are happy using them and aren't really interested in this idea! JC From mikekann at yahoo.com Mon Jul 11 14:03:41 2011 From: mikekann at yahoo.com (Michael Kann) Date: Mon, 11 Jul 2011 11:03:41 -0700 (PDT) Subject: getting from exe to Livecode In-Reply-To: <82DF1CA7-88E7-4813-A98E-0DEE415CE774@numericable.com> Message-ID: <1310407421.36078.YahooMailClassic@web161602.mail.bf1.yahoo.com> Alistair, I checked out the program I recommended and it doesn't work on LiveCode standalones with passwords. As usual, Jacqueline is the go-to person for standalones (and everything else). The analogx website does have some other interesting programs and I've never had any problem with any of them. When TechTV was on, the main guy (can't think of his name) used to always put in a good word for analogx. --- Good program, but NOT for LIVECODE -- http://www.analogx.com/contents/download/Programming/textscan/Freeware.htm --- On Mon, 7/11/11, planix wrote: From: planix Subject: getting from exe to Livecode To: use-revolution at lists.runrev.com Date: Monday, July 11, 2011, 4:53 AM Hi, I have just discovered, after numerous changes of my operating system (finally settled on Ubuntu) for various reasons, that I have deleted (doh!) my latest version of an application that I was working on. I did do backups and archive and all the rest but these files were worked on in a rush and I obviously forgot to put them where they needed to be for capture (damn and blast). I usually also print out a hard copy of the scripts that I am writing but, wouldn't you know it, I didn't do that this time. I do, however, have a windows exe file of the last version which I (happily enough) emailed to a student to use in his project. I bet you can see where this is going. Can anyone tell me whether there is a straightforward (not necessarily simple) way of reverse engineering the exe file so that I can get at the scripts? cheers Alistair Campbell Townsville From stephenREVOLUTION2 at barncard.com Mon Jul 11 14:11:21 2011 From: stephenREVOLUTION2 at barncard.com (stephen barncard) Date: Mon, 11 Jul 2011 11:11:21 -0700 Subject: Mac OS X missing controls in the LiveCode IDE In-Reply-To: <32257BFE-617E-4FC9-ABD7-92D81E9A60E5@numericable.com> References: <32257BFE-617E-4FC9-ABD7-92D81E9A60E5@numericable.com> Message-ID: crap...I hope I didn't blow my NDA... I thought it was a feature announced/added for 4.6.2 gm1 but yes... that would change things. That would be incentive for me or anyone else to cram and to learn enough Obj-C to put these suggested xcmds to work. xcmds would be a solution to keep up with the rapidly changing and platform specific UI components. However this is accomplished with the cost of losing cross-platform compatibility. Since what we are talking about is tapping the very specific 'furniture' of the OS, it should be noted these objects do not stand alone, they build on libraries deep in the gut of the OS and cannot be cross-compiled. Also there would be copyright issues here. So it's clear that if one is considering deployment on a number of platforms in Livecode, that constructing *custom controls* is still the way to go. I know it's possible to create customs that look and work *exactly *like the native controls ( the amazing datagrid comes to mind) - but many of us (myself included) get carried away with custom controls and make "improvements" to the look which gives it away. Sometimes there is just too much complexity in the information needed for a dialog to use the stock dialogs, and one has to break the rules. Custom controls are *cool*. One can make any UI gadget work just the way one wants and clone hundreds of them and run them from a single behavior script. And make a Droptool? to share with others. On 11 July 2011 10:06, Ren? Micout wrote: > > Le 11 juil. 2011 ? 18:46, stephen barncard a ?crit : > > > Perhaps with the new externals interface we can get access to these > directly > > from the OS. > > Thank you Stephen... > I am not informed about "new externals interface"... What is it? and where > can I find information about that? > > > I"d like to use the system font and size menu panel like the one that's > used > > almost everywhere like TextEdit and TexEdit. > > YES !!! > _______________________________________________ > use-livecode mailing list > use-livecode at lists.runrev.com > Please visit this url to subscribe, unsubscribe and manage your > subscription preferences: > http://lists.runrev.com/mailman/listinfo/use-livecode > -- Stephen Barncard San Francisco Ca. USA more about sqb From admin at FlexibleLearning.com Mon Jul 11 14:32:38 2011 From: admin at FlexibleLearning.com (FlexibleLearning) Date: Mon, 11 Jul 2011 19:32:38 +0100 Subject: getting from exe to Livecode In-Reply-To: Message-ID: Try... on mouseUp answer file "Select a Standalone..." if it is "cancel" or it = "" then exit to top put url ("binfile:"&it) into tStack repeat forever -- there's more than one stackfile in there which is interesting put offset("#!/bin/sh",char 10 to -1 of tStack) into tOff if tOff = 0 then exit repeat put char tOff+9 to -1 of tStack into tStack end repeat ask file "Name the Stack..." if it is "cancel" or it = "" then exit to top set the fileType to "RevoRSTK" put tStack into url ("binfile:"&it) answer "Conversion finished" with "OK" end mouseUp >From the Scripter's Scrapbook Hugh Senior FLCo Alistair Campbell wrote: Hi, I have just discovered, after numerous changes of my operating system (finally settled on Ubuntu) for various reasons, that I have deleted (doh!) my latest version of an application that I was working on. I did do backups and archive and all the rest but these files were worked on in a rush and I obviously forgot to put them where they needed to be for capture (damn and blast). I usually also print out a hard copy of the scripts that I am writing but, wouldn't you know it, I didn't do that this time. I do, however, have a windows exe file of the last version which I (happily enough) emailed to a student to use in his project. I bet you can see where this is going. Can anyone tell me whether there is a straightforward (not necessarily simple) way of reverse engineering the exe file so that I can get at the scripts? cheers Alistair Campbell Townsville From kee at kagi.com Mon Jul 11 15:18:02 2011 From: kee at kagi.com (Kee Nethery) Date: Mon, 11 Jul 2011 12:18:02 -0700 Subject: how to make answer and ask work in revweb? In-Reply-To: References: Message-ID: When I create a revlet using revweb, and access it via a browser, it appears that answer and ask don't work 100%. In "ask", I can type in the field presented and completely replace what is in the field but I cannot edit what is in the dialog field. It also appears that I cannot click on the buttons, I have to hit the enter key to "press" the default button. The "Cancel" button is not clickable (far as I can tell). for example: ask "Your name is:" with "John Dough" will display "John Dough" but if you try to edit "Dough" to "Doe" all you can do is retype the entire string to fix it "John Doe". I find that if I move the dialog around on the screen, sometimes that lets me select a button on it, but not always. In the answer dialog, I can select the default but selecting any other button seems problematic. Again, it seems that if I move the dialog around on the screen, sometimes that will let me select a button with the mouse. Is there a way to make these work or am I out of luck when trying to use ask and answer within a revlet? Thanks, Kee From m.schonewille at economy-x-talk.com Mon Jul 11 15:39:35 2011 From: m.schonewille at economy-x-talk.com (Mark Schonewille) Date: Mon, 11 Jul 2011 21:39:35 +0200 Subject: how to make answer and ask work in revweb? In-Reply-To: References: Message-ID: Hi Kee, There is a focus problem that either doesn't let you type text in the dialog or doesn't let you use the browser after the dialog disappears. I think that the best way to do this is by making a group with a custom dialog. Simply show and hide the group and grab the info you need from that group. Btw, although I have a license for it, I don't use the revweb plugin stuff anymore because it is 32 bit only, which is quite deprecated by now. All companies that ask me to do web stuff have 64 bit machines (except for schools and universities, which won't install the plugin anyway). -- Best regards, Mark Schonewille Economy-x-Talk Consulting and Software Engineering Homepage: http://economy-x-talk.com Twitter: http://twitter.com/xtalkprogrammer KvK: 50277553 New: Download the Installer Maker Plugin 1.6 for LiveCode here http://qery.us/ce On 11 jul 2011, at 21:18, Kee Nethery wrote: > When I create a revlet using revweb, and access it via a browser, it appears that answer and ask don't work 100%. > > In "ask", I can type in the field presented and completely replace what is in the field but I cannot edit what is in the dialog field. It also appears that I cannot click on the buttons, I have to hit the enter key to "press" the default button. The "Cancel" button is not clickable (far as I can tell). > > for example: > ask "Your name is:" with "John Dough" > will display "John Dough" but if you try to edit "Dough" to "Doe" all you can do is retype the entire string to fix it "John Doe". > > I find that if I move the dialog around on the screen, sometimes that lets me select a button on it, but not always. > > In the answer dialog, I can select the default but selecting any other button seems problematic. Again, it seems that if I move the dialog around on the screen, sometimes that will let me select a button with the mouse. > > Is there a way to make these work or am I out of luck when trying to use ask and answer within a revlet? > > Thanks, > Kee > From psahores at free.fr Mon Jul 11 15:49:17 2011 From: psahores at free.fr (Pierre Sahores) Date: Mon, 11 Jul 2011 21:49:17 +0200 Subject: Rev Customer Databased Hacked? In-Reply-To: <4E1B1B5B.8020109@fourthworld.com> References: <1310396624.47204.YahooMailClassic@web161603.mail.bf1.yahoo.com> <4E1B1B5B.8020109@fourthworld.com> Message-ID: An useful link to check our n-tier stuffs against those 25 rules ! Thanks Richard. Pierre Le 11 juil. 2011 ? 17:48, Richard Gaskin a ?crit : > Michael Kann wrote: > > > I was more concerned that any data was obtainable from the outside > > at all. > > A concern, but not a surprise, given the range of software components that comprise modern web apps. I think it speaks well of the RunRev that passwords weren't compromised. > > Oddly enough I was writing this morning's post to the LiveCode Journal blog about security when this thread showed up here. The post includes a couple helpful links, the best one being to a recent report of the 25 Most Dangerous Software Errors: > > > > -- > Richard Gaskin > Fourth World > LiveCode training and consulting: http://www.fourthworld.com > Webzine for LiveCode developers: http://www.LiveCodeJournal.com > LiveCode Journal blog: http://LiveCodejournal.com/blog.irv > > _______________________________________________ > use-livecode mailing list > use-livecode at lists.runrev.com > Please visit this url to subscribe, unsubscribe and manage your subscription preferences: > http://lists.runrev.com/mailman/listinfo/use-livecode > -- Pierre Sahores mobile : (33) 6 03 95 77 70 www.woooooooords.com www.sahores-conseil.com From SparkOutYNY at gmail.com Mon Jul 11 16:03:07 2011 From: SparkOutYNY at gmail.com (SparkOut) Date: Mon, 11 Jul 2011 13:03:07 -0700 (PDT) Subject: Rev Customer Databased Hacked? In-Reply-To: <95BCF900-4221-40B1-B145-19043F33BDF6@earthlink.net> References: <1E064547-F17F-4887-92CD-1A73766F0113@videotron.ca> <2EF08577-CEBA-4EF4-83BC-3E55B42D20C7@twft.com> <80A09283-D8D8-4599-8057-735A02C21137@earthlink.net> <4E1B29D4.4010509@gmail.com> <95BCF900-4221-40B1-B145-19043F33BDF6@earthlink.net> Message-ID: <1310414587230-3660677.post@n4.nabble.com> The advice to change password was *not* because of any success by any hacker at accessing your (our) password information. BUT because the hacker now has username and on-rev domain name, *if* you have a weak password it would be wise to change it to one that may be harder to try and attack by dictionary/brute force, should the hacker try in the future to use the list of usernames to find a weak nut to crack. -- View this message in context: http://runtime-revolution.278305.n4.nabble.com/Rev-Customer-Databased-Hacked-tp3659552p3660677.html Sent from the Revolution - User mailing list archive at Nabble.com. From andre at andregarzia.com Mon Jul 11 16:09:21 2011 From: andre at andregarzia.com (Andre Garzia) Date: Mon, 11 Jul 2011 17:09:21 -0300 Subject: Rev Customer Databased Hacked? In-Reply-To: <1310414587230-3660677.post@n4.nabble.com> References: <1E064547-F17F-4887-92CD-1A73766F0113@videotron.ca> <2EF08577-CEBA-4EF4-83BC-3E55B42D20C7@twft.com> <80A09283-D8D8-4599-8057-735A02C21137@earthlink.net> <4E1B29D4.4010509@gmail.com> <95BCF900-4221-40B1-B145-19043F33BDF6@earthlink.net> <1310414587230-3660677.post@n4.nabble.com> Message-ID: On Mon, Jul 11, 2011 at 5:03 PM, SparkOut wrote: > The advice to change password was *not* because of any success by any > hacker > at accessing your (our) password information. > > BUT because the hacker now has username and on-rev domain name, *if* you > have a weak password it would be wise to change it to one that may be > harder > to try and attack by dictionary/brute force, should the hacker try in the > future to use the list of usernames to find a weak nut to crack. > You are actually correct!!!! I will change it to something awfully large completely random... argh haven't thought about brute force attacks > > -- > View this message in context: > http://runtime-revolution.278305.n4.nabble.com/Rev-Customer-Databased-Hacked-tp3659552p3660677.html > Sent from the Revolution - User mailing list archive at Nabble.com. > > _______________________________________________ > use-livecode mailing list > use-livecode at lists.runrev.com > Please visit this url to subscribe, unsubscribe and manage your > subscription preferences: > http://lists.runrev.com/mailman/listinfo/use-livecode > -- http://www.andregarzia.com All We Do Is Code. From psahores at free.fr Mon Jul 11 16:37:45 2011 From: psahores at free.fr (Pierre Sahores) Date: Mon, 11 Jul 2011 22:37:45 +0200 Subject: Fwd: Rev Customer Databased Hacked? References: <1310414587230-3660677.post@n4.nabble.com> Message-ID: <02BF4792-C049-4877-8DD2-195CBD251062@free.fr> I changed all mine, even if they went in theory full safe. It's realy best for all of us to verify that our passwords are at least trusted as 100% safe by the cPanel AJAX tester. Any mix of letters, numbers and itemdels are always more trusty than only letters + numbers ;-) Best, Pierre D?but du message r?exp?di? : > The advice to change password was *not* because of any success by any hacker > at accessing your (our) password information. > > BUT because the hacker now has username and on-rev domain name, *if* you > have a weak password it would be wise to change it to one that may be harder > to try and attack by dictionary/brute force, should the hacker try in the > future to use the list of usernames to find a weak nut to crack. -- Pierre Sahores mobile : (33) 6 03 95 77 70 www.woooooooords.com www.sahores-conseil.com From andre at andregarzia.com Mon Jul 11 17:12:38 2011 From: andre at andregarzia.com (Andre Garzia) Date: Mon, 11 Jul 2011 18:12:38 -0300 Subject: Rev Customer Databased Hacked? In-Reply-To: <02BF4792-C049-4877-8DD2-195CBD251062@free.fr> References: <1310414587230-3660677.post@n4.nabble.com> <02BF4792-C049-4877-8DD2-195CBD251062@free.fr> Message-ID: On Mon, Jul 11, 2011 at 5:37 PM, Pierre Sahores wrote: > I changed all mine, even if they went in theory full safe. It's realy best > for all of us to verify that our passwords are at least trusted as 100% safe > by the cPanel AJAX tester. Any mix of letters, numbers and itemdels are > always more trusty than only letters + numbers ;-) > > I used http://strongpasswordgenerator.com/ to generate mine... :-) > Best, Pierre > > D?but du message r?exp?di? : > > > The advice to change password was *not* because of any success by any > hacker > > at accessing your (our) password information. > > > > BUT because the hacker now has username and on-rev domain name, *if* you > > have a weak password it would be wise to change it to one that may be > harder > > to try and attack by dictionary/brute force, should the hacker try in the > > future to use the list of usernames to find a weak nut to crack. > > -- > Pierre Sahores > mobile : (33) 6 03 95 77 70 > > www.woooooooords.com > www.sahores-conseil.com > > > > > > _______________________________________________ > use-livecode mailing list > use-livecode at lists.runrev.com > Please visit this url to subscribe, unsubscribe and manage your > subscription preferences: > http://lists.runrev.com/mailman/listinfo/use-livecode > -- http://www.andregarzia.com All We Do Is Code. From shaosean at wehostmacs.com Mon Jul 11 17:48:46 2011 From: shaosean at wehostmacs.com (Shao Sean) Date: Mon, 11 Jul 2011 17:48:46 -0400 Subject: Mac OS X missing controls in the LiveCode IDE Message-ID: > Perhaps with the new externals interface we can get access to these > directly you can with the old SDK.. the SDK that uses Obj-C seems to be pretty flakey and I have not been able to do anything with it other than build their example.. Mind you, you can use Obj-C in the old SDK so not real loss there.. Or is there yet another new SDK coming that actually allows us to do interface features like HC/SC/Rb/every other language ? > 24. Puff of smoke animation done > 25. Answer font could be fun, i had started work on the text palette > 26. Toolbar and his items (separator toolbar, print toolbar item, show less fun but it is doable.. I had actually found some code that would have made this so easy to "fake", but the non-standardness of Rev causes it to fail (works so nice in REALbasic).. From shaosean at wehostmacs.com Mon Jul 11 18:06:09 2011 From: shaosean at wehostmacs.com (Shao Sean) Date: Mon, 11 Jul 2011 18:06:09 -0400 Subject: [ANN] MobGUI V0.12 : tab bar and buttons Message-ID: <36D243B1-97AE-4190-AE76-10724C22F3EE@wehostmacs.com> Using the new iOS external SDK you can (hopefully) tap in to the system and use the built-in icons (in the Constants section, scroll down for images).. http://developer.apple.com/library/ios/documentation/UIKit/Reference/UIBarButtonItem_Class/Reference/Reference.html#/ /apple_ref/doc/uid/TP40007519-CH3-DontLinkElementID_2 From rene.micout at numericable.com Mon Jul 11 18:24:21 2011 From: rene.micout at numericable.com (=?iso-8859-1?Q?Ren=E9_Micout?=) Date: Tue, 12 Jul 2011 00:24:21 +0200 Subject: Mac OS X missing controls in the LiveCode IDE In-Reply-To: References: Message-ID: <66D7379B-1F3A-453E-B3BD-124853312728@numericable.com> Le 11 juil. 2011 ? 23:48, Shao Sean a ?crit : >> Perhaps with the new externals interface we can get access to these directly > you can with the old SDK.. the SDK that uses Obj-C seems to be pretty flakey and I have not been able to do anything with it other than build their example.. Mind you, you can use Obj-C in the old SDK so not real loss there.. Or is there yet another new SDK coming that actually allows us to do interface features like HC/SC/Rb/every other language ? Here I am overwhelmed !! > >> 24. Puff of smoke animation > done Good ! But where ? > >> 25. Answer font > could be fun, i had started work on the text palette Good ! > >> 26. Toolbar and his items (separator toolbar, print toolbar item, show > less fun but it is doable.. I had actually found some code that would have made this so easy to "fake", but the non-standardness of Rev causes it to fail (works so nice in REALbasic).. It would be nice ! ... From tsj at unimelb.edu.au Mon Jul 11 18:46:44 2011 From: tsj at unimelb.edu.au (Terry Judd) Date: Tue, 12 Jul 2011 08:46:44 +1000 Subject: Rev Customer Databased Hacked? In-Reply-To: References: <1E064547-F17F-4887-92CD-1A73766F0113@videotron.ca> Message-ID: Same here. Terry... On 12/07/2011, at 1:44 AM, "Jeff Massung" wrote: > I would just like to say that I haven't been an On-Rev customer for over a > year now, and I want to thank the Rev team for still including me in this > email (so, yes, I got one, too). > > Jeff M. > _______________________________________________ > use-livecode mailing list > use-livecode at lists.runrev.com > Please visit this url to subscribe, unsubscribe and manage your subscription preferences: > http://lists.runrev.com/mailman/listinfo/use-livecode > From john at splash21.com Mon Jul 11 18:59:24 2011 From: john at splash21.com (John Craig) Date: Mon, 11 Jul 2011 23:59:24 +0100 Subject: [ANN] MobGUI V0.12 : tab bar and buttons In-Reply-To: <36D243B1-97AE-4190-AE76-10724C22F3EE@wehostmacs.com> References: <36D243B1-97AE-4190-AE76-10724C22F3EE@wehostmacs.com> Message-ID: <82BFF7EF-EC17-4F97-BD11-AA9427918EB0@splash21.com> Thanks for the link - will check it out! Sent from my iPhone On 11 Jul 2011, at 23:06, Shao Sean wrote: > Using the new iOS external SDK you can (hopefully) tap in to the system and use the built-in icons (in the Constants section, scroll down for images).. > > http://developer.apple.com/library/ios/documentation/UIKit/Reference/UIBarButtonItem_Class/Reference/Reference.html#//apple_ref/doc/uid/TP40007519-CH3-DontLinkElementID_2 > > _______________________________________________ > use-livecode mailing list > use-livecode at lists.runrev.com > Please visit this url to subscribe, unsubscribe and manage your subscription preferences: > http://lists.runrev.com/mailman/listinfo/use-livecode From shaosean at wehostmacs.com Mon Jul 11 19:05:15 2011 From: shaosean at wehostmacs.com (Shao Sean) Date: Mon, 11 Jul 2011 19:05:15 -0400 Subject: Mac OS X missing controls in the LiveCode IDE Message-ID: <605A5842-D0C2-4052-8E73-6F815E2AFB92@wehostmacs.com> > >> 24. Puff of smoke animation > > done > Good ! But where ? external > >> 26. Toolbar and his items (separator toolbar, print toolbar item, > show > > less fun but it is doable.. I had actually found some code that > would have made this so easy to "fake", but the non-standardness of > Rev causes it to fail (works so nice in REALbasic).. > It would be nice ! yes it would From kee at kagi.com Mon Jul 11 20:28:09 2011 From: kee at kagi.com (Kee Nethery) Date: Mon, 11 Jul 2011 17:28:09 -0700 Subject: how to make answer and ask work in revweb? In-Reply-To: References: Message-ID: <48E1900C-1FBA-4B15-8491-60EBF787ADA8@kagi.com> On Jul 11, 2011, at 12:39 PM, Mark Schonewille wrote: > Hi Kee, > > There is a focus problem that either doesn't let you type text in the dialog or doesn't let you use the browser after the dialog disappears. yep > I think that the best way to do this is by making a group with a custom dialog. Simply show and hide the group and grab the info you need from that group. I thought so too except that I had ask and answer in code that waits for the answer and then continues with the results obtained. For example, click on a line in a table, ask what should happen, and then do it to that line of data in that table. Kinda difficult to simulate that with a group without a bunch of message passing and such and breaking up the code in strange ways. But it will have to be done. > Btw, although I have a license for it, I don't use the revweb plugin stuff anymore because it is 32 bit only, which is quite deprecated by now. All companies that ask me to do web stuff have 64 bit machines (except for schools and universities, which won't install the plugin anyway). I am lucky in that the target audience is 32bit. Thanks, Kee From chipp at chipp.com Mon Jul 11 20:37:03 2011 From: chipp at chipp.com (Chipp Walters) Date: Mon, 11 Jul 2011 19:37:03 -0500 Subject: getting from exe to Livecode In-Reply-To: References: Message-ID: If that works, please PURCHASE Scripter's Scrapbook! (Even if it doesn't you should still purchase it as it has a ton of GREAT scripts!) On Mon, Jul 11, 2011 at 1:32 PM, FlexibleLearning < admin at flexiblelearning.com> wrote: > Try... > > on mouseUp > answer file "Select a Standalone..." > if it is "cancel" or it = "" then exit to top > put url ("binfile:"&it) into tStack > repeat forever > -- there's more than one stackfile in there which is interesting > put offset("#!/bin/sh",char 10 to -1 of tStack) into tOff > if tOff = 0 then exit repeat > put char tOff+9 to -1 of tStack into tStack > end repeat > ask file "Name the Stack..." > if it is "cancel" or it = "" then exit to top > set the fileType to "RevoRSTK" > put tStack into url ("binfile:"&it) > answer "Conversion finished" with "OK" > end mouseUp > > From the Scripter's Scrapbook > > Hugh Senior > FLCo > > > > Alistair Campbell wrote: > > Hi, > > I have just discovered, after numerous changes of my operating system > (finally settled on Ubuntu) for various reasons, that I have deleted (doh!) > my latest version of an application that I was working on. I did do backups > and archive and all the rest but these files were worked on in a rush and I > obviously forgot to put them where they needed to be for capture (damn and > blast). I usually also print out a hard copy of the scripts that I am > writing but, wouldn't you know it, I didn't do that this time. > > I do, however, have a windows exe file of the last version which I (happily > enough) emailed to a student to use in his project. I bet you can see where > this is going. Can anyone tell me whether there is a straightforward (not > necessarily simple) way of reverse engineering the exe file so that I can > get at the scripts? > > cheers > > Alistair Campbell > Townsville > > _______________________________________________ > use-livecode mailing list > use-livecode at lists.runrev.com > Please visit this url to subscribe, unsubscribe and manage your > subscription preferences: > http://lists.runrev.com/mailman/listinfo/use-livecode > -- Chipp Walters CEO, Shafer Walters Group, Inc. From mwieder at ahsoftware.net Mon Jul 11 20:39:14 2011 From: mwieder at ahsoftware.net (Mark Wieder) Date: Mon, 11 Jul 2011 17:39:14 -0700 Subject: Mac OS X missing controls in the LiveCode IDE In-Reply-To: References: <32257BFE-617E-4FC9-ABD7-92D81E9A60E5@numericable.com> Message-ID: <44809919468.20110711173914@ahsoftware.net> stephen- Monday, July 11, 2011, 11:11:21 AM, you wrote: > crap...I hope I didn't blow my NDA... I thought it was a feature > announced/added for 4.6.2 gm1 Not to worry... this was announced and demonstrated at the conference. -- -Mark Wieder mwieder at ahsoftware.net From shaosean at wehostmacs.com Mon Jul 11 20:46:43 2011 From: shaosean at wehostmacs.com (Shao Sean) Date: Mon, 11 Jul 2011 20:46:43 -0400 Subject: Mac OS X missing controls in the LiveCode IDE Message-ID: If it is not covered under NDA feel free to let us know about it.. Some of us have been waiting since 4.0 for the new SDK.. From chipp at chipp.com Mon Jul 11 20:47:55 2011 From: chipp at chipp.com (Chipp Walters) Date: Mon, 11 Jul 2011 19:47:55 -0500 Subject: Mac OS X missing controls in the LiveCode IDE In-Reply-To: References: Message-ID: Shao, I believe the altButton control is better now than ButtonGadget as it can do much of what BG can do, and all of it's properties are accessible by script. Also, the gradient property for the button is also settable by script for altButton. I'm EOL'ing ButtonGadget in favor of altButton. I'll use the base altButton object as a point of departure for other controls. Also, please consider allowing any controls you create to work with Ken's fabulous DropTools architecture. It would be nice to have some of the wonderful things you make available as DropTools-- your work speaks for itself! On Mon, Jul 11, 2011 at 9:31 AM, Ren? Micout wrote: > I made an inventory of missing Mac OS X controls, see list below : > 1. Round button > 2. ? Textured button > > http://blog.chipp.com/new-altbutton-control-for-livecode/ (?) > 3. ? Recessed button > I have a project on it > 4. ? Disclosure button > I have a project on it > 5. ? Gradient button > http://www.buttongadget.com (?) > 6. ? Rounded rect button > > http://blog.chipp.com/new-altbutton-control-for-livecode/ (?) > 7. ? Rounded textured button > > http://blog.chipp.com/new-altbutton-control-for-livecode/ (?) > 8. ? Help button > I have a project on it > 9. ? Bevel button > > http://blog.chipp.com/new-altbutton-control-for-livecode/ (?) > 10. ? Radio button > size mini > I work on it > 11. ? Check box > size mini > I work on it > 12. ? Stepper > size mini > I work on it > 13. Date picker > 14. Segmented control > 15. ? Search field > http://droptools.sonsothunder.com/all.irev > 16. Token field > 17. ? Image Well > http://droptools.sonsothunder.com/all.irev > 18. Color well > 19. ? Slider (vertical and horizontal) > size : small and mini > I work on > it > 20. ? Circular slider > I have a project on it > 21. ? Circular progress indicator > > http://revonline2.runrev.com/stack/440/Spinning-wheel > 22. Vertical and horizontal split view > 23. ? HUD panel > http://revonline2.runrev.com/stack/574/HUD-Panel-Factory > 24. Puff of smoke animation > 25. Answer font > 26. Toolbar and his items (separator toolbar, print toolbar item, show > color toolbar item, show font toolbar item, customize toolbar item, search > toolbar item). > Some have their "custom control"... But not all... Is my list up to date, > incomplete or wrong? > Thank you by advance... > Ren? > _______________________________________________ > use-livecode mailing list > use-livecode at lists.runrev.com > Please visit this url to subscribe, unsubscribe and manage your > subscription preferences: > http://lists.runrev.com/mailman/listinfo/use-livecode -- Chipp Walters CEO, Shafer Walters Group, Inc. From chipp at chipp.com Mon Jul 11 20:58:24 2011 From: chipp at chipp.com (Chipp Walters) Date: Mon, 11 Jul 2011 19:58:24 -0500 Subject: how to make answer and ask work in revweb? In-Reply-To: <48E1900C-1FBA-4B15-8491-60EBF787ADA8@kagi.com> References: <48E1900C-1FBA-4B15-8491-60EBF787ADA8@kagi.com> Message-ID: Kee, I just finished creating a generic dialog sheet created with groups. It's modal but does not use windows. The function call is: altAnswerDialog (*dialog icon, dialog title, dialog msg, any number of buttons)* * * It supports icons: help,information,warning,about,error,question If the msg is html, then it will display as htmltext It uses the altButton object, but could be modified to use regular buttons It slides down from the top of the window, and adjusts it's height to how large the message. If taller than the window, it will instead put a scrollbar on the msg field. it is used like this: *get* altAnswerDialog("Question","Logoff or Quit","Do you want to Logoff or Quit Now?","Logoff","Quit","Cancel") if it is "Cancel" then exit to top If this is of interest, holler at me offline and I'll send you copy. I'm using it in an app which has to work on Mac, PC, and iPad, thus the requirement of not using real dialogs. On Mon, Jul 11, 2011 at 7:28 PM, Kee Nethery wrote: > > On Jul 11, 2011, at 12:39 PM, Mark Schonewille wrote: > > > Hi Kee, > > > > There is a focus problem that either doesn't let you type text in the > dialog or doesn't let you use the browser after the dialog disappears. > > yep > > > I think that the best way to do this is by making a group with a custom > dialog. Simply show and hide the group and grab the info you need from that > group. > > I thought so too except that I had ask and answer in code that waits for > the answer and then continues with the results obtained. > > For example, click on a line in a table, ask what should happen, and then > do it to that line of data in that table. Kinda difficult to simulate that > with a group without a bunch of message passing and such and breaking up the > code in strange ways. But it will have to be done. > > > Btw, although I have a license for it, I don't use the revweb plugin > stuff anymore because it is 32 bit only, which is quite deprecated by now. > All companies that ask me to do web stuff have 64 bit machines (except for > schools and universities, which won't install the plugin anyway). > > I am lucky in that the target audience is 32bit. > > Thanks, Kee > _______________________________________________ > use-livecode mailing list > use-livecode at lists.runrev.com > Please visit this url to subscribe, unsubscribe and manage your > subscription preferences: > http://lists.runrev.com/mailman/listinfo/use-livecode > -- Chipp Walters CEO, Shafer Walters Group, Inc. From capellan2000 at gmail.com Mon Jul 11 22:22:52 2011 From: capellan2000 at gmail.com (Alejandro Tejada) Date: Mon, 11 Jul 2011 22:22:52 -0400 Subject: [ANN] AndreGarzia.Com new web page Message-ID: Hi Andre, Your new website looks great! Congratulations! Only one question: Does it really took 24 hours? :-) Alejandro From jiml at netrin.com Mon Jul 11 22:34:04 2011 From: jiml at netrin.com (Jim Lambert) Date: Mon, 11 Jul 2011 19:34:04 -0700 Subject: AW: How to test if an image is empty? In-Reply-To: References: Message-ID: Ken, Great explanation. Thanks, JimL From andre at andregarzia.com Tue Jul 12 00:02:44 2011 From: andre at andregarzia.com (Andre Garzia) Date: Tue, 12 Jul 2011 01:02:44 -0300 Subject: [ANN] AndreGarzia.Com new web page In-Reply-To: References: Message-ID: On Mon, Jul 11, 2011 at 11:22 PM, Alejandro Tejada wrote: > Hi Andre, > > Your new website looks great! > Congratulations! > > Only one question: > Does it really took 24 hours? :-) > Alejandro, It actually took less than 24h... I will write a little journal on how it was built shortly. :-) > > Alejandro > > _______________________________________________ > use-livecode mailing list > use-livecode at lists.runrev.com > Please visit this url to subscribe, unsubscribe and manage your > subscription preferences: > http://lists.runrev.com/mailman/listinfo/use-livecode > -- http://www.andregarzia.com All We Do Is Code. From jacque at hyperactivesw.com Tue Jul 12 00:06:13 2011 From: jacque at hyperactivesw.com (J. Landman Gay) Date: Mon, 11 Jul 2011 23:06:13 -0500 Subject: getting from exe to Livecode In-Reply-To: References: Message-ID: <4E1BC835.9070501@hyperactivesw.com> On 7/11/11 7:37 PM, Chipp Walters wrote: > If that works, please PURCHASE Scripter's Scrapbook! (Even if it doesn't you > should still purchase it as it has a ton of GREAT scripts!) I'm afraid the extraction method in the script has been obsolete for some years, so it won't work any more. That's no reflection on SS by any means, though. -- Jacqueline Landman Gay | jacque at hyperactivesw.com HyperActive Software | http://www.hyperactivesw.com From dr.alistair at gmail.com Tue Jul 12 03:13:35 2011 From: dr.alistair at gmail.com (planix) Date: Tue, 12 Jul 2011 00:13:35 -0700 (PDT) Subject: getting from exe to Livecode In-Reply-To: <4E1B2F24.3070306@hyperactivesw.com> References: <1310377983813-3659096.post@n4.nabble.com> <4E1B2F24.3070306@hyperactivesw.com> Message-ID: <1310454815760-3661610.post@n4.nabble.com> Hi, Thanks for all the good ideas. I didn't password protect the stack and I have swept through the exe with a hex editor and a text editor. I can't find any english language version of my code. I built the exe in LC4.5. I wonder if there is some encryption that now just occurs automagicaly? cheers -- View this message in context: http://runtime-revolution.278305.n4.nabble.com/getting-from-exe-to-Livecode-tp3659096p3661610.html Sent from the Revolution - User mailing list archive at Nabble.com. From dr.alistair at gmail.com Tue Jul 12 03:20:29 2011 From: dr.alistair at gmail.com (planix) Date: Tue, 12 Jul 2011 00:20:29 -0700 (PDT) Subject: getting from exe to Livecode In-Reply-To: References: <82DF1CA7-88E7-4813-A98E-0DEE415CE774@numericable.com> <1310407421.36078.YahooMailClassic@web161602.mail.bf1.yahoo.com> Message-ID: <1310455229676-3661619.post@n4.nabble.com> That would have been nice. But it just returns a corrupted file. -- View this message in context: http://runtime-revolution.278305.n4.nabble.com/ANN-Update-3-custom-controls-on-RevOnLine-tp3659664p3661619.html Sent from the Revolution - User mailing list archive at Nabble.com. From dr.alistair at gmail.com Tue Jul 12 03:22:28 2011 From: dr.alistair at gmail.com (planix) Date: Tue, 12 Jul 2011 00:22:28 -0700 (PDT) Subject: getting from exe to Livecode In-Reply-To: References: <82DF1CA7-88E7-4813-A98E-0DEE415CE774@numericable.com> <1310407421.36078.YahooMailClassic@web161602.mail.bf1.yahoo.com> Message-ID: <1310455348925-3661623.post@n4.nabble.com> Hi Chipp, Nup, doesn't work. But, I already have purchased SS. Great tool but I have to get my head organised to use it properly. cheers -- View this message in context: http://runtime-revolution.278305.n4.nabble.com/ANN-Update-3-custom-controls-on-RevOnLine-tp3659664p3661623.html Sent from the Revolution - User mailing list archive at Nabble.com. From admin at FlexibleLearning.com Tue Jul 12 03:28:22 2011 From: admin at FlexibleLearning.com (FlexibleLearning) Date: Tue, 12 Jul 2011 08:28:22 +0100 Subject: getting from exe to Livecode In-Reply-To: Message-ID: You could try asking RunRev if they can reverse-engineer your exe. It used to be a service offered 'in extreemis'. Hugh Senior FLCo On 7/11/11 7:37 PM, Chipp Walters wrote: > If that works, please PURCHASE Scripter's Scrapbook! (Even if it doesn't you > should still purchase it as it has a ton of GREAT scripts!) Jacqueline wrote: >I'm afraid the extraction method in the script has been obsolete for >some years, so it won't work any more. That's no reflection on SS by any >means, though. From psahores at free.fr Tue Jul 12 03:58:38 2011 From: psahores at free.fr (Pierre Sahores) Date: Tue, 12 Jul 2011 09:58:38 +0200 Subject: [ANN] AndreGarzia.Com new web page In-Reply-To: References: Message-ID: <4CD56108-048D-4401-A26E-4405FC88530F@free.fr> Andre, Seems i will learn some important ways to go in reading it ;D KR, Pierre Le 12 juil. 2011 ? 06:02, Andre Garzia a ?crit : > On Mon, Jul 11, 2011 at 11:22 PM, Alejandro Tejada > wrote: > >> Hi Andre, >> >> Your new website looks great! >> Congratulations! >> >> Only one question: >> Does it really took 24 hours? :-) >> > > > Alejandro, > > It actually took less than 24h... I will write a little journal on how it > was built shortly. > > :-) > > > > >> >> Alejandro >> >> _______________________________________________ >> use-livecode mailing list >> use-livecode at lists.runrev.com >> Please visit this url to subscribe, unsubscribe and manage your >> subscription preferences: >> http://lists.runrev.com/mailman/listinfo/use-livecode >> > > > > -- > http://www.andregarzia.com All We Do Is Code. > _______________________________________________ > use-livecode mailing list > use-livecode at lists.runrev.com > Please visit this url to subscribe, unsubscribe and manage your subscription preferences: > http://lists.runrev.com/mailman/listinfo/use-livecode > -- Pierre Sahores mobile : (33) 6 03 95 77 70 www.woooooooords.com www.sahores-conseil.com From toolbook at kestner.de Tue Jul 12 04:40:28 2011 From: toolbook at kestner.de (Tiemo Hollmann TB) Date: Tue, 12 Jul 2011 10:40:28 +0200 Subject: AW: AW: How to test if an image is empty? In-Reply-To: <71E960B1-1ACD-4600-94F6-4DA4B438B82B@sonsothunder.com> References: <003e01cc3fae$66f78a60$34e69f20$@de> <004601cc3fbb$b38f50d0$1aadf270$@de> <71E960B1-1ACD-4600-94F6-4DA4B438B82B@sonsothunder.com> Message-ID: <000c01cc406f$5e459df0$1ad0d9d0$@de> Great explanation, I wasn't aware of that. thank you Ken Tiemo > -----Urspr?ngliche Nachricht----- > Von: use-livecode-bounces at lists.runrev.com [mailto:use-livecode- > bounces at lists.runrev.com] Im Auftrag von Ken Ray > Gesendet: Montag, 11. Juli 2011 15:29 > An: How to use LiveCode > Betreff: Re: AW: How to test if an image is empty? > > > Thanks Mark and Scott, > > "the text" ... hmm, not the first to think about with an image :) > > If it helps, Tiemo, you can use this: > > put "" into image 1 > > and > > if image 1 is empty then... > > The "text" property is implied in the same was as for fields ("put field 1 > into.." vs. "put the text of field 1 into..."). > > Here's a good way to think about images: there's a distinction between the > actual image binary data and what is used to show the user what the image > currently looks like (including scaling, rotation, rendering in various > quality levels, the "paintCompression", etc.). The binary data is the image's > *contents* (the 'text' of the image in this case), and what is shown to the > user is the 'imageData'+'maskData'+'alphaData' (which I'll just call "image > data"). If you empty out the image data, the binary data still exists - all > you've done is to say (in effect) "don't show this image to the user", or more > accurately: "show blank to the user". > > You can have binary data without image data but not the other way around; once > you set the image data of an image, the binary data is *created* to support > the image data you set. This can be very useful in keeping a scaled version of > an image in a stack without holding on to all the extra "weight" of the > original image. > > So for example, if you imported a 100K image that was 1000 x 1000 pixels and > then scaled it down to 100 x 100 (and set the lockLoc to true so it doesn't > "pop" back to its original size), the user would see 100 x 100 pixels of image > data, but the image would be storing 1000 x 1000 pixels of binary data. > However, if you created a new blank 100 x 100 image an then executed: > > set the imageData of img 2 to the imageData of img 1 > set the alphaData of img 2 to the alphaData of img 1 > set the maskData of img 2 to the maskData of img 1 > > the binary data for img 2 would only be what is necessary to support what the > user sees (100 x 100 pixels). It would look exactly like image 1, but would be > only 1% of the original number of pixels and would only take up 1K instead of > 100K. You could then delete image 1 and you'd have exactly what you started > with but storing a bunch less space. > > This is great for working with thumbnails of full-resolution images; of course > if you *need* to keep the full-res image around because the image might scale > *up* from 100x100 to 1000x1000 (or any size in between) then you want to work > with the full-res image and not make a "cheap copy", but you get the idea. > > I have a very old (but still mostly accurate) primer on imageData, alphaData, > and maskData here: > > http://www.sonsothunder.com/devres/revolution/tips/imag003.htm > > Hope this helps, > > Ken Ray > Sons of Thunder Software, Inc. > Email: kray at sonsothunder.com > Web Site: http://www.sonsothunder.com/ > > > > > Thanks for your quick response > > Tiemo > > > >> -----Urspr?ngliche Nachricht----- > >> Von: use-livecode-bounces at lists.runrev.com [mailto:use-livecode- > >> bounces at lists.runrev.com] Im Auftrag von Scott Rossi > >> Gesendet: Montag, 11. Juli 2011 11:48 > >> An: LiveCode Mail List > >> Betreff: Re: How to test if an image is empty? > >> > >> Recently, Tiemo Hollmann TB wrote: > >> > >>> when I put empty into img "foo" the imagedata of img "foo" is still not > >>> empty, there are still any binary data in the image. > >>> > >>> I can't test if the imagedata of img "foo" is empty" nor can I test if > > the > >>> img "foo" is empty. How can I test if I have put empty into an image? > > Can I > >>> test it binary? > >> > >> To "truly" empty an image, I set the text property of the image to empty, > >> and also check that property to make sure it's empty. > >> > >> return (the text of img 1 is empty) > >> > >> Regards, > >> > >> Scott Rossi > >> Creative Director > >> Tactile Media, UX Design > >> > >> > >> > >> _______________________________________________ > >> use-livecode mailing list > >> use-livecode at lists.runrev.com > >> Please visit this url to subscribe, unsubscribe and manage your > > subscription > >> preferences: > >> http://lists.runrev.com/mailman/listinfo/use-livecode > > > > > > _______________________________________________ > > use-livecode mailing list > > use-livecode at lists.runrev.com > > Please visit this url to subscribe, unsubscribe and manage your subscription > preferences: > > http://lists.runrev.com/mailman/listinfo/use-livecode > > > > _______________________________________________ > use-livecode mailing list > use-livecode at lists.runrev.com > Please visit this url to subscribe, unsubscribe and manage your subscription > preferences: > http://lists.runrev.com/mailman/listinfo/use-livecode From mblivecode at harbourhosting.co.uk Tue Jul 12 05:05:22 2011 From: mblivecode at harbourhosting.co.uk (Martin Baxter) Date: Tue, 12 Jul 2011 10:05:22 +0100 Subject: Rev Customer Databased Hacked? In-Reply-To: <1E064547-F17F-4887-92CD-1A73766F0113@videotron.ca> References: <1E064547-F17F-4887-92CD-1A73766F0113@videotron.ca> Message-ID: <4E1C0E52.60907@harbourhosting.co.uk> As well as changing your password, which I think should be done as a matter of course, I would suggest also changing your onrev contact email if you can. Ideally it should be a new one only used as contact email for that account and maybe redirected to a real mailbox. This is because there is danger of this stolen info being used in a spear-phishing email, perhaps purporting to be from runrev about "your onrev account" and containing a poisoned URL of some sort or some other trickery. If the contact email is unique to the account (and non-obvious), then it makes it easier to identify any fraudulent emails of this sort. IMO, online security is a contradiction in terms. Even if you are smart, your security is nowadays crowdsourced and therefore dependent on so many unknown others, many of whom are not smart, that it is wise to assume everything online will eventually be compromised, and plan accordingly. We are now in the "steal everything" era of online crime. It always irritates me when online credentials consist of any factors which cannot be changed if they are compromised - as in this case the account id. This is the problem with biometric credentials, once they are compromised, how do you change them to repair your security? The weakest link is in-between the chair and the keyboard and, unfortunately, cannot easily be upgraded or patched. ;-) Martin Baxter On 11/07/2011 14:34, Gregory Lypny wrote: > Hello everyone, > > Have any of you received this message from Heather? Implications? > > Gregory > > >> Dear Gregory Lypny, From toolbook at kestner.de Tue Jul 12 05:33:22 2011 From: toolbook at kestner.de (Tiemo Hollmann TB) Date: Tue, 12 Jul 2011 11:33:22 +0200 Subject: why don't I get mouse events with brush tool? Message-ID: <001301cc4076$c3136ad0$493a4070$@de> Hello, some days ago Bj?rnke taught me that it?s not a bug but a feature that I can?t pick the mousecolor while having the brush tool activated. Now I am experiencing, that I also don?t get even a mouseDown or mouseUp with the brush tool. The dictionary doesn?t says anything about this behavior. What I want to do is: The user can paint with the brush tool on an image. To supply the user an undo with several states, I wanted to make a backup of the image each time at mouseDown, so that the user can roll back every newly painted state. I tried to choose the brush tool with mouseEnter and make the backup with mouseDown (I don?t get the mouseDown event). I tried to make the backup with mouseDown and choose brush tool afterwards (works only once until the brush tool is choosen in the first mouseDown) I tried to switch back to the browse tool with every mouseUp to make the backup with next mouseDown (I don?t get the mouseUp event, when having selected the brush tool) Probably I am again too narrow minded and looking into the wrong direction, can anybody point me into another direction, how I can realize my undo function? Thanks Tiemo From rene.micout at numericable.com Tue Jul 12 05:46:15 2011 From: rene.micout at numericable.com (=?iso-8859-1?Q?Ren=E9_Micout?=) Date: Tue, 12 Jul 2011 11:46:15 +0200 Subject: Rev Customer Databased Hacked? In-Reply-To: <4E1C0E52.60907@harbourhosting.co.uk> References: <1E064547-F17F-4887-92CD-1A73766F0113@videotron.ca> <4E1C0E52.60907@harbourhosting.co.uk> Message-ID: <87166697-4422-4618-A4AB-30BB67728840@numericable.com> Le 12 juil. 2011 ? 11:05, Martin Baxter a ?crit : > The weakest link is in-between the chair and the keyboard and, > unfortunately, cannot easily be upgraded or patched. ;-) No, but, (un)fortunately "it" is initialized from time to time... ;-) or :-( Bon souvenir de Paris Ren? From rene.micout at numericable.com Tue Jul 12 05:47:59 2011 From: rene.micout at numericable.com (=?iso-8859-1?Q?Ren=E9_Micout?=) Date: Tue, 12 Jul 2011 11:47:59 +0200 Subject: Mac OS X missing controls in the LiveCode IDE In-Reply-To: <605A5842-D0C2-4052-8E73-6F815E2AFB92@wehostmacs.com> References: <605A5842-D0C2-4052-8E73-6F815E2AFB92@wehostmacs.com> Message-ID: <3473D022-46BB-4646-B54B-46C0A535A4AA@numericable.com> Le 12 juil. 2011 ? 01:05, Shao Sean a ?crit : >> >> 24. Puff of smoke animation >> > done >> Good ! But where ? > external Thank you Shao Sean... Where can find it ? From rene.micout at numericable.com Tue Jul 12 05:59:40 2011 From: rene.micout at numericable.com (=?iso-8859-1?Q?Ren=E9_Micout?=) Date: Tue, 12 Jul 2011 11:59:40 +0200 Subject: Mac OS X missing controls in the LiveCode IDE In-Reply-To: References: <32257BFE-617E-4FC9-ABD7-92D81E9A60E5@numericable.com> Message-ID: <1EF1F66A-D8F6-4F1B-8B7D-627FAA297D54@numericable.com> Le 11 juil. 2011 ? 20:11, stephen barncard a ?crit : > Custom controls are *cool*. One can make any UI gadget work just the way one > wants and clone hundreds of them and run them from a single behavior script. > And make a Droptool? to share with others. I work on it... [OT] > I see your young face on jacket disc "If I Could Only Remenber My Name" of David Crosby under Grace Slick 1971 > Ah ! Mes vingt ans ! Bon souvenir de Paris Ren? ? twenty years old... in 1971... :-( From pmbrig at gmail.com Tue Jul 12 11:19:00 2011 From: pmbrig at gmail.com (Peter Brigham MD) Date: Tue, 12 Jul 2011 11:19:00 -0400 Subject: Rev Customer Databased Hacked? In-Reply-To: References: <1310414587230-3660677.post@n4.nabble.com> <02BF4792-C049-4877-8DD2-195CBD251062@free.fr> Message-ID: <2CFF3D9C-1008-463B-92CC-A50D8FB0898D@gmail.com> On Jul 11, 2011, at 5:12 PM, Andre Garzia wrote: > On Mon, Jul 11, 2011 at 5:37 PM, Pierre Sahores wrote: > >> I changed all mine, even if they went in theory full safe. It's realy best >> for all of us to verify that our passwords are at least trusted as 100% safe >> by the cPanel AJAX tester. Any mix of letters, numbers and itemdels are >> always more trusty than only letters + numbers ;-) >> >> > I used http://strongpasswordgenerator.com/ to generate mine... :-) I made a simple stack to generate strong passwords. Options include choosing length of PW and whether to use punctuation characters or not. Should be pretty robust. Using this kind of password really means you have to have a secure repository for your passwords, since random passwords are rarely easy to remember. (Here is a plug for Bill Vlahos' InfoWallet!) Mac: http://db.tt/RieJv91 Windows: http://db.tt/PEaUKCY (sorry, no Linux) -- Peter Peter M. Brigham pmbrig at gmail.com http://home.comcast.net/~pmbrig From livfoss at mac.com Tue Jul 12 12:19:32 2011 From: livfoss at mac.com (Graham Samuel) Date: Tue, 12 Jul 2011 18:19:32 +0200 Subject: C programmer wanted to help with (Windows) LiveCode project Message-ID: <9EC75F90-EC46-4E3F-B4B0-4F611A816743@mac.com> Folks, I want to get my current LC project to read images from PDF files - this is a small subset of the functionality of most PDF readers and library components on sale today. As my current project is for Windows customers, I am content for now with a Windows-only solution (although I would love it if LC could natively read more image formats, including both PDF and TIFF, on all supported platforms). After a lot of discussion with RunRev and others, it seems the short-term solution particularly in the absence of an up-do-date externals interface, is to link an appropriate DLL to the LiveCode engine with some C code 'glue'. I believe I have a good candidate for the DLL, but as I know very little about C programming, I would like to pay a professional (OK, or a gifted amateur) to do the gluing. If anyone on this list would like to help or has an appropriate contact, please write to me off-list. TIA Graham From stephenREVOLUTION2 at barncard.com Tue Jul 12 13:07:29 2011 From: stephenREVOLUTION2 at barncard.com (stephen barncard) Date: Tue, 12 Jul 2011 10:07:29 -0700 Subject: Mac OS X missing controls in the LiveCode IDE In-Reply-To: <1EF1F66A-D8F6-4F1B-8B7D-627FAA297D54@numericable.com> References: <32257BFE-617E-4FC9-ABD7-92D81E9A60E5@numericable.com> <1EF1F66A-D8F6-4F1B-8B7D-627FAA297D54@numericable.com> Message-ID: it's true. I'm almost famous! But Grace only sang in a choir with others. Jerry Garcia was the biggest influence on the record. sqb On 12 July 2011 02:59, Ren? Micout wrote: > > Le 11 juil. 2011 ? 20:11, stephen barncard a ?crit : > > > Custom controls are *cool*. One can make any UI gadget work just the way > one > > wants and clone hundreds of them and run them from a single behavior > script. > > And make a Droptool? to share with others. > > > I work on it... > > [OT] > I see your young face on jacket disc "If I Could Only Remenber My > Name" of David Crosby under Grace Slick > 1971 > Ah ! Mes vingt ans ! > > Bon souvenir de Paris > Ren? ? twenty years old... in 1971... :-( > _______________________________________________ > use-livecode mailing list > use-livecode at lists.runrev.com > Please visit this url to subscribe, unsubscribe and manage your > subscription preferences: > http://lists.runrev.com/mailman/listinfo/use-livecode > -- Stephen Barncard San Francisco Ca. USA more about sqb From arietext at mac.com Tue Jul 12 13:12:26 2011 From: arietext at mac.com (Arie van der Ent) Date: Tue, 12 Jul 2011 19:12:26 +0200 Subject: Rev Customer Databased Hacked? In-Reply-To: <2CFF3D9C-1008-463B-92CC-A50D8FB0898D@gmail.com> References: <1310414587230-3660677.post@n4.nabble.com> <02BF4792-C049-4877-8DD2-195CBD251062@free.fr> <2CFF3D9C-1008-463B-92CC-A50D8FB0898D@gmail.com> Message-ID: <451228D7-B522-4EAB-8DE3-B15B48269DF9@mac.com> Ik ben helemaal gerustgesteld. Fijne vakantie. Arie Verstuurd vanaf mijn iPad Op 12 jul. 2011 om 17:19 heeft Peter Brigham MD het volgende geschreven: > On Jul 11, 2011, at 5:12 PM, Andre Garzia wrote: > >> On Mon, Jul 11, 2011 at 5:37 PM, Pierre Sahores wrote: >> >>> I changed all mine, even if they went in theory full safe. It's realy best >>> for all of us to verify that our passwords are at least trusted as 100% safe >>> by the cPanel AJAX tester. Any mix of letters, numbers and itemdels are >>> always more trusty than only letters + numbers ;-) >>> >>> >> I used http://strongpasswordgenerator.com/ to generate mine... :-) > > I made a simple stack to generate strong passwords. Options include choosing length of PW and whether to use punctuation characters or not. Should be pretty robust. Using this kind of password really means you have to have a secure repository for your passwords, since random passwords are rarely easy to remember. (Here is a plug for Bill Vlahos' InfoWallet!) > > Mac: > http://db.tt/RieJv91 > > Windows: > http://db.tt/PEaUKCY > > (sorry, no Linux) > > -- Peter > > Peter M. Brigham > pmbrig at gmail.com > http://home.comcast.net/~pmbrig > > > > _______________________________________________ > use-livecode mailing list > use-livecode at lists.runrev.com > Please visit this url to subscribe, unsubscribe and manage your subscription preferences: > http://lists.runrev.com/mailman/listinfo/use-livecode From bobs at twft.com Tue Jul 12 13:30:13 2011 From: bobs at twft.com (Bob Sneidar) Date: Tue, 12 Jul 2011 10:30:13 -0700 Subject: defining and using globals in an application In-Reply-To: References: Message-ID: Hey now! Those aren't arrays!! Bob On Jul 9, 2011, at 3:05 AM, Francis Nugent Dixon wrote: > Of course, I begin to grab arrays when I have another (Irish) > Whiskey ! > > -Francis From bobs at twft.com Tue Jul 12 13:37:42 2011 From: bobs at twft.com (Bob Sneidar) Date: Tue, 12 Jul 2011 10:37:42 -0700 Subject: global variable change in substack not available to main stack In-Reply-To: <000d01cc3f0d$d3f2ef20$7bd8cd60$@lexiconbridge.com> References: <000601cc3ec6$efa35fd0$ceea1f70$@com> <143663480734.20110710005836@ahsoftware.net> <000d01cc3f0d$d3f2ef20$7bd8cd60$@lexiconbridge.com> Message-ID: There is another place in the message box where you can execute multiple commands. I think you can also execute multiple commands by separating them with a semicolon, but I might be wrong about that. Bob On Jul 10, 2011, at 7:29 AM, Slava Paperno wrote: > I'm curious about globals and the message box. I've been doing both > (omitting the global declaration and including it) and I haven't seen any > difference, but I've seen conflicting opinions in the posts here as well as > some forums. What's the skinny on this? > > Slava > >>> In the Message box: >> >>> put gBgColor >>> --0, 0, 0 >> >> You have to declare the global variable. In the message box try >> >> global gBgColor; put gBgColor > > > _______________________________________________ > use-livecode mailing list > use-livecode at lists.runrev.com > Please visit this url to subscribe, unsubscribe and manage your subscription preferences: > http://lists.runrev.com/mailman/listinfo/use-livecode From bobs at twft.com Tue Jul 12 13:40:07 2011 From: bobs at twft.com (Bob Sneidar) Date: Tue, 12 Jul 2011 10:40:07 -0700 Subject: global variable change in substack not available to main stack In-Reply-To: References: <000601cc3ec6$efa35fd0$ceea1f70$@com> <000001cc3ed4$75c3b210$614b1630$@com> <000f01cc3f10$25cc14a0$71643de0$@lexiconbridge.com> <165694213203.20110710093048@ahsoftware.net> Message-ID: Isn't that a system variable? Bob On Jul 10, 2011, at 1:34 PM, Pete wrote: > Here's another twist on this. The Message Box has the ability to create > "global variables" - click the 4th icon from the left in the toolbar at the > top left of the message bar. > > If you define a global variable with a name that starts with a $ sign, it > seems to be available from any script without declaring it, thus making it > truly global. It also shows up in the Variables tab of the script editor > window. From bobs at twft.com Tue Jul 12 13:43:18 2011 From: bobs at twft.com (Bob Sneidar) Date: Tue, 12 Jul 2011 10:43:18 -0700 Subject: [ANN] AndreGarzia.Com new web page. In-Reply-To: References: Message-ID: <8B578B8A-461C-480B-9465-8804CB4A4BA1@twft.com> Your site must be hacked. All I get is a streaming video of monkeys throwing poo. J/K! Looks great!! Bob On Jul 9, 2011, at 9:05 PM, Andre Garzia wrote: > Hello Folks, > > It is full of joy and pride that I announce my website redesign here. Those > that accessed my home page recently knew that it was not being updated > often, actually, it was not being updated at all. I always wanted to > recreate everything with LiveCode and now I've did it. I started yesterday, > Friday and after a little more than 24h I've got a brand new website. This > is a testimonial of how easy it is to build web stuff with the new LiveCode > Server. > > This new page will serve as a hub for all my new developments and I plan to > release many many many things during the next days and weeks. I welcome > feedback and comments. > > http://andregarzia.com > > =) > > > For those that want to know, this homepage is driven with LiveCode Server. > It uses RevIgniter Framework. It uses no database at all, the pages are all > text files with HTML and special syntax in it. Keeping it simple allowed me > to deliver a full website in less than 24h. The site is easy to extend and > change. Since I am the developer and the user, I don't need an > administration interface, I can create pages by simply dropping plain text > files in the correct place and tweek things by editing special files. > > -- > http://www.andregarzia.com All We Do Is Code. > _______________________________________________ > use-livecode mailing list > use-livecode at lists.runrev.com > Please visit this url to subscribe, unsubscribe and manage your subscription preferences: > http://lists.runrev.com/mailman/listinfo/use-livecode From bobs at twft.com Tue Jul 12 13:49:46 2011 From: bobs at twft.com (Bob Sneidar) Date: Tue, 12 Jul 2011 10:49:46 -0700 Subject: Mac OS X missing controls in the LiveCode IDE In-Reply-To: References: <32257BFE-617E-4FC9-ABD7-92D81E9A60E5@numericable.com> <1EF1F66A-D8F6-4F1B-8B7D-627FAA297D54@numericable.com> Message-ID: One of the very many things I have forgotten from those hazy days. Bob On Jul 12, 2011, at 10:07 AM, stephen barncard wrote: > it's true. I'm almost famous! But Grace only sang in a choir with others. > Jerry Garcia was the biggest influence on the record. > > sqb From bobs at twft.com Tue Jul 12 13:51:35 2011 From: bobs at twft.com (Bob Sneidar) Date: Tue, 12 Jul 2011 10:51:35 -0700 Subject: Rev Customer Databased Hacked? In-Reply-To: <1310396624.47204.YahooMailClassic@web161603.mail.bf1.yahoo.com> References: <1310396624.47204.YahooMailClassic@web161603.mail.bf1.yahoo.com> Message-ID: I did too, but I got the message. Thankfully, all the info I have there is in an SQL database with bogus information. The best counter intel is bad. :-) Bob On Jul 11, 2011, at 8:03 AM, Michael Kann wrote: > Mike, > You make a good point. I bought the lifetime membership so perhaps my info is in a different pile than yours. I was more concerned that any data was obtainable from the outside at all. > Mike From klaus at major.on-rev.com Tue Jul 12 13:52:56 2011 From: klaus at major.on-rev.com (Klaus on-rev) Date: Tue, 12 Jul 2011 19:52:56 +0200 Subject: global variable change in substack not available to main stack In-Reply-To: References: <000601cc3ec6$efa35fd0$ceea1f70$@com> <143663480734.20110710005836@ahsoftware.net> <000d01cc3f0d$d3f2ef20$7bd8cd60$@lexiconbridge.com> Message-ID: <33FE3FA9-CCBD-4B3D-8231-63DDFE8AE295@major.on-rev.com> Hi Bob, Am 12.07.2011 um 19:37 schrieb Bob Sneidar: > There is another place in the message box where you can execute multiple commands. > I think you can also execute multiple commands by separating them with a semicolon, > but I might be wrong about that. you ain't :-) > Bob Best Klaus -- Klaus Major http://www.major-k.de klaus at major.on-rev.com From bobs at twft.com Tue Jul 12 13:58:33 2011 From: bobs at twft.com (Bob Sneidar) Date: Tue, 12 Jul 2011 10:58:33 -0700 Subject: Rev Customer Databased Hacked? In-Reply-To: <02BF4792-C049-4877-8DD2-195CBD251062@free.fr> References: <1310414587230-3660677.post@n4.nabble.com> <02BF4792-C049-4877-8DD2-195CBD251062@free.fr> Message-ID: <3DFDDC34-7EDA-4FC5-A67F-0A1A5DDDF99A@twft.com> For the record, I have a password philosophy that has served me well. I have one set of credentials I use for local logons, like computer accounts and file servers. I have a second set for anything that accesses the internet, but does not contain information that can hurt me. I have a third set that I use for internet accounts where I can be hurt. I NEVER use one set in another environment. I take that back. I DID use a game account in what I thot was a trusted forum on a server I thot was owned by one of the guys in game that I knew for years. Turns out it was a hosted forum, and my game account got hacked within two weeks. Live and learn. Bob On Jul 11, 2011, at 1:37 PM, Pierre Sahores wrote: > I changed all mine, even if they went in theory full safe. It's realy best for all of us to verify that our passwords are at least trusted as 100% safe by the cPanel AJAX tester. Any mix of letters, numbers and itemdels are always more trusty than only letters + numbers ;-) > > Best, Pierre > > D?but du message r?exp?di? : > >> The advice to change password was *not* because of any success by any hacker >> at accessing your (our) password information. >> >> BUT because the hacker now has username and on-rev domain name, *if* you >> have a weak password it would be wise to change it to one that may be harder >> to try and attack by dictionary/brute force, should the hacker try in the >> future to use the list of usernames to find a weak nut to crack. > > -- > Pierre Sahores > mobile : (33) 6 03 95 77 70 > > www.woooooooords.com > www.sahores-conseil.com > > > > > > _______________________________________________ > use-livecode mailing list > use-livecode at lists.runrev.com > Please visit this url to subscribe, unsubscribe and manage your subscription preferences: > http://lists.runrev.com/mailman/listinfo/use-livecode From bobs at twft.com Tue Jul 12 14:00:48 2011 From: bobs at twft.com (Bob Sneidar) Date: Tue, 12 Jul 2011 11:00:48 -0700 Subject: Rev Customer Databased Hacked? In-Reply-To: <4E1C0E52.60907@harbourhosting.co.uk> References: <1E064547-F17F-4887-92CD-1A73766F0113@videotron.ca> <4E1C0E52.60907@harbourhosting.co.uk> Message-ID: <8AB890CF-23F1-4C36-8CB4-48B5E4DC7F9C@twft.com> I tried to upgrade me once. I crashed. Had to flash the bios to the original firmware. It hurt. Bob On Jul 12, 2011, at 2:05 AM, Martin Baxter wrote: > The weakest link is in-between the chair and the keyboard and, > unfortunately, cannot easily be upgraded or patched. ;-) > > Martin Baxter From rene.micout at numericable.com Tue Jul 12 14:04:42 2011 From: rene.micout at numericable.com (=?iso-8859-1?Q?Ren=E9_Micout?=) Date: Tue, 12 Jul 2011 20:04:42 +0200 Subject: Rev Customer Databased Hacked? In-Reply-To: <3DFDDC34-7EDA-4FC5-A67F-0A1A5DDDF99A@twft.com> References: <1310414587230-3660677.post@n4.nabble.com> <02BF4792-C049-4877-8DD2-195CBD251062@free.fr> <3DFDDC34-7EDA-4FC5-A67F-0A1A5DDDF99A@twft.com> Message-ID: <1D098DE5-EB7D-4161-B160-10539803A3E9@numericable.com> Le 12 juil. 2011 ? 19:58, Bob Sneidar a ?crit : > For the record, I have a password philosophy that has served me well. I have one set of credentials I use for local logons, like computer accounts and file servers. I have a second set for anything that accesses the internet, but does not contain information that can hurt me. I have a third set that I use for internet accounts where I can be hurt. I NEVER use one set in another environment. Thank you ! I will follow your example... From keith.clarke at clarkeandclarke.co.uk Tue Jul 12 14:21:05 2011 From: keith.clarke at clarkeandclarke.co.uk (Keith Clarke) Date: Tue, 12 Jul 2011 19:21:05 +0100 Subject: Rev Customer Databased Hacked? In-Reply-To: <3DFDDC34-7EDA-4FC5-A67F-0A1A5DDDF99A@twft.com> References: <1310414587230-3660677.post@n4.nabble.com> <02BF4792-C049-4877-8DD2-195CBD251062@free.fr> <3DFDDC34-7EDA-4FC5-A67F-0A1A5DDDF99A@twft.com> Message-ID: Whilst we're sharing tips, I've been using 1Password from http://agilebits.com/ across my Macs and iPhone devices for a few years, recently adding their PC version for my Parallels virtual PCs (no Linux, except web servers). All my passwords are unique - I use 1Password's inbuilt generator and these are always the longest, most random alpha + numeric + symbol combinations that each service allows. I use 1Password's tagging capabilities to remember what all the services and passwords are for. My encrypted 1Password data file lives in DropBox and so is synchronised across platforms and accessible anywhere. I then have 1 very hard password for 1 Password alone - and I am extremely careful with this. Oh, and I always remember that "There are only two people who can keep a secret - and one of them is dead!" ;-) Best, Keith.. On 12 Jul 2011, at 18:58, Bob Sneidar wrote: > For the record, I have a password philosophy that has served me well. I have one set of credentials I use for local logons, like computer accounts and file servers. I have a second set for anything that accesses the internet, but does not contain information that can hurt me. I have a third set that I use for internet accounts where I can be hurt. I NEVER use one set in another environment. > > I take that back. I DID use a game account in what I thot was a trusted forum on a server I thot was owned by one of the guys in game that I knew for years. Turns out it was a hosted forum, and my game account got hacked within two weeks. Live and learn. > > Bob > > > On Jul 11, 2011, at 1:37 PM, Pierre Sahores wrote: > >> I changed all mine, even if they went in theory full safe. It's realy best for all of us to verify that our passwords are at least trusted as 100% safe by the cPanel AJAX tester. Any mix of letters, numbers and itemdels are always more trusty than only letters + numbers ;-) >> >> Best, Pierre >> >> D?but du message r?exp?di? : >> >>> The advice to change password was *not* because of any success by any hacker >>> at accessing your (our) password information. >>> >>> BUT because the hacker now has username and on-rev domain name, *if* you >>> have a weak password it would be wise to change it to one that may be harder >>> to try and attack by dictionary/brute force, should the hacker try in the >>> future to use the list of usernames to find a weak nut to crack. From pete at mollysrevenge.com Tue Jul 12 14:22:21 2011 From: pete at mollysrevenge.com (Pete) Date: Tue, 12 Jul 2011 11:22:21 -0700 Subject: bugs in union and intersect? In-Reply-To: References: Message-ID: Just catching up with some posts.... It's true that casesensitive only applies to text comparisons, specifically those listed in the dictionary, but the dictionary also claims that keys to arrays and custom property names are affected by the casesensitive setting, which confuses things in my mind. I'd be tempted to put in an enhancement request for this - it seems like union and intersect must do text comparisons under the covers so it's reasonable that they should obey the casesensitive setting. I guess you could implement case sensitive versions of union and intersect with a repeat loop on the keys of one array, testing if it was "among" the lines of the keys of the other array, since "among" does obey the case sensitive setting, something like: command myUnion @parray,ptestarray put the keys of ptestarray into myKeys set the casesensitive to true repeat for each line myKey in myKeys if myKey is not among the lines of the keys of parray then put ptestarray[myKey] into parray[myKey] end if end repeat end myUnion I tried that on Dick's test case and it produced the right results. Pete Molly's Revenge On Tue, Jun 28, 2011 at 6:24 AM, Ken Ray wrote: > > They both fail with caseSensitive true. > > > > on mouseUp > > local a,b,c,d > > set the caseSensitive to "true" > > put 1 into a["a"] > > put 2 into b["b"] > > put 3 into b["B"] > > union a with b > > -- missing a["b"] > > > > put 1 into c["c"] > > put 2 into c["C"] > > put 3 into d["c"] > > intersect c with d > > -- should not have c["C"] > > end mouseUp > > > > > > Confirmation? > > > > Bug report? Enhancement request? Warnings in documentation? > > Confirmed, but caseSensitive is only used for text comparisons (as stated > in > the docs), and the key to an array is apparently not "text" so it has no > effect. So if anything I'd say "warning in documentation"... > > Ken Ray > Sons of Thunder Software, Inc. > Email: kray at sonsothunder.com > Web Site: http://www.sonsothunder.com/ > > > > _______________________________________________ > use-livecode mailing list > use-livecode at lists.runrev.com > Please visit this url to subscribe, unsubscribe and manage your > subscription preferences: > http://lists.runrev.com/mailman/listinfo/use-livecode > > From pete at mollysrevenge.com Tue Jul 12 14:28:25 2011 From: pete at mollysrevenge.com (Pete) Date: Tue, 12 Jul 2011 11:28:25 -0700 Subject: global variable change in substack not available to main stack In-Reply-To: References: <000601cc3ec6$efa35fd0$ceea1f70$@com> <000001cc3ed4$75c3b210$614b1630$@com> <000f01cc3f10$25cc14a0$71643de0$@lexiconbridge.com> <165694213203.20110710093048@ahsoftware.net> Message-ID: Yes it is - which makes available until you log out. Still have to be careful what you use it for between stacks of course but at least you don't have to declare it everywhere it's referenced. Pete Molly's Revenge On Tue, Jul 12, 2011 at 10:40 AM, Bob Sneidar wrote: > Isn't that a system variable? > > Bob > > > On Jul 10, 2011, at 1:34 PM, Pete wrote: > > > Here's another twist on this. The Message Box has the ability to create > > "global variables" - click the 4th icon from the left in the toolbar at > the > > top left of the message bar. > > > > If you define a global variable with a name that starts with a $ sign, it > > seems to be available from any script without declaring it, thus making > it > > truly global. It also shows up in the Variables tab of the script editor > > window. > > > _______________________________________________ > use-livecode mailing list > use-livecode at lists.runrev.com > Please visit this url to subscribe, unsubscribe and manage your > subscription preferences: > http://lists.runrev.com/mailman/listinfo/use-livecode > > From alex at tweedly.net Tue Jul 12 14:28:56 2011 From: alex at tweedly.net (Alex Tweedly) Date: Tue, 12 Jul 2011 19:28:56 +0100 Subject: Rev Customer Databased Hacked? In-Reply-To: <3DFDDC34-7EDA-4FC5-A67F-0A1A5DDDF99A@twft.com> References: <1310414587230-3660677.post@n4.nabble.com> <02BF4792-C049-4877-8DD2-195CBD251062@free.fr> <3DFDDC34-7EDA-4FC5-A67F-0A1A5DDDF99A@twft.com> Message-ID: <4E1C9268.806@tweedly.net> On 12/07/2011 18:58, Bob Sneidar wrote: > For the record, I have a password philosophy that has served me well. I have one set of credentials I use for local logons, like computer accounts and file servers. I have a second set for anything that accesses the internet, but does not contain information that can hurt me. I have a third set that I use for internet accounts where I can be hurt. I NEVER use one set in another environment. That's pretty close to what I do, though I have two levels of "hurt" (general websites that could be compromised, another for banking, credit cards, etc.) I will also willingly share my method for generating easily-remembered passwords. I take - the car license number of a car I used to own (or a friend used to own) - treat it as N groups of letters, or of numbers - change the order of those groups - reverse the order of one of the groups - append or prepend the year I bought that car in (and also add punctuation, but I won't go into details on that). This generates passwords I can easily remember (or easily re-create from what I do remember), but would be difficult for anyone else to find out (and will be generally hard to guess by brute force). It does help that I've owned cars in more than one country :-) -- Alex. From pmbrig at gmail.com Tue Jul 12 18:13:48 2011 From: pmbrig at gmail.com (Peter Brigham MD) Date: Tue, 12 Jul 2011 18:13:48 -0400 Subject: Mac OS X missing controls in the LiveCode IDE In-Reply-To: References: <32257BFE-617E-4FC9-ABD7-92D81E9A60E5@numericable.com> <1EF1F66A-D8F6-4F1B-8B7D-627FAA297D54@numericable.com> Message-ID: <986A2524-6769-4A0F-9A17-5F4B5BC64F0D@gmail.com> At the risk of embarrassing Stephen, here is a link to his older life in music: http://www.allmusic.com/artist/stephen-barncard-p54338/credits -- Peter Peter M. Brigham pmbrig at gmail.com http://home.comcast.net/~pmbrig On Jul 12, 2011, at 1:07 PM, stephen barncard wrote: > it's true. I'm almost famous! But Grace only sang in a choir with others. > Jerry Garcia was the biggest influence on the record. > > sqb > > On 12 July 2011 02:59, Ren? Micout wrote: > >> >> Le 11 juil. 2011 ? 20:11, stephen barncard a ?crit : >> >>> Custom controls are *cool*. One can make any UI gadget work just the way >> one >>> wants and clone hundreds of them and run them from a single behavior >> script. >>> And make a Droptool? to share with others. >> >> >> I work on it... >> >> [OT] > I see your young face on jacket disc "If I Could Only Remenber My >> Name" of David Crosby under Grace Slick >> 1971 > Ah ! Mes vingt ans ! >> >> Bon souvenir de Paris >> Ren? ? twenty years old... in 1971... :-( From pmbrig at gmail.com Tue Jul 12 18:15:16 2011 From: pmbrig at gmail.com (Peter Brigham MD) Date: Tue, 12 Jul 2011 18:15:16 -0400 Subject: defining and using globals in an application In-Reply-To: References: Message-ID: <62F2A4F0-A714-4A7D-9A89-868B603B3956@gmail.com> I've been telling my bosses I could use arrays.... -- Peter Peter M. Brigham pmbrig at gmail.com http://home.comcast.net/~pmbrig On Jul 12, 2011, at 1:30 PM, Bob Sneidar wrote: > Hey now! Those aren't arrays!! > > Bob > > > On Jul 9, 2011, at 3:05 AM, Francis Nugent Dixon wrote: > >> Of course, I begin to grab arrays when I have another (Irish) >> Whiskey ! >> >> -Francis > > > _______________________________________________ > use-livecode mailing list > use-livecode at lists.runrev.com > Please visit this url to subscribe, unsubscribe and manage your subscription preferences: > http://lists.runrev.com/mailman/listinfo/use-livecode From rene.micout at numericable.com Tue Jul 12 18:49:16 2011 From: rene.micout at numericable.com (=?iso-8859-1?Q?Ren=E9_Micout?=) Date: Wed, 13 Jul 2011 00:49:16 +0200 Subject: Mac OS X missing controls in the LiveCode IDE In-Reply-To: <986A2524-6769-4A0F-9A17-5F4B5BC64F0D@gmail.com> References: <32257BFE-617E-4FC9-ABD7-92D81E9A60E5@numericable.com> <1EF1F66A-D8F6-4F1B-8B7D-627FAA297D54@numericable.com> <986A2524-6769-4A0F-9A17-5F4B5BC64F0D@gmail.com> Message-ID: Le 13 juil. 2011 ? 00:13, Peter Brigham MD a ?crit : > At the risk of embarrassing Stephen, here is a link to his older life in music: > > http://www.allmusic.com/artist/stephen-barncard-p54338/credits for me > Http/1.1 Service Unavailable :-( From bobs at twft.com Tue Jul 12 19:06:48 2011 From: bobs at twft.com (Bob Sneidar) Date: Tue, 12 Jul 2011 16:06:48 -0700 Subject: Mac OS X missing controls in the LiveCode IDE In-Reply-To: <986A2524-6769-4A0F-9A17-5F4B5BC64F0D@gmail.com> References: <32257BFE-617E-4FC9-ABD7-92D81E9A60E5@numericable.com> <1EF1F66A-D8F6-4F1B-8B7D-627FAA297D54@numericable.com> <986A2524-6769-4A0F-9A17-5F4B5BC64F0D@gmail.com> Message-ID: <3FB632A0-579D-4964-BB12-094EA94D36D7@twft.com> I wonder if the CSN guys ever look up Richie Furay. I hear Stephen has kept in touch. Richie is pastoring a Calvary Chapel in Colorado, Broomfield I think. I've seen Richie at a couple of Pastor's conferences out here in Costa Mesa CA, but I never tried to meet him. I bet everyone who knows who he is does that. I really love all the music from that crowd. Bob On Jul 12, 2011, at 3:13 PM, Peter Brigham MD wrote: > At the risk of embarrassing Stephen, here is a link to his older life in music: > > http://www.allmusic.com/artist/stephen-barncard-p54338/credits > > -- Peter > > Peter M. Brigham > pmbrig at gmail.com > http://home.comcast.net/~pmbrig > > > On Jul 12, 2011, at 1:07 PM, stephen barncard wrote: > >> it's true. I'm almost famous! But Grace only sang in a choir with others. >> Jerry Garcia was the biggest influence on the record. >> >> sqb >> >> On 12 July 2011 02:59, Ren? Micout wrote: >> >>> >>> Le 11 juil. 2011 ? 20:11, stephen barncard a ?crit : >>> >>>> Custom controls are *cool*. One can make any UI gadget work just the way >>> one >>>> wants and clone hundreds of them and run them from a single behavior >>> script. >>>> And make a Droptool? to share with others. >>> >>> >>> I work on it... >>> >>> [OT] > I see your young face on jacket disc "If I Could Only Remenber My >>> Name" of David Crosby under Grace Slick >>> 1971 > Ah ! Mes vingt ans ! >>> >>> Bon souvenir de Paris >>> Ren? ? twenty years old... in 1971... :-( > > > _______________________________________________ > use-livecode mailing list > use-livecode at lists.runrev.com > Please visit this url to subscribe, unsubscribe and manage your subscription preferences: > http://lists.runrev.com/mailman/listinfo/use-livecode From andre at andregarzia.com Tue Jul 12 21:38:08 2011 From: andre at andregarzia.com (Andre Garzia) Date: Tue, 12 Jul 2011 22:38:08 -0300 Subject: what is the last LiveCode version to run on G5? Message-ID: Hey Folks, I am not keeping track of the universality of LiveCode engines, what is the latest version able to run on a G5 machine? Cheers andre -- http://www.andregarzia.com All We Do Is Code. From jhj at jhj.com Tue Jul 12 21:50:49 2011 From: jhj at jhj.com (Jerry J) Date: Tue, 12 Jul 2011 18:50:49 -0700 Subject: what is the last LiveCode version to run on G5? In-Reply-To: References: Message-ID: <417098CC-9DDE-49F7-AFA1-D5AAD8425C2D@jhj.com> I am running 4.6.2 on a G3 PowerBook Pismo which works just fine! Out in a blockhouse in a field, unattended, taking data and sending it home via satellite. --Jerry Jensen On Jul 12, 2011, at 6:38 PM, Andre Garzia wrote: > Hey Folks, > > I am not keeping track of the universality of LiveCode engines, what is the > latest version able to run on a G5 machine? > > Cheers > andre > > -- > http://www.andregarzia.com All We Do Is Code. > _______________________________________________ > use-livecode mailing list > use-livecode at lists.runrev.com > Please visit this url to subscribe, unsubscribe and manage your subscription preferences: > http://lists.runrev.com/mailman/listinfo/use-livecode From stephenREVOLUTION2 at barncard.com Tue Jul 12 21:52:36 2011 From: stephenREVOLUTION2 at barncard.com (stephen barncard) Date: Tue, 12 Jul 2011 18:52:36 -0700 Subject: what is the last LiveCode version to run on G5? In-Reply-To: References: Message-ID: Unless something has changed, I think the package is still uiniversal. My main machine was a G5 up until a couple of months ago an Livecode ran fine.... On 12 July 2011 18:38, Andre Garzia wrote: > Hey Folks, > > I am not keeping track of the universality of LiveCode engines, what is the > latest version able to run on a G5 machine? > > Cheers > andre > > Stephen Barncard San Francisco Ca. USA more about sqb From andre at andregarzia.com Tue Jul 12 22:18:39 2011 From: andre at andregarzia.com (Andre Garzia) Date: Tue, 12 Jul 2011 23:18:39 -0300 Subject: what is the last LiveCode version to run on G5? In-Reply-To: References: Message-ID: oh great!!!! A friend of a friend here is selling a dual core G5... I am thinking about snatching it, I have a long love for PPC machines, my favorite laptop for work is my G4 even though I own a 2009 macbook pro, this intel machines fell a little souless to me. My previous intel machine is giving signs of breaking, the intel machine before that broke twice... My two G4 machines are alive and kicking, my iMac G4 works great.... I am thinking about picking this dual G5... :-) PS: this is specially interesting because I don't like Lion so I will be staying with snow leopard/leopard for a while.... On Tue, Jul 12, 2011 at 10:52 PM, stephen barncard < stephenREVOLUTION2 at barncard.com> wrote: > Unless something has changed, I think the package is still uiniversal. My > main machine was a G5 up until a couple of months ago an Livecode ran > fine.... > > On 12 July 2011 18:38, Andre Garzia wrote: > > > Hey Folks, > > > > I am not keeping track of the universality of LiveCode engines, what is > the > > latest version able to run on a G5 machine? > > > > Cheers > > andre > > > > > > > Stephen Barncard > San Francisco Ca. USA > > more about sqb > _______________________________________________ > use-livecode mailing list > use-livecode at lists.runrev.com > Please visit this url to subscribe, unsubscribe and manage your > subscription preferences: > http://lists.runrev.com/mailman/listinfo/use-livecode > -- http://www.andregarzia.com All We Do Is Code. From shaosean at wehostmacs.com Tue Jul 12 22:34:51 2011 From: shaosean at wehostmacs.com (Shao Sean) Date: Tue, 12 Jul 2011 22:34:51 -0400 Subject: what is the last LiveCode version to run on G5? Message-ID: The release notes for 4.5.0 stated: "The supported platforms that will be supported in 5.0 is currently being reviewed. However, it is likely that the following will be the case: ? The engine will no longer support Mac OS 10.3.9 (Panther) " but this information is not in the 4.6.x release notes.. From pete at mollysrevenge.com Wed Jul 13 00:11:03 2011 From: pete at mollysrevenge.com (Pete) Date: Tue, 12 Jul 2011 21:11:03 -0700 Subject: Resizing the objects in a group In-Reply-To: References: Message-ID: It seems that this problem is not restricted to when there is just one control in a group. If I create a group with any number of controls in it then try to resize one of the controls, the group does not expand to accommodate the new control size. Moving a control resizes the group, resizing it does not. If I change the size of a control in the group by script, the group does expand accordingly. The lockLocation property of the group is false. Is there some other property of the group I need to set/unset to allow resizing of a control? I fnot, I plan to report this as a bug. Pete Molly's Revenge On Sun, Jul 10, 2011 at 5:55 PM, Pete wrote: > According to the dictionary: "If the *lockLocation* of a group is false, > the group is automatically resized when the objects in it are moved, > resized, hidden, or shown." > > I have a group that has just one control in it - a datagrid - and its > lockloc is false. If I resize the datagrid by dragging its handles, the > group does not resize and the parts of the datagrid that are outside the > group are no longer visible. > > With some experimentation, this seems to happen with any group that has > only one control in it, no matter what type of control it is. I know it's > not normal to have a group with just one object in it but any ideas why this > might be happening? > > Pete > Molly's Revenge > > > From mwieder at ahsoftware.net Wed Jul 13 00:22:52 2011 From: mwieder at ahsoftware.net (Mark Wieder) Date: Tue, 12 Jul 2011 21:22:52 -0700 Subject: what is the last LiveCode version to run on G5? In-Reply-To: References: Message-ID: <147909737093.20110712212252@ahsoftware.net> Sean- Tuesday, July 12, 2011, 7:34:51 PM, you wrote: > ? The engine will no longer support Mac OS 10.3.9 (Panther) " > but this information is not in the 4.6.x release notes.. Look in the release notes under "Noteworthy changes" in the Maintenance section: "Mac OS 10.3.9 is no longer supported Support has been dropped for Mac OS 10.3.9." -- -Mark Wieder mwieder at ahsoftware.net From jacque at hyperactivesw.com Wed Jul 13 00:35:26 2011 From: jacque at hyperactivesw.com (J. Landman Gay) Date: Tue, 12 Jul 2011 23:35:26 -0500 Subject: Resizing the objects in a group In-Reply-To: References: Message-ID: <4E1D208E.5040901@hyperactivesw.com> On 7/12/11 11:11 PM, Pete wrote: > It seems that this problem is not restricted to when there is just one > control in a group. If I create a group with any number of controls in it > then try to resize one of the controls, the group does not expand to > accommodate the new control size. Moving a control resizes the > group, resizing it does not. If I change the size of a control in the group > by script, the group does expand accordingly. > > The lockLocation property of the group is false. Is there some other > property of the group I need to set/unset to allow resizing of a control? I > fnot, I plan to report this as a bug. I'm not sure whether it's a bug or not, but the behavior has been there since the engine was MetaCard. It may be partly a byproduct of the boundingrect property. I've gotten so used to it I don't even think about it any more; if I want a group to expand to fit the objects, I just click twice on Edit Group in the toolbar. That pops me in and out of edit mode quickly, and when coming back out of edit mode, the group adjusts to fit the objects. Any kind of redraw will do the same thing, like going to the next card and back. -- Jacqueline Landman Gay | jacque at hyperactivesw.com HyperActive Software | http://www.hyperactivesw.com From chipp at chipp.com Wed Jul 13 00:37:14 2011 From: chipp at chipp.com (Chipp Walters) Date: Tue, 12 Jul 2011 23:37:14 -0500 Subject: Resizing the objects in a group In-Reply-To: References: Message-ID: if you edit the group, then close it, it will auto resize to the bounds of all the controls-- assuming the group's lockloc prop is not true. On Tue, Jul 12, 2011 at 11:11 PM, Pete wrote: > It seems that this problem is not restricted to when there is just one > control in a group. If I create a group with any number of controls in it > then try to resize one of the controls, the group does not expand to > accommodate the new control size. Moving a control resizes the > group, resizing it does not. If I change the size of a control in the > group > by script, the group does expand accordingly. > > The lockLocation property of the group is false. Is there some other > property of the group I need to set/unset to allow resizing of a control? > I > fnot, I plan to report this as a bug. > > Pete > Molly's Revenge > > > > > On Sun, Jul 10, 2011 at 5:55 PM, Pete wrote: > > > According to the dictionary: "If the *lockLocation* of a group is false, > > the group is automatically resized when the objects in it are moved, > > resized, hidden, or shown." > > > > I have a group that has just one control in it - a datagrid - and its > > lockloc is false. If I resize the datagrid by dragging its handles, the > > group does not resize and the parts of the datagrid that are outside the > > group are no longer visible. > > > > With some experimentation, this seems to happen with any group that has > > only one control in it, no matter what type of control it is. I know it's > > not normal to have a group with just one object in it but any ideas why > this > > might be happening? > > > > Pete > > Molly's Revenge > > > > > > > _______________________________________________ > use-livecode mailing list > use-livecode at lists.runrev.com > Please visit this url to subscribe, unsubscribe and manage your > subscription preferences: > http://lists.runrev.com/mailman/listinfo/use-livecode > -- Chipp Walters CEO, Shafer Walters Group, Inc. From pete at mollysrevenge.com Wed Jul 13 02:01:42 2011 From: pete at mollysrevenge.com (Pete) Date: Tue, 12 Jul 2011 23:01:42 -0700 Subject: Resizing the objects in a group In-Reply-To: <4E1D208E.5040901@hyperactivesw.com> References: <4E1D208E.5040901@hyperactivesw.com> Message-ID: Thanks guys. The double click on Edit Group seems to be the easiest way around this. I do believe it is a bug irrespective of how long it's been around. Not necessarily a code bug - maybe just fixing the dictionary entry bug would make it right. I'll enter something at the QC just so it's on record. Pete Molly's Revenge On Tue, Jul 12, 2011 at 9:35 PM, J. Landman Gay wrote: > On 7/12/11 11:11 PM, Pete wrote: > >> It seems that this problem is not restricted to when there is just one >> control in a group. If I create a group with any number of controls in it >> then try to resize one of the controls, the group does not expand to >> accommodate the new control size. Moving a control resizes the >> group, resizing it does not. If I change the size of a control in the >> group >> by script, the group does expand accordingly. >> >> The lockLocation property of the group is false. Is there some other >> property of the group I need to set/unset to allow resizing of a control? >> I >> fnot, I plan to report this as a bug. >> > > I'm not sure whether it's a bug or not, but the behavior has been there > since the engine was MetaCard. It may be partly a byproduct of the > boundingrect property. I've gotten so used to it I don't even think about it > any more; if I want a group to expand to fit the objects, I just click twice > on Edit Group in the toolbar. That pops me in and out of edit mode quickly, > and when coming back out of edit mode, the group adjusts to fit the objects. > Any kind of redraw will do the same thing, like going to the next card and > back. > > -- > Jacqueline Landman Gay | jacque at hyperactivesw.com > HyperActive Software | http://www.hyperactivesw.com > > ______________________________**_________________ > use-livecode mailing list > use-livecode at lists.runrev.com > Please visit this url to subscribe, unsubscribe and manage your > subscription preferences: > http://lists.runrev.com/**mailman/listinfo/use-livecode > > From toolbook at kestner.de Wed Jul 13 03:37:13 2011 From: toolbook at kestner.de (Tiemo Hollmann TB) Date: Wed, 13 Jul 2011 09:37:13 +0200 Subject: AW: why don't I get mouse events with brush tool? In-Reply-To: <001301cc4076$c3136ad0$493a4070$@de> References: <001301cc4076$c3136ad0$493a4070$@de> Message-ID: <000c01cc412f$b2e2d420$18a87c60$@de> Just FYI After having run against the front door without success, I now followed Bj?rnkes approach to recreate the paint tool with a drag loop and here it was easy to implement my undo function: on mouseDown xStoreUndo -- mybackup handler put the mouseLoc into tOldLoc choose brush tool repeat while the mouse is down wait with messages drag from tOldLoc to the mouseLoc put the mouseLoc into tOldLoc end repeat choose browse tool pass mouseDown end mouseDown but the question remains. Is it an unimplemented feature to capture the mouse events also with the paint tools, or is there any deeper sense behind the sceens not to do it? Tiemo > -----Urspr?ngliche Nachricht----- > Von: use-livecode-bounces at lists.runrev.com [mailto:use-livecode- > bounces at lists.runrev.com] Im Auftrag von Tiemo Hollmann TB > Gesendet: Dienstag, 12. Juli 2011 11:33 > An: 'How to use LiveCode' > Betreff: why don't I get mouse events with brush tool? > > Hello, > > some days ago Bj?rnke taught me that it?s not a bug but a feature that I > can?t pick the mousecolor while having the brush tool activated. > > Now I am experiencing, that I also don?t get even a mouseDown or mouseUp > with the brush tool. The dictionary doesn?t says anything about this > behavior. What I want to do is: > > The user can paint with the brush tool on an image. To supply the user an > undo with several states, I wanted to make a backup of the image each time > at mouseDown, so that the user can roll back every newly painted state. > > I tried to choose the brush tool with mouseEnter and make the backup with > mouseDown (I don?t get the mouseDown event). > > I tried to make the backup with mouseDown and choose brush tool afterwards > (works only once until the brush tool is choosen in the first mouseDown) > > I tried to switch back to the browse tool with every mouseUp to make the > backup with next mouseDown (I don?t get the mouseUp event, when having > selected the brush tool) > > Probably I am again too narrow minded and looking into the wrong direction, > can anybody point me into another direction, how I can realize my undo > function? > > Thanks > > Tiemo > > > > > > _______________________________________________ > use-livecode mailing list > use-livecode at lists.runrev.com > Please visit this url to subscribe, unsubscribe and manage your subscription > preferences: > http://lists.runrev.com/mailman/listinfo/use-livecode From andre at andregarzia.com Wed Jul 13 03:47:18 2011 From: andre at andregarzia.com (Andre Garzia) Date: Wed, 13 Jul 2011 04:47:18 -0300 Subject: New post detailing the technology and decisions behind my new website Message-ID: Folks, The blog is now live and has its first useful post. http://andregarzia.com/blog You can see a little journal on the decisions behind the system that drives my new website by going to: http://andregarzia.com/page/bootstrappingacms It is a long read but it is useful for those trying to understand. Hope you guys like, any feedback is appreciated. Cheers andre -- http://www.andregarzia.com All We Do Is Code. From rene.micout at numericable.com Wed Jul 13 03:55:08 2011 From: rene.micout at numericable.com (=?iso-8859-1?Q?Ren=E9_Micout?=) Date: Wed, 13 Jul 2011 09:55:08 +0200 Subject: New post detailing the technology and decisions behind my new website In-Reply-To: References: Message-ID: <06A0907C-A3FD-40F7-B398-60B9A6F3987B@numericable.com> Wow !! Impressive ! Le 13 juil. 2011 ? 09:47, Andre Garzia a ?crit : > Folks, > > The blog is now live and has its first useful post. > > http://andregarzia.com/blog > > You can see a little journal on the decisions behind the system that drives > my new website by going to: > > http://andregarzia.com/page/bootstrappingacms > > It is a long read but it is useful for those trying to understand. > > Hope you guys like, any feedback is appreciated. > > Cheers > andre > > -- > http://www.andregarzia.com All We Do Is Code. > _______________________________________________ > use-livecode mailing list > use-livecode at lists.runrev.com > Please visit this url to subscribe, unsubscribe and manage your subscription preferences: > http://lists.runrev.com/mailman/listinfo/use-livecode From andre at andregarzia.com Wed Jul 13 04:00:41 2011 From: andre at andregarzia.com (Andre Garzia) Date: Wed, 13 Jul 2011 05:00:41 -0300 Subject: New post detailing the technology and decisions behind my new website In-Reply-To: <06A0907C-A3FD-40F7-B398-60B9A6F3987B@numericable.com> References: <06A0907C-A3FD-40F7-B398-60B9A6F3987B@numericable.com> Message-ID: On Wed, Jul 13, 2011 at 4:55 AM, Ren? Micout wrote: > Wow !! Impressive ! > If you read it then please give me some feedback! :-) > > Le 13 juil. 2011 ? 09:47, Andre Garzia a ?crit : > > > Folks, > > > > The blog is now live and has its first useful post. > > > > http://andregarzia.com/blog > > > > You can see a little journal on the decisions behind the system that > drives > > my new website by going to: > > > > http://andregarzia.com/page/bootstrappingacms > > > > It is a long read but it is useful for those trying to understand. > > > > Hope you guys like, any feedback is appreciated. > > > > Cheers > > andre > > > > -- > > http://www.andregarzia.com All We Do Is Code. > > _______________________________________________ > > use-livecode mailing list > > use-livecode at lists.runrev.com > > Please visit this url to subscribe, unsubscribe and manage your > subscription preferences: > > http://lists.runrev.com/mailman/listinfo/use-livecode > > > _______________________________________________ > use-livecode mailing list > use-livecode at lists.runrev.com > Please visit this url to subscribe, unsubscribe and manage your > subscription preferences: > http://lists.runrev.com/mailman/listinfo/use-livecode > -- http://www.andregarzia.com All We Do Is Code. From rene.micout at numericable.com Wed Jul 13 04:19:31 2011 From: rene.micout at numericable.com (=?iso-8859-1?Q?Ren=E9_Micout?=) Date: Wed, 13 Jul 2011 10:19:31 +0200 Subject: New post detailing the technology and decisions behind my new website In-Reply-To: References: <06A0907C-A3FD-40F7-B398-60B9A6F3987B@numericable.com> Message-ID: <449D7446-1192-4DB8-A14D-9C8BCC8B93D4@numericable.com> Hello Andre, Your work is not mine (music / interface) and I do not have enough time to read closely (and translate in French to understand all the subtleties of about). I just wanted to express my admiration for the work even if I do not understand the very substance (la substantifique mo?lle disait Rabelais)... Bravo (again !) Bon souvenir de Paris Ren? Le 13 juil. 2011 ? 10:00, Andre Garzia a ?crit : > On Wed, Jul 13, 2011 at 4:55 AM, Ren? Micout wrote: > >> Wow !! Impressive ! >> > > If you read it then please give me some feedback! :-) > > > >> >> Le 13 juil. 2011 ? 09:47, Andre Garzia a ?crit : >> >>> Folks, >>> >>> The blog is now live and has its first useful post. >>> >>> http://andregarzia.com/blog >>> >>> You can see a little journal on the decisions behind the system that >> drives >>> my new website by going to: >>> >>> http://andregarzia.com/page/bootstrappingacms >>> >>> It is a long read but it is useful for those trying to understand. >>> >>> Hope you guys like, any feedback is appreciated. >>> >>> Cheers >>> andre >>> >>> -- >>> http://www.andregarzia.com All We Do Is Code. >>> _______________________________________________ >>> use-livecode mailing list >>> use-livecode at lists.runrev.com >>> Please visit this url to subscribe, unsubscribe and manage your >> subscription preferences: >>> http://lists.runrev.com/mailman/listinfo/use-livecode >> >> >> _______________________________________________ >> use-livecode mailing list >> use-livecode at lists.runrev.com >> Please visit this url to subscribe, unsubscribe and manage your >> subscription preferences: >> http://lists.runrev.com/mailman/listinfo/use-livecode >> > > > > -- > http://www.andregarzia.com All We Do Is Code. > _______________________________________________ > use-livecode mailing list > use-livecode at lists.runrev.com > Please visit this url to subscribe, unsubscribe and manage your subscription preferences: > http://lists.runrev.com/mailman/listinfo/use-livecode From andre at andregarzia.com Wed Jul 13 04:27:16 2011 From: andre at andregarzia.com (Andre Garzia) Date: Wed, 13 Jul 2011 05:27:16 -0300 Subject: New post detailing the technology and decisions behind my new website In-Reply-To: <449D7446-1192-4DB8-A14D-9C8BCC8B93D4@numericable.com> References: <06A0907C-A3FD-40F7-B398-60B9A6F3987B@numericable.com> <449D7446-1192-4DB8-A14D-9C8BCC8B93D4@numericable.com> Message-ID: On Wed, Jul 13, 2011 at 5:19 AM, Ren? Micout wrote: > Hello Andre, > Your work is not mine (music / interface) and I do not have enough time to > read closely (and translate in French to understand all the subtleties of > about). > I just wanted to express my admiration for the work even if I do not > understand the very substance (la substantifique mo?lle disait Rabelais)... > Bravo (again !) > Bon souvenir de Paris > Ren? > > Thanks for the kind words Ren?!!! I am just giving back to the community that has given me so much! :-) Bonsoir my friend PS: Its 5:26 AM in here... off to bed! :-) > Le 13 juil. 2011 ? 10:00, Andre Garzia a ?crit : > > > On Wed, Jul 13, 2011 at 4:55 AM, Ren? Micout < > rene.micout at numericable.com>wrote: > > > >> Wow !! Impressive ! > >> > > > > If you read it then please give me some feedback! :-) > > > > > > > >> > >> Le 13 juil. 2011 ? 09:47, Andre Garzia a ?crit : > >> > >>> Folks, > >>> > >>> The blog is now live and has its first useful post. > >>> > >>> http://andregarzia.com/blog > >>> > >>> You can see a little journal on the decisions behind the system that > >> drives > >>> my new website by going to: > >>> > >>> http://andregarzia.com/page/bootstrappingacms > >>> > >>> It is a long read but it is useful for those trying to understand. > >>> > >>> Hope you guys like, any feedback is appreciated. > >>> > >>> Cheers > >>> andre > >>> > >>> -- > >>> http://www.andregarzia.com All We Do Is Code. > >>> _______________________________________________ > >>> use-livecode mailing list > >>> use-livecode at lists.runrev.com > >>> Please visit this url to subscribe, unsubscribe and manage your > >> subscription preferences: > >>> http://lists.runrev.com/mailman/listinfo/use-livecode > >> > >> > >> _______________________________________________ > >> use-livecode mailing list > >> use-livecode at lists.runrev.com > >> Please visit this url to subscribe, unsubscribe and manage your > >> subscription preferences: > >> http://lists.runrev.com/mailman/listinfo/use-livecode > >> > > > > > > > > -- > > http://www.andregarzia.com All We Do Is Code. > > _______________________________________________ > > use-livecode mailing list > > use-livecode at lists.runrev.com > > Please visit this url to subscribe, unsubscribe and manage your > subscription preferences: > > http://lists.runrev.com/mailman/listinfo/use-livecode > > > _______________________________________________ > use-livecode mailing list > use-livecode at lists.runrev.com > Please visit this url to subscribe, unsubscribe and manage your > subscription preferences: > http://lists.runrev.com/mailman/listinfo/use-livecode > -- http://www.andregarzia.com All We Do Is Code. From rene.micout at numericable.com Wed Jul 13 06:12:56 2011 From: rene.micout at numericable.com (=?iso-8859-1?Q?Ren=E9_Micout?=) Date: Wed, 13 Jul 2011 12:12:56 +0200 Subject: New post detailing the technology and decisions behind my new website In-Reply-To: References: <06A0907C-A3FD-40F7-B398-60B9A6F3987B@numericable.com> <449D7446-1192-4DB8-A14D-9C8BCC8B93D4@numericable.com> Message-ID: <26732886-A3FE-4555-A5D8-61048A42014C@numericable.com> Le 13 juil. 2011 ? 10:27, Andre Garzia a ?crit : > I am just giving back to the community that has given me so much! :-) It is also my case... I try to make it a little with my modest tools on RevOnLine. > Bonsoir my friend Bonne nuit (si je puis dire !) it is noon now in Paris... > PS: Its 5:26 AM in here... off to bed! :-) What health !! (what else ?) ;-) From keith.clarke at clarkeandclarke.co.uk Wed Jul 13 06:46:03 2011 From: keith.clarke at clarkeandclarke.co.uk (Keith Clarke) Date: Wed, 13 Jul 2011 11:46:03 +0100 Subject: New post detailing the technology and decisions behind my new website In-Reply-To: References: Message-ID: Thanks for the 'How-to' article Andre - great insights for developing lightweight CMS features. ...and you have mail with some further feedback. Best, Keith.. On 13 Jul 2011, at 08:47, Andre Garzia wrote: > Folks, > > The blog is now live and has its first useful post. > > http://andregarzia.com/blog > > You can see a little journal on the decisions behind the system that drives > my new website by going to: > > http://andregarzia.com/page/bootstrappingacms > > It is a long read but it is useful for those trying to understand. > > Hope you guys like, any feedback is appreciated. > > Cheers > andre > > -- > http://www.andregarzia.com All We Do Is Code. > _______________________________________________ > use-livecode mailing list > use-livecode at lists.runrev.com > Please visit this url to subscribe, unsubscribe and manage your subscription preferences: > http://lists.runrev.com/mailman/listinfo/use-livecode From henshaw at me.com Wed Jul 13 07:17:38 2011 From: henshaw at me.com (Andy Henshaw) Date: Wed, 13 Jul 2011 12:17:38 +0100 Subject: Approved :) - iOS App upgrade from just iPhone to both iPhone and iPad Message-ID: <05785487-7E73-4D18-A3DF-C04814A93CB6@me.com> Just got the email from Apple to say the update to my Livecode app 'Tracker2Go' has been approved. I wouldn't normally post this, but I thought I would as its the first I've seen where the original '1.0' app was just for iPhone, and the '1.1' version is for iPhone and iPad. Beyond the changes in Livecode to include the iPad (i.e. create new high resolution iPad screens and link the code etc, then compile to include the iPad), as far as the submission went it was all very simple. I just created a new version, uploaded iPad screenshots to that version then uploaded the binary to Apple. Once approved it just popped into the iTunes store in both the iPad and iPhone section, so it is possible to work for one platform, then upgrade the app so it works on both. Since release Ive found a few bugs, so 1.1.1 is already 'waiting for review' and I'm sure 1.1.2 will follow shortly after that! Andy From coiin at verizon.net Wed Jul 13 07:30:39 2011 From: coiin at verizon.net (Colin Holgate) Date: Wed, 13 Jul 2011 07:30:39 -0400 Subject: Approved :) - iOS App upgrade from just iPhone to both iPhone and iPad In-Reply-To: <05785487-7E73-4D18-A3DF-C04814A93CB6@me.com> References: <05785487-7E73-4D18-A3DF-C04814A93CB6@me.com> Message-ID: <6B91F8BE-692A-484E-BD51-CB2081592388@verizon.net> They like that a lot more than if you tried to go from supporting a device to not supporting it anymore. On Jul 13, 2011, at 7:17 AM, Andy Henshaw wrote: > >I wouldn't normally post this, but I thought I would as its the first I've seen where the original '1.0' app was just for iPhone, and the '1.1' version is for iPhone and iPad. From capellan2000 at gmail.com Wed Jul 13 10:05:26 2011 From: capellan2000 at gmail.com (Alejandro Tejada) Date: Wed, 13 Jul 2011 10:05:26 -0400 Subject: New post detailing the technology and decisions behind my new website Message-ID: Hi Andre, Many thanks for the useful insights in your decision process! I like that you had chosen time as a constrain for the creation of your new website. For many years, I have been educating my graphic design clients about the two main constrains in their projects: Scheduled time and assigned budget. Keep Up your great work! :-D Al on Wed, 13 Jul 2011 04:47:18 -0300 Andre Garzia wrote: > The blog is now live and has its first useful post. > You can see a little journal on the decisions behind the system that drives > my new website. > It is a long read but it is useful for those trying to understand. > Hope you guys like, any feedback is appreciated. From bobs at twft.com Wed Jul 13 11:51:06 2011 From: bobs at twft.com (Bob Sneidar) Date: Wed, 13 Jul 2011 08:51:06 -0700 Subject: what is the last LiveCode version to run on G5? In-Reply-To: References: Message-ID: Stable little things aren't they? Bob On Jul 12, 2011, at 7:18 PM, Andre Garzia wrote: > oh great!!!! > > A friend of a friend here is selling a dual core G5... I am thinking about > snatching it, I have a long love for PPC machines, my favorite laptop for > work is my G4 even though I own a 2009 macbook pro, this intel machines fell > a little souless to me. My previous intel machine is giving signs of > breaking, the intel machine before that broke twice... My two G4 machines > are alive and kicking, my iMac G4 works great.... I am thinking about > picking this dual G5... :-) > > PS: this is specially interesting because I don't like Lion so I will be > staying with snow leopard/leopard for a while.... > > On Tue, Jul 12, 2011 at 10:52 PM, stephen barncard < > stephenREVOLUTION2 at barncard.com> wrote: > >> Unless something has changed, I think the package is still uiniversal. My >> main machine was a G5 up until a couple of months ago an Livecode ran >> fine.... >> >> On 12 July 2011 18:38, Andre Garzia wrote: >> >>> Hey Folks, >>> >>> I am not keeping track of the universality of LiveCode engines, what is >> the >>> latest version able to run on a G5 machine? >>> >>> Cheers >>> andre >>> >>> >> >> >> Stephen Barncard >> San Francisco Ca. USA >> >> more about sqb >> _______________________________________________ >> use-livecode mailing list >> use-livecode at lists.runrev.com >> Please visit this url to subscribe, unsubscribe and manage your >> subscription preferences: >> http://lists.runrev.com/mailman/listinfo/use-livecode >> > > > > -- > http://www.andregarzia.com All We Do Is Code. > _______________________________________________ > use-livecode mailing list > use-livecode at lists.runrev.com > Please visit this url to subscribe, unsubscribe and manage your subscription preferences: > http://lists.runrev.com/mailman/listinfo/use-livecode From stephenREVOLUTION2 at barncard.com Wed Jul 13 13:18:06 2011 From: stephenREVOLUTION2 at barncard.com (stephen barncard) Date: Wed, 13 Jul 2011 10:18:06 -0700 Subject: what is the last LiveCode version to run on G5? In-Reply-To: References: Message-ID: I'm keeping my G5 for a long time... yeah, Panther was a lot of cats ago... On 13 July 2011 08:51, Bob Sneidar wrote: > Stable little things aren't they? > > Bob > > > On Jul 12, 2011, at 7:18 PM, Andre Garzia wrote: > > > oh great!!!! > > > > A friend of a friend here is selling a dual core G5... I am thinking > about > > snatching it, I have a long love for PPC machines, my favorite laptop for > Stephen Barncard San Francisco Ca. USA more about sqb From andre at andregarzia.com Wed Jul 13 13:18:13 2011 From: andre at andregarzia.com (Andre Garzia) Date: Wed, 13 Jul 2011 14:18:13 -0300 Subject: New post detailing the technology and decisions behind my new website In-Reply-To: References: Message-ID: Folks, I've just added comments to the blog system by using Disqus.com I think it works and it is easier than writting my own. :-) From andre at andregarzia.com Wed Jul 13 13:25:02 2011 From: andre at andregarzia.com (Andre Garzia) Date: Wed, 13 Jul 2011 14:25:02 -0300 Subject: what is the last LiveCode version to run on G5? In-Reply-To: References: Message-ID: On Wed, Jul 13, 2011 at 2:18 PM, stephen barncard < stephenREVOLUTION2 at barncard.com> wrote: > I'm keeping my G5 for a long time... > yeah, Panther was a lot of cats ago... > Hey I had a working G3 and a Newton until last year when someone sat on my newton and someone moved my G3 to a damp place and it destroyed the screen... my G4s are working really well... I miss how well built those things were, they just seem to last longer than the current intel machines. > > On 13 July 2011 08:51, Bob Sneidar wrote: > > > Stable little things aren't they? > > > > Bob > > > > > > On Jul 12, 2011, at 7:18 PM, Andre Garzia wrote: > > > > > oh great!!!! > > > > > > A friend of a friend here is selling a dual core G5... I am thinking > > about > > > snatching it, I have a long love for PPC machines, my favorite laptop > for > > > > Stephen Barncard > San Francisco Ca. USA > > more about sqb > _______________________________________________ > use-livecode mailing list > use-livecode at lists.runrev.com > Please visit this url to subscribe, unsubscribe and manage your > subscription preferences: > http://lists.runrev.com/mailman/listinfo/use-livecode > -- http://www.andregarzia.com All We Do Is Code. From psahores at free.fr Wed Jul 13 13:42:21 2011 From: psahores at free.fr (Pierre Sahores) Date: Wed, 13 Jul 2011 19:42:21 +0200 Subject: New post detailing the technology and decisions behind my new website In-Reply-To: References: Message-ID: <41412C55-2CDB-4209-8698-C219CD5C776A@free.fr> Well done ! Will probably follow you in adding it (as an option) to the widestep.fr platform. ;-) Le 13 juil. 2011 ? 19:18, Andre Garzia a ?crit : > Folks, > > I've just added comments to the blog system by using Disqus.com I think it > works and it is easier than writting my own. > > :-) > _______________________________________________ > use-livecode mailing list > use-livecode at lists.runrev.com > Please visit this url to subscribe, unsubscribe and manage your subscription preferences: > http://lists.runrev.com/mailman/listinfo/use-livecode > -- Pierre Sahores mobile : (33) 6 03 95 77 70 www.woooooooords.com www.sahores-conseil.com From keith.clarke at clarkeandclarke.co.uk Wed Jul 13 13:47:05 2011 From: keith.clarke at clarkeandclarke.co.uk (Keith Clarke) Date: Wed, 13 Jul 2011 18:47:05 +0100 Subject: New post detailing the technology and decisions behind my new website In-Reply-To: References: Message-ID: <1A9AB09E-1639-4F37-AE35-0ECE70ACB251@clarkeandclarke.co.uk> ...so after your epic 'le Mans, 24-hour' CMS coding challenge, you didn't fancy the Brazilian Grand-prix challenge to code a comments mgt. system in 2.5-hours? ;-) Best, Keith.. On 13 Jul 2011, at 18:18, Andre Garzia wrote: > Folks, > > I've just added comments to the blog system by using Disqus.com I think it > works and it is easier than writting my own. > > :-) > _______________________________________________ > use-livecode mailing list > use-livecode at lists.runrev.com > Please visit this url to subscribe, unsubscribe and manage your subscription preferences: > http://lists.runrev.com/mailman/listinfo/use-livecode From andre at andregarzia.com Wed Jul 13 14:02:22 2011 From: andre at andregarzia.com (Andre Garzia) Date: Wed, 13 Jul 2011 15:02:22 -0300 Subject: New post detailing the technology and decisions behind my new website In-Reply-To: <1A9AB09E-1639-4F37-AE35-0ECE70ACB251@clarkeandclarke.co.uk> References: <1A9AB09E-1639-4F37-AE35-0ECE70ACB251@clarkeandclarke.co.uk> Message-ID: On Wed, Jul 13, 2011 at 2:47 PM, Keith Clarke < keith.clarke at clarkeandclarke.co.uk> wrote: > ...so after your epic 'le Mans, 24-hour' CMS coding challenge, you didn't > fancy the Brazilian Grand-prix challenge to code a comments mgt. system in > 2.5-hours? ;-) > comment system would require a database or some clever engineering of text files and folders (like old fido boards), I opted for a cost effective solution... Brazilian Cheating :-) > Best, > Keith.. > > On 13 Jul 2011, at 18:18, Andre Garzia wrote: > > > Folks, > > > > I've just added comments to the blog system by using Disqus.com I think > it > > works and it is easier than writting my own. > > > > :-) > > _______________________________________________ > > use-livecode mailing list > > use-livecode at lists.runrev.com > > Please visit this url to subscribe, unsubscribe and manage your > subscription preferences: > > http://lists.runrev.com/mailman/listinfo/use-livecode > > > _______________________________________________ > use-livecode mailing list > use-livecode at lists.runrev.com > Please visit this url to subscribe, unsubscribe and manage your > subscription preferences: > http://lists.runrev.com/mailman/listinfo/use-livecode > -- http://www.andregarzia.com All We Do Is Code. From richmondmathewson at gmail.com Wed Jul 13 16:26:04 2011 From: richmondmathewson at gmail.com (Richmond Mathewson) Date: Wed, 13 Jul 2011 23:26:04 +0300 Subject: [OT] HyperNext Android Creator Message-ID: <4E1DFF5C.1010607@gmail.com> http://tigabyte.com/ interesting. From ambassador at fourthworld.com Wed Jul 13 16:40:05 2011 From: ambassador at fourthworld.com (Richard Gaskin) Date: Wed, 13 Jul 2011 13:40:05 -0700 Subject: [OT] HyperNext Android Creator In-Reply-To: <4E1DFF5C.1010607@gmail.com> References: <4E1DFF5C.1010607@gmail.com> Message-ID: <4E1E02A5.8020109@fourthworld.com> Richmond Mathewson wrote: > http://tigabyte.com/ > > interesting. Even more interesting if it becomes RealBASIC's Digital Chisel. ;) -- Richard Gaskin Fourth World LiveCode training and consulting: http://www.fourthworld.com Webzine for LiveCode developers: http://www.LiveCodeJournal.com LiveCode Journal blog: http://LiveCodejournal.com/blog.irv From lfredricks at proactive-intl.com Wed Jul 13 18:10:35 2011 From: lfredricks at proactive-intl.com (Lynn Fredricks) Date: Wed, 13 Jul 2011 15:10:35 -0700 Subject: [OT] HyperNext Android Creator In-Reply-To: <4E1E02A5.8020109@fourthworld.com> References: <4E1DFF5C.1010607@gmail.com> <4E1E02A5.8020109@fourthworld.com> Message-ID: <29E728C6AC11413996BED44D4FCEE402@GATEWAY> > Richmond Mathewson wrote: > > > http://tigabyte.com/ > > > > interesting. > > Even more interesting if it becomes RealBASIC's Digital Chisel. ;) I think the subset of what you can do with it is much more limited because it uses RBScript. The funny thing about DC is that it was a much better tool for education than SuperCard was. Those were the days ;-) Best regards, Lynn Fredricks President Paradigma Software http://www.paradigmasoft.com Valentina SQL Server: The Ultra-fast, Royalty Free Database Server From bobs at twft.com Wed Jul 13 19:24:43 2011 From: bobs at twft.com (Bob Sneidar) Date: Wed, 13 Jul 2011 16:24:43 -0700 Subject: Hotel Management Software Message-ID: Hi all. We have several conference centers around, but custom apps people have built for scheduling have been woefully inadequate. We need a kind of Hotel Scheduling solution, and everything out there is insanely expensive, or else charges exorbitant fees to host, based upon how many conferences, and how many attendees a year you will have. If anyone has done any work on this sort of thing, I would purchase what you have if I could be able to customize and expand on it. It's a long shot I know, but it's just the sort of thing Livecode would excel at. Barring that, does anyone do any freelance user interface work? I am not very good at building user interfaces. Maybe if someone had a template for forms that I could build upon, that would be a good starting point. Bob From niconiko at gmail.com Wed Jul 13 19:41:46 2011 From: niconiko at gmail.com (Nicolas Cueto) Date: Thu, 14 Jul 2011 08:41:46 +0900 Subject: outerGlow color differs by Win OS Message-ID: <90A62286-22FF-4FF6-8784-50009019AC2C@gmail.com> Hello list. On XP and Win98 the outerGlow color of objects is blue, as I'd set them. On Vista, it's red. Exact same stack, though. And to make the color blue, I must reset outerGlow color by choosing what on the palette appears as red but which on the stack becomes blue. Could someone explain why? I tried different blendmode and filter settings but no go. Using LC 4.6.1. Thanks. -- Nicolas Cueto (iPhone) From dr.alistair at gmail.com Wed Jul 13 21:01:24 2011 From: dr.alistair at gmail.com (planix) Date: Wed, 13 Jul 2011 18:01:24 -0700 (PDT) Subject: Hotel Management Software In-Reply-To: References: Message-ID: <1310605284681-3666426.post@n4.nabble.com> Hi, have you had a look at the Open Conference System (http://pkp.sfu.ca/?q=ocs)? I have used the Open Journal System (http://pkp.sfu.ca/?q=ojs) and found it to be excellent. cheers Alistair Townsville -- View this message in context: http://runtime-revolution.278305.n4.nabble.com/Hotel-Management-Software-tp3666333p3666426.html Sent from the Revolution - User mailing list archive at Nabble.com. From gandalf at doctorTimothyMiller.com Wed Jul 13 21:01:44 2011 From: gandalf at doctorTimothyMiller.com (Timothy Miller) Date: Wed, 13 Jul 2011 18:01:44 -0700 Subject: repeat with i= Message-ID: Hi, Let's say that I want to do something like repeat with i = 1 to 6 do "whatever" & i end repeat Except, I want 1 to 6 to be in random sequence I could think of a few kludgy ways to do this. Is there a standard approach? A simple approach? Thanks in advance. Tim From roger.e.eller at sealedair.com Wed Jul 13 21:12:08 2011 From: roger.e.eller at sealedair.com (Roger Eller) Date: Wed, 13 Jul 2011 21:12:08 -0400 Subject: repeat with i= In-Reply-To: References: Message-ID: On Wed, Jul 13, 2011 at 9:01 PM, Timothy Miller wrote: > Hi, > > Let's say that I want to do something like > > repeat with i = 1 to 6 > do "whatever" & i > end repeat > > Except, I want 1 to 6 to be in random sequence > > I could think of a few kludgy ways to do this. > > Is there a standard approach? A simple approach? > > Thanks in advance. > > Tim > This won't guarantee that all 6 numbers will get used (because it is random), but it will loop 6 times. repeat with i = 1 to 6 put random(6) into x answer "whatever" && x end repeat ?Roger From pete at mollysrevenge.com Wed Jul 13 21:25:10 2011 From: pete at mollysrevenge.com (Pete) Date: Wed, 13 Jul 2011 18:25:10 -0700 Subject: repeat with i= In-Reply-To: References: Message-ID: Maybe (untested): put "1,2,3,4,5,6" into x repeat with i=1 to 6 put item random(the number of items in x) into y put empty into item y of x end repeat Pete Molly's Revenge On Wed, Jul 13, 2011 at 6:01 PM, Timothy Miller < gandalf at doctortimothymiller.com> wrote: > Hi, > > Let's say that I want to do something like > > repeat with i = 1 to 6 > do "whatever" & i > end repeat > > Except, I want 1 to 6 to be in random sequence > > I could think of a few kludgy ways to do this. > > Is there a standard approach? A simple approach? > > Thanks in advance. > > Tim > > > > _______________________________________________ > use-livecode mailing list > use-livecode at lists.runrev.com > Please visit this url to subscribe, unsubscribe and manage your > subscription preferences: > http://lists.runrev.com/mailman/listinfo/use-livecode > > From jhj at jhj.com Wed Jul 13 21:37:51 2011 From: jhj at jhj.com (Jerry J) Date: Wed, 13 Jul 2011 18:37:51 -0700 Subject: repeat with i= In-Reply-To: References: Message-ID: <9B283C6F-5117-4CE0-9263-43076A785C7A@jhj.com> On Jul 13, 2011, at 6:01 PM, Timothy Miller wrote: > Hi, > > Let's say that I want to do something like > > repeat with i = 1 to 6 > do "whatever" & i > end repeat > > Except, I want 1 to 6 to be in random sequence > > I could think of a few kludgy ways to do this. > > Is there a standard approach? A simple approach? > > Thanks in advance. on mouseUp local theNums put "1,2,3,4,5,6" into theNums sort items of theNums numeric by random(1000000) repeat for each item I in theNums --do "whatever" & I end repeat put theNums end mouseUp From roger.e.eller at sealedair.com Wed Jul 13 21:39:25 2011 From: roger.e.eller at sealedair.com (Roger Eller) Date: Wed, 13 Jul 2011 21:39:25 -0400 Subject: repeat with i= In-Reply-To: References: Message-ID: This one guarantees that each of the 6 random numbers is only used once. put "1,2,3,4,5,6" into x replace comma with cr in x repeat until (the number of lines of t) = 6 -- be cautious with until loops put random(6) into tLine if tLine is not among the lines of t then put (line tLine of x) & cr after t answer "whatever" && (line tLine of x) end if end repeat ?Roger From m.schonewille at economy-x-talk.com Wed Jul 13 21:52:04 2011 From: m.schonewille at economy-x-talk.com (Mark Schonewille) Date: Thu, 14 Jul 2011 03:52:04 +0200 Subject: LiveCode.tv Event #31 Wrap-Up Message-ID: <75AE8A46-4733-4CE5-BB63-0DD92887E57F@economy-x-talk.com> Dear LiveCoders, Last Saturday, 9 July, we had another nice on-line gathering of LiveCode fans. This time we had 2 great presentations by Claudi and Jim. Claudi Cornaz showed his ?stackAlly?, which analyses your code and stacks, manages stack versions and archives code. One amazing feature is a tree view of events, showing all messages triggered by a single event in your code. You can watch Claudi's video here: http://www.ustream.tv/recorded/15893197 Jim Ault showed a simple UDP messaging example. He introduced us to the UDP protocol and gave a demonstration using a modified version of a stack by Alex Tweedly. You can find Jim's presentation at http://www.ustream.tv/recorded/15894157 The European HyperCard User Group (eHUG, http://www.ehug.info) raffled off an e-book courtesy of TidBITS. The e-book was won by Matthias. You can find more info on the TC series at http://www.takecontrolbooks.com. We are still looking for more people to prepare presentations. If you would like to participate in the event, now or later, please head over to http://qery.us/u0 for ideas and contact Mark or Bj?rnke. At http://qery.us/du you can find a web form that makes it really easy to contact us. (If you contacted me already, I'll be writing you tomorrow). I hope to see you all at the event next time. We will be making announcements on this mailing list and at http://livecode.tv where you can (soon) find a copy of this text. (Sorry for the current garbage on the blog; it seems it got hacked or spammed). -- Best regards, Mark Schonewille Economy-x-Talk Consulting and Software Engineering Homepage: http://economy-x-talk.com Twitter: http://twitter.com/xtalkprogrammer KvK: 50277553 New: Download the Installer Maker Plugin 1.6 for LiveCode here http://qery.us/ce From jacque at hyperactivesw.com Wed Jul 13 21:56:23 2011 From: jacque at hyperactivesw.com (J. Landman Gay) Date: Wed, 13 Jul 2011 20:56:23 -0500 Subject: outerGlow color differs by Win OS In-Reply-To: <90A62286-22FF-4FF6-8784-50009019AC2C@gmail.com> References: <90A62286-22FF-4FF6-8784-50009019AC2C@gmail.com> Message-ID: <4E1E4CC7.8060909@hyperactivesw.com> On 7/13/11 6:41 PM, Nicolas Cueto wrote: > Hello list. > > On XP and Win98 the outerGlow color of objects is blue, as I'd set > them. > > On Vista, it's red. Exact same stack, though. And to make the color > blue, I must reset outerGlow color by choosing what on the palette > appears as red but which on the stack becomes blue. > > Could someone explain why? It's a bug in the engine. On Android, it's yellow. :) -- Jacqueline Landman Gay | jacque at hyperactivesw.com HyperActive Software | http://www.hyperactivesw.com From form at nonsanity.com Wed Jul 13 22:53:34 2011 From: form at nonsanity.com (Nonsanity) Date: Wed, 13 Jul 2011 22:53:34 -0400 Subject: repeat with i= In-Reply-To: References: Message-ID: I like Pete's best - closest to what I was thinking - but I'd change the "put empty" line to "delete item y of x". I think the put empty would just put "" into that item, but the item would still be there, like: 1,2,,4,5,6 ~ Chris Innanen ~ Nonsanity On Wed, Jul 13, 2011 at 9:25 PM, Pete wrote: > Maybe (untested): > > put "1,2,3,4,5,6" into x > repeat with i=1 to 6 > put item random(the number of items in x) into y > > put empty into item y of x > end repeat > > Pete > Molly's Revenge > > > > > On Wed, Jul 13, 2011 at 6:01 PM, Timothy Miller < > gandalf at doctortimothymiller.com> wrote: > > > Hi, > > > > Let's say that I want to do something like > > > > repeat with i = 1 to 6 > > do "whatever" & i > > end repeat > > > > Except, I want 1 to 6 to be in random sequence > > > > I could think of a few kludgy ways to do this. > > > > Is there a standard approach? A simple approach? > > > > Thanks in advance. > > > > Tim > > > > > > > > _______________________________________________ > > use-livecode mailing list > > use-livecode at lists.runrev.com > > Please visit this url to subscribe, unsubscribe and manage your > > subscription preferences: > > http://lists.runrev.com/mailman/listinfo/use-livecode > > > > > _______________________________________________ > use-livecode mailing list > use-livecode at lists.runrev.com > Please visit this url to subscribe, unsubscribe and manage your > subscription preferences: > http://lists.runrev.com/mailman/listinfo/use-livecode > From dick.kriesel at mail.com Thu Jul 14 00:02:13 2011 From: dick.kriesel at mail.com (Dick Kriesel) Date: Wed, 13 Jul 2011 21:02:13 -0700 Subject: repeat with i= In-Reply-To: References: Message-ID: <99561597-0FF4-4534-BAA8-9B87D8B239EF@mail.com> On Jul 13, 2011, at 7:53 PM, Nonsanity wrote: > I like Pete's best - closest to what I was thinking - I like Jerry's best - it's better than I was thinking ... Jerry's is less code executed less often: one statement once versus two statements for each item. -- Dick From kee at kagi.com Thu Jul 14 00:20:57 2011 From: kee at kagi.com (Kee Nethery) Date: Wed, 13 Jul 2011 21:20:57 -0700 Subject: multi-lingual and the .lproj folders inside apps? In-Reply-To: <1310605284681-3666426.post@n4.nabble.com> References: <1310605284681-3666426.post@n4.nabble.com> Message-ID: How does one determine the preferred written language for a user's computer? For example, when to display French versus English or Italian? How does one typically make a multi-lingual Livecode app? Put all the translations in custom properties and set everything when the language changes? Kee Nethery From pete at mollysrevenge.com Thu Jul 14 00:33:25 2011 From: pete at mollysrevenge.com (Pete) Date: Wed, 13 Jul 2011 21:33:25 -0700 Subject: repeat with i= In-Reply-To: References: Message-ID: You're right, should probably be something like "replace y & comma with empty in x". Pete Molly's Revenge On Wed, Jul 13, 2011 at 7:53 PM, Nonsanity
wrote: > I like Pete's best - closest to what I was thinking - but I'd change the > "put empty" line to "delete item y of x". I think the put empty would just > put "" into that item, but the item would still be there, like: 1,2,,4,5,6 > > ~ Chris Innanen > ~ Nonsanity > > > On Wed, Jul 13, 2011 at 9:25 PM, Pete wrote: > > > Maybe (untested): > > > > put "1,2,3,4,5,6" into x > > repeat with i=1 to 6 > > put item random(the number of items in x) into y > > > > put empty into item y of x > > end repeat > > > > Pete > > Molly's Revenge > > > > > > > > > > On Wed, Jul 13, 2011 at 6:01 PM, Timothy Miller < > > gandalf at doctortimothymiller.com> wrote: > > > > > Hi, > > > > > > Let's say that I want to do something like > > > > > > repeat with i = 1 to 6 > > > do "whatever" & i > > > end repeat > > > > > > Except, I want 1 to 6 to be in random sequence > > > > > > I could think of a few kludgy ways to do this. > > > > > > Is there a standard approach? A simple approach? > > > > > > Thanks in advance. > > > > > > Tim > > > > > > > > > > > > _______________________________________________ > > > use-livecode mailing list > > > use-livecode at lists.runrev.com > > > Please visit this url to subscribe, unsubscribe and manage your > > > subscription preferences: > > > http://lists.runrev.com/mailman/listinfo/use-livecode > > > > > > > > _______________________________________________ > > use-livecode mailing list > > use-livecode at lists.runrev.com > > Please visit this url to subscribe, unsubscribe and manage your > > subscription preferences: > > http://lists.runrev.com/mailman/listinfo/use-livecode > > > _______________________________________________ > use-livecode mailing list > use-livecode at lists.runrev.com > Please visit this url to subscribe, unsubscribe and manage your > subscription preferences: > http://lists.runrev.com/mailman/listinfo/use-livecode > > From johnpatten at mac.com Thu Jul 14 00:53:39 2011 From: johnpatten at mac.com (John Patten) Date: Wed, 13 Jul 2011 21:53:39 -0700 Subject: Set image/data/source via an image on web? Message-ID: <5D58E5A5-29B4-452E-91A3-CBEA76E8C129@mac.com> Hi All... I have a script where I'm creating a number of image objects on the fly and then attempting to set them to jpg on a web server. Something like: put URL "http://webserver.on-rev.com/conference/images/" & (line x of tBadgeList) & ".jpg" into tImageData put tImageData into last image ... It appears to create the correct target file "http://webserver.on-rev.com/conference/images/11.jpg" Anybody have a tip on how to go about doing this? Thank you! John Patten SUSD From jacque at hyperactivesw.com Thu Jul 14 00:58:31 2011 From: jacque at hyperactivesw.com (J. Landman Gay) Date: Wed, 13 Jul 2011 23:58:31 -0500 Subject: repeat with i= In-Reply-To: References: Message-ID: <4E1E7777.8020308@hyperactivesw.com> On 7/13/11 11:33 PM, Pete wrote: > You're right, should probably be something like "replace y& comma with > empty in x". Plain old "delete" does it. Also, not everyone knows about the "any" keyword but it's really handy for lines like this: put item random(the number of items in x) of x into y Which is more readable as: put any item of x into y I'm very fond of "any". -- Jacqueline Landman Gay | jacque at hyperactivesw.com HyperActive Software | http://www.hyperactivesw.com From jacque at hyperactivesw.com Thu Jul 14 01:08:53 2011 From: jacque at hyperactivesw.com (J. Landman Gay) Date: Thu, 14 Jul 2011 00:08:53 -0500 Subject: Set image/data/source via an image on web? In-Reply-To: <5D58E5A5-29B4-452E-91A3-CBEA76E8C129@mac.com> References: <5D58E5A5-29B4-452E-91A3-CBEA76E8C129@mac.com> Message-ID: <4E1E79E5.1070807@hyperactivesw.com> On 7/13/11 11:53 PM, John Patten wrote: > Hi All... > > I have a script where I'm creating a number of image objects on the > fly and then attempting to set them to jpg on a web server. > > Something like: > > put URL "http://webserver.on-rev.com/conference/images/"& (line x > of tBadgeList)& ".jpg" into tImageData > > put tImageData into last image > > ... > > It appears to create the correct target file > "http://webserver.on-rev.com/conference/images/11.jpg" > > Anybody have a tip on how to go about doing this? Don't use imagedata, just put the image itself, or alternately use "the text" of the image. Either of these should work: put URL "http://webserver.on-rev.com/conference/images/"& (line x of tBadgeList)& ".jpg" into last image put the text of URL "http://webserver.on-rev.com/conference/images/"& (line x of tBadgeList)& ".jpg" into last image -- Jacqueline Landman Gay | jacque at hyperactivesw.com HyperActive Software | http://www.hyperactivesw.com From pete at mollysrevenge.com Thu Jul 14 01:27:13 2011 From: pete at mollysrevenge.com (Pete) Date: Wed, 13 Jul 2011 22:27:13 -0700 Subject: repeat with i= In-Reply-To: <4E1E7777.8020308@hyperactivesw.com> References: <4E1E7777.8020308@hyperactivesw.com> Message-ID: Great, learned two things form this - delete and any! Pete Molly's Revenge On Wed, Jul 13, 2011 at 9:58 PM, J. Landman Gay wrote: > On 7/13/11 11:33 PM, Pete wrote: > >> You're right, should probably be something like "replace y& comma with >> empty in x". >> > > Plain old "delete" does it. > > Also, not everyone knows about the "any" keyword but it's really handy for > lines like this: > > put item random(the number of items in x) of x into y > > Which is more readable as: > > put any item of x into y > > I'm very fond of "any". > > -- > Jacqueline Landman Gay | jacque at hyperactivesw.com > HyperActive Software | http://www.hyperactivesw.com > > > ______________________________**_________________ > use-livecode mailing list > use-livecode at lists.runrev.com > Please visit this url to subscribe, unsubscribe and manage your > subscription preferences: > http://lists.runrev.com/**mailman/listinfo/use-livecode > > From scott at elementarysoftware.com Thu Jul 14 02:31:48 2011 From: scott at elementarysoftware.com (Scott Morrow) Date: Wed, 13 Jul 2011 23:31:48 -0700 Subject: repeat with i= In-Reply-To: <4E1E7777.8020308@hyperactivesw.com> References: <4E1E7777.8020308@hyperactivesw.com> Message-ID: Cool, I didn't know about "any". Thanks, Jacque! Scott Morrow On Jul 13, 2011, at 9:58 PM, J. Landman Gay wrote: > On 7/13/11 11:33 PM, Pete wrote: >> You're right, should probably be something like "replace y& comma with >> empty in x". > > Plain old "delete" does it. > > Also, not everyone knows about the "any" keyword but it's really handy for lines like this: > > put item random(the number of items in x) of x into y > > Which is more readable as: > > put any item of x into y > > I'm very fond of "any". > > -- > Jacqueline Landman Gay | jacque at hyperactivesw.com > HyperActive Software | http://www.hyperactivesw.com > > _______________________________________________ > use-livecode mailing list > use-livecode at lists.runrev.com > Please visit this url to subscribe, unsubscribe and manage your subscription preferences: > http://lists.runrev.com/mailman/listinfo/use-livecode From keith.clarke at clarkeandclarke.co.uk Thu Jul 14 03:05:33 2011 From: keith.clarke at clarkeandclarke.co.uk (Keith Clarke) Date: Thu, 14 Jul 2011 08:05:33 +0100 Subject: repeat with i= In-Reply-To: <4E1E7777.8020308@hyperactivesw.com> References: <4E1E7777.8020308@hyperactivesw.com> Message-ID: ...so for a random selection, enforcing the use of all 6 items, would this work? put "1,2,3,4,5,6" into x repeat until x is empty put any item of x into y do something delete item y from x end repeat Best, Keith.. On 14 Jul 2011, at 05:58, J. Landman Gay wrote: > On 7/13/11 11:33 PM, Pete wrote: >> You're right, should probably be something like "replace y& comma with >> empty in x". > > Plain old "delete" does it. > > Also, not everyone knows about the "any" keyword but it's really handy for lines like this: > > put item random(the number of items in x) of x into y > > Which is more readable as: > > put any item of x into y > > I'm very fond of "any". > > -- > Jacqueline Landman Gay | jacque at hyperactivesw.com > HyperActive Software | http://www.hyperactivesw.com > > _______________________________________________ > use-livecode mailing list > use-livecode at lists.runrev.com > Please visit this url to subscribe, unsubscribe and manage your subscription preferences: > http://lists.runrev.com/mailman/listinfo/use-livecode From jimaultwins at yahoo.com Thu Jul 14 03:23:15 2011 From: jimaultwins at yahoo.com (Jim Ault) Date: Thu, 14 Jul 2011 00:23:15 -0700 Subject: repeat with i= In-Reply-To: References: <4E1E7777.8020308@hyperactivesw.com> Message-ID: On Jul 14, 2011, at 12:05 AM, Keith Clarke wrote: > ...so for a random selection, enforcing the use of all 6 items, > would this work? > > put "1,2,3,4,5,6" into x > repeat until x is empty > put any item of x into y > do something > delete item y from x > end repeat > > No, since the first pass could choose '2', then the next pass could choose '6' and produce the error "item 6 does not exist. My preference would be to do the put "1,2,3,4,5,6" into theNums sort items of theNums numeric by random(1000000) repeat for each item Y of theNums do "some command" & Y end repeat Jim Ault Las Vegas From revolution at derbrill.de Thu Jul 14 03:37:47 2011 From: revolution at derbrill.de (Malte Brill) Date: Thu, 14 Jul 2011 09:37:47 +0200 Subject: unicode (again) This time: Tooltips In-Reply-To: References: Message-ID: <83386A56-7E4C-4CCC-A1E1-E46292A51A16@derbrill.de> Hi all, if I wanted to display russian Tooltips, how would I go about that? Is it possible at all? Thanks in advance, Malte From SparkOutYNY at gmail.com Thu Jul 14 04:33:06 2011 From: SparkOutYNY at gmail.com (SparkOut) Date: Thu, 14 Jul 2011 01:33:06 -0700 (PDT) Subject: Hotel Management Software In-Reply-To: References: Message-ID: <1310632386393-3666911.post@n4.nabble.com> Also have a look at http://rapla.org/ perhaps. -- View this message in context: http://runtime-revolution.278305.n4.nabble.com/Hotel-Management-Software-tp3666333p3666911.html Sent from the Revolution - User mailing list archive at Nabble.com. From keith.clarke at clarkeandclarke.co.uk Thu Jul 14 05:12:06 2011 From: keith.clarke at clarkeandclarke.co.uk (Keith Clarke) Date: Thu, 14 Jul 2011 10:12:06 +0100 Subject: repeat with i= In-Reply-To: References: <4E1E7777.8020308@hyperactivesw.com> Message-ID: <8CE761D5-E040-464C-AE62-94A56AAFA487@clarkeandclarke.co.uk> ...ah yes, of course, thanks Jim - I forgot that 'delete item y' is not the same as 'delete item *called* y' ! So, would itemoffset help...? put "1,2,3,4,5,6" into theNumbers repeat until theNumbers is empty put any item theNumbers into n do something delete (itemOffset(n, theNumbers)) from theNumbers end repeat I realise that this may not be the most efficient layout (one more line than yours!) but it reads nicely and could help in educational settings when teaching the application of scripts to formulae that use finite n-based series - I'm thinking here about permutations (nCr), combinations (nPr), binomials, factorials, statistical analysis, etc. Just a thought and hopefully not too much of a hijacking of the thread :-) Best, Keith.. On 14 Jul 2011, at 08:23, Jim Ault wrote: > > On Jul 14, 2011, at 12:05 AM, Keith Clarke wrote: > >> ...so for a random selection, enforcing the use of all 6 items, would this work? >> >> put "1,2,3,4,5,6" into x >> repeat until x is empty >> put any item of x into y >> do something >> delete item y from x >> end repeat >> >> > > No, since the first pass could choose '2', > then the next pass could choose '6' > and produce the error "item 6 does not exist. > > My preference would be to do the > > put "1,2,3,4,5,6" into theNums > sort items of theNums numeric by random(1000000) > repeat for each item Y of theNums > do "some command" & Y > end repeat > > > Jim Ault > Las Vegas > > > > _______________________________________________ > use-livecode mailing list > use-livecode at lists.runrev.com > Please visit this url to subscribe, unsubscribe and manage your subscription preferences: > http://lists.runrev.com/mailman/listinfo/use-livecode From klaus at major.on-rev.com Thu Jul 14 06:49:56 2011 From: klaus at major.on-rev.com (Klaus on-rev) Date: Thu, 14 Jul 2011 12:49:56 +0200 Subject: Set image/data/source via an image on web? In-Reply-To: <4E1E79E5.1070807@hyperactivesw.com> References: <5D58E5A5-29B4-452E-91A3-CBEA76E8C129@mac.com> <4E1E79E5.1070807@hyperactivesw.com> Message-ID: Hi John, Am 14.07.2011 um 07:08 schrieb J. Landman Gay: > On 7/13/11 11:53 PM, John Patten wrote: >> Hi All... >> >> I have a script where I'm creating a number of image objects on the >> fly and then attempting to set them to jpg on a web server. >> Something like: >> put URL "http://webserver.on-rev.com/conference/images/"& (line x >> of tBadgeList)& ".jpg" into tImageData >> put tImageData into last image >> ... >> >> It appears to create the correct target file >> "http://webserver.on-rev.com/conference/images/11.jpg" >> >> Anybody have a tip on how to go about doing this? > > Don't use imagedata, just put the image itself, or alternately use "the text" of the image. Either of these should work: > put URL "http://webserver.on-rev.com/conference/images/"& (line x of tBadgeList)& ".jpg" into last image > put the text of URL "http://webserver.on-rev.com/conference/images/"& (line x of tBadgeList)& ".jpg" into last image or just set "the filename" with a one-liner: ... set the filename of last img to ("http://webserver.on-rev.com/conference/images/"& (line x of tBadgeList) & ".jpg") ... Best Klaus -- Klaus Major http://www.major-k.de klaus at major.on-rev.com From mazzapaolo at libero.it Thu Jul 14 07:00:52 2011 From: mazzapaolo at libero.it (paolo mazza) Date: Thu, 14 Jul 2011 13:00:52 +0200 Subject: repeat with i= In-Reply-To: <8CE761D5-E040-464C-AE62-94A56AAFA487@clarkeandclarke.co.uk> References: <4E1E7777.8020308@hyperactivesw.com> <8CE761D5-E040-464C-AE62-94A56AAFA487@clarkeandclarke.co.uk> Message-ID: Tha's interesting. I think you have to use delete.. of instead delete.... from . To me this is correct. put "1,2,3,4,5,6" into theNumbers repeat until theNumbers is empty put any item theNumbers into n put n after theNumberList delete item (itemOffset(n, theNumbers)) of theNumbers end repeat put theNumberList All the best. Paolo From index at kenjikojima.com Thu Jul 14 07:17:10 2011 From: index at kenjikojima.com (Kenji Kojima) Date: Thu, 14 Jul 2011 07:17:10 -0400 Subject: unicode (again) This time: Tooltips In-Reply-To: <83386A56-7E4C-4CCC-A1E1-E46292A51A16@derbrill.de> References: <83386A56-7E4C-4CCC-A1E1-E46292A51A16@derbrill.de> Message-ID: <803D121F-66F1-44C3-BB2B-D17D41E94B9B@kenjikojima.com> Hi Malte, This Japanese Tooltips. go to url "http://kenjikojima.com/runrev/handbook/download/jpTooltip.rev" You can change it to Russian. Best, -- Kenji Kojima / ???? http://www.kenjikojima.com/ On Jul 14, 2011, at 3:37 AM, Malte Brill wrote: > Hi all, > > if I wanted to display russian Tooltips, how would I go about that? Is it possible at all? > > Thanks in advance, > > Malte > _______________________________________________ > use-livecode mailing list > use-livecode at lists.runrev.com > Please visit this url to subscribe, unsubscribe and manage your subscription preferences: > http://lists.runrev.com/mailman/listinfo/use-livecode From curry at pair.com Thu Jul 14 07:21:23 2011 From: curry at pair.com (Curry Kenworthy) Date: Thu, 14 Jul 2011 06:21:23 -0500 Subject: Hotel Management Software In-Reply-To: References: Message-ID: <4E1ED133.1000509@pair.com> > We need a kind of Hotel Scheduling solution Howdy, I couldn't resist posting on this subject! I worked hotel reservations office for many years and used several industry products in my work, tested others to review for management, and of course over time built up my own ideas of "a better way" for hotel management and reservation interface. (After seeing lots of worse ways....) In spare time I started some work on reservations app interface screen ideas, at very early stages and mainly focused on guest room rentals rather than conference slots, but anyway although I don't have any quickie templates, I know very well where you're coming from and would be happy to help you build a neat solution. Drop me a line if you'd like to plan something in LiveCode. Best wishes, Curry Kenworthy Custom LiveCode Software Development http://curryk.com/consulting/ Email: info at curryk.com Alt. email: curry at pair.com From john at splash21.com Thu Jul 14 08:52:14 2011 From: john at splash21.com (John Craig) Date: Thu, 14 Jul 2011 13:52:14 +0100 Subject: Fun DropTool Message-ID: <4E1EE67E.3010000@splash21.com> I've just had a first attempt at creating a custom control with the rather nicely engineered DropTools palette from Sons of Thunder. It's a scalable android logo constructed from LC graphic objects - just a bit of fun! You can get it here if you're interested; http://mobgui.com/downloads/Android.zip From revolution at derbrill.de Thu Jul 14 08:56:52 2011 From: revolution at derbrill.de (Malte Brill) Date: Thu, 14 Jul 2011 14:56:52 +0200 Subject: unicode (again) This time: Tooltips In-Reply-To: References: Message-ID: <09958975-E36A-4765-8968-43E5794CF566@derbrill.de> Kenji! thanks so much! This saves my day. Once you know how to tackle all this stuff, it gets a little less complicated. So as a short recap: tooltips -> UTF8 fields -> UTF16 Button Text -> UTF16 and need to set the appropriate font Button labels -> UTF16 and need to set the appropriate font Window Title: set the unicodeTitle of stack "myStack" to UTF16 encoded string Anything I did forget / not get right in here? Cheers, Malte From jimaultwins at yahoo.com Thu Jul 14 10:13:27 2011 From: jimaultwins at yahoo.com (Jim Ault) Date: Thu, 14 Jul 2011 07:13:27 -0700 Subject: repeat with i= In-Reply-To: <8CE761D5-E040-464C-AE62-94A56AAFA487@clarkeandclarke.co.uk> References: <4E1E7777.8020308@hyperactivesw.com> <8CE761D5-E040-464C-AE62-94A56AAFA487@clarkeandclarke.co.uk> Message-ID: <150D60F4-F76E-4FF6-8C8D-E46AA457A97A@yahoo.com> On Jul 14, 2011, at 2:12 AM, Keith Clarke wrote: > ...ah yes, of course, thanks Jim - I forgot that 'delete item y' is > not the same as 'delete item *called* y' ! > > So, would itemoffset help...? > > put "1,2,3,4,5,6" into theNumbers > repeat until theNumbers is empty > put any item theNumbers into n > do something > delete (itemOffset(n, theNumbers)) from theNumbers > end repeat > > I realise that this may not be the most efficient layout (one more > line than yours!) but it reads nicely and could help in educational > settings when teaching the application of scripts to formulae that > use finite n-based series - I'm thinking here about permutations > (nCr), combinations (nPr), binomials, factorials, statistical > analysis, etc. Just a thought and hopefully not too much of a > hijacking of the thread :-) > Best, Yes, and you could use a more visual logic... put "1,2,3,4,5,6" into theNumbers repeat until sum(theNumbers) is 0 put any item theNumbers into N if N is not 0 then do something put 0 into item N of theNumbers end if end repeat > > On 14 Jul 2011, at 08:23, Jim Ault wrote: > >> >> On Jul 14, 2011, at 12:05 AM, Keith Clarke wrote: >> >>> ...so for a random selection, enforcing the use of all 6 items, >>> would this work? >>> >>> put "1,2,3,4,5,6" into x >>> repeat until x is empty >>> put any item of x into y >>> do something >>> delete item y from x >>> end repeat >>> >>> >> >> No, since the first pass could choose '2', >> then the next pass could choose '6' >> and produce the error "item 6 does not exist. >> >> My preference would be to do the >> >> put "1,2,3,4,5,6" into theNums >> sort items of theNums numeric by random(1000000) >> repeat for each item Y of theNums >> do "some command" & Y >> end repeat Jim Ault Las Vegas From zellner at tamu.edu Thu Jul 14 11:06:01 2011 From: zellner at tamu.edu (Ronald Zellner) Date: Thu, 14 Jul 2011 10:06:01 -0500 Subject: repeat with i= Message-ID: Jim's seems to be the most efficient and accurate, but I needed this change to make it work. ' of ' changed to 'in' I like to add something to create quick feedback- like the field "chosen". Easier to see the result. Then switch to the complete function when the code is working. on mouseUp local Y, theNums put empty into field "chosen" put "1,2,3,4,5,6" into theNums sort items of theNums numeric by random(1000000) repeat for each item Y in theNums put Y & return after field "chosen" end repeat end mouseUp This is a variant to Keith's: It stops at 6 items, but it still has the problem of repeating some numbers that are still in the list. on mouseUp local x, y, z put empty into field "chosen" put "1,2,3,4,5,6" into x repeat with z = 1 to 6 put any item of x into y put y & return after field "chosen" delete item y of x end repeat end mouseUp Ron From keith.clarke at clarkeandclarke.co.uk Thu Jul 14 11:19:08 2011 From: keith.clarke at clarkeandclarke.co.uk (Keith Clarke) Date: Thu, 14 Jul 2011 16:19:08 +0100 Subject: repeat with i= In-Reply-To: References: Message-ID: Ronald, Sorry, don't use my first attempt - it had an error. You'd need the updated version that used itemoffset to delete the the specific number in the list, rather than it's position. To see what was used in each loop in real time, you can either put y or answer y To see the choices made retrospectively, just put y and CR after tSelected within the loop before the end repeat and then use answer tSelected or put tSelected after the end repeat. Best, Keith.. On 14 Jul 2011, at 16:06, Ronald Zellner wrote: > Jim's seems to be the most efficient and accurate, > but I needed this change to make it work. ' of ' changed to 'in' > > I like to add something to create quick feedback- like the field "chosen". > Easier to see the result. > Then switch to the complete function when the code is working. > > on mouseUp > local Y, theNums > put empty into field "chosen" > > put "1,2,3,4,5,6" into theNums > sort items of theNums numeric by random(1000000) > repeat for each item Y in theNums > put Y & return after field "chosen" > end repeat > end mouseUp > > > This is a variant to Keith's: > It stops at 6 items, but it still has the problem of repeating some numbers that are still in the list. > > on mouseUp > local x, y, z > put empty into field "chosen" > put "1,2,3,4,5,6" into x > repeat with z = 1 to 6 > put any item of x into y > put y & return after field "chosen" > delete item y of x > end repeat > end mouseUp > > Ron > _______________________________________________ > use-livecode mailing list > use-livecode at lists.runrev.com > Please visit this url to subscribe, unsubscribe and manage your subscription preferences: > http://lists.runrev.com/mailman/listinfo/use-livecode From klaus at major.on-rev.com Thu Jul 14 11:35:14 2011 From: klaus at major.on-rev.com (Klaus on-rev) Date: Thu, 14 Jul 2011 17:35:14 +0200 Subject: repeat with i= In-Reply-To: References: Message-ID: Hi Ron, Am 14.07.2011 um 17:06 schrieb Ronald Zellner: > Jim's seems to be the most efficient and accurate, > but I needed this change to make it work. ' of ' changed to 'in' > > I like to add something to create quick feedback- like the field "chosen". > Easier to see the result. > Then switch to the complete function when the code is working. > ... > > This is a variant to Keith's: > It stops at 6 items, but it still has the problem of repeating some numbers that are still in the list. Sure: any(y) probably <> z :-) > on mouseUp > local x, y, z > put empty into field "chosen" > put "1,2,3,4,5,6" into x > repeat with z = 1 to 6 > put any item of x into y > put y & return after field "chosen" > delete item y of x > end repeat > end mouseUp on mouseUp local x, y, z put empty into field "chosen" put "1,2,3,4,5,6" into x repeat with z = 1 to 6 put random(the num of items of x) into y put item y of x & return after field "chosen" delete item y of x end repeat end mouseUp > Ron Best Klaus -- Klaus Major http://www.major-k.de klaus at major.on-rev.com From roger.e.eller at sealedair.com Thu Jul 14 12:21:56 2011 From: roger.e.eller at sealedair.com (Roger Eller) Date: Thu, 14 Jul 2011 12:21:56 -0400 Subject: Fun DropTool In-Reply-To: <4E1EE67E.3010000@splash21.com> References: <4E1EE67E.3010000@splash21.com> Message-ID: On Thu, Jul 14, 2011 at 8:52 AM, John Craig wrote: > I've just had a first attempt at creating a custom control with the rather > nicely engineered DropTools palette from Sons of Thunder. It's a scalable > android logo constructed from LC graphic objects - just a bit of fun! > > > You can get it here if you're interested; > > http://mobgui.com/downloads/**Android.zip > > Nice graphic! However, there must be a problem with how Android renders LC graphic objects because the rounded corners don't maintain the same radius as in the IDE. Also in the Android emulator, Mr. Android loses his head (literally). I'll send you an off-list screenshot. Thanks for making this. ~Roger From jacque at hyperactivesw.com Thu Jul 14 13:02:02 2011 From: jacque at hyperactivesw.com (J. Landman Gay) Date: Thu, 14 Jul 2011 12:02:02 -0500 Subject: repeat with i= In-Reply-To: References: Message-ID: <4E1F210A.8000507@hyperactivesw.com> On 7/14/11 10:06 AM, Ronald Zellner wrote: > Jim's seems to be the most efficient and accurate, I agree. Sorting the list before the loop and then peeling off the items one by one is the most efficient way to do it. I mentioned "any" because it's such a nice addition to the language, but I didn't mean to endorse it for this situation. -- Jacqueline Landman Gay | jacque at hyperactivesw.com HyperActive Software | http://www.hyperactivesw.com From rdimola at evergreeninfo.net Thu Jul 14 13:11:49 2011 From: rdimola at evergreeninfo.net (Ralph DiMola) Date: Thu, 14 Jul 2011 13:11:49 -0400 Subject: [Android] Why so quiet? Who's developing Android apps? In-Reply-To: References: Message-ID: <015f01cc4249$1ebf6f90$5c3e4eb0$@net> OK then let's get noisy I purchased LC almost exclusively for IOS and Android. LC seems like a marvelous development platform so far. My first Android app progressing. After I have it fully functional I will fire up the MAC Book to generate the iOS app and load it onto the iPad for testing and flush out platform specific issues My first app uses SQLite and I must say that it works seamlessly on the IDE as well as on the Android HW. I still have some major road blocks. I will not be testing on iOS until the Android issues are ironed out. Thanks in advance for all you help and insight. I am running the IDE on Win XP SP3 32 bit. Testing on a Droid X running 2.2 Froyo. I have not updated to 2.3 Gingerbread because no root exploit has been developed and without access to the elusive data folder I would be working blind. Bugs? 1) In desktop shortcut for LC, the "Start in" folder points to "C:\Program Files\RunRev\LiveCode 4.6.x\". This seems normal enough. But if you specify a Keystore file in the Android standalone applications screen the IDE can't find when building the app it unless you change the "Start in" folder to the folder where the .livecode file lives and restart LC. Also if you change the current directory to anything else in your scripts while testing, again the keystore file can't be found and LC has to be restarted. Also when doing a "Save as" the default folder is not where you opened the .livecode file from but is instead pointed to the LC shortcut "Start in" folder or the folder one last changed the directory to in a script. 2) If you try to exit the app via a close stack on the Android HW the app freezes. If you start the app again you get a black screen. Then if you try to test a new version(via test button) the HW locks up and only a battery removal will get you running again. 3) One still seems to need the old plug-in to see error messages and results of a put on the Android HW. Is this going to be integrated into the IDE? Filtering messages so you only see events associated with the app would be helpful. But the bug here is on a crash the stack dump exceeds the scrolling history length and the beginning of the crash has scrolled off the top of the history. 4) SQLite has stopped working in 4.6.3 (dp1,2,3). If you do a "revOpenDatabase("sqlite", tDatabasePath, , , , )" the app just quits on the Android HW. And the origin of the of the crash has scrolled off the log viewers history. The IDE still works fine. 4.6.2 works fine. 5) There are still references to iOS in the Android release notes in 4.6.3(DP3). See Pg 14 and 19. 6) On the Android HW I can set the directory to the virtual specialfolders("engine") and list files and folders. If I try to set the folder to a 'specialfolders("engine") & Slash & "subfolder"' as documented in the release notes, the script just dies with no error. Questions: 1) Where is the list of (in VB ez) "properties" and "methods" for objects? Maybe I'm missing it in the docs. 2) Where is the list of possible LC internal events for each object? Such as openCard, preopenCard, mouseUp.... Some are obvious but others are obscure to me. I pick many of them up from this forum and online examples. 3) What event fires off when the use hits the physical Android "Menu" key? 4) My first card is a splash screen with an image object. Image displays in the IDE but even when I include it in the copy files the app on the Android HW just shows a blank screen. I have seen it in the APK assets folder and in the root of specialfolders("engine") virtual folder. But still can't find out where the app is looking for it. Should I just be loading the image into the image object at run-time? 8) Is there a way to list files and folders without actually setting the current directory to the folder and then listing files or folders? Looking to the Future: 9) When will socket communication be available on Android? 10) When will ODBC database access be available on Android? 11) Any update for GPS access on Android? Have the zip==>lat/lon database. Know the great circle equations. Just need the lat/lon from the GPS. Also is this the appropriate list to post these on? Should I post one email per question or stack them up like this email? Just want to follow the proper protocol. Thanks Ralph -----Original Message----- From: use-livecode-bounces at lists.runrev.com [mailto:use-livecode-bounces at lists.runrev.com] On Behalf Of Roger Eller Sent: Saturday, July 09, 2011 11:17 AM To: How to use LiveCode Subject: [Android] Why so quiet? Who's developing Android apps? Just wondering if I am alone... echo... echo... ;-) What are your plans or aspirations regarding the Android platform and LiveCode? Those of you who build for iOS, are you also building your apps for Android? Are the capabilities you need present in the latest version of LiveCode? I am still happy that RunRev added Android to our toolbelt, but where is the community? How about a show of hands if you are using it, and if not, please share with us why. ~Roger _______________________________________________ use-livecode mailing list use-livecode at lists.runrev.com Please visit this url to subscribe, unsubscribe and manage your subscription preferences: http://lists.runrev.com/mailman/listinfo/use-livecode From jacque at hyperactivesw.com Thu Jul 14 13:19:15 2011 From: jacque at hyperactivesw.com (J. Landman Gay) Date: Thu, 14 Jul 2011 12:19:15 -0500 Subject: Fun DropTool In-Reply-To: <4E1EE67E.3010000@splash21.com> References: <4E1EE67E.3010000@splash21.com> Message-ID: <4E1F2513.5010003@hyperactivesw.com> On 7/14/11 7:52 AM, John Craig wrote: > I've just had a first attempt at creating a custom control with the > rather nicely engineered DropTools palette from Sons of Thunder. It's a > scalable android logo constructed from LC graphic objects - just a bit > of fun! Works great here. I'm not having any trouble with the head resizing, btw. -- Jacqueline Landman Gay | jacque at hyperactivesw.com HyperActive Software | http://www.hyperactivesw.com From jhurley0305 at sbcglobal.net Thu Jul 14 13:41:51 2011 From: jhurley0305 at sbcglobal.net (James Hurley) Date: Thu, 14 Jul 2011 10:41:51 -0700 Subject: repeat with i= In-Reply-To: References: Message-ID: I haven't been following this thread so my apologies if I'm missing the point, but a while back I had to deal with the task of selecting m random people for a voter pole from a list N registered voters, where N was over 100,000. Check out "Random pick" in Rev Online. I found using an array 3 times factor than picking from a list. It takes time to pick a number out of a list. Including the time to convert the list into an array. Jim Hurley From bobs at twft.com Thu Jul 14 14:26:20 2011 From: bobs at twft.com (Bob Sneidar) Date: Thu, 14 Jul 2011 11:26:20 -0700 Subject: Hotel Management Software In-Reply-To: <1310605284681-3666426.post@n4.nabble.com> References: <1310605284681-3666426.post@n4.nabble.com> Message-ID: Thanks I will look at that. Bob On Jul 13, 2011, at 6:01 PM, planix wrote: > Hi, > > have you had a look at the Open Conference System > (http://pkp.sfu.ca/?q=ocs)? > > I have used the Open Journal System (http://pkp.sfu.ca/?q=ojs) and found it > to be excellent. > > cheers > > Alistair > Townsville > > -- > View this message in context: http://runtime-revolution.278305.n4.nabble.com/Hotel-Management-Software-tp3666333p3666426.html > Sent from the Revolution - User mailing list archive at Nabble.com. > > _______________________________________________ > use-livecode mailing list > use-livecode at lists.runrev.com > Please visit this url to subscribe, unsubscribe and manage your subscription preferences: > http://lists.runrev.com/mailman/listinfo/use-livecode From bobs at twft.com Thu Jul 14 14:26:39 2011 From: bobs at twft.com (Bob Sneidar) Date: Thu, 14 Jul 2011 11:26:39 -0700 Subject: Hotel Management Software In-Reply-To: <1310632386393-3666911.post@n4.nabble.com> References: <1310632386393-3666911.post@n4.nabble.com> Message-ID: <606261F6-AAB6-4C66-AFFE-7C43F3A49F7F@twft.com> Thanks you. Looks interesting, no accounting though. Bob On Jul 14, 2011, at 1:33 AM, SparkOut wrote: > Also have a look at http://rapla.org/ perhaps. > > -- > View this message in context: http://runtime-revolution.278305.n4.nabble.com/Hotel-Management-Software-tp3666333p3666911.html > Sent from the Revolution - User mailing list archive at Nabble.com. > > _______________________________________________ > use-livecode mailing list > use-livecode at lists.runrev.com > Please visit this url to subscribe, unsubscribe and manage your subscription preferences: > http://lists.runrev.com/mailman/listinfo/use-livecode From john at splash21.com Thu Jul 14 14:40:07 2011 From: john at splash21.com (John Craig) Date: Thu, 14 Jul 2011 19:40:07 +0100 Subject: Fun DropTool In-Reply-To: <4E1F2513.5010003@hyperactivesw.com> References: <4E1EE67E.3010000@splash21.com> <4E1F2513.5010003@hyperactivesw.com> Message-ID: <4E1F3807.10201@splash21.com> Bummer! - I get the same as Roger - an angry looking droid with big eyebrows, and a missing head! (And the odd looking radius ends) I'm set at platform 2.2 API level 8 - is this still the current setup? Just to rub salt in the wound, it renders perfectly on iOS!! On 14/07/2011 18:19, J. Landman Gay wrote: > On 7/14/11 7:52 AM, John Craig wrote: >> I've just had a first attempt at creating a custom control with the >> rather nicely engineered DropTools palette from Sons of Thunder. It's a >> scalable android logo constructed from LC graphic objects - just a bit >> of fun! > > Works great here. I'm not having any trouble with the head resizing, btw. > From bvg at mac.com Thu Jul 14 14:56:21 2011 From: bvg at mac.com (=?iso-8859-1?Q?Bj=F6rnke_von_Gierke?=) Date: Thu, 14 Jul 2011 20:56:21 +0200 Subject: Poll for LiveCode.tv event #32 In-Reply-To: <081FD717-0A48-447D-90AA-A8F370B14F43@mac.com> References: <6B865405-BBC0-491C-AD25-BFB1C85A2D58@mac.com> <7EC6A85B-DD6B-4138-A597-512A194F4453@mac.com> <89782C22-C6E3-4245-95D1-A48F69423753@mac.com> <061D9182-B34B-4ABB-A7D6-9C2D09DA8755@mac.com> <73F84954-890B-4627-859A-702C8054F13B@mac.com> <093DEF5E-6C0F-4924-A262-F6DBE748ED45@mac.com> <3FB852C9-5675-4A19-9506-543733BB2547@mac.com> <914DF999-596E-4023-A213-9469C6A69FA0@mac.com> <2B105965-FB27-41D0-B95F-1F4C84ADD53B@mac.com> <163225D1-67D1-4CE7-8049-E85A8D94D177@mac.com> <225B0941-5D11-434A-BC0B-CD61B998E9F8@mac.com> <201756EB-9601-417D-856E-5128C5256EF1@mac.com> <081FD717-0A48-447D-90AA-A8F370B14F43@mac.com> Message-ID: <913B7E78-3052-4CF0-9883-C1CDD9F8BDFA@mac.com> please vote about which add ons you'd like to see covered next saturday: http://blog.livecode.tv/2011/07/poll-for-event-32/ -- official ChatRev page: http://bjoernke.com/chatrev Chat with other RunRev developers: go stack URL "http://bjoernke.com/chatrev/chatrev1.3b3.rev" From john at splash21.com Thu Jul 14 15:35:34 2011 From: john at splash21.com (John Craig) Date: Thu, 14 Jul 2011 20:35:34 +0100 Subject: Fun DropTool In-Reply-To: <4E1EE67E.3010000@splash21.com> References: <4E1EE67E.3010000@splash21.com> Message-ID: <4E1F4506.6060805@splash21.com> I've discovered that the head ( an oval graphic with startAngle = 0 and arcAngle = 180 ) gets displayed upside down on my android simulator, so the head is merging in with the body. If I change the startAngle to 180 then the effect is reversed: no head in the IDE, but I can see it in the simulator. I wonder what's different in Jacque's setup that it works - Any clues, Jacque?? I'll file a bug report. John. From andre at andregarzia.com Thu Jul 14 15:42:27 2011 From: andre at andregarzia.com (Andre Garzia) Date: Thu, 14 Jul 2011 16:42:27 -0300 Subject: Fun DropTool In-Reply-To: <4E1F4506.6060805@splash21.com> References: <4E1EE67E.3010000@splash21.com> <4E1F4506.6060805@splash21.com> Message-ID: paraphrasing his Steveness: "you are holding it wrong!" try holding the phone upside down... (joking) On Thu, Jul 14, 2011 at 4:35 PM, John Craig wrote: > I've discovered that the head ( an oval graphic with startAngle = 0 and > arcAngle = 180 ) gets displayed upside down on my android simulator, so the > head is merging in with the body. If I change the startAngle to 180 then > the effect is reversed: no head in the IDE, but I can see it in the > simulator. I wonder what's different in Jacque's setup that it works - Any > clues, Jacque?? I'll file a bug report. > > > John. > > > ______________________________**_________________ > use-livecode mailing list > use-livecode at lists.runrev.com > Please visit this url to subscribe, unsubscribe and manage your > subscription preferences: > http://lists.runrev.com/**mailman/listinfo/use-livecode > -- http://www.andregarzia.com All We Do Is Code. From jacque at hyperactivesw.com Thu Jul 14 15:53:32 2011 From: jacque at hyperactivesw.com (J. Landman Gay) Date: Thu, 14 Jul 2011 14:53:32 -0500 Subject: Fun DropTool In-Reply-To: <4E1F4506.6060805@splash21.com> References: <4E1EE67E.3010000@splash21.com> <4E1F4506.6060805@splash21.com> Message-ID: <4E1F493C.9070203@hyperactivesw.com> On 7/14/11 2:35 PM, John Craig wrote: > I've discovered that the head ( an oval graphic with startAngle = 0 and > arcAngle = 180 ) gets displayed upside down on my android simulator, so > the head is merging in with the body. If I change the startAngle to 180 > then the effect is reversed: no head in the IDE, but I can see it in the > simulator. I wonder what's different in Jacque's setup that it works - > Any clues, Jacque?? I'll file a bug report. Uh...I suspect the main difference is that I didn't notice the problem was only on the simulator, so I didn't test it there. Embarrassing. Ignore me. -- Jacqueline Landman Gay | jacque at hyperactivesw.com HyperActive Software | http://www.hyperactivesw.com From roger.e.eller at sealedair.com Thu Jul 14 17:02:04 2011 From: roger.e.eller at sealedair.com (Roger Eller) Date: Thu, 14 Jul 2011 17:02:04 -0400 Subject: Fun DropTool In-Reply-To: <4E1F3807.10201@splash21.com> References: <4E1EE67E.3010000@splash21.com> <4E1F2513.5010003@hyperactivesw.com> <4E1F3807.10201@splash21.com> Message-ID: On Thu, Jul 14, 2011 at 2:40 PM, John Craig wrote: > Bummer! - I get the same as Roger - an angry looking droid with big > eyebrows, and a missing head! (And the odd looking radius ends) > I'm set at platform 2.2 API level 8 - is this still the current setup? > > Just to rub salt in the wound, it renders perfectly on iOS!! > My off-list screenshot was of platform 2.3.1 API level 9 (I think). So, the result was the same. That salt burns. Where's quality assurance when you need 'em? ;) ~Roger From revolution at derbrill.de Thu Jul 14 17:11:10 2011 From: revolution at derbrill.de (Malte Brill) Date: Thu, 14 Jul 2011 23:11:10 +0200 Subject: Unicode again, this time comboboxes Message-ID: <1EC73B21-02B7-4AE6-A531-DCD41933A1B3@derbrill.de> Hi all, please bear with me, while I ask more (seemingly stupid) unicode questions. This time it is about comboboxes. The only way to get the choice of this is to ask for the selectedtext of btn "myCombobox). This will either return an UTF16 or an Ansii string. Now, is there a way to put that stuff into a variable and check which one it is? I really go nuts over all this stuff... All the best, Malte From m.schonewille at economy-x-talk.com Thu Jul 14 17:17:34 2011 From: m.schonewille at economy-x-talk.com (Mark Schonewille) Date: Thu, 14 Jul 2011 23:17:34 +0200 Subject: Unicode again, this time comboboxes In-Reply-To: <1EC73B21-02B7-4AE6-A531-DCD41933A1B3@derbrill.de> References: <1EC73B21-02B7-4AE6-A531-DCD41933A1B3@derbrill.de> Message-ID: <8C3FBFC5-B56A-4EC6-A165-89DB1AFC1676@economy-x-talk.com> Hi Malte, Can't you use the menuhistory and avoid using text? -- Best regards, Mark Schonewille Economy-x-Talk Consulting and Software Engineering Homepage: http://economy-x-talk.com Twitter: http://twitter.com/xtalkprogrammer KvK: 50277553 New: Download the Installer Maker Plugin 1.6 for LiveCode here http://qery.us/ce On 14 jul 2011, at 23:11, Malte Brill wrote: > Hi all, > > please bear with me, while I ask more (seemingly stupid) unicode questions. This time it is about comboboxes. > > The only way to get the choice of this is to ask for the selectedtext of btn "myCombobox). This will either return an UTF16 or an Ansii string. Now, is there a way to put that stuff into a variable and check which one it is? > > I really go nuts over all this stuff... > > All the best, > > > Malte From revolution at derbrill.de Thu Jul 14 17:21:23 2011 From: revolution at derbrill.de (Malte Brill) Date: Thu, 14 Jul 2011 23:21:23 +0200 Subject: Unicode again, this time comboboxes Message-ID: <0DBDF372-8C93-4AFE-9C21-FE511E51712B@derbrill.de> Hi Mark, not in a combobox I am afraid, as there is free text entry for that control... *sigh* From m.schonewille at economy-x-talk.com Thu Jul 14 17:25:02 2011 From: m.schonewille at economy-x-talk.com (Mark Schonewille) Date: Thu, 14 Jul 2011 23:25:02 +0200 Subject: Unicode again, this time comboboxes In-Reply-To: <0DBDF372-8C93-4AFE-9C21-FE511E51712B@derbrill.de> References: <0DBDF372-8C93-4AFE-9C21-FE511E51712B@derbrill.de> Message-ID: Hi Malte, As you might know, I made my own combobox (List Search Field). Perhaps you can look at how I did it and adjust it for unicode. That might be easier than trying to make the combobox work with unicode. (garlic) -- Best regards, Mark Schonewille Economy-x-Talk Consulting and Software Engineering Homepage: http://economy-x-talk.com Twitter: http://twitter.com/xtalkprogrammer KvK: 50277553 New: Download the Installer Maker Plugin 1.6 for LiveCode here http://qery.us/ce On 14 jul 2011, at 23:21, Malte Brill wrote: > Hi Mark, > > not in a combobox I am afraid, as there is free text entry for that control... > > *sigh* From slava at lexiconbridge.com Thu Jul 14 18:07:54 2011 From: slava at lexiconbridge.com (Slava Paperno) Date: Thu, 14 Jul 2011 18:07:54 -0400 Subject: Unicode again, this time comboboxes In-Reply-To: <1EC73B21-02B7-4AE6-A531-DCD41933A1B3@derbrill.de> References: <1EC73B21-02B7-4AE6-A531-DCD41933A1B3@derbrill.de> Message-ID: <001201cc4272$7e0bcfb0$7a236f10$@com> I tried to use Unicode in comboboxes and gave up because it is a hybrid between a button and a field. Perhaps someone else knows how to get around the problems, but I ended up simulating a combobox by using a one-line, no-wrap text input field and below it a list field that becomes visible when the user clicks in the inut field. This gives me access to the text of both fields. Slava > -----Original Message----- > From: use-livecode-bounces at lists.runrev.com [mailto:use-livecode- > bounces at lists.runrev.com] On Behalf Of Malte Brill > Sent: Thursday, July 14, 2011 5:11 PM > To: use-livecode at lists.runrev.com > Subject: Unicode again, this time comboboxes > > Hi all, > > please bear with me, while I ask more (seemingly stupid) unicode > questions. This time it is about comboboxes. > > The only way to get the choice of this is to ask for the selectedtext > of btn "myCombobox). This will either return an UTF16 or an Ansii > string. Now, is there a way to put that stuff into a variable and check > which one it is? > > I really go nuts over all this stuff... > > All the best, > > > Malte > _______________________________________________ > use-livecode mailing list > use-livecode at lists.runrev.com > Please visit this url to subscribe, unsubscribe and manage your > subscription preferences: > http://lists.runrev.com/mailman/listinfo/use-livecode From slava at lexiconbridge.com Thu Jul 14 18:10:55 2011 From: slava at lexiconbridge.com (Slava Paperno) Date: Thu, 14 Jul 2011 18:10:55 -0400 Subject: Unicode again, this time comboboxes In-Reply-To: References: <0DBDF372-8C93-4AFE-9C21-FE511E51712B@derbrill.de> Message-ID: <001901cc4272$ea4fe030$beefa090$@com> Mark--where can that List Search Field be seen? Slava > -----Original Message----- > From: use-livecode-bounces at lists.runrev.com [mailto:use-livecode- > bounces at lists.runrev.com] On Behalf Of Mark Schonewille > Sent: Thursday, July 14, 2011 5:25 PM > To: How to use LiveCode > Subject: Re: Unicode again, this time comboboxes > > Hi Malte, > > As you might know, I made my own combobox (List Search Field). Perhaps > you can look at how I did it and adjust it for unicode. That might be > easier than trying to make the combobox work with unicode. > > (garlic) > > -- > Best regards, > > Mark Schonewille > > Economy-x-Talk Consulting and Software Engineering > Homepage: http://economy-x-talk.com > Twitter: http://twitter.com/xtalkprogrammer > KvK: 50277553 > > New: Download the Installer Maker Plugin 1.6 for LiveCode here > http://qery.us/ce > > On 14 jul 2011, at 23:21, Malte Brill wrote: > > > Hi Mark, > > > > not in a combobox I am afraid, as there is free text entry for that > control... > > > > *sigh* > > > > _______________________________________________ > use-livecode mailing list > use-livecode at lists.runrev.com > Please visit this url to subscribe, unsubscribe and manage your > subscription preferences: > http://lists.runrev.com/mailman/listinfo/use-livecode From jhj at jhj.com Thu Jul 14 18:35:40 2011 From: jhj at jhj.com (Jerry J) Date: Thu, 14 Jul 2011 15:35:40 -0700 Subject: repeat with i= In-Reply-To: <8CE761D5-E040-464C-AE62-94A56AAFA487@clarkeandclarke.co.uk> References: <4E1E7777.8020308@hyperactivesw.com> <8CE761D5-E040-464C-AE62-94A56AAFA487@clarkeandclarke.co.uk> Message-ID: On Jul 14, 2011, at 2:12 AM, Keith Clarke wrote: > ...ah yes, of course, thanks Jim - I forgot that 'delete item y' is not the same as 'delete item *called* y' ! > > So, would itemoffset help...? > > put "1,2,3,4,5,6" into theNumbers > repeat until theNumbers is empty > put any item theNumbers into n > do something > delete (itemOffset(n, theNumbers)) from theNumbers > end repeat In the case of 1,2,3,4,5,6 itemOffset will work, but a warning about itemOffset: itemOffset("2","23,24,2") returns 1 because the string "2" matches the first 2 of 23. This is not applicable to this exercise, but might save somebody some trouble. In a previous project I assumed that it looked for a matching *item*, but in fact it looks to match a *string*. Arrgh. The docs are correct, by the way. --jhj From pete at mollysrevenge.com Thu Jul 14 18:51:25 2011 From: pete at mollysrevenge.com (Pete) Date: Thu, 14 Jul 2011 15:51:25 -0700 Subject: repeat with i= In-Reply-To: References: <4E1E7777.8020308@hyperactivesw.com> <8CE761D5-E040-464C-AE62-94A56AAFA487@clarkeandclarke.co.uk> Message-ID: Doesn't setting the wholematches to true deal with this? Pete Molly's Revenge On Thu, Jul 14, 2011 at 3:35 PM, Jerry J wrote: > On Jul 14, 2011, at 2:12 AM, Keith Clarke wrote: > > > ...ah yes, of course, thanks Jim - I forgot that 'delete item y' is not > the same as 'delete item *called* y' ! > > > > So, would itemoffset help...? > > > > put "1,2,3,4,5,6" into theNumbers > > repeat until theNumbers is empty > > put any item theNumbers into n > > do something > > delete (itemOffset(n, theNumbers)) from theNumbers > > end repeat > > In the case of 1,2,3,4,5,6 itemOffset will work, but a warning about > itemOffset: > itemOffset("2","23,24,2") returns 1 because the string "2" matches the > first 2 of 23. This is not applicable to this exercise, but might save > somebody some trouble. In a previous project I assumed that it looked for a > matching *item*, but in fact it looks to match a *string*. Arrgh. The docs > are correct, by the way. > > --jhj > > > _______________________________________________ > use-livecode mailing list > use-livecode at lists.runrev.com > Please visit this url to subscribe, unsubscribe and manage your > subscription preferences: > http://lists.runrev.com/mailman/listinfo/use-livecode > > From jhj at jhj.com Thu Jul 14 18:58:30 2011 From: jhj at jhj.com (Jerry J) Date: Thu, 14 Jul 2011 15:58:30 -0700 Subject: repeat with i= In-Reply-To: References: <4E1E7777.8020308@hyperactivesw.com> <8CE761D5-E040-464C-AE62-94A56AAFA487@clarkeandclarke.co.uk> Message-ID: <5825F329-4C90-4C34-A53D-738E2D1FF0AA@jhj.com> Aha! The docs say so! Learned something new today. Thanks Pete, Jerry On Jul 14, 2011, at 3:51 PM, Pete wrote: > Doesn't setting the wholematches to true deal with this? > Pete > Molly's Revenge > > > > > On Thu, Jul 14, 2011 at 3:35 PM, Jerry J wrote: > >> On Jul 14, 2011, at 2:12 AM, Keith Clarke wrote: >> >>> ...ah yes, of course, thanks Jim - I forgot that 'delete item y' is not >> the same as 'delete item *called* y' ! >>> >>> So, would itemoffset help...? >>> >>> put "1,2,3,4,5,6" into theNumbers >>> repeat until theNumbers is empty >>> put any item theNumbers into n >>> do something >>> delete (itemOffset(n, theNumbers)) from theNumbers >>> end repeat >> >> In the case of 1,2,3,4,5,6 itemOffset will work, but a warning about >> itemOffset: >> itemOffset("2","23,24,2") returns 1 because the string "2" matches the >> first 2 of 23. This is not applicable to this exercise, but might save >> somebody some trouble. In a previous project I assumed that it looked for a >> matching *item*, but in fact it looks to match a *string*. Arrgh. The docs >> are correct, by the way. >> >> --jhj >> >> >> _______________________________________________ >> use-livecode mailing list >> use-livecode at lists.runrev.com >> Please visit this url to subscribe, unsubscribe and manage your >> subscription preferences: >> http://lists.runrev.com/mailman/listinfo/use-livecode >> >> > _______________________________________________ > use-livecode mailing list > use-livecode at lists.runrev.com > Please visit this url to subscribe, unsubscribe and manage your subscription preferences: > http://lists.runrev.com/mailman/listinfo/use-livecode From pete at mollysrevenge.com Thu Jul 14 19:11:41 2011 From: pete at mollysrevenge.com (Pete) Date: Thu, 14 Jul 2011 16:11:41 -0700 Subject: repeat with i= In-Reply-To: <5825F329-4C90-4C34-A53D-738E2D1FF0AA@jhj.com> References: <4E1E7777.8020308@hyperactivesw.com> <8CE761D5-E040-464C-AE62-94A56AAFA487@clarkeandclarke.co.uk> <5825F329-4C90-4C34-A53D-738E2D1FF0AA@jhj.com> Message-ID: No problem. I'd definitely try it though - I've been bitten before by the docs saying things that weren't quite true! Pete Molly's Revenge On Thu, Jul 14, 2011 at 3:58 PM, Jerry J wrote: > Aha! The docs say so! Learned something new today. > > Thanks Pete, > Jerry > > On Jul 14, 2011, at 3:51 PM, Pete wrote: > > > Doesn't setting the wholematches to true deal with this? > > Pete > > Molly's Revenge > > > > > > > > > > On Thu, Jul 14, 2011 at 3:35 PM, Jerry J wrote: > > > >> On Jul 14, 2011, at 2:12 AM, Keith Clarke wrote: > >> > >>> ...ah yes, of course, thanks Jim - I forgot that 'delete item y' is not > >> the same as 'delete item *called* y' ! > >>> > >>> So, would itemoffset help...? > >>> > >>> put "1,2,3,4,5,6" into theNumbers > >>> repeat until theNumbers is empty > >>> put any item theNumbers into n > >>> do something > >>> delete (itemOffset(n, theNumbers)) from theNumbers > >>> end repeat > >> > >> In the case of 1,2,3,4,5,6 itemOffset will work, but a warning about > >> itemOffset: > >> itemOffset("2","23,24,2") returns 1 because the string "2" matches the > >> first 2 of 23. This is not applicable to this exercise, but might save > >> somebody some trouble. In a previous project I assumed that it looked > for a > >> matching *item*, but in fact it looks to match a *string*. Arrgh. The > docs > >> are correct, by the way. > >> > >> --jhj > >> > >> > >> _______________________________________________ > >> use-livecode mailing list > >> use-livecode at lists.runrev.com > >> Please visit this url to subscribe, unsubscribe and manage your > >> subscription preferences: > >> http://lists.runrev.com/mailman/listinfo/use-livecode > >> > >> > > _______________________________________________ > > use-livecode mailing list > > use-livecode at lists.runrev.com > > Please visit this url to subscribe, unsubscribe and manage your > subscription preferences: > > http://lists.runrev.com/mailman/listinfo/use-livecode > > > _______________________________________________ > use-livecode mailing list > use-livecode at lists.runrev.com > Please visit this url to subscribe, unsubscribe and manage your > subscription preferences: > http://lists.runrev.com/mailman/listinfo/use-livecode > > From revolution at derbrill.de Thu Jul 14 19:15:33 2011 From: revolution at derbrill.de (Malte Brill) Date: Fri, 15 Jul 2011 01:15:33 +0200 Subject: Unicode again, this time comboboxes Message-ID: The function mentioned by Ron Barber here: http://lists.runrev.com/pipermail/use-livecode//2011-June/157016.html helps. At least if it is either or... No good results for mixed languages though. local tUni,tAsUTF16 put the selectedText of btn "combo" into tUni put RawDataToUTF16(tUni) into tAsUTF16 *sigh*. Garlic... From jhj at jhj.com Thu Jul 14 19:17:53 2011 From: jhj at jhj.com (Jerry J) Date: Thu, 14 Jul 2011 16:17:53 -0700 Subject: repeat with i= In-Reply-To: References: <4E1E7777.8020308@hyperactivesw.com> <8CE761D5-E040-464C-AE62-94A56AAFA487@clarkeandclarke.co.uk> <5825F329-4C90-4C34-A53D-738E2D1FF0AA@jhj.com> Message-ID: <745822E1-48D8-4A70-BEEF-B5514A35D0A3@jhj.com> Works in the message box. On Jul 14, 2011, at 4:11 PM, Pete wrote: > No problem. I'd definitely try it though - I've been bitten before by the > docs saying things that weren't quite true! > Pete > Molly's Revenge > > > > > On Thu, Jul 14, 2011 at 3:58 PM, Jerry J wrote: > >> Aha! The docs say so! Learned something new today. >> >> Thanks Pete, >> Jerry >> >> On Jul 14, 2011, at 3:51 PM, Pete wrote: >> >>> Doesn't setting the wholematches to true deal with this? >>> Pete >>> Molly's Revenge >>> >>> >>> >>> >>> On Thu, Jul 14, 2011 at 3:35 PM, Jerry J wrote: >>> >>>> On Jul 14, 2011, at 2:12 AM, Keith Clarke wrote: >>>> >>>>> ...ah yes, of course, thanks Jim - I forgot that 'delete item y' is not >>>> the same as 'delete item *called* y' ! >>>>> >>>>> So, would itemoffset help...? >>>>> >>>>> put "1,2,3,4,5,6" into theNumbers >>>>> repeat until theNumbers is empty >>>>> put any item theNumbers into n >>>>> do something >>>>> delete (itemOffset(n, theNumbers)) from theNumbers >>>>> end repeat >>>> >>>> In the case of 1,2,3,4,5,6 itemOffset will work, but a warning about >>>> itemOffset: >>>> itemOffset("2","23,24,2") returns 1 because the string "2" matches the >>>> first 2 of 23. This is not applicable to this exercise, but might save >>>> somebody some trouble. In a previous project I assumed that it looked >> for a >>>> matching *item*, but in fact it looks to match a *string*. Arrgh. The >> docs >>>> are correct, by the way. >>>> >>>> --jhj >>>> >>>> >>>> _______________________________________________ >>>> use-livecode mailing list >>>> use-livecode at lists.runrev.com >>>> Please visit this url to subscribe, unsubscribe and manage your >>>> subscription preferences: >>>> http://lists.runrev.com/mailman/listinfo/use-livecode >>>> >>>> >>> _______________________________________________ >>> use-livecode mailing list >>> use-livecode at lists.runrev.com >>> Please visit this url to subscribe, unsubscribe and manage your >> subscription preferences: >>> http://lists.runrev.com/mailman/listinfo/use-livecode >> >> >> _______________________________________________ >> use-livecode mailing list >> use-livecode at lists.runrev.com >> Please visit this url to subscribe, unsubscribe and manage your >> subscription preferences: >> http://lists.runrev.com/mailman/listinfo/use-livecode >> >> > _______________________________________________ > use-livecode mailing list > use-livecode at lists.runrev.com > Please visit this url to subscribe, unsubscribe and manage your subscription preferences: > http://lists.runrev.com/mailman/listinfo/use-livecode From alex at tweedly.net Thu Jul 14 19:42:26 2011 From: alex at tweedly.net (Alex Tweedly) Date: Fri, 15 Jul 2011 00:42:26 +0100 Subject: [OT} Re: New post detailing the technology and decisions behind my new website In-Reply-To: References: Message-ID: <4E1F7EE2.2070306@tweedly.net> Sorry for using the list for this .... Andre - I tried to email you some feedback, initially to andre at andregarzia.com which gave me a "mail delivery failure" of "no such person at this address". I then tried contact at andregarzia.com (as specified in the Contact page of the site),and that too failed with the same error. -- Alex. On 13/07/2011 08:47, Andre Garzia wrote: > Folks, > > The blog is now live and has its first useful post. > > http://andregarzia.com/blog > > You can see a little journal on the decisions behind the system that drives > my new website by going to: > > http://andregarzia.com/page/bootstrappingacms > > It is a long read but it is useful for those trying to understand. > > Hope you guys like, any feedback is appreciated. > > Cheers > andre > From revolution at derbrill.de Thu Jul 14 20:04:23 2011 From: revolution at derbrill.de (Malte Brill) Date: Fri, 15 Jul 2011 02:04:23 +0200 Subject: Unicode again, this time comboboxes Message-ID: <322E9EB7-0924-4468-999C-4B2447D38FE4@derbrill.de> Forget what I said earlier. No good results at all. For now I give up. :( From bvg at mac.com Thu Jul 14 22:12:18 2011 From: bvg at mac.com (=?iso-8859-1?Q?Bj=F6rnke_von_Gierke?=) Date: Fri, 15 Jul 2011 04:12:18 +0200 Subject: [ANN] LiveCode.tv event #32 In-Reply-To: <913B7E78-3052-4CF0-9883-C1CDD9F8BDFA@mac.com> References: <6B865405-BBC0-491C-AD25-BFB1C85A2D58@mac.com> <7EC6A85B-DD6B-4138-A597-512A194F4453@mac.com> <89782C22-C6E3-4245-95D1-A48F69423753@mac.com> <061D9182-B34B-4ABB-A7D6-9C2D09DA8755@mac.com> <73F84954-890B-4627-859A-702C8054F13B@mac.com> <093DEF5E-6C0F-4924-A262-F6DBE748ED45@mac.com> <3FB852C9-5675-4A19-9506-543733BB2547@mac.com> <914DF999-596E-4023-A213-9469C6A69FA0@mac.com> <2B105965-FB27-41D0-B95F-1F4C84ADD53B@mac.com> <163225D1-67D1-4CE7-8049-E85A8D94D177@mac.com> <225B0941-5D11-434A-BC0B-CD61B998E9F8@mac.com> <201756EB-9601-417D-856E-5128C5256EF1@mac.com> <081FD717-0A48-447D-90AA-A8F370B14F43@mac.com> <913B7E78-3052-4CF0-9883-C1CDD9F8BDFA@mac.com> Message-ID: Join us once again, when we show LiveCode at it's best. This Saturday, Pete Haworth will update us on his qd[SQL] development. qd[SQL] is a currently in development, and will allow you to easily create databases without using any SQL at all, by linking LiveCodes object/group/card metaphor to a backend Database automatically. See also his explanation on Google Docs: https://docs.google.com/document/d/1Rl7B3JJrnmOGctXqd-OL26a37jN10cVD43lujyF26uE/edit?hl=en_US Then, the European HyperCard User Group (eHUG, http://www.ehug.info) will raffle off an e-book courtesy of TidBITS. This time around it?s ?Take Control of TextExpander?. You can find more info on the TC series here: http://www.takecontrolbooks.com Finally, Bj?rnke von Gierke will take you on a tour trough some of the add-ons from the 3 for 2 special deal ( http://www.runrev.com/mailers/3for2/3for2.html ), currently offered on the RunRev store with the coupon code "THREEFORTWO". Some of the creator of those plugins will also be available in chat, so keep those questions ready. Don't forget that you can vote on which plug-in you are interested in: http://blog.livecode.tv/2011/07/poll-for-event-32/ All this and more on Saturdays Livecode.tv show: Sat. 22:00 Moscow Sat. 20:00 Paris Sat. 19:00h GMT Sat. 14:00 New York Sat. 11:00 Los Angeles Sun. 04:00 Sydney Sun. 03:00 Tokyo Sun. 2:00 Beijing Make sure to join ChatRev on the right time, otherwise you won?t know where to watch, because the correct streams will be announced in ChatRev. ChatRev is open 24/7 for your chatting needs. Go to http://bjoernke.com/chatrev/ or enter in the message box: go stack URL ?http://bjoernke.com/chatrev/chatrev1.3b3.rev? If you would like to show something in an event, now or somewhen, please head over to the participation page for ideas, or contact us: http://blog.livecode.tv/participate/ http://www3.economy-x-talk.com/file.php?node=contact cheers Bj?rnke From andre at andregarzia.com Fri Jul 15 10:45:56 2011 From: andre at andregarzia.com (Andre Garzia) Date: Fri, 15 Jul 2011 11:45:56 -0300 Subject: [OT} Re: New post detailing the technology and decisions behind my new website In-Reply-To: <4E1F7EE2.2070306@tweedly.net> References: <4E1F7EE2.2070306@tweedly.net> Message-ID: Hey, I am here. That is my primary account that I use everyday. Strange. If you can't email me at that account, please send it to soapdog at mac dot com. Cheers Andre Sent from my iPad On Jul 14, 2011, at 8:42 PM, Alex Tweedly wrote: > Sorry for using the list for this .... > > Andre - I tried to email you some feedback, initially to andre at andregarzia.com which gave me a "mail delivery failure" of "no such person at this address". > > I then tried contact at andregarzia.com (as specified in the Contact page of the site),and that too failed with the same error. > > -- Alex. > > On 13/07/2011 08:47, Andre Garzia wrote: >> Folks, >> >> The blog is now live and has its first useful post. >> >> http://andregarzia.com/blog >> >> You can see a little journal on the decisions behind the system that drives >> my new website by going to: >> >> http://andregarzia.com/page/bootstrappingacms >> >> It is a long read but it is useful for those trying to understand. >> >> Hope you guys like, any feedback is appreciated. >> >> Cheers >> andre >> > > > _______________________________________________ > use-livecode mailing list > use-livecode at lists.runrev.com > Please visit this url to subscribe, unsubscribe and manage your subscription preferences: > http://lists.runrev.com/mailman/listinfo/use-livecode From revolution at derbrill.de Fri Jul 15 05:25:36 2011 From: revolution at derbrill.de (Malte Brill) Date: Fri, 15 Jul 2011 11:25:36 +0200 Subject: Unicode again, this time comboboxes References: <555F0D66-AE17-49C3-A981-6B40EA5BAF5C@DERBRILL.DE> Message-ID: Ok, getting a step further with this. This does work, but is rather ugly: To Read out a current label of the comboBox: on mouseUp lock screen local tProps,tSelection select the text of btn "Combo" focus on btn "Combo" answer the selection put the unicodeText of the selection into tProps set the unicodeText of fld "Test" to tProps unlock screen end mouseUp I have no idea why this works. I have no idea why setting the focus is necessary. Without the focus, it might break every once in a while. It works for both ANSII and UTF16 inputs. It works for mixed inputs. Just totally puzzled as to the why... Now trying to set stuff by script. Cheers, Malte From th.douez at gmail.com Fri Jul 15 10:05:41 2011 From: th.douez at gmail.com (Thierry Douez) Date: Fri, 15 Jul 2011 16:05:41 +0200 Subject: [ANN] alien camel in your LiveCode Message-ID: Hi, I'm pleased to announce SunnYperl. Download today a trial version from http://sunny-tdz.com Formerly called rev2perl, SunnYperl is an external which embeds Perl inside your LiveCode engine. Or in other words, you can write externals in Perl. For Perlers, it's a LiveCode proxy: fast and direct access to all LiveCode objects (widgets, handlers, properties,...) For LiveCoders, imagine using all the libraries you want from the CPAN(*) plus an easy communication with Perl. I've been using SunnYperl in the industry for years. One of my realizations is BioArchimed, a complex set of tools used by biologists (Cancer Research) on a daily basis since 2003. You can read more about it on my website. In few days, SunnYperl for MacIntel 10.6 will be available for purchase, versions for Linux and Windows OS will follow. Regards, Thierry Douez http://sunny-tdz.com - freelance developer * CPAN: Comprehensive Perl Archive Network ** Camel: Perl Logo From jhurley0305 at sbcglobal.net Fri Jul 15 10:47:42 2011 From: jhurley0305 at sbcglobal.net (James Hurley) Date: Fri, 15 Jul 2011 07:47:42 -0700 Subject: repeat with i= In-Reply-To: References: Message-ID: > -------------------------- > > Message: 7 > Date: Thu, 14 Jul 2011 10:06:01 -0500 > From: Ronald Zellner > To: use-livecode at lists.runrev.com > Subject: Re: repeat with i= > Message-ID: > Content-Type: text/plain; charset=us-ascii > > Jim's seems to be the most efficient and accurate, Ron, Have you compared replacing one array element with another? I believe it is much more efficient than deleting an item in a list. The engine has to do a lot of work to delete an item from a list. The whole list has to be reconstructed. Check out "Random Pick" in RevOnline. Jim Hurley > From rene.micout at numericable.com Fri Jul 15 11:42:12 2011 From: rene.micout at numericable.com (=?iso-8859-1?Q?Ren=E9_Micout?=) Date: Fri, 15 Jul 2011 17:42:12 +0200 Subject: [ANN] alien camel in your LiveCode In-Reply-To: References: Message-ID: <0B5E72EE-135C-43AF-BE30-ACD229B0CA49@numericable.com> Bravo Thierry, m?me si je n'y comprends pas grand chose (even if I do not understand much) Ren? Le 15 juil. 2011 ? 16:05, Thierry Douez a ?crit : > Hi, > > I'm pleased to announce SunnYperl. > > Download today a trial version from http://sunny-tdz.com > > Formerly called rev2perl, SunnYperl is an external > which embeds Perl inside your LiveCode engine. > > Or in other words, you can write externals in Perl. > > For Perlers, it's a LiveCode proxy: > fast and direct access to all LiveCode objects (widgets, handlers, > properties,...) > > For LiveCoders, imagine using all the libraries you want from the CPAN(*) > plus an easy communication with Perl. > > > I've been using SunnYperl in the industry for years. > One of my realizations is BioArchimed, > a complex set of tools used by biologists (Cancer Research) > on a daily basis since 2003. You can read more about it on my website. > > In few days, SunnYperl for MacIntel 10.6 will be available for purchase, > versions for Linux and Windows OS will follow. > > Regards, > > Thierry Douez > http://sunny-tdz.com - freelance developer > > > * CPAN: Comprehensive Perl Archive Network > ** Camel: Perl Logo > _______________________________________________ > use-livecode mailing list > use-livecode at lists.runrev.com > Please visit this url to subscribe, unsubscribe and manage your subscription preferences: > http://lists.runrev.com/mailman/listinfo/use-livecode From ruslan_zasukhin at valentina-db.com Fri Jul 15 14:00:16 2011 From: ruslan_zasukhin at valentina-db.com (Ruslan Zasukhin) Date: Fri, 15 Jul 2011 21:00:16 +0300 Subject: [ANN] iValentina 4.9 for iPhone and iPad APPROVED. FREE. Message-ID: Hi Guys, I want inform you that 4.9 update of iValentina was approved today. You can use get it for FREE here. ipad: http://itunes.apple.com/app/ivalentina-for-ipad/id415367982?mt=8 iphone: http://itunes.apple.com/app/ivalentina/id415381086?mt=8 ============== What's New in Version 4.9 * Added TAB panels * Added Keyboard Extension to type faster SQL commands * Compiled against 4.2 SDK, so * Added Multi-tasking support * Added Table Editor * Improved a lots speed of Table Editor thanks to caching in VCLIENT. -- Best regards, Ruslan Zasukhin VP Engineering and New Technology Paradigma Software, Inc Valentina - Joining Worlds of Information http://www.paradigmasoft.com [I feel the need: the need for speed] From pete at mollysrevenge.com Fri Jul 15 14:41:44 2011 From: pete at mollysrevenge.com (Pete) Date: Fri, 15 Jul 2011 11:41:44 -0700 Subject: Nested behaviors Message-ID: I'm not sure if I should expect this to work or not, but it doesn't! I have a custom column behavior for a datagrid column that contains an option menu. The option menu has a behavior set for it. In the custom column behavior FillInData handler, I dispatch a message to the option menu which should be handled by the option menu's behavior script, but it never makes it there and the contents of the "it" variable after the dispatch are "Unhandled". I guess I'm in a behavior (the datagrid's custom column behavior) when I send the message to the behavior for a different object. Am I expecting too much of LC's behavior concept? Pete Molly's Revenge From pete at mollysrevenge.com Fri Jul 15 16:29:13 2011 From: pete at mollysrevenge.com (Pete) Date: Fri, 15 Jul 2011 13:29:13 -0700 Subject: How/When are behavior references resolved? Message-ID: I'm having a problem with behaviors not being resolved in my application stack. I have two stack files, lets call them library.livecode and application.livecode. The script of the main stack of library.livecode has handlers that I use in application.livecode by issuing a "start using" command. library.livecode also contains a substack named Behaviors whose card has a bunch of buttons whose scripts I want to use as behaviors in application.livecode butthe "start using" command doesn't seem to make the behaviors available to the controls in application.livecode, by which I mean they act like they don't have a behavior defined. I can fix this by setting the stackFiles of the main stack of application.livecode to "Behaviors,". I have to reference the Behaviors substack, not the main stack of library.livecode for this to work. I guess I'm wondering if this is the appropriate way to deal with this situation or if there's a cleaner way to do it. Pete Molly's Revenge From richmondmathewson at gmail.com Fri Jul 15 16:47:22 2011 From: richmondmathewson at gmail.com (Richmond Mathewson) Date: Fri, 15 Jul 2011 23:47:22 +0300 Subject: [OT] calling non-numbered Unicode glyphs Message-ID: <4E20A75A.40601@gmail.com> Normally if one wants to put a unicode char into a field one does something like this: set the unicodeText of fld "MyBigFatUnicodeTextFld" to numToChat(2327) & numToChar(9745) or set the unicodeText of fld "MyBigFatUnicodeTextFld" to the unicodeText of fld "MyBigFatUnicodeTextFld" & numToChar(67234) and that works reasonably well . . . HOWEVER; if one cracks open a Unicode font with a font editing program such as Fontographer [the best way to see this is to use the LastResort font from the Unicode consortium] one finds that there are glyphs/chars that belong to font families that have names such as "kannada_31" but no hex address that is required for LiveCode to call it. Now one CAN generate Unicode addresses for those chars: but the big and burning question is if those ARE the real Unicode addresses or just locally assigned values. From johnpatten at mac.com Fri Jul 15 17:52:37 2011 From: johnpatten at mac.com (John Patten) Date: Fri, 15 Jul 2011 14:52:37 -0700 Subject: Viewing Large Images Using Groups and Adding Buttons to the Map Group? Message-ID: Hi All! I have a few questions about large images, buttons and groups.... I have a card that I want to place a large map image on. I know I can put it into a group, size the group, and turn the scroll bars on to navigate the large image. Is it possible to navigate map in a group with just by dragging/Grab me? Similar to what you might due on a tablet and iOS/Android? I need to be able to navigate the image and have my button markers stick to the image not float on top. Now, if I attach a "grab me" to the image, the image moves but the button marker float above. If i attach a grab me to the group, the whole group moves but not the map. Ideas? Also, if i want to add cd buttons to the map group one at a time, how would I go about doing that? Lock screen set the rect of group "map" to the rect of image "largeMapImage" put 1 into x repeat 5 add cd button x to group "map" of cd 1 -- this does not seem to work for me??? set the loc of cd button x to line x of tListofLocations -- retrieving the loc coordinations from a database and storing them in tListLocations add 1 to x end repeat (Most importantly, how do I add buttons to a the map group, with out throwing off the locs of the current button markers in the group.) The buttons need to be buttons because they are used to navigate to information regarding that specific location on the map and some can change their loc via user placement (grab me). In order to do what I want to do, is it only possible by assigning the buttons to the map group (?) and then using the scroll bars of the group to navigate the map? Would that work inside iOS and Android (that = group scroll bars) too? Thank you! John Patten SUSD From scott at tactilemedia.com Fri Jul 15 18:15:51 2011 From: scott at tactilemedia.com (Scott Rossi) Date: Fri, 15 Jul 2011 15:15:51 -0700 Subject: Viewing Large Images Using Groups and Adding Buttons to the Map Group? In-Reply-To: Message-ID: Hi John: Maybe this is a start for what you're trying to do: {in your message box) go url "http://www.tactilemedia.com/download/dragscroll.rev" Regards, Scott Rossi Creative Director Tactile Media, UX Design Recently, John Patten wrote: > Hi All! > > I have a few questions about large images, buttons and groups.... > > I have a card that I want to place a large map image on. I know I can > put it into a group, size the group, and turn the scroll bars on to > navigate the large image. > > Is it possible to navigate map in a group with just by dragging/Grab > me? Similar to what you might due on a tablet and iOS/Android? I need > to be able to navigate the image and have my button markers stick to > the image not float on top. > > Now, if I attach a "grab me" to the image, the image moves but the > button marker float above. If i attach a grab me to the group, the > whole group moves but not the map. Ideas? > > > Also, if i want to add cd buttons to the map group one at a time, how > would I go about doing that? > > Lock screen > set the rect of group "map" to the rect of image "largeMapImage" > put 1 into x > repeat 5 > > add cd button x to group "map" of cd 1 -- this does not seem to work > for me??? > > set the loc of cd button x to line x of tListofLocations -- retrieving > the loc coordinations from a database and storing them in tListLocations > add 1 to x > end repeat > > (Most importantly, how do I add buttons to a the map group, with out > throwing off the locs of the current button markers in the group.) > > The buttons need to be buttons because they are used to navigate to > information regarding that specific location on the map and some can > change their loc via user placement (grab me). > > In order to do what I want to do, is it only possible by assigning the > buttons to the map group (?) and then using the scroll bars of the > group to navigate the map? Would that work inside iOS and Android > (that = group scroll bars) too? > > Thank you! > > John Patten > SUSD > > > > > > > > > > > _______________________________________________ > use-livecode mailing list > use-livecode at lists.runrev.com > Please visit this url to subscribe, unsubscribe and manage your subscription > preferences: > http://lists.runrev.com/mailman/listinfo/use-livecode From brent at juicydevelopment.com Fri Jul 15 18:17:48 2011 From: brent at juicydevelopment.com (Brent Anderson) Date: Fri, 15 Jul 2011 16:17:48 -0600 Subject: Viewing Large Images Using Groups and Adding Buttons to the Map Group? In-Reply-To: References: Message-ID: <4DD0486D-5DB5-4E1A-B2E1-D643272ED7BE@juicydevelopment.com> I actually did this on iOS recently. Incidentally, I scrapped the grouped map/button concept in favor of an HTML implementation (which, depending on your application, could be a good idea). But, if you're interested in using buttons and an image in a grouped implementation, I used two groups. One group, called "mapLayer", contained the map and pins I dropped on the map. This group contained the "grab me" code, and it was also full size (no scrollbars). It was then grouped within another group called "mapContainer". This "mapContainer" group was of fixed size and was the "window" to the map, including scrollbars, etc. You cannot group a group using LiveCode's development environment, but you can use the message box to issue a group command to stick the mapLayer inside the mapContainer: group grp "mapLayer" Then rename the resulting group as "mapContainer". Alternatively, you can create another object like a button and group it with the mapLayer group. Edit this new group, delete the button so all that is left is the mapLayer, and rename your new group "mapContainer" (or whatever you prefer, these were the names I used). Cheers, Brent Anderson Juicy Development On Jul 15, 2011, at 3:52 PM, John Patten wrote: > Hi All! > > I have a few questions about large images, buttons and groups.... > > I have a card that I want to place a large map image on. I know I can put it into a group, size the group, and turn the scroll bars on to navigate the large image. > > Is it possible to navigate map in a group with just by dragging/Grab me? Similar to what you might due on a tablet and iOS/Android? I need to be able to navigate the image and have my button markers stick to the image not float on top. > > Now, if I attach a "grab me" to the image, the image moves but the button marker float above. If i attach a grab me to the group, the whole group moves but not the map. Ideas? > > > Also, if i want to add cd buttons to the map group one at a time, how would I go about doing that? > > Lock screen > set the rect of group "map" to the rect of image "largeMapImage" > put 1 into x > repeat 5 > > add cd button x to group "map" of cd 1 -- this does not seem to work for me??? > > set the loc of cd button x to line x of tListofLocations -- retrieving the loc coordinations from a database and storing them in tListLocations > add 1 to x > end repeat > > (Most importantly, how do I add buttons to a the map group, with out throwing off the locs of the current button markers in the group.) > > The buttons need to be buttons because they are used to navigate to information regarding that specific location on the map and some can change their loc via user placement (grab me). > > In order to do what I want to do, is it only possible by assigning the buttons to the map group (?) and then using the scroll bars of the group to navigate the map? Would that work inside iOS and Android (that = group scroll bars) too? > > Thank you! > > John Patten > SUSD > > > > > > > > > > > _______________________________________________ > use-livecode mailing list > use-livecode at lists.runrev.com > Please visit this url to subscribe, unsubscribe and manage your subscription preferences: > http://lists.runrev.com/mailman/listinfo/use-livecode From m.schonewille at economy-x-talk.com Fri Jul 15 18:29:07 2011 From: m.schonewille at economy-x-talk.com (Mark Schonewille) Date: Sat, 16 Jul 2011 00:29:07 +0200 Subject: [OT] calling non-numbered Unicode glyphs In-Reply-To: <4E20A75A.40601@gmail.com> References: <4E20A75A.40601@gmail.com> Message-ID: <5E2013EA-5809-42F9-A9E6-DDFFD38911EB@economy-x-talk.com> Hi Richmond, I think that Fontographer will show you the UTF8 values of the font values. You can take those values and convert them to binary using the binaryEncode function. Convert the binary value to UTF8. It is probably easier to use Mac OS X's Character Viewer. This palette will allowyou to view the code tables. You can look up unicode character 31f0 for example. On intel machines, you need to reverse this hex number before you can use it. 31f0 becomes f031, e201 becomes 01e2 etc. I believe you don't need to do this on a PowerMac. If you use these functions, you can just enter a sequence of hex numbers and set the unicodetext of a field to the value returned. function hexToUnicode theHex put binaryEncode("H*",theHex) into myUnicode if the processor contains "x86" then repeat with x = 1 to the number of bytes of myUnicode step 2 put char x+1 of myUnicode & char x of myUnicode after myNewUnicode end repeat return myNewUnicode else return myUnicode end if end hexToUnicode function unicodeToHex theUnicode if the processor contains "x86" then repeat with x = 1 to the number of bytes of theUnicode step 2 put char x+1 of theUnicode & char x of theUnicode after myNewUnicode end repeat if binaryDecode("H*",myNewUnicode,myHex) is 1 then return myHex end if else if binaryDecode("H*",theUnicode,myHex) is 1 then return myHex end if end if return empty end unicodeToHex Here's an example you might like: on mouseUp put empty into fld 1 set the textFont of fld 1 to empty set the traversalOn of fld 1 to true put hexToUnicode("041f044004380432043504423000042004380449043c044e043d0434") into myUnicode lock screen set the unicodeText of fld 1 to myUnicode set the textFont of fld 1 to ",Cyrillic" select the text of fld 1 set the textSize of the selection to 18 select empty unlock screen end mouseUp Unfortunately, Unicode in LiveCode is a very, very dirty thing. Hence all those hacks, such as put empty into fld 1, select the text of fld 1, etc. -- Best regards, Mark Schonewille Economy-x-Talk Consulting and Software Engineering Homepage: http://economy-x-talk.com Twitter: http://twitter.com/xtalkprogrammer KvK: 50277553 New: Download the Installer Maker Plugin 1.6 for LiveCode here http://qery.us/ce On 15 jul 2011, at 22:47, Richmond Mathewson wrote: > Normally if one wants to put a unicode char into a field one does something > like this: > > set the unicodeText of fld "MyBigFatUnicodeTextFld" to numToChat(2327) & numToChar(9745) > > or > > set the unicodeText of fld "MyBigFatUnicodeTextFld" to the unicodeText of fld "MyBigFatUnicodeTextFld" & numToChar(67234) > > and that works reasonably well . . . > > HOWEVER; if one cracks open a Unicode font with a font editing program such as > Fontographer > > [the best way to see this is to use the LastResort font from the Unicode consortium] > > one finds that there are glyphs/chars that belong to font families that have names > such as "kannada_31" but no hex address that is required for LiveCode to call it. > > Now one CAN generate Unicode addresses for those chars: but the big and burning question is if those ARE the real Unicode addresses or just locally assigned values. > > > _______________________________________________ > use-livecode mailing list > use-livecode at lists.runrev.com > Please visit this url to subscribe, unsubscribe and manage your subscription preferences: > http://lists.runrev.com/mailman/listinfo/use-livecode From m.schonewille at economy-x-talk.com Fri Jul 15 18:32:59 2011 From: m.schonewille at economy-x-talk.com (Mark Schonewille) Date: Sat, 16 Jul 2011 00:32:59 +0200 Subject: [OT] calling non-numbered Unicode glyphs In-Reply-To: <5E2013EA-5809-42F9-A9E6-DDFFD38911EB@economy-x-talk.com> References: <4E20A75A.40601@gmail.com> <5E2013EA-5809-42F9-A9E6-DDFFD38911EB@economy-x-talk.com> Message-ID: Oh darn, that first sentence.... let me rewrite that. I think that Fontographer will show you the UTF8 values of the glyphs. You can take those values and convert them to binary using the binaryEncode function. Convert the binary value to UTF16. -- Good night, Mark Schonewille Economy-x-Talk Consulting and Software Engineering Homepage: http://economy-x-talk.com Twitter: http://twitter.com/xtalkprogrammer KvK: 50277553 New: Download the Installer Maker Plugin 1.6 for LiveCode here http://qery.us/ce On 16 jul 2011, at 00:29, Mark Schonewille wrote: > Hi Richmond, > > I think that Fontographer will show you the UTF8 values of the font values. You can take those values and convert them to binary using the binaryEncode function. Convert the binary value to UTF8. > > It is probably easier to use Mac OS X's Character Viewer. This palette will allowyou to view the code tables. You can look up unicode character 31f0 for example. On intel machines, you need to reverse this hex number before you can use it. 31f0 becomes f031, e201 becomes 01e2 etc. I believe you don't need to do this on a PowerMac. > > If you use these functions, you can just enter a sequence of hex numbers and set the unicodetext of a field to the value returned. From gandalf at doctorTimothyMiller.com Fri Jul 15 18:40:23 2011 From: gandalf at doctorTimothyMiller.com (Timothy Miller) Date: Fri, 15 Jul 2011 15:40:23 -0700 Subject: repeat with i= In-Reply-To: References: Message-ID: Thanks for all the suggestions. I think I'll go with: on mouseUp local theNums put "1,2,3,4,5,6" into theNums sort items of theNums numeric by random(1000000) repeat with i = 1 to 6 do "whatever" & item i of theNums end repeat end mouseUp This is probably erroneous but possibly the right general idea. There are better ways to do it, no doubt. The advantage is I'll understand it if I have to work on it again. Best regards, Tim From pete at mollysrevenge.com Fri Jul 15 18:56:41 2011 From: pete at mollysrevenge.com (Pete) Date: Fri, 15 Jul 2011 15:56:41 -0700 Subject: repeat with i= In-Reply-To: References: Message-ID: Sounds like the best solution. Thanks for posting the question - I learend about "delete", "any", and more about "sort"! Pete Molly's Revenge On Fri, Jul 15, 2011 at 3:40 PM, Timothy Miller < gandalf at doctortimothymiller.com> wrote: > Thanks for all the suggestions. > > I think I'll go with: > > on mouseUp > local theNums > put "1,2,3,4,5,6" into theNums > sort items of theNums numeric by random(1000000) > repeat with i = 1 to 6 > do "whatever" & item i of theNums > end repeat > end mouseUp > > This is probably erroneous but possibly the right general idea. There are > better ways to do it, no doubt. The advantage is I'll understand it if I > have to work on it again. > > Best regards, > > > Tim > _______________________________________________ > use-livecode mailing list > use-livecode at lists.runrev.com > Please visit this url to subscribe, unsubscribe and manage your > subscription preferences: > http://lists.runrev.com/mailman/listinfo/use-livecode > > From roger.e.eller at sealedair.com Fri Jul 15 19:02:06 2011 From: roger.e.eller at sealedair.com (Roger Eller) Date: Fri, 15 Jul 2011 19:02:06 -0400 Subject: repeat with i= In-Reply-To: References: Message-ID: On Fri, Jul 15, 2011 at 6:56 PM, Pete wrote: > Sounds like the best solution. Thanks for posting the question - I learned > about "delete", "any", and more about "sort"! > Pete > Molly's Revenge > These little challenges are great for the LC community. I learn something every time one shows up here. Something like this would be a great episode on livecode.tv with full audience participation to solve a puzzle or find the most efficient way. ?Roger From kray at sonsothunder.com Fri Jul 15 19:59:35 2011 From: kray at sonsothunder.com (Ken Ray) Date: Fri, 15 Jul 2011 18:59:35 -0500 Subject: How/When are behavior references resolved? In-Reply-To: References: Message-ID: <34773085-5E4E-41AE-9F9C-427B2D98AB31@sonsothunder.com> On Jul 15, 2011, at 3:29 PM, Pete wrote: > I'm having a problem with behaviors not being resolved in my application > stack. > > I have two stack files, lets call them library.livecode and > application.livecode. The script of the main stack of library.livecode has > handlers that I use in application.livecode by issuing a "start using" > command. > > library.livecode also contains a substack named Behaviors whose card has a > bunch of buttons whose scripts I want to use as behaviors in > application.livecode butthe "start using" command doesn't seem to make the > behaviors available to the controls in application.livecode, by which I mean > they act like they don't have a behavior defined. > > I can fix this by setting the stackFiles of the main stack of > application.livecode to "Behaviors,". I have to > reference the Behaviors substack, not the main stack of library.livecode for > this to work. > > I guess I'm wondering if this is the appropriate way to deal with this > situation or if there's a cleaner way to do it. You're right - putting a stack into use doesn't load it into memory (e.g. it's not in the mainstacks), so the behaviors go unresolved. Here's two ways to do it (other than the one you described): 1) Open the library invisibly (I do this by trapping "on libraryStack": on libraryStack if the target is me then open inv stack (the short name of me) end if end libraryStack 2) The "datagrid" trick - if you use "there is a stack" then it loads a stack into memory but doesn't actually open it: get (there is a stack ) Ken Ray Sons of Thunder Software, Inc. Email: kray at sonsothunder.com Web Site: http://www.sonsothunder.com/ From johnpatten at mac.com Fri Jul 15 20:09:08 2011 From: johnpatten at mac.com (John Patten) Date: Fri, 15 Jul 2011 17:09:08 -0700 Subject: Floating City and K12 Game Theory Model... Message-ID: <65848462-D50C-4B62-AB9D-BA2EFD26AA34@mac.com> Thanks Scott and Brent...I'll look at both of your solutions regarding large images and groups... FWIW, I've been looking at how an online game called The Floating City (http://www.floatingcity.com was put together...and how some of the principle game activities could be incorporated into an instructional model in the K12 education environment. Naturally, the easiest way to duplicate their model is doing it in LiveCode ;-) I put together the following example in a very short amount of time. Obviously, if I actually create something I'll use it will take much longer, designing the narrative, graphics, securing the code, etc. etc. But I was pretty much floored how quickly and how easy it was to create with LiveCode. I think I only posted questions to the list a couple of times! :-) I've put down more of a description of what I'm attempting to do here if interested: http://edutonica.blogspot.com/ Thanks everyone! John Patten SUSD From kray at sonsothunder.com Fri Jul 15 20:15:38 2011 From: kray at sonsothunder.com (Ken Ray) Date: Fri, 15 Jul 2011 19:15:38 -0500 Subject: Viewing Large Images Using Groups and Adding Buttons to the Map Group? In-Reply-To: <4DD0486D-5DB5-4E1A-B2E1-D643272ED7BE@juicydevelopment.com> References: <4DD0486D-5DB5-4E1A-B2E1-D643272ED7BE@juicydevelopment.com> Message-ID: > You cannot group a group using LiveCode's development environment, but you can use the message box to issue a group command to stick the mapLayer inside the mapContainer: Actually you *can*, but you need to add a dummy object, select them both, group them, and then delete the dummy object. It's easier to do with the message box, of course... Ken Ray Sons of Thunder Software, Inc. Email: kray at sonsothunder.com Web Site: http://www.sonsothunder.com/ From dunbarx at aol.com Fri Jul 15 22:07:31 2011 From: dunbarx at aol.com (dunbarx at aol.com) Date: Fri, 15 Jul 2011 22:07:31 -0400 (EDT) Subject: How/When are behavior references resolved? In-Reply-To: <34773085-5E4E-41AE-9F9C-427B2D98AB31@sonsothunder.com> References: <34773085-5E4E-41AE-9F9C-427B2D98AB31@sonsothunder.com> Message-ID: <8CE117441578253-1E48-538A1@Webmail-m110.sysops.aol.com> I believe putting a stack into use will only place the stack script in the hierarchy. Objects, like buttons, will not be "seen". Craig Newman -----Original Message----- From: Ken Ray To: How to use LiveCode Sent: Fri, Jul 15, 2011 4:02 pm Subject: Re: How/When are behavior references resolved? On Jul 15, 2011, at 3:29 PM, Pete wrote: > I'm having a problem with behaviors not being resolved in my application > stack. > > I have two stack files, lets call them library.livecode and > application.livecode. The script of the main stack of library.livecode has > handlers that I use in application.livecode by issuing a "start using" > command. > > library.livecode also contains a substack named Behaviors whose card has a > bunch of buttons whose scripts I want to use as behaviors in > application.livecode butthe "start using" command doesn't seem to make the > behaviors available to the controls in application.livecode, by which I mean > they act like they don't have a behavior defined. > > I can fix this by setting the stackFiles of the main stack of > application.livecode to "Behaviors,". I have to > reference the Behaviors substack, not the main stack of library.livecode for > this to work. > > I guess I'm wondering if this is the appropriate way to deal with this > situation or if there's a cleaner way to do it. You're right - putting a stack into use doesn't load it into memory (e.g. it's not in the mainstacks), so the behaviors go unresolved. Here's two ways to do it (other than the one you described): 1) Open the library invisibly (I do this by trapping "on libraryStack": on libraryStack if the target is me then open inv stack (the short name of me) end if end libraryStack 2) The "datagrid" trick - if you use "there is a stack" then it loads a stack into memory but doesn't actually open it: get (there is a stack ) Ken Ray Sons of Thunder Software, Inc. Email: kray at sonsothunder.com Web Site: http://www.sonsothunder.com/ _______________________________________________ use-livecode mailing list use-livecode at lists.runrev.com Please visit this url to subscribe, unsubscribe and manage your subscription preferences: http://lists.runrev.com/mailman/listinfo/use-livecode From bvlahos at mac.com Fri Jul 15 22:54:50 2011 From: bvlahos at mac.com (Bill Vlahos) Date: Fri, 15 Jul 2011 19:54:50 -0700 Subject: How to detect zoombox click Message-ID: <3BE8C33E-08C0-4656-BAD7-5654FDCE7F7B@mac.com> I want to detect when a user clicks the zoombox (green light on a Mac) and do something similar to iTunes where if you click the zoombox the program doesn't just resize itself but goes into a different mode. The only message I see is a resizeStack message but that happens if the user resizes the window manually. Bill Vlahos _________________ InfoWallet (http://www.infowallet.com) is about keeping your important life information with you, accessible, and secure. From jacque at hyperactivesw.com Fri Jul 15 22:59:01 2011 From: jacque at hyperactivesw.com (J. Landman Gay) Date: Fri, 15 Jul 2011 21:59:01 -0500 Subject: repeat with i= In-Reply-To: References: Message-ID: <4E20FE75.70509@hyperactivesw.com> On 7/15/11 6:02 PM, Roger Eller wrote: > On Fri, Jul 15, 2011 at 6:56 PM, Pete wrote: > >> Sounds like the best solution. Thanks for posting the question - I learned >> about "delete", "any", and more about "sort"! >> Pete >> Molly's Revenge >> > > These little challenges are great for the LC community. I learn something > every time one shows up here. Something like this would be a great episode > on livecode.tv with full audience participation to solve a puzzle or find > the most efficient way. We used to do this on the HyperCard list and it was a big hit. Every once in a while someone would just make up a scripting challenge to solve, and everyone would pile on. It was fun. -- Jacqueline Landman Gay | jacque at hyperactivesw.com HyperActive Software | http://www.hyperactivesw.com From revdev at pdslabs.net Sat Jul 16 00:39:52 2011 From: revdev at pdslabs.net (Phil Davis) Date: Fri, 15 Jul 2011 21:39:52 -0700 Subject: How/When are behavior references resolved? In-Reply-To: <8CE117441578253-1E48-538A1@Webmail-m110.sysops.aol.com> References: <34773085-5E4E-41AE-9F9C-427B2D98AB31@sonsothunder.com> <8CE117441578253-1E48-538A1@Webmail-m110.sysops.aol.com> Message-ID: <4E211618.9070204@pdslabs.net> On 7/15/11 7:07 PM, dunbarx at aol.com wrote: > I believe putting a stack into use will only place the stack script in the hierarchy. Objects, like buttons, will not be "seen". > > > Craig Newman Yet after they are in use, the objects they contain (images at least) can be successfully referenced as button icons in other stacks that are open. Phil Davis > > > > > -----Original Message----- > From: Ken Ray > To: How to use LiveCode > Sent: Fri, Jul 15, 2011 4:02 pm > Subject: Re: How/When are behavior references resolved? > > > > On Jul 15, 2011, at 3:29 PM, Pete wrote: > >> I'm having a problem with behaviors not being resolved in my application >> stack. >> >> I have two stack files, lets call them library.livecode and >> application.livecode. The script of the main stack of library.livecode has >> handlers that I use in application.livecode by issuing a "start using" >> command. >> >> library.livecode also contains a substack named Behaviors whose card has a >> bunch of buttons whose scripts I want to use as behaviors in >> application.livecode butthe "start using" command doesn't seem to make the >> behaviors available to the controls in application.livecode, by which I mean >> they act like they don't have a behavior defined. >> >> I can fix this by setting the stackFiles of the main stack of >> application.livecode to "Behaviors,". I have to >> reference the Behaviors substack, not the main stack of library.livecode for >> this to work. >> >> I guess I'm wondering if this is the appropriate way to deal with this >> situation or if there's a cleaner way to do it. > You're right - putting a stack into use doesn't load it into memory (e.g. it's > not in the mainstacks), so the behaviors go unresolved. Here's two ways to do it > (other than the one you described): > > 1) Open the library invisibly (I do this by trapping "on libraryStack": > > on libraryStack > if the target is me then > open inv stack (the short name of me) > end if > end libraryStack > > 2) The "datagrid" trick - if you use "there is a stack" then it loads a stack > into memory but doesn't actually open it: > > get (there is a stack) > > > > Ken Ray > Sons of Thunder Software, Inc. > Email: kray at sonsothunder.com > Web Site: http://www.sonsothunder.com/ > > _______________________________________________ > use-livecode mailing list > use-livecode at lists.runrev.com > Please visit this url to subscribe, unsubscribe and manage your subscription > preferences: > http://lists.runrev.com/mailman/listinfo/use-livecode > > > > _______________________________________________ > use-livecode mailing list > use-livecode at lists.runrev.com > Please visit this url to subscribe, unsubscribe and manage your subscription preferences: > http://lists.runrev.com/mailman/listinfo/use-livecode > -- Phil Davis PDS Labs Professional Software Development http://pdslabs.net From john.allijn at alice.nl Sat Jul 16 02:29:24 2011 From: john.allijn at alice.nl (John Allijn) Date: Sat, 16 Jul 2011 08:29:24 +0200 Subject: [ANN] iOS and Android apps Message-ID: <1F7C1EFB-CF74-44A7-874D-1FE13262B690@alice.nl> Hi, Apple just approved two updates of my apps: READ is now universal (one executable for iPhone and iPad). It has also some minor updates in the way it displays text. The app can be downloaded here: http://itunes.apple.com/us/app/read/id433493823?ls=1&mt=8 For the Dutch readers on the list: Dordrecht.HD is a city walk through Hollands oldest city. I did some minor updates on the interface and corrected quite a few typo's. http://itunes.apple.com/us/app/dordrecht.hd/id418542297?ls=1&mt=8 New for me is the release of READ for Android. Except for some minor adjustments (the rounded corners of rectangles display differently on android then on iOS) publication was easy. Here is a link to the app in the marketplace: https://market.android.com/details?id=com.allijn.read&feature=search_result regards, John From shaosean at wehostmacs.com Sat Jul 16 08:42:16 2011 From: shaosean at wehostmacs.com (Shao Sean) Date: Sat, 16 Jul 2011 08:42:16 -0400 Subject: How/When are behavior references resolved? Message-ID: This was the information given when behaviors (aka parentScripts) were introduced in 4.0, not certain how much has changed in later releases as I have not upgraded.. You might want to see if they ever did get around to adding the "resolutionError" message and see if it is getting fired (which will help you track down the issue).. PARENT SCRIPTS - RESOLUTION A control's parent script reference is saved in the stackfile as three pieces of information: 1) button id 2) stack name 3) mainstack name (if stack is substack) This is the minimum required information to uniquely identify a button within a running Revolution environment. Immediately after loading a stack file, an attempt is made to resolve all parentScript references - the engine acts as if it constructs a control reference: button id of stack [ of stack ] And attempts to access it. Thus, the stackFiles property will be searched as appropriate and any needed stacks will be loaded. At present, if a parentScript fails to resolve it fails silently. In the next dp, the engine will send a 'resolutionError' message in a similar way to how it sends a 'scriptError' message when a loaded stack's script fails to compile. Once resolution has failed for a parentScript reference, no attempt will be made to resolve it again *unless* the parentScript of an object is set to the same reference. A parentScript reference does *not* track changes to the name of the stack and/or substack - if the names of stacks change that contain objects being used as parentScripts then all parentScript references to those objects will be broken. This apparant strictness of parentScript resolution is necessary to ensure that references remain consistent throughout a Revolution session. From richmondmathewson at gmail.com Sat Jul 16 10:24:49 2011 From: richmondmathewson at gmail.com (Richmond Mathewson) Date: Sat, 16 Jul 2011 17:24:49 +0300 Subject: [OT] calling non-numbered Unicode glyphs In-Reply-To: References: <4E20A75A.40601@gmail.com> <5E2013EA-5809-42F9-A9E6-DDFFD38911EB@economy-x-talk.com> Message-ID: <4E219F31.5010004@gmail.com> That tip about the Mac OS character viewer is worth buckets: Thanks a lot! > Oh darn, that first sentence.... let me rewrite that. > > I think that Fontographer will show you the UTF8 values of the glyphs. You can take those values and convert them to binary using the binaryEncode function. Convert the binary value to UTF16. > > -- > Good night, > > Mark Schonewille > > Economy-x-Talk Consulting and Software Engineering > Homepage: http://economy-x-talk.com > Twitter: http://twitter.com/xtalkprogrammer > KvK: 50277553 > > New: Download the Installer Maker Plugin 1.6 for LiveCode here http://qery.us/ce > > On 16 jul 2011, at 00:29, Mark Schonewille wrote: > >> Hi Richmond, >> >> I think that Fontographer will show you the UTF8 values of the font values. You can take those values and convert them to binary using the binaryEncode function. Convert the binary value to UTF8. >> >> It is probably easier to use Mac OS X's Character Viewer. This palette will allowyou to view the code tables. You can look up unicode character 31f0 for example. On intel machines, you need to reverse this hex number before you can use it. 31f0 becomes f031, e201 becomes 01e2 etc. I believe you don't need to do this on a PowerMac. >> >> If you use these functions, you can just enter a sequence of hex numbers and set the unicodetext of a field to the value returned. > > _______________________________________________ > use-livecode mailing list > use-livecode at lists.runrev.com > Please visit this url to subscribe, unsubscribe and manage your subscription preferences: > http://lists.runrev.com/mailman/listinfo/use-livecode From ambassador at fourthworld.com Sat Jul 16 11:16:20 2011 From: ambassador at fourthworld.com (Richard Gaskin) Date: Sat, 16 Jul 2011 08:16:20 -0700 Subject: How/When are behavior references resolved? In-Reply-To: References: Message-ID: <4E21AB44.1010301@fourthworld.com> Behaviors are very powerful, but also very picky about how and when they're initially resolved. If you don't architect just so, they will not be resolved correctly and your code won't work. I've submitted a request which would give developer much greater flexibility and minimal cost to RunRev: Add "resolve behaviors" command The idea here is that when the proposed "resolve behaviors" command is called, it merely triggers the existing code in the engine which resolves behavior references. This would be exactly as what happens when a stack is first loaded, but by being able to invoke resolution at any time the developer has much more flexibility with WHEN that resolution happens, fixing a good many initialization issues developers experience with behaviors. -- Richard Gaskin Fourth World LiveCode training and consulting: http://www.fourthworld.com Webzine for LiveCode developers: http://www.LiveCodeJournal.com LiveCode Journal blog: http://LiveCodejournal.com/blog.irv From richmondmathewson at gmail.com Sat Jul 16 11:22:09 2011 From: richmondmathewson at gmail.com (Richmond Mathewson) Date: Sat, 16 Jul 2011 18:22:09 +0300 Subject: Latest Unicode "smile" Message-ID: <4E21ACA1.1010409@gmail.com> So there I am merrily entering vowel signs into my textFld with scripts rather like this; set the unicodeText of the selectedText to (numToChar(105) & numToChar(772)) and you can see the result at: http://andregarzia.on-rev.com/richmond/INPROGRESS/jumpy.jpg as you will see; simple vowels seem to 'fly' above those with overscore diacritics which is, frankly, a farce! Apart from the obvious joke about long vowels being weighed down by their diacritics, does anyone have any idea what is going on, and how to sort this out/ Richmond. From pete at mollysrevenge.com Sat Jul 16 12:34:18 2011 From: pete at mollysrevenge.com (Pete) Date: Sat, 16 Jul 2011 09:34:18 -0700 Subject: How/When are behavior references resolved? In-Reply-To: <4E21AB44.1010301@fourthworld.com> References: <4E21AB44.1010301@fourthworld.com> Message-ID: Great idea, I just added my 5 votes for it. This thing is driving me crazy and will affect the livecode tv presentation I'm doing later this morning. Not insurmountable, just means restarting LC when I shouldn't need to. Pete Molly's Revenge On Sat, Jul 16, 2011 at 8:16 AM, Richard Gaskin wrote: > Behaviors are very powerful, but also very picky about how and when they're > initially resolved. If you don't architect just so, they will not be > resolved correctly and your code won't work. > > I've submitted a request which would give developer much greater > flexibility and minimal cost to RunRev: > > Add "resolve behaviors" command > > > > > The idea here is that when the proposed "resolve behaviors" command is > called, it merely triggers the existing code in the engine which resolves > behavior references. This would be exactly as what happens when a stack is > first loaded, but by being able to invoke resolution at any time the > developer has much more flexibility with WHEN that resolution happens, > fixing a good many initialization issues developers experience with > behaviors. > > -- > Richard Gaskin > Fourth World > LiveCode training and consulting: http://www.fourthworld.com > Webzine for LiveCode developers: http://www.LiveCodeJournal.com > LiveCode Journal blog: http://LiveCodejournal.com/**blog.irv > > > ______________________________**_________________ > use-livecode mailing list > use-livecode at lists.runrev.com > Please visit this url to subscribe, unsubscribe and manage your > subscription preferences: > http://lists.runrev.com/**mailman/listinfo/use-livecode > > From bvg at mac.com Sat Jul 16 12:55:58 2011 From: bvg at mac.com (=?iso-8859-1?Q?Bj=F6rnke_von_Gierke?=) Date: Sat, 16 Jul 2011 18:55:58 +0200 Subject: [ANN] LiveCode.tv event #32 In-Reply-To: References: <6B865405-BBC0-491C-AD25-BFB1C85A2D58@mac.com> <7EC6A85B-DD6B-4138-A597-512A194F4453@mac.com> <89782C22-C6E3-4245-95D1-A48F69423753@mac.com> <061D9182-B34B-4ABB-A7D6-9C2D09DA8755@mac.com> <73F84954-890B-4627-859A-702C8054F13B@mac.com> <093DEF5E-6C0F-4924-A262-F6DBE748ED45@mac.com> <3FB852C9-5675-4A19-9506-543733BB2547@mac.com> <914DF999-596E-4023-A213-9469C6A69FA0@mac.com> <2B105965-FB27-41D0-B95F-1F4C84ADD53B@mac.com> <163225D1-67D1-4CE7-8049-E85A8D94D177@mac.com> <225B0941-5D11-434A-BC0B-CD61B998E9F8@mac.com> <201756EB-9601-417D-856E-5128C5256EF1@mac.com> <081FD717-0A48-447D-90AA-A8F370B14F43@mac.com> <913B7E78-3052-4CF0-9883-C1CDD9F8BDFA@mac.com> Message-ID: Reminder: The event will commence in about an hour. Everyone go and vote (even if you won't attend): http://blog.livecode.tv/2011/07/poll-for-event-32/ On 15 Jul 2011, at 04:12, Bj?rnke von Gierke wrote: > Join us once again, when we show LiveCode at it's best. > > This Saturday, Pete Haworth will update us on his qd[SQL] development. qd[SQL] is a currently in development, and will allow you to easily create databases without using any SQL at all, by linking LiveCodes object/group/card metaphor to a backend Database automatically. > > See also his explanation on Google Docs: https://docs.google.com/document/d/1Rl7B3JJrnmOGctXqd-OL26a37jN10cVD43lujyF26uE/edit?hl=en_US > > Then, the European HyperCard User Group (eHUG, http://www.ehug.info) will raffle off an e-book courtesy of TidBITS. This time around it?s ?Take Control of TextExpander?. > You can find more info on the TC series here: > http://www.takecontrolbooks.com > > Finally, Bj?rnke von Gierke will take you on a tour trough some of the add-ons from the 3 for 2 special deal ( http://www.runrev.com/mailers/3for2/3for2.html ), currently offered on the RunRev store with the coupon code "THREEFORTWO". Some of the creator of those plugins will also be available in chat, so keep those questions ready. > > Don't forget that you can vote on which plug-in you are interested in: > http://blog.livecode.tv/2011/07/poll-for-event-32/ > > > All this and more on Saturdays Livecode.tv show: > Sat. 22:00 Moscow > Sat. 20:00 Paris > Sat. 19:00h GMT > Sat. 14:00 New York > Sat. 11:00 Los Angeles > Sun. 04:00 Sydney > Sun. 03:00 Tokyo > Sun. 2:00 Beijing > > Make sure to join ChatRev on the right time, otherwise you won?t know where to watch, because the correct streams will be announced in ChatRev. ChatRev is open 24/7 for your chatting needs. > Go to > http://bjoernke.com/chatrev/ > or enter in the message box: > go stack URL ?http://bjoernke.com/chatrev/chatrev1.3b3.rev? > > If you would like to show something in an event, now or somewhen, please head over to the participation page for ideas, or contact us: > http://blog.livecode.tv/participate/ > http://www3.economy-x-talk.com/file.php?node=contact > > cheers > Bj?rnke > _______________________________________________ > use-livecode mailing list > use-livecode at lists.runrev.com > Please visit this url to subscribe, unsubscribe and manage your subscription preferences: > http://lists.runrev.com/mailman/listinfo/use-livecode From ambassador at fourthworld.com Sat Jul 16 13:16:47 2011 From: ambassador at fourthworld.com (Richard Gaskin) Date: Sat, 16 Jul 2011 10:16:47 -0700 Subject: Floating City and K12 Game Theory Model... In-Reply-To: <65848462-D50C-4B62-AB9D-BA2EFD26AA34@mac.com> References: <65848462-D50C-4B62-AB9D-BA2EFD26AA34@mac.com> Message-ID: <4E21C77F.6070205@fourthworld.com> John Patten wrote: > I've put down more of a description of what I'm attempting to do here > if interested: http://edutonica.blogspot.com/ A good read - thanks for posting that. You may find this article interesting, about a shared simulation environment built in LiveCode by Richard Herz: -- Richard Gaskin Fourth World LiveCode training and consulting: http://www.fourthworld.com Webzine for LiveCode developers: http://www.LiveCodeJournal.com LiveCode Journal blog: http://LiveCodejournal.com/blog.irv From pete at mollysrevenge.com Sat Jul 16 13:31:07 2011 From: pete at mollysrevenge.com (Pete) Date: Sat, 16 Jul 2011 10:31:07 -0700 Subject: How/When are behavior references resolved? In-Reply-To: References: Message-ID: Thanks Sean. I tried trapping the ResolutionError message but never got triggered so I guess they didn't put it in there. Pete Molly's Revenge On Sat, Jul 16, 2011 at 5:42 AM, Shao Sean wrote: > This was the information given when behaviors (aka parentScripts) were > introduced in 4.0, not certain how much has changed in later releases as I > have not upgraded.. You might want to see if they ever did get around to > adding the "resolutionError" message and see if it is getting fired (which > will help you track down the issue).. > > > > PARENT SCRIPTS - RESOLUTION > > A control's parent script reference is saved in the stackfile as three > pieces of information: > 1) button id > 2) stack name > 3) mainstack name (if stack is substack) > This is the minimum required information to uniquely identify a button > within a running Revolution environment. > > Immediately after loading a stack file, an attempt is made to resolve all > parentScript references - the engine acts as if it constructs a control > reference: > button id of stack [ of stack ] > And attempts to access it. Thus, the stackFiles property will be searched > as appropriate and any needed stacks will be loaded. > > At present, if a parentScript fails to resolve it fails silently. In the > next dp, the engine will send a 'resolutionError' message in a similar way > to how it sends a 'scriptError' message when a loaded stack's script fails > to compile. Once resolution has failed for a parentScript reference, no > attempt will be made to resolve it again *unless* the parentScript of an > object is set to the same reference. > > A parentScript reference does *not* track changes to the name of the stack > and/or substack - if the names of stacks change that contain objects being > used as parentScripts then all parentScript references to those objects will > be broken. > > This apparant strictness of parentScript resolution is necessary to ensure > that references remain consistent throughout a Revolution session. > > > ______________________________**_________________ > use-livecode mailing list > use-livecode at lists.runrev.com > Please visit this url to subscribe, unsubscribe and manage your > subscription preferences: > http://lists.runrev.com/**mailman/listinfo/use-livecode > > From pete at mollysrevenge.com Sat Jul 16 13:37:07 2011 From: pete at mollysrevenge.com (Pete) Date: Sat, 16 Jul 2011 10:37:07 -0700 Subject: How/When are behavior references resolved? In-Reply-To: <34773085-5E4E-41AE-9F9C-427B2D98AB31@sonsothunder.com> References: <34773085-5E4E-41AE-9F9C-427B2D98AB31@sonsothunder.com> Message-ID: Thanks Ken, I'm still having issues with this and I suspect it's because of just when I issue these commands. Right now, the "start using" command is the preOpenCard handler of the first card of my main stack. If I refresh the Application Browser, I see my library file listed there. I added the "open invisible" command right after the "start using" but that still didn't fix the problem. The only way I can make things work reliably right now is to manually open my library file before opening my application file, which is OK for now in my development environment but need to find a better solution somehow. Pete Molly's Revenge On Fri, Jul 15, 2011 at 4:59 PM, Ken Ray wrote: > > On Jul 15, 2011, at 3:29 PM, Pete wrote: > > > I'm having a problem with behaviors not being resolved in my application > > stack. > > > > I have two stack files, lets call them library.livecode and > > application.livecode. The script of the main stack of library.livecode > has > > handlers that I use in application.livecode by issuing a "start using" > > command. > > > > library.livecode also contains a substack named Behaviors whose card has > a > > bunch of buttons whose scripts I want to use as behaviors in > > application.livecode butthe "start using" command doesn't seem to make > the > > behaviors available to the controls in application.livecode, by which I > mean > > they act like they don't have a behavior defined. > > > > I can fix this by setting the stackFiles of the main stack of > > application.livecode to "Behaviors,". I have > to > > reference the Behaviors substack, not the main stack of library.livecode > for > > this to work. > > > > I guess I'm wondering if this is the appropriate way to deal with this > > situation or if there's a cleaner way to do it. > > You're right - putting a stack into use doesn't load it into memory (e.g. > it's not in the mainstacks), so the behaviors go unresolved. Here's two ways > to do it (other than the one you described): > > 1) Open the library invisibly (I do this by trapping "on libraryStack": > > on libraryStack > if the target is me then > open inv stack (the short name of me) > end if > end libraryStack > > 2) The "datagrid" trick - if you use "there is a stack" then it loads a > stack into memory but doesn't actually open it: > > get (there is a stack ) > > > > Ken Ray > Sons of Thunder Software, Inc. > Email: kray at sonsothunder.com > Web Site: http://www.sonsothunder.com/ > > _______________________________________________ > use-livecode mailing list > use-livecode at lists.runrev.com > Please visit this url to subscribe, unsubscribe and manage your > subscription preferences: > http://lists.runrev.com/mailman/listinfo/use-livecode > > From john at splash21.com Sat Jul 16 15:20:56 2011 From: john at splash21.com (John Craig) Date: Sat, 16 Jul 2011 20:20:56 +0100 Subject: Problem testing on android simulator (Windows) Message-ID: <4E21E498.5060902@splash21.com> I just set up LiveCode + JDK + android SDK on a Windows machine. Everything looks OK, android AVD manager is running, Test target is set, but when I click 'Test' I get the following; "Unable to build app for testing: could not generate package manifest" Is there another step I need to take somewhere? Thanks for any tips, John. From roger.e.eller at sealedair.com Sat Jul 16 15:56:44 2011 From: roger.e.eller at sealedair.com (Roger Eller) Date: Sat, 16 Jul 2011 15:56:44 -0400 Subject: Problem testing on android simulator (Windows) In-Reply-To: <4E21E498.5060902@splash21.com> References: <4E21E498.5060902@splash21.com> Message-ID: On Sat, Jul 16, 2011 at 3:20 PM, John Craig wrote: > I just set up LiveCode + JDK + android SDK on a Windows machine. > Everything looks OK, android AVD manager is running, Test target is set, > but when I click 'Test' I get the following; > > "Unable to build app for testing: could not generate package manifest" > > Is there another step I need to take somewhere? > > > Thanks for any tips, > > John. > Even though the AVD manager is running, you 'must' have a virtual Android device fully booted, and have it selected under the Development > Test Target menu. ?Roger From john at splash21.com Sat Jul 16 16:06:59 2011 From: john at splash21.com (John Craig) Date: Sat, 16 Jul 2011 21:06:59 +0100 Subject: Problem testing on android simulator (Windows) In-Reply-To: References: <4E21E498.5060902@splash21.com> Message-ID: <4E21EF63.9070702@splash21.com> It's selected under "Dev => Test target" and the emulator is running on screen. The test button is enabled, but when I click it I get; Building classes... Assembling assets... The it stops with the "Unable to build app for testing: could not generate package manifest" error :( On 16/07/2011 20:56, Roger Eller wrote: > On Sat, Jul 16, 2011 at 3:20 PM, John Craig wrote: > >> I just set up LiveCode + JDK + android SDK on a Windows machine. >> Everything looks OK, android AVD manager is running, Test target is set, >> but when I click 'Test' I get the following; >> >> "Unable to build app for testing: could not generate package manifest" >> >> Is there another step I need to take somewhere? >> >> >> Thanks for any tips, >> >> John. >> > Even though the AVD manager is running, you 'must' have a virtual Android > device fully booted, and have it selected under the Development> Test > Target menu. > > ?Roger > _______________________________________________ > use-livecode mailing list > use-livecode at lists.runrev.com > Please visit this url to subscribe, unsubscribe and manage your subscription preferences: > http://lists.runrev.com/mailman/listinfo/use-livecode > From roger.e.eller at sealedair.com Sat Jul 16 16:28:32 2011 From: roger.e.eller at sealedair.com (Roger Eller) Date: Sat, 16 Jul 2011 16:28:32 -0400 Subject: Problem testing on android simulator (Windows) In-Reply-To: <4E21EF63.9070702@splash21.com> References: <4E21E498.5060902@splash21.com> <4E21EF63.9070702@splash21.com> Message-ID: On Sat, Jul 16, 2011 at 4:06 PM, John Craig wrote: > It's selected under "Dev => Test target" and the emulator is running on > screen. > The test button is enabled, but when I click it I get; > > Building classes... > Assembling assets... > > The it stops with the "Unable to build app for testing: could not generate > package manifest" error > > > :( > > > On 16/07/2011 20:56, Roger Eller wrote: > >> On Sat, Jul 16, 2011 at 3:20 PM, John Craig wrote: >> >> I just set up LiveCode + JDK + android SDK on a Windows machine. >>> Everything looks OK, android AVD manager is running, Test target is set, >>> but when I click 'Test' I get the following; >>> >>> "Unable to build app for testing: could not generate package manifest" >>> >>> Is there another step I need to take somewhere? >>> >>> >>> Thanks for any tips, >>> >>> John. >>> >>> Even though the AVD manager is running, you 'must' have a virtual >> Android >> device fully booted, and have it selected under the Development> Test >> Target menu. >> >> ?Roger >> > Do you have a unique identifier for "com.yourcompany.yourapp", and what other requirements have you included, and are you signing the app for development only? All under the Standalone Application Settings? ?Roger From john at splash21.com Sat Jul 16 16:46:20 2011 From: john at splash21.com (John Craig) Date: Sat, 16 Jul 2011 21:46:20 +0100 Subject: Problem testing on android simulator (Windows) In-Reply-To: References: <4E21E498.5060902@splash21.com> <4E21EF63.9070702@splash21.com> Message-ID: <4E21F89C.7010401@splash21.com> I've tried with a unique id, not signing with anything - just want to run in emulator. All works OK on Mac - does Windows require anything extra? I don't see anything so far in the android release notes. :| > Do you have a unique identifier for "com.yourcompany.yourapp", and what > other requirements have you included, and are you signing the app for > development only? All under the Standalone Application Settings? > > ?Roger From roger.e.eller at sealedair.com Sat Jul 16 17:01:09 2011 From: roger.e.eller at sealedair.com (Roger Eller) Date: Sat, 16 Jul 2011 17:01:09 -0400 Subject: Problem testing on android simulator (Windows) In-Reply-To: <4E21F89C.7010401@splash21.com> References: <4E21E498.5060902@splash21.com> <4E21EF63.9070702@splash21.com> <4E21F89C.7010401@splash21.com> Message-ID: On Sat, Jul 16, 2011 at 4:46 PM, John Craig wrote: > I've tried with a unique id, not signing with anything - just want to run > in emulator. > All works OK on Mac - does Windows require anything extra? I don't see > anything so far in the android release notes. > > :| That's so frustrating. I'm sorry I don't know what else to check. It has worked the same for me on both WinXP and Mac. Is your AVD built with all the same options as on your Mac? Try making a new AVD with minimal options. I hope Mark Waddingham or Jacque will chime in and save the day. :) ?Roger From john at splash21.com Sat Jul 16 17:48:09 2011 From: john at splash21.com (John Craig) Date: Sat, 16 Jul 2011 22:48:09 +0100 Subject: Problem testing on android simulator (Windows) In-Reply-To: References: <4E21E498.5060902@splash21.com> <4E21EF63.9070702@splash21.com> <4E21F89C.7010401@splash21.com> Message-ID: <9B1373F1-EA6D-4942-978E-B9A3788235AF@splash21.com> Thanks for the suggestions, Roger. I'll have another go tomorrow! Sent from my iPhone On 16 Jul 2011, at 22:01, Roger Eller wrote: > On Sat, Jul 16, 2011 at 4:46 PM, John Craig wrote: > >> I've tried with a unique id, not signing with anything - just want to run >> in emulator. >> All works OK on Mac - does Windows require anything extra? I don't see >> anything so far in the android release notes. >> >> :| > > > That's so frustrating. I'm sorry I don't know what else to check. It has > worked the same for me on both WinXP and Mac. Is your AVD built with all > the same options as on your Mac? Try making a new AVD with minimal options. > I hope Mark Waddingham or Jacque will chime in and save the day. :) > > ?Roger > _______________________________________________ > use-livecode mailing list > use-livecode at lists.runrev.com > Please visit this url to subscribe, unsubscribe and manage your subscription preferences: > http://lists.runrev.com/mailman/listinfo/use-livecode From shaosean at wehostmacs.com Sat Jul 16 18:08:47 2011 From: shaosean at wehostmacs.com (Shao Sean) Date: Sat, 16 Jul 2011 18:08:47 -0400 Subject: How/When are behavior references resolved? Message-ID: For compiled applications you should make use of the "startUp" message to load libraries and whatnot that you need to have in memory for your application.. http://docs.runrev.com/Message/startup From pete at mollysrevenge.com Sat Jul 16 19:13:09 2011 From: pete at mollysrevenge.com (Pete) Date: Sat, 16 Jul 2011 16:13:09 -0700 Subject: How/When are behavior references resolved? In-Reply-To: References: Message-ID: Thanks Sean, I just found that message eariler today and it will be useful for standalones I seem to be back in a stable situation again with the "start using" and defining my behaviors stack filepath in the Stackfiles property. I've tried so many things in the last few hours, I honestly don;t know why it stopped working in the first place and what I did to make it start working again. I hate that because if it happesn again, I will have no idea how to fix it but for now, all is well! Pete Molly's Revenge On Sat, Jul 16, 2011 at 3:08 PM, Shao Sean wrote: > For compiled applications you should make use of the "startUp" message to > load libraries and whatnot that you need to have in memory for your > application.. > > http://docs.runrev.com/**Message/startup > > > ______________________________**_________________ > use-livecode mailing list > use-livecode at lists.runrev.com > Please visit this url to subscribe, unsubscribe and manage your > subscription preferences: > http://lists.runrev.com/**mailman/listinfo/use-livecode > > From jacque at hyperactivesw.com Sat Jul 16 21:45:53 2011 From: jacque at hyperactivesw.com (J. Landman Gay) Date: Sat, 16 Jul 2011 20:45:53 -0500 Subject: Problem testing on android simulator (Windows) In-Reply-To: References: <4E21E498.5060902@splash21.com> <4E21EF63.9070702@splash21.com> <4E21F89C.7010401@splash21.com> Message-ID: <4E223ED1.4050504@hyperactivesw.com> On 7/16/11 4:01 PM, Roger Eller wrote: > On Sat, Jul 16, 2011 at 4:46 PM, John Craig wrote: > >> I've tried with a unique id, not signing with anything - just want to run >> in emulator. >> All works OK on Mac - does Windows require anything extra? I don't see >> anything so far in the android release notes. >> >> :| > > > That's so frustrating. I'm sorry I don't know what else to check. It has > worked the same for me on both WinXP and Mac. Is your AVD built with all > the same options as on your Mac? Try making a new AVD with minimal options. > I hope Mark Waddingham or Jacque will chime in and save the day. :) I can't chime in, sorry. I couldn't even get the emulator to run in Windows and I gave up. It works on my Mac so I just stayed with that. Could it be a permissions thing? It's a stab in the dark. -- Jacqueline Landman Gay | jacque at hyperactivesw.com HyperActive Software | http://www.hyperactivesw.com From roger.e.eller at sealedair.com Sat Jul 16 22:16:25 2011 From: roger.e.eller at sealedair.com (Roger Eller) Date: Sat, 16 Jul 2011 22:16:25 -0400 Subject: Problem testing on android simulator (Windows) In-Reply-To: <4E223ED1.4050504@hyperactivesw.com> References: <4E21E498.5060902@splash21.com> <4E21EF63.9070702@splash21.com> <4E21F89C.7010401@splash21.com> <4E223ED1.4050504@hyperactivesw.com> Message-ID: On Sat, Jul 16, 2011 at 9:45 PM, J. Landman Gay wrote: > On 7/16/11 4:01 PM, Roger Eller wrote: > >> That's so frustrating. I'm sorry I don't know what else to check. It has >> worked the same for me on both WinXP and Mac. Is your AVD built with all >> the same options as on your Mac? Try making a new AVD with minimal >> options. >> I hope Mark Waddingham or Jacque will chime in and save the day. :) >> > > I can't chime in, sorry. I couldn't even get the emulator to run in Windows > and I gave up. It works on my Mac so I just stayed with that. > > Could it be a permissions thing? It's a stab in the dark. > > -- > Jacqueline Landman Gay | jacque at hyperactivesw.com > HyperActive Software | http://www.hyperactivesw.com I haven't seen any bugzilla reports of it failing to test on Windows. I wonder what conditions cause it to break. ?Roger From dunbarx at aol.com Sat Jul 16 23:01:50 2011 From: dunbarx at aol.com (dunbarx at aol.com) Date: Sat, 16 Jul 2011 23:01:50 -0400 (EDT) Subject: How to detect zoombox click In-Reply-To: <3BE8C33E-08C0-4656-BAD7-5654FDCE7F7B@mac.com> References: <3BE8C33E-08C0-4656-BAD7-5654FDCE7F7B@mac.com> Message-ID: <8CE12450228C4AD-1E48-5FECD@Webmail-m110.sysops.aol.com> A movestack message is also sent. Not sure why, though. Might this help, since resizing the window manually does not send this message? Craig Newman -----Original Message----- From: Bill Vlahos To: How to use LiveCode Sent: Fri, Jul 15, 2011 6:57 pm Subject: How to detect zoombox click I want to detect when a user clicks the zoombox (green light on a Mac) and do something similar to iTunes where if you click the zoombox the program doesn't just resize itself but goes into a different mode. The only message I see is a resizeStack message but that happens if the user resizes the window manually. Bill Vlahos _________________ InfoWallet (http://www.infowallet.com) is about keeping your important life information with you, accessible, and secure. _______________________________________________ use-livecode mailing list use-livecode at lists.runrev.com Please visit this url to subscribe, unsubscribe and manage your subscription preferences: http://lists.runrev.com/mailman/listinfo/use-livecode From mfstuart at cox.net Sat Jul 16 23:14:27 2011 From: mfstuart at cox.net (Mark Stuart) Date: Sat, 16 Jul 2011 20:14:27 -0700 Subject: [ANN] iOS and Android apps Message-ID: <8BB1992E24184EB08F1B2BFEECD04C2D@stuart> Hi John, I'd like to download your app to my Android phone, But after using the search word "READ!", there were so many in the result list I gave up. Are there some key words that I can search for your app with? Regards, Mark Stuart From pepetoo at cox.net Sat Jul 16 23:42:21 2011 From: pepetoo at cox.net (Joe Lewis Wilkins) Date: Sat, 16 Jul 2011 20:42:21 -0700 Subject: spam Message-ID: <8EF270B6-DFDC-49F6-AFB0-34E8C6E7723F@cox.net> Have you people noticed an excessive number of spam emails from gMail accounts in the pass couple of weeks? Others too. I flag them as spam with my ISP (cox), but it does no good. I'm using Apple's Mail application. Any help with this would be greatly appreciated. I'm about to drop Cox and an email address I've been using for at least 15 years because of this. TIA, Joe Wilkins From bvlahos at mac.com Sat Jul 16 23:41:58 2011 From: bvlahos at mac.com (Bill Vlahos) Date: Sat, 16 Jul 2011 20:41:58 -0700 Subject: How to detect zoombox click In-Reply-To: <8CE12450228C4AD-1E48-5FECD@Webmail-m110.sysops.aol.com> References: <3BE8C33E-08C0-4656-BAD7-5654FDCE7F7B@mac.com> <8CE12450228C4AD-1E48-5FECD@Webmail-m110.sysops.aol.com> Message-ID: Craig, That is interesting. Dragging the stack window also generates a movestack message just like resizing a window generates a resizestack message. Is there a way to test for a simultaneous movestack and resizestack messages? Unless I find a unique message for that control detecting simultaneous movestack and resizestack messages should do the trick. On my Mac the resizeStack message is sent first then the moveStack message. The following script works but seems like a huge kludge. Is that order consistent between different versions or Rev/LC and on different platforms? To test put this script in the stack and create a field called "one" and then manipulate the stack window. global vZoom on resizeStack put the seconds into vZoom end resizeStack on moveStack if vZoom - the seconds = 0 then put "ZoomBox click" into field "one" else put "Move or resize" into field "one" end moveStack Is there a more elegant way to detect simultaneous messages? Bill Vlahos _________________ InfoWallet (http://www.infowallet.com) is about keeping your important life information with you, accessible, and secure. On Jul 16, 2011, at 8:01 PM, DunbarX at aol.com wrote: > A movestack message is also sent. Not sure why, though. > > > Might this help, since resizing the window manually does not send this message? > > > Craig Newman > > > > > > -----Original Message----- > From: Bill Vlahos > To: How to use LiveCode > Sent: Fri, Jul 15, 2011 6:57 pm > Subject: How to detect zoombox click > > > I want to detect when a user clicks the zoombox (green light on a Mac) and do > something similar to iTunes where if you click the zoombox the program doesn't > just resize itself but goes into a different mode. > > The only message I see is a resizeStack message but that happens if the user > resizes the window manually. > > Bill Vlahos > _________________ > InfoWallet (http://www.infowallet.com) is about keeping your important life > information with you, accessible, and secure. > > > _______________________________________________ > use-livecode mailing list > use-livecode at lists.runrev.com > Please visit this url to subscribe, unsubscribe and manage your subscription > preferences: > http://lists.runrev.com/mailman/listinfo/use-livecode > > > > _______________________________________________ > use-livecode mailing list > use-livecode at lists.runrev.com > Please visit this url to subscribe, unsubscribe and manage your subscription preferences: > http://lists.runrev.com/mailman/listinfo/use-livecode From pete at mollysrevenge.com Sun Jul 17 00:23:17 2011 From: pete at mollysrevenge.com (Pete) Date: Sat, 16 Jul 2011 21:23:17 -0700 Subject: spam In-Reply-To: <8EF270B6-DFDC-49F6-AFB0-34E8C6E7723F@cox.net> References: <8EF270B6-DFDC-49F6-AFB0-34E8C6E7723F@cox.net> Message-ID: I just started getting emails asking if I want an invitation join Google+ - pretty sure they're spam. Pete Molly's Revenge On Sat, Jul 16, 2011 at 8:42 PM, Joe Lewis Wilkins wrote: > Have you people noticed an excessive number of spam emails from gMail > accounts in the pass couple of weeks? Others too. I flag them as spam with > my ISP (cox), but it does no good. I'm using Apple's Mail application. Any > help with this would be greatly appreciated. I'm about to drop Cox and an > email address I've been using for at least 15 years because of this. TIA, > > Joe Wilkins > > > _______________________________________________ > use-livecode mailing list > use-livecode at lists.runrev.com > Please visit this url to subscribe, unsubscribe and manage your > subscription preferences: > http://lists.runrev.com/mailman/listinfo/use-livecode > > From pepetoo at cox.net Sun Jul 17 00:34:41 2011 From: pepetoo at cox.net (Joe Lewis Wilkins) Date: Sat, 16 Jul 2011 21:34:41 -0700 Subject: spam In-Reply-To: References: <8EF270B6-DFDC-49F6-AFB0-34E8C6E7723F@cox.net> Message-ID: I did too, Pete and trashed them. Joe Wilkins On Jul 16, 2011, at 9:23 PM, Pete wrote: > I just started getting emails asking if I want an invitation join Google+ - > pretty sure they're spam. > Pete > Molly's Revenge > > On Sat, Jul 16, 2011 at 8:42 PM, Joe Lewis Wilkins wrote: > >> Have you people noticed an excessive number of spam emails from gMail >> accounts in the pass couple of weeks? Others too. I flag them as spam with >> my ISP (cox), but it does no good. I'm using Apple's Mail application. Any >> help with this would be greatly appreciated. I'm about to drop Cox and an >> email address I've been using for at least 15 years because of this. TIA, >> >> Joe Wilkins From shaosean at wehostmacs.com Sun Jul 17 01:53:02 2011 From: shaosean at wehostmacs.com (Shao Sean) Date: Sun, 17 Jul 2011 01:53:02 -0400 Subject: How to detect zoombox click Message-ID: <7C790A8D-3026-40EE-9CDC-1D22BAC48FCD@wehostmacs.com> I know Mac OS X sends a message for each of the window buttons and I am pretty sure that Windows does as well.. Could be a simple external for someone to write or for Rev to add it to the engine themselves.. From kray at sonsothunder.com Sun Jul 17 01:57:37 2011 From: kray at sonsothunder.com (Ken Ray) Date: Sun, 17 Jul 2011 00:57:37 -0500 Subject: How to detect zoombox click In-Reply-To: References: <3BE8C33E-08C0-4656-BAD7-5654FDCE7F7B@mac.com> <8CE12450228C4AD-1E48-5FECD@Webmail-m110.sysops.aol.com> Message-ID: <42B4DDA5-044F-4CAB-B0CE-4854832F6EF9@sonsothunder.com> Bill, I have this in my Scripter's Scrapbook, but admittedly it's from 2006 so it may not work... but give it a shot, it detects a maximized and restored window: on resizeStack newW,newH,oldW,oldH if the platform is "MacOS" then put 20 into tAdjust else put 0 into tAdjust if newW = (item 3 of the windowBoundingRect)-tAdjust then put "Maximized" else if oldW = (item 3 of the windowBoundingRect)-tAdjust then put "Restored" else put oldW,oldH,newW,newH end if end if end resizeStack Ken Ray Sons of Thunder Software, Inc. Email: kray at sonsothunder.com Web Site: http://www.sonsothunder.com/ On Jul 16, 2011, at 10:41 PM, Bill Vlahos wrote: > Craig, > > That is interesting. > > Dragging the stack window also generates a movestack message just like resizing a window generates a resizestack message. > > Is there a way to test for a simultaneous movestack and resizestack messages? Unless I find a unique message for that control detecting simultaneous movestack and resizestack messages should do the trick. > > On my Mac the resizeStack message is sent first then the moveStack message. The following script works but seems like a huge kludge. Is that order consistent between different versions or Rev/LC and on different platforms? > > To test put this script in the stack and create a field called "one" and then manipulate the stack window. > > global vZoom > > on resizeStack > put the seconds into vZoom > end resizeStack > > on moveStack > if vZoom - the seconds = 0 then put "ZoomBox click" into field "one" > else put "Move or resize" into field "one" > end moveStack > > Is there a more elegant way to detect simultaneous messages? > > > Bill Vlahos > _________________ > InfoWallet (http://www.infowallet.com) is about keeping your important life information with you, accessible, and secure. > > On Jul 16, 2011, at 8:01 PM, DunbarX at aol.com wrote: > >> A movestack message is also sent. Not sure why, though. >> >> >> Might this help, since resizing the window manually does not send this message? >> >> >> Craig Newman >> >> >> >> >> >> -----Original Message----- >> From: Bill Vlahos >> To: How to use LiveCode >> Sent: Fri, Jul 15, 2011 6:57 pm >> Subject: How to detect zoombox click >> >> >> I want to detect when a user clicks the zoombox (green light on a Mac) and do >> something similar to iTunes where if you click the zoombox the program doesn't >> just resize itself but goes into a different mode. >> >> The only message I see is a resizeStack message but that happens if the user >> resizes the window manually. >> >> Bill Vlahos >> _________________ >> InfoWallet (http://www.infowallet.com) is about keeping your important life >> information with you, accessible, and secure. >> >> >> _______________________________________________ >> use-livecode mailing list >> use-livecode at lists.runrev.com >> Please visit this url to subscribe, unsubscribe and manage your subscription >> preferences: >> http://lists.runrev.com/mailman/listinfo/use-livecode >> >> >> >> _______________________________________________ >> use-livecode mailing list >> use-livecode at lists.runrev.com >> Please visit this url to subscribe, unsubscribe and manage your subscription preferences: >> http://lists.runrev.com/mailman/listinfo/use-livecode > > > _______________________________________________ > use-livecode mailing list > use-livecode at lists.runrev.com > Please visit this url to subscribe, unsubscribe and manage your subscription preferences: > http://lists.runrev.com/mailman/listinfo/use-livecode From kray at sonsothunder.com Sun Jul 17 03:10:48 2011 From: kray at sonsothunder.com (Ken Ray) Date: Sun, 17 Jul 2011 02:10:48 -0500 Subject: [ANN] DropTools 1.1 Update Message-ID: <0CEFEA99-D5F0-4501-BC88-EBCA0FF98F11@sonsothunder.com> Just a quick note to let you know that there's an update to the DropTools Palette that has the following changes/additions: - Optimized the loading/display of DropTools by not reloading images on every launch - Optimized the storing of the DropTools palette location (removes sluggish response when initially dragging the palette) - Added API calls for "DropTools_VersionCheck" (which lets your DropTool check to see if the user has a late enough version of the DropTools Palette) and "DropTools_TruncID()" (which returns a truncated object identifier)* - Will automatically disable DropTool icons in the DropTools Palette if "DropTools_VersionCheck" is executed and the DropTools Palette is older than the version provided. - Folders in the "DropTools" folder that start with "(" will be ignored (so they can be used as support for specific DropTools). To update, click on the gear icon in the DropTools Palette, click on the About DropTools tab and then click "Check For Update". Ken Ray Sons of Thunder Software, Inc. Email: kray at sonsothunder.com Web Site: http://www.sonsothunder.com/ * See http://droptools.sonsothunder.com/developer/quickref.irev for more info on these API calls. From kray at sonsothunder.com Sun Jul 17 03:15:33 2011 From: kray at sonsothunder.com (Ken Ray) Date: Sun, 17 Jul 2011 02:15:33 -0500 Subject: [ANN] stsRating DropTool Message-ID: <4B2005B8-AE34-40BC-9D34-ABE018981AA7@sonsothunder.com> I'm happy to be able to announce the release of the stsRating DropTool, a general purpose rating and choice selection control. It can be used for a "star" rating system, to display a thermometer, to create linear gauges or bars, or even used in questionnaires to have the user make a selection from a number of choices. The control can be for display only or interactive, and can be oriented vertically or horizontally, and draw from any origin point (left, right, top, or bottom). The DropTool comes bundled with 18 different image sets including stars (generic, iTunes, Windows Media Player), colored spheres, hearts, and examples of a simple thermometer and questionnaire selection. You can create your own image sets or just use existing image objects "on the fly". Information and full documentation is at the DropTools web site: http://droptools.sonsothunder.com/sts/sts-ra.irev Enjoy! Ken Ray Sons of Thunder Software, Inc. Email: kray at sonsothunder.com Web Site: http://www.sonsothunder.com/ From stephenREVOLUTION2 at barncard.com Sun Jul 17 03:46:03 2011 From: stephenREVOLUTION2 at barncard.com (stephen barncard) Date: Sun, 17 Jul 2011 00:46:03 -0700 Subject: spam In-Reply-To: <8EF270B6-DFDC-49F6-AFB0-34E8C6E7723F@cox.net> References: <8EF270B6-DFDC-49F6-AFB0-34E8C6E7723F@cox.net> Message-ID: I don't know about the google spam addresses, they are probably spoofed. Reply-To's are often lies. My advice is to start using gmail. Best spam filters in the business. You can still use your old email address. You will be amazed. I'm able to use some of my email addresses that haven't been usable for years. sqb On 16 July 2011 20:42, Joe Lewis Wilkins wrote: > Have you people noticed an excessive number of spam emails from gMail > accounts in the pass couple of weeks? Others too. I flag them as spam with > my ISP (cox), but it does no good. I'm using Apple's Mail application. Any > help with this would be greatly appreciated. I'm about to drop Cox and an > email address I've been using for at least 15 years because of this. TIA, > > Joe Wilkins > > > _______________________________________________ > use-livecode mailing list > use-livecode at lists.runrev.com > Please visit this url to subscribe, unsubscribe and manage your > subscription preferences: > http://lists.runrev.com/mailman/listinfo/use-livecode > -- Stephen Barncard San Francisco Ca. USA more about sqb From richmondmathewson at gmail.com Sun Jul 17 03:49:44 2011 From: richmondmathewson at gmail.com (Richmond Mathewson) Date: Sun, 17 Jul 2011 10:49:44 +0300 Subject: spam In-Reply-To: <8EF270B6-DFDC-49F6-AFB0-34E8C6E7723F@cox.net> References: <8EF270B6-DFDC-49F6-AFB0-34E8C6E7723F@cox.net> Message-ID: <4E229418.9080806@gmail.com> On 07/17/2011 06:42 AM, Joe Lewis Wilkins wrote: > Have you people noticed an excessive number of spam emails from gMail accounts in the pass couple of weeks? Others too. I flag them as spam with my ISP (cox), but it does no good. I'm using Apple's Mail application. Any help with this would be greatly appreciated. I'm about to drop Cox and an email address I've been using for at least 15 years because of this. TIA, > > Joe Wilkins > > Not noticeably. I use Thunderbird, both on Mac and Linux, to look at my e-mail; through a crappy Bulgarian ISP that does not filter anything whatsoever. I average about 2 spam e-mails a week. Now my Yahoo account is another story altogether, attracting about 3-4 spams a day. Frankly, my experience with Apple's Mail app was distinctly negative, added to the fact that I am Biplatformal (work that one out) and like to minimise the cognitive effort across platforms, I have been using Thunderbird for about 4 years now to access my gmail account with never a backward look. From rene.micout at numericable.com Sun Jul 17 04:09:50 2011 From: rene.micout at numericable.com (=?iso-8859-1?Q?Ren=E9_Micout?=) Date: Sun, 17 Jul 2011 10:09:50 +0200 Subject: [ANN] stsRating DropTool In-Reply-To: <4B2005B8-AE34-40BC-9D34-ABE018981AA7@sonsothunder.com> References: <4B2005B8-AE34-40BC-9D34-ABE018981AA7@sonsothunder.com> Message-ID: <080E9BE8-08C1-4676-A9FD-AC8E35D58C76@numericable.com> Hello Ken, The real web site is: http://droptools.sonsothunder.com/products/sts/sts-ra.irev Bon souvenir de Paris Ren? Le 17 juil. 2011 ? 09:15, Ken Ray a ?crit : > I'm happy to be able to announce the release of the stsRating DropTool, a general purpose rating and choice selection control. It can be used for a "star" rating system, to display a thermometer, to create linear gauges or bars, or even used in questionnaires to have the user make a selection from a number of choices. The control can be for display only or interactive, and can be oriented vertically or horizontally, and draw from any origin point (left, right, top, or bottom). > > The DropTool comes bundled with 18 different image sets including stars (generic, iTunes, Windows Media Player), colored spheres, hearts, and examples of a simple thermometer and questionnaire selection. You can create your own image sets or just use existing image objects "on the fly". > > Information and full documentation is at the DropTools web site: > > http://droptools.sonsothunder.com/sts/sts-ra.irev > > Enjoy! > > Ken Ray > Sons of Thunder Software, Inc. > Email: kray at sonsothunder.com > Web Site: http://www.sonsothunder.com/ > > _______________________________________________ > use-livecode mailing list > use-livecode at lists.runrev.com > Please visit this url to subscribe, unsubscribe and manage your subscription preferences: > http://lists.runrev.com/mailman/listinfo/use-livecode From kray at sonsothunder.com Sun Jul 17 04:35:56 2011 From: kray at sonsothunder.com (Ken Ray) Date: Sun, 17 Jul 2011 03:35:56 -0500 Subject: [ANN] stsRating DropTool In-Reply-To: <080E9BE8-08C1-4676-A9FD-AC8E35D58C76@numericable.com> References: <4B2005B8-AE34-40BC-9D34-ABE018981AA7@sonsothunder.com> <080E9BE8-08C1-4676-A9FD-AC8E35D58C76@numericable.com> Message-ID: > The real web site is: > > http://droptools.sonsothunder.com/products/sts/sts-ra.irev Whoops! Thanks, Ren?! Ken Ray Sons of Thunder Software, Inc. Email: kray at sonsothunder.com Web Site: http://www.sonsothunder.com/ From john.allijn at alice.nl Sun Jul 17 05:56:45 2011 From: john.allijn at alice.nl (John Allijn) Date: Sun, 17 Jul 2011 11:56:45 +0200 Subject: [ANN] iOS and Android apps In-Reply-To: <8BB1992E24184EB08F1B2BFEECD04C2D@stuart> References: <8BB1992E24184EB08F1B2BFEECD04C2D@stuart> Message-ID: Hi Mark, Sure: you can access it here directly: https://market.android.com/details?id=com.allijn.read&feature=search_result Thanks for your interest! John. Sent from my iPad On Jul 17, 2011, at 5:14, "Mark Stuart" wrote: > Hi John, > > I'd like to download your app to my Android phone, But after using the > search word "READ!", there were so many in the result list I gave up. > > Are there some key words that I can search for your app with? > > > > Regards, > > Mark Stuart > > _______________________________________________ > use-livecode mailing list > use-livecode at lists.runrev.com > Please visit this url to subscribe, unsubscribe and manage your subscription preferences: > http://lists.runrev.com/mailman/listinfo/use-livecode From john at splash21.com Sun Jul 17 07:52:28 2011 From: john at splash21.com (John Craig) Date: Sun, 17 Jul 2011 12:52:28 +0100 Subject: Problem testing on android simulator (Windows) In-Reply-To: <4E223ED1.4050504@hyperactivesw.com> References: <4E21E498.5060902@splash21.com> <4E21EF63.9070702@splash21.com> <4E21F89C.7010401@splash21.com> <4E223ED1.4050504@hyperactivesw.com> Message-ID: <4E22CCFC.20005@splash21.com> That was a good stab, Jacque - I tried running everything as administrator, but it gave me the same error :o I'll try cleaning everything out and do another install from scratch On 17/07/2011 02:45, J. Landman Gay wrote: > On 7/16/11 4:01 PM, Roger Eller wrote: >> On Sat, Jul 16, 2011 at 4:46 PM, John Craig wrote: >> >>> I've tried with a unique id, not signing with anything - just want >>> to run >>> in emulator. >>> All works OK on Mac - does Windows require anything extra? I don't see >>> anything so far in the android release notes. >>> >>> :| >> >> >> That's so frustrating. I'm sorry I don't know what else to check. >> It has >> worked the same for me on both WinXP and Mac. Is your AVD built with >> all >> the same options as on your Mac? Try making a new AVD with minimal >> options. >> I hope Mark Waddingham or Jacque will chime in and save the day. :) > > I can't chime in, sorry. I couldn't even get the emulator to run in > Windows and I gave up. It works on my Mac so I just stayed with that. > > Could it be a permissions thing? It's a stab in the dark. > From john at splash21.com Sun Jul 17 07:57:44 2011 From: john at splash21.com (John Craig) Date: Sun, 17 Jul 2011 12:57:44 +0100 Subject: Problem testing on android simulator (Windows) In-Reply-To: References: <4E21E498.5060902@splash21.com> <4E21EF63.9070702@splash21.com> <4E21F89C.7010401@splash21.com> <4E223ED1.4050504@hyperactivesw.com> Message-ID: <4E22CE38.7060207@splash21.com> Could even be java related, it only seems to get stuck at the last hurdle as it tries to launch in the emulator. I'll let you know if the clean install is successful. On 17/07/2011 03:16, Roger Eller wrote: > On Sat, Jul 16, 2011 at 9:45 PM, J. Landman Gay wrote: > >> On 7/16/11 4:01 PM, Roger Eller wrote: >> >>> That's so frustrating. I'm sorry I don't know what else to check. It has >>> worked the same for me on both WinXP and Mac. Is your AVD built with all >>> the same options as on your Mac? Try making a new AVD with minimal >>> options. >>> I hope Mark Waddingham or Jacque will chime in and save the day. :) >>> >> I can't chime in, sorry. I couldn't even get the emulator to run in Windows >> and I gave up. It works on my Mac so I just stayed with that. >> >> Could it be a permissions thing? It's a stab in the dark. >> >> -- >> Jacqueline Landman Gay | jacque at hyperactivesw.com >> HyperActive Software | http://www.hyperactivesw.com > > I haven't seen any bugzilla reports of it failing to test on Windows. I > wonder what conditions cause it to break. > > ?Roger > _______________________________________________ > use-livecode mailing list > use-livecode at lists.runrev.com > Please visit this url to subscribe, unsubscribe and manage your subscription preferences: > http://lists.runrev.com/mailman/listinfo/use-livecode > From john at splash21.com Sun Jul 17 08:01:57 2011 From: john at splash21.com (John Craig) Date: Sun, 17 Jul 2011 13:01:57 +0100 Subject: [ANN] iOS and Android apps In-Reply-To: <1F7C1EFB-CF74-44A7-874D-1FE13262B690@alice.nl> References: <1F7C1EFB-CF74-44A7-874D-1FE13262B690@alice.nl> Message-ID: <4E22CF35.2050503@splash21.com> Hey, John - Love the READ app - I bet you were a teacher's nightmare at school ;p On 16/07/2011 07:29, John Allijn wrote: > Hi, > > Apple just approved two updates of my apps: > > READ is now universal (one executable for iPhone and iPad). It has also some minor updates in the way it displays text. > The app can be downloaded here: > http://itunes.apple.com/us/app/read/id433493823?ls=1&mt=8 > > > For the Dutch readers on the list: Dordrecht.HD is a city walk through Hollands oldest city. I did some minor updates on the interface and corrected quite a few typo's. > http://itunes.apple.com/us/app/dordrecht.hd/id418542297?ls=1&mt=8 > > > New for me is the release of READ for Android. Except for some minor adjustments (the rounded corners of rectangles display differently on android then on iOS) publication was easy. > > Here is a link to the app in the marketplace: > https://market.android.com/details?id=com.allijn.read&feature=search_result > > regards, > John > _______________________________________________ > use-livecode mailing list > use-livecode at lists.runrev.com > Please visit this url to subscribe, unsubscribe and manage your subscription preferences: > http://lists.runrev.com/mailman/listinfo/use-livecode > From john.allijn at alice.nl Sun Jul 17 08:38:13 2011 From: john.allijn at alice.nl (John Allijn) Date: Sun, 17 Jul 2011 14:38:13 +0200 Subject: [ANN] iOS and Android apps In-Reply-To: <4E22CF35.2050503@splash21.com> References: <1F7C1EFB-CF74-44A7-874D-1FE13262B690@alice.nl> <4E22CF35.2050503@splash21.com> Message-ID: <4E7DA5CB-F434-4282-BF06-BDB27E891CB9@alice.nl> LOL No comment :) Send from my iPad On Jul 17, 2011, at 14:01, John Craig wrote: > Hey, John - Love the READ app - I bet you were a teacher's nightmare at school ;p > > > On 16/07/2011 07:29, John Allijn wrote: >> Hi, >> >> Apple just approved two updates of my apps: >> >> READ is now universal (one executable for iPhone and iPad). It has also some minor updates in the way it displays text. >> The app can be downloaded here: >> http://itunes.apple.com/us/app/read/id433493823?ls=1&mt=8 >> >> >> For the Dutch readers on the list: Dordrecht.HD is a city walk through Hollands oldest city. I did some minor updates on the interface and corrected quite a few typo's. >> http://itunes.apple.com/us/app/dordrecht.hd/id418542297?ls=1&mt=8 >> >> >> New for me is the release of READ for Android. Except for some minor adjustments (the rounded corners of rectangles display differently on android then on iOS) publication was easy. >> >> Here is a link to the app in the marketplace: >> https://market.android.com/details?id=com.allijn.read&feature=search_result >> >> regards, >> John >> _______________________________________________ >> use-livecode mailing list >> use-livecode at lists.runrev.com >> Please visit this url to subscribe, unsubscribe and manage your subscription preferences: >> http://lists.runrev.com/mailman/listinfo/use-livecode >> > > _______________________________________________ > use-livecode mailing list > use-livecode at lists.runrev.com > Please visit this url to subscribe, unsubscribe and manage your subscription preferences: > http://lists.runrev.com/mailman/listinfo/use-livecode From coiin at verizon.net Sun Jul 17 10:08:18 2011 From: coiin at verizon.net (Colin Holgate) Date: Sun, 17 Jul 2011 10:08:18 -0400 Subject: spam In-Reply-To: References: <8EF270B6-DFDC-49F6-AFB0-34E8C6E7723F@cox.net> Message-ID: <470E6067-DD30-4063-844D-B3CAD674789C@verizon.net> You'll find that as more people you know get on Google Plus, you'll get more invite-like emails, that may well be genuine. If this images pastes in here ok, this is what part of it looks like: -------------- next part -------------- The Change, View, and Block links show real plus.google.com addresses. From coiin at verizon.net Sun Jul 17 10:15:38 2011 From: coiin at verizon.net (Colin Holgate) Date: Sun, 17 Jul 2011 10:15:38 -0400 Subject: spam In-Reply-To: <470E6067-DD30-4063-844D-B3CAD674789C@verizon.net> References: <8EF270B6-DFDC-49F6-AFB0-34E8C6E7723F@cox.net> <470E6067-DD30-4063-844D-B3CAD674789C@verizon.net> Message-ID: <9B39DA8F-AECE-491B-A891-52117D33795A@verizon.net> The image didn't show up. It's this one: http://xfiles.funnygarbage.com/~colinholgate/images/gplus.png On Jul 17, 2011, at 10:08 AM, Colin Holgate wrote: > You'll find that as more people you know get on Google Plus, you'll get more invite-like emails, that may well be genuine. If this images pastes in here ok, this is what part of it looks like: From jacque at hyperactivesw.com Sun Jul 17 12:22:45 2011 From: jacque at hyperactivesw.com (J. Landman Gay) Date: Sun, 17 Jul 2011 11:22:45 -0500 Subject: Problem testing on android simulator (Windows) In-Reply-To: <4E22CE38.7060207@splash21.com> References: <4E21E498.5060902@splash21.com> <4E21EF63.9070702@splash21.com> <4E21F89C.7010401@splash21.com> <4E223ED1.4050504@hyperactivesw.com> <4E22CE38.7060207@splash21.com> Message-ID: <4E230C55.9080904@hyperactivesw.com> On 7/17/11 6:57 AM, John Craig wrote: > Could even be java related, it only seems to get stuck at the last > hurdle as it tries to launch in the emulator. > I'll let you know if the clean install is successful. That triggered a vague memory. Someone in the forums said they had to install the latest Java, or at least a particular version of it. Sorry that's all I remember. A search over there might turn it up. -- Jacqueline Landman Gay | jacque at hyperactivesw.com HyperActive Software | http://www.hyperactivesw.com From jacque at hyperactivesw.com Sun Jul 17 12:28:58 2011 From: jacque at hyperactivesw.com (J. Landman Gay) Date: Sun, 17 Jul 2011 11:28:58 -0500 Subject: [ANN] DropTools 1.1 Update In-Reply-To: <0CEFEA99-D5F0-4501-BC88-EBCA0FF98F11@sonsothunder.com> References: <0CEFEA99-D5F0-4501-BC88-EBCA0FF98F11@sonsothunder.com> Message-ID: <4E230DCA.7070200@hyperactivesw.com> On 7/17/11 2:10 AM, Ken Ray wrote: > To update, click on the gear icon in the DropTools Palette, click on > the About DropTools tab and then click "Check For Update". Very slick and seamless, it updated without a hitch. Nice job. -- Jacqueline Landman Gay | jacque at hyperactivesw.com HyperActive Software | http://www.hyperactivesw.com From bvlahos at mac.com Sun Jul 17 12:32:55 2011 From: bvlahos at mac.com (Bill Vlahos) Date: Sun, 17 Jul 2011 09:32:55 -0700 Subject: How to detect zoombox click In-Reply-To: <7C790A8D-3026-40EE-9CDC-1D22BAC48FCD@wehostmacs.com> References: <7C790A8D-3026-40EE-9CDC-1D22BAC48FCD@wehostmacs.com> Message-ID: It seems to me like LC already knows what the zoombox is supposed to do and just does it. I was hoping there would be a specific message generated for it. Maybe they just did the expected behavior instead of generating a message. Bill Vlahos _________________ InfoWallet (http://www.infowallet.com) is about keeping your important life information with you, accessible, and secure. On Jul 16, 2011, at 10:53 PM, Shao Sean wrote: > I know Mac OS X sends a message for each of the window buttons and I am pretty sure that Windows does as well.. Could be a simple external for someone to write or for Rev to add it to the engine themselves.. > > _______________________________________________ > use-livecode mailing list > use-livecode at lists.runrev.com > Please visit this url to subscribe, unsubscribe and manage your subscription preferences: > http://lists.runrev.com/mailman/listinfo/use-livecode From roger.e.eller at sealedair.com Sun Jul 17 13:48:13 2011 From: roger.e.eller at sealedair.com (Roger Eller) Date: Sun, 17 Jul 2011 13:48:13 -0400 Subject: revMail to Gmail when it's not the default client Message-ID: I need to pass a revMail or mailTo string to Gmail even when another email client is configured as the default. Any suggestions? ?Roger From kray at sonsothunder.com Sun Jul 17 13:55:50 2011 From: kray at sonsothunder.com (Ken Ray) Date: Sun, 17 Jul 2011 12:55:50 -0500 Subject: [ANN] DropTools 1.1 Update: Problem with update fixed - Please update again In-Reply-To: <4E230DCA.7070200@hyperactivesw.com> References: <0CEFEA99-D5F0-4501-BC88-EBCA0FF98F11@sonsothunder.com> <4E230DCA.7070200@hyperactivesw.com> Message-ID: <8DE88CCB-C0CC-4E3D-9C28-06B2A4B85C77@sonsothunder.com> On Jul 17, 2011, at 11:28 AM, J. Landman Gay wrote: > On 7/17/11 2:10 AM, Ken Ray wrote: > >> To update, click on the gear icon in the DropTools Palette, click on >> the About DropTools tab and then click "Check For Update". > > Very slick and seamless, it updated without a hitch. Nice job. Thanks, but I just found that the main download was updated to 1.1 but the "check for updates" one wasn't... To anyone who ran the update before you read this email: if you check the "About DropTools Palette" and it doesn't say "1.1", please do another "Check for Updates" and you should get the new one. Sorry about the mixup, Ken Ray Sons of Thunder Software, Inc. Email: kray at sonsothunder.com Web Site: http://www.sonsothunder.com/ From kray at sonsothunder.com Sun Jul 17 13:56:31 2011 From: kray at sonsothunder.com (Ken Ray) Date: Sun, 17 Jul 2011 12:56:31 -0500 Subject: revMail to Gmail when it's not the default client In-Reply-To: References: Message-ID: On Jul 17, 2011, at 12:48 PM, Roger Eller wrote: > I need to pass a revMail or mailTo string to Gmail even when another email > client is configured as the default. Any suggestions? What platform(s) do you need this on? Ken Ray Sons of Thunder Software, Inc. Email: kray at sonsothunder.com Web Site: http://www.sonsothunder.com/ From pete at mollysrevenge.com Sun Jul 17 14:16:41 2011 From: pete at mollysrevenge.com (Pete) Date: Sun, 17 Jul 2011 11:16:41 -0700 Subject: revMail to Gmail when it's not the default client In-Reply-To: References: Message-ID: There's a Gmail plugin available from Google that makes Gmail come up when you click a mail link instead of the default for the computer, but I don't know how that would interact with revMail Pete Molly's Revenge On Sun, Jul 17, 2011 at 10:48 AM, Roger Eller wrote: > I need to pass a revMail or mailTo string to Gmail even when another email > client is configured as the default. Any suggestions? > > ?Roger > _______________________________________________ > use-livecode mailing list > use-livecode at lists.runrev.com > Please visit this url to subscribe, unsubscribe and manage your > subscription preferences: > http://lists.runrev.com/mailman/listinfo/use-livecode > > From roger.e.eller at sealedair.com Sun Jul 17 14:26:54 2011 From: roger.e.eller at sealedair.com (Roger Eller) Date: Sun, 17 Jul 2011 14:26:54 -0400 Subject: revMail to Gmail when it's not the default client In-Reply-To: References: Message-ID: On Sun, Jul 17, 2011 at 1:56 PM, Ken Ray wrote: > > On Jul 17, 2011, at 12:48 PM, Roger Eller wrote: > > > I need to pass a revMail or mailTo string to Gmail even when another > email > > client is configured as the default. Any suggestions? > > What platform(s) do you need this on? > > > Ken Ray > Sons of Thunder Software, Inc. > Email: kray at sonsothunder.com > Web Site: http://www.sonsothunder.com/ > Windows XP and Windows 7. ?Roger From roger.e.eller at sealedair.com Sun Jul 17 14:29:13 2011 From: roger.e.eller at sealedair.com (Roger Eller) Date: Sun, 17 Jul 2011 14:29:13 -0400 Subject: revMail to Gmail when it's not the default client In-Reply-To: References: Message-ID: On Sun, Jul 17, 2011 at 2:16 PM, Pete wrote: > There's a Gmail plugin available from Google that makes Gmail come up when > you click a mail link instead of the default for the computer, but I don't > know how that would interact with revMail > Pete > Molly's Revenge > > > On Sun, Jul 17, 2011 at 10:48 AM, Roger Eller > wrote: > > > I need to pass a revMail or mailTo string to Gmail even when another > email > > client is configured as the default. Any suggestions? > > > > ?Roger > Thanks Pete. I'll see what I can find about that. ?Roger From pete at mollysrevenge.com Sun Jul 17 14:44:33 2011 From: pete at mollysrevenge.com (Pete) Date: Sun, 17 Jul 2011 11:44:33 -0700 Subject: revMail to Gmail when it's not the default client In-Reply-To: References: Message-ID: Sorry Roger, should have included a link - here it is: http://mail.google.com/support/bin/answer.py?answer=10966 It's actually part of the Gmail Notifier plugin which works on Windows and Mac. Pete Molly's Revenge On Sun, Jul 17, 2011 at 11:29 AM, Roger Eller wrote: > On Sun, Jul 17, 2011 at 2:16 PM, Pete wrote: > > > There's a Gmail plugin available from Google that makes Gmail come up > when > > you click a mail link instead of the default for the computer, but I > don't > > know how that would interact with revMail > > Pete > > Molly's Revenge > > > > > > On Sun, Jul 17, 2011 at 10:48 AM, Roger Eller > > wrote: > > > > > I need to pass a revMail or mailTo string to Gmail even when another > > email > > > client is configured as the default. Any suggestions? > > > > > > ?Roger > > > > Thanks Pete. I'll see what I can find about that. > > ?Roger > _______________________________________________ > use-livecode mailing list > use-livecode at lists.runrev.com > Please visit this url to subscribe, unsubscribe and manage your > subscription preferences: > http://lists.runrev.com/mailman/listinfo/use-livecode > > From roger.e.eller at sealedair.com Sun Jul 17 15:57:44 2011 From: roger.e.eller at sealedair.com (Roger Eller) Date: Sun, 17 Jul 2011 15:57:44 -0400 Subject: revMail to Gmail when it's not the default client In-Reply-To: References: Message-ID: On Sun, Jul 17, 2011 at 2:44 PM, Pete wrote: > Sorry Roger, should have included a link - here it is: > > http://mail.google.com/support/bin/answer.py?answer=10966 > > It's actually part of the Gmail Notifier plugin which works on Windows and > Mac. > > Pete > Molly's Revenge > > Works great! I did find that when the message body is greater than about 37 lines the plugin fails and shows a broken robot illustration. Anyway, this is still very helpful. Thanks. ~Roger From bvlahos at mac.com Sun Jul 17 16:12:27 2011 From: bvlahos at mac.com (Bill Vlahos) Date: Sun, 17 Jul 2011 13:12:27 -0700 Subject: How to detect zoombox click In-Reply-To: <8CE12450228C4AD-1E48-5FECD@Webmail-m110.sysops.aol.com> References: <3BE8C33E-08C0-4656-BAD7-5654FDCE7F7B@mac.com> <8CE12450228C4AD-1E48-5FECD@Webmail-m110.sysops.aol.com> Message-ID: Actually in Linux and Windows if you resize the left side (maybe the top too but I didn't check that) it also generates both messages. That means I can't reliably tell the difference between a zoombox click and resizing the left side. Is there an Event Manager in LC? I can flushEvents but I don't know how to see them. Bill Vlahos _________________ InfoWallet (http://www.infowallet.com) is about keeping your important life information with you, accessible, and secure. On Jul 16, 2011, at 8:01 PM, DunbarX at aol.com wrote: > A movestack message is also sent. Not sure why, though. > > > Might this help, since resizing the window manually does not send this message? > > > Craig Newman > > > > > > -----Original Message----- > From: Bill Vlahos > To: How to use LiveCode > Sent: Fri, Jul 15, 2011 6:57 pm > Subject: How to detect zoombox click > > > I want to detect when a user clicks the zoombox (green light on a Mac) and do > something similar to iTunes where if you click the zoombox the program doesn't > just resize itself but goes into a different mode. > > The only message I see is a resizeStack message but that happens if the user > resizes the window manually. > > Bill Vlahos > _________________ > InfoWallet (http://www.infowallet.com) is about keeping your important life > information with you, accessible, and secure. > > > _______________________________________________ > use-livecode mailing list > use-livecode at lists.runrev.com > Please visit this url to subscribe, unsubscribe and manage your subscription > preferences: > http://lists.runrev.com/mailman/listinfo/use-livecode > > > > _______________________________________________ > use-livecode mailing list > use-livecode at lists.runrev.com > Please visit this url to subscribe, unsubscribe and manage your subscription preferences: > http://lists.runrev.com/mailman/listinfo/use-livecode From dunbarx at aol.com Sun Jul 17 23:37:24 2011 From: dunbarx at aol.com (dunbarx at aol.com) Date: Sun, 17 Jul 2011 23:37:24 -0400 (EDT) Subject: How to detect zoombox click In-Reply-To: References: <3BE8C33E-08C0-4656-BAD7-5654FDCE7F7B@mac.com><8CE12450228C4AD-1E48-5FECD@Webmail-m110.sysops.aol.com> Message-ID: <8CE131324B2C5F9-1C5C-370AB@webmail-m012.sysops.aol.com> Odd. On my mac this message is not sent. I think this can be used to advantage. But if you have other platforms to worry about, and you are correct in that the message is in fact sent there, then no good. But why would this behavior be platform specific? Craig Newman -----Original Message----- From: Bill Vlahos To: How to use LiveCode Sent: Sun, Jul 17, 2011 12:14 pm Subject: Re: How to detect zoombox click Actually in Linux and Windows if you resize the left side (maybe the top too but I didn't check that) it also generates both messages. That means I can't reliably tell the difference between a zoombox click and resizing the left side. Is there an Event Manager in LC? I can flushEvents but I don't know how to see them. Bill Vlahos _________________ InfoWallet (http://www.infowallet.com) is about keeping your important life information with you, accessible, and secure. On Jul 16, 2011, at 8:01 PM, DunbarX at aol.com wrote: > A movestack message is also sent. Not sure why, though. > > > Might this help, since resizing the window manually does not send this message? > > > Craig Newman > > > > > > -----Original Message----- > From: Bill Vlahos > To: How to use LiveCode > Sent: Fri, Jul 15, 2011 6:57 pm > Subject: How to detect zoombox click > > > I want to detect when a user clicks the zoombox (green light on a Mac) and do > something similar to iTunes where if you click the zoombox the program doesn't > just resize itself but goes into a different mode. > > The only message I see is a resizeStack message but that happens if the user > resizes the window manually. > > Bill Vlahos > _________________ > InfoWallet (http://www.infowallet.com) is about keeping your important life > information with you, accessible, and secure. > > > _______________________________________________ > use-livecode mailing list > use-livecode at lists.runrev.com > Please visit this url to subscribe, unsubscribe and manage your subscription > preferences: > http://lists.runrev.com/mailman/listinfo/use-livecode > > > > _______________________________________________ > use-livecode mailing list > use-livecode at lists.runrev.com > Please visit this url to subscribe, unsubscribe and manage your subscription preferences: > http://lists.runrev.com/mailman/listinfo/use-livecode _______________________________________________ use-livecode mailing list use-livecode at lists.runrev.com Please visit this url to subscribe, unsubscribe and manage your subscription preferences: http://lists.runrev.com/mailman/listinfo/use-livecode From shaosean at wehostmacs.com Mon Jul 18 00:55:13 2011 From: shaosean at wehostmacs.com (Shao Sean) Date: Mon, 18 Jul 2011 00:55:13 -0400 Subject: How to detect zoombox click Message-ID: Now that Lion will allow resizing from any window edge, how are the messages on that being sent? From pete at mollysrevenge.com Mon Jul 18 01:13:51 2011 From: pete at mollysrevenge.com (Pete) Date: Sun, 17 Jul 2011 22:13:51 -0700 Subject: security code number generation In-Reply-To: <56E8486D-1F05-4291-826D-8C6DB390D12A@gmail.com> References: <56E8486D-1F05-4291-826D-8C6DB390D12A@gmail.com> Message-ID: Peter, I'm looking for something to generate license codes for some software I'm planning to sell. Do you think this would work for that purpose? Pete Molly's Revenge On Sat, Apr 2, 2011 at 9:51 AM, Peter Brigham MD wrote: > For anyone who might have the need, I have a handler I use to generate a > security code, in my case for printed prescriptions. It takes the name of > the patient, the date of the prescription, the medication and med strength > and hashes all that to produce a ten-digit alphanumeric string (using 0-9, > a-z, A-Z). If there is any question about the validity of the prescription I > can retrieve the correct code from the rx entry in my database with a > mouseclick (actually recalculating the code from the stored rx data) and > confirm it with the pharmacy. This has proved useful on two occasions when a > pt was playing fast and loose with his prescriptions. > > The algorithm is fast in LC, sufficiently obscure that I'm pretty sure it > would be hard to hack -- though of course few things are bulletproof in > encryption if someone wants to try hard enough -- and discontinuous in the > sense that similar inputs do not generate similar outputs, eg, change one > character in the input and the code number is completely different. The > probability of coming up with the correct security number by chance alone is > 1 in 10^15 (a million billion to 1). It could be adapted to any number of > purposes. I am not posting the handler here, since it would be unwise to let > it be archived and available, eg, with a Nabble search, but if anyone is > interested, let me know and I'll share it. > > -- Peter > > Peter M. Brigham > pmbrig at gmail.com > http://home.comcast.net/~**pmbrig > > > ______________________________**_________________ > use-livecode mailing list > use-livecode at lists.runrev.com > Please visit this url to subscribe, unsubscribe and manage your > subscription preferences: > http://lists.runrev.com/**mailman/listinfo/use-livecode > > From kray at sonsothunder.com Mon Jul 18 01:48:42 2011 From: kray at sonsothunder.com (Ken Ray) Date: Mon, 18 Jul 2011 00:48:42 -0500 Subject: security code number generation In-Reply-To: References: <56E8486D-1F05-4291-826D-8C6DB390D12A@gmail.com> Message-ID: > I'm looking for something to generate license codes for some software I'm > planning to sell. Do you think this would work for that purpose? Take a look at Zygodact; it does exactly this plus it has a DropTool component to make it a snap to work with. http://www.runrev.com/store/product/zygodact-1-0-4/ Ken Ray Sons of Thunder Software, Inc. Email: kray at sonsothunder.com Web Site: http://www.sonsothunder.com/ > Pete > Molly's Revenge > > > > > On Sat, Apr 2, 2011 at 9:51 AM, Peter Brigham MD wrote: > >> For anyone who might have the need, I have a handler I use to generate a >> security code, in my case for printed prescriptions. It takes the name of >> the patient, the date of the prescription, the medication and med strength >> and hashes all that to produce a ten-digit alphanumeric string (using 0-9, >> a-z, A-Z). If there is any question about the validity of the prescription I >> can retrieve the correct code from the rx entry in my database with a >> mouseclick (actually recalculating the code from the stored rx data) and >> confirm it with the pharmacy. This has proved useful on two occasions when a >> pt was playing fast and loose with his prescriptions. >> >> The algorithm is fast in LC, sufficiently obscure that I'm pretty sure it >> would be hard to hack -- though of course few things are bulletproof in >> encryption if someone wants to try hard enough -- and discontinuous in the >> sense that similar inputs do not generate similar outputs, eg, change one >> character in the input and the code number is completely different. The >> probability of coming up with the correct security number by chance alone is >> 1 in 10^15 (a million billion to 1). It could be adapted to any number of >> purposes. I am not posting the handler here, since it would be unwise to let >> it be archived and available, eg, with a Nabble search, but if anyone is >> interested, let me know and I'll share it. >> >> -- Peter >> >> Peter M. Brigham >> pmbrig at gmail.com >> http://home.comcast.net/~**pmbrig >> >> >> ______________________________**_________________ >> use-livecode mailing list >> use-livecode at lists.runrev.com >> Please visit this url to subscribe, unsubscribe and manage your >> subscription preferences: >> http://lists.runrev.com/**mailman/listinfo/use-livecode >> >> > _______________________________________________ > use-livecode mailing list > use-livecode at lists.runrev.com > Please visit this url to subscribe, unsubscribe and manage your subscription preferences: > http://lists.runrev.com/mailman/listinfo/use-livecode From shaosean at wehostmacs.com Mon Jul 18 02:27:10 2011 From: shaosean at wehostmacs.com (Shao Sean) Date: Mon, 18 Jul 2011 02:27:10 -0400 Subject: security code number generation Message-ID: <4DA4D9E3-BBBA-4736-834B-862E106A29C7@wehostmacs.com> I have a weird algorithm that will produce different output, even with the same input, and can be fully reversed back to the pieces that built the puzzle.. From pete at mollysrevenge.com Mon Jul 18 03:15:02 2011 From: pete at mollysrevenge.com (Pete) Date: Mon, 18 Jul 2011 00:15:02 -0700 Subject: security code number generation In-Reply-To: References: <56E8486D-1F05-4291-826D-8C6DB390D12A@gmail.com> Message-ID: Thanks Ken, that looks good. I guess the other missing piece is how to control "demo" versions. Expiration dates seem to be the most common, or maybe some limited function set. Pete Molly's Revenge On Sun, Jul 17, 2011 at 10:48 PM, Ken Ray wrote: > > I'm looking for something to generate license codes for some software I'm > > planning to sell. Do you think this would work for that purpose? > > Take a look at Zygodact; it does exactly this plus it has a DropTool > component to make it a snap to work with. > > http://www.runrev.com/store/product/zygodact-1-0-4/ > > > Ken Ray > Sons of Thunder Software, Inc. > Email: kray at sonsothunder.com > Web Site: http://www.sonsothunder.com/ > > > > Pete > > Molly's Revenge > > > > > > > > > > On Sat, Apr 2, 2011 at 9:51 AM, Peter Brigham MD > wrote: > > > >> For anyone who might have the need, I have a handler I use to generate a > >> security code, in my case for printed prescriptions. It takes the name > of > >> the patient, the date of the prescription, the medication and med > strength > >> and hashes all that to produce a ten-digit alphanumeric string (using > 0-9, > >> a-z, A-Z). If there is any question about the validity of the > prescription I > >> can retrieve the correct code from the rx entry in my database with a > >> mouseclick (actually recalculating the code from the stored rx data) and > >> confirm it with the pharmacy. This has proved useful on two occasions > when a > >> pt was playing fast and loose with his prescriptions. > >> > >> The algorithm is fast in LC, sufficiently obscure that I'm pretty sure > it > >> would be hard to hack -- though of course few things are bulletproof in > >> encryption if someone wants to try hard enough -- and discontinuous in > the > >> sense that similar inputs do not generate similar outputs, eg, change > one > >> character in the input and the code number is completely different. The > >> probability of coming up with the correct security number by chance > alone is > >> 1 in 10^15 (a million billion to 1). It could be adapted to any number > of > >> purposes. I am not posting the handler here, since it would be unwise to > let > >> it be archived and available, eg, with a Nabble search, but if anyone is > >> interested, let me know and I'll share it. > >> > >> -- Peter > >> > >> Peter M. Brigham > >> pmbrig at gmail.com > >> http://home.comcast.net/~**pmbrig > >> > >> > >> ______________________________**_________________ > >> use-livecode mailing list > >> use-livecode at lists.runrev.com > >> Please visit this url to subscribe, unsubscribe and manage your > >> subscription preferences: > >> http://lists.runrev.com/**mailman/listinfo/use-livecode< > http://lists.runrev.com/mailman/listinfo/use-livecode> > >> > >> > > _______________________________________________ > > use-livecode mailing list > > use-livecode at lists.runrev.com > > Please visit this url to subscribe, unsubscribe and manage your > subscription preferences: > > http://lists.runrev.com/mailman/listinfo/use-livecode > > > > _______________________________________________ > use-livecode mailing list > use-livecode at lists.runrev.com > Please visit this url to subscribe, unsubscribe and manage your > subscription preferences: > http://lists.runrev.com/mailman/listinfo/use-livecode > > From toolbook at kestner.de Mon Jul 18 03:43:27 2011 From: toolbook at kestner.de (Tiemo Hollmann TB) Date: Mon, 18 Jul 2011 09:43:27 +0200 Subject: AW: security code number generation In-Reply-To: References: <56E8486D-1F05-4291-826D-8C6DB390D12A@gmail.com> Message-ID: <001a01cc451e$64d1aca0$2e7505e0$@de> First: Congratulations to Shao to the winner of the FIFA womens soccer world championship: Japan! As I understand Zygodact, or Peters or Shaos approach the user can pass on his "name, etc." and the generated code to everyone else to unlock the software. How do you handle this issue? Is it just something "as is", is this scenario so negligible in your customer base that you just can ignore it, or do you count on the good in the people that they won't do it? Would be of interest on how you handle this. Tiemo > -----Urspr?ngliche Nachricht----- > Von: use-livecode-bounces at lists.runrev.com [mailto:use-livecode- > bounces at lists.runrev.com] Im Auftrag von Pete > Gesendet: Montag, 18. Juli 2011 09:15 > An: How to use LiveCode > Betreff: Re: security code number generation > > Thanks Ken, that looks good. I guess the other missing piece is how to > control "demo" versions. Expiration dates seem to be the most common, or > maybe some limited function set. > Pete > Molly's Revenge > > > > > On Sun, Jul 17, 2011 at 10:48 PM, Ken Ray wrote: > > > > I'm looking for something to generate license codes for some software I'm > > > planning to sell. Do you think this would work for that purpose? > > > > Take a look at Zygodact; it does exactly this plus it has a DropTool > > component to make it a snap to work with. > > > > http://www.runrev.com/store/product/zygodact-1-0-4/ > > > > > > Ken Ray > > Sons of Thunder Software, Inc. > > Email: kray at sonsothunder.com > > Web Site: http://www.sonsothunder.com/ > > > > > > > Pete > > > Molly's Revenge > > > > > > > > > > > > > > > On Sat, Apr 2, 2011 at 9:51 AM, Peter Brigham MD > > wrote: > > > > > >> For anyone who might have the need, I have a handler I use to generate a > > >> security code, in my case for printed prescriptions. It takes the name > > of > > >> the patient, the date of the prescription, the medication and med > > strength > > >> and hashes all that to produce a ten-digit alphanumeric string (using > > 0-9, > > >> a-z, A-Z). If there is any question about the validity of the > > prescription I > > >> can retrieve the correct code from the rx entry in my database with a > > >> mouseclick (actually recalculating the code from the stored rx data) and > > >> confirm it with the pharmacy. This has proved useful on two occasions > > when a > > >> pt was playing fast and loose with his prescriptions. > > >> > > >> The algorithm is fast in LC, sufficiently obscure that I'm pretty sure > > it > > >> would be hard to hack -- though of course few things are bulletproof in > > >> encryption if someone wants to try hard enough -- and discontinuous in > > the > > >> sense that similar inputs do not generate similar outputs, eg, change > > one > > >> character in the input and the code number is completely different. The > > >> probability of coming up with the correct security number by chance > > alone is > > >> 1 in 10^15 (a million billion to 1). It could be adapted to any number > > of > > >> purposes. I am not posting the handler here, since it would be unwise to > > let > > >> it be archived and available, eg, with a Nabble search, but if anyone is > > >> interested, let me know and I'll share it. > > >> > > >> -- Peter > > >> > > >> Peter M. Brigham > > >> pmbrig at gmail.com > > >> http://home.comcast.net/~**pmbrig > > >> > > >> > > >> ______________________________**_________________ > > >> use-livecode mailing list > > >> use-livecode at lists.runrev.com > > >> Please visit this url to subscribe, unsubscribe and manage your > > >> subscription preferences: > > >> http://lists.runrev.com/**mailman/listinfo/use-livecode< > > http://lists.runrev.com/mailman/listinfo/use-livecode> > > >> > > >> > > > _______________________________________________ > > > use-livecode mailing list > > > use-livecode at lists.runrev.com > > > Please visit this url to subscribe, unsubscribe and manage your > > subscription preferences: > > > http://lists.runrev.com/mailman/listinfo/use-livecode > > > > > > > > _______________________________________________ > > use-livecode mailing list > > use-livecode at lists.runrev.com > > Please visit this url to subscribe, unsubscribe and manage your > > subscription preferences: > > http://lists.runrev.com/mailman/listinfo/use-livecode > > > > > _______________________________________________ > use-livecode mailing list > use-livecode at lists.runrev.com > Please visit this url to subscribe, unsubscribe and manage your subscription > preferences: > http://lists.runrev.com/mailman/listinfo/use-livecode From mwieder at ahsoftware.net Mon Jul 18 04:20:06 2011 From: mwieder at ahsoftware.net (Mark Wieder) Date: Mon, 18 Jul 2011 01:20:06 -0700 Subject: GIGO In-Reply-To: <4DA4D9E3-BBBA-4736-834B-862E106A29C7@wehostmacs.com> References: <4DA4D9E3-BBBA-4736-834B-862E106A29C7@wehostmacs.com> Message-ID: <10829415718.20110718012006@ahsoftware.net> Sean- Sunday, July 17, 2011, 11:27:10 PM, you wrote: > I have a weird algorithm that will produce different output, even with > the same input Garbage Out? > and can be fully reversed back to the pieces that > built the puzzle.. Garbage In? -- -Mark Wieder mwieder at ahsoftware.net From shaosean at wehostmacs.com Mon Jul 18 06:18:43 2011 From: shaosean at wehostmacs.com (Shao Sean) Date: Mon, 18 Jul 2011 06:18:43 -0400 Subject: security code number generation Message-ID: Thanks Tiemo, but Shao is Chinese ;-) In regards to your second point, most password schemes just run an algorithm based on the inputted information to determine that the serial number is correct, or run an algorithm to check that the serial number entered matches a pattern/checksum.. The one that I did (it is just a proof of concept that I did for fun) basically encodes any information you want it to contain.. Once you input your information and the "serial number" you can compare the information that was encoded against the data entered by the user.. As previously mentioned, the exact same input data creates a different encoded output (which was fun to do :-) But the amount of time and energy spent trying to stop people from pirating your software can be better spent making your program worth being paid for.. Keep things simple for the end user to part with their money and to license their software and they will be more inclined to do so (look at all the headaches of DRM ;-) http://images.ncix.com/forumimages/F96CFF63-FCCF-4B43-B25F0CCB4AA06AB4.jpg -Sean From shaosean at wehostmacs.com Mon Jul 18 06:20:08 2011 From: shaosean at wehostmacs.com (Shao Sean) Date: Mon, 18 Jul 2011 06:20:08 -0400 Subject: GIGO Message-ID: If you want to run garbage (in terms of data, not the real stuff) through it ;-) From keith.clarke at clarkeandclarke.co.uk Mon Jul 18 06:35:47 2011 From: keith.clarke at clarkeandclarke.co.uk (Keith Clarke) Date: Mon, 18 Jul 2011 11:35:47 +0100 Subject: How to make LiveCode multidimensional array contents available to Javascript? Message-ID: Hi folks, I have a LiveCode array that contains interesting intranet web site page navigation and page-specific parameters - that I can pass into a revBrowser 'hosted' within my local stack. It's now time to test this resource with users who can't use revlets. Security implications mean that it's not practical in the short-term to turn the array into some kind of web service on my VPS or on-rev server. So, I'm thinking to FTP this array data back to the intranet server as some kind of static resource so that Javascript can access its contents from within standard browser pages. What are the options for packaging-up the LiveCode multi-dimensional array for easy access via Javascript? Is there some kind of binary 'array' file type standard that both languages understand or do I have to use text/XML files or similar? Any insights or research links would be much appreciated. Best, Keith.. From keith.clarke at clarkeandclarke.co.uk Mon Jul 18 06:41:41 2011 From: keith.clarke at clarkeandclarke.co.uk (Keith Clarke) Date: Mon, 18 Jul 2011 11:41:41 +0100 Subject: How to make LiveCode multidimensional array contents available to Javascript? In-Reply-To: References: Message-ID: <9E29F00B-D824-4066-A4E9-635428DA3005@clarkeandclarke.co.uk> ...whoops, forgot to ask whether JSON is one of the options I should be researching. Best, Keith.. On 18 Jul 2011, at 11:35, Keith Clarke wrote: > Hi folks, > I have a LiveCode array that contains interesting intranet web site page navigation and page-specific parameters - that I can pass into a revBrowser 'hosted' within my local stack. > > It's now time to test this resource with users who can't use revlets. Security implications mean that it's not practical in the short-term to turn the array into some kind of web service on my VPS or on-rev server. So, I'm thinking to FTP this array data back to the intranet server as some kind of static resource so that Javascript can access its contents from within standard browser pages. > > What are the options for packaging-up the LiveCode multi-dimensional array for easy access via Javascript? Is there some kind of binary 'array' file type standard that both languages understand or do I have to use text/XML files or similar? > > Any insights or research links would be much appreciated. > Best, > Keith.. From toolbook at kestner.de Mon Jul 18 07:02:16 2011 From: toolbook at kestner.de (Tiemo Hollmann TB) Date: Mon, 18 Jul 2011 13:02:16 +0200 Subject: AW: security code number generation In-Reply-To: References: Message-ID: <002301cc453a$2d649d10$882dd730$@de> Gosh! Sorry Sean for having put you into the wrong country, should have had a look on google maps :) Nevertheless the Japanese women showed a great contest here in germany... Tiemo > -----Urspr?ngliche Nachricht----- > Von: use-livecode-bounces at lists.runrev.com [mailto:use-livecode- > bounces at lists.runrev.com] Im Auftrag von Shao Sean > Gesendet: Montag, 18. Juli 2011 12:19 > An: use-livecode at lists.runrev.com > Betreff: Re: security code number generation > > Thanks Tiemo, but Shao is Chinese ;-) > > In regards to your second point, most password schemes just run an > algorithm based on the inputted information to determine that the > serial number is correct, or run an algorithm to check that the serial > number entered matches a pattern/checksum.. > > The one that I did (it is just a proof of concept that I did for fun) > basically encodes any information you want it to contain.. Once you > input your information and the "serial number" you can compare the > information that was encoded against the data entered by the user.. As > previously mentioned, the exact same input data creates a different > encoded output (which was fun to do :-) > > But the amount of time and energy spent trying to stop people from > pirating your software can be better spent making your program worth > being paid for.. Keep things simple for the end user to part with > their money and to license their software and they will be more > inclined to do so (look at all the headaches of DRM ;-) > > http://images.ncix.com/forumimages/F96CFF63-FCCF-4B43-B25F0CCB4AA06AB4.jpg > > -Sean > > _______________________________________________ > use-livecode mailing list > use-livecode at lists.runrev.com > Please visit this url to subscribe, unsubscribe and manage your subscription > preferences: > http://lists.runrev.com/mailman/listinfo/use-livecode From m.schonewille at economy-x-talk.com Mon Jul 18 07:13:28 2011 From: m.schonewille at economy-x-talk.com (Mark Schonewille) Date: Mon, 18 Jul 2011 13:13:28 +0200 Subject: security code number generation In-Reply-To: References: Message-ID: <01F5861F-E2B6-479F-95B1-FCA51A5C1480@economy-x-talk.com> Hi, Three years ago, I wrote an article, in two parts, which will help you understand the principles of a registration mechanism. http://qery.us/v2 http://qery.us/v3 -- Best regards, Mark Schonewille Economy-x-Talk Consulting and Software Engineering Homepage: http://economy-x-talk.com Twitter: http://twitter.com/xtalkprogrammer KvK: 50277553 New: Download the Installer Maker Plugin 1.6 for LiveCode here http://qery.us/ce From keith.clarke at clarkeandclarke.co.uk Mon Jul 18 07:36:35 2011 From: keith.clarke at clarkeandclarke.co.uk (Keith Clarke) Date: Mon, 18 Jul 2011 12:36:35 +0100 Subject: How to make LiveCode multidimensional array contents available to Javascript? In-Reply-To: <9E29F00B-D824-4066-A4E9-635428DA3005@clarkeandclarke.co.uk> References: <9E29F00B-D824-4066-A4E9-635428DA3005@clarkeandclarke.co.uk> Message-ID: <40DD52AF-5054-45F6-907C-A1104D5C3556@clarkeandclarke.co.uk> So, I discovered the thread on Mark Smith's LibJson project via the list archives and subsequently downloaded v1.0b it from revOnline. This looks like it might be an efficient mechanism to serialise the LiveCode array(s) to subsequently write into a text file, zip, ftp - and then see if Javascript can find (and hopefully understand) the result! Kudos to Mark for sharing. Best, Keith.. On 18 Jul 2011, at 11:41, Keith Clarke wrote: > ...whoops, forgot to ask whether JSON is one of the options I should be researching. > Best, > Keith.. > > On 18 Jul 2011, at 11:35, Keith Clarke wrote: > >> Hi folks, >> I have a LiveCode array that contains interesting intranet web site page navigation and page-specific parameters - that I can pass into a revBrowser 'hosted' within my local stack. >> >> It's now time to test this resource with users who can't use revlets. Security implications mean that it's not practical in the short-term to turn the array into some kind of web service on my VPS or on-rev server. So, I'm thinking to FTP this array data back to the intranet server as some kind of static resource so that Javascript can access its contents from within standard browser pages. >> >> What are the options for packaging-up the LiveCode multi-dimensional array for easy access via Javascript? Is there some kind of binary 'array' file type standard that both languages understand or do I have to use text/XML files or similar? >> >> Any insights or research links would be much appreciated. >> Best, >> Keith.. > From kray at sonsothunder.com Mon Jul 18 09:00:20 2011 From: kray at sonsothunder.com (Ken Ray) Date: Mon, 18 Jul 2011 08:00:20 -0500 Subject: security code number generation In-Reply-To: <01F5861F-E2B6-479F-95B1-FCA51A5C1480@economy-x-talk.com> References: <01F5861F-E2B6-479F-95B1-FCA51A5C1480@economy-x-talk.com> Message-ID: On Jul 18, 2011, at 6:13 AM, Mark Schonewille wrote: > Hi, > > Three years ago, I wrote an article, in two parts, which will help you understand the principles of a registration mechanism. > > http://qery.us/v2 > http://qery.us/v3 Also, although old, this article by Ambrosia Software on the "Plain Truth About Piracy" is still worth a read: http://www.ambrosiasw.com/forums/index.php?showtopic=34059 Ken Ray Sons of Thunder Software, Inc. Email: kray at sonsothunder.com Web Site: http://www.sonsothunder.com/ From kee at kagi.com Mon Jul 18 09:44:54 2011 From: kee at kagi.com (Kee Nethery) Date: Mon, 18 Jul 2011 06:44:54 -0700 Subject: security code number generation In-Reply-To: References: <56E8486D-1F05-4291-826D-8C6DB390D12A@gmail.com> Message-ID: <45D45451-9A90-4788-AC56-9C8D767B99D9@kagi.com> Most unlock algorithms have a method of confirming that the unlock code is correct. One or more verification steps. That verification code lives in the application that users (and crackers) have access to. If someone really wanted to create a code generator for your software they'll be able to by looking at the verification code in the application. How do you tell the difference between a good and bogus unlock code when someone releases am unlock code generator for your software? One method of handling this is to create the unlock code with say a dozen verification parameters. For example, a verification check might be that the 4th char is always mod 3 of the 8th char. In your application, include checks for only 6 of the parameters. If someone releases a code generator, it will create codes that pass the 6 validation steps but it is highly unlikely that they will pass the 6 that you did not check for in your application. The next version of the application can add 2 of the additional parameters to the verification step and you will still be able to tell the difference between good and bogus codes. But, in general, I agree with the previous comment. Time spent building code to prevent people from using your software is typically better spent building code that makes people want to use your software. For most software, people who pirate it were never going to pay for it and probably are not actually using it. Focus on building market share. Kee Nethery From pmbrig at gmail.com Mon Jul 18 09:46:32 2011 From: pmbrig at gmail.com (Peter Brigham MD) Date: Mon, 18 Jul 2011 09:46:32 -0400 Subject: AW: security code number generation In-Reply-To: <001a01cc451e$64d1aca0$2e7505e0$@de> References: <56E8486D-1F05-4291-826D-8C6DB390D12A@gmail.com> <001a01cc451e$64d1aca0$2e7505e0$@de> Message-ID: <0DB64C8F-8BF1-4BB2-AE37-8C9F9C78FDC4@gmail.com> You could include (without telling the user) the contents of $user as part of what is inputted to generate the code. This would make it more secure but cause problems with portability, etc. I agree with the later posts pointing out the trade-offs between security and user-frendliness. Consider your decisions in that light. -- Peter Peter M. Brigham pmbrig at gmail.com http://home.comcast.net/~pmbrig On Jul 18, 2011, at 3:43 AM, Tiemo Hollmann TB wrote: > First: Congratulations to Shao to the winner of the FIFA womens soccer world > championship: Japan! > > As I understand Zygodact, or Peters or Shaos approach the user can pass on > his "name, etc." and the generated code to everyone else to unlock the > software. > How do you handle this issue? Is it just something "as is", is this scenario > so negligible in your customer base that you just can ignore it, or do you > count on the good in the people that they won't do it? > Would be of interest on how you handle this. > Tiemo > > > >> -----Urspr?ngliche Nachricht----- >> Von: use-livecode-bounces at lists.runrev.com [mailto:use-livecode- >> bounces at lists.runrev.com] Im Auftrag von Pete >> Gesendet: Montag, 18. Juli 2011 09:15 >> An: How to use LiveCode >> Betreff: Re: security code number generation >> >> Thanks Ken, that looks good. I guess the other missing piece is how to >> control "demo" versions. Expiration dates seem to be the most common, or >> maybe some limited function set. >> Pete >> Molly's Revenge >> >> >> >> >> On Sun, Jul 17, 2011 at 10:48 PM, Ken Ray wrote: >> >>>> I'm looking for something to generate license codes for some software > I'm >>>> planning to sell. Do you think this would work for that purpose? >>> >>> Take a look at Zygodact; it does exactly this plus it has a DropTool >>> component to make it a snap to work with. >>> >>> http://www.runrev.com/store/product/zygodact-1-0-4/ >>> >>> >>> Ken Ray >>> Sons of Thunder Software, Inc. >>> Email: kray at sonsothunder.com >>> Web Site: http://www.sonsothunder.com/ >>> >>> >>>> Pete >>>> Molly's Revenge >>>> >>>> >>>> >>>> >>>> On Sat, Apr 2, 2011 at 9:51 AM, Peter Brigham MD >>> wrote: >>>> >>>>> For anyone who might have the need, I have a handler I use to > generate a >>>>> security code, in my case for printed prescriptions. It takes the > name >>> of >>>>> the patient, the date of the prescription, the medication and med >>> strength >>>>> and hashes all that to produce a ten-digit alphanumeric string (using >>> 0-9, >>>>> a-z, A-Z). If there is any question about the validity of the >>> prescription I >>>>> can retrieve the correct code from the rx entry in my database with a >>>>> mouseclick (actually recalculating the code from the stored rx data) > and >>>>> confirm it with the pharmacy. This has proved useful on two occasions >>> when a >>>>> pt was playing fast and loose with his prescriptions. >>>>> >>>>> The algorithm is fast in LC, sufficiently obscure that I'm pretty > sure >>> it >>>>> would be hard to hack -- though of course few things are bulletproof > in >>>>> encryption if someone wants to try hard enough -- and discontinuous > in >>> the >>>>> sense that similar inputs do not generate similar outputs, eg, change >>> one >>>>> character in the input and the code number is completely different. > The >>>>> probability of coming up with the correct security number by chance >>> alone is >>>>> 1 in 10^15 (a million billion to 1). It could be adapted to any > number >>> of >>>>> purposes. I am not posting the handler here, since it would be unwise > to >>> let >>>>> it be archived and available, eg, with a Nabble search, but if anyone > is >>>>> interested, let me know and I'll share it. >>>>> >>>>> -- Peter >>>>> >>>>> Peter M. Brigham >>>>> pmbrig at gmail.com >>>>> http://home.comcast.net/~**pmbrig >>>>> >>>>> >>>>> ______________________________**_________________ >>>>> use-livecode mailing list >>>>> use-livecode at lists.runrev.com >>>>> Please visit this url to subscribe, unsubscribe and manage your >>>>> subscription preferences: >>>>> http://lists.runrev.com/**mailman/listinfo/use-livecode< >>> http://lists.runrev.com/mailman/listinfo/use-livecode> >>>>> >>>>> >>>> _______________________________________________ >>>> use-livecode mailing list >>>> use-livecode at lists.runrev.com >>>> Please visit this url to subscribe, unsubscribe and manage your >>> subscription preferences: >>>> http://lists.runrev.com/mailman/listinfo/use-livecode >>> >>> >>> >>> _______________________________________________ >>> use-livecode mailing list >>> use-livecode at lists.runrev.com >>> Please visit this url to subscribe, unsubscribe and manage your >>> subscription preferences: >>> http://lists.runrev.com/mailman/listinfo/use-livecode >>> >>> >> _______________________________________________ >> use-livecode mailing list >> use-livecode at lists.runrev.com >> Please visit this url to subscribe, unsubscribe and manage your > subscription >> preferences: >> http://lists.runrev.com/mailman/listinfo/use-livecode > > > _______________________________________________ > use-livecode mailing list > use-livecode at lists.runrev.com > Please visit this url to subscribe, unsubscribe and manage your subscription preferences: > http://lists.runrev.com/mailman/listinfo/use-livecode From andre at andregarzia.com Mon Jul 18 11:07:34 2011 From: andre at andregarzia.com (Andre Garzia) Date: Mon, 18 Jul 2011 12:07:34 -0300 Subject: New blog post: setting up Livecode server at Dreamhost VPS Message-ID: Folks, I know people struggled with this in the past: http://andregarzia.com/page/dreamhostps Showing how I setup Livecode Server on a Dreamhost VPS. The usual method did not work there so I had to go extra miles to make it work. Cheers andre -- http://www.andregarzia.com All We Do Is Code. From jacque at hyperactivesw.com Mon Jul 18 11:40:01 2011 From: jacque at hyperactivesw.com (J. Landman Gay) Date: Mon, 18 Jul 2011 10:40:01 -0500 Subject: AW: security code number generation In-Reply-To: <001a01cc451e$64d1aca0$2e7505e0$@de> References: <56E8486D-1F05-4291-826D-8C6DB390D12A@gmail.com> <001a01cc451e$64d1aca0$2e7505e0$@de> Message-ID: <4E2453D1.2070003@hyperactivesw.com> On 7/18/11 2:43 AM, Tiemo Hollmann TB wrote: > As I understand Zygodact, or Peters or Shaos approach the user can pass on > his "name, etc." and the generated code to everyone else to unlock the > software. > How do you handle this issue? Is it just something "as is", is this scenario > so negligible in your customer base that you just can ignore it, or do you > count on the good in the people that they won't do it? > Would be of interest on how you handle this. I agree with everyone who said there is a limit to how much effort you should put into piracy protection. However, Zygodact includes a CGI script that can generate serial keys from the server. The script can be altered to check a database on the server. If there are too many registrations for a particular name and serial key, the script could return a message to the app that the user needs to contact you, or do something else to notify you. This technique isn't perfect either, because it requires that the end user has internet access when registering (or at some point anyway) and some users get very upset when an app "calls home". So again, the trade-off is something to consider. If the app is one that works only with the internet, then it's probably okay because access will be expected. But if the app is a self-contained game, for example, users may lose trust, and sometimes they broadcast that in comments or ratings. -- Jacqueline Landman Gay | jacque at hyperactivesw.com HyperActive Software | http://www.hyperactivesw.com From capellan2000 at gmail.com Mon Jul 18 13:07:19 2011 From: capellan2000 at gmail.com (Alejandro Tejada) Date: Mon, 18 Jul 2011 13:07:19 -0400 Subject: [ANN] alien camel in your LiveCode Message-ID: Hi Thierry, > on Fri Jul 15 2011 > Thierry Douez wrote: > I'm pleased to announce SunnYperl. > Download today a trial version from http://sunny-tdz.com > Formerly called rev2perl, SunnYperl is an external > which embeds Perl inside your LiveCode engine. > Or in other words, you can write externals in Perl. This is the more recent landmarking announcement in this platform. You have put in our developers hands all the Perl wisdom available to each stack! :-D My warmest Congratulations! and Thanks for publishing your landmarking work with Perl. As everybody notices, most developers are really busy with the iOS and Android platforms, so do not worry if your announcement does not generate one thousands answers. My platform is Windows, so I want to see more working examples for this platform, in the next weeks. Thanks again for sharing your work with your fellow LiveCode developers! Al From pete at mollysrevenge.com Mon Jul 18 13:16:07 2011 From: pete at mollysrevenge.com (Pete) Date: Mon, 18 Jul 2011 10:16:07 -0700 Subject: AW: security code number generation In-Reply-To: <4E2453D1.2070003@hyperactivesw.com> References: <56E8486D-1F05-4291-826D-8C6DB390D12A@gmail.com> <001a01cc451e$64d1aca0$2e7505e0$@de> <4E2453D1.2070003@hyperactivesw.com> Message-ID: Thanks to everyone for the thoughts on this. I agree that you can never 100% prevent piracy from those who are determined enough and can waste a lot of time and effort trying to do so. I guess that's a plus for Zygodact - I buy the product, spend perhaps a couple of hours doing whatever customisation I think I need, and I'm done. The target audience for the product I'm working on ( a Livecode/SQL database application development tool) is almost exclusively Livecode developers, not the end users of the applications they develop. I think developers, in general, are a pretty honest bunch so I'm not overly concerned about licensing/piracy but seems like there needs to be something in place, if only as a reminder that their demo period is about to expire and it's time to pay if they like what they've seen (that's certainly prodded me into payment in the past). I'm also reminded of the recent discussion on the use of password protected stacks. I'm tempted to go that route, not for reasons of piracy protection, but mainly because I could see huge support problems for me if people change my code for some reason then find that they've broken it. On the other hand, they could improve it :-) Pete Molly's Revenge On Mon, Jul 18, 2011 at 8:40 AM, J. Landman Gay wrote: > On 7/18/11 2:43 AM, Tiemo Hollmann TB wrote: > > As I understand Zygodact, or Peters or Shaos approach the user can pass on >> his "name, etc." and the generated code to everyone else to unlock the >> software. >> How do you handle this issue? Is it just something "as is", is this >> scenario >> so negligible in your customer base that you just can ignore it, or do you >> count on the good in the people that they won't do it? >> Would be of interest on how you handle this. >> > > I agree with everyone who said there is a limit to how much effort you > should put into piracy protection. However, Zygodact includes a CGI script > that can generate serial keys from the server. The script can be altered to > check a database on the server. If there are too many registrations for a > particular name and serial key, the script could return a message to the app > that the user needs to contact you, or do something else to notify you. > > This technique isn't perfect either, because it requires that the end user > has internet access when registering (or at some point anyway) and some > users get very upset when an app "calls home". So again, the trade-off is > something to consider. If the app is one that works only with the internet, > then it's probably okay because access will be expected. But if the app is a > self-contained game, for example, users may lose trust, and sometimes they > broadcast that in comments or ratings. > > -- > Jacqueline Landman Gay | jacque at hyperactivesw.com > HyperActive Software | http://www.hyperactivesw.com > > ______________________________**_________________ > > use-livecode mailing list > use-livecode at lists.runrev.com > Please visit this url to subscribe, unsubscribe and manage your > subscription preferences: > http://lists.runrev.com/**mailman/listinfo/use-livecode > > From stephenREVOLUTION2 at barncard.com Mon Jul 18 13:19:55 2011 From: stephenREVOLUTION2 at barncard.com (stephen barncard) Date: Mon, 18 Jul 2011 10:19:55 -0700 Subject: How to make LiveCode multidimensional array contents available to Javascript? In-Reply-To: <40DD52AF-5054-45F6-907C-A1104D5C3556@clarkeandclarke.co.uk> References: <9E29F00B-D824-4066-A4E9-635428DA3005@clarkeandclarke.co.uk> <40DD52AF-5054-45F6-907C-A1104D5C3556@clarkeandclarke.co.uk> Message-ID: Yeah, send out an APB - I've been trying to contact him about his audio work. s q b On 18 July 2011 04:36, Keith Clarke wrote: > So, I discovered the thread on Mark Smith's LibJson project via the list > archives and subsequently downloaded v1.0b it from revOnline. > > This looks like it might be an efficient mechanism to serialise the > LiveCode array(s) to subsequently write into a text file, zip, ftp - and > then see if Javascript can find (and hopefully understand) the result! > > Kudos to Mark for sharing. > Best, > Keith.. > > On 18 Jul 2011, at 11:41, Keith Clarke wrote: > > > ...whoops, forgot to ask whether JSON is one of the options I should be > researching. > > Best, > > Keith.. > > > > On 18 Jul 2011, at 11:35, Keith Clarke wrote: > > > >> Hi folks, > >> I have a LiveCode array that contains interesting intranet web site page > navigation and page-specific parameters - that I can pass into a revBrowser > 'hosted' within my local stack. > >> > >> It's now time to test this resource with users who can't use revlets. > Security implications mean that it's not practical in the short-term to turn > the array into some kind of web service on my VPS or on-rev server. So, I'm > thinking to FTP this array data back to the intranet server as some kind of > static resource so that Javascript can access its contents from within > standard browser pages. > >> > >> What are the options for packaging-up the LiveCode multi-dimensional > array for easy access via Javascript? Is there some kind of binary 'array' > file type standard that both languages understand or do I have to use > text/XML files or similar? > >> > >> Any insights or research links would be much appreciated. > >> Best, > >> Keith.. > > > > > _______________________________________________ > use-livecode mailing list > use-livecode at lists.runrev.com > Please visit this url to subscribe, unsubscribe and manage your > subscription preferences: > http://lists.runrev.com/mailman/listinfo/use-livecode > -- Stephen Barncard San Francisco Ca. USA more about sqb From th.douez at gmail.com Mon Jul 18 13:43:57 2011 From: th.douez at gmail.com (Thierry Douez) Date: Mon, 18 Jul 2011 19:43:57 +0200 Subject: [ANN] alien camel in your LiveCode In-Reply-To: References: Message-ID: 2011/7/18 Alejandro Tejada > > > Thierry Douez wrote: > > > I'm pleased to announce SunnYperl. > > Download today a trial version from http://sunny-tdz.com > > Formerly called rev2perl, SunnYperl is an external > > which embeds Perl inside your LiveCode engine. > > Or in other words, you can write externals in Perl. > > This is the more recent landmarking announcement in > this platform. > > You have put in our developers hands all the Perl wisdom > available to each stack! :-D > :) > > My warmest Congratulations! and Thanks for publishing > your landmarking work with Perl. > > As everybody notices, most developers are really busy > with the iOS and Android platforms, so do not worry > if your announcement does not generate one thousands > answers. > Actually, I don't expect that much of answers :) > > My platform is Windows, so I want to see more working > examples for this platform, in the next weeks. > > Thanks again for sharing your work with > your fellow LiveCode developers! > Thanks for your words. Let me know off-list your Perl configuration ( distribution and version) and I'll make the first one on Windows works for you. Actually, I've started to build the external for windows with ActivePerl 5.12.4 ( free pre-build perl and easy installation from ActiveState ) For the records, in the first days ( 2003 ), SunnYperl has been coded first for Linux, then for Windows. That said, as soon as I have some times for that, it should come fast. Regards, Thierry http://sunny-tdz.com - freelance developer From bobs at twft.com Mon Jul 18 13:57:11 2011 From: bobs at twft.com (Bob Sneidar) Date: Mon, 18 Jul 2011 10:57:11 -0700 Subject: Floating City and K12 Game Theory Model... In-Reply-To: <65848462-D50C-4B62-AB9D-BA2EFD26AA34@mac.com> References: <65848462-D50C-4B62-AB9D-BA2EFD26AA34@mac.com> Message-ID: <64141D5D-E183-4B38-A649-2C0590A77443@twft.com> Fascinating. I noticed however, the description of The Poison City on a Macintosh running Firefox, exceeded the lower bounds of it's box, and there is no method for scrolling down. It seems fixed. Bob On Jul 15, 2011, at 5:09 PM, John Patten wrote: > Thanks Scott and Brent...I'll look at both of your solutions regarding large images and groups... > > FWIW, I've been looking at how an online game called The Floating City (http://www.floatingcity.com was put together...and how some of the principle game activities could be incorporated into an instructional model in the K12 education environment. Naturally, the easiest way to duplicate their model is doing it in LiveCode ;-) > > I put together the following example in a very short amount of time. Obviously, if I actually create something I'll use it will take much longer, designing the narrative, graphics, securing the code, etc. etc. But I was pretty much floored how quickly and how easy it was to create with LiveCode. I think I only posted questions to the list a couple of times! :-) > > I've put down more of a description of what I'm attempting to do here if interested: http://edutonica.blogspot.com/ > > > Thanks everyone! > > John Patten > SUSD > _______________________________________________ > use-livecode mailing list > use-livecode at lists.runrev.com > Please visit this url to subscribe, unsubscribe and manage your subscription preferences: > http://lists.runrev.com/mailman/listinfo/use-livecode From chipp at chipp.com Mon Jul 18 14:19:15 2011 From: chipp at chipp.com (Chipp Walters) Date: Mon, 18 Jul 2011 13:19:15 -0500 Subject: AW: security code number generation In-Reply-To: <4E2453D1.2070003@hyperactivesw.com> References: <56E8486D-1F05-4291-826D-8C6DB390D12A@gmail.com> <001a01cc451e$64d1aca0$2e7505e0$@de> <4E2453D1.2070003@hyperactivesw.com> Message-ID: Our own home rolled scheme relies on the customers email address as the first part of the key. So, a customer has to enter their email address, then the regcode. If they want to share the app, they need to share their email address. This has a couple of intended consequences. First, I think a customer may be less inclined to share their application when they know it may be their PayPal email address which is also being shared. Secondly, if I run across one of my 'apps in the wild,' and it includes a full regkey, I can see quickly who shared the app. Of course, hard core regcode generator hackers may still create their own codes-- but nothing I will ever do will stop them anyway. It ends up turning into an arms escalation scenario. What I'm after is trying to stop the 'casual' pirate-- the guy who 'lends' their software to a friend. And, the last thing I want to do is to have my copy protection get in the way of my customers use experience. Like Jacque, our system also has a PHP version (which Chris wrote) so we can automate the delivery of regcodes. From m.schonewille at economy-x-talk.com Mon Jul 18 18:58:29 2011 From: m.schonewille at economy-x-talk.com (Mark Schonewille) Date: Tue, 19 Jul 2011 00:58:29 +0200 Subject: ANN: LiveCodeErrors for iPhone Message-ID: Dear LiveCode programmers, Economy-x-Talk is pleased to announce a simple little app for iPhone to help you check the meaning of a LiveCode error. Whenever you see an error code like 573,2,1,mouseOp, the LiveCodeErrors app will tell what it means ("handler not found", in this example). How often have you seen an inexplicable error while running your app on your computer and wishing for a separate device to check the meaning of that error? Now you can look up that error code, simply by entering the first item of the code in the LiveCodeErrors iPhone app and pressing Search. You'll instantly know what your LiveCode script is trying to tell you. I have been using the app myself for several weeks now and already I can't do without it anymore. If you buy this app and have any comments, please let me know. Just click the e-mail link in the app. Follow this link http://qery.us/v4 to find LiveCodeErrors in the iTunes store. -- Best regards, Mark Schonewille Economy-x-Talk Consulting and Software Engineering Homepage: http://economy-x-talk.com Twitter: http://twitter.com/xtalkprogrammer KvK: 50277553 New: Download the Installer Maker Plugin 1.6 for LiveCode here http://qery.us/ce From mpezzo at gmail.com Tue Jul 19 00:00:04 2011 From: mpezzo at gmail.com (AcidJazz) Date: Mon, 18 Jul 2011 21:00:04 -0700 (PDT) Subject: revmpossible? Message-ID: <1311048004923-3677225.post@n4.nabble.com> Hey Andre, When do you think you'll be posting your new and incredible revimpossible on your new (and impressive) website. In the back of my mind I fear that it was all a joke and that's why you called it revimpossible, but for those of us who have all but given up on a working web plugin, you have given use some hope... Best, Mark -- View this message in context: http://runtime-revolution.278305.n4.nabble.com/revmpossible-tp3677225p3677225.html Sent from the Revolution - User mailing list archive at Nabble.com. From andre at andregarzia.com Tue Jul 19 00:32:49 2011 From: andre at andregarzia.com (Andre Garzia) Date: Tue, 19 Jul 2011 01:32:49 -0300 Subject: revmpossible? In-Reply-To: <1311048004923-3677225.post@n4.nabble.com> References: <1311048004923-3677225.post@n4.nabble.com> Message-ID: Mark, It was not a joke but it is not a replacement for the web plugin. I am working on the social networking libraries and the CURL external first. I will only resume work on revImpossible when those are more or less working. revImpossible is a way to create websites from inside LiveCode IDE using LiveCode Server + RevIgniter. It allows you to design your site using stacks and it will generate HTML/CSS/JS and more from your stacks but it is not a replacement for the webplugin, you can't pick a normal stack and pass it thru it. It is made to create sites and not to convert stacks to the web. :-) I will try to post a video of the prototype working this weekend. Cheers andre On Tue, Jul 19, 2011 at 1:00 AM, AcidJazz wrote: > Hey Andre, > > When do you think you'll be posting your new and incredible revimpossible > on > your new (and impressive) website. In the back of my mind I fear that it > was all a joke and that's why you called it revimpossible, but for those of > us who have all but given up on a working web plugin, you have given use > some hope... > > Best, > Mark > > -- > View this message in context: > http://runtime-revolution.278305.n4.nabble.com/revmpossible-tp3677225p3677225.html > Sent from the Revolution - User mailing list archive at Nabble.com. > > _______________________________________________ > use-livecode mailing list > use-livecode at lists.runrev.com > Please visit this url to subscribe, unsubscribe and manage your > subscription preferences: > http://lists.runrev.com/mailman/listinfo/use-livecode > -- http://www.andregarzia.com All We Do Is Code. From tsj at unimelb.edu.au Tue Jul 19 00:38:42 2011 From: tsj at unimelb.edu.au (Terry Judd) Date: Tue, 19 Jul 2011 14:38:42 +1000 Subject: Adding cookies to RevBrowser Message-ID: My university has just implemented a Single Sign-On service which is probably a good thing for most but breaks all my existing routines for authenticating against the university library (to allow me to display library resources either in a revBrowser instance or in the user?s default browser). Anyway, I can apparently request a token (by posting a username and password combination to a particular url) and use this to set a browser cookie, which should then allow me to avoid displaying an SSO login screen to the user when they first try to access a library resource. Can someone give me some clues as to how I go about manually setting a named cookie for RevBrowser and/or the user?s default browser? Cheers, Terry... -- Dr Terry Judd | Senior Lecturer in Medical Education Medical Education Unit Melbourne Medical School The University of Melbourne From slava at lexiconbridge.com Tue Jul 19 01:08:21 2011 From: slava at lexiconbridge.com (Slava Paperno) Date: Tue, 19 Jul 2011 01:08:21 -0400 Subject: Adding cookies to RevBrowser In-Reply-To: References: Message-ID: <000901cc45d1$e4a2d5e0$ade881a0$@com> You can manipulate the HTML of the Web page that is about to load in your revBrowser instance, and thus add a JS function to that Web page. The function can set the cookie that you need. You can call the function either from the onload trigger in the body tag, or by calling it from LC. Slava > -----Original Message----- > From: use-livecode-bounces at lists.runrev.com [mailto:use-livecode- > bounces at lists.runrev.com] On Behalf Of Terry Judd > Sent: Tuesday, July 19, 2011 12:39 AM > To: How to use LiveCode > Subject: Adding cookies to RevBrowser > > My university has just implemented a Single Sign-On service which is > probably a good thing for most but breaks all my existing routines for > authenticating against the university library (to allow me to display > library resources either in a revBrowser instance or in the user?s > default > browser). > > Anyway, I can apparently request a token (by posting a username and > password > combination to a particular url) and use this to set a browser cookie, > which > should then allow me to avoid displaying an SSO login screen to the > user > when they first try to access a library resource. Can someone give me > some > clues as to how I go about manually setting a named cookie for > RevBrowser > and/or the user?s default browser? > > Cheers, > > Terry... > > -- > Dr Terry Judd | Senior Lecturer in Medical Education > Medical Education Unit > Melbourne Medical School > The University of Melbourne > > > _______________________________________________ > use-livecode mailing list > use-livecode at lists.runrev.com > Please visit this url to subscribe, unsubscribe and manage your > subscription preferences: > http://lists.runrev.com/mailman/listinfo/use-livecode From tsj at unimelb.edu.au Tue Jul 19 01:48:46 2011 From: tsj at unimelb.edu.au (Terry Judd) Date: Tue, 19 Jul 2011 15:48:46 +1000 Subject: Adding cookies to RevBrowser In-Reply-To: <000901cc45d1$e4a2d5e0$ade881a0$@com> Message-ID: Thanks Slava - looks like I'll have to brush up on my javascript. Cheers, Terry... On 19/07/2011 03:08 PM, "Slava Paperno" wrote: > You can manipulate the HTML of the Web page that is about to load in your > revBrowser instance, and thus add a JS function to that Web page. The > function can set the cookie that you need. You can call the function either > from the onload trigger in the body tag, or by calling it from LC. > > Slava > >> -----Original Message----- >> From: use-livecode-bounces at lists.runrev.com [mailto:use-livecode- >> bounces at lists.runrev.com] On Behalf Of Terry Judd >> Sent: Tuesday, July 19, 2011 12:39 AM >> To: How to use LiveCode >> Subject: Adding cookies to RevBrowser >> >> My university has just implemented a Single Sign-On service which is >> probably a good thing for most but breaks all my existing routines for >> authenticating against the university library (to allow me to display >> library resources either in a revBrowser instance or in the user?s >> default >> browser). >> >> Anyway, I can apparently request a token (by posting a username and >> password >> combination to a particular url) and use this to set a browser cookie, >> which >> should then allow me to avoid displaying an SSO login screen to the >> user >> when they first try to access a library resource. Can someone give me >> some >> clues as to how I go about manually setting a named cookie for >> RevBrowser >> and/or the user?s default browser? >> >> Cheers, >> >> Terry... >> >> -- >> Dr Terry Judd | Senior Lecturer in Medical Education >> Medical Education Unit >> Melbourne Medical School >> The University of Melbourne >> >> >> _______________________________________________ >> use-livecode mailing list >> use-livecode at lists.runrev.com >> Please visit this url to subscribe, unsubscribe and manage your >> subscription preferences: >> http://lists.runrev.com/mailman/listinfo/use-livecode > > > > _______________________________________________ > use-livecode mailing list > use-livecode at lists.runrev.com > Please visit this url to subscribe, unsubscribe and manage your subscription > preferences: > http://lists.runrev.com/mailman/listinfo/use-livecode > -- Dr Terry Judd | Senior Lecturer in Medical Education Medical Education Unit Melbourne Medical School The University of Melbourne From dixonja at hotmail.co.uk Tue Jul 19 05:23:13 2011 From: dixonja at hotmail.co.uk (John Dixon) Date: Tue, 19 Jul 2011 10:23:13 +0100 Subject: Record Sound iOS In-Reply-To: <4E21C77F.6070205@fourthworld.com> References: <65848462-D50C-4B62-AB9D-BA2EFD26AA34@mac.com>, <4E21C77F.6070205@fourthworld.com> Message-ID: Hi... Has anyone got 'record sound' to work under iOS. If you have done would you give me a nudge in the right direction...:-) be well From ambassador at fourthworld.com Tue Jul 19 09:03:27 2011 From: ambassador at fourthworld.com (Richard Gaskin) Date: Tue, 19 Jul 2011 06:03:27 -0700 Subject: ANN: LiveCodeErrors for iPhone In-Reply-To: References: Message-ID: <4E25809F.7080803@fourthworld.com> Mark Schonewille wrote: > How often have you seen an inexplicable error while running your > app on your computer and wishing for a separate device to check > the meaning of that error? Here, never. Under what circumstances does this happen? What is the RQCC report for this? -- Richard Gaskin Fourth World LiveCode training and consulting: http://www.fourthworld.com Webzine for LiveCode developers: http://www.LiveCodeJournal.com LiveCode Journal blog: http://LiveCodejournal.com/blog.irv From m.schonewille at economy-x-talk.com Tue Jul 19 09:19:01 2011 From: m.schonewille at economy-x-talk.com (Mark Schonewille) Date: Tue, 19 Jul 2011 15:19:01 +0200 Subject: ANN: LiveCodeErrors for iPhone In-Reply-To: <4E25809F.7080803@fourthworld.com> References: <4E25809F.7080803@fourthworld.com> Message-ID: Good for you, Richard. -- Best regards, Mark Schonewille Economy-x-Talk Consulting and Software Engineering Homepage: http://economy-x-talk.com Twitter: http://twitter.com/xtalkprogrammer KvK: 50277553 New: Download the Installer Maker Plugin 1.6 for LiveCode here http://qery.us/ce On 19 jul 2011, at 15:03, Richard Gaskin wrote: > Mark Schonewille wrote: > > > How often have you seen an inexplicable error while running your > > app on your computer and wishing for a separate device to check > > the meaning of that error? > > Here, never. > > Under what circumstances does this happen? > > What is the RQCC report for this? > > -- > Richard Gaskin > Fourth World > LiveCode training and consulting: http://www.fourthworld.com > Webzine for LiveCode developers: http://www.LiveCodeJournal.com > LiveCode Journal blog: http://LiveCodejournal.com/blog.irv > > _______________________________________________ > use-livecode mailing list > use-livecode at lists.runrev.com > Please visit this url to subscribe, unsubscribe and manage your subscription preferences: > http://lists.runrev.com/mailman/listinfo/use-livecode From kray at sonsothunder.com Tue Jul 19 09:50:44 2011 From: kray at sonsothunder.com (Ken Ray) Date: Tue, 19 Jul 2011 08:50:44 -0500 Subject: Mac OS X missing controls in the LiveCode IDE In-Reply-To: References: Message-ID: <4A0BC648-2559-4EE2-88B8-13F369E16F52@sonsothunder.com> On Jul 11, 2011, at 9:31 AM, Ren? Micout wrote: > I made an inventory of missing Mac OS X controls, see list below : > 1. Round button > 2. ? Textured button > http://blog.chipp.com/new-altbutton-control-for-livecode/ (?) > 3. ? Recessed button > I have a project on it > 4. ? Disclosure button > I have a project on it > 5. ? Gradient button > http://www.buttongadget.com (?) > 6. ? Rounded rect button > http://blog.chipp.com/new-altbutton-control-for-livecode/ (?) > 7. ? Rounded textured button > http://blog.chipp.com/new-altbutton-control-for-livecode/ (?) > 8. ? Help button > I have a project on it > 9. ? Bevel button > http://blog.chipp.com/new-altbutton-control-for-livecode/ (?) > 10. ? Radio button > size mini > I work on it > 11. ? Check box > size mini > I work on it > 12. ? Stepper > size mini > I work on it > 13. Date picker > 14. Segmented control > 15. ? Search field > http://droptools.sonsothunder.com/all.irev > 16. Token field > 17. ? Image Well > http://droptools.sonsothunder.com/all.irev > 18. Color well > 19. ? Slider (vertical and horizontal) > size : small and mini > I work on it > 20. ? Circular slider > I have a project on it > 21. ? Circular progress indicator > http://revonline2.runrev.com/stack/440/Spinning-wheel > 22. Vertical and horizontal split view > 23. ? HUD panel > http://revonline2.runrev.com/stack/574/HUD-Panel-Factory > 24. Puff of smoke animation > 25. Answer font > 26. Toolbar and his items (separator toolbar, print toolbar item, show color toolbar item, show font toolbar item, customize toolbar item, search toolbar item). > Some have their "custom control"... But not all... Is my list up to date, incomplete or wrong? Sorry to be so late to the party, but the OS X HI Guidelines have quite a few more controls than in your list, like action menu butttons, path controls, etc. - you should download the guidelines (or view them online) to fill in the gaps... here's the direct URL to the "Controls" portion of the HIG: http://tinyurl.com/2boaz8x If you don't have access, let me know and I'll send you a link to the PDF version to download. Ken Ray Sons of Thunder Software, Inc. Email: kray at sonsothunder.com Web Site: http://www.sonsothunder.com/ From ambassador at fourthworld.com Tue Jul 19 10:04:45 2011 From: ambassador at fourthworld.com (Richard Gaskin) Date: Tue, 19 Jul 2011 07:04:45 -0700 Subject: ANN: LiveCodeErrors for iPhone In-Reply-To: References: Message-ID: <4E258EFD.9010103@fourthworld.com> I appreciate your quick reply, but my questions were in earnest: Under what circumstances does this happen? What is the RQCC report for this? It would seem more useful to address these with RunRev rather than give everyone using the App Store the impression that LiveCode's error dialog doesn't exist. If there's a recipe for reproducing an error which isn't trapped by the error dialog I would imagine the folks at RunRev would want to repair that ASAP. If the issue has been reported and remains outstanding, it may be that the recipe provided has been difficult to reproduce. If you would kindly provide the recipe or the URL to the report which includes it I would be happy to verify it to help the issue get addressed quickly. Thanks in advance for your help on this - -- Richard Gaskin Fourth World LiveCode training and consulting: http://www.fourthworld.com Webzine for LiveCode developers: http://www.LiveCodeJournal.com LiveCode Journal blog: http://LiveCodejournal.com/blog.irv Mark Schonewille wrote: > Good for you, Richard. > > -- > Best regards, > > Mark Schonewille > > Economy-x-Talk Consulting and Software Engineering > Homepage: http://economy-x-talk.com > Twitter: http://twitter.com/xtalkprogrammer > KvK: 50277553 > > New: Download the Installer Maker Plugin 1.6 for LiveCode here http://qery.us/ce > > On 19 jul 2011, at 15:03, Richard Gaskin wrote: > >> Mark Schonewille wrote: >> >> > How often have you seen an inexplicable error while running your >> > app on your computer and wishing for a separate device to check >> > the meaning of that error? >> >> Here, never. >> >> Under what circumstances does this happen? >> >> What is the RQCC report for this? >> From rene.micout at numericable.com Tue Jul 19 10:09:19 2011 From: rene.micout at numericable.com (=?iso-8859-1?Q?Ren=E9_Micout?=) Date: Tue, 19 Jul 2011 16:09:19 +0200 Subject: Mac OS X missing controls in the LiveCode IDE In-Reply-To: <4A0BC648-2559-4EE2-88B8-13F369E16F52@sonsothunder.com> References: <4A0BC648-2559-4EE2-88B8-13F369E16F52@sonsothunder.com> Message-ID: <20E980F0-BBD2-4A9E-8FA3-86FAB2FEBB27@numericable.com> Thank you Ken... I knew my list was not exhaustive, I would have to update, I expect the (imminent) Lion release to make the point ... Also, I do not lose sight of the need to update my tools to be compatible withDropTools ... Bon souvenir de Paris Ren? Le 19 juil. 2011 ? 15:50, Ken Ray a ?crit : > > On Jul 11, 2011, at 9:31 AM, Ren? Micout wrote: > >> I made an inventory of missing Mac OS X controls, see list below : >> 1. Round button >> 2. ? Textured button > http://blog.chipp.com/new-altbutton-control-for-livecode/ (?) >> 3. ? Recessed button > I have a project on it >> 4. ? Disclosure button > I have a project on it >> 5. ? Gradient button > http://www.buttongadget.com (?) >> 6. ? Rounded rect button > http://blog.chipp.com/new-altbutton-control-for-livecode/ (?) >> 7. ? Rounded textured button > http://blog.chipp.com/new-altbutton-control-for-livecode/ (?) >> 8. ? Help button > I have a project on it >> 9. ? Bevel button > http://blog.chipp.com/new-altbutton-control-for-livecode/ (?) >> 10. ? Radio button > size mini > I work on it >> 11. ? Check box > size mini > I work on it >> 12. ? Stepper > size mini > I work on it >> 13. Date picker >> 14. Segmented control >> 15. ? Search field > http://droptools.sonsothunder.com/all.irev >> 16. Token field >> 17. ? Image Well > http://droptools.sonsothunder.com/all.irev >> 18. Color well >> 19. ? Slider (vertical and horizontal) > size : small and mini > I work on it >> 20. ? Circular slider > I have a project on it >> 21. ? Circular progress indicator > http://revonline2.runrev.com/stack/440/Spinning-wheel >> 22. Vertical and horizontal split view >> 23. ? HUD panel > http://revonline2.runrev.com/stack/574/HUD-Panel-Factory >> 24. Puff of smoke animation >> 25. Answer font >> 26. Toolbar and his items (separator toolbar, print toolbar item, show color toolbar item, show font toolbar item, customize toolbar item, search toolbar item). >> Some have their "custom control"... But not all... Is my list up to date, incomplete or wrong? > > Sorry to be so late to the party, but the OS X HI Guidelines have quite a few more controls than in your list, like action menu butttons, path controls, etc. - you should download the guidelines (or view them online) to fill in the gaps... here's the direct URL to the "Controls" portion of the HIG: > > http://tinyurl.com/2boaz8x > > If you don't have access, let me know and I'll send you a link to the PDF version to download. > > Ken Ray > Sons of Thunder Software, Inc. > Email: kray at sonsothunder.com > Web Site: http://www.sonsothunder.com/ > > _______________________________________________ > use-livecode mailing list > use-livecode at lists.runrev.com > Please visit this url to subscribe, unsubscribe and manage your subscription preferences: > http://lists.runrev.com/mailman/listinfo/use-livecode From m.schonewille at economy-x-talk.com Tue Jul 19 10:19:48 2011 From: m.schonewille at economy-x-talk.com (Mark Schonewille) Date: Tue, 19 Jul 2011 16:19:48 +0200 Subject: ANN: LiveCodeErrors for iPhone In-Reply-To: <4E258EFD.9010103@fourthworld.com> References: <4E258EFD.9010103@fourthworld.com> Message-ID: <215B0FE5-038A-47C7-9F24-B9D29CEDD816@economy-x-talk.com> Richard, Since you are a professional LiveCode programmer, just like me, I know that you know that one may see an error code of the form ###,###,###,xxxx once in a while, where # is a number and x is a letter. Seeing this error isn't necessarily a LiveCode bug. I have no idea why you assume it is. For further questions and comments on this product, you are welcome to contact me off-list. -- Best regards, Mark Schonewille Economy-x-Talk Consulting and Software Engineering Homepage: http://economy-x-talk.com Twitter: http://twitter.com/xtalkprogrammer KvK: 50277553 New: Download the Installer Maker Plugin 1.6 for LiveCode here http://qery.us/ce On 19 jul 2011, at 16:04, Richard Gaskin wrote: > I appreciate your quick reply, but my questions were in earnest: > > Under what circumstances does this happen? > What is the RQCC report for this? > > It would seem more useful to address these with RunRev rather than give everyone using the App Store the impression that LiveCode's error dialog doesn't exist. > > If there's a recipe for reproducing an error which isn't trapped by the error dialog I would imagine the folks at RunRev would want to repair that ASAP. > > If the issue has been reported and remains outstanding, it may be that the recipe provided has been difficult to reproduce. > > If you would kindly provide the recipe or the URL to the report which includes it I would be happy to verify it to help the issue get addressed quickly. > > Thanks in advance for your help on this - From rabit at dimensionB.de Tue Jul 19 11:05:54 2011 From: rabit at dimensionB.de (Ralf Bitter) Date: Tue, 19 Jul 2011 17:05:54 +0200 Subject: [ANN] revIgniter v1.4b Message-ID: revIgniter v1.4b has been released. This is the first version which respects the latest engine changes (LiveCode server version 4.6.3). Lots of adjustments were made and the whole Encrypt library is revised. Keep in mind that this version is not compatible with revServer (engine versions before LiveCode server 4.6.3). Info and download at: http://www.revigniter.com/ Ralf From andre at andregarzia.com Tue Jul 19 11:11:51 2011 From: andre at andregarzia.com (Andre Garzia) Date: Tue, 19 Jul 2011 12:11:51 -0300 Subject: [ANN] revIgniter v1.4b In-Reply-To: References: Message-ID: Ralf, For now I think its better to have two links, one for 1.4 and one for 1.3.latest because people on On-Rev will not transition to the new engine for a while and there are a lot of guys out there using RevServer. I think it is better to offer both branches while the engine is in pre-release. :-) Cheers andre On Tue, Jul 19, 2011 at 12:05 PM, Ralf Bitter wrote: > > revIgniter v1.4b has been released. > > This is the first version which respects the latest > engine changes (LiveCode server version 4.6.3). > Lots of adjustments were made and the whole > Encrypt library is revised. > > Keep in mind that this version is not compatible with > revServer (engine versions before LiveCode server 4.6.3). > > Info and download at: http://www.revigniter.com/ > > > > Ralf > > > _______________________________________________ > use-livecode mailing list > use-livecode at lists.runrev.com > Please visit this url to subscribe, unsubscribe and manage your > subscription preferences: > http://lists.runrev.com/mailman/listinfo/use-livecode > -- http://www.andregarzia.com All We Do Is Code. From ambassador at fourthworld.com Tue Jul 19 11:55:12 2011 From: ambassador at fourthworld.com (Richard Gaskin) Date: Tue, 19 Jul 2011 08:55:12 -0700 Subject: ANN: LiveCodeErrors for iPhone In-Reply-To: <215B0FE5-038A-47C7-9F24-B9D29CEDD816@economy-x-talk.com> References: <215B0FE5-038A-47C7-9F24-B9D29CEDD816@economy-x-talk.com> Message-ID: <4E25A8E0.6090704@fourthworld.com> Mark Schonewille wrote: > Richard, > > Since you are a professional LiveCode programmer, just like me, > I know that you know that one may see an error code of the form > ###,###,###,xxxx once in a while, where # is a number and x is > a letter. Seeing this error isn't necessarily a LiveCode bug. > I have no idea why you assume it is. True, the format you describe is what the engine outputs when errors are encountered, sent as the argument to the errorDialog message. But the IDE's revBackScript includes an errorDialog handler which translates that into a conveniently readable form, also made available for inclusion in standalones in the Standalone Builder. Given that RunRev provides this lookup automatically, no one should ever see the raw error data in either the IDE or a standalone. If they do it means the errorDialog handler either wasn't triggered properly, or is somehow failing itself. So my interest is in identifying a recipe in which an errorDialog message is sent but the info is being displayed in raw form rather than the more readable format LiveCode normally provides. > For further questions and comments on this product, you are welcome > to contact me off-list. My question here was about the perception that LiveCode isn't reporting errors correctly. If I have questions about your product I'll write you off-list. -- Richard Gaskin Fourth World LiveCode training and consulting: http://www.fourthworld.com Webzine for LiveCode developers: http://www.LiveCodeJournal.com LiveCode Journal blog: http://LiveCodejournal.com/blog.irv From andre.bisseret at wanadoo.fr Tue Jul 19 12:23:26 2011 From: andre.bisseret at wanadoo.fr (=?iso-8859-1?Q?Andr=E9_Bisseret?=) Date: Tue, 19 Jul 2011 18:23:26 +0200 Subject: age calculation on Windows vs Mac Message-ID: Bonjour, I am a Mac user but I recently bought a PC (windows 7) in order to test my standalones for windows more quickly. I am fighting for hours now with the following problem : In an app., I have a handler that calculates the age from the short birth date (thanks to Jim Ault who gave it on the list). The birth date is jj/mm/AAAA or jj/mm/AA as well. This handler works fine on Mac, but not on windows. On windows, the age is invariably 100 if the birth year < 1970. (with JJ/MM/AA or JJ/MM/AAAA (as bad ;-( If the birth year > = 1970 then the age is correct. I tried to play with the centuryCutOff but without any success (on windows). I am not familiar with Windows : what am I missing ? Is it a matter of computer setting or should I program differently for windows than for mac !? Thanks a lot in advance for any help Best regards from Grenoble Andr? P.S. By the way, I used to use a search engine (if I well remember, it was in the help menu of livecode) I do not find it any more; has it been suppressed ? (or here again, I am missing something !) From rabit at dimensionB.de Tue Jul 19 12:40:01 2011 From: rabit at dimensionB.de (Ralf Bitter) Date: Tue, 19 Jul 2011 18:40:01 +0200 Subject: [ANN] revIgniter v1.4b In-Reply-To: References: Message-ID: <527C7219-72AE-4552-B8DD-43336F45923C@dimensionB.de> Agreed . . . On 19.07.2011, at 17:11, Andre Garzia wrote: > Ralf, > > For now I think its better to have two links, one for 1.4 and one for > 1.3.latest because people on On-Rev will not transition to the new engine > for a while and there are a lot of guys out there using RevServer. I think > it is better to offer both branches while the engine is in pre-release. > > :-) > > Cheers > andre > From pete at mollysrevenge.com Tue Jul 19 12:40:45 2011 From: pete at mollysrevenge.com (Pete) Date: Tue, 19 Jul 2011 09:40:45 -0700 Subject: age calculation on Windows vs Mac In-Reply-To: References: Message-ID: Hi Andre, I don't have a definitive answer for you but this sounds like it is connected to "epoch" dates which are the number of days since Jan 1, 1970. I'm guessing that somewhere along the line, your dates are being converted to the epoch format on the WIndows platform that would certainly cause a problem for dates prior to 1970. I don't know why that would be different in Windows than OS X. Hope that might help track down the problem. Pete Molly's Revenge On Tue, Jul 19, 2011 at 9:23 AM, Andr? Bisseret wrote: > Bonjour, > > I am a Mac user but I recently bought a PC (windows 7) in order to test my > standalones for windows more quickly. > > I am fighting for hours now with the following problem : > > In an app., I have a handler that calculates the age from the short birth > date (thanks to Jim Ault who gave it on the list). > The birth date is jj/mm/AAAA or jj/mm/AA as well. > > This handler works fine on Mac, but not on windows. > > On windows, the age is invariably 100 if the birth year < 1970. (with > JJ/MM/AA or JJ/MM/AAAA (as bad ;-( > If the birth year > = 1970 then the age is correct. > > I tried to play with the centuryCutOff but without any success (on > windows). > > I am not familiar with Windows : what am I missing ? Is it a matter of > computer setting or should I program differently for windows than for mac !? > > Thanks a lot in advance for any help > > Best regards from Grenoble > > Andr? > > P.S. By the way, I used to use a search engine (if I well remember, it was > in the help menu of livecode) I do not find it any more; has it been > suppressed ? (or here again, I am missing something !) > _______________________________________________ > use-livecode mailing list > use-livecode at lists.runrev.com > Please visit this url to subscribe, unsubscribe and manage your > subscription preferences: > http://lists.runrev.com/mailman/listinfo/use-livecode > > From bvg at mac.com Tue Jul 19 12:43:38 2011 From: bvg at mac.com (=?iso-8859-1?Q?Bj=F6rnke_von_Gierke?=) Date: Tue, 19 Jul 2011 18:43:38 +0200 Subject: ANN: LiveCodeErrors for iPhone In-Reply-To: <4E25A8E0.6090704@fourthworld.com> References: <215B0FE5-038A-47C7-9F24-B9D29CEDD816@economy-x-talk.com> <4E25A8E0.6090704@fourthworld.com> Message-ID: I think one way to fuck up the error dialog is to supress messages when closing stacks by script, and then an error happens. At least this was a case I had about 3 years ago. There migth have been other particular settings or behaviour that triggered it, I don't remember exactly. And Rev error reporting is ass in general, not only when it fails to work as intendet :P On 19 Jul 2011, at 17:55, Richard Gaskin wrote: > Mark Schonewille wrote: > > > Richard, > > > > Since you are a professional LiveCode programmer, just like me, > > I know that you know that one may see an error code of the form > > ###,###,###,xxxx once in a while, where # is a number and x is > > a letter. Seeing this error isn't necessarily a LiveCode bug. > > I have no idea why you assume it is. > > True, the format you describe is what the engine outputs when errors are encountered, sent as the argument to the errorDialog message. > > But the IDE's revBackScript includes an errorDialog handler which translates that into a conveniently readable form, also made available for inclusion in standalones in the Standalone Builder. > > Given that RunRev provides this lookup automatically, no one should ever see the raw error data in either the IDE or a standalone. > > If they do it means the errorDialog handler either wasn't triggered properly, or is somehow failing itself. > > So my interest is in identifying a recipe in which an errorDialog message is sent but the info is being displayed in raw form rather than the more readable format LiveCode normally provides. > > > > For further questions and comments on this product, you are welcome > > to contact me off-list. > > My question here was about the perception that LiveCode isn't reporting errors correctly. If I have questions about your product I'll write you off-list. > > -- > Richard Gaskin > Fourth World > LiveCode training and consulting: http://www.fourthworld.com > Webzine for LiveCode developers: http://www.LiveCodeJournal.com > LiveCode Journal blog: http://LiveCodejournal.com/blog.irv > > _______________________________________________ > use-livecode mailing list > use-livecode at lists.runrev.com > Please visit this url to subscribe, unsubscribe and manage your subscription preferences: > http://lists.runrev.com/mailman/listinfo/use-livecode From andrew at rjdfarm.com Tue Jul 19 12:50:09 2011 From: andrew at rjdfarm.com (Andrew Kluthe) Date: Tue, 19 Jul 2011 09:50:09 -0700 (PDT) Subject: [ANN] revIgniter v1.4b In-Reply-To: <527C7219-72AE-4552-B8DD-43336F45923C@dimensionB.de> References: <527C7219-72AE-4552-B8DD-43336F45923C@dimensionB.de> Message-ID: <1311094209355-3678624.post@n4.nabble.com> RevServer got updated? Is there a changelog? Does On-Rev use Livecode Server 4.6.3? -- View this message in context: http://runtime-revolution.278305.n4.nabble.com/ANN-revIgniter-v1-4b-tp3678359p3678624.html Sent from the Revolution - User mailing list archive at Nabble.com. From andre at andregarzia.com Tue Jul 19 12:51:56 2011 From: andre at andregarzia.com (Andre Garzia) Date: Tue, 19 Jul 2011 13:51:56 -0300 Subject: ANN: LiveCodeErrors for iPhone In-Reply-To: References: <215B0FE5-038A-47C7-9F24-B9D29CEDD816@economy-x-talk.com> <4E25A8E0.6090704@fourthworld.com> Message-ID: Using try/catch blocks, fiddling with the exception and throwing it again is a guaranteed form to destroy the errordialog window. 2011/7/19 Bj?rnke von Gierke > I think one way to fuck up the error dialog is to supress messages when > closing stacks by script, and then an error happens. At least this was a > case I had about 3 years ago. There migth have been other particular > settings or behaviour that triggered it, I don't remember exactly. > > And Rev error reporting is ass in general, not only when it fails to work > as intendet :P > > On 19 Jul 2011, at 17:55, Richard Gaskin wrote: > > > Mark Schonewille wrote: > > > > > Richard, > > > > > > Since you are a professional LiveCode programmer, just like me, > > > I know that you know that one may see an error code of the form > > > ###,###,###,xxxx once in a while, where # is a number and x is > > > a letter. Seeing this error isn't necessarily a LiveCode bug. > > > I have no idea why you assume it is. > > > > True, the format you describe is what the engine outputs when errors are > encountered, sent as the argument to the errorDialog message. > > > > But the IDE's revBackScript includes an errorDialog handler which > translates that into a conveniently readable form, also made available for > inclusion in standalones in the Standalone Builder. > > > > Given that RunRev provides this lookup automatically, no one should ever > see the raw error data in either the IDE or a standalone. > > > > If they do it means the errorDialog handler either wasn't triggered > properly, or is somehow failing itself. > > > > So my interest is in identifying a recipe in which an errorDialog message > is sent but the info is being displayed in raw form rather than the more > readable format LiveCode normally provides. > > > > > > > For further questions and comments on this product, you are welcome > > > to contact me off-list. > > > > My question here was about the perception that LiveCode isn't reporting > errors correctly. If I have questions about your product I'll write you > off-list. > > > > -- > > Richard Gaskin > > Fourth World > > LiveCode training and consulting: http://www.fourthworld.com > > Webzine for LiveCode developers: http://www.LiveCodeJournal.com > > LiveCode Journal blog: http://LiveCodejournal.com/blog.irv > > > > _______________________________________________ > > use-livecode mailing list > > use-livecode at lists.runrev.com > > Please visit this url to subscribe, unsubscribe and manage your > subscription preferences: > > http://lists.runrev.com/mailman/listinfo/use-livecode > > > _______________________________________________ > use-livecode mailing list > use-livecode at lists.runrev.com > Please visit this url to subscribe, unsubscribe and manage your > subscription preferences: > http://lists.runrev.com/mailman/listinfo/use-livecode > -- http://www.andregarzia.com All We Do Is Code. From chipp at chipp.com Tue Jul 19 12:58:45 2011 From: chipp at chipp.com (Chipp Walters) Date: Tue, 19 Jul 2011 11:58:45 -0500 Subject: ANN: LiveCodeErrors for iPhone In-Reply-To: References: <215B0FE5-038A-47C7-9F24-B9D29CEDD816@economy-x-talk.com> <4E25A8E0.6090704@fourthworld.com> Message-ID: Interesting. I have to say, in the past 10 years or so since I've been programming commercial applications for my clients and our company, I do not recall seeing such an error. In fact, this is the first time I've even heard of them. I, too, would like to know under what circumstances they are fired. It seems like a bug to me in LC's error handling code if they continually pop up. I suspect Richard knew about this from all his work with the MetaCard IDE. 2011/7/19 Bj?rnke von Gierke > I think one way to fuck up the error dialog is to supress messages when > closing stacks by script, and then an error happens. At least this was a > case I had about 3 years ago. There migth have been other particular > settings or behaviour that triggered it, I don't remember exactly. > > And Rev error reporting is ass in general, not only when it fails to work > as intendet :P > > On 19 Jul 2011, at 17:55, Richard Gaskin wrote: > > > Mark Schonewille wrote: > > > > > Richard, > > > > > > Since you are a professional LiveCode programmer, just like me, > > > I know that you know that one may see an error code of the form > > > ###,###,###,xxxx once in a while, where # is a number and x is > > > a letter. Seeing this error isn't necessarily a LiveCode bug. > > > I have no idea why you assume it is. > > > > True, the format you describe is what the engine outputs when errors are > encountered, sent as the argument to the errorDialog message. > > > > But the IDE's revBackScript includes an errorDialog handler which > translates that into a conveniently readable form, also made available for > inclusion in standalones in the Standalone Builder. > > > > Given that RunRev provides this lookup automatically, no one should ever > see the raw error data in either the IDE or a standalone. > > > > If they do it means the errorDialog handler either wasn't triggered > properly, or is somehow failing itself. > > > > So my interest is in identifying a recipe in which an errorDialog message > is sent but the info is being displayed in raw form rather than the more > readable format LiveCode normally provides. > > > > > > > For further questions and comments on this product, you are welcome > > > to contact me off-list. > > > > My question here was about the perception that LiveCode isn't reporting > errors correctly. If I have questions about your product I'll write you > off-list. > > > > -- > > Richard Gaskin > > Fourth World > > LiveCode training and consulting: http://www.fourthworld.com > > Webzine for LiveCode developers: http://www.LiveCodeJournal.com > > LiveCode Journal blog: http://LiveCodejournal.com/blog.irv > > > > _______________________________________________ > > use-livecode mailing list > > use-livecode at lists.runrev.com > > Please visit this url to subscribe, unsubscribe and manage your > subscription preferences: > > http://lists.runrev.com/mailman/listinfo/use-livecode > > > _______________________________________________ > use-livecode mailing list > use-livecode at lists.runrev.com > Please visit this url to subscribe, unsubscribe and manage your > subscription preferences: > http://lists.runrev.com/mailman/listinfo/use-livecode > -- Chipp Walters CEO, Shafer Walters Group, Inc. From andre at andregarzia.com Tue Jul 19 13:00:21 2011 From: andre at andregarzia.com (Andre Garzia) Date: Tue, 19 Jul 2011 14:00:21 -0300 Subject: [ANN] revIgniter v1.4b In-Reply-To: <1311094209355-3678624.post@n4.nabble.com> References: <527C7219-72AE-4552-B8DD-43336F45923C@dimensionB.de> <1311094209355-3678624.post@n4.nabble.com> Message-ID: Andrew, I just received an email from the mothership, it is more serious than that, RevServer was released!!!!! \O/ LiveCode Server 4.6.3 On Tue, Jul 19, 2011 at 1:50 PM, Andrew Kluthe wrote: > RevServer got updated? Is there a changelog? Does On-Rev use Livecode > Server > 4.6.3? > > -- > View this message in context: > http://runtime-revolution.278305.n4.nabble.com/ANN-revIgniter-v1-4b-tp3678359p3678624.html > Sent from the Revolution - User mailing list archive at Nabble.com. > > _______________________________________________ > use-livecode mailing list > use-livecode at lists.runrev.com > Please visit this url to subscribe, unsubscribe and manage your > subscription preferences: > http://lists.runrev.com/mailman/listinfo/use-livecode > -- http://www.andregarzia.com All We Do Is Code. From andre.bisseret at wanadoo.fr Tue Jul 19 13:06:22 2011 From: andre.bisseret at wanadoo.fr (=?iso-8859-1?Q?Andr=E9_Bisseret?=) Date: Tue, 19 Jul 2011 19:06:22 +0200 Subject: age calculation on Windows vs Mac In-Reply-To: References: Message-ID: <9C1D5443-02BA-4C2E-ABD3-AC20EA2D8C12@wanadoo.fr> Thanks a lot Pete for this information; i was not aware of these "epoch" dates format I am going to dig around that I much appreciate your prompt help Andr? Le 19 juil. 2011 ? 18:40, Pete a ?crit : > Hi Andre, > I don't have a definitive answer for you but this sounds like it is > connected to "epoch" dates which are the number of days since Jan 1, 1970. > I'm guessing that somewhere along the line, your dates are being converted > to the epoch format on the WIndows platform that would certainly cause a > problem for dates prior to 1970. I don't know why that would be different > in Windows than OS X. > > Hope that might help track down the problem. > > Pete > Molly's Revenge > > > > > On Tue, Jul 19, 2011 at 9:23 AM, Andr? Bisseret > wrote: > >> Bonjour, >> >> I am a Mac user but I recently bought a PC (windows 7) in order to test my >> standalones for windows more quickly. >> >> I am fighting for hours now with the following problem : >> >> In an app., I have a handler that calculates the age from the short birth >> date (thanks to Jim Ault who gave it on the list). >> The birth date is jj/mm/AAAA or jj/mm/AA as well. >> >> This handler works fine on Mac, but not on windows. >> >> On windows, the age is invariably 100 if the birth year < 1970. (with >> JJ/MM/AA or JJ/MM/AAAA (as bad ;-( >> If the birth year > = 1970 then the age is correct. >> >> I tried to play with the centuryCutOff but without any success (on >> windows). >> >> I am not familiar with Windows : what am I missing ? Is it a matter of >> computer setting or should I program differently for windows than for mac !? >> >> Thanks a lot in advance for any help >> >> Best regards from Grenoble >> >> Andr? >> >> P.S. By the way, I used to use a search engine (if I well remember, it was >> in the help menu of livecode) I do not find it any more; has it been >> suppressed ? (or here again, I am missing something !) >> _______________________________________________ >> use-livecode mailing list >> use-livecode at lists.runrev.com >> Please visit this url to subscribe, unsubscribe and manage your >> subscription preferences: >> http://lists.runrev.com/mailman/listinfo/use-livecode >> >> > _______________________________________________ > use-livecode mailing list > use-livecode at lists.runrev.com > Please visit this url to subscribe, unsubscribe and manage your subscription preferences: > http://lists.runrev.com/mailman/listinfo/use-livecode From pete at mollysrevenge.com Tue Jul 19 13:07:47 2011 From: pete at mollysrevenge.com (Pete) Date: Tue, 19 Jul 2011 10:07:47 -0700 Subject: Mac OS X missing controls in the LiveCode IDE In-Reply-To: <4A0BC648-2559-4EE2-88B8-13F369E16F52@sonsothunder.com> References: <4A0BC648-2559-4EE2-88B8-13F369E16F52@sonsothunder.com> Message-ID: Ken, Thanks for that link - very interesting. A lot of the details in there are, as you say, to do with controls that we don't have available to us natively in Livecode but there's a couple of other issues. The document talks about "window-frame" controls, which include various controls that you can place in the "toolbar" which, to the user, appears to be part of the window title bar. Is there a way to do that in LC? The only things I can find are ways set the decorations and the title of the title bar. The document also mentions that the window frame include a bottom-bar and once again, I can't find any mention of how to implement that in the docs. I can imagine that both of these could be implemented with some combination of other LC controls but am I right that there's no LC native way to implement them? Pete Molly's Revenge > Sorry to be so late to the party, but the OS X HI Guidelines have quite a > few more controls than in your list, like action menu butttons, path > controls, etc. - you should download the guidelines (or view them online) to > fill in the gaps... here's the direct URL to the "Controls" portion of the > HIG: > > http://tinyurl.com/2boaz8x > > If you don't have access, let me know and I'll send you a link to the PDF > version to download. > > Ken Ray > Sons of Thunder Software, Inc. > Email: kray at sonsothunder.com > Web Site: http://www.sonsothunder.com/ > > _______________________________________________ > use-livecode mailing list > use-livecode at lists.runrev.com > Please visit this url to subscribe, unsubscribe and manage your > subscription preferences: > http://lists.runrev.com/mailman/listinfo/use-livecode From andrew at rjdfarm.com Tue Jul 19 13:15:48 2011 From: andrew at rjdfarm.com (Andrew Kluthe) Date: Tue, 19 Jul 2011 10:15:48 -0700 (PDT) Subject: [ANN] revIgniter v1.4b In-Reply-To: References: <527C7219-72AE-4552-B8DD-43336F45923C@dimensionB.de> <1311094209355-3678624.post@n4.nabble.com> Message-ID: <1311095748965-3678689.post@n4.nabble.com> Whoa. I just got the email too. I am excited about built in encryption feature. This makes webservice apps much easier. I wonder if this means SQL Yoga will work with revServer. I am a little confused on how the stack support works though. Can someone help summarize this feature for me? -- View this message in context: http://runtime-revolution.278305.n4.nabble.com/ANN-revIgniter-v1-4b-tp3678359p3678689.html Sent from the Revolution - User mailing list archive at Nabble.com. From andre at andregarzia.com Tue Jul 19 13:31:07 2011 From: andre at andregarzia.com (Andre Garzia) Date: Tue, 19 Jul 2011 14:31:07 -0300 Subject: [ANN] revIgniter v1.4b In-Reply-To: <1311095748965-3678689.post@n4.nabble.com> References: <527C7219-72AE-4552-B8DD-43336F45923C@dimensionB.de> <1311094209355-3678624.post@n4.nabble.com> <1311095748965-3678689.post@n4.nabble.com> Message-ID: On Tue, Jul 19, 2011 at 2:15 PM, Andrew Kluthe wrote: > Whoa. I just got the email too. > > > I am excited about built in encryption feature. This makes webservice apps > much easier. I wonder if this means SQL Yoga will work with revServer. > > I am a little confused on how the stack support works though. Can someone > help summarize this feature for me? > Andrew, the stack works just like library stacks. It is just a convenience, you can start using a stack and have its script placed in the message path. No GUI stuff. :-) > > -- > View this message in context: > http://runtime-revolution.278305.n4.nabble.com/ANN-revIgniter-v1-4b-tp3678359p3678689.html > Sent from the Revolution - User mailing list archive at Nabble.com. > > _______________________________________________ > use-livecode mailing list > use-livecode at lists.runrev.com > Please visit this url to subscribe, unsubscribe and manage your > subscription preferences: > http://lists.runrev.com/mailman/listinfo/use-livecode > -- http://www.andregarzia.com All We Do Is Code. From stephenREVOLUTION2 at barncard.com Tue Jul 19 13:35:06 2011 From: stephenREVOLUTION2 at barncard.com (stephen barncard) Date: Tue, 19 Jul 2011 10:35:06 -0700 Subject: [ANN] revIgniter v1.4b In-Reply-To: References: <527C7219-72AE-4552-B8DD-43336F45923C@dimensionB.de> <1311094209355-3678624.post@n4.nabble.com> <1311095748965-3678689.post@n4.nabble.com> Message-ID: and CUSTOM PROPERTIES! On 19 July 2011 10:31, Andre Garzia wrote: > On Tue, Jul 19, 2011 at 2:15 PM, Andrew Kluthe wrote: > > > Whoa. I just got the email too. > > > > > > I am excited about built in encryption feature. This makes webservice > apps > > much easier. I wonder if this means SQL Yoga will work with revServer. > > > > I am a little confused on how the stack support works though. Can someone > > help summarize this feature for me? > > > > Andrew, > > the stack works just like library stacks. It is just a convenience, you can > start using a stack and have its script placed in the message path. No GUI > stuff. > > :-) > > > > > > > -- > > View this message in context: > > > http://runtime-revolution.278305.n4.nabble.com/ANN-revIgniter-v1-4b-tp3678359p3678689.html > > Sent from the Revolution - User mailing list archive at Nabble.com. > > > > _______________________________________________ > > use-livecode mailing list > > use-livecode at lists.runrev.com > > Please visit this url to subscribe, unsubscribe and manage your > > subscription preferences: > > http://lists.runrev.com/mailman/listinfo/use-livecode > > > > > > -- > http://www.andregarzia.com All We Do Is Code. > _______________________________________________ > use-livecode mailing list > use-livecode at lists.runrev.com > Please visit this url to subscribe, unsubscribe and manage your > subscription preferences: > http://lists.runrev.com/mailman/listinfo/use-livecode > -- Stephen Barncard San Francisco Ca. USA more about sqb From cszasz at mac.com Tue Jul 19 13:35:59 2011 From: cszasz at mac.com (Charles Szasz) Date: Tue, 19 Jul 2011 13:35:59 -0400 Subject: Output error in standalone Message-ID: <3D74C673-C31E-4462-A2C2-5A1F7253CC13@mac.com> I am using Rev 4.0 Build 950. I just finishing working on a project. It consists of a splash stack and an app stack. I set up my Standalone Applications settings to create builds for Mac OS X (Universal) and Windows. When I tried to compile my app I got the following error message: There was was an error while saving the standalone application. Build failed for MacOSX: could not open output file. When I unchecked making a Mac OS X Universal and just made a windows build, I did not get this error and the windows app was created. I have not yet tried the windows build to see if it is okay. I have never gotten this error message before when compiling. Anybody have any suggestions on how I can correct this problem? Thanks for your time! Charles Szasz cszasz at mac.com From andrew at rjdfarm.com Tue Jul 19 13:40:03 2011 From: andrew at rjdfarm.com (Andrew Kluthe) Date: Tue, 19 Jul 2011 10:40:03 -0700 (PDT) Subject: [ANN] revIgniter v1.4b In-Reply-To: References: <527C7219-72AE-4552-B8DD-43336F45923C@dimensionB.de> <1311094209355-3678624.post@n4.nabble.com> <1311095748965-3678689.post@n4.nabble.com> Message-ID: <1311097203523-3678755.post@n4.nabble.com> I don't need GUI stuff if I can send messages and write my code in a colorized code editor instead of notepad++. Very excited. -- View this message in context: http://runtime-revolution.278305.n4.nabble.com/ANN-revIgniter-v1-4b-tp3678359p3678755.html Sent from the Revolution - User mailing list archive at Nabble.com. From stephenREVOLUTION2 at barncard.com Tue Jul 19 13:44:12 2011 From: stephenREVOLUTION2 at barncard.com (stephen barncard) Date: Tue, 19 Jul 2011 10:44:12 -0700 Subject: [ANN] revIgniter v1.4b In-Reply-To: <1311097203523-3678755.post@n4.nabble.com> References: <527C7219-72AE-4552-B8DD-43336F45923C@dimensionB.de> <1311094209355-3678624.post@n4.nabble.com> <1311095748965-3678689.post@n4.nabble.com> <1311097203523-3678755.post@n4.nabble.com> Message-ID: ?? there is no editor in Livecode server... On 19 July 2011 10:40, Andrew Kluthe wrote: > I don't need GUI stuff if I can send messages and write my code in a > colorized code editor instead of notepad++. Very excited. > > -- > Stephen Barncard San Francisco Ca. USA more about sqb From bobs at twft.com Tue Jul 19 13:46:53 2011 From: bobs at twft.com (Bob Sneidar) Date: Tue, 19 Jul 2011 10:46:53 -0700 Subject: [ANN] revIgniter v1.4b In-Reply-To: References: <527C7219-72AE-4552-B8DD-43336F45923C@dimensionB.de> <1311094209355-3678624.post@n4.nabble.com> Message-ID: Riiiiight... and aliens from Zartha just landed in my mother's living room proclaiming to be the ancient ancestors of Elvis Presley, wondering why their whale songs haven't been answered. Bob On Jul 19, 2011, at 10:00 AM, Andre Garzia wrote: > Andrew, > > I just received an email from the mothership, it is more serious than that, > RevServer was released!!!!! > > \O/ From andre at andregarzia.com Tue Jul 19 13:49:04 2011 From: andre at andregarzia.com (Andre Garzia) Date: Tue, 19 Jul 2011 14:49:04 -0300 Subject: [ANN] revIgniter v1.4b In-Reply-To: <1311097203523-3678755.post@n4.nabble.com> References: <527C7219-72AE-4552-B8DD-43336F45923C@dimensionB.de> <1311094209355-3678624.post@n4.nabble.com> <1311095748965-3678689.post@n4.nabble.com> <1311097203523-3678755.post@n4.nabble.com> Message-ID: LiveCode IDE can't handle LiveCode Server scripts... :-( On Tue, Jul 19, 2011 at 2:40 PM, Andrew Kluthe wrote: > I don't need GUI stuff if I can send messages and write my code in a > colorized code editor instead of notepad++. Very excited. > > -- > View this message in context: > http://runtime-revolution.278305.n4.nabble.com/ANN-revIgniter-v1-4b-tp3678359p3678755.html > Sent from the Revolution - User mailing list archive at Nabble.com. > > _______________________________________________ > use-livecode mailing list > use-livecode at lists.runrev.com > Please visit this url to subscribe, unsubscribe and manage your > subscription preferences: > http://lists.runrev.com/mailman/listinfo/use-livecode > -- http://www.andregarzia.com All We Do Is Code. From shaosean at wehostmacs.com Tue Jul 19 14:05:23 2011 From: shaosean at wehostmacs.com (Shao Sean) Date: Tue, 19 Jul 2011 14:05:23 -0400 Subject: Mac OS X missing controls in the LiveCode IDE Message-ID: <52F8A9C6-5A7A-4160-90AB-22B438238E45@wehostmacs.com> > The document talks about "window-frame" controls, which include > various controls that you can place in the "toolbar" which, to the > user, appears to be part of the window title bar. Is there a way to > do that in LC? Nothing in Rev to allow you to do it natively and I have found some code that would make features like this dead easy to do in Rev except for the issue of the stack's/card's "background" not allowing the system to draw in the content area of the window - we can easily draw native controls in the content area and have them visible, we cannot get the changes to the window frame to be visible (the changes do take effect though).. I currently am having a game of ping-pong with support in regards to this issue.. From andrew at rjdfarm.com Tue Jul 19 14:18:21 2011 From: andrew at rjdfarm.com (Andrew Kluthe) Date: Tue, 19 Jul 2011 11:18:21 -0700 (PDT) Subject: [ANN] revIgniter v1.4b In-Reply-To: References: <527C7219-72AE-4552-B8DD-43336F45923C@dimensionB.de> <1311094209355-3678624.post@n4.nabble.com> <1311095748965-3678689.post@n4.nabble.com> <1311097203523-3678755.post@n4.nabble.com> Message-ID: <1311099501844-3678875.post@n4.nabble.com> Bummer. So we aren't talking about binary stacks, just a different way of interacting between script files? How do custom properties work then? -- View this message in context: http://runtime-revolution.278305.n4.nabble.com/ANN-revIgniter-v1-4b-tp3678359p3678875.html Sent from the Revolution - User mailing list archive at Nabble.com. From stephenREVOLUTION2 at barncard.com Tue Jul 19 14:20:46 2011 From: stephenREVOLUTION2 at barncard.com (stephen barncard) Date: Tue, 19 Jul 2011 11:20:46 -0700 Subject: [ANN] revIgniter v1.4b In-Reply-To: References: <527C7219-72AE-4552-B8DD-43336F45923C@dimensionB.de> <1311094209355-3678624.post@n4.nabble.com> <1311095748965-3678689.post@n4.nabble.com> <1311097203523-3678755.post@n4.nabble.com> Message-ID: Textmate and Ralf Bitter's free Livecode plugin for Textmate rocks for live iRev editing. sqb On 19 July 2011 10:49, Andre Garzia wrote: > LiveCode IDE can't handle LiveCode Server scripts... :-( > > On Tue, Jul 19, 2011 at 2:40 PM, Andrew Kluthe wrote: > > > I don't need GUI stuff if I can send messages and write my code in a > > colorized code editor instead of notepad++. Very excited. > > > > -- > > View this message in context: > > > http://runtime-revolution.278305.n4.nabble.com/ANN-revIgniter-v1-4b-tp3678359p3678755.html > > Sent from the Revolution - User mailing list archive at Nabble.com. > > > > _______________________________________________ > > use-livecode mailing list > > use-livecode at lists.runrev.com > > Please visit this url to subscribe, unsubscribe and manage your > > subscription preferences: > > http://lists.runrev.com/mailman/listinfo/use-livecode > > > > > > -- > http://www.andregarzia.com All We Do Is Code. > _______________________________________________ > use-livecode mailing list > use-livecode at lists.runrev.com > Please visit this url to subscribe, unsubscribe and manage your > subscription preferences: > http://lists.runrev.com/mailman/listinfo/use-livecode > -- Stephen Barncard San Francisco Ca. USA more about sqb From stephenREVOLUTION2 at barncard.com Tue Jul 19 14:22:11 2011 From: stephenREVOLUTION2 at barncard.com (stephen barncard) Date: Tue, 19 Jul 2011 11:22:11 -0700 Subject: [ANN] revIgniter v1.4b In-Reply-To: <1311099501844-3678875.post@n4.nabble.com> References: <527C7219-72AE-4552-B8DD-43336F45923C@dimensionB.de> <1311094209355-3678624.post@n4.nabble.com> <1311095748965-3678689.post@n4.nabble.com> <1311097203523-3678755.post@n4.nabble.com> <1311099501844-3678875.post@n4.nabble.com> Message-ID: maybe I'm wrong but there should be at least custom properties of the stack. I am not able to test drive it yet. sqb On 19 July 2011 11:18, Andrew Kluthe wrote: > Bummer. So we aren't talking about binary stacks, just a different way of > interacting between script files? How do custom properties work then? > > -- > View this message in context: > http://runtime-revolution.278305.n4.nabble.com/ANN-revIgniter-v1-4b-tp3678359p3678875.html > Sent from the Revolution - User mailing list archive at Nabble.com. > > _______________________________________________ > use-livecode mailing list > use-livecode at lists.runrev.com > Please visit this url to subscribe, unsubscribe and manage your > subscription preferences: > http://lists.runrev.com/mailman/listinfo/use-livecode > -- Stephen Barncard San Francisco Ca. USA more about sqb From keith.clarke at clarkeandclarke.co.uk Tue Jul 19 14:24:37 2011 From: keith.clarke at clarkeandclarke.co.uk (Keith Clarke) Date: Tue, 19 Jul 2011 19:24:37 +0100 Subject: [ANN] revIgniter v1.4b In-Reply-To: References: <527C7219-72AE-4552-B8DD-43336F45923C@dimensionB.de> <1311094209355-3678624.post@n4.nabble.com> <1311095748965-3678689.post@n4.nabble.com> <1311097203523-3678755.post@n4.nabble.com> Message-ID: <2C47327A-A1E8-43AA-8145-4CFEA1D27EC5@clarkeandclarke.co.uk> ...that makes me so pleased that I recently bought Coda ;-) On 19 Jul 2011, at 19:20, stephen barncard wrote: > Textmate and Ralf Bitter's free Livecode plugin for Textmate rocks for live > iRev editing. > > sqb > > On 19 July 2011 10:49, Andre Garzia wrote: > >> LiveCode IDE can't handle LiveCode Server scripts... :-( >> >> On Tue, Jul 19, 2011 at 2:40 PM, Andrew Kluthe wrote: >> >>> I don't need GUI stuff if I can send messages and write my code in a >>> colorized code editor instead of notepad++. Very excited. >>> >>> -- >>> View this message in context: >>> >> http://runtime-revolution.278305.n4.nabble.com/ANN-revIgniter-v1-4b-tp3678359p3678755.html >>> Sent from the Revolution - User mailing list archive at Nabble.com. From andre at andregarzia.com Tue Jul 19 14:25:28 2011 From: andre at andregarzia.com (Andre Garzia) Date: Tue, 19 Jul 2011 15:25:28 -0300 Subject: [ANN] revIgniter v1.4b In-Reply-To: <1311099501844-3678875.post@n4.nabble.com> References: <527C7219-72AE-4552-B8DD-43336F45923C@dimensionB.de> <1311094209355-3678624.post@n4.nabble.com> <1311095748965-3678689.post@n4.nabble.com> <1311097203523-3678755.post@n4.nabble.com> <1311099501844-3678875.post@n4.nabble.com> Message-ID: Andrew, it is binary stacks, you use stack libraries just like you do on the IDE but you can't access the graphics layer. Andre On Tue, Jul 19, 2011 at 3:18 PM, Andrew Kluthe wrote: > Bummer. So we aren't talking about binary stacks, just a different way of > interacting between script files? How do custom properties work then? > > -- > View this message in context: > http://runtime-revolution.278305.n4.nabble.com/ANN-revIgniter-v1-4b-tp3678359p3678875.html > Sent from the Revolution - User mailing list archive at Nabble.com. > > _______________________________________________ > use-livecode mailing list > use-livecode at lists.runrev.com > Please visit this url to subscribe, unsubscribe and manage your > subscription preferences: > http://lists.runrev.com/mailman/listinfo/use-livecode > -- http://www.andregarzia.com All We Do Is Code. From andre at andregarzia.com Tue Jul 19 14:25:53 2011 From: andre at andregarzia.com (Andre Garzia) Date: Tue, 19 Jul 2011 15:25:53 -0300 Subject: [ANN] revIgniter v1.4b In-Reply-To: References: <527C7219-72AE-4552-B8DD-43336F45923C@dimensionB.de> <1311094209355-3678624.post@n4.nabble.com> <1311095748965-3678689.post@n4.nabble.com> <1311097203523-3678755.post@n4.nabble.com> <1311099501844-3678875.post@n4.nabble.com> Message-ID: custom props should work fine On Tue, Jul 19, 2011 at 3:22 PM, stephen barncard < stephenREVOLUTION2 at barncard.com> wrote: > maybe I'm wrong but there should be at least custom properties of the > stack. > > I am not able to test drive it yet. > > sqb > > On 19 July 2011 11:18, Andrew Kluthe wrote: > > > Bummer. So we aren't talking about binary stacks, just a different way of > > interacting between script files? How do custom properties work then? > > > > -- > > View this message in context: > > > http://runtime-revolution.278305.n4.nabble.com/ANN-revIgniter-v1-4b-tp3678359p3678875.html > > Sent from the Revolution - User mailing list archive at Nabble.com. > > > > _______________________________________________ > > use-livecode mailing list > > use-livecode at lists.runrev.com > > Please visit this url to subscribe, unsubscribe and manage your > > subscription preferences: > > http://lists.runrev.com/mailman/listinfo/use-livecode > > > > > > -- > > > > Stephen Barncard > San Francisco Ca. USA > > more about sqb > _______________________________________________ > use-livecode mailing list > use-livecode at lists.runrev.com > Please visit this url to subscribe, unsubscribe and manage your > subscription preferences: > http://lists.runrev.com/mailman/listinfo/use-livecode > -- http://www.andregarzia.com All We Do Is Code. From andre at andregarzia.com Tue Jul 19 14:34:51 2011 From: andre at andregarzia.com (Andre Garzia) Date: Tue, 19 Jul 2011 15:34:51 -0300 Subject: [ANN] revIgniter v1.4b In-Reply-To: <2C47327A-A1E8-43AA-8145-4CFEA1D27EC5@clarkeandclarke.co.uk> References: <527C7219-72AE-4552-B8DD-43336F45923C@dimensionB.de> <1311094209355-3678624.post@n4.nabble.com> <1311095748965-3678689.post@n4.nabble.com> <1311097203523-3678755.post@n4.nabble.com> <2C47327A-A1E8-43AA-8145-4CFEA1D27EC5@clarkeandclarke.co.uk> Message-ID: Coda is a great product by one of the greatest if not the greatest software house (that publishes only for the mac). Audiom was my default music player since Mac OS 9, now I've migrated to Grooveshark. To edit my own LiveCode Server stuff I use Transmit + TextMate or my own tools sometimes... FTP Commander is a quick editor for irev files but I need to fix some stuff in it before it can be used for more serious work. On Tue, Jul 19, 2011 at 3:24 PM, Keith Clarke < keith.clarke at clarkeandclarke.co.uk> wrote: > ...that makes me so pleased that I recently bought Coda ;-) > > On 19 Jul 2011, at 19:20, stephen barncard wrote: > > > Textmate and Ralf Bitter's free Livecode plugin for Textmate rocks for > live > > iRev editing. > > > > sqb > > > > On 19 July 2011 10:49, Andre Garzia wrote: > > > >> LiveCode IDE can't handle LiveCode Server scripts... :-( > >> > >> On Tue, Jul 19, 2011 at 2:40 PM, Andrew Kluthe > wrote: > >> > >>> I don't need GUI stuff if I can send messages and write my code in a > >>> colorized code editor instead of notepad++. Very excited. > >>> > >>> -- > >>> View this message in context: > >>> > >> > http://runtime-revolution.278305.n4.nabble.com/ANN-revIgniter-v1-4b-tp3678359p3678755.html > >>> Sent from the Revolution - User mailing list archive at Nabble.com. > > > _______________________________________________ > use-livecode mailing list > use-livecode at lists.runrev.com > Please visit this url to subscribe, unsubscribe and manage your > subscription preferences: > http://lists.runrev.com/mailman/listinfo/use-livecode > -- http://www.andregarzia.com All We Do Is Code. From stephenREVOLUTION2 at barncard.com Tue Jul 19 14:36:25 2011 From: stephenREVOLUTION2 at barncard.com (stephen barncard) Date: Tue, 19 Jul 2011 11:36:25 -0700 Subject: [ANN] revIgniter v1.4b In-Reply-To: <2C47327A-A1E8-43AA-8145-4CFEA1D27EC5@clarkeandclarke.co.uk> References: <527C7219-72AE-4552-B8DD-43336F45923C@dimensionB.de> <1311094209355-3678624.post@n4.nabble.com> <1311095748965-3678689.post@n4.nabble.com> <1311097203523-3678755.post@n4.nabble.com> <2C47327A-A1E8-43AA-8145-4CFEA1D27EC5@clarkeandclarke.co.uk> Message-ID: Coda is a great editor too - and does CSS nicely. I made a SubEthaEdit editor plugin that works in Coda. It should be on their website. It works ok - (I modified a template for Director!) -- but it could use some improvement. Runtime Revolution (includes style) feel free to improve. http://www.codingmonkeys.de/subethaedit/mode.html On 19 July 2011 11:24, Keith Clarke wrote: > ...that makes me so pleased that I recently bought Coda ;-) > > On 19 Jul 2011, at 19:20, stephen barncard wrote: > > > Textmate and Ralf Bitter's free Livecode plugin for Textmate rocks for > live > > iRev editing. > > > > sqb > > > > On 19 July 2011 10:49, Andre Garzia wrote: > > > >> LiveCode IDE can't handle LiveCode Server scripts... :-( > >> > >> On Tue, Jul 19, 2011 at 2:40 PM, Andrew Kluthe > wrote: > >> > >>> I don't need GUI stuff if I can send messages and write my code in a > >>> colorized code editor instead of notepad++. Very excited. > >>> > >>> -- > >>> View this message in context: > >>> > >> > http://runtime-revolution.278305.n4.nabble.com/ANN-revIgniter-v1-4b-tp3678359p3678755.html > >>> Sent from the Revolution - User mailing list archive at Nabble.com. > > > _______________________________________________ > use-livecode mailing list > use-livecode at lists.runrev.com > Please visit this url to subscribe, unsubscribe and manage your > subscription preferences: > http://lists.runrev.com/mailman/listinfo/use-livecode > -- Stephen Barncard San Francisco Ca. USA more about sqb From rman at free.fr Tue Jul 19 14:59:21 2011 From: rman at free.fr (Robert Mann) Date: Tue, 19 Jul 2011 11:59:21 -0700 (PDT) Subject: [ANN] revIgniter v1.4b In-Reply-To: References: <527C7219-72AE-4552-B8DD-43336F45923C@dimensionB.de> <1311094209355-3678624.post@n4.nabble.com> <1311095748965-3678689.post@n4.nabble.com> <1311097203523-3678755.post@n4.nabble.com> <2C47327A-A1E8-43AA-8145-4CFEA1D27EC5@clarkeandclarke.co.uk> Message-ID: <1311101961514-3679017.post@n4.nabble.com> I open up a new thread on the subject of revServer stacks :: how does it work? -- View this message in context: http://runtime-revolution.278305.n4.nabble.com/ANN-revIgniter-v1-4b-tp3678359p3679017.html Sent from the Revolution - User mailing list archive at Nabble.com. From stephenREVOLUTION2 at barncard.com Tue Jul 19 15:02:40 2011 From: stephenREVOLUTION2 at barncard.com (stephen barncard) Date: Tue, 19 Jul 2011 14:02:40 -0500 Subject: [ANN] revIgniter v1.4b In-Reply-To: References: <527C7219-72AE-4552-B8DD-43336F45923C@dimensionB.de> <1311094209355-3678624.post@n4.nabble.com> <1311095748965-3678689.post@n4.nabble.com> <1311097203523-3678755.post@n4.nabble.com> <2C47327A-A1E8-43AA-8145-4CFEA1D27EC5@clarkeandclarke.co.uk> Message-ID: yeah PANIC Software. I love those guys. Great code and a sense of humor. Did you read their story about how Audion almost became the framework for iTunes? They actually had an awkward meeting with the great one, and turned him down! On 19 July 2011 11:34, Andre Garzia wrote: > Coda is a great product by one of the greatest if not the greatest software > house (that publishes only for the mac). Audiom was my default music player > since Mac OS 9, now I've migrated to Grooveshark. > > To edit my own LiveCode Server stuff I use Transmit + TextMate or my own > tools sometimes... FTP Commander is a quick editor for irev files but I > need > to fix some stuff in it before it can be used for more serious work. > > On Tue, Jul 19, 2011 at 3:24 PM, Keith Clarke < > keith.clarke at clarkeandclarke.co.uk> wrote: > > > ...that makes me so pleased that I recently bought Coda ;-) > > > > On 19 Jul 2011, at 19:20, stephen barncard wrote: > > > > > Textmate and Ralf Bitter's free Livecode plugin for Textmate rocks for > > live > > > iRev editing. > > > > > > sqb > > > > > > On 19 July 2011 10:49, Andre Garzia wrote: > > > > > >> LiveCode IDE can't handle LiveCode Server scripts... :-( > > >> > > >> On Tue, Jul 19, 2011 at 2:40 PM, Andrew Kluthe > > wrote: > > >> > > >>> I don't need GUI stuff if I can send messages and write my code in a > > >>> colorized code editor instead of notepad++. Very excited. > > >>> > > >>> -- > > >>> View this message in context: > > >>> > > >> > > > http://runtime-revolution.278305.n4.nabble.com/ANN-revIgniter-v1-4b-tp3678359p3678755.html > > >>> Sent from the Revolution - User mailing list archive at Nabble.com. > > > > > > _______________________________________________ > > use-livecode mailing list > > use-livecode at lists.runrev.com > > Please visit this url to subscribe, unsubscribe and manage your > > subscription preferences: > > http://lists.runrev.com/mailman/listinfo/use-livecode > > > > > > -- > http://www.andregarzia.com All We Do Is Code. > _______________________________________________ > use-livecode mailing list > use-livecode at lists.runrev.com > Please visit this url to subscribe, unsubscribe and manage your > subscription preferences: > http://lists.runrev.com/mailman/listinfo/use-livecode > -- Stephen Barncard San Francisco Ca. USA more about sqb From andre at andregarzia.com Tue Jul 19 15:10:09 2011 From: andre at andregarzia.com (Andre Garzia) Date: Tue, 19 Jul 2011 16:10:09 -0300 Subject: [ANN] revIgniter v1.4b In-Reply-To: References: <527C7219-72AE-4552-B8DD-43336F45923C@dimensionB.de> <1311094209355-3678624.post@n4.nabble.com> <1311095748965-3678689.post@n4.nabble.com> <1311097203523-3678755.post@n4.nabble.com> <2C47327A-A1E8-43AA-8145-4CFEA1D27EC5@clarkeandclarke.co.uk> Message-ID: On Tue, Jul 19, 2011 at 4:02 PM, stephen barncard < stephenREVOLUTION2 at barncard.com> wrote: > They actually had an awkward meeting with the great one, and turned > him down! > instead of great one, I read Great Old One and some weird Lovecraftian part of my mind suddenly screamed in horror.... :-) Panic Rox! -- http://www.andregarzia.com All We Do Is Code. From jacque at hyperactivesw.com Tue Jul 19 15:14:50 2011 From: jacque at hyperactivesw.com (J. Landman Gay) Date: Tue, 19 Jul 2011 14:14:50 -0500 Subject: [ANN] revIgniter v1.4b In-Reply-To: References: <527C7219-72AE-4552-B8DD-43336F45923C@dimensionB.de> <1311094209355-3678624.post@n4.nabble.com> <1311095748965-3678689.post@n4.nabble.com> <1311097203523-3678755.post@n4.nabble.com> Message-ID: <4E25D7AA.6080005@hyperactivesw.com> On 7/19/11 1:20 PM, stephen barncard wrote: > Textmate and Ralf Bitter's free Livecode plugin for Textmate rocks for live > iRev editing. I know Trevor made one for BBEdit, is Ralf's a new one? Will it work with BBEdit (and do you have a link?) -- Jacqueline Landman Gay | jacque at hyperactivesw.com HyperActive Software | http://www.hyperactivesw.com From stephenREVOLUTION2 at barncard.com Tue Jul 19 15:18:35 2011 From: stephenREVOLUTION2 at barncard.com (stephen barncard) Date: Tue, 19 Jul 2011 14:18:35 -0500 Subject: [ANN] revIgniter v1.4b In-Reply-To: <4E25D7AA.6080005@hyperactivesw.com> References: <527C7219-72AE-4552-B8DD-43336F45923C@dimensionB.de> <1311094209355-3678624.post@n4.nabble.com> <1311095748965-3678689.post@n4.nabble.com> <1311097203523-3678755.post@n4.nabble.com> <4E25D7AA.6080005@hyperactivesw.com> Message-ID: Strictly for Textmate. On 19 July 2011 14:14, J. Landman Gay wrote: > On 7/19/11 1:20 PM, stephen barncard wrote: > >> Textmate and Ralf Bitter's free Livecode plugin for Textmate rocks for >> live >> iRev editing. >> > > I know Trevor made one for BBEdit, is Ralf's a new one? Will it work with > BBEdit (and do you have a link?) > > -- > Jacqueline Landman Gay | jacque at hyperactivesw.com > HyperActive Software | http://www.hyperactivesw.com > > > ______________________________**_________________ > use-livecode mailing list > use-livecode at lists.runrev.com > Please visit this url to subscribe, unsubscribe and manage your > subscription preferences: > http://lists.runrev.com/**mailman/listinfo/use-livecode > -- Stephen Barncard San Francisco Ca. USA more about sqb From stephenREVOLUTION2 at barncard.com Tue Jul 19 15:19:45 2011 From: stephenREVOLUTION2 at barncard.com (stephen barncard) Date: Tue, 19 Jul 2011 14:19:45 -0500 Subject: [ANN] revIgniter v1.4b In-Reply-To: References: <527C7219-72AE-4552-B8DD-43336F45923C@dimensionB.de> <1311094209355-3678624.post@n4.nabble.com> <1311095748965-3678689.post@n4.nabble.com> <1311097203523-3678755.post@n4.nabble.com> <4E25D7AA.6080005@hyperactivesw.com> Message-ID: http://revigniter.com/accessory On 19 July 2011 14:18, stephen barncard wrote: > Strictly for Textmate. > > > On 19 July 2011 14:14, J. Landman Gay wrote: > >> On 7/19/11 1:20 PM, stephen barncard wrote: >> >>> Textmate and Ralf Bitter's free Livecode plugin for Textmate rocks for >>> live >>> iRev editing. >>> >> >> I know Trevor made one for BBEdit, is Ralf's a new one? Will it work with >> BBEdit (and do you have a link?) >> >> -- >> Jacqueline Landman Gay | jacque at hyperactivesw.com >> > Stephen Barncard San Francisco Ca. USA more about sqb From keith.clarke at clarkeandclarke.co.uk Tue Jul 19 15:23:24 2011 From: keith.clarke at clarkeandclarke.co.uk (Keith Clarke) Date: Tue, 19 Jul 2011 20:23:24 +0100 Subject: [ANN] revIgniter v1.4b In-Reply-To: References: <527C7219-72AE-4552-B8DD-43336F45923C@dimensionB.de> <1311094209355-3678624.post@n4.nabble.com> <1311095748965-3678689.post@n4.nabble.com> <1311097203523-3678755.post@n4.nabble.com> <2C47327A-A1E8-43AA-8145-4CFEA1D27EC5@clarkeandclarke.co.uk> Message-ID: ...I was using Text Wrangler with CSSEdit (which I like a lot) and Transmit, but as a coding numpty, I was looking for a visual, more declarative (point-and-click instead of type) toolset, with inbuilt irev language awareness, alongside HTML, Javascript and CSS. I really like Flux and would love to see an irev-aware Flux (Andre, check it out as the early design prototypes for revImpossible http://theescapers.com/screencast/ !) So, I went the Coda all-in-one route rather than replacing TextWrangler with TextMate. I've not got my head around it yet - not have I added the SubEthaEdit plugin, but I guess that may be out of date for the 4.6.3 version of irev anyway. Does TextMate have a plugin for .irev language awareness? Best, Keith.. On 19 Jul 2011, at 19:36, stephen barncard wrote: > Coda is a great editor too - and does CSS nicely. > > I made a SubEthaEdit editor plugin that works in Coda. It should be on their > website. It works ok - (I modified a template for Director!) -- but it > could use some improvement. > > Runtime Revolution (includes > style) > > > feel free to improve. > > http://www.codingmonkeys.de/subethaedit/mode.html > > On 19 July 2011 11:24, Keith Clarke wrote: > >> ...that makes me so pleased that I recently bought Coda ;-) From keith.clarke at clarkeandclarke.co.uk Tue Jul 19 15:25:31 2011 From: keith.clarke at clarkeandclarke.co.uk (Keith Clarke) Date: Tue, 19 Jul 2011 20:25:31 +0100 Subject: [ANN] revIgniter v1.4b In-Reply-To: References: <527C7219-72AE-4552-B8DD-43336F45923C@dimensionB.de> <1311094209355-3678624.post@n4.nabble.com> <1311095748965-3678689.post@n4.nabble.com> <1311097203523-3678755.post@n4.nabble.com> <2C47327A-A1E8-43AA-8145-4CFEA1D27EC5@clarkeandclarke.co.uk> Message-ID: <75850846-1EFA-4392-A425-40CDAE8EEF85@clarkeandclarke.co.uk> Doh! there I go sending when I should have been reading about Ralf's Textmate plugin. Note to self - learn to type quicker! Best, Keith.. On 19 Jul 2011, at 20:23, Keith Clarke wrote: > ...I was using Text Wrangler with CSSEdit (which I like a lot) and Transmit, but as a coding numpty, I was looking for a visual, more declarative (point-and-click instead of type) toolset, with inbuilt irev language awareness, alongside HTML, Javascript and CSS. > > I really like Flux and would love to see an irev-aware Flux (Andre, check it out as the early design prototypes for revImpossible http://theescapers.com/screencast/ !) > > So, I went the Coda all-in-one route rather than replacing TextWrangler with TextMate. I've not got my head around it yet - not have I added the SubEthaEdit plugin, but I guess that may be out of date for the 4.6.3 version of irev anyway. Does TextMate have a plugin for .irev language awareness? > Best, > Keith.. > > On 19 Jul 2011, at 19:36, stephen barncard wrote: > >> Coda is a great editor too - and does CSS nicely. >> >> I made a SubEthaEdit editor plugin that works in Coda. It should be on their >> website. It works ok - (I modified a template for Director!) -- but it >> could use some improvement. >> >> Runtime Revolution (includes >> style) >> >> >> feel free to improve. >> >> http://www.codingmonkeys.de/subethaedit/mode.html >> >> On 19 July 2011 11:24, Keith Clarke wrote: >> >>> ...that makes me so pleased that I recently bought Coda ;-) > From kray at sonsothunder.com Tue Jul 19 15:29:41 2011 From: kray at sonsothunder.com (Ken Ray) Date: Tue, 19 Jul 2011 14:29:41 -0500 Subject: age calculation on Windows vs Mac In-Reply-To: References: Message-ID: On Jul 19, 2011, at 11:40 AM, Pete wrote: > Hi Andre, > I don't have a definitive answer for you but this sounds like it is > connected to "epoch" dates which are the number of days since Jan 1, 1970. > I'm guessing that somewhere along the line, your dates are being converted > to the epoch format on the WIndows platform that would certainly cause a > problem for dates prior to 1970. I don't know why that would be different > in Windows than OS X. I'm not sure either, since on Mac OS 9 whenever a date got messed up it would always be set to "12/31/1969" (the day before the Epoch). For Windows, though, this is actually documented under the 'convert' command: "Note: The range of dates that the convertcommand can handle is limited by the operating system's date routines. In particular, Windows systems are limited to dates after 1/1/1970." You'll have to use some other method to calculate your dates, unfortunately. FYI: Here's the info on the Unix Epoch: http://en.wikipedia.org/wiki/Unix_time Ken Ray Sons of Thunder Software, Inc. Email: kray at sonsothunder.com Web Site: http://www.sonsothunder.com/ From kray at sonsothunder.com Tue Jul 19 15:33:31 2011 From: kray at sonsothunder.com (Ken Ray) Date: Tue, 19 Jul 2011 14:33:31 -0500 Subject: [ANN] revIgniter v1.4b In-Reply-To: <75850846-1EFA-4392-A425-40CDAE8EEF85@clarkeandclarke.co.uk> References: <527C7219-72AE-4552-B8DD-43336F45923C@dimensionB.de> <1311094209355-3678624.post@n4.nabble.com> <1311095748965-3678689.post@n4.nabble.com> <1311097203523-3678755.post@n4.nabble.com> <2C47327A-A1E8-43AA-8145-4CFEA1D27EC5@clarkeandclarke.co.uk> <75850846-1EFA-4392-A425-40CDAE8EEF85@clarkeandclarke.co.uk> Message-ID: <90D0E5A5-B90B-460A-A9E5-322CC1BDE95E@sonsothunder.com> BTW, There's more modules for a few other editors - originally designed to work with the STS/MLXEditor that allows you to use a text editor for editing instead of the IDE automatically. Here's the link: http://www.sonsothunder.com/devres/livecode/downloads/stsMLXEditor.htm#Plugins Ken Ray Sons of Thunder Software, Inc. Email: kray at sonsothunder.com Web Site: http://www.sonsothunder.com/ From andre at andregarzia.com Tue Jul 19 15:33:53 2011 From: andre at andregarzia.com (Andre Garzia) Date: Tue, 19 Jul 2011 16:33:53 -0300 Subject: [ANN] revIgniter v1.4b In-Reply-To: References: <527C7219-72AE-4552-B8DD-43336F45923C@dimensionB.de> <1311094209355-3678624.post@n4.nabble.com> <1311095748965-3678689.post@n4.nabble.com> <1311097203523-3678755.post@n4.nabble.com> <2C47327A-A1E8-43AA-8145-4CFEA1D27EC5@clarkeandclarke.co.uk> Message-ID: On Tue, Jul 19, 2011 at 4:23 PM, Keith Clarke < keith.clarke at clarkeandclarke.co.uk> wrote: > ...I was using Text Wrangler with CSSEdit (which I like a lot) and > Transmit, but as a coding numpty, I was looking for a visual, more > declarative (point-and-click instead of type) toolset, with inbuilt irev > language awareness, alongside HTML, Javascript and CSS. > > I really like Flux and would love to see an irev-aware Flux (Andre, check > it out as the early design prototypes for revImpossible > http://theescapers.com/screencast/ !) > > I have licenses for Flux since their early betas... it is a great idea but I think it is a poor implementation. The software (Flux 1 and 2) crashes a lot, I cannot tolerate software crashing like that. Also, once you have dozens of nested divs, things become confusing. That is not what I aim with revImpossible.... man, I'd better make a video... :-) -- http://www.andregarzia.com All We Do Is Code. From andre at andregarzia.com Tue Jul 19 15:38:16 2011 From: andre at andregarzia.com (Andre Garzia) Date: Tue, 19 Jul 2011 16:38:16 -0300 Subject: [ANN] revIgniter v1.4b In-Reply-To: <90D0E5A5-B90B-460A-A9E5-322CC1BDE95E@sonsothunder.com> References: <527C7219-72AE-4552-B8DD-43336F45923C@dimensionB.de> <1311094209355-3678624.post@n4.nabble.com> <1311095748965-3678689.post@n4.nabble.com> <1311097203523-3678755.post@n4.nabble.com> <2C47327A-A1E8-43AA-8145-4CFEA1D27EC5@clarkeandclarke.co.uk> <75850846-1EFA-4392-A425-40CDAE8EEF85@clarkeandclarke.co.uk> <90D0E5A5-B90B-460A-A9E5-322CC1BDE95E@sonsothunder.com> Message-ID: Ken, do you still use STS/MLXEditor? Cheers andre On Tue, Jul 19, 2011 at 4:33 PM, Ken Ray wrote: > BTW, > > There's more modules for a few other editors - originally designed to work > with the STS/MLXEditor that allows you to use a text editor for editing > instead of the IDE automatically. Here's the link: > > > http://www.sonsothunder.com/devres/livecode/downloads/stsMLXEditor.htm#Plugins > > > Ken Ray > Sons of Thunder Software, Inc. > Email: kray at sonsothunder.com > Web Site: http://www.sonsothunder.com/ > _______________________________________________ > use-livecode mailing list > use-livecode at lists.runrev.com > Please visit this url to subscribe, unsubscribe and manage your > subscription preferences: > http://lists.runrev.com/mailman/listinfo/use-livecode > -- http://www.andregarzia.com All We Do Is Code. From keith.clarke at clarkeandclarke.co.uk Tue Jul 19 15:53:44 2011 From: keith.clarke at clarkeandclarke.co.uk (Keith Clarke) Date: Tue, 19 Jul 2011 20:53:44 +0100 Subject: [ANN] revIgniter v1.4b In-Reply-To: References: <527C7219-72AE-4552-B8DD-43336F45923C@dimensionB.de> <1311094209355-3678624.post@n4.nabble.com> <1311095748965-3678689.post@n4.nabble.com> <1311097203523-3678755.post@n4.nabble.com> <2C47327A-A1E8-43AA-8145-4CFEA1D27EC5@clarkeandclarke.co.uk> Message-ID: <8F79C897-94F3-4C97-B564-9F8674FBAAD0@clarkeandclarke.co.uk> ...yes, nice concept but Flux 2 was quite unstable. It got better with V3 - but everything is relative! On 19 Jul 2011, at 20:33, Andre Garzia wrote: > On Tue, Jul 19, 2011 at 4:23 PM, Keith Clarke < > keith.clarke at clarkeandclarke.co.uk> wrote: > >> ...I was using Text Wrangler with CSSEdit (which I like a lot) and >> Transmit, but as a coding numpty, I was looking for a visual, more >> declarative (point-and-click instead of type) toolset, with inbuilt irev >> language awareness, alongside HTML, Javascript and CSS. >> >> I really like Flux and would love to see an irev-aware Flux (Andre, check >> it out as the early design prototypes for revImpossible >> http://theescapers.com/screencast/ !) >> >> > I have licenses for Flux since their early betas... it is a great idea but I > think it is a poor implementation. The software (Flux 1 and 2) crashes a > lot, I cannot tolerate software crashing like that. Also, once you have > dozens of nested divs, things become confusing. > > That is not what I aim with revImpossible.... man, I'd better make a > video... > > :-) From kray at sonsothunder.com Tue Jul 19 16:19:17 2011 From: kray at sonsothunder.com (Ken Ray) Date: Tue, 19 Jul 2011 15:19:17 -0500 Subject: [ANN] revIgniter v1.4b In-Reply-To: References: <527C7219-72AE-4552-B8DD-43336F45923C@dimensionB.de> <1311094209355-3678624.post@n4.nabble.com> <1311095748965-3678689.post@n4.nabble.com> <1311097203523-3678755.post@n4.nabble.com> <2C47327A-A1E8-43AA-8145-4CFEA1D27EC5@clarkeandclarke.co.uk> <75850846-1EFA-4392-A425-40CDAE8EEF85@clarkeandclarke.co.uk> <90D0E5A5-B90B-460A-A9E5-322CC1BDE95E@sonsothunder.com> Message-ID: <772AC3DE-295B-4480-BC0C-64CE14B2AF50@sonsothunder.com> On Jul 19, 2011, at 2:38 PM, Andre Garzia wrote: > Ken, > > do you still use STS/MLXEditor? I haven't in a while - but it still works with the latest LC just as it did with Revolution... :D Ken > > On Tue, Jul 19, 2011 at 4:33 PM, Ken Ray wrote: > >> BTW, >> >> There's more modules for a few other editors - originally designed to work >> with the STS/MLXEditor that allows you to use a text editor for editing >> instead of the IDE automatically. Here's the link: >> >> >> http://www.sonsothunder.com/devres/livecode/downloads/stsMLXEditor.htm#Plugins >> >> >> Ken Ray >> Sons of Thunder Software, Inc. >> Email: kray at sonsothunder.com >> Web Site: http://www.sonsothunder.com/ >> _______________________________________________ >> use-livecode mailing list >> use-livecode at lists.runrev.com >> Please visit this url to subscribe, unsubscribe and manage your >> subscription preferences: >> http://lists.runrev.com/mailman/listinfo/use-livecode >> > > > > -- > http://www.andregarzia.com All We Do Is Code. > _______________________________________________ > use-livecode mailing list > use-livecode at lists.runrev.com > Please visit this url to subscribe, unsubscribe and manage your subscription preferences: > http://lists.runrev.com/mailman/listinfo/use-livecode Ken Ray Sons of Thunder Software, Inc. Email: kray at sonsothunder.com Web Site: http://www.sonsothunder.com/ From pete at mollysrevenge.com Tue Jul 19 17:34:10 2011 From: pete at mollysrevenge.com (Pete) Date: Tue, 19 Jul 2011 14:34:10 -0700 Subject: age calculation on Windows vs Mac In-Reply-To: References: Message-ID: Sounds like time for using Julian Days for the calculation. Per Ken, take a look at Sarah's date conversion routines at www.troz.net(they include Julian day conversions). Also, if you happen to be using an sql database in your application, there are SELECT statements that will do date calculations for you. Pete Molly's Revenge On Tue, Jul 19, 2011 at 12:29 PM, Ken Ray wrote: > > On Jul 19, 2011, at 11:40 AM, Pete wrote: > > > Hi Andre, > > I don't have a definitive answer for you but this sounds like it is > > connected to "epoch" dates which are the number of days since Jan 1, > 1970. > > I'm guessing that somewhere along the line, your dates are being > converted > > to the epoch format on the WIndows platform that would certainly cause a > > problem for dates prior to 1970. I don't know why that would be > different > > in Windows than OS X. > > I'm not sure either, since on Mac OS 9 whenever a date got messed up it > would always be set to "12/31/1969" (the day before the Epoch). For Windows, > though, this is actually documented under the 'convert' command: > > "Note: The range of dates that the convertcommand can handle is limited by > the operating system's date routines. In particular, Windows systems are > limited to dates after 1/1/1970." > > You'll have to use some other method to calculate your dates, > unfortunately. FYI: Here's the info on the Unix Epoch: > > http://en.wikipedia.org/wiki/Unix_time > > > Ken Ray > Sons of Thunder Software, Inc. > Email: kray at sonsothunder.com > Web Site: http://www.sonsothunder.com/ > > > _______________________________________________ > use-livecode mailing list > use-livecode at lists.runrev.com > Please visit this url to subscribe, unsubscribe and manage your > subscription preferences: > http://lists.runrev.com/mailman/listinfo/use-livecode > > From psahores at free.fr Tue Jul 19 17:58:57 2011 From: psahores at free.fr (Pierre Sahores) Date: Tue, 19 Jul 2011 23:58:57 +0200 Subject: [ANN] revIgniter v1.4b In-Reply-To: References: Message-ID: Thank you for your outstanding and major contribution to the LC-server frameworks main tools, Ralf ! All the best, Le 19 juil. 2011 ? 17:05, Ralf Bitter a ?crit : > > revIgniter v1.4b has been released. > > This is the first version which respects the latest > engine changes (LiveCode server version 4.6.3). > Lots of adjustments were made and the whole > Encrypt library is revised. > > Keep in mind that this version is not compatible with > revServer (engine versions before LiveCode server 4.6.3). > > Info and download at: http://www.revigniter.com/ > > > > Ralf > > > _______________________________________________ > use-livecode mailing list > use-livecode at lists.runrev.com > Please visit this url to subscribe, unsubscribe and manage your subscription preferences: > http://lists.runrev.com/mailman/listinfo/use-livecode > -- Pierre Sahores mobile : (33) 6 03 95 77 70 www.woooooooords.com www.sahores-conseil.com From rabit at dimensionB.de Tue Jul 19 18:18:05 2011 From: rabit at dimensionB.de (Ralf Bitter) Date: Wed, 20 Jul 2011 00:18:05 +0200 Subject: [ANN] revIgniter v1.4b In-Reply-To: References: Message-ID: <42EA23FE-EE94-4B5E-A819-5E2550A0993B@dimensionB.de> Pierre, thanks very much for the kind words. On 19.07.2011, at 23:58, Pierre Sahores wrote: > Thank you for your outstanding and major contribution to the LC-server frameworks main tools, Ralf ! > > All the best, > From capellan2000 at gmail.com Tue Jul 19 18:31:40 2011 From: capellan2000 at gmail.com (Alejandro Tejada) Date: Tue, 19 Jul 2011 18:31:40 -0400 Subject: [ANN] alien camel in your LiveCode Message-ID: Hi Thierry, On Tue, Jul 19, 2011 Thierry Douez wrote: > Let me know off-list your Perl configuration ( distribution and version) > and I'll make the first one on Windows works for you. > Actually, I've started to build the external for windows with > ActivePerl 5.12.4 ?( free pre-build perl and easy installation from > ActiveState ) Actually, I do not have a Perl distribution installed. I use a portable edition: http://portableapps.com/node/12595 If you include a recommended distribution in the instructions, I will install that distribution. Keep up your Great Work! :-) Al From m.schonewille at economy-x-talk.com Tue Jul 19 18:35:54 2011 From: m.schonewille at economy-x-talk.com (Mark Schonewille) Date: Wed, 20 Jul 2011 00:35:54 +0200 Subject: ANN: LiveCodeErrors for iPhone In-Reply-To: References: <215B0FE5-038A-47C7-9F24-B9D29CEDD816@economy-x-talk.com> <4E25A8E0.6090704@fourthworld.com> Message-ID: <2E567827-7552-4FF2-BDFC-446D6330FB6B@economy-x-talk.com> Hi Chipp, Weird. I would expect all pros to know about it. I'd say, give it a try. execute the following script on foo try executeNonExistingHandler catch myErr put myErr end try end foo The first item in the message box will be a number. Download my app from http://qery.us/v4 and enter the number in the app. It'll tell you that the handler executeNonExistingHandler could not be found. If you contact me off-list, I'll send you a promo code. -- Best regards, Mark Schonewille Economy-x-Talk Consulting and Software Engineering Homepage: http://economy-x-talk.com Twitter: http://twitter.com/xtalkprogrammer KvK: 50277553 New: Download the Installer Maker Plugin 1.6 for LiveCode here http://qery.us/ce On 19 jul 2011, at 18:58, Chipp Walters wrote: > Interesting. > > I have to say, in the past 10 years or so since I've been programming > commercial applications for my clients and our company, I do not recall > seeing such an error. In fact, this is the first time I've even heard of > them. > > I, too, would like to know under what circumstances they are fired. It seems > like a bug to me in LC's error handling code if they continually pop up. I > suspect Richard knew about this from all his work with the MetaCard IDE. From SparkOutYNY at gmail.com Tue Jul 19 19:18:40 2011 From: SparkOutYNY at gmail.com (SparkOut) Date: Tue, 19 Jul 2011 16:18:40 -0700 (PDT) Subject: age calculation on Windows vs Mac In-Reply-To: References: Message-ID: <1311117520617-3679589.post@n4.nabble.com> I don't know why this is the case, because Windows can obviously calculate dates prior to the 1970 start of epoch in other applications... but something to do with the method LC uses to hook into a date function based on the epoch will fail. Try: put the date into tDate convert tDate to dateItems put 1928 into item 1 of tDate convert tDate to short date if tDate is not a date then answer tDate && "is not a date" else answer tDate && "is a date" end if On Windows, this will always result in "not a date". I know a great percentage of LC users are Mac oriented but there are plenty of cross-platform and Windows developers, so I am rather surprised that there is so little attention paid to this - and the old, old, old bugzilla entry #4941 from 2007. -- View this message in context: http://runtime-revolution.278305.n4.nabble.com/age-calculation-on-Windows-vs-Mac-tp3678552p3679589.html Sent from the Revolution - User mailing list archive at Nabble.com. From ambassador at fourthworld.com Tue Jul 19 19:21:45 2011 From: ambassador at fourthworld.com (Richard Gaskin) Date: Tue, 19 Jul 2011 16:21:45 -0700 Subject: ANN: LiveCodeErrors for iPhone In-Reply-To: <2E567827-7552-4FF2-BDFC-446D6330FB6B@economy-x-talk.com> References: <2E567827-7552-4FF2-BDFC-446D6330FB6B@economy-x-talk.com> Message-ID: <4E261189.9050009@fourthworld.com> Mark Schonewille wrote: > Hi Chipp, > > Weird. I would expect all pros to know about it. > > I'd say, give it a try. execute the following script > > on foo > try > executeNonExistingHandler > catch myErr > put myErr > end try > end foo I doubt many pros would need a lookup to figure out that a non-existent handler would result in a "can't find handler" error. ;) But seriously, most devs using try/catch setups won't be executing random code, they'll be working on things that are somewhat well defined, and use their own error messages to report to the user. I don't know of any dev who report RunRev's messages to their users; they just don't provide enough guidance for the user. Those so inclined can just use the same list everything else in the LiveCode world uses, the one RunRev provides for us in their revErrorDisplay stack: get line (item 1 of tErr) of the cErrorsList of cd 1 \ of stack "revErrorDisplay" into tList Section 10.4 ("Custom Error handling") of the LiveCode User Guide covers this well. -- Richard Gaskin Fourth World LiveCode training and consulting: http://www.fourthworld.com Webzine for LiveCode developers: http://www.LiveCodeJournal.com LiveCode Journal blog: http://LiveCodejournal.com/blog.irv From bvlahos at mac.com Tue Jul 19 19:24:15 2011 From: bvlahos at mac.com (Bill Vlahos) Date: Tue, 19 Jul 2011 16:24:15 -0700 Subject: age calculation on Windows vs Mac In-Reply-To: References: Message-ID: The problem is the epoch date in Windows is 1970 and LiveCode can't deal with dates earlier than that in Windows. You will need to use a VBScript. Here is what I use in InfoWallet which handles Windows, Mac, and Linux correctly which was based on code provided by folks on this list. on calcAge tDate,asOf -- returns the current age based on a DOB of tDate -- as of the date asOf, or if asOf = empty, as of today if tDate = empty then return empty if the platform is "WIn32" then isADate tDate if the result then put "sdate = " & quote & tDate & quote into vFive put return & "result = DateDiff(" & quote & "d" & quote & ",sdate,Date)" after vFive put return & "trueage = result / 365" after vFive put return & "result = Left(trueage,2)" after vFive do vFive as vbscript return the result else return empty end if else if tDate is not a date then return empty if asOf = empty then put the date into asOf end if set the itemdelimiter to "/" put item -1 of the short date into nowYr set the centurycutoff to nowYr+1 set the itemdelimiter to comma convert tDate to dateItems convert asOf to dateitems put item 1 of asOf - item 1 of tDate - 1 into tAge put item 1 of asOf into item 1 of tDate convert tDate to seconds convert asOf to seconds if tDate <= asOf then add 1 to tAge return tAge end if end calcAge Bill Vlahos _________________ InfoWallet (http://www.infowallet.com) is about keeping your important life information with you, accessible, and secure. On Jul 19, 2011, at 9:23 AM, Andr? Bisseret wrote: > Bonjour, > > I am a Mac user but I recently bought a PC (windows 7) in order to test my standalones for windows more quickly. > > I am fighting for hours now with the following problem : > > In an app., I have a handler that calculates the age from the short birth date (thanks to Jim Ault who gave it on the list). > The birth date is jj/mm/AAAA or jj/mm/AA as well. > > This handler works fine on Mac, but not on windows. > > On windows, the age is invariably 100 if the birth year < 1970. (with JJ/MM/AA or JJ/MM/AAAA (as bad ;-( > If the birth year > = 1970 then the age is correct. > > I tried to play with the centuryCutOff but without any success (on windows). > > I am not familiar with Windows : what am I missing ? Is it a matter of computer setting or should I program differently for windows than for mac !? > > Thanks a lot in advance for any help > > Best regards from Grenoble > > Andr? > > P.S. By the way, I used to use a search engine (if I well remember, it was in the help menu of livecode) I do not find it any more; has it been suppressed ? (or here again, I am missing something !) > _______________________________________________ > use-livecode mailing list > use-livecode at lists.runrev.com > Please visit this url to subscribe, unsubscribe and manage your subscription preferences: > http://lists.runrev.com/mailman/listinfo/use-livecode From m.schonewille at economy-x-talk.com Tue Jul 19 19:26:20 2011 From: m.schonewille at economy-x-talk.com (Mark Schonewille) Date: Wed, 20 Jul 2011 01:26:20 +0200 Subject: ANN: LiveCodeErrors for iPhone In-Reply-To: <4E261189.9050009@fourthworld.com> References: <2E567827-7552-4FF2-BDFC-446D6330FB6B@economy-x-talk.com> <4E261189.9050009@fourthworld.com> Message-ID: Sure, Richard, it is clear that you won't buy that app. Others will, already did, and I'm glad I was able to provide them with something useful. -- Best regards, Mark Schonewille Economy-x-Talk Consulting and Software Engineering Homepage: http://economy-x-talk.com Twitter: http://twitter.com/xtalkprogrammer KvK: 50277553 New: Download the Installer Maker Plugin 1.6 for LiveCode here http://qery.us/ce On 20 jul 2011, at 01:21, Richard Gaskin wrote: > Mark Schonewille wrote: > >> Hi Chipp, >> >> Weird. I would expect all pros to know about it. >> >> I'd say, give it a try. execute the following script >> >> on foo >> try >> executeNonExistingHandler >> catch myErr >> put myErr >> end try >> end foo > > I doubt many pros would need a lookup to figure out that a non-existent handler would result in a "can't find handler" error. ;) > > But seriously, most devs using try/catch setups won't be executing random code, they'll be working on things that are somewhat well defined, and use their own error messages to report to the user. I don't know of any dev who report RunRev's messages to their users; they just don't provide enough guidance for the user. > > Those so inclined can just use the same list everything else in the LiveCode world uses, the one RunRev provides for us in their revErrorDisplay stack: > > get line (item 1 of tErr) of the cErrorsList of cd 1 \ > of stack "revErrorDisplay" into tList > > > Section 10.4 ("Custom Error handling") of the LiveCode User Guide covers this well. > > -- > Richard Gaskin > Fourth World > LiveCode training and consulting: http://www.fourthworld.com > Webzine for LiveCode developers: http://www.LiveCodeJournal.com > LiveCode Journal blog: http://LiveCodejournal.com/blog.irv > > _______________________________________________ > use-livecode mailing list > use-livecode at lists.runrev.com > Please visit this url to subscribe, unsubscribe and manage your subscription preferences: > http://lists.runrev.com/mailman/listinfo/use-livecode From kray at sonsothunder.com Tue Jul 19 21:26:46 2011 From: kray at sonsothunder.com (Ken Ray) Date: Tue, 19 Jul 2011 20:26:46 -0500 Subject: LiveCode Server Release and On-Rev Message-ID: <848DF372-D3F3-4658-8A13-14D4C7852B67@sonsothunder.com> Just curious... does the recent release of LiveCode Server affect subscribers to On-Rev? If so, how and what should we be aware of? Ken Ray Sons of Thunder Software, Inc. Email: kray at sonsothunder.com Web Site: http://www.sonsothunder.com/ From roger.e.eller at sealedair.com Tue Jul 19 21:34:40 2011 From: roger.e.eller at sealedair.com (Roger Eller) Date: Tue, 19 Jul 2011 21:34:40 -0400 Subject: Countdown HH:MM:SS Message-ID: I know I can convert the time to seconds, do the math, split it up into HH:MM:SS, and repeat a "send myCounter in 1 sec" to update a countdown timer field, but before I begin chiseling on a new square wheel, is there a better (shorter) way? ?Roger From dunbarx at aol.com Tue Jul 19 23:00:11 2011 From: dunbarx at aol.com (dunbarx at aol.com) Date: Tue, 19 Jul 2011 23:00:11 -0400 (EDT) Subject: Countdown HH:MM:SS In-Reply-To: References: Message-ID: <8CE14A04637B482-1AA4-201D3@webmail-d058.sysops.aol.com> Hi. Shorter? It already can be pretty short. MUCH shorter? Where's Colin? Craig Newman -----Original Message----- From: Roger Eller To: How to use LiveCode Sent: Tue, Jul 19, 2011 5:36 pm Subject: Countdown HH:MM:SS I know I can convert the time to seconds, do the math, split it up into HH:MM:SS, and repeat a "send myCounter in 1 sec" to update a countdown timer field, but before I begin chiseling on a new square wheel, is there a better (shorter) way? ?Roger _______________________________________________ use-livecode mailing list use-livecode at lists.runrev.com Please visit this url to subscribe, unsubscribe and manage your subscription preferences: http://lists.runrev.com/mailman/listinfo/use-livecode From massung at gmail.com Tue Jul 19 23:02:47 2011 From: massung at gmail.com (Jeff Massung) Date: Tue, 19 Jul 2011 21:02:47 -0600 Subject: Countdown HH:MM:SS In-Reply-To: <8CE14A04637B482-1AA4-201D3@webmail-d058.sysops.aol.com> References: <8CE14A04637B482-1AA4-201D3@webmail-d058.sysops.aol.com> Message-ID: Lookup the "convert" function... dateAndTime and look at the format "dateItems". That probably has what you care about. HTH, Jeff M. From andre at andregarzia.com Tue Jul 19 23:35:25 2011 From: andre at andregarzia.com (Andre Garzia) Date: Wed, 20 Jul 2011 00:35:25 -0300 Subject: externals error under linux Message-ID: Folks, Anyone seen this: warning: implicit declaration of function ?getXtable? [-Wimplicit-function-declaration] It is related to some macro error I think, or the linker is not linking against the correct stuff... any clue? -- http://www.andregarzia.com All We Do Is Code. From kray at sonsothunder.com Wed Jul 20 02:12:21 2011 From: kray at sonsothunder.com (Ken Ray) Date: Wed, 20 Jul 2011 01:12:21 -0500 Subject: Countdown HH:MM:SS In-Reply-To: <8CE14A04637B482-1AA4-201D3@webmail-d058.sysops.aol.com> References: <8CE14A04637B482-1AA4-201D3@webmail-d058.sysops.aol.com> Message-ID: <61EE076F-3EE3-42D4-A552-2ADF6526034A@sonsothunder.com> On Jul 19, 2011, at 10:00 PM, DunbarX at aol.com wrote: > Hi. > > > Shorter? It already can be pretty short. MUCH shorter? Where's Colin? Here's the fastest way I know: on mouseUp DisplayTime end mouseUp on DisplayTime put the long time into tTime convert tTime to dateItems split tTime by "," put format("%02d:%02d:%02d",tTime[4],tTime[5],tTime[6]) into fld "Time" send "DisplayTime" to me in 1 second end DisplayTime Ken Ray Sons of Thunder Software, Inc. Email: kray at sonsothunder.com Web Site: http://www.sonsothunder.com/ From palcibiades-first at yahoo.co.uk Wed Jul 20 02:14:33 2011 From: palcibiades-first at yahoo.co.uk (Peter Alcibiades) Date: Tue, 19 Jul 2011 23:14:33 -0700 (PDT) Subject: [ANN] revIgniter v1.4b - Geany! In-Reply-To: References: <527C7219-72AE-4552-B8DD-43336F45923C@dimensionB.de> <1311094209355-3678624.post@n4.nabble.com> <1311095748965-3678689.post@n4.nabble.com> <1311097203523-3678755.post@n4.nabble.com> <2C47327A-A1E8-43AA-8145-4CFEA1D27EC5@clarkeandclarke.co.uk> Message-ID: <1311142473032-3680064.post@n4.nabble.com> What would be lovely would be if someone would do a plug-in for Geany. Its cross platform, so it would be usable in windows as well as the penguin. -- View this message in context: http://runtime-revolution.278305.n4.nabble.com/ANN-revIgniter-v1-4b-tp3678359p3680064.html Sent from the Revolution - User mailing list archive at Nabble.com. From revdev at pdslabs.net Wed Jul 20 02:47:59 2011 From: revdev at pdslabs.net (Phil Davis) Date: Tue, 19 Jul 2011 23:47:59 -0700 Subject: iPhone + Arduino Message-ID: <4E267A1F.1050604@pdslabs.net> I just saw this: http://www.tuaw.com/2011/07/19/cord-from-iphone-to-arduino-now-available/ Redpark (maker of the cable) has a Serial SDK for app dev with the cable. I assume it could be made into an external for mobile dev (?), but their licensing is for private use only. Bummer. -- Phil Davis PDS Labs Professional Software Development http://pdslabs.net From andre.bisseret at wanadoo.fr Wed Jul 20 03:25:59 2011 From: andre.bisseret at wanadoo.fr (=?iso-8859-1?Q?Andr=E9_Bisseret?=) Date: Wed, 20 Jul 2011 09:25:59 +0200 Subject: age calculation on Windows vs Mac In-Reply-To: References: Message-ID: <3A4F01A5-D32C-4D89-ADA5-9DB3E7FC4AA1@wanadoo.fr> Pete, Ken, SparkOut, Bill, Thank you very much for your attention and your answers. This morning, I am going to change my handler in order to get windows accepting it!! I will let you know my result Wonderful list ;-)) Andr? Le 20 juil. 2011 ? 01:24, Bill Vlahos a ?crit : > The problem is the epoch date in Windows is 1970 and LiveCode can't deal with dates earlier than that in Windows. You will need to use a VBScript. > > Here is what I use in InfoWallet which handles Windows, Mac, and Linux correctly which was based on code provided by folks on this list. > > on calcAge tDate,asOf > -- returns the current age based on a DOB of tDate > -- as of the date asOf, or if asOf = empty, as of today > if tDate = empty then return empty > if the platform is "WIn32" then > isADate tDate > if the result then > put "sdate = " & quote & tDate & quote into vFive > put return & "result = DateDiff(" & quote & "d" & quote & ",sdate,Date)" after vFive > put return & "trueage = result / 365" after vFive > put return & "result = Left(trueage,2)" after vFive > do vFive as vbscript > return the result > else > return empty > end if > else > if tDate is not a date then return empty > if asOf = empty then > put the date into asOf > end if > set the itemdelimiter to "/" > put item -1 of the short date into nowYr > set the centurycutoff to nowYr+1 > set the itemdelimiter to comma > convert tDate to dateItems > convert asOf to dateitems > put item 1 of asOf - item 1 of tDate - 1 into tAge > put item 1 of asOf into item 1 of tDate > convert tDate to seconds > convert asOf to seconds > if tDate <= asOf then add 1 to tAge > return tAge > end if > end calcAge > > > Bill Vlahos > _________________ > InfoWallet (http://www.infowallet.com) is about keeping your important life information with you, accessible, and secure. > > On Jul 19, 2011, at 9:23 AM, Andr? Bisseret wrote: > >> Bonjour, >> >> I am a Mac user but I recently bought a PC (windows 7) in order to test my standalones for windows more quickly. >> >> I am fighting for hours now with the following problem : >> >> In an app., I have a handler that calculates the age from the short birth date (thanks to Jim Ault who gave it on the list). >> The birth date is jj/mm/AAAA or jj/mm/AA as well. >> >> This handler works fine on Mac, but not on windows. >> >> On windows, the age is invariably 100 if the birth year < 1970. (with JJ/MM/AA or JJ/MM/AAAA (as bad ;-( >> If the birth year > = 1970 then the age is correct. >> >> I tried to play with the centuryCutOff but without any success (on windows). >> >> I am not familiar with Windows : what am I missing ? Is it a matter of computer setting or should I program differently for windows than for mac !? >> >> Thanks a lot in advance for any help >> >> Best regards from Grenoble >> >> Andr? >> >> P.S. By the way, I used to use a search engine (if I well remember, it was in the help menu of livecode) I do not find it any more; has it been suppressed ? (or here again, I am missing something !) >> _______________________________________________ >> use-livecode mailing list >> use-livecode at lists.runrev.com >> Please visit this url to subscribe, unsubscribe and manage your subscription preferences: >> http://lists.runrev.com/mailman/listinfo/use-livecode > > > _______________________________________________ > use-livecode mailing list > use-livecode at lists.runrev.com > Please visit this url to subscribe, unsubscribe and manage your subscription preferences: > http://lists.runrev.com/mailman/listinfo/use-livecode From SparkOutYNY at gmail.com Wed Jul 20 03:49:37 2011 From: SparkOutYNY at gmail.com (SparkOut) Date: Wed, 20 Jul 2011 00:49:37 -0700 (PDT) Subject: age calculation on Windows vs Mac In-Reply-To: <3A4F01A5-D32C-4D89-ADA5-9DB3E7FC4AA1@wanadoo.fr> References: <3A4F01A5-D32C-4D89-ADA5-9DB3E7FC4AA1@wanadoo.fr> Message-ID: <1311148177376-3680176.post@n4.nabble.com> As well as changing your handler, which is unfortunately necessary at the moment, if you have access to the QCC you could vote for bug #4941 and/or message support and bring awareness back to this. It's not that Livecode "can't" determine pre epoch dates on Windows - it's just that the implementation needs to be changed in order for it to happen. -- View this message in context: http://runtime-revolution.278305.n4.nabble.com/age-calculation-on-Windows-vs-Mac-tp3678552p3680176.html Sent from the Revolution - User mailing list archive at Nabble.com. From m.schonewille at economy-x-talk.com Wed Jul 20 05:01:17 2011 From: m.schonewille at economy-x-talk.com (Mark Schonewille) Date: Wed, 20 Jul 2011 11:01:17 +0200 Subject: Output error in standalone In-Reply-To: <3D74C673-C31E-4462-A2C2-5A1F7253CC13@mac.com> References: <3D74C673-C31E-4462-A2C2-5A1F7253CC13@mac.com> Message-ID: Hi Charles, You might want to check out this http://qery.us/v9 blog post. Are you sure that the Mac OS X binaries are still inside the Runtime folder of your Revolution application folder? Is any Mac OS X item at all created by the standalone builder? -- Best regards, Mark Schonewille Economy-x-Talk Consulting and Software Engineering Homepage: http://economy-x-talk.com Twitter: http://twitter.com/xtalkprogrammer KvK: 50277553 New: Download the Installer Maker Plugin 1.6 for LiveCode here http://qery.us/ce On 19 jul 2011, at 19:35, Charles Szasz wrote: > I am using Rev 4.0 Build 950. I just finishing working on a project. It consists of a splash stack and an app stack. I set up my Standalone Applications settings to create builds for Mac OS X (Universal) and Windows. When I tried to compile my app I got the following error message: > > There was was an error while saving the standalone application. Build failed for MacOSX: could not open output file. > > When I unchecked making a Mac OS X Universal and just made a windows build, I did not get this error and the windows app was created. I have not yet tried the windows build to see if it is okay. > > I have never gotten this error message before when compiling. Anybody have any suggestions on how I can correct this problem? Thanks for your time! > > Charles Szasz > cszasz at mac.com From toolbook at kestner.de Wed Jul 20 06:09:37 2011 From: toolbook at kestner.de (Tiemo Hollmann TB) Date: Wed, 20 Jul 2011 12:09:37 +0200 Subject: AW: ANN: LiveCodeErrors for iPhone In-Reply-To: References: <2E567827-7552-4FF2-BDFC-446D6330FB6B@economy-x-talk.com> <4E261189.9050009@fourthworld.com> Message-ID: <003701cc46c5$27e56b30$77b04190$@de> I just can't recall the circumstances, but I had some of these error messages just with codes in the last years and was happy about Marks web interface to decode them. My 2 cents Tiemo > -----Urspr?ngliche Nachricht----- > Von: use-livecode-bounces at lists.runrev.com [mailto:use-livecode- > bounces at lists.runrev.com] Im Auftrag von Mark Schonewille > Gesendet: Mittwoch, 20. Juli 2011 01:26 > An: How to use LiveCode > Betreff: Re: ANN: LiveCodeErrors for iPhone > > Sure, Richard, it is clear that you won't buy that app. Others will, already > did, and I'm glad I was able to provide them with something useful. > > -- > Best regards, > > Mark Schonewille > > Economy-x-Talk Consulting and Software Engineering > Homepage: http://economy-x-talk.com > Twitter: http://twitter.com/xtalkprogrammer > KvK: 50277553 > > New: Download the Installer Maker Plugin 1.6 for LiveCode here > http://qery.us/ce > > On 20 jul 2011, at 01:21, Richard Gaskin wrote: > > > Mark Schonewille wrote: > > > >> Hi Chipp, > >> > >> Weird. I would expect all pros to know about it. > >> > >> I'd say, give it a try. execute the following script > >> > >> on foo > >> try > >> executeNonExistingHandler > >> catch myErr > >> put myErr > >> end try > >> end foo > > > > I doubt many pros would need a lookup to figure out that a non-existent > handler would result in a "can't find handler" error. ;) > > > > But seriously, most devs using try/catch setups won't be executing random > code, they'll be working on things that are somewhat well defined, and use > their own error messages to report to the user. I don't know of any dev who > report RunRev's messages to their users; they just don't provide enough > guidance for the user. > > > > Those so inclined can just use the same list everything else in the LiveCode > world uses, the one RunRev provides for us in their revErrorDisplay stack: > > > > get line (item 1 of tErr) of the cErrorsList of cd 1 \ > > of stack "revErrorDisplay" into tList > > > > > > Section 10.4 ("Custom Error handling") of the LiveCode User Guide covers > this well. > > > > -- > > Richard Gaskin > > Fourth World > > LiveCode training and consulting: http://www.fourthworld.com > > Webzine for LiveCode developers: http://www.LiveCodeJournal.com > > LiveCode Journal blog: http://LiveCodejournal.com/blog.irv > > > > _______________________________________________ > > use-livecode mailing list > > use-livecode at lists.runrev.com > > Please visit this url to subscribe, unsubscribe and manage your subscription > preferences: > > http://lists.runrev.com/mailman/listinfo/use-livecode > > > _______________________________________________ > use-livecode mailing list > use-livecode at lists.runrev.com > Please visit this url to subscribe, unsubscribe and manage your subscription > preferences: > http://lists.runrev.com/mailman/listinfo/use-livecode From andre.bisseret at wanadoo.fr Wed Jul 20 06:35:41 2011 From: andre.bisseret at wanadoo.fr (=?iso-8859-1?Q?Andr=E9_Bisseret?=) Date: Wed, 20 Jul 2011 12:35:41 +0200 Subject: age calculation on Windows vs Mac In-Reply-To: <1311148177376-3680176.post@n4.nabble.com> References: <3A4F01A5-D32C-4D89-ADA5-9DB3E7FC4AA1@wanadoo.fr> <1311148177376-3680176.post@n4.nabble.com> Message-ID: <5373D644-BEA1-4FBC-B05E-48E9E4C9CC8B@wanadoo.fr> Le 20 juil. 2011 ? 09:49, SparkOut a ?crit : > As well as changing your handler, which is unfortunately necessary at the > moment, it is a pity indeed ; I am wasting a lot of time :-(( > if you have access to the QCC you could vote for bug #4941 and/or > message support and bring awareness back to this OK I am doing that today > . It's not that Livecode > "can't" determine pre epoch dates on Windows - it's just that the > implementation needs to be changed in order for it to happen. OK I thank you very much SpartOut Andr? From roger.e.eller at sealedair.com Wed Jul 20 07:56:09 2011 From: roger.e.eller at sealedair.com (Roger Eller) Date: Wed, 20 Jul 2011 07:56:09 -0400 Subject: Countdown HH:MM:SS In-Reply-To: <61EE076F-3EE3-42D4-A552-2ADF6526034A@sonsothunder.com> References: <8CE14A04637B482-1AA4-201D3@webmail-d058.sysops.aol.com> <61EE076F-3EE3-42D4-A552-2ADF6526034A@sonsothunder.com> Message-ID: On Wed, Jul 20, 2011 at 2:12 AM, Ken Ray wrote: > > On Jul 19, 2011, at 10:00 PM, DunbarX at aol.com wrote: > > > Hi. > > > > > > Shorter? It already can be pretty short. MUCH shorter? Where's Colin? > > Here's the fastest way I know: > > on mouseUp > DisplayTime > end mouseUp > > on DisplayTime > put the long time into tTime > convert tTime to dateItems > split tTime by "," > put format("%02d:%02d:%02d",tTime[4],tTime[5],tTime[6]) into fld "Time" > send "DisplayTime" to me in 1 second > end DisplayTime > > > Ken Ray > Sons of Thunder Software, Inc. > Email: kray at sonsothunder.com > Web Site: http://www.sonsothunder.com/ > > Using the dateItems is definitely the right direction, but my objective is to count backwards from for example; 2 hours, 45 minutes, and 59 seconds while updating the display field every second. When the countdown reaches 00:00:00, I would play a sound of do other actions. ?Roger From andre.bisseret at wanadoo.fr Wed Jul 20 08:25:07 2011 From: andre.bisseret at wanadoo.fr (=?iso-8859-1?Q?Andr=E9_Bisseret?=) Date: Wed, 20 Jul 2011 14:25:07 +0200 Subject: age calculation on Windows vs Mac In-Reply-To: <1311148177376-3680176.post@n4.nabble.com> References: <3A4F01A5-D32C-4D89-ADA5-9DB3E7FC4AA1@wanadoo.fr> <1311148177376-3680176.post@n4.nabble.com> Message-ID: <8A196B3F-805F-4A5A-BFFF-F15BAEEAC7AC@wanadoo.fr> Le 20 juil. 2011 ? 09:49, SparkOut a ?crit : > As well as changing your handler, which is unfortunately necessary at the > moment, if you have access to the QCC you could vote for bug #4941 Done; in fact, I discovered that I had already put 5 votes for this bug, quite a long time ago ;-)) > and/or > message support and bring awareness back to this. Done Best Andr? > It's not that Livecode > "can't" determine pre epoch dates on Windows - it's just that the > implementation needs to be changed in order for it to happen. > From bdrunrev at gmail.com Wed Jul 20 09:05:08 2011 From: bdrunrev at gmail.com (Bernard Devlin) Date: Wed, 20 Jul 2011 14:05:08 +0100 Subject: ANN: LiveCodeErrors for iPhone In-Reply-To: <4E261189.9050009@fourthworld.com> References: <2E567827-7552-4FF2-BDFC-446D6330FB6B@economy-x-talk.com> <4E261189.9050009@fourthworld.com> Message-ID: Whilst I don't have a use for this incarnation of Mark's app (I'm avoiding the new Apple ecosystem as much as possible), I've used his web service for looking up such errors, and I bought a copy of his library errorLib years ago for the same purpose. It's kind of weird that some of us should get such errors on an almost daily basis, yet others never get them. Just goes to show, different folks have different programming practices. I would think that after a decade of using Livecode I too count as a "pro", and I am very grateful for the existence of Mark's work in this area. Such bizarre error codes do manifest themselves, and they do not just happen through the contrived example that Mark provided -- and even then, without being able to lookup the meaning of the error codes, one would not necessarily know that a non-existent handler is where the error lies in the specific example that Mark gives (it could be that the executeNoneExistingHandler had a misspelling). Yesterday I had a problem occur in a far more prosaic scenario. A loop's end condition was controlled by the value in a field. The field was only 1 line high, and inadvertently two numbers were entered in the field, thus the code was failing as the loop end condition failed (it was no longer a number but two lines of numbers). Because this occured within a try/catch, I received only the obscure error numbers. (This was not in a production app, just something I was working on within the IDE, hence no real attempt at type-checking the data.) There was nothing immediately obvious to me that was wrong with the loop, and I hadn't noticed the tiny little magnifiying glass in the IDE variables pane that should have indicated to me that there was more to the variable containing the loop end condition than a mere integer (a quick glance at the variables pane showed that the loop end condition was a number). A quick shot to Mark's online service, and I got the meaning of the error code and could immediately see where the problem was. I am certain Mark is providing his library, his web service, and now his iPhone app in good faith. Like you, Mark has invested a huge portion of his professional career in Hypertext/Livecode. You are both people who set about trying to spread the love of Livecode, and I'm exceedingly grateful to both of you. Bernard On Wed, Jul 20, 2011 at 12:21 AM, Richard Gaskin wrote: >> on foo >> try >> executeNonExistingHandler >> catch myErr >> put myErr >> end try >> end foo > > I doubt many pros would need a lookup to figure out that a non-existent > handler would result in a "can't find handler" error. ;) > > But seriously, most devs using try/catch setups won't be executing random > code, they'll be working on things that are somewhat well defined, and use > their own error messages to report to the user. I don't know of any dev who > report RunRev's messages to their users; they just don't provide enough > guidance for the user. From chipp at chipp.com Wed Jul 20 10:02:10 2011 From: chipp at chipp.com (Chipp Walters) Date: Wed, 20 Jul 2011 09:02:10 -0500 Subject: ANN: LiveCodeErrors for iPhone In-Reply-To: <2E567827-7552-4FF2-BDFC-446D6330FB6B@economy-x-talk.com> References: <215B0FE5-038A-47C7-9F24-B9D29CEDD816@economy-x-talk.com> <4E25A8E0.6090704@fourthworld.com> <2E567827-7552-4FF2-BDFC-446D6330FB6B@economy-x-talk.com> Message-ID: Mark, Thanks for your generous offer, but frankly, since I don't recall seeing those errors, I'm sure Richard's technique and your web application will suffice for now. I suspect I may have actually run across them, but instantly know what the problem is- or have just gotten used to ignoring them. That said, I can't recall ever having a customer send me a bug report with mention of them in it. Best, Chipp On Tue, Jul 19, 2011 at 5:35 PM, Mark Schonewille < m.schonewille at economy-x-talk.com> wrote: > Hi Chipp, > > Weird. I would expect all pros to know about it. > > I'd say, give it a try. execute the following script > > on foo > try > executeNonExistingHandler > catch myErr > put myErr > end try > end foo > > The first item in the message box will be a number. Download my app from > http://qery.us/v4 and enter the number in the app. It'll tell you that the > handler executeNonExistingHandler could not be found. If you contact me > off-list, I'll send you a promo code. > > -- > Best regards, > > Mark Schonewille > > Economy-x-Talk Consulting and Software Engineering > Homepage: http://economy-x-talk.com > Twitter: http://twitter.com/xtalkprogrammer > KvK: 50277553 > > New: Download the Installer Maker Plugin 1.6 for LiveCode here > http://qery.us/ce > > On 19 jul 2011, at 18:58, Chipp Walters wrote: > > > Interesting. > > > > I have to say, in the past 10 years or so since I've been programming > > commercial applications for my clients and our company, I do not recall > > seeing such an error. In fact, this is the first time I've even heard of > > them. > > > > I, too, would like to know under what circumstances they are fired. It > seems > > like a bug to me in LC's error handling code if they continually pop up. > I > > suspect Richard knew about this from all his work with the MetaCard IDE. > > > _______________________________________________ > use-livecode mailing list > use-livecode at lists.runrev.com > Please visit this url to subscribe, unsubscribe and manage your > subscription preferences: > http://lists.runrev.com/mailman/listinfo/use-livecode > -- Chipp Walters CEO, Shafer Walters Group, Inc. From m.schonewille at economy-x-talk.com Wed Jul 20 10:19:38 2011 From: m.schonewille at economy-x-talk.com (Mark Schonewille) Date: Wed, 20 Jul 2011 16:19:38 +0200 Subject: ANN: LiveCodeErrors for iPhone In-Reply-To: References: <215B0FE5-038A-47C7-9F24-B9D29CEDD816@economy-x-talk.com> <4E25A8E0.6090704@fourthworld.com> <2E567827-7552-4FF2-BDFC-446D6330FB6B@economy-x-talk.com> Message-ID: Chipp, True, your customer won't and shouldn't see those errors, unless you display them on purpose. -- Best regards, Mark Schonewille Economy-x-Talk Consulting and Software Engineering Homepage: http://economy-x-talk.com Twitter: http://twitter.com/xtalkprogrammer KvK: 50277553 New: Download the Installer Maker Plugin 1.6 for LiveCode here http://qery.us/ce On 20 jul 2011, at 16:02, Chipp Walters wrote: > Mark, > > Thanks for your generous offer, but frankly, since I don't recall seeing > those errors, I'm sure Richard's technique and your web application will > suffice for now. I suspect I may have actually run across them, but > instantly know what the problem is- or have just gotten used to ignoring > them. That said, I can't recall ever having a customer send me a bug report > with mention of them in it. > > Best, > > Chipp From revolution at derbrill.de Wed Jul 20 10:22:48 2011 From: revolution at derbrill.de (Malte Brill) Date: Wed, 20 Jul 2011 16:22:48 +0200 Subject: ANN: LiveCodeErrors for iPhone Message-ID: <8DD263FD-4C40-4801-ACB9-1F000BF06495@derbrill.de> > True, your customer won't and shouldn't see those errors, unless you display them on purpose. Which I frankly do quite often. :-) (Not that my software ever has errors... mwahahaha) Malte From andrew at rjdfarm.com Wed Jul 20 10:36:40 2011 From: andrew at rjdfarm.com (Andrew Kluthe) Date: Wed, 20 Jul 2011 07:36:40 -0700 (PDT) Subject: LiveCode Server Release and On-Rev In-Reply-To: <848DF372-D3F3-4658-8A13-14D4C7852B67@sonsothunder.com> References: <848DF372-D3F3-4658-8A13-14D4C7852B67@sonsothunder.com> Message-ID: <1311172600076-3680996.post@n4.nabble.com> Heather says On-Rev users will be upgraded to the new livecode server in a few days. -- View this message in context: http://runtime-revolution.278305.n4.nabble.com/LiveCode-Server-Release-and-On-Rev-tp3679789p3680996.html Sent from the Revolution - User mailing list archive at Nabble.com. From jacque at hyperactivesw.com Wed Jul 20 10:59:04 2011 From: jacque at hyperactivesw.com (J. Landman Gay) Date: Wed, 20 Jul 2011 09:59:04 -0500 Subject: ANN: LiveCodeErrors for iPhone In-Reply-To: <2E567827-7552-4FF2-BDFC-446D6330FB6B@economy-x-talk.com> References: <215B0FE5-038A-47C7-9F24-B9D29CEDD816@economy-x-talk.com> <4E25A8E0.6090704@fourthworld.com> <2E567827-7552-4FF2-BDFC-446D6330FB6B@economy-x-talk.com> Message-ID: <4E26ED38.2020109@hyperactivesw.com> I also have a free web interface for error lookup. But what bothers me is that everyone who has LiveCode already has a copy of the error list, extracting it to a field takes one line of script in the message box, and the lookup script is two lines of code. Even the most basic newcomer could do this without much trouble. Charging money for something that is already free and available appears to take advantage of the naivete of new users who don't know that. While that's pretty common in the general public, in our community it isn't. -- Jacqueline Landman Gay | jacque at hyperactivesw.com HyperActive Software | http://www.hyperactivesw.com From andre.bisseret at wanadoo.fr Wed Jul 20 11:04:34 2011 From: andre.bisseret at wanadoo.fr (=?iso-8859-1?Q?Andr=E9_Bisseret?=) Date: Wed, 20 Jul 2011 17:04:34 +0200 Subject: age calculation on Windows vs Mac In-Reply-To: <1311148177376-3680176.post@n4.nabble.com> References: <3A4F01A5-D32C-4D89-ADA5-9DB3E7FC4AA1@wanadoo.fr> <1311148177376-3680176.post@n4.nabble.com> Message-ID: <0B43FE20-7D6A-4BC4-B01E-8C2A2B8A8000@wanadoo.fr> Bonjour, After following the quoted adv!ces from SparkOut, I found a handler given in 2006 by Peter Brigham on nabble; it is running well on PC (it avoid the 1970 barrier!) Thank you Peter ;-)) When age < 4 (baby), I need to get it in month so I keep the handler from Jim Ault (thank you Jim ;-)) So that I have something like : set the itemDelimiter to slash if item 3 of birthdate < 1970 then -- Peter' handler else --Jim' handler end if my stack script is beginning to be like "une usine ? gaz" as we say in french ("a gas factory"? don't know the english equivalent ;-)) Hope some changes in LC, so that it be possible to get it more simple! Thanks again to those who replied to my first post on this topic Best to all Andr? Le 20 juil. 2011 ? 09:49, SparkOut a ?crit : > As well as changing your handler, which is unfortunately necessary at the > moment, if you have access to the QCC you could vote for bug #4941 and/or > message support and bring awareness back to this. It's not that Livecode > "can't" determine pre epoch dates on Windows - it's just that the > implementation needs to be changed in order for it to happen. From m.schonewille at economy-x-talk.com Wed Jul 20 11:10:57 2011 From: m.schonewille at economy-x-talk.com (Mark Schonewille) Date: Wed, 20 Jul 2011 17:10:57 +0200 Subject: ANN: LiveCodeErrors for iPhone In-Reply-To: <4E26ED38.2020109@hyperactivesw.com> References: <215B0FE5-038A-47C7-9F24-B9D29CEDD816@economy-x-talk.com> <4E25A8E0.6090704@fourthworld.com> <2E567827-7552-4FF2-BDFC-446D6330FB6B@economy-x-talk.com> <4E26ED38.2020109@hyperactivesw.com> Message-ID: <43BCFD29-5011-4B84-B815-B5FD64FFE392@economy-x-talk.com> Hi Jacque, I really believe you should write people off-list, when you accuse them. I still believe I made a useful app, since I use it myself. -- Best regards, Mark Schonewille Economy-x-Talk Consulting and Software Engineering Homepage: http://economy-x-talk.com Twitter: http://twitter.com/xtalkprogrammer KvK: 50277553 New: Download the Installer Maker Plugin 1.6 for LiveCode here http://qery.us/ce On 20 jul 2011, at 16:59, J. Landman Gay wrote: > I also have a free web interface for error lookup. But what bothers me is that everyone who has LiveCode already has a copy of the error list, extracting it to a field takes one line of script in the message box, and the lookup script is two lines of code. Even the most basic newcomer could do this without much trouble. > > Charging money for something that is already free and available appears to take advantage of the naivete of new users who don't know that. While that's pretty common in the general public, in our community it isn't. > > -- > Jacqueline Landman Gay | jacque at hyperactivesw.com > HyperActive Software | http://www.hyperactivesw.com From roger.e.eller at sealedair.com Wed Jul 20 11:13:08 2011 From: roger.e.eller at sealedair.com (Roger Eller) Date: Wed, 20 Jul 2011 11:13:08 -0400 Subject: ANN: LiveCodeErrors for iPhone In-Reply-To: <4E26ED38.2020109@hyperactivesw.com> References: <215B0FE5-038A-47C7-9F24-B9D29CEDD816@economy-x-talk.com> <4E25A8E0.6090704@fourthworld.com> <2E567827-7552-4FF2-BDFC-446D6330FB6B@economy-x-talk.com> <4E26ED38.2020109@hyperactivesw.com> Message-ID: On Wed, Jul 20, 2011 at 10:59 AM, J. Landman Gay wrote: > I also have a free web interface for error lookup. But what bothers me is > that everyone who has LiveCode already has a copy of the error list, > extracting it to a field takes one line of script in the message box, and > the lookup script is two lines of code. Even the most basic newcomer could > do this without much trouble. > > Charging money for something that is already free and available appears to > take advantage of the naivete of new users who don't know that. While that's > pretty common in the general public, in our community it isn't. > > -- > Jacqueline Landman Gay | jacque at hyperactivesw.com > HyperActive Software | http://www.hyperactivesw.com > > This is an awesome community, and a generous community when it comes to sharing knowledge to help each other along. Your statement is well said Jacque! Bravo! ~Roger From m.schonewille at economy-x-talk.com Wed Jul 20 11:21:26 2011 From: m.schonewille at economy-x-talk.com (Mark Schonewille) Date: Wed, 20 Jul 2011 17:21:26 +0200 Subject: ANN: LiveCodeErrors for iPhone In-Reply-To: References: <215B0FE5-038A-47C7-9F24-B9D29CEDD816@economy-x-talk.com> <4E25A8E0.6090704@fourthworld.com> <2E567827-7552-4FF2-BDFC-446D6330FB6B@economy-x-talk.com> <4E26ED38.2020109@hyperactivesw.com> Message-ID: I think it is really mean what you guys are doing. I have taken all free LiveCode utilities and libraries off-line. From now on, you will have to pay if you want to use something. -- Best regards, Mark Schonewille Economy-x-Talk Consulting and Software Engineering Homepage: http://economy-x-talk.com Twitter: http://twitter.com/xtalkprogrammer KvK: 50277553 New: Download the Installer Maker Plugin 1.6 for LiveCode here http://qery.us/ce On 20 jul 2011, at 17:13, Roger Eller wrote: > On Wed, Jul 20, 2011 at 10:59 AM, J. Landman Gay wrote: > >> I also have a free web interface for error lookup. But what bothers me is >> that everyone who has LiveCode already has a copy of the error list, >> extracting it to a field takes one line of script in the message box, and >> the lookup script is two lines of code. Even the most basic newcomer could >> do this without much trouble. >> >> Charging money for something that is already free and available appears to >> take advantage of the naivete of new users who don't know that. While that's >> pretty common in the general public, in our community it isn't. >> >> -- >> Jacqueline Landman Gay | jacque at hyperactivesw.com >> HyperActive Software | http://www.hyperactivesw.com >> >> > This is an awesome community, and a generous community when it comes to > sharing knowledge to help each other along. Your statement is well said > Jacque! Bravo! > > ~Roger > _______________________________________________ > use-livecode mailing list > use-livecode at lists.runrev.com > Please visit this url to subscribe, unsubscribe and manage your subscription preferences: > http://lists.runrev.com/mailman/listinfo/use-livecode From jhurley0305 at sbcglobal.net Wed Jul 20 11:29:45 2011 From: jhurley0305 at sbcglobal.net (James Hurley) Date: Wed, 20 Jul 2011 08:29:45 -0700 Subject: Countdown HH:MM:SS In-Reply-To: References: Message-ID: <04539BD7-007A-4621-BFA9-64DCB8F81169@sbcglobal.net> Roger Is this what you are looking for? on mouseUp set itemdel to ":" put the long time into tTime put char 1 to -4 of tTime into field 1 convert tTime to dateItems set the hms of me to tTime countDown end mouseUp on countDown put the hms of me into temp subtract 1 from item -2 of temp --The dateItems will take care of the negative seconds convert temp to dateItems set the hms of me to temp put temp into tDisplayTIme put item -4 to -2 of tDisplayTIme into temp2 replace "," with ":" in temp2 put temp2 into field 1 send countDown to me in 1 sec --Change the "1 sec" to "1 Minute" when you are sure it works for you. end countDown Jim Hurley From andre at andregarzia.com Wed Jul 20 11:45:57 2011 From: andre at andregarzia.com (Andre Garzia) Date: Wed, 20 Jul 2011 12:45:57 -0300 Subject: ANN: LiveCodeErrors for iPhone In-Reply-To: References: <215B0FE5-038A-47C7-9F24-B9D29CEDD816@economy-x-talk.com> <4E25A8E0.6090704@fourthworld.com> <2E567827-7552-4FF2-BDFC-446D6330FB6B@economy-x-talk.com> <4E26ED38.2020109@hyperactivesw.com> Message-ID: irgh, Hey Guys, Is it friday 13th here? What is happening? Mark made an mobile application that he likes and some other people find useful, props to him. Others don't see the need, that is ok. Mark is not forcing anyone to buy anything, actually, for someone to want his app, this person needs to know more than day-1 livecode, it needs to know about LiveCode error codes. I see the utility in his application. In some stuff I did, I allow exceptions to bubble up in the following manner: I try to figure out what is happening and if it is recoverable, if it is not, then, I present my own error message followed by the error codes. Also if you are doing LiveCode Server, sometimes, the engine will spit out error codes, I don't have a recipe but it happened to me before. I know enough LiveCode to have my own systems to deal with it but none of my systems work on mobile. I can see someone not in front of his machine, using this. I don't think it is fair to accuse Mark of exploring the naivete of people. He is just offering a mobile app for some bucks. For someone that does not have a RevMobile license and thus can't recreate those two lines, this might be useful. I think there is a terrible miscommunication going on here. Richard wanted to know if there was any standard case where the errordialog from LiveCode would fail to display human readable errors. He wanted that because he wanted to file a QA report at the quality center since LiveCode own dialogbox should handle everything. Mark created an application for the cases where the error codes are not passing thru LiveCode dialogboxes (maybe their are caught in a try/catch block) and the developer did not pick the errors from the list. There is no harm here guys. For errors people always had options: * Let livecode error dialog translate them * use one of the web front ends available (Mark had one, Jacque had one) * rolll your own * don't care, panic, rollback and recover (<-- my personal tools are like this) Now what started at as a very good thread where one developer launched a product and another developer asked if there was a case where the built-in tools failed, derailed into a very bad thread that will soon be locked because the internet is not meant for arguments but for discussions. come on guys, we just launched 4.6.3. The new LiveCode Server is around the corner and many scripts need to be patched. Mobile is growing and 2012 is the Year of the LiveCode RevLet!!!!! (I Want To Believe). We have better things to do then to keep this thread on. Products are like standards, when you don't like them, you just don't use them. Can we go back to a normal sunny day in Rio? From matthias_livecode at me.com Wed Jul 20 11:53:03 2011 From: matthias_livecode at me.com (Matthias Rebbe) Date: Wed, 20 Jul 2011 17:53:03 +0200 Subject: ANN: LiveCodeErrors for iPhone In-Reply-To: <4E26ED38.2020109@hyperactivesw.com> References: <215B0FE5-038A-47C7-9F24-B9D29CEDD816@economy-x-talk.com> <4E25A8E0.6090704@fourthworld.com> <2E567827-7552-4FF2-BDFC-446D6330FB6B@economy-x-talk.com> <4E26ED38.2020109@hyperactivesw.com> Message-ID: Hi, although i understand what you mean. I cannot completely agree. Nobody is forced to buy it. Mark made an iOS tool which can be used offline on the iOS devices. He spent time and effort into it. So why not selling? Everyone is free to use the free webInterfaces. But if one wants to buy, just let him/her without talking him/her out of it. Just my 2cents. Regards, Matthias Am 20.07.2011 um 16:59 schrieb J. Landman Gay: > I also have a free web interface for error lookup. But what bothers me is that everyone who has LiveCode already has a copy of the error list, extracting it to a field takes one line of script in the message box, and the lookup script is two lines of code. Even the most basic newcomer could do this without much trouble. > > Charging money for something that is already free and available appears to take advantage of the naivete of new users who don't know that. While that's pretty common in the general public, in our community it isn't. > > -- > Jacqueline Landman Gay | jacque at hyperactivesw.com > HyperActive Software | http://www.hyperactivesw.com > > _______________________________________________ > use-livecode mailing list > use-livecode at lists.runrev.com > Please visit this url to subscribe, unsubscribe and manage your subscription preferences: > http://lists.runrev.com/mailman/listinfo/use-livecode From jhurley0305 at sbcglobal.net Wed Jul 20 11:55:03 2011 From: jhurley0305 at sbcglobal.net (James Hurley) Date: Wed, 20 Jul 2011 08:55:03 -0700 Subject: Countdown HH:MM:SS In-Reply-To: References: Message-ID: Roger, I forgot the stop clause. Maybe: if field 1 is "0:0:0" then --or whatever --Do your business else send countDown to me in 1 sec end if > Roger > > Is this what you are looking for? > > on mouseUp > set itemdel to ":" > put the long time into tTime > put char 1 to -4 of tTime into field 1 > convert tTime to dateItems > set the hms of me to tTime > countDown > end mouseUp > > on countDown > put the hms of me into temp > subtract 1 from item -2 of temp > --The dateItems will take care of the negative seconds > convert temp to dateItems > set the hms of me to temp > put temp into tDisplayTIme > put item -4 to -2 of tDisplayTIme into temp2 > replace "," with ":" in temp2 > put temp2 into field 1 > send countDown to me in 1 sec > --Change the "1 sec" to "1 Minute" when you are sure it works for you. > end countDown > > Jim Hurley > From jperryl at ecs.fullerton.edu Wed Jul 20 11:56:32 2011 From: jperryl at ecs.fullerton.edu (Judy Perry) Date: Wed, 20 Jul 2011 08:56:32 -0700 (PDT) Subject: ANN: LiveCodeErrors for iPhone In-Reply-To: <4E26ED38.2020109@hyperactivesw.com> References: <215B0FE5-038A-47C7-9F24-B9D29CEDD816@economy-x-talk.com> <4E25A8E0.6090704@fourthworld.com> <2E567827-7552-4FF2-BDFC-446D6330FB6B@economy-x-talk.com> <4E26ED38.2020109@hyperactivesw.com> Message-ID: I've followed this thread with some interest as it is something I intended to purchase once my vacation here in Florida is done. What amazes, no, horrifies me is the pure vitriol and outright personal nastiness that has greeted this nice product. Sure, the rest of you Titans could figure out this stuff pretty easily. Every time I get one of those things I just throw up my hands and give up. Because I have neither the time nor the inclination to go looking 4,000 different places to find out why something doesn't work/what's the way to make it work/ is it a bucket, a mop, a lesson, a tutorial, a blog posting, something mentioned in passing somewhere in 10+ years of use-list postings, on the forums that are always asking for some other password I can never remember, etc. etc. etc. It's not taking advantage of people's naivete to put arcane stuff together in once place for a reasonable price. It's what's called a SERVICE. Why not just tell all the beginners/non-programmers to blow off and go use HyperStudio? IS THAT WHAT YOU ALL REALLY WANT? Because that's what you're doing. You don't need it? Don't buy it. All kinds of people don't need Scott Rossi's stuff either or any number of other people who sell stuff to the community, but nobody ever craps all over those people. All of you people doing the complaining -- how many sessions have YOU done for Livecode.tv? I'm betting zero. How many have you organized? How many have you troubleshot? Zero and zero. How surprising. :-( Judy On Wed, 20 Jul 2011, J. Landman Gay wrote: > I also have a free web interface for error lookup. But what bothers me is > that everyone who has LiveCode already has a copy of the error list, > extracting it to a field takes one line of script in the message box, and the > lookup script is two lines of code. Even the most basic newcomer could do > this without much trouble. > > Charging money for something that is already free and available appears to > take advantage of the naivete of new users who don't know that. While that's > pretty common in the general public, in our community it isn't. From matthias_livecode at me.com Wed Jul 20 11:58:16 2011 From: matthias_livecode at me.com (Matthias Rebbe) Date: Wed, 20 Jul 2011 17:58:16 +0200 Subject: ANN: LiveCodeErrors for iPhone In-Reply-To: References: <215B0FE5-038A-47C7-9F24-B9D29CEDD816@economy-x-talk.com> <4E25A8E0.6090704@fourthworld.com> <2E567827-7552-4FF2-BDFC-446D6330FB6B@economy-x-talk.com> <4E26ED38.2020109@hyperactivesw.com> Message-ID: Well spoken, Andre ! Am 20.07.2011 um 17:45 schrieb Andre Garzia: > irgh, > > Hey Guys, > > Is it friday 13th here? What is happening? Mark made an mobile application > that he likes and some other people find useful, props to him. Others don't > see the need, that is ok. Mark is not forcing anyone to buy anything, > actually, for someone to want his app, this person needs to know more than > day-1 livecode, it needs to know about LiveCode error codes. > > I see the utility in his application. In some stuff I did, I allow > exceptions to bubble up in the following manner: I try to figure out what is > happening and if it is recoverable, if it is not, then, I present my own > error message followed by the error codes. Also if you are doing LiveCode > Server, sometimes, the engine will spit out error codes, I don't have a > recipe but it happened to me before. > > I know enough LiveCode to have my own systems to deal with it but none of my > systems work on mobile. I can see someone not in front of his machine, using > this. > > I don't think it is fair to accuse Mark of exploring the naivete of people. > He is just offering a mobile app for some bucks. For someone that does not > have a RevMobile license and thus can't recreate those two lines, this might > be useful. > > I think there is a terrible miscommunication going on here. Richard wanted > to know if there was any standard case where the errordialog from LiveCode > would fail to display human readable errors. He wanted that because he > wanted to file a QA report at the quality center since LiveCode own > dialogbox should handle everything. Mark created an application for the > cases where the error codes are not passing thru LiveCode dialogboxes (maybe > their are caught in a try/catch block) and the developer did not pick the > errors from the list. There is no harm here guys. > > For errors people always had options: > * Let livecode error dialog translate them > * use one of the web front ends available (Mark had one, Jacque had one) > * rolll your own > * don't care, panic, rollback and recover (<-- my personal tools are like > this) > > Now what started at as a very good thread where one developer launched a > product and another developer asked if there was a case where the built-in > tools failed, derailed into a very bad thread that will soon be locked > because the internet is not meant for arguments but for discussions. > > come on guys, we just launched 4.6.3. The new LiveCode Server is around the > corner and many scripts need to be patched. Mobile is growing and 2012 is > the Year of the LiveCode RevLet!!!!! (I Want To Believe). > > We have better things to do then to keep this thread on. Products are like > standards, when you don't like them, you just don't use them. Can we go back > to a normal sunny day in Rio? > _______________________________________________ > use-livecode mailing list > use-livecode at lists.runrev.com > Please visit this url to subscribe, unsubscribe and manage your subscription preferences: > http://lists.runrev.com/mailman/listinfo/use-livecode From jacque at hyperactivesw.com Wed Jul 20 12:02:26 2011 From: jacque at hyperactivesw.com (J. Landman Gay) Date: Wed, 20 Jul 2011 11:02:26 -0500 Subject: ANN: LiveCodeErrors for iPhone In-Reply-To: References: <215B0FE5-038A-47C7-9F24-B9D29CEDD816@economy-x-talk.com> <4E25A8E0.6090704@fourthworld.com> <2E567827-7552-4FF2-BDFC-446D6330FB6B@economy-x-talk.com> <4E26ED38.2020109@hyperactivesw.com> Message-ID: <4E26FC12.50201@hyperactivesw.com> I apologize for the public posting. I agree that anyone can sell anything they like, and anyone can buy anything they like. End of thread for me. -- Jacqueline Landman Gay | jacque at hyperactivesw.com HyperActive Software | http://www.hyperactivesw.com From kevin at runrev.com Wed Jul 20 12:05:25 2011 From: kevin at runrev.com (Kevin Miller) Date: Wed, 20 Jul 2011 17:05:25 +0100 Subject: ANN: LiveCodeErrors for iPhone In-Reply-To: Message-ID: Hi folks, I have every confidence in the free market to determine whether or not people find this tool useful. I'm certainly not going to make a judgement either way and I know that we have so many different types of people using LC these days. Lets drop this thread now, please. Kind regards, Kevin Kevin Miller ~ kevin at runrev.com ~ http://www.runrev.com/ LiveCode: Compile-free coding, the faster path to better apps From m.schonewille at economy-x-talk.com Wed Jul 20 12:15:16 2011 From: m.schonewille at economy-x-talk.com (Mark Schonewille) Date: Wed, 20 Jul 2011 18:15:16 +0200 Subject: ANN: LiveCodeErrors for iPhone In-Reply-To: <4E26FC12.50201@hyperactivesw.com> References: <215B0FE5-038A-47C7-9F24-B9D29CEDD816@economy-x-talk.com> <4E25A8E0.6090704@fourthworld.com> <2E567827-7552-4FF2-BDFC-446D6330FB6B@economy-x-talk.com> <4E26ED38.2020109@hyperactivesw.com> <4E26FC12.50201@hyperactivesw.com> Message-ID: <8D90BD61-4E6E-4D4B-96A8-9ECC60BF66C8@economy-x-talk.com> Thank you, Jacque. -- Best regards, Mark Schonewille Economy-x-Talk Consulting and Software Engineering Homepage: http://economy-x-talk.com Twitter: http://twitter.com/xtalkprogrammer KvK: 50277553 New: Download the Installer Maker Plugin 1.6 for LiveCode here http://qery.us/ce On 20 jul 2011, at 18:02, J. Landman Gay wrote: > I apologize for the public posting. I agree that anyone can sell anything they like, and anyone can buy anything they like. > > End of thread for me. > > -- > Jacqueline Landman Gay | jacque at hyperactivesw.com > HyperActive Software | http://www.hyperactivesw.com From bobs at twft.com Wed Jul 20 12:18:20 2011 From: bobs at twft.com (Bob Sneidar) Date: Wed, 20 Jul 2011 09:18:20 -0700 Subject: externals error under linux In-Reply-To: References: Message-ID: No but I LOVE the word "wimplicit"!! I am going to have to work that into a sentence soon. Bob On Jul 19, 2011, at 8:35 PM, Andre Garzia wrote: > Folks, > > Anyone seen this: > > warning: implicit declaration of function ?getXtable? > [-Wimplicit-function-declaration] > > It is related to some macro error I think, or the linker is not linking > against the correct stuff... > > any clue? > > -- > http://www.andregarzia.com All We Do Is Code. > _______________________________________________ > use-livecode mailing list > use-livecode at lists.runrev.com > Please visit this url to subscribe, unsubscribe and manage your subscription preferences: > http://lists.runrev.com/mailman/listinfo/use-livecode From bobs at twft.com Wed Jul 20 12:21:29 2011 From: bobs at twft.com (Bob Sneidar) Date: Wed, 20 Jul 2011 09:21:29 -0700 Subject: age calculation on Windows vs Mac In-Reply-To: References: Message-ID: <5E93AEDF-9294-4C6F-9865-87F940B9B264@twft.com> I purchased Infowallet, (it's awesome) and so feel it's alright to scavenge your bit of code. :-) Bob On Jul 19, 2011, at 4:24 PM, Bill Vlahos wrote: > The problem is the epoch date in Windows is 1970 and LiveCode can't deal with dates earlier than that in Windows. You will need to use a VBScript. > > Here is what I use in InfoWallet which handles Windows, Mac, and Linux correctly which was based on code provided by folks on this list. > > on calcAge tDate,asOf > -- returns the current age based on a DOB of tDate > -- as of the date asOf, or if asOf = empty, as of today > if tDate = empty then return empty > if the platform is "WIn32" then > isADate tDate > if the result then > put "sdate = " & quote & tDate & quote into vFive > put return & "result = DateDiff(" & quote & "d" & quote & ",sdate,Date)" after vFive > put return & "trueage = result / 365" after vFive > put return & "result = Left(trueage,2)" after vFive > do vFive as vbscript > return the result > else > return empty > end if > else > if tDate is not a date then return empty > if asOf = empty then > put the date into asOf > end if > set the itemdelimiter to "/" > put item -1 of the short date into nowYr > set the centurycutoff to nowYr+1 > set the itemdelimiter to comma > convert tDate to dateItems > convert asOf to dateitems > put item 1 of asOf - item 1 of tDate - 1 into tAge > put item 1 of asOf into item 1 of tDate > convert tDate to seconds > convert asOf to seconds > if tDate <= asOf then add 1 to tAge > return tAge > end if > end calcAge > > > Bill Vlahos > _________________ > InfoWallet (http://www.infowallet.com) is about keeping your important life information with you, accessible, and secure. > > On Jul 19, 2011, at 9:23 AM, Andr? Bisseret wrote: > >> Bonjour, >> >> I am a Mac user but I recently bought a PC (windows 7) in order to test my standalones for windows more quickly. >> >> I am fighting for hours now with the following problem : >> >> In an app., I have a handler that calculates the age from the short birth date (thanks to Jim Ault who gave it on the list). >> The birth date is jj/mm/AAAA or jj/mm/AA as well. >> >> This handler works fine on Mac, but not on windows. >> >> On windows, the age is invariably 100 if the birth year < 1970. (with JJ/MM/AA or JJ/MM/AAAA (as bad ;-( >> If the birth year > = 1970 then the age is correct. >> >> I tried to play with the centuryCutOff but without any success (on windows). >> >> I am not familiar with Windows : what am I missing ? Is it a matter of computer setting or should I program differently for windows than for mac !? >> >> Thanks a lot in advance for any help >> >> Best regards from Grenoble >> >> Andr? >> >> P.S. By the way, I used to use a search engine (if I well remember, it was in the help menu of livecode) I do not find it any more; has it been suppressed ? (or here again, I am missing something !) >> _______________________________________________ >> use-livecode mailing list >> use-livecode at lists.runrev.com >> Please visit this url to subscribe, unsubscribe and manage your subscription preferences: >> http://lists.runrev.com/mailman/listinfo/use-livecode > > > _______________________________________________ > use-livecode mailing list > use-livecode at lists.runrev.com > Please visit this url to subscribe, unsubscribe and manage your subscription preferences: > http://lists.runrev.com/mailman/listinfo/use-livecode From bobs at twft.com Wed Jul 20 12:32:00 2011 From: bobs at twft.com (Bob Sneidar) Date: Wed, 20 Jul 2011 09:32:00 -0700 Subject: Countdown HH:MM:SS In-Reply-To: <04539BD7-007A-4621-BFA9-64DCB8F81169@sbcglobal.net> References: <04539BD7-007A-4621-BFA9-64DCB8F81169@sbcglobal.net> Message-ID: Hey as long as we are talking about dates and times, I am going to need soon a method to determine if a block of time intersects with another. Specifically I will need to do this in an SQL query. I suppose the best way would be to store all dates in julian format, then have a query that, given savedStartDate and savedEndDate are columns in a table, goes something like : select * where (startDate > savedStartDate and startDate < savedEndDate) or (endDate > savedStartDate and endDate < savedEndDate) Is that about right? Any other ideas? Bob On Jul 20, 2011, at 8:29 AM, James Hurley wrote: > Roger > > Is this what you are looking for? > > on mouseUp > set itemdel to ":" > put the long time into tTime > put char 1 to -4 of tTime into field 1 > convert tTime to dateItems > set the hms of me to tTime > countDown > end mouseUp > > on countDown > put the hms of me into temp > subtract 1 from item -2 of temp > --The dateItems will take care of the negative seconds > convert temp to dateItems > set the hms of me to temp > put temp into tDisplayTIme > put item -4 to -2 of tDisplayTIme into temp2 > replace "," with ":" in temp2 > put temp2 into field 1 > send countDown to me in 1 sec > --Change the "1 sec" to "1 Minute" when you are sure it works for you. > end countDown > > Jim Hurley > > > _______________________________________________ > use-livecode mailing list > use-livecode at lists.runrev.com > Please visit this url to subscribe, unsubscribe and manage your subscription preferences: > http://lists.runrev.com/mailman/listinfo/use-livecode From chris at altuit.com Wed Jul 20 12:36:28 2011 From: chris at altuit.com (Chris Bohnert) Date: Wed, 20 Jul 2011 11:36:28 -0500 Subject: externals error under linux In-Reply-To: References: Message-ID: Andre, This is the compiler warning you that it can't find a header prototype for the getXtable function. In this case the function (which used to be in XCmdGlue.c) will serve as the declaration and you can remove the warning by changing the compiler flag -Wimplicit-function-declaration to -Wno-implicit. In the past there were a number of ifdefs in the XCmdGlue headers that were controlling the inclusion of the different function prototypes. I haven't looked into the headers recently to make sure this is still the same. You might check them to see if you're missing a platform or language definition that is causing this warning to be thrown. -- cb On Tue, Jul 19, 2011 at 10:35 PM, Andre Garzia wrote: > Folks, > > Anyone seen this: > > warning: implicit declaration of function ?getXtable? > [-Wimplicit-function-declaration] > > It is related to some macro error I think, or the linker is not linking > against the correct stuff... > > any clue? > > -- > http://www.andregarzia.com All We Do Is Code. > _______________________________________________ > use-livecode mailing list > use-livecode at lists.runrev.com > Please visit this url to subscribe, unsubscribe and manage your > subscription preferences: > http://lists.runrev.com/mailman/listinfo/use-livecode > From andre at andregarzia.com Wed Jul 20 12:42:14 2011 From: andre at andregarzia.com (Andre Garzia) Date: Wed, 20 Jul 2011 13:42:14 -0300 Subject: externals error under linux In-Reply-To: References: Message-ID: Chris, Thank you very much! I will look into that. :-) The Linux Externals SDK is not used much, so I guess there might be bugs. :-) On Wed, Jul 20, 2011 at 1:36 PM, Chris Bohnert wrote: > Andre, > > This is the compiler warning you that it can't find a header prototype for > the getXtable function. In this case the function (which used to be in > XCmdGlue.c) will serve as the declaration and you can remove the warning by > changing the compiler flag -Wimplicit-function-declaration to > -Wno-implicit. > > In the past there were a number of ifdefs in the XCmdGlue headers that were > controlling the inclusion of the different function prototypes. I haven't > looked into the headers recently to make sure this is still the same. You > might check them to see if you're missing a platform or language definition > that is causing this warning to be thrown. > > -- > cb > > On Tue, Jul 19, 2011 at 10:35 PM, Andre Garzia >wrote: > > > Folks, > > > > Anyone seen this: > > > > warning: implicit declaration of function ?getXtable? > > [-Wimplicit-function-declaration] > > > > It is related to some macro error I think, or the linker is not linking > > against the correct stuff... > > > > any clue? > > > > -- > > http://www.andregarzia.com All We Do Is Code. > > _______________________________________________ > > use-livecode mailing list > > use-livecode at lists.runrev.com > > Please visit this url to subscribe, unsubscribe and manage your > > subscription preferences: > > http://lists.runrev.com/mailman/listinfo/use-livecode > > > _______________________________________________ > use-livecode mailing list > use-livecode at lists.runrev.com > Please visit this url to subscribe, unsubscribe and manage your > subscription preferences: > http://lists.runrev.com/mailman/listinfo/use-livecode > -- http://www.andregarzia.com All We Do Is Code. From kray at sonsothunder.com Wed Jul 20 12:46:38 2011 From: kray at sonsothunder.com (Ken Ray) Date: Wed, 20 Jul 2011 11:46:38 -0500 Subject: LiveCode Server Release and On-Rev In-Reply-To: <1311172600076-3680996.post@n4.nabble.com> References: <848DF372-D3F3-4658-8A13-14D4C7852B67@sonsothunder.com> <1311172600076-3680996.post@n4.nabble.com> Message-ID: On Jul 20, 2011, at 9:36 AM, Andrew Kluthe wrote: > Heather says On-Rev users will be upgraded to the new livecode server in a > few days. That's great to hear, but should I assume that current .irev files won't be affected by the upgrade? I just want to know if I need to change anything before the upgrade since I have some clients that are dependent on the .irev files working as usual. Ken Ray Sons of Thunder Software, Inc. Email: kray at sonsothunder.com Web Site: http://www.sonsothunder.com/ From lfredricks at proactive-intl.com Wed Jul 20 12:49:47 2011 From: lfredricks at proactive-intl.com (Lynn Fredricks) Date: Wed, 20 Jul 2011 09:49:47 -0700 Subject: Your Icon Requests for Android and iOS Development Message-ID: <15E29E64CFE644EE8A745BEDAF99CD21@GATEWAY> Hello all, A partner of mine makes the icons for IconPeople - many of you have the Valerian icon set, either purchased individually or received in a bundle. Since we now have iOS and Android deployment for LiveCode, we've been thinking about what icons would be useful to extend Valerian for use on these devices. That way, at least a modest UX experience can be had between your apps running on the desktop and the devices. I believe that both devices already handle the "roundedness" and "glossiness" aspects of icon rendering. Can you correct me if I am wrong about that? Also, I think there are good lists of supporting sizes. Are there any specific icons you'd like to see included? Best regards, Lynn Fredricks President Proactive International, LLC - Because it is about who you know.(tm) http://www.proactive-intl.com From revdev at pdslabs.net Wed Jul 20 12:54:30 2011 From: revdev at pdslabs.net (Phil Davis) Date: Wed, 20 Jul 2011 09:54:30 -0700 Subject: iPhone + Arduino In-Reply-To: <4E267A1F.1050604@pdslabs.net> References: <4E267A1F.1050604@pdslabs.net> Message-ID: <4E270846.6040203@pdslabs.net> I emailed Redpark and heard back from them: > Apple policy established the "private use only" approach. This was not > defined by Redpark. So there you have it. Phil On 7/19/11 11:47 PM, Phil Davis wrote: > I just saw this: > http://www.tuaw.com/2011/07/19/cord-from-iphone-to-arduino-now-available/ > > Redpark (maker of the cable) has a Serial SDK for app dev with the cable. I > assume it could be made into an external for mobile dev (?), but their > licensing is for private use only. Bummer. > -- Phil Davis PDS Labs Professional Software Development http://pdslabs.net From kray at sonsothunder.com Wed Jul 20 13:06:05 2011 From: kray at sonsothunder.com (Ken Ray) Date: Wed, 20 Jul 2011 12:06:05 -0500 Subject: Countdown HH:MM:SS In-Reply-To: References: <8CE14A04637B482-1AA4-201D3@webmail-d058.sysops.aol.com> <61EE076F-3EE3-42D4-A552-2ADF6526034A@sonsothunder.com> Message-ID: > Using the dateItems is definitely the right direction, but my objective is > to count backwards from for example; 2 hours, 45 minutes, and 59 seconds > while updating the display field every second. When the countdown reaches > 00:00:00, I would play a sound of do other actions. Sorry about that... this will work if you just want to count down from a fixed amount of time: on mouseUp DoCountDown "2:45:59" end mouseUp on DoCountDown pTime convert pTime to seconds subtract 1 from pTime convert pTime to dateItems split pTime by "," put format("%02d:%02d:%02d",pTime[4],pTime[5],pTime[6]) into tNewTime put tNewTime into fld "Time" if tNewTime = "00:00:00" then TimesUp else send "DoCountDown tNewTime" to me in 1 second end if end DoCountDown on TimesUp answer "Done" end TimesUp Ken Ray Sons of Thunder Software, Inc. Email: kray at sonsothunder.com Web Site: http://www.sonsothunder.com/ From roger.e.eller at sealedair.com Wed Jul 20 13:06:34 2011 From: roger.e.eller at sealedair.com (Roger Eller) Date: Wed, 20 Jul 2011 13:06:34 -0400 Subject: ANN: LiveCodeErrors for iPhone In-Reply-To: <4E26FC12.50201@hyperactivesw.com> References: <215B0FE5-038A-47C7-9F24-B9D29CEDD816@economy-x-talk.com> <4E25A8E0.6090704@fourthworld.com> <2E567827-7552-4FF2-BDFC-446D6330FB6B@economy-x-talk.com> <4E26ED38.2020109@hyperactivesw.com> <4E26FC12.50201@hyperactivesw.com> Message-ID: On Wed, Jul 20, 2011 at 12:02 PM, J. Landman Gay wrote: > I apologize for the public posting. I agree that anyone can sell anything > they like, and anyone can buy anything they like. > > End of thread for me. > > -- > Jacqueline Landman Gay | jacque at hyperactivesw.com > HyperActive Software | http://www.hyperactivesw.com > > Me too. I hope Mark and every LiveCode user here becomes very prosperous with their products. The End. ~Roger From andre at andregarzia.com Wed Jul 20 13:29:51 2011 From: andre at andregarzia.com (Andre Garzia) Date: Wed, 20 Jul 2011 14:29:51 -0300 Subject: LiveCode Server Release and On-Rev In-Reply-To: References: <848DF372-D3F3-4658-8A13-14D4C7852B67@sonsothunder.com> <1311172600076-3680996.post@n4.nabble.com> Message-ID: There is a way to have both RevServer and the new LiveCode Server installed side by side with no interference. You can bind one to .lc and the other to .irev. I don't know how RunRev will install things on the On-Rev server, but this can be done. On Wed, Jul 20, 2011 at 1:46 PM, Ken Ray wrote: > > On Jul 20, 2011, at 9:36 AM, Andrew Kluthe wrote: > > > Heather says On-Rev users will be upgraded to the new livecode server in > a > > few days. > > That's great to hear, but should I assume that current .irev files won't be > affected by the upgrade? I just want to know if I need to change anything > before the upgrade since I have some clients that are dependent on the .irev > files working as usual. > > Ken Ray > Sons of Thunder Software, Inc. > Email: kray at sonsothunder.com > Web Site: http://www.sonsothunder.com/ > > _______________________________________________ > use-livecode mailing list > use-livecode at lists.runrev.com > Please visit this url to subscribe, unsubscribe and manage your > subscription preferences: > http://lists.runrev.com/mailman/listinfo/use-livecode > -- http://www.andregarzia.com All We Do Is Code. From rene.micout at numericable.com Wed Jul 20 13:34:47 2011 From: rene.micout at numericable.com (=?iso-8859-1?Q?Ren=E9_Micout?=) Date: Wed, 20 Jul 2011 19:34:47 +0200 Subject: Mac OS X Lion Message-ID: Hello Mac addict(s) ! First impression after installing Mac OS X Lion on my Macintosh... LiveCode 4.6.3 run well... New buttons, new radio buttons... A little problem: the scroll bar have the two aspect, the last one (borders) ans the new one (grey bar moving)... Tomorrow morning a more complete review of the new environment... Good evening from Paris Ren? From klaus at major.on-rev.com Wed Jul 20 13:42:52 2011 From: klaus at major.on-rev.com (Klaus on-rev) Date: Wed, 20 Jul 2011 19:42:52 +0200 Subject: Mac OS X Lion In-Reply-To: References: Message-ID: Bonsoir Ren?, Am 20.07.2011 um 19:34 schrieb Ren? Micout: > Hello Mac addict(s) ! > First impression after installing Mac OS X Lion on my Macintosh... > LiveCode 4.6.3 run well... > New buttons, new radio buttons... > A little problem: the scroll bar have the two aspect, the last one (borders) ans the new one (grey bar moving)... > Tomorrow morning a more complete review of the new environment... thanks for the info! Sounds good! I think I will update tomorrow :-) > Good evening from Paris > Ren? Best Klaus -- Klaus Major http://www.major-k.de klaus at major.on-rev.com From pete at mollysrevenge.com Wed Jul 20 13:50:06 2011 From: pete at mollysrevenge.com (Pete) Date: Wed, 20 Jul 2011 10:50:06 -0700 Subject: Countdown HH:MM:SS In-Reply-To: References: <04539BD7-007A-4621-BFA9-64DCB8F81169@sbcglobal.net> Message-ID: Hi Bob, Depending on your requirements, your tests might ned to be >= and <=. Also, SQL has a BETWEEN operator that would make your code a little more readable: "SELECT * FROM xyz WHERE startDate BETWEEN savedStartDate AND savedEndDate...". I don't think you need to store your dates in julian format. The standard SQL storage format of YYYY-MM-DD should work for your comparisons as long as startdate is in that format also (I think). If not, SQL (at least sqlite) has date conversion expressions you can use to convert the dates to Julian before the comparison: SELECT * FROM xyz WHERE strftime(%J,startDate) BETWEEN strftime(%J,savedStartDate) AND strftime(%J,savedEndDate)..." mySQL does not have a Julian day conversion expression, but it does have a TO_DAYS function so, as long as your dates are not before 1582 (!!!): "SELECT * FROM xyz WHERE TO_DAYS(startDate) BETWEEN TO_DAYS(savedStartDate) AND TO_DAYS(SavedEndDate)..." For a system that is supposed to adhere to a standard, SQL has a remarkable number of variations in its various implementations! Pete Molly's Revenge On Wed, Jul 20, 2011 at 9:32 AM, Bob Sneidar wrote: > Hey as long as we are talking about dates and times, I am going to need > soon a method to determine if a block of time intersects with another. > Specifically I will need to do this in an SQL query. I suppose the best way > would be to store all dates in julian format, then have a query that, given > savedStartDate and savedEndDate are columns in a table, goes something like > : > > select * where (startDate > savedStartDate and startDate < savedEndDate) or > (endDate > savedStartDate and endDate < savedEndDate) > > Is that about right? Any other ideas? > > Bob > > > On Jul 20, 2011, at 8:29 AM, James Hurley wrote: > > > Roger > > > > Is this what you are looking for? > > > > on mouseUp > > set itemdel to ":" > > put the long time into tTime > > put char 1 to -4 of tTime into field 1 > > convert tTime to dateItems > > set the hms of me to tTime > > countDown > > end mouseUp > > > > on countDown > > put the hms of me into temp > > subtract 1 from item -2 of temp > > --The dateItems will take care of the negative seconds > > convert temp to dateItems > > set the hms of me to temp > > put temp into tDisplayTIme > > put item -4 to -2 of tDisplayTIme into temp2 > > replace "," with ":" in temp2 > > put temp2 into field 1 > > send countDown to me in 1 sec > > --Change the "1 sec" to "1 Minute" when you are sure it works for you. > > end countDown > > > > Jim Hurley > > > > > > _______________________________________________ > > use-livecode mailing list > > use-livecode at lists.runrev.com > > Please visit this url to subscribe, unsubscribe and manage your > subscription preferences: > > http://lists.runrev.com/mailman/listinfo/use-livecode > > > _______________________________________________ > use-livecode mailing list > use-livecode at lists.runrev.com > Please visit this url to subscribe, unsubscribe and manage your > subscription preferences: > http://lists.runrev.com/mailman/listinfo/use-livecode > > From andre at andregarzia.com Wed Jul 20 13:53:35 2011 From: andre at andregarzia.com (Andre Garzia) Date: Wed, 20 Jul 2011 14:53:35 -0300 Subject: Mac OS X Lion In-Reply-To: References: Message-ID: I don't like lion or the new scrollbars... :-( will stay with snow leopard for a while :-) On Wed, Jul 20, 2011 at 2:42 PM, Klaus on-rev wrote: > Bonsoir Ren?, > > Am 20.07.2011 um 19:34 schrieb Ren? Micout: > > > Hello Mac addict(s) ! > > First impression after installing Mac OS X Lion on my Macintosh... > > LiveCode 4.6.3 run well... > > New buttons, new radio buttons... > > A little problem: the scroll bar have the two aspect, the last one > (borders) ans the new one (grey bar moving)... > > Tomorrow morning a more complete review of the new environment... > > thanks for the info! > Sounds good! > > I think I will update tomorrow :-) > > > Good evening from Paris > > Ren? > > Best > > Klaus > > -- > Klaus Major > http://www.major-k.de > klaus at major.on-rev.com > > > _______________________________________________ > use-livecode mailing list > use-livecode at lists.runrev.com > Please visit this url to subscribe, unsubscribe and manage your > subscription preferences: > http://lists.runrev.com/mailman/listinfo/use-livecode > -- http://www.andregarzia.com All We Do Is Code. From ambassador at fourthworld.com Wed Jul 20 14:11:55 2011 From: ambassador at fourthworld.com (Richard Gaskin) Date: Wed, 20 Jul 2011 11:11:55 -0700 Subject: Mac OS X Lion In-Reply-To: References: Message-ID: <4E271A6B.7000509@fourthworld.com> Andre wrote: > I don't like lion or the new scrollbars... :-( For better or worse, it seems those scrollbars are the wave of the future: What concerns me more about Lion is how auto-save is supposed to work with our apps.... -- Richard Gaskin Fourth World LiveCode training and consulting: http://www.fourthworld.com Webzine for LiveCode developers: http://www.LiveCodeJournal.com LiveCode Journal blog: http://LiveCodejournal.com/blog.irv From mpezzo at gmail.com Wed Jul 20 14:13:41 2011 From: mpezzo at gmail.com (AcidJazz) Date: Wed, 20 Jul 2011 11:13:41 -0700 (PDT) Subject: revmpossible? In-Reply-To: References: <1311048004923-3677225.post@n4.nabble.com> Message-ID: <1311185621681-3681693.post@n4.nabble.com> Cool. A video would be great. It's not entirely clear to me yet what revimpossible could *not* do that a plugin could do, but that indicates my lack of understanding. Thanks, Mark -- View this message in context: http://runtime-revolution.278305.n4.nabble.com/revmpossible-tp3677225p3681693.html Sent from the Revolution - User mailing list archive at Nabble.com. From olivierdussutour at gmail.com Wed Jul 20 14:27:28 2011 From: olivierdussutour at gmail.com (Olivier Dussutour) Date: Wed, 20 Jul 2011 20:27:28 +0200 Subject: revIgniter References: <527C7219-72AE-4552-B8DD-43336F45923C@dimensionB.de><1311094209355-3678624.post@n4.nabble.com><1311095748965-3678689.post@n4.nabble.com> <1311097203523-3678755.post@n4.nabble.com> Message-ID: <973D0DBD2B584E40A0993E7BF0F20CC0@portablefip> I do not quite understand what is revigniter? Is it locally installed with WampServer? ps: I am beginner and French with a deplorable level of English so please write slowly ;-) Cordialement, Olivier Dussutour olivierdussutour at gmail.com From bobs at twft.com Wed Jul 20 14:27:31 2011 From: bobs at twft.com (Bob Sneidar) Date: Wed, 20 Jul 2011 11:27:31 -0700 Subject: Countdown HH:MM:SS In-Reply-To: References: <04539BD7-007A-4621-BFA9-64DCB8F81169@sbcglobal.net> Message-ID: <00121B5D-E28B-40C0-B8B1-B8AF3050B097@twft.com> Ah! Very useful thank you! Bob On Jul 20, 2011, at 10:50 AM, Pete wrote: > Hi Bob, > Depending on your requirements, your tests might ned to be >= and <=. Also, > SQL has a BETWEEN operator that would make your code a little more > readable: > > "SELECT * FROM xyz WHERE startDate BETWEEN savedStartDate AND > savedEndDate...". > > I don't think you need to store your dates in julian format. The standard > SQL storage format of YYYY-MM-DD should work for your comparisons as long as > startdate is in that format also (I think). If not, SQL (at least sqlite) > has date conversion expressions you can use to convert the dates to Julian > before the comparison: > > SELECT * FROM xyz WHERE strftime(%J,startDate) BETWEEN > strftime(%J,savedStartDate) AND strftime(%J,savedEndDate)..." > > mySQL does not have a Julian day conversion expression, but it does have a > TO_DAYS function so, as long as your dates are not before 1582 (!!!): > > "SELECT * FROM xyz WHERE TO_DAYS(startDate) BETWEEN TO_DAYS(savedStartDate) > AND TO_DAYS(SavedEndDate)..." > > For a system that is supposed to adhere to a standard, SQL has a remarkable > number of variations in its various implementations! > > > Pete > Molly's Revenge > > > > > On Wed, Jul 20, 2011 at 9:32 AM, Bob Sneidar wrote: > >> Hey as long as we are talking about dates and times, I am going to need >> soon a method to determine if a block of time intersects with another. >> Specifically I will need to do this in an SQL query. I suppose the best way >> would be to store all dates in julian format, then have a query that, given >> savedStartDate and savedEndDate are columns in a table, goes something like >> : >> >> select * where (startDate > savedStartDate and startDate < savedEndDate) or >> (endDate > savedStartDate and endDate < savedEndDate) >> >> Is that about right? Any other ideas? >> >> Bob >> >> >> On Jul 20, 2011, at 8:29 AM, James Hurley wrote: >> >>> Roger >>> >>> Is this what you are looking for? >>> >>> on mouseUp >>> set itemdel to ":" >>> put the long time into tTime >>> put char 1 to -4 of tTime into field 1 >>> convert tTime to dateItems >>> set the hms of me to tTime >>> countDown >>> end mouseUp >>> >>> on countDown >>> put the hms of me into temp >>> subtract 1 from item -2 of temp >>> --The dateItems will take care of the negative seconds >>> convert temp to dateItems >>> set the hms of me to temp >>> put temp into tDisplayTIme >>> put item -4 to -2 of tDisplayTIme into temp2 >>> replace "," with ":" in temp2 >>> put temp2 into field 1 >>> send countDown to me in 1 sec >>> --Change the "1 sec" to "1 Minute" when you are sure it works for you. >>> end countDown >>> >>> Jim Hurley >>> >>> >>> _______________________________________________ >>> use-livecode mailing list >>> use-livecode at lists.runrev.com >>> Please visit this url to subscribe, unsubscribe and manage your >> subscription preferences: >>> http://lists.runrev.com/mailman/listinfo/use-livecode >> >> >> _______________________________________________ >> use-livecode mailing list >> use-livecode at lists.runrev.com >> Please visit this url to subscribe, unsubscribe and manage your >> subscription preferences: >> http://lists.runrev.com/mailman/listinfo/use-livecode >> >> > _______________________________________________ > use-livecode mailing list > use-livecode at lists.runrev.com > Please visit this url to subscribe, unsubscribe and manage your subscription preferences: > http://lists.runrev.com/mailman/listinfo/use-livecode From warren at warrensweb.us Wed Jul 20 14:30:36 2011 From: warren at warrensweb.us (Warren Samples) Date: Wed, 20 Jul 2011 13:30:36 -0500 Subject: Mac OS X Lion In-Reply-To: <4E271A6B.7000509@fourthworld.com> References: <4E271A6B.7000509@fourthworld.com> Message-ID: <201107201330.36610.warren@warrensweb.us> On Wednesday, July 20, 2011 01:11:55 PM Richard Gaskin wrote: > For better or worse, it seems those scrollbars are the wave of the future: Stevie was really enthuastic about his drawers, too. For a while. Warren From richmondmathewson at gmail.com Wed Jul 20 14:42:11 2011 From: richmondmathewson at gmail.com (Richmond Mathewson) Date: Wed, 20 Jul 2011 21:42:11 +0300 Subject: Mac OS X Lion In-Reply-To: <201107201330.36610.warren@warrensweb.us> References: <4E271A6B.7000509@fourthworld.com> <201107201330.36610.warren@warrensweb.us> Message-ID: <4E272183.90904@gmail.com> Not wishing to appear catty or anything . . . But, surely, from the point of view of the average RunRev/Livecode developer unless one has a desperate urge to leverage any new capabilities in 'Lion' it is just business as usual? From richmondmathewson at gmail.com Wed Jul 20 15:14:20 2011 From: richmondmathewson at gmail.com (Richmond Mathewson) Date: Wed, 20 Jul 2011 22:14:20 +0300 Subject: Mac OS X Lion In-Reply-To: <4E272183.90904@gmail.com> References: <4E272183.90904@gmail.com> Message-ID: <4E27290C.6050400@gmail.com> http://www.apple.com/macosx/whats-new/ well; I don't know what to say, but on the basis of that webpage there seems to be nothing much more than eye-candy. The only thing that seems vaguely new is the set of finger-twiddles one can perform on one's trackpad: just thinking about them makes me reach for my mouse and Belkin Nostromo. For those who are excited about multi-touch guestures, they can get these working with Touchegg in Linux for a few minutes work, rather than rushing out to buy 'Lion'. From roger.e.eller at sealedair.com Wed Jul 20 15:19:07 2011 From: roger.e.eller at sealedair.com (Roger Eller) Date: Wed, 20 Jul 2011 15:19:07 -0400 Subject: Countdown HH:MM:SS In-Reply-To: References: <8CE14A04637B482-1AA4-201D3@webmail-d058.sysops.aol.com> <61EE076F-3EE3-42D4-A552-2ADF6526034A@sonsothunder.com> Message-ID: On Wed, Jul 20, 2011 at 1:06 PM, Ken Ray wrote: > > Using the dateItems is definitely the right direction, but my objective > is > > to count backwards from for example; 2 hours, 45 minutes, and 59 seconds > > while updating the display field every second. When the countdown > reaches > > 00:00:00, I would play a sound of do other actions. > > Sorry about that... this will work if you just want to count down from a > fixed amount of time: > > on mouseUp > DoCountDown "2:45:59" > end mouseUp > > on DoCountDown pTime > convert pTime to seconds > subtract 1 from pTime > convert pTime to dateItems > split pTime by "," > put format("%02d:%02d:%02d",pTime[4],pTime[5],pTime[6]) into tNewTime > put tNewTime into fld "Time" > if tNewTime = "00:00:00" then > TimesUp > else > send "DoCountDown tNewTime" to me in 1 second > end if > end DoCountDown > > on TimesUp > answer "Done" > end TimesUp > > Ken Ray > Sons of Thunder Software, Inc. > Email: kray at sonsothunder.com > Web Site: http://www.sonsothunder.com/ > > With only a few tweaks like handling a cancel/reset request, this script does what I need. James' was almost the one, but the time wasn't starting from a pre-set amount of time such as 1 hour, etc. Thanks to everyone that contributed! ~Roger From andre at andregarzia.com Wed Jul 20 15:37:08 2011 From: andre at andregarzia.com (Andre Garzia) Date: Wed, 20 Jul 2011 16:37:08 -0300 Subject: Mac OS X Lion In-Reply-To: <201107201330.36610.warren@warrensweb.us> References: <4E271A6B.7000509@fourthworld.com> <201107201330.36610.warren@warrensweb.us> Message-ID: On Wed, Jul 20, 2011 at 3:30 PM, Warren Samples wrote: > On Wednesday, July 20, 2011 01:11:55 PM Richard Gaskin wrote: > > For better or worse, it seems those scrollbars are the wave of the > future: > > > Stevie was really enthuastic about his drawers, too. For a while. > > I like drawers :-) > Warren > > _______________________________________________ > use-livecode mailing list > use-livecode at lists.runrev.com > Please visit this url to subscribe, unsubscribe and manage your > subscription preferences: > http://lists.runrev.com/mailman/listinfo/use-livecode > -- http://www.andregarzia.com All We Do Is Code. From andre at andregarzia.com Wed Jul 20 15:38:04 2011 From: andre at andregarzia.com (Andre Garzia) Date: Wed, 20 Jul 2011 16:38:04 -0300 Subject: Mac OS X Lion In-Reply-To: <4E271A6B.7000509@fourthworld.com> References: <4E271A6B.7000509@fourthworld.com> Message-ID: On Wed, Jul 20, 2011 at 3:11 PM, Richard Gaskin wrote: > Andre wrote: > > I don't like lion or the new scrollbars... :-( >> > > For better or worse, it seems those scrollbars are the wave of the future: > > **> > > What concerns me more about Lion is how auto-save is supposed to work with > our apps.... > I have no clue either... The new APIs are so Objective-C/Cocoa specific, using delegates and whatnot that I think it will be a tad hard for our little carbon engine to leverage on them. > > -- > Richard Gaskin > Fourth World > LiveCode training and consulting: http://www.fourthworld.com > Webzine for LiveCode developers: http://www.LiveCodeJournal.com > LiveCode Journal blog: http://LiveCodejournal.com/**blog.irv > > > ______________________________**_________________ > use-livecode mailing list > use-livecode at lists.runrev.com > Please visit this url to subscribe, unsubscribe and manage your > subscription preferences: > http://lists.runrev.com/**mailman/listinfo/use-livecode > -- http://www.andregarzia.com All We Do Is Code. From bobs at twft.com Wed Jul 20 15:41:14 2011 From: bobs at twft.com (Bob Sneidar) Date: Wed, 20 Jul 2011 12:41:14 -0700 Subject: Mac OS X Lion In-Reply-To: <4E27290C.6050400@gmail.com> References: <4E272183.90904@gmail.com> <4E27290C.6050400@gmail.com> Message-ID: You mean besides the new pricing which is WAAAY more affordable than prior version? Or besides the new Remote Access that allows me to have a session which does not disturb the current local session? Or the fact that Remote Access Server is no longer $500 unlimited, but somewhere in the area of $80? Or the fact that Xsan Client is built in and Xsan Controller is also insanely cheaper? (I could go on). Aside from all of that, I don't think we will be doing a wholesale upgrade just yet. There are applications that will certainly break (I ran the first gold RC) and upgrading Lion also means, for some upgrading their software too. Bob On Jul 20, 2011, at 12:14 PM, Richmond Mathewson wrote: > http://www.apple.com/macosx/whats-new/ > > well; I don't know what to say, but on the basis of that webpage there seems to be > nothing much more than eye-candy. > > The only thing that seems vaguely new is the set of finger-twiddles one can > perform on one's trackpad: just thinking about them makes me reach for my mouse and Belkin Nostromo. > > For those who are excited about multi-touch guestures, they can get these working > with Touchegg in Linux for a few minutes work, rather than rushing out to buy 'Lion'. > > _______________________________________________ > use-livecode mailing list > use-livecode at lists.runrev.com > Please visit this url to subscribe, unsubscribe and manage your subscription preferences: > http://lists.runrev.com/mailman/listinfo/use-livecode From richmondmathewson at gmail.com Wed Jul 20 15:45:54 2011 From: richmondmathewson at gmail.com (Richmond Mathewson) Date: Wed, 20 Jul 2011 22:45:54 +0300 Subject: Mac OS X Lion In-Reply-To: References: <4E272183.90904@gmail.com> <4E27290C.6050400@gmail.com> Message-ID: <4E273072.5020908@gmail.com> On 07/20/2011 10:41 PM, Bob Sneidar wrote: > You mean besides the new pricing which is WAAAY more affordable than prior version? Or besides the new Remote Access that allows me to have a session which does not disturb the current local session? Or the fact that Remote Access Server is no longer $500 unlimited, but somewhere in the area of $80? Or the fact that Xsan Client is built in and Xsan Controller is also insanely cheaper? (I could go on). I was writing about interface advances; not about money. Certainly Remote access and so forth are, just a bit, cheaper with Linux, so there have to be other reasons for sticking with Mac. From what I can see, should I splurge on one of the new Mac-Minis, the first thing I would do is install Leopard (10.5) on the thing. > Aside from all of that, I don't think we will be doing a wholesale upgrade just yet. There are applications that will certainly break (I ran the first gold RC) and upgrading Lion also means, for some upgrading their software too. > > Bob > > > On Jul 20, 2011, at 12:14 PM, Richmond Mathewson wrote: > >> http://www.apple.com/macosx/whats-new/ >> >> well; I don't know what to say, but on the basis of that webpage there seems to be >> nothing much more than eye-candy. >> >> The only thing that seems vaguely new is the set of finger-twiddles one can >> perform on one's trackpad: just thinking about them makes me reach for my mouse and Belkin Nostromo. >> >> For those who are excited about multi-touch guestures, they can get these working >> with Touchegg in Linux for a few minutes work, rather than rushing out to buy 'Lion'. >> >> _______________________________________________ >> use-livecode mailing list >> use-livecode at lists.runrev.com >> Please visit this url to subscribe, unsubscribe and manage your subscription preferences: >> http://lists.runrev.com/mailman/listinfo/use-livecode > > _______________________________________________ > use-livecode mailing list > use-livecode at lists.runrev.com > Please visit this url to subscribe, unsubscribe and manage your subscription preferences: > http://lists.runrev.com/mailman/listinfo/use-livecode From andre at andregarzia.com Wed Jul 20 15:45:54 2011 From: andre at andregarzia.com (Andre Garzia) Date: Wed, 20 Jul 2011 16:45:54 -0300 Subject: Mac OS X Lion In-Reply-To: References: <4E272183.90904@gmail.com> <4E27290C.6050400@gmail.com> Message-ID: The last Lion DP was pretty bad on my machine. I am going to wait for a long time... On Wed, Jul 20, 2011 at 4:41 PM, Bob Sneidar wrote: > You mean besides the new pricing which is WAAAY more affordable than prior > version? Or besides the new Remote Access that allows me to have a session > which does not disturb the current local session? Or the fact that Remote > Access Server is no longer $500 unlimited, but somewhere in the area of $80? > Or the fact that Xsan Client is built in and Xsan Controller is also > insanely cheaper? (I could go on). > > Aside from all of that, I don't think we will be doing a wholesale upgrade > just yet. There are applications that will certainly break (I ran the first > gold RC) and upgrading Lion also means, for some upgrading their software > too. > > Bob > > > On Jul 20, 2011, at 12:14 PM, Richmond Mathewson wrote: > > > http://www.apple.com/macosx/whats-new/ > > > > well; I don't know what to say, but on the basis of that webpage there > seems to be > > nothing much more than eye-candy. > > > > The only thing that seems vaguely new is the set of finger-twiddles one > can > > perform on one's trackpad: just thinking about them makes me reach for my > mouse and Belkin Nostromo. > > > > For those who are excited about multi-touch guestures, they can get these > working > > with Touchegg in Linux for a few minutes work, rather than rushing out to > buy 'Lion'. > > > > _______________________________________________ > > use-livecode mailing list > > use-livecode at lists.runrev.com > > Please visit this url to subscribe, unsubscribe and manage your > subscription preferences: > > http://lists.runrev.com/mailman/listinfo/use-livecode > > > _______________________________________________ > use-livecode mailing list > use-livecode at lists.runrev.com > Please visit this url to subscribe, unsubscribe and manage your > subscription preferences: > http://lists.runrev.com/mailman/listinfo/use-livecode > -- http://www.andregarzia.com All We Do Is Code. From warren at warrensweb.us Wed Jul 20 15:47:43 2011 From: warren at warrensweb.us (Warren Samples) Date: Wed, 20 Jul 2011 14:47:43 -0500 Subject: Mac OS X Lion In-Reply-To: References: <201107201330.36610.warren@warrensweb.us> Message-ID: <201107201447.43312.warren@warrensweb.us> On Wednesday, July 20, 2011 02:37:08 PM Andre Garzia wrote: > I like drawers :-) Drawers are indispensible for certain purposes. I have many and use them every day, but hated them on my computer ;) Warren From form at nonsanity.com Wed Jul 20 15:51:50 2011 From: form at nonsanity.com (Nonsanity) Date: Wed, 20 Jul 2011 15:51:50 -0400 Subject: Countdown HH:MM:SS In-Reply-To: References: <8CE14A04637B482-1AA4-201D3@webmail-d058.sysops.aol.com> <61EE076F-3EE3-42D4-A552-2ADF6526034A@sonsothunder.com> Message-ID: I wouldn't rely on the send in time structure to actually fire every second. It's going to slip over time as the computer takes periodic spikes of use. If you want New Years Eve accuracy of the final countdown, you should probably re-calculate the delta based on the current time continuously. Here's the code from a quick test stack's single button. There are also three fields. on mouseup put the internet date into fld 1 -- display the current time get fld 2 -- get the target time convert it to seconds set the targetTime of me to it UpdateCountdown end mouseup on UpdateCountdown get the targetTime of me put CountdownCalc( it ) into fld 3 -- display the countdown -- always best to make sure this timer loop hasn't already been started if "UpdateDountdown" is not in the pendingmessages then send "UpdateCountdown" to me in 1 second end UpdateCountdown function CountdownCalc targetTime -- in seconds put targetTime - the seconds into deltaTime put trunc(deltaTime / 3600) into h -- hours put trunc((deltaTime - (h * 3600)) / 60) into m -- minutes put deltaTime - (h * 3600) - (m * 60) into s -- seconds put format( "%i:%02i:%02i", h, m, s ) into countdown return countdown end CountdownCalc Pretty much the same as the other samples, but more accurate over the long haul in those final seconds. ~ Chris Innanen ~ Nonsanity On Wed, Jul 20, 2011 at 3:19 PM, Roger Eller wrote: > On Wed, Jul 20, 2011 at 1:06 PM, Ken Ray wrote: > > > > Using the dateItems is definitely the right direction, but my objective > > is > > > to count backwards from for example; 2 hours, 45 minutes, and 59 > seconds > > > while updating the display field every second. When the countdown > > reaches > > > 00:00:00, I would play a sound of do other actions. > > > > Sorry about that... this will work if you just want to count down from a > > fixed amount of time: > > > > on mouseUp > > DoCountDown "2:45:59" > > end mouseUp > > > > on DoCountDown pTime > > convert pTime to seconds > > subtract 1 from pTime > > convert pTime to dateItems > > split pTime by "," > > put format("%02d:%02d:%02d",pTime[4],pTime[5],pTime[6]) into tNewTime > > put tNewTime into fld "Time" > > if tNewTime = "00:00:00" then > > TimesUp > > else > > send "DoCountDown tNewTime" to me in 1 second > > end if > > end DoCountDown > > > > on TimesUp > > answer "Done" > > end TimesUp > > > > Ken Ray > > Sons of Thunder Software, Inc. > > Email: kray at sonsothunder.com > > Web Site: http://www.sonsothunder.com/ > > > > > With only a few tweaks like handling a cancel/reset request, this script > does what I need. James' was almost the one, but the time wasn't starting > from a pre-set amount of time such as 1 hour, etc. Thanks to everyone that > contributed! > > ~Roger > _______________________________________________ > use-livecode mailing list > use-livecode at lists.runrev.com > Please visit this url to subscribe, unsubscribe and manage your > subscription preferences: > http://lists.runrev.com/mailman/listinfo/use-livecode > From bobs at twft.com Wed Jul 20 15:57:40 2011 From: bobs at twft.com (Bob Sneidar) Date: Wed, 20 Jul 2011 12:57:40 -0700 Subject: Mac OS X Lion In-Reply-To: <201107201447.43312.warren@warrensweb.us> References: <201107201330.36610.warren@warrensweb.us> <201107201447.43312.warren@warrensweb.us> Message-ID: <71607FED-80BD-45A2-A85C-9891F02F430F@twft.com> It's Drawer Implementation and Maintenance that always had me stumped. Where should I install them? Should they always be implemented? There are different schools of thought on the subject. I believe in European circles, some don't see the need for drawers at all. I'm not sure I would go that far. The older I get, the more I use I have for drawers. ;-) Bob On Jul 20, 2011, at 12:47 PM, Warren Samples wrote: > On Wednesday, July 20, 2011 02:37:08 PM Andre Garzia wrote: >> I like drawers :-) > > > Drawers are indispensible for certain purposes. I have many and use them every day, but hated them on my > computer ;) > > Warren > > _______________________________________________ > use-livecode mailing list > use-livecode at lists.runrev.com > Please visit this url to subscribe, unsubscribe and manage your subscription preferences: > http://lists.runrev.com/mailman/listinfo/use-livecode From richmondmathewson at gmail.com Wed Jul 20 16:07:12 2011 From: richmondmathewson at gmail.com (Richmond Mathewson) Date: Wed, 20 Jul 2011 23:07:12 +0300 Subject: Mac OS X Lion In-Reply-To: <71607FED-80BD-45A2-A85C-9891F02F430F@twft.com> References: <201107201330.36610.warren@warrensweb.us> <201107201447.43312.warren@warrensweb.us> <71607FED-80BD-45A2-A85C-9891F02F430F@twft.com> Message-ID: <4E273570.50604@gmail.com> On 07/20/2011 10:57 PM, Bob Sneidar wrote: > It's Drawer Implementation and Maintenance that always had me stumped. Where should I install them? Should they always be implemented? There are different schools of thought on the subject. I believe in European circles, That is because drawers are long and baggy; not circular. > some don't see the need for drawers at all. I'm not sure I would go that far. The older I get, the more I use I have for drawers. ;-) Certainly, as it is 40 degs Celsius here in Plovdiv, Bulgaria, I wish I could wear a kilt without people chucking bricks at me. It seems that with trousers one keeps one's drawers close to oneself, and with a kilt, other things are drawn to one. > > Bob > > > From pete at mollysrevenge.com Wed Jul 20 16:13:33 2011 From: pete at mollysrevenge.com (Pete) Date: Wed, 20 Jul 2011 13:13:33 -0700 Subject: Mac OS X Lion In-Reply-To: References: <4E271A6B.7000509@fourthworld.com> Message-ID: There's been recent discussion of just how many OSX controls are not natively available in LC and Lion just seems to add to that list. Will LC ever catch up? Pete Molly's Revenge On Wed, Jul 20, 2011 at 12:38 PM, Andre Garzia wrote: > On Wed, Jul 20, 2011 at 3:11 PM, Richard Gaskin > wrote: > > > Andre wrote: > > > > I don't like lion or the new scrollbars... :-( > >> > > > > For better or worse, it seems those scrollbars are the wave of the > future: > > http://livecodejournal.com/blog.irv?pid=1307545265.586975> > > **> > > > > What concerns me more about Lion is how auto-save is supposed to work > with > > our apps.... > > > > I have no clue either... The new APIs are so Objective-C/Cocoa specific, > using delegates and whatnot that I think it will be a tad hard for our > little carbon engine to leverage on them. > > > > > > > -- > > Richard Gaskin > > Fourth World > > LiveCode training and consulting: http://www.fourthworld.com > > Webzine for LiveCode developers: http://www.LiveCodeJournal.com > > LiveCode Journal blog: http://LiveCodejournal.com/**blog.irv< > http://LiveCodejournal.com/blog.irv> > > > > > > ______________________________**_________________ > > use-livecode mailing list > > use-livecode at lists.runrev.com > > Please visit this url to subscribe, unsubscribe and manage your > > subscription preferences: > > http://lists.runrev.com/**mailman/listinfo/use-livecode< > http://lists.runrev.com/mailman/listinfo/use-livecode> > > > > > > -- > http://www.andregarzia.com All We Do Is Code. > _______________________________________________ > use-livecode mailing list > use-livecode at lists.runrev.com > Please visit this url to subscribe, unsubscribe and manage your > subscription preferences: > http://lists.runrev.com/mailman/listinfo/use-livecode > > From andre at andregarzia.com Wed Jul 20 16:19:23 2011 From: andre at andregarzia.com (Andre Garzia) Date: Wed, 20 Jul 2011 17:19:23 -0300 Subject: Mac OS X Lion In-Reply-To: References: <4E271A6B.7000509@fourthworld.com> Message-ID: On Wed, Jul 20, 2011 at 5:13 PM, Pete wrote: > There's been recent discussion of just how many OSX controls are not > natively available in LC and Lion just seems to add to that list. Will LC > ever catch up? > LC is Carbon and has a lot of emulated stuff to make is cross-platform, this is a very hard race to win... :-) > Pete > Molly's Revenge > > > > > On Wed, Jul 20, 2011 at 12:38 PM, Andre Garzia >wrote: > > > On Wed, Jul 20, 2011 at 3:11 PM, Richard Gaskin > > wrote: > > > > > Andre wrote: > > > > > > I don't like lion or the new scrollbars... :-( > > >> > > > > > > For better or worse, it seems those scrollbars are the wave of the > > future: > > > > http://livecodejournal.com/blog.irv?pid=1307545265.586975> > > > **> > > > > > > What concerns me more about Lion is how auto-save is supposed to work > > with > > > our apps.... > > > > > > > I have no clue either... The new APIs are so Objective-C/Cocoa specific, > > using delegates and whatnot that I think it will be a tad hard for our > > little carbon engine to leverage on them. > > > > > > > > > > > > -- > > > Richard Gaskin > > > Fourth World > > > LiveCode training and consulting: http://www.fourthworld.com > > > Webzine for LiveCode developers: http://www.LiveCodeJournal.com > > > LiveCode Journal blog: http://LiveCodejournal.com/**blog.irv< > > http://LiveCodejournal.com/blog.irv> > > > > > > > > > ______________________________**_________________ > > > use-livecode mailing list > > > use-livecode at lists.runrev.com > > > Please visit this url to subscribe, unsubscribe and manage your > > > subscription preferences: > > > http://lists.runrev.com/**mailman/listinfo/use-livecode< > > http://lists.runrev.com/mailman/listinfo/use-livecode> > > > > > > > > > > > -- > > http://www.andregarzia.com All We Do Is Code. > > _______________________________________________ > > use-livecode mailing list > > use-livecode at lists.runrev.com > > Please visit this url to subscribe, unsubscribe and manage your > > subscription preferences: > > http://lists.runrev.com/mailman/listinfo/use-livecode > > > > > _______________________________________________ > use-livecode mailing list > use-livecode at lists.runrev.com > Please visit this url to subscribe, unsubscribe and manage your > subscription preferences: > http://lists.runrev.com/mailman/listinfo/use-livecode > -- http://www.andregarzia.com All We Do Is Code. From andrew at rjdfarm.com Wed Jul 20 16:30:10 2011 From: andrew at rjdfarm.com (Andrew Kluthe) Date: Wed, 20 Jul 2011 13:30:10 -0700 (PDT) Subject: revIgniter In-Reply-To: <973D0DBD2B584E40A0993E7BF0F20CC0@portablefip> References: <527C7219-72AE-4552-B8DD-43336F45923C@dimensionB.de> <1311094209355-3678624.post@n4.nabble.com> <1311095748965-3678689.post@n4.nabble.com> <1311097203523-3678755.post@n4.nabble.com> <973D0DBD2B584E40A0993E7BF0F20CC0@portablefip> Message-ID: <1311193810106-3681975.post@n4.nabble.com> It is a web app framework for revServer (Now named LiveCode Server). It is very much like CodeIgniter for PHP. -- View this message in context: http://runtime-revolution.278305.n4.nabble.com/ANN-revIgniter-v1-4b-tp3678359p3681975.html Sent from the Revolution - User mailing list archive at Nabble.com. From andre at andregarzia.com Wed Jul 20 16:34:42 2011 From: andre at andregarzia.com (Andre Garzia) Date: Wed, 20 Jul 2011 17:34:42 -0300 Subject: revIgniter In-Reply-To: <1311193810106-3681975.post@n4.nabble.com> References: <527C7219-72AE-4552-B8DD-43336F45923C@dimensionB.de> <1311094209355-3678624.post@n4.nabble.com> <1311095748965-3678689.post@n4.nabble.com> <1311097203523-3678755.post@n4.nabble.com> <973D0DBD2B584E40A0993E7BF0F20CC0@portablefip> <1311193810106-3681975.post@n4.nabble.com> Message-ID: On Wed, Jul 20, 2011 at 5:30 PM, Andrew Kluthe wrote: > It is a web app framework for revServer (Now named LiveCode Server). It is > very much like CodeIgniter for PHP. > let me correct that for you dear Sir, It is a *BEST* web app framework for revServer (Now named LiveCode Server). It is very much like CodeIgniter for PHP. thanks you, carry on, keep calm and stay with snow leopard. > -- > View this message in context: > http://runtime-revolution.278305.n4.nabble.com/ANN-revIgniter-v1-4b-tp3678359p3681975.html > Sent from the Revolution - User mailing list archive at Nabble.com. > > _______________________________________________ > use-livecode mailing list > use-livecode at lists.runrev.com > Please visit this url to subscribe, unsubscribe and manage your > subscription preferences: > http://lists.runrev.com/mailman/listinfo/use-livecode > -- http://www.andregarzia.com All We Do Is Code. From form at nonsanity.com Wed Jul 20 16:34:51 2011 From: form at nonsanity.com (Nonsanity) Date: Wed, 20 Jul 2011 16:34:51 -0400 Subject: Countdown HH:MM:SS In-Reply-To: References: <8CE14A04637B482-1AA4-201D3@webmail-d058.sysops.aol.com> <61EE076F-3EE3-42D4-A552-2ADF6526034A@sonsothunder.com> Message-ID: Or with the other features you mentioned: on mouseup -- start 1 hour timer button get the seconds put it into now convert now to internet date put now into fld 1 -- display the current time add 3600 to it -- one hour from now set the targetTime of me to it convert it to internet date put it into fld 2 -- display the target time UpdateCountdown end mouseup on UpdateCountdown get the targetTime of me if it is empty then exit UpdateCountdown put CountdownCalc( it ) into res if char 1 of res is "-" or res = "0:00:00" then -- timer ended set the targetTime of me to empty put "0:00:00" into res end if put res into fld 3 -- display the countdown -- always best to make sure this timer loop hasn't already been started if "UpdateDountdown" is not in the pendingmessages then send "UpdateCountdown" to me in 1 second end UpdateCountdown function CountdownCalc targetTime -- in seconds put targetTime - the seconds into deltaTime put trunc(deltaTime / 3600) into h -- hours put trunc((deltaTime - (h * 3600)) / 60) into m -- minutes put deltaTime - (h * 3600) - (m * 60) into s -- seconds put format( "%01i:%02i:%02i", h, m, s ) into countdown return countdown end CountdownCalc on MouseUp -- cancel button set the targetTime of me to empty end MouseUp ~ Chris Innanen ~ Nonsanity > On Wed, Jul 20, 2011 at 3:19 PM, Roger Eller wrote: > >> >> With only a few tweaks like handling a cancel/reset request, this script >> does what I need. James' was almost the one, but the time wasn't starting >> from a pre-set amount of time such as 1 hour, etc. Thanks to everyone >> that >> contributed! >> >> ~Roger >> > From pete at mollysrevenge.com Wed Jul 20 16:50:08 2011 From: pete at mollysrevenge.com (Pete) Date: Wed, 20 Jul 2011 13:50:08 -0700 Subject: Mac OS X Lion In-Reply-To: References: <4E271A6B.7000509@fourthworld.com> Message-ID: I agree and look forward seeing the results of the work that some LC developers are doing on making the missing controls available. Pete Molly's Revenge On Wed, Jul 20, 2011 at 1:19 PM, Andre Garzia wrote: > On Wed, Jul 20, 2011 at 5:13 PM, Pete wrote: > > > There's been recent discussion of just how many OSX controls are not > > natively available in LC and Lion just seems to add to that list. Will > LC > > ever catch up? > > > > LC is Carbon and has a lot of emulated stuff to make is cross-platform, > this > is a very hard race to win... :-) > > > > > Pete > > Molly's Revenge > > > > > > > > > > On Wed, Jul 20, 2011 at 12:38 PM, Andre Garzia > >wrote: > > > > > On Wed, Jul 20, 2011 at 3:11 PM, Richard Gaskin > > > wrote: > > > > > > > Andre wrote: > > > > > > > > I don't like lion or the new scrollbars... :-( > > > >> > > > > > > > > For better or worse, it seems those scrollbars are the wave of the > > > future: > > > > > > http://livecodejournal.com/blog.irv?pid=1307545265.586975> > > > > **> > > > > > > > > What concerns me more about Lion is how auto-save is supposed to work > > > with > > > > our apps.... > > > > > > > > > > I have no clue either... The new APIs are so Objective-C/Cocoa > specific, > > > using delegates and whatnot that I think it will be a tad hard for our > > > little carbon engine to leverage on them. > > > > > > > > > > > > > > > > > -- > > > > Richard Gaskin > > > > Fourth World > > > > LiveCode training and consulting: http://www.fourthworld.com > > > > Webzine for LiveCode developers: http://www.LiveCodeJournal.com > > > > LiveCode Journal blog: http://LiveCodejournal.com/**blog.irv< > > > http://LiveCodejournal.com/blog.irv> > > > > > > > > > > > > ______________________________**_________________ > > > > use-livecode mailing list > > > > use-livecode at lists.runrev.com > > > > Please visit this url to subscribe, unsubscribe and manage your > > > > subscription preferences: > > > > http://lists.runrev.com/**mailman/listinfo/use-livecode< > > > http://lists.runrev.com/mailman/listinfo/use-livecode> > > > > > > > > > > > > > > > > -- > > > http://www.andregarzia.com All We Do Is Code. > > > _______________________________________________ > > > use-livecode mailing list > > > use-livecode at lists.runrev.com > > > Please visit this url to subscribe, unsubscribe and manage your > > > subscription preferences: > > > http://lists.runrev.com/mailman/listinfo/use-livecode > > > > > > > > _______________________________________________ > > use-livecode mailing list > > use-livecode at lists.runrev.com > > Please visit this url to subscribe, unsubscribe and manage your > > subscription preferences: > > http://lists.runrev.com/mailman/listinfo/use-livecode > > > > > > -- > http://www.andregarzia.com All We Do Is Code. > _______________________________________________ > use-livecode mailing list > use-livecode at lists.runrev.com > Please visit this url to subscribe, unsubscribe and manage your > subscription preferences: > http://lists.runrev.com/mailman/listinfo/use-livecode > > From pete at mollysrevenge.com Wed Jul 20 17:31:01 2011 From: pete at mollysrevenge.com (Pete) Date: Wed, 20 Jul 2011 14:31:01 -0700 Subject: Mac OS X Lion In-Reply-To: References: <4E271A6B.7000509@fourthworld.com> Message-ID: I was just told the following about Lion: 1) It will only install correctly if you are on the very latest version of Snow Leopard (10.6.8) 2) Apple have discontinued Snow Leopard If that is true and you don't already have Snow Leopard installed, seems like you're screwed. Can anyone confirm this? Pete Molly's Revenge On Wed, Jul 20, 2011 at 1:50 PM, Pete wrote: > I agree and look forward seeing the results of the work that some LC > developers are doing on making the missing controls available. > Pete > Molly's Revenge > > > > > On Wed, Jul 20, 2011 at 1:19 PM, Andre Garzia wrote: > >> On Wed, Jul 20, 2011 at 5:13 PM, Pete wrote: >> >> > There's been recent discussion of just how many OSX controls are not >> > natively available in LC and Lion just seems to add to that list. Will >> LC >> > ever catch up? >> > >> >> LC is Carbon and has a lot of emulated stuff to make is cross-platform, >> this >> is a very hard race to win... :-) >> >> >> >> > Pete >> > Molly's Revenge >> > >> > >> > >> > >> > On Wed, Jul 20, 2011 at 12:38 PM, Andre Garzia > > >wrote: >> > >> > > On Wed, Jul 20, 2011 at 3:11 PM, Richard Gaskin >> > > wrote: >> > > >> > > > Andre wrote: >> > > > >> > > > I don't like lion or the new scrollbars... :-( >> > > >> >> > > > >> > > > For better or worse, it seems those scrollbars are the wave of the >> > > future: >> > > > > > > http://livecodejournal.com/blog.irv?pid=1307545265.586975> >> > > > **> >> > > > >> > > > What concerns me more about Lion is how auto-save is supposed to >> work >> > > with >> > > > our apps.... >> > > > >> > > >> > > I have no clue either... The new APIs are so Objective-C/Cocoa >> specific, >> > > using delegates and whatnot that I think it will be a tad hard for our >> > > little carbon engine to leverage on them. >> > > >> > > >> > > >> > > > >> > > > -- >> > > > Richard Gaskin >> > > > Fourth World >> > > > LiveCode training and consulting: http://www.fourthworld.com >> > > > Webzine for LiveCode developers: http://www.LiveCodeJournal.com >> > > > LiveCode Journal blog: http://LiveCodejournal.com/**blog.irv< >> > > http://LiveCodejournal.com/blog.irv> >> > > > >> > > > >> > > > ______________________________**_________________ >> > > > use-livecode mailing list >> > > > use-livecode at lists.runrev.com >> > > > Please visit this url to subscribe, unsubscribe and manage your >> > > > subscription preferences: >> > > > http://lists.runrev.com/**mailman/listinfo/use-livecode< >> > > http://lists.runrev.com/mailman/listinfo/use-livecode> >> > > > >> > > >> > > >> > > >> > > -- >> > > http://www.andregarzia.com All We Do Is Code. >> > > _______________________________________________ >> > > use-livecode mailing list >> > > use-livecode at lists.runrev.com >> > > Please visit this url to subscribe, unsubscribe and manage your >> > > subscription preferences: >> > > http://lists.runrev.com/mailman/listinfo/use-livecode >> > > >> > > >> > _______________________________________________ >> > use-livecode mailing list >> > use-livecode at lists.runrev.com >> > Please visit this url to subscribe, unsubscribe and manage your >> > subscription preferences: >> > http://lists.runrev.com/mailman/listinfo/use-livecode >> > >> >> >> >> -- >> http://www.andregarzia.com All We Do Is Code. >> _______________________________________________ >> use-livecode mailing list >> use-livecode at lists.runrev.com >> Please visit this url to subscribe, unsubscribe and manage your >> subscription preferences: >> http://lists.runrev.com/mailman/listinfo/use-livecode >> >> > From andre at andregarzia.com Wed Jul 20 17:45:09 2011 From: andre at andregarzia.com (Andre Garzia) Date: Wed, 20 Jul 2011 18:45:09 -0300 Subject: Mac OS X Lion In-Reply-To: References: <4E271A6B.7000509@fourthworld.com> Message-ID: Apple would not discontinue snow leopard, they support OSes for a long time... On Wed, Jul 20, 2011 at 6:31 PM, Pete wrote: > I was just told the following about Lion: > > 1) It will only install correctly if you are on the very latest version of > Snow Leopard (10.6.8) > 2) Apple have discontinued Snow Leopard > > If that is true and you don't already have Snow Leopard installed, seems > like you're screwed. > > Can anyone confirm this? > > Pete > Molly's Revenge > > > > > On Wed, Jul 20, 2011 at 1:50 PM, Pete wrote: > > > I agree and look forward seeing the results of the work that some LC > > developers are doing on making the missing controls available. > > Pete > > Molly's Revenge > > > > > > > > > > On Wed, Jul 20, 2011 at 1:19 PM, Andre Garzia >wrote: > > > >> On Wed, Jul 20, 2011 at 5:13 PM, Pete wrote: > >> > >> > There's been recent discussion of just how many OSX controls are not > >> > natively available in LC and Lion just seems to add to that list. > Will > >> LC > >> > ever catch up? > >> > > >> > >> LC is Carbon and has a lot of emulated stuff to make is cross-platform, > >> this > >> is a very hard race to win... :-) > >> > >> > >> > >> > Pete > >> > Molly's Revenge > >> > > >> > > >> > > >> > > >> > On Wed, Jul 20, 2011 at 12:38 PM, Andre Garzia >> > >wrote: > >> > > >> > > On Wed, Jul 20, 2011 at 3:11 PM, Richard Gaskin > >> > > wrote: > >> > > > >> > > > Andre wrote: > >> > > > > >> > > > I don't like lion or the new scrollbars... :-( > >> > > >> > >> > > > > >> > > > For better or worse, it seems those scrollbars are the wave of the > >> > > future: > >> > > > >> > > http://livecodejournal.com/blog.irv?pid=1307545265.586975> > >> > > > **> > >> > > > > >> > > > What concerns me more about Lion is how auto-save is supposed to > >> work > >> > > with > >> > > > our apps.... > >> > > > > >> > > > >> > > I have no clue either... The new APIs are so Objective-C/Cocoa > >> specific, > >> > > using delegates and whatnot that I think it will be a tad hard for > our > >> > > little carbon engine to leverage on them. > >> > > > >> > > > >> > > > >> > > > > >> > > > -- > >> > > > Richard Gaskin > >> > > > Fourth World > >> > > > LiveCode training and consulting: http://www.fourthworld.com > >> > > > Webzine for LiveCode developers: http://www.LiveCodeJournal.com > >> > > > LiveCode Journal blog: http://LiveCodejournal.com/**blog.irv< > >> > > http://LiveCodejournal.com/blog.irv> > >> > > > > >> > > > > >> > > > ______________________________**_________________ > >> > > > use-livecode mailing list > >> > > > use-livecode at lists.runrev.com > >> > > > Please visit this url to subscribe, unsubscribe and manage your > >> > > > subscription preferences: > >> > > > http://lists.runrev.com/**mailman/listinfo/use-livecode< > >> > > http://lists.runrev.com/mailman/listinfo/use-livecode> > >> > > > > >> > > > >> > > > >> > > > >> > > -- > >> > > http://www.andregarzia.com All We Do Is Code. > >> > > _______________________________________________ > >> > > use-livecode mailing list > >> > > use-livecode at lists.runrev.com > >> > > Please visit this url to subscribe, unsubscribe and manage your > >> > > subscription preferences: > >> > > http://lists.runrev.com/mailman/listinfo/use-livecode > >> > > > >> > > > >> > _______________________________________________ > >> > use-livecode mailing list > >> > use-livecode at lists.runrev.com > >> > Please visit this url to subscribe, unsubscribe and manage your > >> > subscription preferences: > >> > http://lists.runrev.com/mailman/listinfo/use-livecode > >> > > >> > >> > >> > >> -- > >> http://www.andregarzia.com All We Do Is Code. > >> _______________________________________________ > >> use-livecode mailing list > >> use-livecode at lists.runrev.com > >> Please visit this url to subscribe, unsubscribe and manage your > >> subscription preferences: > >> http://lists.runrev.com/mailman/listinfo/use-livecode > >> > >> > > > _______________________________________________ > use-livecode mailing list > use-livecode at lists.runrev.com > Please visit this url to subscribe, unsubscribe and manage your > subscription preferences: > http://lists.runrev.com/mailman/listinfo/use-livecode > -- http://www.andregarzia.com All We Do Is Code. From andre at andregarzia.com Wed Jul 20 17:47:46 2011 From: andre at andregarzia.com (Andre Garzia) Date: Wed, 20 Jul 2011 18:47:46 -0300 Subject: [ANN] New plugin AAG|Workspaces Message-ID: Folks, I've just launched a little plugin. I've used it since forever, hope you guys find it useful. Check out at http://andregarzia.com/page/aagworkspaces Basically it allows you to create "workspaces". Each workspace is a collection of URLs and Files (that can be stacks) that are open when you are in that workspace, so when you switch projects, instead of loosing a lot of time reopening pages and files, you can simply select the workspace in there and all the files and URLs will open. It sounds silly but it is handy. :-) -- http://www.andregarzia.com All We Do Is Code. From francois.chaplais at mines-paristech.fr Wed Jul 20 18:05:08 2011 From: francois.chaplais at mines-paristech.fr (=?iso-8859-1?Q?Fran=E7ois_Chaplais?=) Date: Thu, 21 Jul 2011 00:05:08 +0200 Subject: Mac OS X Lion In-Reply-To: References: <4E271A6B.7000509@fourthworld.com> Message-ID: Le 20 juil. 2011 ? 23:45, Andre Garzia a ?crit : > Apple would not discontinue snow leopard, they support OSes for a long > time... > when I learned that Lion requires snow leopard (which I do not have on all of my machines) I cautiously ordered a copy of snow. Hardest to find is System 7.x for my powermac 8500 ;) From gerry.orkin at gmail.com Wed Jul 20 18:17:14 2011 From: gerry.orkin at gmail.com (Gerry Orkin) Date: Thu, 21 Jul 2011 08:17:14 +1000 Subject: Mac OS X Lion In-Reply-To: References: <4E271A6B.7000509@fourthworld.com> Message-ID: When I try to test an iOS standalone I get an error message: Unknow deployment platform. Any ideas? Gerry From scott at tactilemedia.com Wed Jul 20 18:20:37 2011 From: scott at tactilemedia.com (Scott Rossi) Date: Wed, 20 Jul 2011 15:20:37 -0700 Subject: Countdown HH:MM:SS In-Reply-To: Message-ID: Recently, Nonsanity wrote: > I wouldn't rely on the send in time structure to actually fire every second. > It's going to slip over time as the computer takes periodic spikes of use. > If you want New Years Eve accuracy of the final countdown, you should > probably re-calculate the delta based on the current time continuously. Agreed. A few years back, Geoff Canyon came up with the following which has been reliable for me: send "someMsg" to someObj in (1 - (the long seconds mod 1)) seconds Regards, Scott Rossi Creative Director Tactile Media, UX Design From lists.pete at haworths.org Wed Jul 20 18:22:33 2011 From: lists.pete at haworths.org (Pete Haworth) Date: Wed, 20 Jul 2011 15:22:33 -0700 Subject: Mac OS X Lion In-Reply-To: References: <4E271A6B.7000509@fourthworld.com> Message-ID: I think the info I got meant they stopped selling it, not supporting it. It seemed strange to me also, that's why I was asking if anyone had run into this. I'm already on Snow Leopard so not a problem for me. Pete On Wed, Jul 20, 2011 at 2:45 PM, Andre Garzia wrote: > Apple would not discontinue snow leopard, they support OSes for a long > time... > > From bvlahos at mac.com Wed Jul 20 18:13:40 2011 From: bvlahos at mac.com (Bill Vlahos) Date: Wed, 20 Jul 2011 15:13:40 -0700 Subject: Mac OS X Lion In-Reply-To: <4E271A6B.7000509@fourthworld.com> References: <4E271A6B.7000509@fourthworld.com> Message-ID: <9970C968-A8AA-4B0A-A7C2-5D7D82374675@mac.com> Richard, I believe auto-save only works on applications that specifically support the new API. Bill Vlahos Sent from my iPhone On Jul 20, 2011, at 11:11 AM, Richard Gaskin wrote: > Andre wrote: > >> I don't like lion or the new scrollbars... :-( > > For better or worse, it seems those scrollbars are the wave of the future: > > > What concerns me more about Lion is how auto-save is supposed to work with our apps.... > > -- > Richard Gaskin > Fourth World > LiveCode training and consulting: http://www.fourthworld.com > Webzine for LiveCode developers: http://www.LiveCodeJournal.com > LiveCode Journal blog: http://LiveCodejournal.com/blog.irv > > _______________________________________________ > use-livecode mailing list > use-livecode at lists.runrev.com > Please visit this url to subscribe, unsubscribe and manage your subscription preferences: > http://lists.runrev.com/mailman/listinfo/use-livecode From jhj at jhj.com Wed Jul 20 18:37:55 2011 From: jhj at jhj.com (Jerry J) Date: Wed, 20 Jul 2011 15:37:55 -0700 Subject: Mac OS X Lion In-Reply-To: References: <4E271A6B.7000509@fourthworld.com> Message-ID: <6938341D-4427-4B89-B95E-D082B0F8DFF2@jhj.com> I just checked the Apple Store (online) and Snow Leopard is still for sale there. I think they mean that new computers will not ship with Snow Leopard any longer. The Snow Leopard in the store is an upgrade from Leopard, which is the way it has always been unless you bought the more expensive Box Set that would install SL from scratch. A point of interest is that the Box Set version will NOT install on my recent MBP - must use the DVD that came with it (which will not install on older MBPs). I also saw that soon, next month I think, Apple will offer Lion on a USB flash stick. --Jerry Jensen On Jul 20, 2011, at 3:22 PM, Pete Haworth wrote: > I think the info I got meant they stopped selling it, not supporting it. It > seemed strange to me also, that's why I was asking if anyone had run into > this. I'm already on Snow Leopard so not a problem for me. > Pete > > On Wed, Jul 20, 2011 at 2:45 PM, Andre Garzia wrote: > >> Apple would not discontinue snow leopard, they support OSes for a long >> time... >> >> > _______________________________________________ > use-livecode mailing list > use-livecode at lists.runrev.com > Please visit this url to subscribe, unsubscribe and manage your subscription preferences: > http://lists.runrev.com/mailman/listinfo/use-livecode From shaosean at wehostmacs.com Wed Jul 20 20:49:02 2011 From: shaosean at wehostmacs.com (Shao Sean) Date: Wed, 20 Jul 2011 20:49:02 -0400 Subject: Mac OS X Lion Message-ID: > There's been recent discussion of just how many OSX controls are not > natively available in LC and Lion just seems to add to that list. > Will LC ever catch up? Probably not.. Seems they will not touch anything that is not cross platform.. From roger.e.eller at sealedair.com Wed Jul 20 20:54:39 2011 From: roger.e.eller at sealedair.com (Roger Eller) Date: Wed, 20 Jul 2011 20:54:39 -0400 Subject: Countdown HH:MM:SS In-Reply-To: References: Message-ID: On Wed, Jul 20, 2011 at 6:20 PM, Scott Rossi wrote: > Recently, Nonsanity wrote: > > > I wouldn't rely on the send in time structure to actually fire every > second. > > It's going to slip over time as the computer takes periodic spikes of > use. > > If you want New Years Eve accuracy of the final countdown, you should > > probably re-calculate the delta based on the current time continuously. > > Agreed. A few years back, Geoff Canyon came up with the following which > has > been reliable for me: > > send "someMsg" to someObj in (1 - (the long seconds mod 1)) seconds > > Regards, > > Scott Rossi > Creative Director > Tactile Media, UX Design > The variety of flavors in script methods to achieve a countdown timer are obviously many. The first place I looked for such was in revOnline (user samples). Would those of you in this thread that contributed a version of a solution (Ken, Chris, James, others) be ok with me compiling these into a stack and posting it there? Of course credits would be given to each scriptor for their contribution. I'm thinking this would be useful to both new and seasoned devs. Thanks guys! ?Roger From kray at sonsothunder.com Wed Jul 20 22:02:11 2011 From: kray at sonsothunder.com (Ken Ray) Date: Wed, 20 Jul 2011 21:02:11 -0500 Subject: Countdown HH:MM:SS In-Reply-To: References: Message-ID: <44EE9B1B-25C7-45B3-94B5-A62C5D80F6AE@sonsothunder.com> > > The variety of flavors in script methods to achieve a countdown timer are > obviously many. The first place I looked for such was in revOnline (user > samples). Would those of you in this thread that contributed a version of a > solution (Ken, Chris, James, others) be ok with me compiling these into a > stack and posting it there? Of course credits would be given to each > scriptor for their contribution. I'm thinking this would be useful to both > new and seasoned devs. Thanks guys! Fine by me! Ken Ray Sons of Thunder Software, Inc. Email: kray at sonsothunder.com Web Site: http://www.sonsothunder.com/ From stephenREVOLUTION2 at barncard.com Wed Jul 20 23:22:05 2011 From: stephenREVOLUTION2 at barncard.com (stephen barncard) Date: Wed, 20 Jul 2011 22:22:05 -0500 Subject: [ANN] New plugin AAG|Workspaces In-Reply-To: References: Message-ID: Thanks Andre - I've wanted this since I moved over to using what is now called Remo. Now everyone can have it. sqb On 20 July 2011 16:47, Andre Garzia wrote: > Folks, > > I've just launched a little plugin. I've used it since forever, hope you > guys find it useful. > > Check out at http://andregarzia.com/page/aagworkspaces > > Basically it allows you to create "workspaces". Each workspace is a > collection of URLs and Files (that can be stacks) that are open when you > are > in that workspace, so when you switch projects, instead of loosing a lot of > time reopening pages and files, you can simply select the workspace in > there > and all the files and URLs will open. It sounds silly but it is handy. > > :-) > > -- > http://www.andregarzia.com All We Do Is Code. > _______________________________________________ > use-livecode mailing list > use-livecode at lists.runrev.com > Please visit this url to subscribe, unsubscribe and manage your > subscription preferences: > http://lists.runrev.com/mailman/listinfo/use-livecode > -- Stephen Barncard San Francisco Ca. USA more about sqb From tvogelaar at de-mare.nl Thu Jul 21 00:45:58 2011 From: tvogelaar at de-mare.nl (Terry Vogelaar) Date: Thu, 21 Jul 2011 06:45:58 +0200 Subject: Lion & Xcode 3.2.6 In-Reply-To: References: Message-ID: Hi there, Is anyone experiencing this same problem with Lion? I cannot save a stack as a standalone for iOS anymore since upgrading to Lion. When I go to the preferences and go under Mobile Support and choose 'Location of developer root for iOS 3.2 and above', it says '/Developer' like it should. When I browse to reselect it, it says: 'The chosen folder is not a valid iOS SDK for 3.2 and later. It must be the one that ships with XCode 3.2.4 or later.' I also tried installing Xcode 4.1 for Lion, which is the preferred version for Lion, but that is not supported by RunRev (yet). I'm on Developer Preview 4 of Lion. I wanted to upgrade yesterday, but I couldn't, because it is 'Already installed' according to the App Store. So I will download it at work and bring it here on an USB stick. Hopefully this resolves these issues. Terry From warren at warrensweb.us Thu Jul 21 01:40:11 2011 From: warren at warrensweb.us (Warren Samples) Date: Thu, 21 Jul 2011 00:40:11 -0500 Subject: Lion & Xcode 3.2.6 In-Reply-To: References: Message-ID: <201107210040.11741.warren@warrensweb.us> On Wednesday, July 20, 2011 11:45:58 PM Terry Vogelaar wrote: > I wanted to upgrade yesterday, but I couldn't, because it is 'Already > installed' according to the App Store. Not to make light of the problems you personally are having, Terry, but would this be the same "we provide a seamless path to software installation and upgrade, for the ultimate user experience" Apple App Store that so many people are so excited about? Lordy! Warren From bvlahos at mac.com Thu Jul 21 01:56:13 2011 From: bvlahos at mac.com (Bill Vlahos) Date: Wed, 20 Jul 2011 22:56:13 -0700 Subject: age calculation on Windows vs Mac In-Reply-To: <5E93AEDF-9294-4C6F-9865-87F940B9B264@twft.com> References: <5E93AEDF-9294-4C6F-9865-87F940B9B264@twft.com> Message-ID: Bob, I'm glad you like InfoWallet. Thanks for the kind words. This code originally came from this awesome list. Bill Vlahos _________________ InfoWallet (http://www.infowallet.com) is about keeping your important life information with you, accessible, and secure. On Jul 20, 2011, at 9:21 AM, Bob Sneidar wrote: > I purchased Infowallet, (it's awesome) and so feel it's alright to scavenge your bit of code. :-) > > Bob > > > On Jul 19, 2011, at 4:24 PM, Bill Vlahos wrote: > >> The problem is the epoch date in Windows is 1970 and LiveCode can't deal with dates earlier than that in Windows. You will need to use a VBScript. >> >> Here is what I use in InfoWallet which handles Windows, Mac, and Linux correctly which was based on code provided by folks on this list. >> >> on calcAge tDate,asOf >> -- returns the current age based on a DOB of tDate >> -- as of the date asOf, or if asOf = empty, as of today >> if tDate = empty then return empty >> if the platform is "WIn32" then >> isADate tDate >> if the result then >> put "sdate = " & quote & tDate & quote into vFive >> put return & "result = DateDiff(" & quote & "d" & quote & ",sdate,Date)" after vFive >> put return & "trueage = result / 365" after vFive >> put return & "result = Left(trueage,2)" after vFive >> do vFive as vbscript >> return the result >> else >> return empty >> end if >> else >> if tDate is not a date then return empty >> if asOf = empty then >> put the date into asOf >> end if >> set the itemdelimiter to "/" >> put item -1 of the short date into nowYr >> set the centurycutoff to nowYr+1 >> set the itemdelimiter to comma >> convert tDate to dateItems >> convert asOf to dateitems >> put item 1 of asOf - item 1 of tDate - 1 into tAge >> put item 1 of asOf into item 1 of tDate >> convert tDate to seconds >> convert asOf to seconds >> if tDate <= asOf then add 1 to tAge >> return tAge >> end if >> end calcAge >> >> >> Bill Vlahos >> _________________ >> InfoWallet (http://www.infowallet.com) is about keeping your important life information with you, accessible, and secure. >> >> On Jul 19, 2011, at 9:23 AM, Andr? Bisseret wrote: >> >>> Bonjour, >>> >>> I am a Mac user but I recently bought a PC (windows 7) in order to test my standalones for windows more quickly. >>> >>> I am fighting for hours now with the following problem : >>> >>> In an app., I have a handler that calculates the age from the short birth date (thanks to Jim Ault who gave it on the list). >>> The birth date is jj/mm/AAAA or jj/mm/AA as well. >>> >>> This handler works fine on Mac, but not on windows. >>> >>> On windows, the age is invariably 100 if the birth year < 1970. (with JJ/MM/AA or JJ/MM/AAAA (as bad ;-( >>> If the birth year > = 1970 then the age is correct. >>> >>> I tried to play with the centuryCutOff but without any success (on windows). >>> >>> I am not familiar with Windows : what am I missing ? Is it a matter of computer setting or should I program differently for windows than for mac !? >>> >>> Thanks a lot in advance for any help >>> >>> Best regards from Grenoble >>> >>> Andr? >>> >>> P.S. By the way, I used to use a search engine (if I well remember, it was in the help menu of livecode) I do not find it any more; has it been suppressed ? (or here again, I am missing something !) >>> _______________________________________________ >>> use-livecode mailing list >>> use-livecode at lists.runrev.com >>> Please visit this url to subscribe, unsubscribe and manage your subscription preferences: >>> http://lists.runrev.com/mailman/listinfo/use-livecode >> >> >> _______________________________________________ >> use-livecode mailing list >> use-livecode at lists.runrev.com >> Please visit this url to subscribe, unsubscribe and manage your subscription preferences: >> http://lists.runrev.com/mailman/listinfo/use-livecode > > > _______________________________________________ > use-livecode mailing list > use-livecode at lists.runrev.com > Please visit this url to subscribe, unsubscribe and manage your subscription preferences: > http://lists.runrev.com/mailman/listinfo/use-livecode From m.schonewille at economy-x-talk.com Thu Jul 21 05:12:59 2011 From: m.schonewille at economy-x-talk.com (Mark Schonewille) Date: Thu, 21 Jul 2011 11:12:59 +0200 Subject: Lion & Xcode 3.2.6 In-Reply-To: References: Message-ID: <234EA56A-4C4D-40B8-BC2D-A0A03C33039F@economy-x-talk.com> Hi Terry, Yes, it is possible that the app store can't distinguish a dev preview and the golden master. Downloading Lion directly from your Apple dev account is the right thing to do. I have no trouble building standalones for iOS with XCode 4. It may be unspupported but it works. -- Best regards, Mark Schonewille Economy-x-Talk Consulting and Software Engineering Homepage: http://economy-x-talk.com Twitter: http://twitter.com/xtalkprogrammer KvK: 50277553 New: Download the Installer Maker Plugin 1.6 for LiveCode here http://qery.us/ce On 21 jul 2011, at 06:45, Terry Vogelaar wrote: > Hi there, > > Is anyone experiencing this same problem with Lion? > > I cannot save a stack as a standalone for iOS anymore since upgrading to Lion. When I go to the preferences and go under Mobile Support and choose 'Location of developer root for iOS 3.2 and above', it says '/Developer' like it should. When I browse to reselect it, it says: 'The chosen folder is not a valid iOS SDK for 3.2 and later. It must be the one that ships with XCode 3.2.4 or later.' > > I also tried installing Xcode 4.1 for Lion, which is the preferred version for Lion, but that is not supported by RunRev (yet). > > I'm on Developer Preview 4 of Lion. I wanted to upgrade yesterday, but I couldn't, because it is 'Already installed' according to the App Store. So I will download it at work and bring it here on an USB stick. Hopefully this resolves these issues. > > Terry From gerry.orkin at gmail.com Thu Jul 21 05:47:35 2011 From: gerry.orkin at gmail.com (Gerry Orkin) Date: Thu, 21 Jul 2011 19:47:35 +1000 Subject: Lion & Xcode 3.2.6 In-Reply-To: <234EA56A-4C4D-40B8-BC2D-A0A03C33039F@economy-x-talk.com> References: <234EA56A-4C4D-40B8-BC2D-A0A03C33039F@economy-x-talk.com> Message-ID: <7F365666-56D0-4C11-9A83-8A13A92AD8CA@gmail.com> 4.0? It's broken in 4.1 for me. Gerry On 21/07/2011, at 7:12 PM, Mark Schonewille wrote: > I have no trouble building standalones for iOS with XCode 4. It may be unspupported but it works. From bmmeili at swissonline.ch Thu Jul 21 05:50:08 2011 From: bmmeili at swissonline.ch (martin meili) Date: Thu, 21 Jul 2011 11:50:08 +0200 Subject: save formatted text in a file Message-ID: Hi How can I avoid loosing the formatting of a text, if I save this text to a file? I usually save text in a file as follows: open file the_path_Var for write write TextVar to file the_path_Var ---TextVar contains the formatted text of a textfield; in the clipboard it can be seen as formatted text close file the_path_Var Thanks for any replies! Cheers Martin From m.schonewille at economy-x-talk.com Thu Jul 21 06:08:02 2011 From: m.schonewille at economy-x-talk.com (Mark Schonewille) Date: Thu, 21 Jul 2011 12:08:02 +0200 Subject: Lion & Xcode 3.2.6 In-Reply-To: <7F365666-56D0-4C11-9A83-8A13A92AD8CA@gmail.com> References: <234EA56A-4C4D-40B8-BC2D-A0A03C33039F@economy-x-talk.com> <7F365666-56D0-4C11-9A83-8A13A92AD8CA@gmail.com> Message-ID: Oh, I see, Gerry, I am using XCode 4.0.2 at the moment. -- Best regards, Mark Schonewille Economy-x-Talk Consulting and Software Engineering Homepage: http://economy-x-talk.com Twitter: http://twitter.com/xtalkprogrammer KvK: 50277553 New: Download the Installer Maker Plugin 1.6 for LiveCode here http://qery.us/ce On 21 jul 2011, at 11:47, Gerry Orkin wrote: > 4.0? It's broken in 4.1 for me. > > Gerry > > > On 21/07/2011, at 7:12 PM, Mark Schonewille wrote: > >> I have no trouble building standalones for iOS with XCode 4. It may be unspupported but it works. From m.schonewille at economy-x-talk.com Thu Jul 21 06:19:11 2011 From: m.schonewille at economy-x-talk.com (Mark Schonewille) Date: Thu, 21 Jul 2011 12:19:11 +0200 Subject: save formatted text in a file In-Reply-To: References: Message-ID: <57849C38-D174-49A3-B914-05D04DA4B3C8@economy-x-talk.com> Hi Martin, What do you want to do with your text? If you want to use it in a text editor, then it is often useful to keep unformatted text. If you want to import formatted text into Word or Pages, you could use the rtfText property. put the rtfText of field "Some Field" into myData open file myPath for binary write write myData to file myPath close file myPath Make sure to use binary write. Btw you probably will want to use the .rtf file extension instead of .txt. -- Best regards, Mark Schonewille Economy-x-Talk Consulting and Software Engineering Homepage: http://economy-x-talk.com Twitter: http://twitter.com/xtalkprogrammer KvK: 50277553 New: Download the Installer Maker Plugin 1.6 for LiveCode here http://qery.us/ce On 21 jul 2011, at 11:50, martin meili wrote: > Hi > How can I avoid loosing the formatting of a text, if I save this text to a file? > > I usually save text in a file as follows: > > open file the_path_Var for write > write TextVar to file the_path_Var ---TextVar contains the formatted text of a textfield; in the clipboard it can be seen as formatted text > close file the_path_Var > > Thanks for any replies! > > Cheers > Martin From jhurley0305 at sbcglobal.net Thu Jul 21 06:29:26 2011 From: jhurley0305 at sbcglobal.net (James Hurley) Date: Thu, 21 Jul 2011 03:29:26 -0700 Subject: Countdown HH:MM:SS In-Reply-To: References: Message-ID: > > Message: 15 > Date: Wed, 20 Jul 2011 15:19:07 -0400 > From: Roger Eller > To: How to use LiveCode > Subject: Re: Countdown HH:MM:SS > Message-ID: > > Content-Type: text/plain; charset=ISO-8859-1 > > On Wed, Jul 20, 2011 at 1:06 PM, Ken Ray wrote: > >>> Using the dateItems is definitely the right direction, but my objective >> is >>> to count backwards from for example; 2 hours, 45 minutes, and 59 seconds >>> while updating the display field every second. When the countdown >> reaches >>> 00:00:00, I would play a sound of do other actions. >> >> Sorry about that... this will work if you just want to count down from a >> fixed amount of time: >> >> on mouseUp >> DoCountDown "2:45:59" >> end mouseUp >> >> on DoCountDown pTime >> convert pTime to seconds >> subtract 1 from pTime >> convert pTime to dateItems >> split pTime by "," >> put format("%02d:%02d:%02d",pTime[4],pTime[5],pTime[6]) into tNewTime >> put tNewTime into fld "Time" >> if tNewTime = "00:00:00" then >> TimesUp >> else >> send "DoCountDown tNewTime" to me in 1 second >> end if >> end DoCountDown >> >> on TimesUp >> answer "Done" >> end TimesUp >> >> Ken Ray >> Sons of Thunder Software, Inc. >> Email: kray at sonsothunder.com >> Web Site: http://www.sonsothunder.com/ >> >> > With only a few tweaks like handling a cancel/reset request, this script > does what I need. James' was almost the one, but the time wasn't starting > from a pre-set amount of time such as 1 hour, etc. Thanks to everyone that > contributed! > > ~Roger > Roger, Good decision. Ken's solution is MUCH cleaner. Jim Hurley From gerry.orkin at gmail.com Thu Jul 21 06:32:01 2011 From: gerry.orkin at gmail.com (Gerry Orkin) Date: Thu, 21 Jul 2011 20:32:01 +1000 Subject: Lion & Xcode 3.2.6 In-Reply-To: References: <234EA56A-4C4D-40B8-BC2D-A0A03C33039F@economy-x-talk.com> <7F365666-56D0-4C11-9A83-8A13A92AD8CA@gmail.com> Message-ID: I wonder if I can downgrade? g On 21/07/2011, at 8:08 PM, Mark Schonewille wrote: > Oh, I see, Gerry, I am using XCode 4.0.2 at the moment. > From m.schonewille at economy-x-talk.com Thu Jul 21 06:48:35 2011 From: m.schonewille at economy-x-talk.com (Mark Schonewille) Date: Thu, 21 Jul 2011 12:48:35 +0200 Subject: Lion & Xcode 3.2.6 In-Reply-To: References: <234EA56A-4C4D-40B8-BC2D-A0A03C33039F@economy-x-talk.com> <7F365666-56D0-4C11-9A83-8A13A92AD8CA@gmail.com> Message-ID: <1BBF6997-35E1-4969-AF43-1A7C0E46291D@economy-x-talk.com> Hi, It is possible to use mutliple versions of XCode next to each other, but XCode seems to stop working at some point, while you keep upgrading to newer system versions. You should be able to install multiple SDK's though. If you're lucky, you're only missing an older SDK and will be able to use LC for iOS again after installing that SDK. I think there are a few support documents on Apple's site about it. What error do you see exactly, when you try to build a standalone for iOS? -- Best regards, Mark Schonewille Economy-x-Talk Consulting and Software Engineering Homepage: http://economy-x-talk.com Twitter: http://twitter.com/xtalkprogrammer KvK: 50277553 New: Download the Installer Maker Plugin 1.6 for LiveCode here http://qery.us/ce On 21 jul 2011, at 12:32, Gerry Orkin wrote: > I wonder if I can downgrade? > > g > > > On 21/07/2011, at 8:08 PM, Mark Schonewille wrote: > >> Oh, I see, Gerry, I am using XCode 4.0.2 at the moment. From gerry.orkin at gmail.com Thu Jul 21 07:30:38 2011 From: gerry.orkin at gmail.com (Gerry Orkin) Date: Thu, 21 Jul 2011 21:30:38 +1000 Subject: Lion & Xcode 3.2.6 In-Reply-To: <1BBF6997-35E1-4969-AF43-1A7C0E46291D@economy-x-talk.com> References: <234EA56A-4C4D-40B8-BC2D-A0A03C33039F@economy-x-talk.com> <7F365666-56D0-4C11-9A83-8A13A92AD8CA@gmail.com> <1BBF6997-35E1-4969-AF43-1A7C0E46291D@economy-x-talk.com> Message-ID: <54A3F4BF-3BC2-4205-8EC9-89C816745C95@gmail.com> Mark Sorry, I have been asking about this on a couple of lists, and lost track of where the conversation was up to here?my problem is in getting the simulator running. I can save iOS standalones, but I can't test them. The Test button works but gives me the error "Unknown deployment platform" when I click it. The Test and Test Target menu items are disabled. I'm running the 4.6.3 GM of LC and xCode 4.1 (4.0.1 refused to open in Lion). Cheers Gerry On 21/07/2011, at 8:48 PM, Mark Schonewille wrote: > What error do you see exactly, when you try to build a standalone for iOS? From len-morgan at crcom.net Thu Jul 21 07:34:44 2011 From: len-morgan at crcom.net (Len Morgan) Date: Thu, 21 Jul 2011 06:34:44 -0500 Subject: on-Rev client software Message-ID: <4E280ED4.90502@crcom.net> I've been one of "founders" with on-rev since the offer was first made but haven't really had the time to get into it until now. I've had web sites there but never used Livecode to make them. Now is the time. Has runRev ever released an update to the "on-rev" desktop client? If so, where can I find it. Also, is there ANY kind of tutorial, manual, web page, cave painting, that I can read that will explain how to use it? Thanks! len From m.schonewille at economy-x-talk.com Thu Jul 21 07:38:14 2011 From: m.schonewille at economy-x-talk.com (Mark Schonewille) Date: Thu, 21 Jul 2011 13:38:14 +0200 Subject: Lion & Xcode 3.2.6 In-Reply-To: <54A3F4BF-3BC2-4205-8EC9-89C816745C95@gmail.com> References: <234EA56A-4C4D-40B8-BC2D-A0A03C33039F@economy-x-talk.com> <7F365666-56D0-4C11-9A83-8A13A92AD8CA@gmail.com> <1BBF6997-35E1-4969-AF43-1A7C0E46291D@economy-x-talk.com> <54A3F4BF-3BC2-4205-8EC9-89C816745C95@gmail.com> Message-ID: <82410BA5-56A6-4F3F-80A6-B80CEBDEE50D@economy-x-talk.com> Hi Gerry, Usually, that error appears if you haven't selected a platform in the standalone builder. It probably means that XCode isn't even involved yet. If the error still occurs after selecting the iOS platform in the standalone builder, try an older version of LiveCode (4.6.1). -- Best regards, Mark Schonewille Economy-x-Talk Consulting and Software Engineering Homepage: http://economy-x-talk.com Twitter: http://twitter.com/xtalkprogrammer KvK: 50277553 New: Download the Installer Maker Plugin 1.6 for LiveCode here http://qery.us/ce On 21 jul 2011, at 13:30, Gerry Orkin wrote: > Mark > > Sorry, I have been asking about this on a couple of lists, and lost track of where the conversation was up to here?my problem is in getting the simulator running. I can save iOS standalones, but I can't test them. > > The Test button works but gives me the error "Unknown deployment platform" when I click it. The Test and Test Target menu items are disabled. I'm running the 4.6.3 GM of LC and xCode 4.1 (4.0.1 refused to open in Lion). > > Cheers > > Gerry From gerry.orkin at gmail.com Thu Jul 21 08:27:44 2011 From: gerry.orkin at gmail.com (Gerry Orkin) Date: Thu, 21 Jul 2011 22:27:44 +1000 Subject: Lion & Xcode 3.2.6 In-Reply-To: <82410BA5-56A6-4F3F-80A6-B80CEBDEE50D@economy-x-talk.com> References: <234EA56A-4C4D-40B8-BC2D-A0A03C33039F@economy-x-talk.com> <7F365666-56D0-4C11-9A83-8A13A92AD8CA@gmail.com> <1BBF6997-35E1-4969-AF43-1A7C0E46291D@economy-x-talk.com> <54A3F4BF-3BC2-4205-8EC9-89C816745C95@gmail.com> <82410BA5-56A6-4F3F-80A6-B80CEBDEE50D@economy-x-talk.com> Message-ID: Mark, the error is occurring with 4.6.1, 4.6.2 and 4.6.3. In each case I made sure that the target platform was selected in the standalone settings. Cheers Gerry On 21/07/2011, at 9:38 PM, Mark Schonewille wrote: > Usually, that error appears if you haven't selected a platform in the standalone builder. It probably means that XCode isn't even involved yet. If the error still occurs after selecting the iOS platform in the standalone builder, try an older version of LiveCode (4.6.1). From john at splash21.com Thu Jul 21 08:57:54 2011 From: john at splash21.com (John Craig) Date: Thu, 21 Jul 2011 13:57:54 +0100 Subject: [ANN] New plugin AAG|Workspaces In-Reply-To: References: Message-ID: <4E282252.7080009@splash21.com> Silly?? Sounds like a useful tool - I'm off to give it a go - ta! On 20/07/2011 22:47, Andre Garzia wrote: > Folks, > > I've just launched a little plugin. I've used it since forever, hope you > guys find it useful. > > Check out at http://andregarzia.com/page/aagworkspaces > > Basically it allows you to create "workspaces". Each workspace is a > collection of URLs and Files (that can be stacks) that are open when you are > in that workspace, so when you switch projects, instead of loosing a lot of > time reopening pages and files, you can simply select the workspace in there > and all the files and URLs will open. It sounds silly but it is handy. > > :-) > From andre at andregarzia.com Thu Jul 21 10:08:44 2011 From: andre at andregarzia.com (Andre Garzia) Date: Thu, 21 Jul 2011 11:08:44 -0300 Subject: [ANN] New plugin AAG|Workspaces In-Reply-To: <4E282252.7080009@splash21.com> References: <4E282252.7080009@splash21.com> Message-ID: Thanks folks! Please give me feedback when you run it. I haven't tried this on other machines... :-) Cheers andre On Thu, Jul 21, 2011 at 9:57 AM, John Craig wrote: > Silly?? Sounds like a useful tool - I'm off to give it a go - ta! > > > > > On 20/07/2011 22:47, Andre Garzia wrote: > >> Folks, >> >> I've just launched a little plugin. I've used it since forever, hope you >> guys find it useful. >> >> Check out at http://andregarzia.com/page/**aagworkspaces >> >> Basically it allows you to create "workspaces". Each workspace is a >> collection of URLs and Files (that can be stacks) that are open when you >> are >> in that workspace, so when you switch projects, instead of loosing a lot >> of >> time reopening pages and files, you can simply select the workspace in >> there >> and all the files and URLs will open. It sounds silly but it is handy. >> >> :-) >> >> > ______________________________**_________________ > use-livecode mailing list > use-livecode at lists.runrev.com > Please visit this url to subscribe, unsubscribe and manage your > subscription preferences: > http://lists.runrev.com/**mailman/listinfo/use-livecode > -- http://www.andregarzia.com All We Do Is Code. From mwieder at ahsoftware.net Thu Jul 21 11:36:05 2011 From: mwieder at ahsoftware.net (Mark Wieder) Date: Thu, 21 Jul 2011 08:36:05 -0700 Subject: on-Rev client software In-Reply-To: <4E280ED4.90502@crcom.net> References: <4E280ED4.90502@crcom.net> Message-ID: <11547577484.20110721083605@ahsoftware.net> Len- Thursday, July 21, 2011, 4:34:44 AM, you wrote: > I've been one of "founders" with on-rev since the offer was first made > but haven't really had the time to get into it until now. I've had web > sites there but never used Livecode to make them. Now is the time. You might want to wait a bit until they update the server to the newly released LC server instead. No sense diving into things and then having them all break shortly thereafter. > Has runRev ever released an update to the "on-rev" desktop client? No. > Also, is there ANY kind of tutorial, manual, web page, cave > painting, that I can read that will explain how to use it? No. -- -Mark Wieder mwieder at ahsoftware.net From jacque at hyperactivesw.com Thu Jul 21 11:48:17 2011 From: jacque at hyperactivesw.com (J. Landman Gay) Date: Thu, 21 Jul 2011 10:48:17 -0500 Subject: on-Rev client software In-Reply-To: <4E280ED4.90502@crcom.net> References: <4E280ED4.90502@crcom.net> Message-ID: <4E284A41.6000708@hyperactivesw.com> On 7/21/11 6:34 AM, Len Morgan wrote: > Has runRev ever released an update to the "on-rev" desktop client? If > so, where can I find it. Also, is there ANY kind of tutorial, manual, > web page, cave painting, that I can read that will explain how to use it? I'd love to see cave paintings. :) Yesterday RR released a dozen or so lessons on the new server product. If you go to the Lessons area of the web site and do a search for "server" you'll probably turn up most of them. -- Jacqueline Landman Gay | jacque at hyperactivesw.com HyperActive Software | http://www.hyperactivesw.com From kray at sonsothunder.com Thu Jul 21 12:28:34 2011 From: kray at sonsothunder.com (Ken Ray) Date: Thu, 21 Jul 2011 11:28:34 -0500 Subject: [ANN] stsProgBar DropTool 1.2 - Now with support for Lion! Message-ID: Hey all! With Apple releasing Lion yesterday, they changed the look and feel of virtually every control in the OS, including the "intermediate progress bar" (barber pole). This is why I've upgraded stsProgBar to 1.2 to support the new Lion interface; if you're interested you can get more info at this page: http://droptools.sonsothunder.com/products/sts/sts-pb.irev (NOTE: Those that have already purchased stsProgBar 1.1 have been sent free upgrades to 1.2; if you're reading this before you get the email from me, just hang tight and it will be in your inbox shortly.) Ken Ray Sons of Thunder Software, Inc. Email: kray at sonsothunder.com Web Site: http://www.sonsothunder.com/ From pete at mollysrevenge.com Wed Jul 20 18:11:20 2011 From: pete at mollysrevenge.com (Pete) Date: Wed, 20 Jul 2011 15:11:20 -0700 Subject: Mac OS X Lion In-Reply-To: References: <4E271A6B.7000509@fourthworld.com> Message-ID: I think the info I got meant they stopped selling it, not supporting it. It seemed strange to me also, that's why I was asking if anyone had run into this. I'm already on Snow Leopard so not a problem for me. Pete Molly's Revenge On Wed, Jul 20, 2011 at 2:45 PM, Andre Garzia wrote: > Apple would not discontinue snow leopard, they support OSes for a long > time... > > On Wed, Jul 20, 2011 at 6:31 PM, Pete wrote: > > > I was just told the following about Lion: > > > > 1) It will only install correctly if you are on the very latest version > of > > Snow Leopard (10.6.8) > > 2) Apple have discontinued Snow Leopard > > > > If that is true and you don't already have Snow Leopard installed, seems > > like you're screwed. > > > > Can anyone confirm this? > > > > Pete > > Molly's Revenge > > > > > > > > > > On Wed, Jul 20, 2011 at 1:50 PM, Pete wrote: > > > > > I agree and look forward seeing the results of the work that some LC > > > developers are doing on making the missing controls available. > > > Pete > > > Molly's Revenge > > > > > > > > > > > > > > > On Wed, Jul 20, 2011 at 1:19 PM, Andre Garzia > >wrote: > > > > > >> On Wed, Jul 20, 2011 at 5:13 PM, Pete wrote: > > >> > > >> > There's been recent discussion of just how many OSX controls are not > > >> > natively available in LC and Lion just seems to add to that list. > > Will > > >> LC > > >> > ever catch up? > > >> > > > >> > > >> LC is Carbon and has a lot of emulated stuff to make is > cross-platform, > > >> this > > >> is a very hard race to win... :-) > > >> > > >> > > >> > > >> > Pete > > >> > Molly's Revenge > > >> > > > >> > > > >> > > > >> > > > >> > On Wed, Jul 20, 2011 at 12:38 PM, Andre Garzia < > andre at andregarzia.com > > >> > >wrote: > > >> > > > >> > > On Wed, Jul 20, 2011 at 3:11 PM, Richard Gaskin > > >> > > wrote: > > >> > > > > >> > > > Andre wrote: > > >> > > > > > >> > > > I don't like lion or the new scrollbars... :-( > > >> > > >> > > >> > > > > > >> > > > For better or worse, it seems those scrollbars are the wave of > the > > >> > > future: > > >> > > > > >> > > http://livecodejournal.com/blog.irv?pid=1307545265.586975> > > >> > > > **> > > >> > > > > > >> > > > What concerns me more about Lion is how auto-save is supposed to > > >> work > > >> > > with > > >> > > > our apps.... > > >> > > > > > >> > > > > >> > > I have no clue either... The new APIs are so Objective-C/Cocoa > > >> specific, > > >> > > using delegates and whatnot that I think it will be a tad hard for > > our > > >> > > little carbon engine to leverage on them. > > >> > > > > >> > > > > >> > > > > >> > > > > > >> > > > -- > > >> > > > Richard Gaskin > > >> > > > Fourth World > > >> > > > LiveCode training and consulting: http://www.fourthworld.com > > >> > > > Webzine for LiveCode developers: > http://www.LiveCodeJournal.com > > >> > > > LiveCode Journal blog: http://LiveCodejournal.com/**blog.irv< > > >> > > http://LiveCodejournal.com/blog.irv> > > >> > > > > > >> > > > > > >> > > > ______________________________**_________________ > > >> > > > use-livecode mailing list > > >> > > > use-livecode at lists.runrev.com > > >> > > > Please visit this url to subscribe, unsubscribe and manage your > > >> > > > subscription preferences: > > >> > > > http://lists.runrev.com/**mailman/listinfo/use-livecode< > > >> > > http://lists.runrev.com/mailman/listinfo/use-livecode> > > >> > > > > > >> > > > > >> > > > > >> > > > > >> > > -- > > >> > > http://www.andregarzia.com All We Do Is Code. > > >> > > _______________________________________________ > > >> > > use-livecode mailing list > > >> > > use-livecode at lists.runrev.com > > >> > > Please visit this url to subscribe, unsubscribe and manage your > > >> > > subscription preferences: > > >> > > http://lists.runrev.com/mailman/listinfo/use-livecode > > >> > > > > >> > > > > >> > _______________________________________________ > > >> > use-livecode mailing list > > >> > use-livecode at lists.runrev.com > > >> > Please visit this url to subscribe, unsubscribe and manage your > > >> > subscription preferences: > > >> > http://lists.runrev.com/mailman/listinfo/use-livecode > > >> > > > >> > > >> > > >> > > >> -- > > >> http://www.andregarzia.com All We Do Is Code. > > >> _______________________________________________ > > >> use-livecode mailing list > > >> use-livecode at lists.runrev.com > > >> Please visit this url to subscribe, unsubscribe and manage your > > >> subscription preferences: > > >> http://lists.runrev.com/mailman/listinfo/use-livecode > > >> > > >> > > > > > _______________________________________________ > > use-livecode mailing list > > use-livecode at lists.runrev.com > > Please visit this url to subscribe, unsubscribe and manage your > > subscription preferences: > > http://lists.runrev.com/mailman/listinfo/use-livecode > > > > > > -- > http://www.andregarzia.com All We Do Is Code. > _______________________________________________ > use-livecode mailing list > use-livecode at lists.runrev.com > Please visit this url to subscribe, unsubscribe and manage your > subscription preferences: > http://lists.runrev.com/mailman/listinfo/use-livecode > > From pete at mollysrevenge.com Wed Jul 20 18:15:11 2011 From: pete at mollysrevenge.com (Pete) Date: Wed, 20 Jul 2011 15:15:11 -0700 Subject: [ANN] New plugin AAG|Workspaces In-Reply-To: References: Message-ID: I don't know why but this reminded me of something I would love to see in the script editor - split screen mode, kinda like you see in Word/Excel where you can slide a bar part way up the editing window and each resulting section of the window can be individually scrolled/edited. Pete Molly's Revenge On Wed, Jul 20, 2011 at 2:47 PM, Andre Garzia wrote: > Folks, > > I've just launched a little plugin. I've used it since forever, hope you > guys find it useful. > > Check out at http://andregarzia.com/page/aagworkspaces > > Basically it allows you to create "workspaces". Each workspace is a > collection of URLs and Files (that can be stacks) that are open when you > are > in that workspace, so when you switch projects, instead of loosing a lot of > time reopening pages and files, you can simply select the workspace in > there > and all the files and URLs will open. It sounds silly but it is handy. > > :-) > > -- > http://www.andregarzia.com All We Do Is Code. > _______________________________________________ > use-livecode mailing list > use-livecode at lists.runrev.com > Please visit this url to subscribe, unsubscribe and manage your > subscription preferences: > http://lists.runrev.com/mailman/listinfo/use-livecode > > From lists.pete at haworths.org Wed Jul 20 18:18:37 2011 From: lists.pete at haworths.org (Pete Haworth) Date: Wed, 20 Jul 2011 15:18:37 -0700 Subject: Mac OS X Lion In-Reply-To: References: <4E271A6B.7000509@fourthworld.com> Message-ID: I think the info I got meant they stopped selling it, not supporting it. It seemed strange to me also, that's why I was asking if anyone had run into this. I'm already on Snow Leopard so not a problem for me. Pete On Wed, Jul 20, 2011 at 2:45 PM, Andre Garzia wrote: > Apple would not discontinue snow leopard, they support OSes for a long > time... > > On Wed, Jul 20, 2011 at 6:31 PM, Pete wrote: > > > I was just told the following about Lion: > > > > 1) It will only install correctly if you are on the very latest version > of > > Snow Leopard (10.6.8) > > 2) Apple have discontinued Snow Leopard > > > > If that is true and you don't already have Snow Leopard installed, seems > > like you're screwed. > > > > Can anyone confirm this? > > > > Pete > > Molly's Revenge > > > > > > > > > > On Wed, Jul 20, 2011 at 1:50 PM, Pete wrote: > > > > > I agree and look forward seeing the results of the work that some LC > > > developers are doing on making the missing controls available. > > > Pete > > > Molly's Revenge > > > > > > > > > > > > > > > On Wed, Jul 20, 2011 at 1:19 PM, Andre Garzia > >wrote: > > > > > >> On Wed, Jul 20, 2011 at 5:13 PM, Pete wrote: > > >> > > >> > There's been recent discussion of just how many OSX controls are not > > >> > natively available in LC and Lion just seems to add to that list. > > Will > > >> LC > > >> > ever catch up? > > >> > > > >> > > >> LC is Carbon and has a lot of emulated stuff to make is > cross-platform, > > >> this > > >> is a very hard race to win... :-) > > >> > > >> > > >> > > >> > Pete > > >> > Molly's Revenge > > >> > > > >> > > > >> > > > >> > > > >> > On Wed, Jul 20, 2011 at 12:38 PM, Andre Garzia < > andre at andregarzia.com > > >> > >wrote: > > >> > > > >> > > On Wed, Jul 20, 2011 at 3:11 PM, Richard Gaskin > > >> > > wrote: > > >> > > > > >> > > > Andre wrote: > > >> > > > > > >> > > > I don't like lion or the new scrollbars... :-( > > >> > > >> > > >> > > > > > >> > > > For better or worse, it seems those scrollbars are the wave of > the > > >> > > future: > > >> > > > > >> > > http://livecodejournal.com/blog.irv?pid=1307545265.586975> > > >> > > > **> > > >> > > > > > >> > > > What concerns me more about Lion is how auto-save is supposed to > > >> work > > >> > > with > > >> > > > our apps.... > > >> > > > > > >> > > > > >> > > I have no clue either... The new APIs are so Objective-C/Cocoa > > >> specific, > > >> > > using delegates and whatnot that I think it will be a tad hard for > > our > > >> > > little carbon engine to leverage on them. > > >> > > > > >> > > > > >> > > > > >> > > > > > >> > > > -- > > >> > > > Richard Gaskin > > >> > > > Fourth World > > >> > > > LiveCode training and consulting: http://www.fourthworld.com > > >> > > > Webzine for LiveCode developers: > http://www.LiveCodeJournal.com > > >> > > > LiveCode Journal blog: http://LiveCodejournal.com/**blog.irv< > > >> > > http://LiveCodejournal.com/blog.irv> > > >> > > > > > >> > > > > > >> > > > ______________________________**_________________ > > >> > > > use-livecode mailing list > > >> > > > use-livecode at lists.runrev.com > > >> > > > Please visit this url to subscribe, unsubscribe and manage your > > >> > > > subscription preferences: > > >> > > > http://lists.runrev.com/**mailman/listinfo/use-livecode< > > >> > > http://lists.runrev.com/mailman/listinfo/use-livecode> > > >> > > > > > >> > > > > >> > > > > >> > > > > >> > > -- > > >> > > http://www.andregarzia.com All We Do Is Code. > > >> > > _______________________________________________ > > >> > > use-livecode mailing list > > >> > > use-livecode at lists.runrev.com > > >> > > Please visit this url to subscribe, unsubscribe and manage your > > >> > > subscription preferences: > > >> > > http://lists.runrev.com/mailman/listinfo/use-livecode > > >> > > > > >> > > > > >> > _______________________________________________ > > >> > use-livecode mailing list > > >> > use-livecode at lists.runrev.com > > >> > Please visit this url to subscribe, unsubscribe and manage your > > >> > subscription preferences: > > >> > http://lists.runrev.com/mailman/listinfo/use-livecode > > >> > > > >> > > >> > > >> > > >> -- > > >> http://www.andregarzia.com All We Do Is Code. > > >> _______________________________________________ > > >> use-livecode mailing list > > >> use-livecode at lists.runrev.com > > >> Please visit this url to subscribe, unsubscribe and manage your > > >> subscription preferences: > > >> http://lists.runrev.com/mailman/listinfo/use-livecode > > >> > > >> > > > > > _______________________________________________ > > use-livecode mailing list > > use-livecode at lists.runrev.com > > Please visit this url to subscribe, unsubscribe and manage your > > subscription preferences: > > http://lists.runrev.com/mailman/listinfo/use-livecode > > > > > > -- > http://www.andregarzia.com All We Do Is Code. > _______________________________________________ > use-livecode mailing list > use-livecode at lists.runrev.com > Please visit this url to subscribe, unsubscribe and manage your > subscription preferences: > http://lists.runrev.com/mailman/listinfo/use-livecode > > From jperryl at ecs.fullerton.edu Thu Jul 21 12:52:17 2011 From: jperryl at ecs.fullerton.edu (Judy Perry) Date: Thu, 21 Jul 2011 09:52:17 -0700 (PDT) Subject: Mac OS X Lion In-Reply-To: References: <4E271A6B.7000509@fourthworld.com> Message-ID: System 9 isn't easy to find, either... Judy On Thu, 21 Jul 2011, Fran?ois Chaplais wrote: > when I learned that Lion requires snow leopard (which I do not have on all of my machines) I cautiously ordered a copy of snow. Hardest to find is System 7.x for my powermac 8500 ;) From bobs at twft.com Thu Jul 21 12:59:05 2011 From: bobs at twft.com (Bob Sneidar) Date: Thu, 21 Jul 2011 09:59:05 -0700 Subject: Mac OS X Lion In-Reply-To: References: <4E271A6B.7000509@fourthworld.com> Message-ID: <26629F3A-B9AB-4C69-95B2-53AB8782E68C@twft.com> I have a copy if you need it. Actually, I still have 8.6. ;-) Bob On Jul 21, 2011, at 9:52 AM, Judy Perry wrote: > System 9 isn't easy to find, either... > > Judy > > On Thu, 21 Jul 2011, Fran?ois Chaplais wrote: > >> when I learned that Lion requires snow leopard (which I do not have on all of my machines) I cautiously ordered a copy of snow. Hardest to find is System 7.x for my powermac 8500 ;) > _______________________________________________ > use-livecode mailing list > use-livecode at lists.runrev.com > Please visit this url to subscribe, unsubscribe and manage your subscription preferences: > http://lists.runrev.com/mailman/listinfo/use-livecode From jperryl at ecs.fullerton.edu Thu Jul 21 13:07:10 2011 From: jperryl at ecs.fullerton.edu (Judy Perry) Date: Thu, 21 Jul 2011 10:07:10 -0700 (PDT) Subject: Mac OS X Lion In-Reply-To: <26629F3A-B9AB-4C69-95B2-53AB8782E68C@twft.com> References: <4E271A6B.7000509@fourthworld.com> <26629F3A-B9AB-4C69-95B2-53AB8782E68C@twft.com> Message-ID: Actually, I do. I have a couple of original iMacs I need to revive... My office machine (a G3 blue & white tower) might actually be running 8.x; can't tell as they got rid of the monitor :-( Judy On Thu, 21 Jul 2011, Bob Sneidar wrote: > I have a copy if you need it. Actually, I still have 8.6. ;-) From andre at andregarzia.com Thu Jul 21 13:19:11 2011 From: andre at andregarzia.com (Andre Garzia) Date: Thu, 21 Jul 2011 14:19:11 -0300 Subject: [ANN] New plugin AAG|Workspaces In-Reply-To: References: Message-ID: Someone can correct me but I think GLX2 can do that with even more than two panes... (or it was constellation?) :-) On Wed, Jul 20, 2011 at 7:15 PM, Pete wrote: > I don't know why but this reminded me of something I would love to see in > the script editor - split screen mode, kinda like you see in Word/Excel > where you can slide a bar part way up the editing window and each resulting > section of the window can be individually scrolled/edited. > Pete > Molly's Revenge > > > > > On Wed, Jul 20, 2011 at 2:47 PM, Andre Garzia > wrote: > > > Folks, > > > > I've just launched a little plugin. I've used it since forever, hope you > > guys find it useful. > > > > Check out at http://andregarzia.com/page/aagworkspaces > > > > Basically it allows you to create "workspaces". Each workspace is a > > collection of URLs and Files (that can be stacks) that are open when you > > are > > in that workspace, so when you switch projects, instead of loosing a lot > of > > time reopening pages and files, you can simply select the workspace in > > there > > and all the files and URLs will open. It sounds silly but it is handy. > > > > :-) > > > > -- > > http://www.andregarzia.com All We Do Is Code. > > _______________________________________________ > > use-livecode mailing list > > use-livecode at lists.runrev.com > > Please visit this url to subscribe, unsubscribe and manage your > > subscription preferences: > > http://lists.runrev.com/mailman/listinfo/use-livecode > > > > > _______________________________________________ > use-livecode mailing list > use-livecode at lists.runrev.com > Please visit this url to subscribe, unsubscribe and manage your > subscription preferences: > http://lists.runrev.com/mailman/listinfo/use-livecode > -- http://www.andregarzia.com All We Do Is Code. From bobs at twft.com Thu Jul 21 13:39:49 2011 From: bobs at twft.com (Bob Sneidar) Date: Thu, 21 Jul 2011 10:39:49 -0700 Subject: Mac OS X Lion In-Reply-To: References: <4E271A6B.7000509@fourthworld.com> <26629F3A-B9AB-4C69-95B2-53AB8782E68C@twft.com> Message-ID: <0C9AA6AE-8F7B-4707-8367-261B415BC4F2@twft.com> Let be burn you copies of what I have. Will .img files work for you? Bob On Jul 21, 2011, at 10:07 AM, Judy Perry wrote: > Actually, I do. I have a couple of original iMacs I need to revive... My office machine (a G3 blue & white tower) might actually be running 8.x; can't tell as they got rid of the monitor :-( > > Judy > > On Thu, 21 Jul 2011, Bob Sneidar wrote: > >> I have a copy if you need it. Actually, I still have 8.6. ;-) > > _______________________________________________ > use-livecode mailing list > use-livecode at lists.runrev.com > Please visit this url to subscribe, unsubscribe and manage your subscription preferences: > http://lists.runrev.com/mailman/listinfo/use-livecode From lists.pete at haworths.org Thu Jul 21 13:53:00 2011 From: lists.pete at haworths.org (Pete Haworth) Date: Thu, 21 Jul 2011 10:53:00 -0700 Subject: [ANN] New plugin AAG|Workspaces In-Reply-To: References: Message-ID: Is GLX2 what became tRev... and then Remo? I have tRev but have to admit I haven't used it a lot but don;t remember seeing a split screen editing mode, but then there's not much in the way of documentation for it. Pete On Thu, Jul 21, 2011 at 10:19 AM, Andre Garzia wrote: > Someone can correct me but I think GLX2 can do that with even more than two > panes... (or it was constellation?) > > :-) > > On Wed, Jul 20, 2011 at 7:15 PM, Pete wrote: > > > I don't know why but this reminded me of something I would love to see in > > the script editor - split screen mode, kinda like you see in Word/Excel > > where you can slide a bar part way up the editing window and each > resulting > > section of the window can be individually scrolled/edited. > > Pete > > Molly's Revenge > > > > > > > > > > On Wed, Jul 20, 2011 at 2:47 PM, Andre Garzia > > wrote: > > > > > Folks, > > > > > > I've just launched a little plugin. I've used it since forever, hope > you > > > guys find it useful. > > > > > > Check out at http://andregarzia.com/page/aagworkspaces > > > > > > Basically it allows you to create "workspaces". Each workspace is a > > > collection of URLs and Files (that can be stacks) that are open when > you > > > are > > > in that workspace, so when you switch projects, instead of loosing a > lot > > of > > > time reopening pages and files, you can simply select the workspace in > > > there > > > and all the files and URLs will open. It sounds silly but it is handy. > > > > > > :-) > > > > > > -- > > > http://www.andregarzia.com All We Do Is Code. > > > _______________________________________________ > > > use-livecode mailing list > > > use-livecode at lists.runrev.com > > > Please visit this url to subscribe, unsubscribe and manage your > > > subscription preferences: > > > http://lists.runrev.com/mailman/listinfo/use-livecode > > > > > > > > _______________________________________________ > > use-livecode mailing list > > use-livecode at lists.runrev.com > > Please visit this url to subscribe, unsubscribe and manage your > > subscription preferences: > > http://lists.runrev.com/mailman/listinfo/use-livecode > > > > > > -- > http://www.andregarzia.com All We Do Is Code. > _______________________________________________ > use-livecode mailing list > use-livecode at lists.runrev.com > Please visit this url to subscribe, unsubscribe and manage your > subscription preferences: > http://lists.runrev.com/mailman/listinfo/use-livecode > From stephenREVOLUTION2 at barncard.com Thu Jul 21 14:07:01 2011 From: stephenREVOLUTION2 at barncard.com (stephen barncard) Date: Thu, 21 Jul 2011 11:07:01 -0700 Subject: [ANN] New plugin AAG|Workspaces In-Reply-To: References: Message-ID: tRev (now Remo) doesn't split. GLX2 did, and I think is in the open source version. sqb On 21 July 2011 10:53, Pete Haworth wrote: > Is GLX2 what became tRev... and then Remo? I have tRev but have to admit I > haven't used it a lot but don;t remember seeing a split screen editing > mode, > but then there's not much in the way of documentation for it. > Pete > > On Thu, Jul 21, 2011 at 10:19 AM, Andre Garzia >wrote: > > > Someone can correct me but I think GLX2 can do that with even more than > two > > panes... (or it was constellation?) > > > > :-) > > > > On Wed, Jul 20, 2011 at 7:15 PM, Pete wrote: > > > > > I don't know why but this reminded me of something I would love to see > in > > > the script editor - split screen mode, kinda like you see in Word/Excel > > > where you can slide a bar part way up the editing window and each > > resulting > > > section of the window can be individually scrolled/edited. > > > Pete > > > Molly's Revenge > > > > > > > > > > > > > > > On Wed, Jul 20, 2011 at 2:47 PM, Andre Garzia > > > wrote: > > > > > > > Folks, > > > > > > > > I've just launched a little plugin. I've used it since forever, hope > > you > > > > guys find it useful. > > > > > > > > Check out at http://andregarzia.com/page/aagworkspaces > > > > > > > > Basically it allows you to create "workspaces". Each workspace is a > > > > collection of URLs and Files (that can be stacks) that are open when > > you > > > > are > > > > in that workspace, so when you switch projects, instead of loosing a > > lot > > > of > > > > time reopening pages and files, you can simply select the workspace > in > > > > there > > > > and all the files and URLs will open. It sounds silly but it is > handy. > > > > > > > > :-) > > > > > > > > -- > > > > http://www.andregarzia.com All We Do Is Code. > > > > _______________________________________________ > > > > use-livecode mailing list > > > > use-livecode at lists.runrev.com > > > > Please visit this url to subscribe, unsubscribe and manage your > > > > subscription preferences: > > > > http://lists.runrev.com/mailman/listinfo/use-livecode > > > > > > > > > > > _______________________________________________ > > > use-livecode mailing list > > > use-livecode at lists.runrev.com > > > Please visit this url to subscribe, unsubscribe and manage your > > > subscription preferences: > > > http://lists.runrev.com/mailman/listinfo/use-livecode > > > > > > > > > > > -- > > http://www.andregarzia.com All We Do Is Code. > > _______________________________________________ > > use-livecode mailing list > > use-livecode at lists.runrev.com > > Please visit this url to subscribe, unsubscribe and manage your > > subscription preferences: > > http://lists.runrev.com/mailman/listinfo/use-livecode > > > _______________________________________________ > use-livecode mailing list > use-livecode at lists.runrev.com > Please visit this url to subscribe, unsubscribe and manage your > subscription preferences: > http://lists.runrev.com/mailman/listinfo/use-livecode > -- Stephen Barncard San Francisco Ca. USA more about sqb From andre at andregarzia.com Thu Jul 21 14:09:36 2011 From: andre at andregarzia.com (Andre Garzia) Date: Thu, 21 Jul 2011 15:09:36 -0300 Subject: [ANN] New plugin AAG|Workspaces In-Reply-To: References: Message-ID: Those were all done by the same person, Jerry Daniels (one of the best coders I've seen) but they are not the same. GLX2 is one thing and tRev/Remo is another. GLX2 is now an open source project and Mark Wieder is at the helm. You can get it from https://bitbucket.org/mwieder/glx2 GLX2 is a great editor, it is a different approach than tRev which is an out-of-engine editor. There are different script editors flying around but they don't get much attention. I've built a silly one called ASIE (Andres Simple iRev Editor) that could edit stack scripts and RevServer files but it proved to hard to maintain and ended up not sharing or using it. The cool thing was that it was inspired by EMACS (ASIE also means ASIE Simply Isn't Emacs) with different buffers, minibuffers and those nice Emacs keychords (C-X C-S for saving...) In the end, that means that there is room for improvement in the script editor space... :-) On Thu, Jul 21, 2011 at 2:53 PM, Pete Haworth wrote: > Is GLX2 what became tRev... and then Remo? I have tRev but have to admit I > haven't used it a lot but don;t remember seeing a split screen editing > mode, > but then there's not much in the way of documentation for it. > Pete > > On Thu, Jul 21, 2011 at 10:19 AM, Andre Garzia >wrote: > > > Someone can correct me but I think GLX2 can do that with even more than > two > > panes... (or it was constellation?) > > > > :-) > > > > On Wed, Jul 20, 2011 at 7:15 PM, Pete wrote: > > > > > I don't know why but this reminded me of something I would love to see > in > > > the script editor - split screen mode, kinda like you see in Word/Excel > > > where you can slide a bar part way up the editing window and each > > resulting > > > section of the window can be individually scrolled/edited. > > > Pete > > > Molly's Revenge > > > > > > > > > > > > > > > On Wed, Jul 20, 2011 at 2:47 PM, Andre Garzia > > > wrote: > > > > > > > Folks, > > > > > > > > I've just launched a little plugin. I've used it since forever, hope > > you > > > > guys find it useful. > > > > > > > > Check out at http://andregarzia.com/page/aagworkspaces > > > > > > > > Basically it allows you to create "workspaces". Each workspace is a > > > > collection of URLs and Files (that can be stacks) that are open when > > you > > > > are > > > > in that workspace, so when you switch projects, instead of loosing a > > lot > > > of > > > > time reopening pages and files, you can simply select the workspace > in > > > > there > > > > and all the files and URLs will open. It sounds silly but it is > handy. > > > > > > > > :-) > > > > > > > > -- > > > > http://www.andregarzia.com All We Do Is Code. > > > > _______________________________________________ > > > > use-livecode mailing list > > > > use-livecode at lists.runrev.com > > > > Please visit this url to subscribe, unsubscribe and manage your > > > > subscription preferences: > > > > http://lists.runrev.com/mailman/listinfo/use-livecode > > > > > > > > > > > _______________________________________________ > > > use-livecode mailing list > > > use-livecode at lists.runrev.com > > > Please visit this url to subscribe, unsubscribe and manage your > > > subscription preferences: > > > http://lists.runrev.com/mailman/listinfo/use-livecode > > > > > > > > > > > -- > > http://www.andregarzia.com All We Do Is Code. > > _______________________________________________ > > use-livecode mailing list > > use-livecode at lists.runrev.com > > Please visit this url to subscribe, unsubscribe and manage your > > subscription preferences: > > http://lists.runrev.com/mailman/listinfo/use-livecode > > > _______________________________________________ > use-livecode mailing list > use-livecode at lists.runrev.com > Please visit this url to subscribe, unsubscribe and manage your > subscription preferences: > http://lists.runrev.com/mailman/listinfo/use-livecode > -- http://www.andregarzia.com All We Do Is Code. From andre at andregarzia.com Thu Jul 21 14:11:17 2011 From: andre at andregarzia.com (Andre Garzia) Date: Thu, 21 Jul 2011 15:11:17 -0300 Subject: Mac OS X Lion In-Reply-To: <0C9AA6AE-8F7B-4707-8367-261B415BC4F2@twft.com> References: <4E271A6B.7000509@fourthworld.com> <26629F3A-B9AB-4C69-95B2-53AB8782E68C@twft.com> <0C9AA6AE-8F7B-4707-8367-261B415BC4F2@twft.com> Message-ID: Apple still selling Snow Leopard on their web store. It was EOL only on physical stores. Current macbook pros still come with Snow Leopard. Now that they shipped lion, which cat will be the next OS? I vote for Mac OS X 10.8 Megalomaniac Garfield On Thu, Jul 21, 2011 at 2:39 PM, Bob Sneidar wrote: > Let be burn you copies of what I have. Will .img files work for you? > > Bob > > > On Jul 21, 2011, at 10:07 AM, Judy Perry wrote: > > > Actually, I do. I have a couple of original iMacs I need to revive... > My office machine (a G3 blue & white tower) might actually be running 8.x; > can't tell as they got rid of the monitor :-( > > > > Judy > > > > On Thu, 21 Jul 2011, Bob Sneidar wrote: > > > >> I have a copy if you need it. Actually, I still have 8.6. ;-) > > > > _______________________________________________ > > use-livecode mailing list > > use-livecode at lists.runrev.com > > Please visit this url to subscribe, unsubscribe and manage your > subscription preferences: > > http://lists.runrev.com/mailman/listinfo/use-livecode > > > _______________________________________________ > use-livecode mailing list > use-livecode at lists.runrev.com > Please visit this url to subscribe, unsubscribe and manage your > subscription preferences: > http://lists.runrev.com/mailman/listinfo/use-livecode > -- http://www.andregarzia.com All We Do Is Code. From bobs at twft.com Thu Jul 21 14:28:54 2011 From: bobs at twft.com (Bob Sneidar) Date: Thu, 21 Jul 2011 11:28:54 -0700 Subject: Mac OS X Lion In-Reply-To: References: <4E271A6B.7000509@fourthworld.com> <26629F3A-B9AB-4C69-95B2-53AB8782E68C@twft.com> <0C9AA6AE-8F7B-4707-8367-261B415BC4F2@twft.com> Message-ID: <84C2225B-2FA0-4166-ACB7-24857E7CDE6E@twft.com> How about Tabby, code named Felix? Bob On Jul 21, 2011, at 11:11 AM, Andre Garzia wrote: > Apple still selling Snow Leopard on their web store. > > It was EOL only on physical stores. Current macbook pros still come with > Snow Leopard. > > Now that they shipped lion, which cat will be the next OS? I vote for Mac OS > X 10.8 Megalomaniac Garfield > > On Thu, Jul 21, 2011 at 2:39 PM, Bob Sneidar wrote: > >> Let be burn you copies of what I have. Will .img files work for you? >> >> Bob >> >> >> On Jul 21, 2011, at 10:07 AM, Judy Perry wrote: >> >>> Actually, I do. I have a couple of original iMacs I need to revive... >> My office machine (a G3 blue & white tower) might actually be running 8.x; >> can't tell as they got rid of the monitor :-( >>> >>> Judy >>> >>> On Thu, 21 Jul 2011, Bob Sneidar wrote: >>> >>>> I have a copy if you need it. Actually, I still have 8.6. ;-) >>> >>> _______________________________________________ >>> use-livecode mailing list >>> use-livecode at lists.runrev.com >>> Please visit this url to subscribe, unsubscribe and manage your >> subscription preferences: >>> http://lists.runrev.com/mailman/listinfo/use-livecode >> >> >> _______________________________________________ >> use-livecode mailing list >> use-livecode at lists.runrev.com >> Please visit this url to subscribe, unsubscribe and manage your >> subscription preferences: >> http://lists.runrev.com/mailman/listinfo/use-livecode >> > > > > -- > http://www.andregarzia.com All We Do Is Code. > _______________________________________________ > use-livecode mailing list > use-livecode at lists.runrev.com > Please visit this url to subscribe, unsubscribe and manage your subscription preferences: > http://lists.runrev.com/mailman/listinfo/use-livecode From bobs at twft.com Thu Jul 21 14:30:37 2011 From: bobs at twft.com (Bob Sneidar) Date: Thu, 21 Jul 2011 11:30:37 -0700 Subject: [ANN] New plugin AAG|Workspaces In-Reply-To: References: Message-ID: <182B976C-6FAA-477F-8559-8F74D136AE9E@twft.com> Dya think?? Bob On Jul 21, 2011, at 11:09 AM, Andre Garzia wrote: > > In the end, that means that there is room for improvement in the script > editor space... :-) From lists.pete at haworths.org Thu Jul 21 14:32:49 2011 From: lists.pete at haworths.org (Pete Haworth) Date: Thu, 21 Jul 2011 11:32:49 -0700 Subject: Mac OS X Lion In-Reply-To: References: <4E271A6B.7000509@fourthworld.com> <26629F3A-B9AB-4C69-95B2-53AB8782E68C@twft.com> <0C9AA6AE-8F7B-4707-8367-261B415BC4F2@twft.com> Message-ID: That explains it since the info I got about it being EOL was an email from a vendor. Pete On Thu, Jul 21, 2011 at 11:11 AM, Andre Garzia wrote: > Apple still selling Snow Leopard on their web store. > > It was EOL only on physical stores. Current macbook pros still come with > Snow Leopard. > > Now that they shipped lion, which cat will be the next OS? I vote for Mac > OS > X 10.8 Megalomaniac Garfield > > From richmondmathewson at gmail.com Thu Jul 21 14:34:05 2011 From: richmondmathewson at gmail.com (Richmond Mathewson) Date: Thu, 21 Jul 2011 21:34:05 +0300 Subject: Mac OS X Lion In-Reply-To: <84C2225B-2FA0-4166-ACB7-24857E7CDE6E@twft.com> References: <4E271A6B.7000509@fourthworld.com> <26629F3A-B9AB-4C69-95B2-53AB8782E68C@twft.com> <0C9AA6AE-8F7B-4707-8367-261B415BC4F2@twft.com> <84C2225B-2FA0-4166-ACB7-24857E7CDE6E@twft.com> Message-ID: <4E28711D.6030102@gmail.com> Marsupial sabre-tooth? > How about Tabby, code named Felix? > > Bob > > > On Jul 21, 2011, at 11:11 AM, Andre Garzia wrote: > >> Apple still selling Snow Leopard on their web store. >> >> It was EOL only on physical stores. Current macbook pros still come with >> Snow Leopard. >> >> Now that they shipped lion, which cat will be the next OS? I vote for Mac OS >> X 10.8 Megalomaniac Garfield >> >> From lists.pete at haworths.org Thu Jul 21 14:35:10 2011 From: lists.pete at haworths.org (Pete Haworth) Date: Thu, 21 Jul 2011 11:35:10 -0700 Subject: [ANN] New plugin AAG|Workspaces In-Reply-To: References: Message-ID: Thanks, I'll take a look at GLX2. I think I gave it a whirl a few months back but there were some issues with it at that time. Pete On Thu, Jul 21, 2011 at 11:09 AM, Andre Garzia wrote: > Those were all done by the same person, Jerry Daniels (one of the best > coders I've seen) but they are not the same. GLX2 is one thing and > tRev/Remo > is another. GLX2 is now an open source project and Mark Wieder is at the > helm. You can get it from https://bitbucket.org/mwieder/glx2 > > GLX2 is a great editor, it is a different approach than tRev which is an > out-of-engine editor. > > There are different script editors flying around but they don't get much > attention. > > I've built a silly one called ASIE (Andres Simple iRev Editor) that could > edit stack scripts and RevServer files but it proved to hard to maintain > and > ended up not sharing or using it. The cool thing was that it was inspired > by > EMACS (ASIE also means ASIE Simply Isn't Emacs) with different buffers, > minibuffers and those nice Emacs keychords (C-X C-S for saving...) > > In the end, that means that there is room for improvement in the script > editor space... :-) > > On Thu, Jul 21, 2011 at 2:53 PM, Pete Haworth >wrote: > > > Is GLX2 what became tRev... and then Remo? I have tRev but have to admit > I > > haven't used it a lot but don;t remember seeing a split screen editing > > mode, > > but then there's not much in the way of documentation for it. > > Pete > > > > On Thu, Jul 21, 2011 at 10:19 AM, Andre Garzia > >wrote: > > > > > Someone can correct me but I think GLX2 can do that with even more than > > two > > > panes... (or it was constellation?) > > > > > > :-) > > > > > > On Wed, Jul 20, 2011 at 7:15 PM, Pete wrote: > > > > > > > I don't know why but this reminded me of something I would love to > see > > in > > > > the script editor - split screen mode, kinda like you see in > Word/Excel > > > > where you can slide a bar part way up the editing window and each > > > resulting > > > > section of the window can be individually scrolled/edited. > > > > Pete > > > > Molly's Revenge > > > > > > > > > > > > > > > > > > > > On Wed, Jul 20, 2011 at 2:47 PM, Andre Garzia > > > > > wrote: > > > > > > > > > Folks, > > > > > > > > > > I've just launched a little plugin. I've used it since forever, > hope > > > you > > > > > guys find it useful. > > > > > > > > > > Check out at http://andregarzia.com/page/aagworkspaces > > > > > > > > > > Basically it allows you to create "workspaces". Each workspace is a > > > > > collection of URLs and Files (that can be stacks) that are open > when > > > you > > > > > are > > > > > in that workspace, so when you switch projects, instead of loosing > a > > > lot > > > > of > > > > > time reopening pages and files, you can simply select the workspace > > in > > > > > there > > > > > and all the files and URLs will open. It sounds silly but it is > > handy. > > > > > > > > > > :-) > > > > > > > > > > -- > > > > > http://www.andregarzia.com All We Do Is Code. > > > > > _______________________________________________ > > > > > use-livecode mailing list > > > > > use-livecode at lists.runrev.com > > > > > Please visit this url to subscribe, unsubscribe and manage your > > > > > subscription preferences: > > > > > http://lists.runrev.com/mailman/listinfo/use-livecode > > > > > > > > > > > > > > _______________________________________________ > > > > use-livecode mailing list > > > > use-livecode at lists.runrev.com > > > > Please visit this url to subscribe, unsubscribe and manage your > > > > subscription preferences: > > > > http://lists.runrev.com/mailman/listinfo/use-livecode > > > > > > > > > > > > > > > > -- > > > http://www.andregarzia.com All We Do Is Code. > > > _______________________________________________ > > > use-livecode mailing list > > > use-livecode at lists.runrev.com > > > Please visit this url to subscribe, unsubscribe and manage your > > > subscription preferences: > > > http://lists.runrev.com/mailman/listinfo/use-livecode > > > > > _______________________________________________ > > use-livecode mailing list > > use-livecode at lists.runrev.com > > Please visit this url to subscribe, unsubscribe and manage your > > subscription preferences: > > http://lists.runrev.com/mailman/listinfo/use-livecode > > > > > > -- > http://www.andregarzia.com All We Do Is Code. > _______________________________________________ > use-livecode mailing list > use-livecode at lists.runrev.com > Please visit this url to subscribe, unsubscribe and manage your > subscription preferences: > http://lists.runrev.com/mailman/listinfo/use-livecode > From warren at warrensweb.us Thu Jul 21 14:37:38 2011 From: warren at warrensweb.us (Warren Samples) Date: Thu, 21 Jul 2011 13:37:38 -0500 Subject: Mac OS X Lion In-Reply-To: References: <0C9AA6AE-8F7B-4707-8367-261B415BC4F2@twft.com> Message-ID: <201107211337.38935.warren@warrensweb.us> On Thursday, July 21, 2011 01:11:17 PM Andre Garzia wrote: > I vote for Mac OS > X 10.8 Megalomaniac Garfield Werecat? From bvg at mac.com Thu Jul 21 14:38:45 2011 From: bvg at mac.com (=?iso-8859-1?Q?Bj=F6rnke_von_Gierke?=) Date: Thu, 21 Jul 2011 20:38:45 +0200 Subject: Mac OS X Lion In-Reply-To: References: Message-ID: <4808F5F3-23E8-4520-B343-78777C7F183B@mac.com> I guess it's a question of ROI much more then off a decision on a concise ruleset about what and what not to support. So the basic native controls on the other hand have always been supported, even though sometimes they where broken for short times when a new OS was freshly released. More.. arcane controls like the small edit dot or drawers on the other hand have always been second hand citizens. and therefore have been either partly or not at all or very late after their appearance become implemented. For example, As far as I know RunRev was committed to allow us the creation of mac apps that are apple store compatible. However, with the new signing requirement, this might have changed a bit. All we can do is to wait for a decision from them. On 21 Jul 2011, at 02:49, Shao Sean wrote: >> There's been recent discussion of just how many OSX controls are not natively available in LC and Lion just seems to add to that list. Will LC ever catch up? > > Probably not.. Seems they will not touch anything that is not cross platform.. From cszasz at mac.com Thu Jul 21 14:45:59 2011 From: cszasz at mac.com (Charles Szasz) Date: Thu, 21 Jul 2011 14:45:59 -0400 Subject: Setting a dirty flag for a file Message-ID: I am using Rev. 4.0. How do you set a "dirty flag" for a data file that that has been saved but still open in your app stack? My app creates a data file that saves the contents of 16 fields. I have code to show an initial save dialog when the user makes changes. I want to use a dirty flag to show another save box only when closing the file if the user makes subsequent changes in the data file. Otherwise when the user closes the data file without additional change no save dialog will be presented. Thanks for your time! Charles Szasz cszasz at mac.com From lists.pete at haworths.org Thu Jul 21 14:48:27 2011 From: lists.pete at haworths.org (Pete Haworth) Date: Thu, 21 Jul 2011 11:48:27 -0700 Subject: [ANN] New plugin AAG|Workspaces In-Reply-To: References: Message-ID: Just downloaded GLX2 from bitbucket and I don't see a split screen capability, at least no obvious way of doing it. Pete On Thu, Jul 21, 2011 at 11:07 AM, stephen barncard < stephenREVOLUTION2 at barncard.com> wrote: > tRev (now Remo) doesn't split. GLX2 did, and I think is in the open source > version. > > sqb > > On 21 July 2011 10:53, Pete Haworth wrote: > > > Is GLX2 what became tRev... and then Remo? I have tRev but have to admit > I > > haven't used it a lot but don;t remember seeing a split screen editing > > mode, > > but then there's not much in the way of documentation for it. > > Pete > > > > On Thu, Jul 21, 2011 at 10:19 AM, Andre Garzia > >wrote: > > > > > Someone can correct me but I think GLX2 can do that with even more than > > two > > > panes... (or it was constellation?) > > > > > > :-) > > > > > > On Wed, Jul 20, 2011 at 7:15 PM, Pete wrote: > > > > > > > I don't know why but this reminded me of something I would love to > see > > in > > > > the script editor - split screen mode, kinda like you see in > Word/Excel > > > > where you can slide a bar part way up the editing window and each > > > resulting > > > > section of the window can be individually scrolled/edited. > > > > Pete > > > > Molly's Revenge > > > > > > > > > > > > > > > > > > > > On Wed, Jul 20, 2011 at 2:47 PM, Andre Garzia > > > > > wrote: > > > > > > > > > Folks, > > > > > > > > > > I've just launched a little plugin. I've used it since forever, > hope > > > you > > > > > guys find it useful. > > > > > > > > > > Check out at http://andregarzia.com/page/aagworkspaces > > > > > > > > > > Basically it allows you to create "workspaces". Each workspace is a > > > > > collection of URLs and Files (that can be stacks) that are open > when > > > you > > > > > are > > > > > in that workspace, so when you switch projects, instead of loosing > a > > > lot > > > > of > > > > > time reopening pages and files, you can simply select the workspace > > in > > > > > there > > > > > and all the files and URLs will open. It sounds silly but it is > > handy. > > > > > > > > > > :-) > > > > > > > > > > -- > > > > > http://www.andregarzia.com All We Do Is Code. > > > > > _______________________________________________ > > > > > use-livecode mailing list > > > > > use-livecode at lists.runrev.com > > > > > Please visit this url to subscribe, unsubscribe and manage your > > > > > subscription preferences: > > > > > http://lists.runrev.com/mailman/listinfo/use-livecode > > > > > > > > > > > > > > _______________________________________________ > > > > use-livecode mailing list > > > > use-livecode at lists.runrev.com > > > > Please visit this url to subscribe, unsubscribe and manage your > > > > subscription preferences: > > > > http://lists.runrev.com/mailman/listinfo/use-livecode > > > > > > > > > > > > > > > > -- > > > http://www.andregarzia.com All We Do Is Code. > > > _______________________________________________ > > > use-livecode mailing list > > > use-livecode at lists.runrev.com > > > Please visit this url to subscribe, unsubscribe and manage your > > > subscription preferences: > > > http://lists.runrev.com/mailman/listinfo/use-livecode > > > > > _______________________________________________ > > use-livecode mailing list > > use-livecode at lists.runrev.com > > Please visit this url to subscribe, unsubscribe and manage your > > subscription preferences: > > http://lists.runrev.com/mailman/listinfo/use-livecode > > > > > > -- > > > > Stephen Barncard > San Francisco Ca. USA > > more about sqb > _______________________________________________ > use-livecode mailing list > use-livecode at lists.runrev.com > Please visit this url to subscribe, unsubscribe and manage your > subscription preferences: > http://lists.runrev.com/mailman/listinfo/use-livecode > From m.schonewille at economy-x-talk.com Thu Jul 21 15:13:14 2011 From: m.schonewille at economy-x-talk.com (Mark Schonewille) Date: Thu, 21 Jul 2011 21:13:14 +0200 Subject: Setting a dirty flag for a file In-Reply-To: References: Message-ID: <90FBB019-82D2-4B13-9DF6-2A8E15277AF3@economy-x-talk.com> Hi Charles, Isn't it sufficient to simply no close the file until you're done? You might need to keep records of open files, but if you do this properly, you could open them once and close them all when your app closes or if the user closes some window. on openSomeFIle open file gFile // do stuff end openSomeFile on closeStack close file gFile end coseStack -- Best regards, Mark Schonewille Economy-x-Talk Consulting and Software Engineering Homepage: http://economy-x-talk.com Twitter: http://twitter.com/xtalkprogrammer KvK: 50277553 New: Download the Installer Maker Plugin 1.6 for LiveCode here http://qery.us/ce On 21 jul 2011, at 20:45, Charles Szasz wrote: > I am using Rev. 4.0. How do you set a "dirty flag" for a data file that that has been saved but still open in your app stack? My app creates a data file that saves the contents of 16 fields. I have code to show an initial save dialog when the user makes changes. I want to use a dirty flag to show another save box only when closing the file if the user makes subsequent changes in the data file. Otherwise when the user closes the data file without additional change no save dialog will be presented. > > Thanks for your time! > > Charles Szasz > cszasz at mac.com > From cszasz at mac.com Thu Jul 21 15:19:48 2011 From: cszasz at mac.com (Charles Szasz) Date: Thu, 21 Jul 2011 15:19:48 -0400 Subject: Setting a dirty flag for a file Message-ID: <965FE1CF-E24C-4542-A811-600AD02D1B90@mac.com> Hi Mark, Can you elaborate on what you are suggesting here and how you can do it? Charles Szasz cszasz at mac.com From ambassador at fourthworld.com Thu Jul 21 15:20:16 2011 From: ambassador at fourthworld.com (Richard Gaskin) Date: Thu, 21 Jul 2011 12:20:16 -0700 Subject: Setting a dirty flag for a file In-Reply-To: References: Message-ID: <4E287BF0.50308@fourthworld.com> Charles Szasz wrote: > I am using Rev. 4.0. How do you set a "dirty flag" for a data > file that that has been saved but still open in your app stack? > My app creates a data file that saves the contents of 16 fields. > I have code to show an initial save dialog when the user makes > changes. I want to use a dirty flag to show another save box only > when closing the file if the user makes subsequent changes in the > data file. Otherwise when the user closes the data file without > additional change no save dialog will be presented. You can use a custom property for that, setting it to true whenever a closeField triggers, and clearing it on save. But better still would be to use the modifiedMark property of the stack, which has the additional benefit of indicating unsaved changes by darkening the closeBox on OS X. Whichever flag you use, you'll want to check it in a closeStackRequest handler, so you can prompt the user to save if needed. -- Richard Gaskin Fourth World LiveCode training and consulting: http://www.fourthworld.com Webzine for LiveCode developers: http://www.LiveCodeJournal.com LiveCode Journal blog: http://LiveCodejournal.com/blog.irv From devin_asay at byu.edu Thu Jul 21 15:28:21 2011 From: devin_asay at byu.edu (Devin Asay) Date: Thu, 21 Jul 2011 13:28:21 -0600 Subject: Setting a dirty flag for a file In-Reply-To: <4E287BF0.50308@fourthworld.com> References: <4E287BF0.50308@fourthworld.com> Message-ID: <42262BB8-A865-44F6-8A43-6672C53999AE@byu.edu> On Jul 21, 2011, at 1:20 PM, Richard Gaskin wrote: > Charles Szasz wrote: > >> I am using Rev. 4.0. How do you set a "dirty flag" for a data >> file that that has been saved but still open in your app stack? >> My app creates a data file that saves the contents of 16 fields. >> I have code to show an initial save dialog when the user makes >> changes. I want to use a dirty flag to show another save box only >> when closing the file if the user makes subsequent changes in the >> data file. Otherwise when the user closes the data file without >> additional change no save dialog will be presented. > > You can use a custom property for that, setting it to true whenever a > closeField triggers, and clearing it on save. > > But better still would be to use the modifiedMark property of the stack, > which has the additional benefit of indicating unsaved changes by > darkening the closeBox on OS X. But don't forget that the modifiedMark property only showed up in LC 4.6, and Charles said he's using v. 4.0. > > Whichever flag you use, you'll want to check it in a closeStackRequest > handler, so you can prompt the user to save if needed. For actually setting the flag you can do something as simple as comparing the contents of a field with the contents of an external file with each keystroke: on rawKeyUp # where tFilepath is a variable storing the full path to the saved file if field "mycontent" = url ("file:" & tFilepath) then set the dirtyStack of this stack to true else set the dirtyStack of this stack to false end if end rawKeyUp HTH Devin Devin Asay Humanities Technology and Research Support Center Brigham Young University From cszasz at mac.com Thu Jul 21 15:35:16 2011 From: cszasz at mac.com (Charles Szasz) Date: Thu, 21 Jul 2011 15:35:16 -0400 Subject: Setting a dirty flag for a file Message-ID: <3D4ED896-FECC-430A-AB9D-6BF46BCB51D1@mac.com> Devin and Mark, Thanks for your suggestions! I think this is one of those areas that should be covered by LiveCode in their Lessons. I would not be surprised that others had questions about how to do this task. Charles Szasz cszasz at mac.com From m.schonewille at economy-x-talk.com Thu Jul 21 16:02:21 2011 From: m.schonewille at economy-x-talk.com (Mark Schonewille) Date: Thu, 21 Jul 2011 22:02:21 +0200 Subject: Setting a dirty flag for a file In-Reply-To: <3D4ED896-FECC-430A-AB9D-6BF46BCB51D1@mac.com> References: <3D4ED896-FECC-430A-AB9D-6BF46BCB51D1@mac.com> Message-ID: Hi Charles, I read your e-mail again. I understand that you want to know whether one or more fields have changed, not whether a file is open. Usually, I generate an md5Digest and save that: on closeField makeDigest end closeField on makeDigest put empty into myDigest repeat with x = 1 to number of fields put md5Digest(fld x & myDigest) into myDigest end repeat set the cDigest of this stack to myDigest end makeDigest function dataChanged put empty into myDigest repeat with x = 1 to number of fields put md5Digest(fld x & myDigest) into myDigest end repeat return (the cDigest of this stack is myDigest) end dataChanged You just will have to figure out what is a smart event to check the digest: closing a field, closing a window, opening a window, refreshing the file menu, etc. When you close the window, you might want to do this: on closeStackRequest if dataChanged then answer "Do you want?" with "Don't Save" or "OK" or "No" if it containt "Don't" then exit closeStackRequest else if it is "No" then pass closeStackRequest else // do your saving stuff here // return true if the file was saved if the result is true then pass closeStackRequest end if end if end if end closeStackRequest -- Best regards, Mark Schonewille Economy-x-Talk Consulting and Software Engineering Homepage: http://economy-x-talk.com Twitter: http://twitter.com/xtalkprogrammer KvK: 50277553 New: Download the Installer Maker Plugin 1.6 for LiveCode here http://qery.us/ce On 21 jul 2011, at 21:35, Charles Szasz wrote: > Devin and Mark, > > Thanks for your suggestions! I think this is one of those areas that should be covered by LiveCode in their Lessons. I would not be surprised that others had questions about how to do this task. > > Charles Szasz > cszasz at mac.com > > > > > > _______________________________________________ > use-livecode mailing list > use-livecode at lists.runrev.com > Please visit this url to subscribe, unsubscribe and manage your subscription preferences: > http://lists.runrev.com/mailman/listinfo/use-livecode From lists.pete at haworths.org Thu Jul 21 16:45:44 2011 From: lists.pete at haworths.org (Pete Haworth) Date: Thu, 21 Jul 2011 13:45:44 -0700 Subject: Detecting Different Versions of Livecode Message-ID: Is there a way to detect the license type of the installed version of LiveCode? For example, the Desktop Personal license, or the Commercial Desktop ISV? Pete From pmbrig at gmail.com Thu Jul 21 17:08:52 2011 From: pmbrig at gmail.com (Peter Brigham MD) Date: Thu, 21 Jul 2011 17:08:52 -0400 Subject: Setting a dirty flag for a file In-Reply-To: References: <3D4ED896-FECC-430A-AB9D-6BF46BCB51D1@mac.com> Message-ID: <5EE81E11-ADDB-46F7-8ADA-129366F81E13@gmail.com> On Jul 21, 2011, at 4:02 PM, Mark Schonewille wrote: > Hi Charles, > > I read your e-mail again. I understand that you want to know whether one or more fields have changed, not whether a file is open. > > Usually, I generate an md5Digest and save that: > > on closeField > makeDigest > end closeField > > on makeDigest > put empty into myDigest > repeat with x = 1 to number of fields > put md5Digest(fld x & myDigest) into myDigest > end repeat > set the cDigest of this stack to myDigest > end makeDigest > > function dataChanged > put empty into myDigest > repeat with x = 1 to number of fields > put md5Digest(fld x & myDigest) into myDigest > end repeat > return (the cDigest of this stack is myDigest) > end dataChanged > > You just will have to figure out what is a smart event to check the digest: closing a field, closing a window, opening a window, refreshing the file menu, etc. > > When you close the window, you might want to do this: > > on closeStackRequest > if dataChanged then > answer "Do you want?" with "Don't Save" or "OK" or "No" > if it containt "Don't" then > exit closeStackRequest > else if it is "No" then > pass closeStackRequest > else > // do your saving stuff here > // return true if the file was saved > if the result is true then > pass closeStackRequest > end if > end if > end if > end closeStackRequest I use a frontscript -- wholesale is better than retail: on closefield setDirty pass closefield end closefield on setDirty tf if tf = empty then put true into tf put the version into v replace "." with empty in v if v < 453 then exit setDirty -- version must be 4.5.3 or higher, -- or you just create a new customprop set the modifiedMark of stack "myStack" to tf end setDirty If you want to exclude certain fields, use a customprop to mark fields that shouldn't trigger setDirty when edited: set the the dontFlagDirty of fld "excludedFld" to true Then in the closeField handler in the frontscript, insert as the first line if the dontFlagDirty of the target <> true then pass closefield Changes other than field content that should be saved (radiobuttons or checkboxes, etc) can be handled, eg, with a mouseup handler in the frontscript, and suitable checks on the target. (Pass the mouseup!) I also do: on preopenstack set dirty false ... ... ... end preopenstack -- Peter Peter M. Brigham pmbrig at gmail.com http://home.comcast.net/~pmbrig From bdrunrev at gmail.com Thu Jul 21 17:13:18 2011 From: bdrunrev at gmail.com (Bernard Devlin) Date: Thu, 21 Jul 2011 22:13:18 +0100 Subject: OT: It's Android Jim, but not as we know it (what CPUs does Livecode compile to?) Message-ID: Hi folks, This is a question for those of you who have sallied forth into the world of Android development (or, who have at least considered it). When Livecode creates an app for Android deployment, what CPU architecture does Livecode compile for? My reason for asking is that I like the look of the Asus Transformer. It is an Android tablet that docks into a keyboard. But the processor is the Tegra. I'm guessing that Livecode will not be compiling apps for such an unusual processor. Assuming that Android (ultimately) provides an OS that runs identically (let me dream) across different processor architectures, then it would be great to be able to actually run the IDE on a machine like the Transformer. From what I read, the Transformer is still a little imperfect (in terms of manufacturing quality, and pointer lag when docked into the keyboard). But it has only been around for a few months and shows great promise as a design. http://www.engadget.com/2011/04/18/asus-eee-pad-transformer-uk-edition-review/ http://thisismynext.com/2011/05/12/asus-eee-pad-transformer-tf101-review/#1_undefined,0_ As a tablet it is only marginally heavier than an iPad (about 10%). And as a tablet it has a supposed battery life of 8 hours, plus another 6 when docked to the keyboard (if fully charged on both battery packs, the usage times of both are combined to provide 14 hours or so). Being a dockable touchscreen, it has the benefits of mutliple input modes (I really can't be doing with more than minimal typing with an onscreen keyboard). I held a Transformer today, and it is pretty sweet (although I didn't get to touch the keyboard, as they were sold out). I wasn't very impressed by the quality of the early Asus EEE netbooks, but the Transformer felt like a quality product. The Acer AA1s were superior to the early Asus netbooks - for some months now I've been using a later Acer netbook for portable computing, and I've been surprisingly pleased with the form factor and the performance. Of course, I still find the Macbook Air a very, very seductive design, but Apple's attitudes have been fuelling a love/hate relationship for me for some years now. I guess I'm really hoping for too much to think that Livecode could run on a Tegra (I know the Linux IDE is only compiled for intel). Microsoft just about killed alternative operating systems on netbooks (as OEM installations), but it looks like Linux might rise again on these small laptops using Android. Certainly I've never seen an OEM install of linux on a netbook with the finesse of Android on the Asus Transformer, and I've never seen a netbook with the build quality of the Transformer tablet. And the Transformer (including the dockable keyboard) is about half the price of a Macbook Air. If the Transformer had access to the masses of apps found in a typical Linux distro AND had a Livecode IDE that was at parity with the IDE on Windows or OS X, I think I'd be in a very happy place technologically. The Wikipedia entry on the Tegra says that it is a system on a chip that incorporates an ARM processor, and that Ubuntu will run on it in addition to Android. http://en.wikipedia.org/wiki/Nvidia_Tegra http://tegradeveloper.nvidia.com/tegra/forum/what-operating-systems-are-supported-tegra I think however, the idea of getting even a basic Livecode development platform running (e.g. one that ran within the scriptlimits inside an Android app) might be the most I could hope for. Still, I did not see myself developing apps for an Android mobile phone, but running Livecode apps on an Asus Transformer would be of interest to me. Bernard From cszasz at mac.com Thu Jul 21 17:17:44 2011 From: cszasz at mac.com (Charles Szasz) Date: Thu, 21 Jul 2011 17:17:44 -0400 Subject: Setting a dirty flag for a file Message-ID: <36074EF3-EA17-4DE8-99EA-69448C4D5E93@mac.com> Mark, Thanks very much! I work on this tonight. Sent from my iPad From ambassador at fourthworld.com Thu Jul 21 17:32:11 2011 From: ambassador at fourthworld.com (Richard Gaskin) Date: Thu, 21 Jul 2011 14:32:11 -0700 Subject: OT: It's Android Jim, but not as we know it (what CPUs does Livecode compile to?) In-Reply-To: References: Message-ID: <4E289ADB.7080402@fourthworld.com> Bernard Devlin wrote: > When Livecode creates an app for Android deployment, what CPU > architecture does Livecode compile for? > > My reason for asking is that I like the look of the Asus Transformer. > It is an Android tablet that docks into a keyboard. But the processor > is the Tegra. I'm guessing that Livecode will not be compiling apps > for such an unusual processor. I haven't a clue, but that won't stop me from random conjecture. :) My hunch is that it's like AMD and Intel: different makers, compatible instruction set. I can't imagine the Tegra would expect every developer to recompile for it. If you get one let us know how you like it. Looks like a sweet machine. -- Richard Gaskin Fourth World LiveCode training and consulting: http://www.fourthworld.com Webzine for LiveCode developers: http://www.LiveCodeJournal.com LiveCode Journal blog: http://LiveCodejournal.com/blog.irv From jacque at hyperactivesw.com Thu Jul 21 17:36:24 2011 From: jacque at hyperactivesw.com (J. Landman Gay) Date: Thu, 21 Jul 2011 16:36:24 -0500 Subject: OT: It's Android Jim, but not as we know it (what CPUs does Livecode compile to?) In-Reply-To: References: Message-ID: <4E289BD8.9080908@hyperactivesw.com> On 7/21/11 4:13 PM, Bernard Devlin wrote: > Hi folks, > > This is a question for those of you who have sallied forth into the > world of Android development (or, who have at least considered it). > > When Livecode creates an app for Android deployment, what CPU > architecture does Livecode compile for? > > My reason for asking is that I like the look of the Asus Transformer. > It is an Android tablet that docks into a keyboard. But the processor > is the Tegra. I'm guessing that Livecode will not be compiling apps > for such an unusual processor. I don't know. But if the device can download and run apps from the Android market then it should be able to run anything LiveCode compiles. As I understand it, the processor is immaterial, it's the OS that counts. If the processor runs a standard Android distribution then it should run LiveCode-compiled Android apps. Maybe the first thing to find out is if it can run standard Market apps. -- Jacqueline Landman Gay | jacque at hyperactivesw.com HyperActive Software | http://www.hyperactivesw.com From josh at dvcreators.net Thu Jul 21 17:38:27 2011 From: josh at dvcreators.net (Josh Mellicker) Date: Thu, 21 Jul 2011 14:38:27 -0700 Subject: Lion problem report and fix Message-ID: <373168FE-1908-438D-B206-2BCDF7F99A26@dvcreators.net> Just wanted to let everyone know, Lion permissions default to not allowing Livecode to create folders in "/Library/Application Support/" with the "create folder" command. Here's how we got around it: First, this put "mkdir" && quote & "/Library/Application Support/theNewFolderWeNeed2create" & quote into tCmd get shell (tCmd) put it returns "Permission denied". But this: put "sudo mkdir -p" && quote & "/Library/Application Support/theNewFolderWeNeed2create" & quote into tCmd get shell (tCmd) creates the folder, and oddly, does not prompt for the password, at least in the IDE (haven't tried with a standalone yet). Then, this: put "sudo chmod -Rfv 777" && quote & "/Library/Application Support/theNewFolderWeNeed2create" & quote into tCmd get shell (tCmd) makes it so that folders and items can be freely created in the new folder by Livecode on Lion. So everything is back to normal a la the Leopards. Hope this helps someone out. And, if anyone has a better way, please post! From andre at andregarzia.com Thu Jul 21 17:52:59 2011 From: andre at andregarzia.com (Andre Garzia) Date: Thu, 21 Jul 2011 18:52:59 -0300 Subject: OT: It's Android Jim, but not as we know it (what CPUs does Livecode compile to?) In-Reply-To: <4E289BD8.9080908@hyperactivesw.com> References: <4E289BD8.9080908@hyperactivesw.com> Message-ID: On Thu, Jul 21, 2011 at 6:36 PM, J. Landman Gay wrote: > On 7/21/11 4:13 PM, Bernard Devlin wrote: > >> Hi folks, >> >> This is a question for those of you who have sallied forth into the >> world of Android development (or, who have at least considered it). >> >> When Livecode creates an app for Android deployment, what CPU >> architecture does Livecode compile for? >> >> My reason for asking is that I like the look of the Asus Transformer. >> It is an Android tablet that docks into a keyboard. But the processor >> is the Tegra. I'm guessing that Livecode will not be compiling apps >> for such an unusual processor. >> > > I don't know. But if the device can download and run apps from the Android > market then it should be able to run anything LiveCode compiles. As I > understand it, the processor is immaterial, it's the OS that counts. If the > processor runs a standard Android distribution then it should run > LiveCode-compiled Android apps. > > Not Really Jacque, Applications that target the dalvik virtual machine will run accross different CPUs but as I understand Android has some NDK thing like a Native CPU Specific Development Kit where you can compile C/C++ code such as the LiveCode engine and call it from a dalvik based application. As I understand, LiveCode is probably being built natively for the ARM CPU on Android and maybe x86 to run on the Android Emulator (or the Android Emulator is emulating ARM). This is my guess, I may be wrong though.... > Maybe the first thing to find out is if it can run standard Market apps. > > -- > Jacqueline Landman Gay | jacque at hyperactivesw.com > HyperActive Software | http://www.hyperactivesw.com > > > ______________________________**_________________ > use-livecode mailing list > use-livecode at lists.runrev.com > Please visit this url to subscribe, unsubscribe and manage your > subscription preferences: > http://lists.runrev.com/**mailman/listinfo/use-livecode > -- http://www.andregarzia.com All We Do Is Code. From ambassador at fourthworld.com Thu Jul 21 18:05:28 2011 From: ambassador at fourthworld.com (Richard Gaskin) Date: Thu, 21 Jul 2011 15:05:28 -0700 Subject: Lion problem report and fix In-Reply-To: <373168FE-1908-438D-B206-2BCDF7F99A26@dvcreators.net> References: <373168FE-1908-438D-B206-2BCDF7F99A26@dvcreators.net> Message-ID: <4E28A2A8.4070908@fourthworld.com> Josh Mellicker wrote: > Just wanted to let everyone know, Lion permissions default to not allowing Livecode to create folders in "/Library/Application Support/" with the "create folder" command. I believe that's true with earlier versions as well if the current user is not admin. Apple recommends using the user's App Support folder, where LC's "create folder" seems to work well: Contains all application-specific data and support files. These are the files that your application creates and manages on behalf of the user and can include files that contain user data. By convention, all of these items should be put in a subdirectory whose name matches the bundle identifier of the application. For example, if your application is named MyApp and has the bundle identifier com.example.MyApp, you would put your application?s user-specific data files and resources in the ~/Library/Application Support/com.example.MyApp/ directory. Your application is responsible for creating this directory as needed. There may be special cases where you'd need to use the system-wide App Support folder, but those should ideally invoke the admin dialog, since any access to system directories without that would pose a security risk. -- Richard Gaskin Fourth World LiveCode training and consulting: http://www.fourthworld.com Webzine for LiveCode developers: http://www.LiveCodeJournal.com LiveCode Journal blog: http://LiveCodejournal.com/blog.irv From francois.chaplais at mines-paristech.fr Thu Jul 21 18:14:27 2011 From: francois.chaplais at mines-paristech.fr (=?iso-8859-1?Q?Fran=E7ois_Chaplais?=) Date: Fri, 22 Jul 2011 00:14:27 +0200 Subject: OT: It's Android Jim, but not as we know it (what CPUs does Livecode compile to?) In-Reply-To: References: <4E289BD8.9080908@hyperactivesw.com> Message-ID: <23610190-5630-4839-9BAC-7AB5D0CF0D30@mines-paristech.fr> Le 21 juil. 2011 ? 23:52, Andre Garzia a ?crit : > On Thu, Jul 21, 2011 at 6:36 PM, J. Landman Gay wrote: > >> On 7/21/11 4:13 PM, Bernard Devlin wrote: >> >>> Hi folks, >>> >>> This is a question for those of you who have sallied forth into the >>> world of Android development (or, who have at least considered it). >>> >>> When Livecode creates an app for Android deployment, what CPU >>> architecture does Livecode compile for? >>> >>> My reason for asking is that I like the look of the Asus Transformer. >>> It is an Android tablet that docks into a keyboard. But the processor >>> is the Tegra. I'm guessing that Livecode will not be compiling apps >>> for such an unusual processor. >>> >> >> I don't know. But if the device can download and run apps from the Android >> market then it should be able to run anything LiveCode compiles. As I >> understand it, the processor is immaterial, it's the OS that counts. If the >> processor runs a standard Android distribution then it should run >> LiveCode-compiled Android apps. >> >> > Not Really Jacque, > > Applications that target the dalvik virtual machine will run accross > different CPUs but as I understand Android has some NDK thing like a Native > CPU Specific Development Kit where you can compile C/C++ code such as the > LiveCode engine and call it from a dalvik based application. As I > understand, LiveCode is probably being built natively for the ARM CPU on > Android and maybe x86 to run on the Android Emulator (or the Android > Emulator is emulating ARM). > > This is my guess, I may be wrong though.... > Isn'it some kind of Java? The kind of Java Larry Ellison is suing Google about? From m.schonewille at economy-x-talk.com Thu Jul 21 18:14:34 2011 From: m.schonewille at economy-x-talk.com (Mark Schonewille) Date: Fri, 22 Jul 2011 00:14:34 +0200 Subject: Lion problem report and fix In-Reply-To: <373168FE-1908-438D-B206-2BCDF7F99A26@dvcreators.net> References: <373168FE-1908-438D-B206-2BCDF7F99A26@dvcreators.net> Message-ID: <207022DD-A972-4A26-8486-16552E8BA149@economy-x-talk.com> Hi Josh, That's seems normal, since /Library isn't a user folder. Are you sure this is Lion-specific? Have you tried ~/Library/Application Support? -- Best regards, Mark Schonewille Economy-x-Talk Consulting and Software Engineering Homepage: http://economy-x-talk.com Twitter: http://twitter.com/xtalkprogrammer KvK: 50277553 New: Download the Installer Maker Plugin 1.6 for LiveCode here http://qery.us/ce On 21 jul 2011, at 23:38, Josh Mellicker wrote: > Just wanted to let everyone know, Lion permissions default to not allowing Livecode to create folders in "/Library/Application Support/" with the "create folder" command. > > Here's how we got around it: > > First, this > > put "mkdir" && quote & "/Library/Application Support/theNewFolderWeNeed2create" & quote into tCmd > get shell (tCmd) > put it > > returns "Permission denied". > > > But this: > > put "sudo mkdir -p" && quote & "/Library/Application Support/theNewFolderWeNeed2create" & quote into tCmd > get shell (tCmd) > > creates the folder, and oddly, does not prompt for the password, at least in the IDE (haven't tried with a standalone yet). > > Then, this: > > put "sudo chmod -Rfv 777" && quote & "/Library/Application Support/theNewFolderWeNeed2create" & quote into tCmd > get shell (tCmd) > > makes it so that folders and items can be freely created in the new folder by Livecode on Lion. So everything is back to normal a la the Leopards. > > > Hope this helps someone out. > > > And, if anyone has a better way, please post! From lists.pete at haworths.org Thu Jul 21 18:15:22 2011 From: lists.pete at haworths.org (Pete Haworth) Date: Thu, 21 Jul 2011 15:15:22 -0700 Subject: Lion problem report and fix In-Reply-To: <4E28A2A8.4070908@fourthworld.com> References: <373168FE-1908-438D-B206-2BCDF7F99A26@dvcreators.net> <4E28A2A8.4070908@fourthworld.com> Message-ID: Thanks for that knowledge Richard, very useful. One thing I noticed on my Mac (OS X 10.6.8) is that all the folders inside the Application Support folder are named simply with the application name not with bundle identifier, so in the example quoted, they are in the MyApp folder, not the com.example.MyApp folder. Pete On Thu, Jul 21, 2011 at 3:05 PM, Richard Gaskin wrote: > Josh Mellicker wrote: > > Just wanted to let everyone know, Lion permissions default to not allowing >> Livecode to create folders in "/Library/Application Support/" with the >> "create folder" command. >> > > I believe that's true with earlier versions as well if the current user is > not admin. > > Apple recommends using the user's App Support folder, where LC's "create > folder" seems to work well: > > Contains all application-specific data and support files. These > are the files that your application creates and manages on behalf > of the user and can include files that contain user data. > > By convention, all of these items should be put in a subdirectory > whose name matches the bundle identifier of the application. For > example, if your application is named MyApp and has the bundle > identifier com.example.MyApp, you would put your application?s > user-specific data files and resources in the ~/Library/Application > Support/com.example.MyApp/ directory. Your application is > responsible for creating this directory as needed. > > FileManagement/Conceptual/**FileSystemProgrammingGUide/** > MacOSXDirectories/**MacOSXDirectories.html#//** > apple_ref/doc/uid/TP40010672-**CH10-SW1 > > > > There may be special cases where you'd need to use the system-wide App > Support folder, but those should ideally invoke the admin dialog, since any > access to system directories without that would pose a security risk. > > -- > Richard Gaskin > Fourth World > LiveCode training and consulting: http://www.fourthworld.com > Webzine for LiveCode developers: http://www.LiveCodeJournal.com > LiveCode Journal blog: http://LiveCodejournal.com/**blog.irv > > ______________________________**_________________ > use-livecode mailing list > use-livecode at lists.runrev.com > Please visit this url to subscribe, unsubscribe and manage your > subscription preferences: > http://lists.runrev.com/**mailman/listinfo/use-livecode > From bobs at twft.com Thu Jul 21 18:16:06 2011 From: bobs at twft.com (Bob Sneidar) Date: Thu, 21 Jul 2011 15:16:06 -0700 Subject: OT: It's Android Jim, but not as we know it (what CPUs does Livecode compile to?) In-Reply-To: References: Message-ID: Aslong as this is OT and we are on the subject of Android, has anyone else noticed that Android Wifi Tethering simply hoses Airport Wireless? Sure seems to around here. Bob On Jul 21, 2011, at 2:13 PM, Bernard Devlin wrote: > Hi folks, > > This is a question for those of you who have sallied forth into the > world of Android development (or, who have at least considered it). > > When Livecode creates an app for Android deployment, what CPU > architecture does Livecode compile for? From martyknapp at comcast.net Thu Jul 21 18:17:32 2011 From: martyknapp at comcast.net (Marty Knapp) Date: Thu, 21 Jul 2011 15:17:32 -0700 Subject: Lion problem report and fix In-Reply-To: <4E28A2A8.4070908@fourthworld.com> References: <373168FE-1908-438D-B206-2BCDF7F99A26@dvcreators.net> <4E28A2A8.4070908@fourthworld.com> Message-ID: <4E28A57C.4020504@comcast.net> Interestingly, the user's Library folder, and thus the Application Support folder is invisible on my Lion install. I can write to it OK, though (only tested as admin). Marty Knapp > Josh Mellicker wrote: > >> Just wanted to let everyone know, Lion permissions default to not >> allowing Livecode to create folders in "/Library/Application >> Support/" with the "create folder" command. > > I believe that's true with earlier versions as well if the current > user is not admin. > > Apple recommends using the user's App Support folder, where LC's > "create folder" seems to work well: > > Contains all application-specific data and support files. These > are the files that your application creates and manages on behalf > of the user and can include files that contain user data. > > By convention, all of these items should be put in a subdirectory > whose name matches the bundle identifier of the application. For > example, if your application is named MyApp and has the bundle > identifier com.example.MyApp, you would put your application?s > user-specific data files and resources in the ~/Library/Application > Support/com.example.MyApp/ directory. Your application is > responsible for creating this directory as needed. > > > > > There may be special cases where you'd need to use the system-wide App > Support folder, but those should ideally invoke the admin dialog, > since any access to system directories without that would pose a > security risk. > > -- > Richard Gaskin > Fourth World > LiveCode training and consulting: http://www.fourthworld.com > Webzine for LiveCode developers: http://www.LiveCodeJournal.com > LiveCode Journal blog: http://LiveCodejournal.com/blog.irv From ambassador at fourthworld.com Thu Jul 21 18:18:56 2011 From: ambassador at fourthworld.com (Richard Gaskin) Date: Thu, 21 Jul 2011 15:18:56 -0700 Subject: Lion problem report and fix In-Reply-To: References: Message-ID: <4E28A5D0.6020105@fourthworld.com> Pete Haworth wrote: > Thanks for that knowledge Richard, very useful. One thing I noticed on my > Mac (OS X 10.6.8) is that all the folders inside the Application Support > folder are named simply with the application name not with bundle > identifier, so in the example quoted, they are in the MyApp folder, not the > com.example.MyApp folder. What Apple recommends and what they actually do are not always the same thing. ;) -- Richard Gaskin Fourth World LiveCode training and consulting: http://www.fourthworld.com Webzine for LiveCode developers: http://www.LiveCodeJournal.com LiveCode Journal blog: http://LiveCodejournal.com/blog.irv From ambassador at fourthworld.com Thu Jul 21 18:22:43 2011 From: ambassador at fourthworld.com (Richard Gaskin) Date: Thu, 21 Jul 2011 15:22:43 -0700 Subject: Lion problem report and fix In-Reply-To: <4E28A57C.4020504@comcast.net> References: <4E28A57C.4020504@comcast.net> Message-ID: <4E28A6B3.20308@fourthworld.com> Marty Knapp wrote: > Interestingly, the user's Library folder, and thus the Application > Support folder is invisible on my Lion install. I can write to it OK, > though (only tested as admin). This sort of thing was touched in in the WWDC keynote. In short, Steve believes file systems are confusing to users, so Apple is removing end-user access to anything they don't really need. If you prowl around in Terminal you'll find there are a lot of folders that have been hidden for years. Lion just adds more. -- Richard Gaskin Fourth World LiveCode training and consulting: http://www.fourthworld.com Webzine for LiveCode developers: http://www.LiveCodeJournal.com LiveCode Journal blog: http://LiveCodejournal.com/blog.irv From m.schonewille at economy-x-talk.com Thu Jul 21 18:29:58 2011 From: m.schonewille at economy-x-talk.com (Mark Schonewille) Date: Fri, 22 Jul 2011 00:29:58 +0200 Subject: Lion problem report and fix In-Reply-To: <4E28A57C.4020504@comcast.net> References: <373168FE-1908-438D-B206-2BCDF7F99A26@dvcreators.net> <4E28A2A8.4070908@fourthworld.com> <4E28A57C.4020504@comcast.net> Message-ID: Hi, By default, the ~/Library folder is hidden in Lion. This is a big problem to many of the readers of this site, as many hints involve this folder (especially any involving hidden preferences). You can easily un-hide this folder for easy access. Simply run the following command in Terminal: chflags nohidden ~/Library If, for some reason, you want to re-hide it, you can run the same command using 'hidden' instead of 'nohidden'. You can actually use this command to hide or unhide any folder. Just put it's path in place of ~/Library. source: http://qery.us/vf You can also press shift-command-G and type "~/Library" to get access to the folder in the Finder. -- Best regards, Mark Schonewille Economy-x-Talk Consulting and Software Engineering Homepage: http://economy-x-talk.com Twitter: http://twitter.com/xtalkprogrammer KvK: 50277553 New: Download the Installer Maker Plugin 1.6 for LiveCode here http://qery.us/ce On 22 jul 2011, at 00:17, Marty Knapp wrote: > Interestingly, the user's Library folder, and thus the Application Support folder is invisible on my Lion install. I can write to it OK, though (only tested as admin). > > Marty Knapp From bobs at twft.com Thu Jul 21 18:46:03 2011 From: bobs at twft.com (Bob Sneidar) Date: Thu, 21 Jul 2011 15:46:03 -0700 Subject: How is it? Message-ID: How is it that I can have the selection tool selected, and clicking on an object sends a mouseUp to it and executes the script? Is anyone else seeing wierdness like this in 4.6.3? Other things are going awry as well similar to this. I'm getting scared. Bob From revmaillist at positivme.com Thu Jul 21 18:50:31 2011 From: revmaillist at positivme.com (Thunder) Date: Thu, 21 Jul 2011 15:50:31 -0700 (PDT) Subject: Detecting Different Versions of Livecode In-Reply-To: References: Message-ID: <1311288631267-3685390.post@n4.nabble.com> Try this : revLicenseType() and revLicenseInfo -- View this message in context: http://runtime-revolution.278305.n4.nabble.com/Detecting-Different-Versions-of-Livecode-tp3684990p3685390.html Sent from the Revolution - User mailing list archive at Nabble.com. From bdrunrev at gmail.com Thu Jul 21 19:02:41 2011 From: bdrunrev at gmail.com (Bernard Devlin) Date: Fri, 22 Jul 2011 00:02:41 +0100 Subject: OT: It's Android Jim, but not as we know it (what CPUs does Livecode compile to?) In-Reply-To: References: <4E289BD8.9080908@hyperactivesw.com> Message-ID: Since the Tegra has an ARM processor on the system chip, then it would seem to fundamentally be an ARM processor. It is a rather amazing ecosystem where linux/google/ARM/nvidia/asus combine to produce something like that, which was probably never envisaged by any of them 10 years ago. It does run things from the (or at least "an") Android marketplace (I guess it might be configured to go to a processor-specific marketplace). Since I'm such a total noob in this particular area, I don't know the ins and outs. But certainly it lists "top free apps" and "top paid apps" that can be installed. If someone provides me with an identifiable free app from the Android Marketplace which was coded in Livecode, then I can go back to the store and see if I can find, install and run it. Bernard On Thu, Jul 21, 2011 at 10:52 PM, Andre Garzia wrote: > On Thu, Jul 21, 2011 at 6:36 PM, J. Landman Gay wrote: > >> I don't know. But if the device can download and run apps from the Android >> market then it should be able to run anything LiveCode compiles. As I >> understand it, the processor is immaterial, it's the OS that counts. If the >> processor runs a standard Android distribution then it should run >> LiveCode-compiled Android apps. >> >> > Not Really Jacque, > > Applications that target the dalvik virtual machine will run accross > different CPUs but as I understand Android has some NDK thing like a Native > CPU Specific Development Kit where you can compile C/C++ code such as the > LiveCode engine and call it from a dalvik based application. As I > understand, LiveCode is probably being built natively for the ARM CPU on > Android and maybe x86 to run on the Android Emulator (or the Android > Emulator is emulating ARM). > > This is my guess, I may be wrong though.... > >> Maybe the first thing to find out is if it can run standard Market apps. >> >> -- >> Jacqueline Landman Gay ? ? ? ? | ? ? jacque at hyperactivesw.com From ambassador at fourthworld.com Thu Jul 21 19:03:56 2011 From: ambassador at fourthworld.com (Richard Gaskin) Date: Thu, 21 Jul 2011 16:03:56 -0700 Subject: How is it? In-Reply-To: References: Message-ID: <4E28B05C.9010004@fourthworld.com> Bob Sneidar wrote: > How is it that I can have the selection tool selected, and clicking > on an object sends a mouseUp to it and executes the script? Is anyone > else seeing wierdness like this in 4.6.3? Is this in the IDE or in a standalone only? If in the IDE, does it only happen when "Suspend IDE" is active? When the mouse goes up a mouseUp message is sent, regardless whether the pointer or browse tool is active (U have an RQCC request for a separate suite of messages for the pointer tool like SuperCard provides, but that's another story). Historically, the IDE eats those messages when the pointer tool is active. If this isn't happening that way for you it may be a bug. While it would reflect engine behaviors, it can be confusing to try to author like that. -- Richard Gaskin Fourth World LiveCode training and consulting: http://www.fourthworld.com Webzine for LiveCode developers: http://www.LiveCodeJournal.com LiveCode Journal blog: http://LiveCodejournal.com/blog.irv From josh at dvcreators.net Thu Jul 21 19:06:03 2011 From: josh at dvcreators.net (Josh Mellicker) Date: Thu, 21 Jul 2011 16:06:03 -0700 Subject: Lion problem report and fix In-Reply-To: <207022DD-A972-4A26-8486-16552E8BA149@economy-x-talk.com> References: <373168FE-1908-438D-B206-2BCDF7F99A26@dvcreators.net> <207022DD-A972-4A26-8486-16552E8BA149@economy-x-talk.com> Message-ID: On Jul 21, 2011, at 3:14 PM, Mark Schonewille wrote: > Hi Josh, > > That's seems normal, since /Library isn't a user folder. Are you sure this is Lion-specific? Yes, at least with all the copies of Lion we've encountered so far. On Jul 21, 2011, at 3:14 PM, Mark Schonewille wrote: > Have you tried ~/Library/Application Support? We used to use that folder, but switched because we encountered some cases of multiple users on the same computer (and we wanted to make things work for all users). From ambassador at fourthworld.com Thu Jul 21 19:08:31 2011 From: ambassador at fourthworld.com (Richard Gaskin) Date: Thu, 21 Jul 2011 16:08:31 -0700 Subject: Lion problem report and fix In-Reply-To: References: Message-ID: <4E28B16F.4060300@fourthworld.com> Josh Mellicker wrote: > On Jul 21, 2011, at 3:14 PM, Mark Schonewille wrote: > >> Hi Josh, >> >> That's seems normal, since /Library isn't a user folder. Are you sure this is Lion-specific? > > Yes, at least with all the copies of Lion we've encountered so far. Is it any different in Snow Leopard? -- Richard Gaskin Fourth World LiveCode training and consulting: http://www.fourthworld.com Webzine for LiveCode developers: http://www.LiveCodeJournal.com LiveCode Journal blog: http://LiveCodejournal.com/blog.irv From jacque at hyperactivesw.com Thu Jul 21 19:27:23 2011 From: jacque at hyperactivesw.com (J. Landman Gay) Date: Thu, 21 Jul 2011 18:27:23 -0500 Subject: OT: It's Android Jim, but not as we know it (what CPUs does Livecode compile to?) In-Reply-To: References: <4E289BD8.9080908@hyperactivesw.com> Message-ID: <4E28B5DB.7040204@hyperactivesw.com> On 7/21/11 4:52 PM, Andre Garzia wrote: > Applications that target the dalvik virtual machine will run accross > different CPUs but as I understand Android has some NDK thing like a Native > CPU Specific Development Kit where you can compile C/C++ code such as the > LiveCode engine and call it from a dalvik based application. As I > understand, LiveCode is probably being built natively for the ARM CPU on > Android and maybe x86 to run on the Android Emulator (or the Android > Emulator is emulating ARM). > > This is my guess, I may be wrong though.... You'd know better than me, so you're probably right. But if the machine can run Market apps, would that be a good test? -- Jacqueline Landman Gay | jacque at hyperactivesw.com HyperActive Software | http://www.hyperactivesw.com From lists.pete at haworths.org Thu Jul 21 19:27:34 2011 From: lists.pete at haworths.org (Pete Haworth) Date: Thu, 21 Jul 2011 16:27:34 -0700 Subject: Detecting Different Versions of Livecode In-Reply-To: <1311288631267-3685390.post@n4.nabble.com> References: <1311288631267-3685390.post@n4.nabble.com> Message-ID: Thanks - revLicenseInfo is what I was looking for (not documented in the dictionary) Pete On Thu, Jul 21, 2011 at 3:50 PM, Thunder wrote: > Try this : > > revLicenseType() > > and > > revLicenseInfo > > -- > View this message in context: > http://runtime-revolution.278305.n4.nabble.com/Detecting-Different-Versions-of-Livecode-tp3684990p3685390.html > Sent from the Revolution - User mailing list archive at Nabble.com. > > _______________________________________________ > use-livecode mailing list > use-livecode at lists.runrev.com > Please visit this url to subscribe, unsubscribe and manage your > subscription preferences: > http://lists.runrev.com/mailman/listinfo/use-livecode > From lists.pete at haworths.org Thu Jul 21 19:27:34 2011 From: lists.pete at haworths.org (Pete Haworth) Date: Thu, 21 Jul 2011 16:27:34 -0700 Subject: Detecting Different Versions of Livecode In-Reply-To: <1311288631267-3685390.post@n4.nabble.com> References: <1311288631267-3685390.post@n4.nabble.com> Message-ID: Thanks - revLicenseInfo is what I was looking for (not documented in the dictionary) Pete On Thu, Jul 21, 2011 at 3:50 PM, Thunder wrote: > Try this : > > revLicenseType() > > and > > revLicenseInfo > > -- > View this message in context: > http://runtime-revolution.278305.n4.nabble.com/Detecting-Different-Versions-of-Livecode-tp3684990p3685390.html > Sent from the Revolution - User mailing list archive at Nabble.com. > > _______________________________________________ > use-livecode mailing list > use-livecode at lists.runrev.com > Please visit this url to subscribe, unsubscribe and manage your > subscription preferences: > http://lists.runrev.com/mailman/listinfo/use-livecode > From jacque at hyperactivesw.com Thu Jul 21 19:31:54 2011 From: jacque at hyperactivesw.com (J. Landman Gay) Date: Thu, 21 Jul 2011 18:31:54 -0500 Subject: Lion problem report and fix In-Reply-To: <4E28A57C.4020504@comcast.net> References: <373168FE-1908-438D-B206-2BCDF7F99A26@dvcreators.net> <4E28A2A8.4070908@fourthworld.com> <4E28A57C.4020504@comcast.net> Message-ID: <4E28B6EA.3030709@hyperactivesw.com> On 7/21/11 5:17 PM, Marty Knapp wrote: > Interestingly, the user's Library folder, and thus the Application > Support folder is invisible on my Lion install. I can write to it OK, > though (only tested as admin). I was just reading about this. Apparently you can hold down the Option key while choosing the Go menu in the Finder, and a Library item appears. I've read about several shortcuts (and the usual Terminal fix,) but this one seems fastest. (I haven't installed Lion yet, so all my info is based on reading.) -- Jacqueline Landman Gay | jacque at hyperactivesw.com HyperActive Software | http://www.hyperactivesw.com From bobs at twft.com Thu Jul 21 19:41:29 2011 From: bobs at twft.com (Bob Sneidar) Date: Thu, 21 Jul 2011 16:41:29 -0700 Subject: How is it? In-Reply-To: <4E28B05C.9010004@fourthworld.com> References: <4E28B05C.9010004@fourthworld.com> Message-ID: <287C7F6F-E019-4194-9528-ACA4C49436F6@twft.com> Hi Richard. It's in the IDE. I am running an alt debugger and an alt script editor whose names shall remain unspoken, so it's not clear what is causing the problem. But if someone else was seeing it, and I could determine off list if they were also using the same script editor and debugger, then I could forward the issue to the proper people. Bob On Jul 21, 2011, at 4:03 PM, Richard Gaskin wrote: > Bob Sneidar wrote: > > How is it that I can have the selection tool selected, and clicking > > on an object sends a mouseUp to it and executes the script? Is anyone > > else seeing wierdness like this in 4.6.3? > > Is this in the IDE or in a standalone only? If in the IDE, does it only happen when "Suspend IDE" is active? > > When the mouse goes up a mouseUp message is sent, regardless whether the pointer or browse tool is active (U have an RQCC request for a separate suite of messages for the pointer tool like SuperCard provides, but that's another story). > > Historically, the IDE eats those messages when the pointer tool is active. > > If this isn't happening that way for you it may be a bug. While it would reflect engine behaviors, it can be confusing to try to author like that. > > -- > Richard Gaskin > Fourth World > LiveCode training and consulting: http://www.fourthworld.com > Webzine for LiveCode developers: http://www.LiveCodeJournal.com > LiveCode Journal blog: http://LiveCodejournal.com/blog.irv > > _______________________________________________ > use-livecode mailing list > use-livecode at lists.runrev.com > Please visit this url to subscribe, unsubscribe and manage your subscription preferences: > http://lists.runrev.com/mailman/listinfo/use-livecode From bobs at twft.com Thu Jul 21 19:44:10 2011 From: bobs at twft.com (Bob Sneidar) Date: Thu, 21 Jul 2011 16:44:10 -0700 Subject: Lion problem report and fix In-Reply-To: <4E28B6EA.3030709@hyperactivesw.com> References: <373168FE-1908-438D-B206-2BCDF7F99A26@dvcreators.net> <4E28A2A8.4070908@fourthworld.com> <4E28A57C.4020504@comcast.net> <4E28B6EA.3030709@hyperactivesw.com> Message-ID: <253C2BBC-9D91-49EE-84A8-B2AA1D43F0B7@twft.com> Hmmm... doesn't work like that for me. It still remembers the last thing I had entered. Are you talking about Got to folder? Bob On Jul 21, 2011, at 4:31 PM, J. Landman Gay wrote: > On 7/21/11 5:17 PM, Marty Knapp wrote: >> Interestingly, the user's Library folder, and thus the Application >> Support folder is invisible on my Lion install. I can write to it OK, >> though (only tested as admin). > > I was just reading about this. Apparently you can hold down the Option key while choosing the Go menu in the Finder, and a Library item appears. I've read about several shortcuts (and the usual Terminal fix,) but this one seems fastest. (I haven't installed Lion yet, so all my info is based on reading.) > > -- > Jacqueline Landman Gay | jacque at hyperactivesw.com > HyperActive Software | http://www.hyperactivesw.com > > _______________________________________________ > use-livecode mailing list > use-livecode at lists.runrev.com > Please visit this url to subscribe, unsubscribe and manage your subscription preferences: > http://lists.runrev.com/mailman/listinfo/use-livecode From bvg at mac.com Thu Jul 21 19:44:04 2011 From: bvg at mac.com (=?iso-8859-1?Q?Bj=F6rnke_von_Gierke?=) Date: Fri, 22 Jul 2011 01:44:04 +0200 Subject: LiveCode.tv event #32 wrap up In-Reply-To: References: <6B865405-BBC0-491C-AD25-BFB1C85A2D58@mac.com> <7EC6A85B-DD6B-4138-A597-512A194F4453@mac.com> <89782C22-C6E3-4245-95D1-A48F69423753@mac.com> <061D9182-B34B-4ABB-A7D6-9C2D09DA8755@mac.com> <73F84954-890B-4627-859A-702C8054F13B@mac.com> <093DEF5E-6C0F-4924-A262-F6DBE748ED45@mac.com> <3FB852C9-5675-4A19-9506-543733BB2547@mac.com> <914DF999-596E-4023-A213-9469C6A69FA0@mac.com> <2B105965-FB27-41D0-B95F-1F4C84ADD53B@mac.com> <163225D1-67D1-4CE7-8049-E85A8D94D177@mac.com> <225B0941-5D11-434A-BC0B-CD61B998E9F8@mac.com> <201756EB-9601-417D-856E-5128C5256EF1@mac.com> <081FD717-0A48-447D-90AA-A8F370B14F43@mac.com> <913B7E78-3052-4CF0-9883-C1CDD9F8BDFA@mac.com> Message-ID: <0C1B6D89-1745-46DF-9C49-A5FE2E5B67B9@mac.com> We had a jolly good time First I showed off some of the Runrev.com/store marketplace 3rd party add ons to rev (and I even sticked mostly to the poll). As usual it was a disjointed random mess, and technical difficulties (read: disconnects) abound: http://www.ustream.tv/recorded/16037131 http://www.ustream.tv/recorded/16037272 http://www.ustream.tv/recorded/16037513 http://www.ustream.tv/recorded/16037739 Then, the European HyperCard User Group (eHUG, http://www.ehug.info) and TidBITS gave the eBook ?Take Control of TextExpander? to Roger Eller. You can find more info on the TC by TidBITS series here: http://www.takecontrolbooks.com We also had a nice presentation by Pete, who showed his progress on dynamically creating SQL databases and queries by placing objects on a stack (it's amazing to see). http://www.ustream.tv/recorded/16039034 See you all next time (MAKE A PRESENTATION!!!) on www.livecode.tv Bj?rnke On 15 Jul 2011, at 04:12, Bj?rnke von Gierke wrote: > Join us once again, when we show LiveCode at it's best. > > This Saturday, Pete Haworth will update us on his qd[SQL] development. qd[SQL] is a currently in development, and will allow you to easily create databases without using any SQL at all, by linking LiveCodes object/group/card metaphor to a backend Database automatically. > > See also his explanation on Google Docs: https://docs.google.com/document/d/1Rl7B3JJrnmOGctXqd-OL26a37jN10cVD43lujyF26uE/edit?hl=en_US > > Then, the European HyperCard User Group (eHUG, http://www.ehug.info) will raffle off an e-book courtesy of TidBITS. This time around it?s ?Take Control of TextExpander?. > You can find more info on the TC series here: > http://www.takecontrolbooks.com > > Finally, Bj?rnke von Gierke will take you on a tour trough some of the add-ons from the 3 for 2 special deal ( http://www.runrev.com/mailers/3for2/3for2.html ), currently offered on the RunRev store with the coupon code "THREEFORTWO". Some of the creator of those plugins will also be available in chat, so keep those questions ready. > > Don't forget that you can vote on which plug-in you are interested in: > http://blog.livecode.tv/2011/07/poll-for-event-32/ > > > All this and more on Saturdays Livecode.tv show: > Sat. 22:00 Moscow > Sat. 20:00 Paris > Sat. 19:00h GMT > Sat. 14:00 New York > Sat. 11:00 Los Angeles > Sun. 04:00 Sydney > Sun. 03:00 Tokyo > Sun. 2:00 Beijing > > Make sure to join ChatRev on the right time, otherwise you won?t know where to watch, because the correct streams will be announced in ChatRev. ChatRev is open 24/7 for your chatting needs. > Go to > http://bjoernke.com/chatrev/ > or enter in the message box: > go stack URL ?http://bjoernke.com/chatrev/chatrev1.3b3.rev? > > If you would like to show something in an event, now or somewhen, please head over to the participation page for ideas, or contact us: > http://blog.livecode.tv/participate/ > http://www3.economy-x-talk.com/file.php?node=contact > > cheers > Bj?rnke > _______________________________________________ > use-livecode mailing list > use-livecode at lists.runrev.com > Please visit this url to subscribe, unsubscribe and manage your subscription preferences: > http://lists.runrev.com/mailman/listinfo/use-livecode From lists.pete at haworths.org Thu Jul 21 19:46:01 2011 From: lists.pete at haworths.org (Pete Haworth) Date: Thu, 21 Jul 2011 16:46:01 -0700 Subject: The Multitude of LC web dev tools Message-ID: So far, I've kept myself strictly to desktop apps but wanting to start working on web stuff now (and Android but that's a different email). I see many terms mentioned on this list about LC web tools and I'm not 100% sure as to exactly what they all do so I'm hoping I can get some guidance. Here's what I think everything is: On-Rev - a paid web hosting service from Revolution. For any web applications or just those developed with LC and if the latter, with what tools? revBrowser - a control you can place on a card and use as a web browser to any url you could reach with a standalone browser. revServer - a way to develop web server applications using a mixture of LC stacks and html revIgniter - a third part tool to help with web server development - is that any web server or does it have to be a Revolution based server? I also see reference to a way of running CGI scripts with an older version of Revolution on the web server. And then there's Rodeo although I haven't seen much mention of that recently and I don;t believe it involves LC scripts? How far off base am I with those descriptions? No doubt there's other tools out there as well. So, let's say I have an application in mind that a user will access from his/her favorite web browser either on a desktop or perhaps an Android device. On the server, I need to format pages of html to send to the user. and access data in an SQL database. Some of the pages will be static and others will change depending on database content. What are the recommended tools to achieve this? Pete From kray at sonsothunder.com Thu Jul 21 19:52:08 2011 From: kray at sonsothunder.com (Ken Ray) Date: Thu, 21 Jul 2011 18:52:08 -0500 Subject: Lion 'Gotcha' #1: Creating a Support Folder Message-ID: put (specialFolderPath("asup") & "/Test") into tSupportFolder create folder tSupportFolder This works under Snow Leopard if you are a logged-in admin user without error, and it assigns the currently logged-in user as the owner of the folder. In Lion, you get a "can't create that directory" error in the result. The workaround is to prompt for a password via AppleScript: put "do shell script" && quote & "mkdir '" & tSupportFolder & \ "'" & quote && "with administrator privileges" into tScript do tScript as "AppleScript" if the result <> (quote & quote) then -- report the AppleScript error end if Note that the resulting folder is owned by 'system' not the logged-in user, so you'll need to 'chmod' and 'chown' if you want to set the privileges back to what it was pre-Lion. Ken Ray Sons of Thunder Software, Inc. Email: kray at sonsothunder.com Web Site: http://www.sonsothunder.com/ From bvg at mac.com Thu Jul 21 19:57:16 2011 From: bvg at mac.com (=?iso-8859-1?Q?Bj=F6rnke_von_Gierke?=) Date: Fri, 22 Jul 2011 01:57:16 +0200 Subject: The Multitude of LC web dev tools In-Reply-To: References: Message-ID: > On-Rev - a paid web hosting service from Revolution. For any web > applications or just those developed with LC and if the latter, with what > tools? Any website supports php, perl, cgi and all that other stuff. ANd of course on-rev also has revServer built in. > revServer - a way to develop web server applications using a mixture of LC > stacks and html Using a mixture of LC scripts and html (stacks can be used, but are not integral to the technology, and where added just this month). For example:

it is now:

> revIgniter - a third part tool to help with web server development - is that > any web server or does it have to be a Revolution based server? Only works with RevServer (for example on-rev hosted sites) > I also see reference to a way of running CGI scripts with an older version > of Revolution on the web server. Everyone should mail RunRev and ask for the CGI engine to be given to them. basically this free feature got removed in favour of revServer, but you can still get that older version of Rev (wasn't LC yet), just ask. > And then there's Rodeo although I haven't seen much mention of that recently > and I don;t believe it involves LC scripts? Rodeo is made with LC, but not specific to LC coding. > So, let's say I have an application in mind that a user will access from > his/her favorite web browser either on a desktop or perhaps an Android > device. On the server, I need to format pages of html to send to the user. > and access data in an SQL database. Some of the pages will be static and > others will change depending on database content. What are the recommended > tools to achieve this? use cgi or on-rev, depending on your budget, and how much you value your own time (CGI or revServer on your own host is harder to set up then just buying on-rev). From kray at sonsothunder.com Thu Jul 21 20:00:32 2011 From: kray at sonsothunder.com (Ken Ray) Date: Thu, 21 Jul 2011 19:00:32 -0500 Subject: Setting a dirty flag for a file In-Reply-To: <42262BB8-A865-44F6-8A43-6672C53999AE@byu.edu> References: <4E287BF0.50308@fourthworld.com> <42262BB8-A865-44F6-8A43-6672C53999AE@byu.edu> Message-ID: <68284CA1-2238-4093-8F45-2E8E9101F352@sonsothunder.com> > But don't forget that the modifiedMark property only showed up in LC 4.6, and Charles said he's using v. 4.0. True, but FYI Sean Shao has an external that works with 4.0 that does the same thing. I've been using it in Stykz for quite a while. You can contact her if you need it... Ken Ray Sons of Thunder Software, Inc. Email: kray at sonsothunder.com Web Site: http://www.sonsothunder.com/ From kray at sonsothunder.com Thu Jul 21 20:01:42 2011 From: kray at sonsothunder.com (Ken Ray) Date: Thu, 21 Jul 2011 19:01:42 -0500 Subject: Lion 'Gotcha' #1: Creating a Support Folder In-Reply-To: References: Message-ID: On Jul 21, 2011, at 6:52 PM, Ken Ray wrote: > put (specialFolderPath("asup") & "/Test") into tSupportFolder > create folder tSupportFolder Whoops! Just read Josh Mellicker's report on this... ignore this post! Ken Ray Sons of Thunder Software, Inc. Email: kray at sonsothunder.com Web Site: http://www.sonsothunder.com/ From ambassador at fourthworld.com Thu Jul 21 20:05:01 2011 From: ambassador at fourthworld.com (Richard Gaskin) Date: Thu, 21 Jul 2011 17:05:01 -0700 Subject: The Multitude of LC web dev tools In-Reply-To: References: Message-ID: <4E28BEAD.2070803@fourthworld.com> Pete Haworth wrote: > So, let's say I have an application in mind that a user will access from > his/her favorite web browser either on a desktop or perhaps an Android > device. On the server, I need to format pages of html to send to the user. > and access data in an SQL database. Some of the pages will be static and > others will change depending on database content. What are the recommended > tools to achieve this? PHP. :) But seriously, revBrowser is for client-side desktop apps, so that won't affect what you want to build for use in any browser. revServer is the core LiveCode engine. On-Rev is a hosting option which comes bundled with revServer (though you can run revServer on nearly any Linux host). revIgniter is a framework that uses revServer. So to build what you need with LiveCode you'd use revServer, and optionally revIgniter (a good choice for making short work of complex apps). -- Richard Gaskin Fourth World LiveCode training and consulting: http://www.fourthworld.com Webzine for LiveCode developers: http://www.LiveCodeJournal.com LiveCode Journal blog: http://LiveCodejournal.com/blog.irv From kray at sonsothunder.com Thu Jul 21 20:05:36 2011 From: kray at sonsothunder.com (Ken Ray) Date: Thu, 21 Jul 2011 19:05:36 -0500 Subject: Lion problem report and fix In-Reply-To: <4E28A2A8.4070908@fourthworld.com> References: <373168FE-1908-438D-B206-2BCDF7F99A26@dvcreators.net> <4E28A2A8.4070908@fourthworld.com> Message-ID: On Jul 21, 2011, at 5:05 PM, Richard Gaskin wrote: > Josh Mellicker wrote: > >> Just wanted to let everyone know, Lion permissions default to not allowing Livecode to create folders in "/Library/Application Support/" with the "create folder" command. > > I believe that's true with earlier versions as well if the current user is not admin. I tried doing the same thing as Josh as an admin user and got the same result (see my errant "Lion 'Gotcha' #1" post). It's definitely a Lion thing, although why /Application Support is more secured under Lion than under Snow Leopard is beyond me... Ken Ray Sons of Thunder Software, Inc. Email: kray at sonsothunder.com Web Site: http://www.sonsothunder.com/ From josh at dvcreators.net Thu Jul 21 20:24:17 2011 From: josh at dvcreators.net (Josh Mellicker) Date: Thu, 21 Jul 2011 17:24:17 -0700 Subject: Lion problem report and fix In-Reply-To: <4E28B6EA.3030709@hyperactivesw.com> References: <373168FE-1908-438D-B206-2BCDF7F99A26@dvcreators.net> <4E28A2A8.4070908@fourthworld.com> <4E28A57C.4020504@comcast.net> <4E28B6EA.3030709@hyperactivesw.com> Message-ID: <02B0CFCF-EF8A-4E2E-A661-B2BD34E5E80A@dvcreators.net> On Jul 21, 2011, at 4:31 PM, J. Landman Gay wrote: > On 7/21/11 5:17 PM, Marty Knapp wrote: >> Interestingly, the user's Library folder, and thus the Application >> Support folder is invisible on my Lion install. I can write to it OK, >> though (only tested as admin). > > I was just reading about this. Apparently you can hold down the Option key while choosing the Go menu in the Finder, and a Library item appears. This works on my machine From jacque at hyperactivesw.com Thu Jul 21 20:28:29 2011 From: jacque at hyperactivesw.com (J. Landman Gay) Date: Thu, 21 Jul 2011 19:28:29 -0500 Subject: Lion problem report and fix In-Reply-To: <253C2BBC-9D91-49EE-84A8-B2AA1D43F0B7@twft.com> References: <373168FE-1908-438D-B206-2BCDF7F99A26@dvcreators.net> <4E28A2A8.4070908@fourthworld.com> <4E28A57C.4020504@comcast.net> <4E28B6EA.3030709@hyperactivesw.com> <253C2BBC-9D91-49EE-84A8-B2AA1D43F0B7@twft.com> Message-ID: <4E28C42D.5080402@hyperactivesw.com> On 7/21/11 6:44 PM, Bob Sneidar wrote: > Hmmm... doesn't work like that for me. It still remembers the last > thing I had entered. Are you talking about Got to folder? The Go menu in the Finder: -- Jacqueline Landman Gay | jacque at hyperactivesw.com HyperActive Software | http://www.hyperactivesw.com From josh at dvcreators.net Thu Jul 21 20:30:36 2011 From: josh at dvcreators.net (Josh Mellicker) Date: Thu, 21 Jul 2011 17:30:36 -0700 Subject: Lion problem report and fix In-Reply-To: <4E28B16F.4060300@fourthworld.com> References: <4E28B16F.4060300@fourthworld.com> Message-ID: On Jul 21, 2011, at 4:08 PM, Richard Gaskin wrote: > Josh Mellicker wrote: > >> On Jul 21, 2011, at 3:14 PM, Mark Schonewille wrote: >> >>> Hi Josh, >>> >>> That's seems normal, since /Library isn't a user folder. Are you sure this is Lion-specific? >> >> Yes, at least with all the copies of Lion we've encountered so far. > > Is it any different in Snow Leopard? Yes: In Snow Leopard and previous: 1. the Livecode command "create folder" WORKS in /Library/Application Support/ (note this is not in the user folder) 2. files CAN be downloaded to folders within that folder (As Richard noted, this is when the current logged in user is the admin user, which for us is 99.9% of the time) In Lion: 1. the Livecode command "create folder" does NOT work in /Library/Application Support/ (but executing a shell command sudo mkdir does work) 2. files can NOT be downloaded to folders within that folder (but after a shell command changing permissions to 777, this fixes it) --- On the topic of "where do we put stuff (needed support files) in OS X", I remember Ken Ray had a great article on this? we decided on /Library/Application Support/, it has been working great until yesterday :-) We are now changing on OS X so that support files will be downloaded and housed inside the Mac application package in the Applications directory.) ---- [OT] By the way, many people write web apps in Javascript, HTML5, PHP, etc., and have nightmares about browser incompatibilities, while we are very happy to write desktop apps in Livecode that are far faster to develop and offer a much better user experience. (In a sane world 95% of developers would work in Livecode and only the crazy few would deal with writing web apps.) However, there's always the client objection of "we don't want to make the user install an app". (Whereas the same clients feel it's fine to require the user to install Silverlight, Adobe AIR, a new version of the Flash plugin, a Java app, or some other plugin to use their web app) So, Ken Ray wrote us some ultra-cool installers that make installing our app faster and easier than any of the browser plugin, effectively making a Livecode app just as easy for the user, if not easier than many web applications. If you want an awesome installation experience, Ken is your guy! So, think "Beyond the Browser" (R. Gaskin quote), the web is dead, we are now in the Age of the App! From cszasz at mac.com Thu Jul 21 20:33:20 2011 From: cszasz at mac.com (Charles Szasz) Date: Thu, 21 Jul 2011 20:33:20 -0400 Subject: Your Message-ID: <8652F22F-7CD1-493F-9146-55DC05FFB268@mac.com> Sean, Ken Ray indicated that you have an external for Rev 4.0. Do you have a web site where I can download your MacWindows external? Charles Szasz cszasz at mac.com From jacque at hyperactivesw.com Thu Jul 21 20:46:13 2011 From: jacque at hyperactivesw.com (J. Landman Gay) Date: Thu, 21 Jul 2011 19:46:13 -0500 Subject: OT: It's Android Jim, but not as we know it (what CPUs does Livecode compile to?) In-Reply-To: References: <4E289BD8.9080908@hyperactivesw.com> Message-ID: <4E28C855.5060902@hyperactivesw.com> On 7/21/11 6:02 PM, Bernard Devlin wrote: > If someone provides me with an identifiable free app from the Android > Marketplace which was coded in Livecode, then I can go back to the > store and see if I can find, install and run it. I'm not sure there are any yet. Someone asked on the forums and last I looked there were no responses. I'd be happy to compile the Hello World example for you to test. If you're game for that, drop me an email offlist and we can figure out logistics. -- Jacqueline Landman Gay | jacque at hyperactivesw.com HyperActive Software | http://www.hyperactivesw.com From shaosean at wehostmacs.com Thu Jul 21 20:50:58 2011 From: shaosean at wehostmacs.com (Shao Sean) Date: Thu, 21 Jul 2011 20:50:58 -0400 Subject: Lion problem report and fix Message-ID: <964ED011-A3AA-435F-AD07-D0170CE6D8D7@wehostmacs.com> > On the topic of "where do we put stuff (needed support files) in OS > X", I remember Ken Ray had a great article on this? we decided on / > Library/Application Support/, it has been working great until > yesterday :-) Best to just use the user's application support folder.. If I remember correctly, Apple will deny your Mac App from the store if you try to write to the system application support folder.. > We are now changing on OS X so that support files will be downloaded > and housed inside the Mac application package in the Applications > directory.) Always a bad idea, and this will cause your app to get denied from the app store for sure.. While it is true that the majority of users are running as an admin account on their own single-user machine and you can get around the limitations imposed by Apple by running sudo commands, think about a corporate/educational/shared environment where your attempt to run sudo will fail, the elevated privs through AppleScript will fail (and even Rev's new elevated privs feature will fail).. By coding according to the rules laid out you can save yourself headaches in the future.. From bobs at twft.com Thu Jul 21 20:52:51 2011 From: bobs at twft.com (Bob Sneidar) Date: Thu, 21 Jul 2011 17:52:51 -0700 Subject: Lion problem report and fix In-Reply-To: <4E28C42D.5080402@hyperactivesw.com> References: <373168FE-1908-438D-B206-2BCDF7F99A26@dvcreators.net> <4E28A2A8.4070908@fourthworld.com> <4E28A57C.4020504@comcast.net> <4E28B6EA.3030709@hyperactivesw.com> <253C2BBC-9D91-49EE-84A8-B2AA1D43F0B7@twft.com> <4E28C42D.5080402@hyperactivesw.com> Message-ID: OH! This is if you are running Lion. Not Snow Leopard. Bob On Jul 21, 2011, at 5:28 PM, J. Landman Gay wrote: > On 7/21/11 6:44 PM, Bob Sneidar wrote: >> Hmmm... doesn't work like that for me. It still remembers the last >> thing I had entered. Are you talking about Got to folder? > > The Go menu in the Finder: > > > > -- > Jacqueline Landman Gay | jacque at hyperactivesw.com > HyperActive Software | http://www.hyperactivesw.com > > _______________________________________________ > use-livecode mailing list > use-livecode at lists.runrev.com > Please visit this url to subscribe, unsubscribe and manage your subscription preferences: > http://lists.runrev.com/mailman/listinfo/use-livecode From mwieder at ahsoftware.net Thu Jul 21 20:52:56 2011 From: mwieder at ahsoftware.net (Mark Wieder) Date: Thu, 21 Jul 2011 17:52:56 -0700 Subject: on-Rev client software In-Reply-To: <4E284A41.6000708@hyperactivesw.com> References: <4E280ED4.90502@crcom.net> <4E284A41.6000708@hyperactivesw.com> Message-ID: <180987765.20110721175256@ahsoftware.net> Jacque- Thursday, July 21, 2011, 8:48:17 AM, you wrote: > I'd love to see cave paintings. :) Yesterday RR released a dozen or so > lessons on the new server product. If you go to the Lessons area of the > web site and do a search for "server" you'll probably turn up most of them. Right, but this is about on-rev, not about the LC server product. To my knowledge RR hasn't updated the server installation on the on-rev machines yet. And the transition path hasn't been made clear. And whether the ancient and dreaded on-rev client will work with the new server is an open question. -- -Mark Wieder mwieder at ahsoftware.net From bobs at twft.com Thu Jul 21 20:57:02 2011 From: bobs at twft.com (Bob Sneidar) Date: Thu, 21 Jul 2011 17:57:02 -0700 Subject: Lion problem report and fix In-Reply-To: References: <373168FE-1908-438D-B206-2BCDF7F99A26@dvcreators.net> <4E28A2A8.4070908@fourthworld.com> Message-ID: Just to weigh in here, anytime an application is installed, the OS asks for credentials. I assume this runs the installer with those admin credentials? If that is the case, then I think that anytime you want to put something in a System folder like Library and such, it would be a good idea to shell it using sudo. This assures you will always succeed. I have long thought it a bit irritating when software installs in the current user's profile without even asking, and then cannot be used in other profiles. Bob On Jul 21, 2011, at 5:05 PM, Ken Ray wrote: > > On Jul 21, 2011, at 5:05 PM, Richard Gaskin wrote: > >> Josh Mellicker wrote: >> >>> Just wanted to let everyone know, Lion permissions default to not allowing Livecode to create folders in "/Library/Application Support/" with the "create folder" command. >> >> I believe that's true with earlier versions as well if the current user is not admin. > > I tried doing the same thing as Josh as an admin user and got the same result (see my errant "Lion 'Gotcha' #1" post). It's definitely a Lion thing, although why /Application Support is more secured under Lion than under Snow Leopard is beyond me... > > Ken Ray > Sons of Thunder Software, Inc. > Email: kray at sonsothunder.com > Web Site: http://www.sonsothunder.com/ > > _______________________________________________ > use-livecode mailing list > use-livecode at lists.runrev.com > Please visit this url to subscribe, unsubscribe and manage your subscription preferences: > http://lists.runrev.com/mailman/listinfo/use-livecode From mwieder at ahsoftware.net Thu Jul 21 20:58:19 2011 From: mwieder at ahsoftware.net (Mark Wieder) Date: Thu, 21 Jul 2011 17:58:19 -0700 Subject: Detecting Different Versions of Livecode In-Reply-To: References: <1311288631267-3685390.post@n4.nabble.com> Message-ID: <3081311531.20110721175819@ahsoftware.net> Pete- Thursday, July 21, 2011, 4:27:34 PM, you wrote: > Thanks - revLicenseInfo is what I was looking for (not documented in the > dictionary) revLicenseType() changed with rev 4.5, so don't rely on that one. But can anyone tell me what revLicenseInfo returns for a Personal license? Not having one I can't investigate this myself, and my queries to the rev team have gone unanswered. -- -Mark Wieder mwieder at ahsoftware.net From lists.pete at haworths.org Thu Jul 21 21:03:16 2011 From: lists.pete at haworths.org (Pete Haworth) Date: Thu, 21 Jul 2011 18:03:16 -0700 Subject: Detecting Different Versions of Livecode In-Reply-To: <3081311531.20110721175819@ahsoftware.net> References: <1311288631267-3685390.post@n4.nabble.com> <3081311531.20110721175819@ahsoftware.net> Message-ID: Yep, saw in the dictionary it would be deprecated. Here's the output from revLicenseInfo on my computer - I have a personal desktop license with Android added in: Peter Haworth Personal 1 Android Local Maybe you could tell me what you get for whatever license you have? Pete On Thu, Jul 21, 2011 at 5:58 PM, Mark Wieder wrote: > Pete- > > Thursday, July 21, 2011, 4:27:34 PM, you wrote: > > > Thanks - revLicenseInfo is what I was looking for (not documented in the > > dictionary) > > revLicenseType() changed with rev 4.5, so don't rely on that one. > > But can anyone tell me what revLicenseInfo returns for a Personal > license? Not having one I can't investigate this myself, and my > queries to the rev team have gone unanswered. > > -- > -Mark Wieder > mwieder at ahsoftware.net > > > _______________________________________________ > use-livecode mailing list > use-livecode at lists.runrev.com > Please visit this url to subscribe, unsubscribe and manage your > subscription preferences: > http://lists.runrev.com/mailman/listinfo/use-livecode > From mwieder at ahsoftware.net Thu Jul 21 21:03:20 2011 From: mwieder at ahsoftware.net (Mark Wieder) Date: Thu, 21 Jul 2011 18:03:20 -0700 Subject: [ANN] New plugin AAG|Workspaces In-Reply-To: References: Message-ID: <4081611843.20110721180320@ahsoftware.net> Pete- Thursday, July 21, 2011, 11:48:27 AM, you wrote: > Just downloaded GLX2 from bitbucket and I don't see a split screen > capability, at least no obvious way of doing it. Used to be in there. Lemme poke around a bit and see if it's still in the code. I can't remember how to invoke it either. -- -Mark Wieder mwieder at ahsoftware.net From lists.pete at haworths.org Thu Jul 21 21:07:04 2011 From: lists.pete at haworths.org (Pete Haworth) Date: Thu, 21 Jul 2011 18:07:04 -0700 Subject: Lion problem report and fix In-Reply-To: References: <373168FE-1908-438D-B206-2BCDF7F99A26@dvcreators.net> <4E28A2A8.4070908@fourthworld.com> Message-ID: I guess the question here is where on earth are you supposed to put an application's files if you want them to be available to all users? It seems like /Library/Application Support makes sense but apparently Apple have decided otherwise. I hope they have published a guideline somewhere that explains how to handle this situation. Pete On Thu, Jul 21, 2011 at 5:57 PM, Bob Sneidar wrote: > Just to weigh in here, anytime an application is installed, the OS asks for > credentials. I assume this runs the installer with those admin credentials? > If that is the case, then I think that anytime you want to put something in > a System folder like Library and such, it would be a good idea to shell it > using sudo. This assures you will always succeed. > > I have long thought it a bit irritating when software installs in the > current user's profile without even asking, and then cannot be used in other > profiles. > > Bob > > > On Jul 21, 2011, at 5:05 PM, Ken Ray wrote: > > > > > On Jul 21, 2011, at 5:05 PM, Richard Gaskin wrote: > > > >> Josh Mellicker wrote: > >> > >>> Just wanted to let everyone know, Lion permissions default to not > allowing Livecode to create folders in "/Library/Application Support/" with > the "create folder" command. > >> > >> I believe that's true with earlier versions as well if the current user > is not admin. > > > > I tried doing the same thing as Josh as an admin user and got the same > result (see my errant "Lion 'Gotcha' #1" post). It's definitely a Lion > thing, although why /Application Support is more secured under Lion than > under Snow Leopard is beyond me... > > > > Ken Ray > > Sons of Thunder Software, Inc. > > Email: kray at sonsothunder.com > > Web Site: http://www.sonsothunder.com/ > > > > _______________________________________________ > > use-livecode mailing list > > use-livecode at lists.runrev.com > > Please visit this url to subscribe, unsubscribe and manage your > subscription preferences: > > http://lists.runrev.com/mailman/listinfo/use-livecode > > > _______________________________________________ > use-livecode mailing list > use-livecode at lists.runrev.com > Please visit this url to subscribe, unsubscribe and manage your > subscription preferences: > http://lists.runrev.com/mailman/listinfo/use-livecode > From jacque at hyperactivesw.com Thu Jul 21 21:14:26 2011 From: jacque at hyperactivesw.com (J. Landman Gay) Date: Thu, 21 Jul 2011 20:14:26 -0500 Subject: on-Rev client software In-Reply-To: <180987765.20110721175256@ahsoftware.net> References: <4E280ED4.90502@crcom.net> <4E284A41.6000708@hyperactivesw.com> <180987765.20110721175256@ahsoftware.net> Message-ID: <4E28CEF2.5010102@hyperactivesw.com> On 7/21/11 7:52 PM, Mark Wieder wrote: > Jacque- > > Thursday, July 21, 2011, 8:48:17 AM, you wrote: > >> I'd love to see cave paintings. :) Yesterday RR released a dozen or so >> lessons on the new server product. If you go to the Lessons area of the >> web site and do a search for "server" you'll probably turn up most of them. > > Right, but this is about on-rev, not about the LC server product. Oh. You mean, just like the subject says... -- Jacqueline Landman Gay | jacque at hyperactivesw.com HyperActive Software | http://www.hyperactivesw.com From roger.e.eller at sealedair.com Thu Jul 21 21:39:39 2011 From: roger.e.eller at sealedair.com (Roger Eller) Date: Thu, 21 Jul 2011 21:39:39 -0400 Subject: OT: It's Android Jim, but not as we know it (what CPUs does Livecode compile to?) In-Reply-To: <4E28C855.5060902@hyperactivesw.com> References: <4E289BD8.9080908@hyperactivesw.com> <4E28C855.5060902@hyperactivesw.com> Message-ID: On Thu, Jul 21, 2011 at 8:46 PM, J. Landman Gay wrote: > On 7/21/11 6:02 PM, Bernard Devlin wrote: > > If someone provides me with an identifiable free app from the Android >> Marketplace which was coded in Livecode, then I can go back to the >> store and see if I can find, install and run it. >> > > I'm not sure there are any yet. Someone asked on the forums and last I > looked there were no responses. I'd be happy to compile the Hello World > example for you to test. If you're game for that, drop me an email offlist > and we can figure out logistics. > > -- > Jacqueline Landman Gay | jacque at hyperactivesw.com > HyperActive Software | http://www.hyperactivesw.com > > My Android tablet has the nVidia Tegra2 dual-core CPU (which is ARM based), and my LiveCode apps are working great, as are many apps I have installed from the Android Market. Earlier, Bernard wrote: > I guess I'm really hoping for too much to think that Livecode could run on a Tegra > (I know the Linux IDE is only compiled for intel). I seriously doubt that RunRev intends for the IDE to run on mobile, if that is your intent for getting the Asus Transformer. As an Android testing device, it should be fine. ?Roger From josh at dvcreators.net Thu Jul 21 21:40:56 2011 From: josh at dvcreators.net (Josh Mellicker) Date: Thu, 21 Jul 2011 18:40:56 -0700 Subject: Lion problem report and fix In-Reply-To: <964ED011-A3AA-435F-AD07-D0170CE6D8D7@wehostmacs.com> References: <964ED011-A3AA-435F-AD07-D0170CE6D8D7@wehostmacs.com> Message-ID: <8C150299-D1A5-4D07-82E9-EBBCBB40FA69@dvcreators.net> On Jul 21, 2011, at 5:50 PM, Shao Sean wrote: >> On the topic of "where do we put stuff (needed support files) in OS X", I remember Ken Ray had a great article on this? we decided on /Library/Application Support/, it has been working great until yesterday :-) > > Best to just use the user's application support folder.. If I remember correctly, Apple will deny your Mac App from the store if you try to write to the system application support folder.. > >> We are now changing on OS X so that support files will be downloaded and housed inside the Mac application package in the Applications directory.) > > Always a bad idea, and this will cause your app to get denied from the app store for sure.. > > While it is true that the majority of users are running as an admin account on their own single-user machine and you can get around the limitations imposed by Apple by running sudo commands, think about a corporate/educational/shared environment where your attempt to run sudo will fail, the elevated privs through AppleScript will fail (and even Rev's new elevated privs feature will fail).. By coding according to the rules laid out you can save yourself headaches in the future.. Hmmm?. I stand corrected. Now, we are getting an error message when trying to execute the sudo command. "sudo: no tty present and no askpass program specified" So, we are going to take Shao's sagely advice and try ~/Library/Application support. From bobs at twft.com Thu Jul 21 21:50:15 2011 From: bobs at twft.com (Bob Sneidar) Date: Thu, 21 Jul 2011 18:50:15 -0700 Subject: Lion problem report and fix In-Reply-To: <964ED011-A3AA-435F-AD07-D0170CE6D8D7@wehostmacs.com> References: <964ED011-A3AA-435F-AD07-D0170CE6D8D7@wehostmacs.com> Message-ID: <79B3BC34-D2C0-4B39-A420-F524038BBFA5@twft.com> With respect to all posters, this is silly. If the answer to "How do we make applications available to all users" is, "You can't do that!", then something is broke and it needs fixing. Nothing says "cheesy app from inexperienced programmers made with sub-par development tools" like apps that have no universal install capability. I am not saying anyone qualifies, but that is what it will seem like to anyone you try to sell your applications to. My 2? Bob On Jul 21, 2011, at 5:50 PM, Shao Sean wrote: > Always a bad idea, and this will cause your app to get denied from the app store for sure.. > > While it is true that the majority of users are running as an admin account on their own single-user machine and you can get around the limitations imposed by Apple by running sudo commands, think about a corporate/educational/shared environment where your attempt to run sudo will fail, the elevated privs through AppleScript will fail (and even Rev's new elevated privs feature will fail).. By coding according to the rules laid out you can save yourself headaches in the future.. From livecode.list at gmail.com Thu Jul 21 21:55:58 2011 From: livecode.list at gmail.com (Chip Thomas) Date: Thu, 21 Jul 2011 18:55:58 -0700 Subject: Moving a folder Message-ID: What's the best way to move a folder (with all contents) on a user's system? rename folder tOldFolder to tNewFolder revMoveFolder tOldFolder, tNewFolder or using a shell command Concerned about potential permissions issue that would prevent a proper move. What's the best way around that? From bobs at twft.com Thu Jul 21 22:08:19 2011 From: bobs at twft.com (Bob Sneidar) Date: Thu, 21 Jul 2011 19:08:19 -0700 Subject: Moving a folder In-Reply-To: References: Message-ID: Never done it, but I imagine you could shell out to check the permissions of where you are moving to. I don't know if there is a shell command to check effective permissions of the current user. If there are, do that. Barring that, a safer way would be to copy the files, check to make sure there were no errors, if there are bail, if there are not then delete the original. Bob On Jul 21, 2011, at 6:55 PM, Chip Thomas wrote: > What's the best way to move a folder (with all contents) on a user's system? > > rename folder tOldFolder to tNewFolder > > revMoveFolder tOldFolder, tNewFolder > > or using a shell command > > > Concerned about potential permissions issue that would prevent a proper > move. What's the best way around that? > _______________________________________________ > use-livecode mailing list > use-livecode at lists.runrev.com > Please visit this url to subscribe, unsubscribe and manage your subscription preferences: > http://lists.runrev.com/mailman/listinfo/use-livecode From roger.e.eller at sealedair.com Thu Jul 21 22:15:56 2011 From: roger.e.eller at sealedair.com (Roger Eller) Date: Thu, 21 Jul 2011 22:15:56 -0400 Subject: Moving a folder In-Reply-To: References: Message-ID: On Thu, Jul 21, 2011 at 10:08 PM, Bob Sneidar wrote: > Never done it, but I imagine you could shell out to check the permissions > of where you are moving to. I don't know if there is a shell command to > check effective permissions of the current user. If there are, do that. > Barring that, a safer way would be to copy the files, check to make sure > there were no errors, if there are bail, if there are not then delete the > original. > > Bob > > > On Jul 21, 2011, at 6:55 PM, Chip Thomas wrote: > > > What's the best way to move a folder (with all contents) on a user's > system? > > > > rename folder tOldFolder to tNewFolder > > > > revMoveFolder tOldFolder, tNewFolder > > > > or using a shell command > > > > > > Concerned about potential permissions issue that would prevent a proper > > move. What's the best way around that? > > _______________________________________________ > > use-livecode mailing list > > use-livecode at lists.runrev.com > > Please visit this url to subscribe, unsubscribe and manage your > subscription preferences: > > http://lists.runrev.com/mailman/listinfo/use-livecode > My preference is to not use shell commands unless you really must. By using LC native as much as possible, there is less work when you meed it to work on more than a single platform. If memory serves, the rename command was the better of the two, as one of them would only work with an empty folder, and the other would move a folder and its contents. ?Roger From terryhass at tdheng.com Thu Jul 21 22:19:43 2011 From: terryhass at tdheng.com (TERRY HASS) Date: Thu, 21 Jul 2011 21:19:43 -0500 Subject: Lion and Livecode Message-ID: <296B6549-B3BA-4976-B407-1DD33364D2E0@tdheng.com> I upgraded to Lion yesterday. Now LiveCode is having problems. I can no longer use the iPhone simulator. Lion will not allow LiveCode to "select" the Developer directory. Any suggestions from the experts? Thanks for your help... Terry From bvlahos at mac.com Thu Jul 21 22:29:55 2011 From: bvlahos at mac.com (Bill Vlahos) Date: Thu, 21 Jul 2011 19:29:55 -0700 Subject: Setting a dirty flag for a file In-Reply-To: <68284CA1-2238-4093-8F45-2E8E9101F352@sonsothunder.com> References: <4E287BF0.50308@fourthworld.com> <42262BB8-A865-44F6-8A43-6672C53999AE@byu.edu> <68284CA1-2238-4093-8F45-2E8E9101F352@sonsothunder.com> Message-ID: <5B07EBE3-9E12-4510-8BE5-A6D2CDA0F979@mac.com> I use it in InfoWallet and It is terrific but it isn't available any longer. Bill Vlahos Sent from my iPhone On Jul 21, 2011, at 5:00 PM, Ken Ray wrote: >> But don't forget that the modifiedMark property only showed up in LC 4.6, and Charles said he's using v. 4.0. > > True, but FYI Sean Shao has an external that works with 4.0 that does the same thing. I've been using it in Stykz for quite a while. You can contact her if you need it... > > Ken Ray > Sons of Thunder Software, Inc. > Email: kray at sonsothunder.com > Web Site: http://www.sonsothunder.com/ > > _______________________________________________ > use-livecode mailing list > use-livecode at lists.runrev.com > Please visit this url to subscribe, unsubscribe and manage your subscription preferences: > http://lists.runrev.com/mailman/listinfo/use-livecode From scott at elementarysoftware.com Thu Jul 21 23:28:27 2011 From: scott at elementarysoftware.com (Scott Morrow) Date: Thu, 21 Jul 2011 20:28:27 -0700 Subject: Lion problem report and fix In-Reply-To: <8C150299-D1A5-4D07-82E9-EBBCBB40FA69@dvcreators.net> References: <964ED011-A3AA-435F-AD07-D0170CE6D8D7@wehostmacs.com> <8C150299-D1A5-4D07-82E9-EBBCBB40FA69@dvcreators.net> Message-ID: To provide writable "application" files, one method I have used is to copy these writable files (from the executable) into the user's ~/Library/Application Support/ folder and then makes a note of this in the /Users/Shared folder so that an admin account doing an uninstall can look one place to see all the different user's Application Support folder(s) that have files needing deletion. Having the application do this saves the installer from needing to know about multiple accounts. And if more accounts are added that need to use the application the executable just creates a new set of files. Scott Morrow On Jul 21, 2011, at 6:40 PM, Josh Mellicker wrote: > > On Jul 21, 2011, at 5:50 PM, Shao Sean wrote: > >>> On the topic of "where do we put stuff (needed support files) in OS X", I remember Ken Ray had a great article on this? we decided on /Library/Application Support/, it has been working great until yesterday :-) >> >> Best to just use the user's application support folder.. If I remember correctly, Apple will deny your Mac App from the store if you try to write to the system application support folder.. >> >>> We are now changing on OS X so that support files will be downloaded and housed inside the Mac application package in the Applications directory.) >> >> Always a bad idea, and this will cause your app to get denied from the app store for sure.. >> >> While it is true that the majority of users are running as an admin account on their own single-user machine and you can get around the limitations imposed by Apple by running sudo commands, think about a corporate/educational/shared environment where your attempt to run sudo will fail, the elevated privs through AppleScript will fail (and even Rev's new elevated privs feature will fail).. By coding according to the rules laid out you can save yourself headaches in the future.. > > > Hmmm?. I stand corrected. Now, we are getting an error message when trying to execute the sudo command. > > "sudo: no tty present and no askpass program specified" > > So, we are going to take Shao's sagely advice and try ~/Library/Application support. > > > _______________________________________________ > use-livecode mailing list > use-livecode at lists.runrev.com > Please visit this url to subscribe, unsubscribe and manage your subscription preferences: > http://lists.runrev.com/mailman/listinfo/use-livecode From kray at sonsothunder.com Fri Jul 22 00:07:41 2011 From: kray at sonsothunder.com (Ken Ray) Date: Thu, 21 Jul 2011 23:07:41 -0500 Subject: Lion problem report and fix In-Reply-To: References: <964ED011-A3AA-435F-AD07-D0170CE6D8D7@wehostmacs.com> <8C150299-D1A5-4D07-82E9-EBBCBB40FA69@dvcreators.net> Message-ID: <55C7200B-685B-4426-A5BF-3046429F04B7@sonsothunder.com> Personally I think we may be jumping the gun on assuming Apple's intentions based on enhanced security on the /Library/Application Support folder. Yes, I got burned as well, but I *still* plan on writing "all users" support files in the /Library/Application Support folder, even if I have to go the extra mile to make it happen. Ken Ray Sons of Thunder Software, Inc. Email: kray at sonsothunder.com Web Site: http://www.sonsothunder.com/ On Jul 21, 2011, at 10:28 PM, Scott Morrow wrote: > To provide writable "application" files, one method I have used is to copy these writable files (from the executable) into the user's ~/Library/Application Support/ folder and then makes a note of this in the /Users/Shared folder so that an admin account doing an uninstall can look one place to see all the different user's Application Support folder(s) that have files needing deletion. Having the application do this saves the installer from needing to know about multiple accounts. And if more accounts are added that need to use the application the executable just creates a new set of files. > > Scott Morrow > > On Jul 21, 2011, at 6:40 PM, Josh Mellicker wrote: > >> >> On Jul 21, 2011, at 5:50 PM, Shao Sean wrote: >> >>>> On the topic of "where do we put stuff (needed support files) in OS X", I remember Ken Ray had a great article on this? we decided on /Library/Application Support/, it has been working great until yesterday :-) >>> >>> Best to just use the user's application support folder.. If I remember correctly, Apple will deny your Mac App from the store if you try to write to the system application support folder.. >>> >>>> We are now changing on OS X so that support files will be downloaded and housed inside the Mac application package in the Applications directory.) >>> >>> Always a bad idea, and this will cause your app to get denied from the app store for sure.. >>> >>> While it is true that the majority of users are running as an admin account on their own single-user machine and you can get around the limitations imposed by Apple by running sudo commands, think about a corporate/educational/shared environment where your attempt to run sudo will fail, the elevated privs through AppleScript will fail (and even Rev's new elevated privs feature will fail).. By coding according to the rules laid out you can save yourself headaches in the future.. >> >> >> Hmmm?. I stand corrected. Now, we are getting an error message when trying to execute the sudo command. >> >> "sudo: no tty present and no askpass program specified" >> >> So, we are going to take Shao's sagely advice and try ~/Library/Application support. >> >> >> _______________________________________________ >> use-livecode mailing list >> use-livecode at lists.runrev.com >> Please visit this url to subscribe, unsubscribe and manage your subscription preferences: >> http://lists.runrev.com/mailman/listinfo/use-livecode > > > _______________________________________________ > use-livecode mailing list > use-livecode at lists.runrev.com > Please visit this url to subscribe, unsubscribe and manage your subscription preferences: > http://lists.runrev.com/mailman/listinfo/use-livecode From shaosean at wehostmacs.com Fri Jul 22 01:13:58 2011 From: shaosean at wehostmacs.com (Shao Sean) Date: Fri, 22 Jul 2011 01:13:58 -0400 Subject: Lion problem report and fix Message-ID: Here is the info from Apple in regards to the changes talked about in this thread.. Folder Permissions and Ownership A number of folders in the System and Local file system domains now have different ownership and permissions. Specifically: ? Many folders in the System domain that were previously owned by the admin group are now owned by the wheelgroup. ? Permissions for the root directory (/) are now mode 755 (writable only by root) instead of mode 775 (writable by the admin group). ? Permissions for /Applications/Utilities are now mode 755 (writable only by root) instead of mode 775 (writable by the admin group). ? Permissions for /Library are now mode 755 (writable only by root) instead of mode 775 (writable by the admin group), no longer sticky. All subdirectories within /Library now have mode 755 (writable only by root) permissions instead of mode 775 (writable by the admin group) except: ? /Library/Caches ? /Library/Fonts ? /Library/Java ? /Library/QuickTimeStreaming ? /Library/Receipts ? /Library/Tomcat The subdirectories listed above have the same permissions as in previous versions of Mac OS X (usually mode 775, sometimes with the sticky bit set). ? Permissions for /Network/Applications and /Network/Library are now mode 555 (unwritable even by root) instead of mode 755 (writable only by root). ? Permissions for /var/log/DiagnosticMessages are slightly more lax, with mode 770 (writable by the admin group, unreadable by non-admin users) instead of mode 750 (writable only by root, unreadable by non- admin users). From shaosean at wehostmacs.com Fri Jul 22 01:22:30 2011 From: shaosean at wehostmacs.com (Shao Sean) Date: Fri, 22 Jul 2011 01:22:30 -0400 Subject: Lion problem report and fix Message-ID: <2D201346-CA3E-4B0D-8DA4-0326ED92254E@wehostmacs.com> For those that are submitting Rev apps to the Mac App store, this might be useful to you (and could also be good idea to follow for non- Mac App store downloads as well).. File-System Usage Requirements for the Mac App Store To promote a more consistent user experience, applications submitted to the Mac App Store must follow certain rules about where they write files. Users can be confused when applications cause unexpected side effects on the file system (for example, storing databases in the user?s Documents folder, storing files in the user?s Library folder that are not recognizably associated with your application, storing user data in the user?s Library folder, and so on). Your application must adhere to the following requirements: ? You may use Apple frameworks such as User Defaults, Calendar Store, and Address Book that implicitly write to files in specific locations, including locations is not allowed to access directly. ? Your application may write to temporary paths that you acquire using the appropriate Apple programming interfaces. ? Your application may write to the following directories: ? ~/Library/Application Support/ ? ~/Library/ ? ~/Library/Caches/ where is your application's bundle identifier, its name, or your company?s name. This must exactly match what is in iTunes Connect for the application. Always use Apple programming interfaces such as the URLsForDirectory:inDomains: function to locate these paths rather than hardcoding them. For more information, see File System Programming Guide. ? If your application manages libraries of pictures, music, or movies, the application may also write to the following directories: ? ~/Pictures/ ? ~/Music/ ? ~/Movies/ ? If the user explicitly chooses to save data in an alternate location (using a Save As dialog), your application may write to the chosen location. From mwieder at ahsoftware.net Fri Jul 22 01:26:49 2011 From: mwieder at ahsoftware.net (Mark Wieder) Date: Thu, 21 Jul 2011 22:26:49 -0700 Subject: Detecting Different Versions of Livecode In-Reply-To: References: <1311288631267-3685390.post@n4.nabble.com> <3081311531.20110721175819@ahsoftware.net> Message-ID: <8797421046.20110721222649@ahsoftware.net> Pete- Thursday, July 21, 2011, 6:03:16 PM, you wrote: > Maybe you could tell me what you get for whatever license you have? Mark WIEDER Commercial 1 Windows,Mac OS X,Linux,Web,iOS,Android,Windows Mobile,Linux Mobile Local Mind you, I have no idea what Linux Mobile is, but it sounds good to me. And is that the actual spacing of lines in your version? I had thought that maybe line 3 could differentiate between the Commercial and Person versions, but from your post is looks like "Personal" is line 4. That's annoying if it's true. I have a blank line after my name and then another before the word "Local". And I have no idea why my last name is in all caps. Maybe I'm yelling at myself. Maybe it's a secret handshake the rev team uses to identify troublemakers. -- -Mark Wieder mwieder at ahsoftware.net From shaosean at wehostmacs.com Fri Jul 22 01:29:48 2011 From: shaosean at wehostmacs.com (Shao Sean) Date: Fri, 22 Jul 2011 01:29:48 -0400 Subject: Carbon framework on Lion Message-ID: <67FF6496-12EF-49B1-976F-D81FAECD4F1E@wehostmacs.com> More info from Apple about the changes to Carbon in Lion (Rev is still a Carbon based application) Carbon Framework Release Notes This document describes new features and issues with the Carbon framework in Mac OS X v10.7. Transparent Application Lifecycle Persistent State All Carbon applications support Persistent State restoration across logout/shutdown and login automatically or when the user selects "Quit and Keep Windows" menu item from the application menu. When this menu item is chosen, the OS records a list of all open disk-based documents to re-open at next launch. In addition to the standard state like window size, position and toolbar visibility, your application can preserve custom state such as current selection. See SetWindowProperty and ChangeWindowPropertyAttributes, the attribute kWindowPropertyPersistent, andkEventWindowRestoredAfterRelaunch. Sudden Termination When Sudden termination is enabled (either using the NSSupportsSuddenTermination key in the application?s Info.plist file or the NSProcessInfomethod enableSuddenTermination), the app can be terminated immediately, without sending the kAEQuitApplication AppleEvent. Sudden termination is automatically disabled when a window?s close box shows the modified state indicator (via the SetWindowModified API). Automatic Termination An application may adopt the automatic termination capability by adding the NSSupportsAutomaticTermination key to its Info.plist. The application?s File menu automatically gains a "Close All" menu item which uses the new kHICommandCloseAll and dispatches a kEventWindowCloseAll event. Your application should implement kEventWindowCloseAll to prevent multiple sheets for unsaved changes and allow the user to review unsaved changes in each document singly. With no visible documents and when deactivated, such applications may be converted to UIElements, disappear from the Dock or application switcher, and be terminated to recover memory. If termination should not occur (due to background task), the application may useNSProcessInfo methods disableAutomaticTermination and enableAutomaticTermination. Scroll Bars Carbon now uses the new overlay scrollbar appearance for the standard scrollbar control and for scrollbars drawn with the HITheme API. Carbon does not support the automatic scrollbar hiding behavior provided by AppKit; for Carbon applications, the scrollbars will always remain visible. Known Issues There are a few issues with Carbon in the current seed. There is no need to file bugs on these specific problems: ? Window close/minimize/zoom buttons are positioned too low in the window title bar. ? Resizing a window from any edge or corner is only available for compositing windows. From mwieder at ahsoftware.net Fri Jul 22 01:30:50 2011 From: mwieder at ahsoftware.net (Mark Wieder) Date: Thu, 21 Jul 2011 22:30:50 -0700 Subject: Lion problem report and fix In-Reply-To: <2D201346-CA3E-4B0D-8DA4-0326ED92254E@wehostmacs.com> References: <2D201346-CA3E-4B0D-8DA4-0326ED92254E@wehostmacs.com> Message-ID: <4097662546.20110721223050@ahsoftware.net> Sean- Thursday, July 21, 2011, 10:22:30 PM, you wrote: > where is your application's bundle identifier, its > name, or your company?s name. This must exactly match what is in > iTunes Connect for the application. Since when did iTunes get to be the arbiter of all things OSX? I feel like I've missed a turn and fallen in to some alternate universe... -- -Mark Wieder mwieder at ahsoftware.net From jacque at hyperactivesw.com Fri Jul 22 01:32:55 2011 From: jacque at hyperactivesw.com (J. Landman Gay) Date: Fri, 22 Jul 2011 00:32:55 -0500 Subject: Lion problem report and fix In-Reply-To: <55C7200B-685B-4426-A5BF-3046429F04B7@sonsothunder.com> References: <964ED011-A3AA-435F-AD07-D0170CE6D8D7@wehostmacs.com> <8C150299-D1A5-4D07-82E9-EBBCBB40FA69@dvcreators.net> <55C7200B-685B-4426-A5BF-3046429F04B7@sonsothunder.com> Message-ID: <4E290B87.1040407@hyperactivesw.com> On 7/21/11 11:07 PM, Ken Ray wrote: > Personally I think we may be jumping the gun on assuming Apple's > intentions based on enhanced security on the /Library/Application > Support folder. I've been working my way through the Ars Technica article that was linked here. The Sandboxing section describes how Lion apps must save files: *** A sandboxed application must now include a list of "entitlements" describing exactly what resources it needs in order to do its job. Lion supports about 30 different entitlements which range from basic things like the ability to create a network connection or to listen for incoming network connections (two separate entitlements) to sophisticated tasks like capturing video or still images from a built-in camera. It might seem like any nontrivial document-based Mac application will, at the very least, need to declare an entitlement that will allow it to both read from and write to any directory owned by the current user. After all, how else would the user open and save documents? And if that's the case, wouldn't that entirely defeat the purpose of sandboxing? Apple has chosen to solve this problem by providing heightened permissions to a particular class of actions: those explicitly initiated by the user. Lion includes a trusted daemon process called Powerbox whose job is to present and control open/ save dialog boxes on behalf of sandboxed applications. After the user selects a file or directory into which a file should be saved, Powerbox pokes a hole in the application sandbox that allows it to perform the specific action. A similar mechanism is used to allow access to recently opened files in the "Open Recent" menu, to restore previously open documents when an application is relaunched, to handle drag and drop, and so on. The goal is to prevent applications from having to request entitlements that allow it to read and write arbitrary files. *** It does only talk about saving to the user folder, but maybe the app can request specific permissions/authority from the OS to write to protected locations too. If so, and if standalone builder can be updated to include that request, things could work as before. -- Jacqueline Landman Gay | jacque at hyperactivesw.com HyperActive Software | http://www.hyperactivesw.com From bdrunrev at gmail.com Fri Jul 22 01:43:15 2011 From: bdrunrev at gmail.com (Bernard Devlin) Date: Fri, 22 Jul 2011 06:43:15 +0100 Subject: OT: It's Android Jim, but not as we know it (what CPUs does Livecode compile to?) In-Reply-To: References: <4E289BD8.9080908@hyperactivesw.com> <4E28C855.5060902@hyperactivesw.com> Message-ID: Thanks Roger, that sounds like the first hurdle will be overcome. As a machine that I might carry with me to run admin/monitoring apps written in Livecode, the Transformer is looking better. I've never tried to see how much could be done within the scriptLimits but provided I don't look on it as being a main development machine, then it may be workable ( I have no need to create standalones on it). I too have no real expectation that Runrev would provide an IDE that runs on Android/ARM - not unless such Android tablets/netbooks sweep the world :) Until this week I had pretty much no interest in developing for iOS/Android. Do you have a Livecode app in the Android Marketplace I could use to test on it? If not, I'll take Jacques up on her kind offer offlist. Also, can one install a Livecode app on a machine without going through the hassle of listing it in the Marketplace? Can they be installed e.g. by downloading them from a URL? Looks like Runrev are going to be getting me to upgrade my license once again. Bernard On Fri, Jul 22, 2011 at 2:39 AM, Roger Eller wrote: > My Android tablet has the nVidia Tegra2 dual-core CPU (which is ARM based), > and my LiveCode apps are working great, as are many apps I have installed > from the Android Market. > > Earlier, Bernard wrote: > ? > I guess I'm really hoping for too much to think that Livecode could run > on a Tegra > ? > (I know the Linux IDE is only compiled for intel). > > I seriously doubt that RunRev intends for the IDE to run on mobile, if that > is your intent for getting the Asus Transformer. ?As an Android testing > device, it should be fine. From lists.pete at haworths.org Fri Jul 22 02:11:30 2011 From: lists.pete at haworths.org (Pete Haworth) Date: Thu, 21 Jul 2011 23:11:30 -0700 Subject: Detecting Different Versions of Livecode In-Reply-To: <8797421046.20110721222649@ahsoftware.net> References: <1311288631267-3685390.post@n4.nabble.com> <3081311531.20110721175819@ahsoftware.net> <8797421046.20110721222649@ahsoftware.net> Message-ID: Hi Mark, It's the same spacing - name on line 1 then a blank line then Personal on line 3. I guess your name is in uppercase to differentiate you from the other Mark "lowercase" Wieder (set the casesensitive to true). Pete On Thu, Jul 21, 2011 at 10:26 PM, Mark Wieder wrote: > Pete- > > Thursday, July 21, 2011, 6:03:16 PM, you wrote: > > > Maybe you could tell me what you get for whatever license you have? > > Mark WIEDER > > Commercial > 1 > Windows,Mac OS X,Linux,Web,iOS,Android,Windows Mobile,Linux Mobile > > Local > > Mind you, I have no idea what Linux Mobile is, but it sounds good to > me. And is that the actual spacing of lines in your version? I had > thought that maybe line 3 could differentiate between the Commercial > and Person versions, but from your post is looks like "Personal" is > line 4. That's annoying if it's true. I have a blank line after my > name and then another before the word "Local". > > And I have no idea why my last name is in all caps. Maybe I'm yelling > at myself. Maybe it's a secret handshake the rev team uses to identify > troublemakers. > > -- > -Mark Wieder > mwieder at ahsoftware.net > > > _______________________________________________ > use-livecode mailing list > use-livecode at lists.runrev.com > Please visit this url to subscribe, unsubscribe and manage your > subscription preferences: > http://lists.runrev.com/mailman/listinfo/use-livecode > From mwieder at ahsoftware.net Fri Jul 22 02:38:55 2011 From: mwieder at ahsoftware.net (Mark Wieder) Date: Thu, 21 Jul 2011 23:38:55 -0700 Subject: Detecting Different Versions of Livecode In-Reply-To: References: <1311288631267-3685390.post@n4.nabble.com> <3081311531.20110721175819@ahsoftware.net> <8797421046.20110721222649@ahsoftware.net> Message-ID: <197101747093.20110721233855@ahsoftware.net> Pete- Thursday, July 21, 2011, 11:11:30 PM, you wrote: > I guess your name is in uppercase to differentiate you from the other Mark > "lowercase" Wieder (set the casesensitive to true). Doh! Of course! That guy's always making problems for me. -- -Mark Wieder mwieder at ahsoftware.net From revlist at azurevision.co.uk Fri Jul 22 04:08:13 2011 From: revlist at azurevision.co.uk (Ian Wood) Date: Fri, 22 Jul 2011 09:08:13 +0100 Subject: Lion problem report and fix In-Reply-To: <4097662546.20110721223050@ahsoftware.net> References: <2D201346-CA3E-4B0D-8DA4-0326ED92254E@wehostmacs.com> <4097662546.20110721223050@ahsoftware.net> Message-ID: <71FA3788-7859-4548-866F-FD3D67E56953@azurevision.co.uk> On 22 Jul 2011, at 06:30, Mark Wieder wrote: > Sean- > > Thursday, July 21, 2011, 10:22:30 PM, you wrote: > >> where is your application's bundle identifier, its >> name, or your company?s name. This must exactly match what is in >> iTunes Connect for the application. > > Since when did iTunes get to be the arbiter of all things OSX? I feel > like I've missed a turn and fallen in to some alternate universe... "For those that are submitting Rev apps to the Mac App store" Ian From andre at andregarzia.com Fri Jul 22 05:11:28 2011 From: andre at andregarzia.com (Andre Garzia) Date: Fri, 22 Jul 2011 06:11:28 -0300 Subject: [ANN] New plugin AAG|LayerComps Message-ID: Folks, It is with immense happiness that I announce a new plugin called AAG|LayerComps. For those that know Adobe Photoshop, this is a recreation of its Layer Comps palette for LiveCode. It allows you to record the visibility and rects of controls in a given card and switch between those recorded states (layer comps, aka, layer compositions). Basically it allows you to layout your stack, press a button to save, change the layout, save it again, and then switch between those states (or more). I made a web page and some videos available here: http://andregarzia.com/page/aaglayercomps You can use that for many things, but the most obvious usage scenarios are: * Create different prototype layouts to show to a client or friend and switch between then easily. * Create different layouts for the different mobile resolutions and switch between them as needed. * Create complex interfaces where you reveal or hide parts of it without changing cards. For example: hiding and revealing different sidebars and inspectors. The different recorded states can be activated using the plugin or by code if you start using LibLayerComps stack. The plugin can copy this library stack into your topstack if you want. This plugin is my first commercial plugin priced at USD 30. It is really useful for those doing mobile, revlets or complex interfaces. Hope you guys like it Andre Alves Garzia -- http://www.andregarzia.com All We Do Is Code. From richmondmathewson at gmail.com Fri Jul 22 07:52:25 2011 From: richmondmathewson at gmail.com (Richmond Mathewson) Date: Fri, 22 Jul 2011 14:52:25 +0300 Subject: [OT] SVG graphics Message-ID: <4E296479.4060403@gmail.com> I remember a while back some questions about how to "get at SVG graphics"; this seems to be the best bet at the present: http://xmlgraphics.apache.org/batik/ Richmond. From roger.e.eller at sealedair.com Fri Jul 22 07:58:10 2011 From: roger.e.eller at sealedair.com (Roger Eller) Date: Fri, 22 Jul 2011 07:58:10 -0400 Subject: OT: It's Android Jim, but not as we know it (what CPUs does Livecode compile to?) In-Reply-To: References: <4E289BD8.9080908@hyperactivesw.com> <4E28C855.5060902@hyperactivesw.com> Message-ID: On Fri, Jul 22, 2011 at 1:43 AM, Bernard Devlin wrote: > Thanks Roger, that sounds like the first hurdle will be overcome. As > a machine that I might carry with me to run admin/monitoring apps > written in Livecode, the Transformer is looking better. I've never > tried to see how much could be done within the scriptLimits but > provided I don't look on it as being a main development machine, then > it may be workable ( I have no need to create standalones on it). I > too have no real expectation that Runrev would provide an IDE that > runs on Android/ARM - not unless such Android tablets/netbooks sweep > the world :) Until this week I had pretty much no interest in > developing for iOS/Android. > > Do you have a Livecode app in the Android Marketplace I could use to > test on it? If not, I'll take Jacques up on her kind offer offlist. > > Also, can one install a Livecode app on a machine without going > through the hassle of listing it in the Marketplace? Can they be > installed e.g. by downloading them from a URL? > > Looks like Runrev are going to be getting me to upgrade my license once > again. > > Bernard > > I don't have anything on the Android Market. Unlike iOS, you can easily install an APK that LiveCode has built, so I could email you a test app. You simply check the box in your security settings to allow an install from unknown sources, then go to the attachment in your email to install it, and afterwards uncheck allow unknown sources. ?Roger From ambassador at fourthworld.com Fri Jul 22 09:55:03 2011 From: ambassador at fourthworld.com (Richard Gaskin) Date: Fri, 22 Jul 2011 06:55:03 -0700 Subject: Lion problem report and fix In-Reply-To: References: Message-ID: <4E298137.1060807@fourthworld.com> Pete Haworth wrote: > I guess the question here is where on earth are you supposed to put an > application's files if you want them to be available to all users? It seems > like /Library/Application Support makes sense but apparently Apple have > decided otherwise. I hope they have published a guideline somewhere that > explains how to handle this situation. The rules haven't changed all that much; it's more like a prudent closure of a security exposure. Allowing all processes to write to the system's Library is dangerous. Problem solved in Lion. So what do we do? Nothing much different from how it's been for the last decade: Applications: most of an app's files go here, including the executable and other components like stack files and externals, all the essentials you app needs to run Preferences: user-specific settings for the app Application Support: Apple says, "These are the files that your application creates and manages on behalf of the user and can include files that contain user data." The use of App Support is rather loosely defined, its distinction from Preferences not exactly clear. Given that many users are comfortable ditching the prefs for an app, I would use App Support for those files which are more central to the app than preferences, need to be writable, and are not documents. For example, an RSS reader might puts its URL DB and cache in there. But I must admit I've never yet had a need for it. It's rather specialized; the subset of things that aren't essential to running the app (which should be in the bundle) and aren't user-specific settings (which should be in Prefs) is slim. If I did need to use it, I'd make sure the app contained copies of the files it needs to put there, checked for them at startup, and if not found would write the initial set of files to the user's App Support folder. By putting the smarts for this into the app, all users have the same starting point. In the even slimmer case where you need to have one user's settings affect other users, I would consider the Shared folder. It may or may not be what Apple would suggest, but it's a very rare case anyway and AFAIK neither OS X, Window, or Linux provides a globally-writable space for such things other than the Shared folder or its equivalent on other platforms. -- Richard Gaskin Fourth World LiveCode training and consulting: http://www.fourthworld.com Webzine for LiveCode developers: http://www.LiveCodeJournal.com LiveCode Journal blog: http://LiveCodejournal.com/blog.irv From bobcole at earthlink.net Fri Jul 22 10:02:29 2011 From: bobcole at earthlink.net (Bob Cole) Date: Fri, 22 Jul 2011 09:02:29 -0500 Subject: Detecting Different Versions of Livecode Message-ID: <51827931-7939-455D-AE44-9FA9B9D6A9B8@earthlink.net> Here is the result from revLicenseInfo for me: ---- Bob Cole Commercial 1 Windows,Mac OS X,Linux,Web,iOS,Server Local ---- Bob ---Original Message--- Message: 29 Date: Thu, 21 Jul 2011 22:26:49 -0700 From: Mark Wieder To: How to use LiveCode Subject: Re: Detecting Different Versions of Livecode Message-ID: <8797421046.20110721222649 at ahsoftware.net> Content-Type: text/plain; charset=us-ascii Pete- Thursday, July 21, 2011, 6:03:16 PM, you wrote: > Maybe you could tell me what you get for whatever license you have? Mark WIEDER Commercial 1 Windows,Mac OS X,Linux,Web,iOS,Android,Windows Mobile,Linux Mobile Local Mind you, I have no idea what Linux Mobile is, but it sounds good to me. And is that the actual spacing of lines in your version? I had thought that maybe line 3 could differentiate between the Commercial and Person versions, but from your post is looks like "Personal" is line 4. That's annoying if it's true. I have a blank line after my name and then another before the word "Local". And I have no idea why my last name is in all caps. Maybe I'm yelling at myself. Maybe it's a secret handshake the rev team uses to identify troublemakers. -- -Mark Wieder mwieder at ahsoftware.net From bobs at twft.com Fri Jul 22 11:34:46 2011 From: bobs at twft.com (Bob Sneidar) Date: Fri, 22 Jul 2011 08:34:46 -0700 Subject: [OT] SVG graphics In-Reply-To: <4E296479.4060403@gmail.com> References: <4E296479.4060403@gmail.com> Message-ID: <22599C28-DB6C-4C61-90EF-A1784AC014DB@twft.com> Isn't this the format that CAD software sometimes uses? This could have implications for apps who want to employ scrollable floorplans and such. Bob On Jul 22, 2011, at 4:52 AM, Richmond Mathewson wrote: > I remember a while back some questions about how to "get at SVG graphics"; > > this seems to be the best bet at the present: > > http://xmlgraphics.apache.org/batik/ > > Richmond. > > _______________________________________________ > use-livecode mailing list > use-livecode at lists.runrev.com > Please visit this url to subscribe, unsubscribe and manage your subscription preferences: > http://lists.runrev.com/mailman/listinfo/use-livecode From bobs at twft.com Fri Jul 22 11:38:49 2011 From: bobs at twft.com (Bob Sneidar) Date: Fri, 22 Jul 2011 08:38:49 -0700 Subject: Lion problem report and fix In-Reply-To: References: Message-ID: <6A60A897-4FF1-4C0A-8DC5-936E0F44412F@twft.com> Wha??? Root is no longer the Super User?? That is what this amounts to. I am ready to issue a ban on Lion on this network until I can determine the impact this will have on usability. I think this is a step in the wrong direction for Apple. Bob On Jul 21, 2011, at 10:13 PM, Shao Sean wrote: > ? Permissions for /Network/Applications and /Network/Library are now mode 555 (unwritable even by root) instead of mode 755 (writable only by root). From ambassador at fourthworld.com Fri Jul 22 11:45:45 2011 From: ambassador at fourthworld.com (Richard Gaskin) Date: Fri, 22 Jul 2011 08:45:45 -0700 Subject: Lion problem report and fix In-Reply-To: <6A60A897-4FF1-4C0A-8DC5-936E0F44412F@twft.com> References: <6A60A897-4FF1-4C0A-8DC5-936E0F44412F@twft.com> Message-ID: <4E299B29.8010801@fourthworld.com> Read-only is distinct from owner. Root can change the permissions to allow writing, and then write. But I wouldn't do it. If Apple feels those should be read-only, there's probably a good reason for it. -- Richard Gaskin Fourth World LiveCode training and consulting: http://www.fourthworld.com Webzine for LiveCode developers: http://www.LiveCodeJournal.com LiveCode Journal blog: http://LiveCodejournal.com/blog.irv Bob Sneidar wrote: > Wha??? Root is no longer the Super User?? That is what this amounts to. I am ready to issue a ban on Lion on this network until I can determine the impact this will have on usability. I think this is a step in the wrong direction for Apple. > > Bob > > On Jul 21, 2011, at 10:13 PM, Shao Sean wrote: > >> ? Permissions for /Network/Applications and /Network/Library are now mode 555 (unwritable even by root) instead of mode 755 (writable only by root). > > From bobs at twft.com Fri Jul 22 11:49:53 2011 From: bobs at twft.com (Bob Sneidar) Date: Fri, 22 Jul 2011 08:49:53 -0700 Subject: Detecting Different Versions of Livecode In-Reply-To: <197101747093.20110721233855@ahsoftware.net> References: <1311288631267-3685390.post@n4.nabble.com> <3081311531.20110721175819@ahsoftware.net> <8797421046.20110721222649@ahsoftware.net> <197101747093.20110721233855@ahsoftware.net> Message-ID: <2BFFCF15-A2BD-4AAC-9521-1A161F504D44@twft.com> I also have an evil twin. Although I have never met him, my associates assure me he exists. You can always tell it's him however. Ask him to spell his first name. He always spells it backwards. ;-) Bob On Jul 21, 2011, at 11:38 PM, Mark Wieder wrote: > Pete- > > Thursday, July 21, 2011, 11:11:30 PM, you wrote: > >> I guess your name is in uppercase to differentiate you from the other Mark >> "lowercase" Wieder (set the casesensitive to true). > > Doh! Of course! That guy's always making problems for me. > > -- > -Mark Wieder > mwieder at ahsoftware.net > > > _______________________________________________ > use-livecode mailing list > use-livecode at lists.runrev.com > Please visit this url to subscribe, unsubscribe and manage your subscription preferences: > http://lists.runrev.com/mailman/listinfo/use-livecode From lists.pete at haworths.org Fri Jul 22 12:57:32 2011 From: lists.pete at haworths.org (Pete Haworth) Date: Fri, 22 Jul 2011 09:57:32 -0700 Subject: Lion problem report and fix In-Reply-To: <4E298137.1060807@fourthworld.com> References: <4E298137.1060807@fourthworld.com> Message-ID: I don't think the rules have changed at all if I understand things correctly. Apple have just made it more difficult for people to abide by their own guidelines. They say Application Support is the place to put certain types of user files (and I assume that applies to both the user and system locations of that folder), then stop you from following their guidelines without jumping through hoops. >From the description, I'd say that SQL database files would be a good candidate to put in Application Support? If so and I have a database I want to be available to all users of a specific Mac, it seems like the system Application Support folder would be the place to put it. As an admin user, I would expect to be able to do that with the usual prompt for Admin user password and not have to issue Unix commands from Terminal. It's not the end of the world, just another installation gotcha that has to be dealt with. Pete On Fri, Jul 22, 2011 at 6:55 AM, Richard Gaskin wrote: > > > The rules haven't changed all that much; it's more like a prudent closure > of a security exposure. > > Allowing all processes to write to the system's Library is dangerous. > Problem solved in Lion. > > So what do we do? > > Nothing much different from how it's been for the last decade: > > Applications: most of an app's files go here, including the executable > and other components like stack files and externals, all > the essentials you app needs to run > > Preferences: user-specific settings for the app > > Application Support: Apple says, "These are the files that your > application creates and manages on behalf of the user > and can include files that contain user data." > > From bobs at twft.com Fri Jul 22 14:25:55 2011 From: bobs at twft.com (Bob Sneidar) Date: Fri, 22 Jul 2011 11:25:55 -0700 Subject: Lion problem report and fix In-Reply-To: <4E299B29.8010801@fourthworld.com> References: <6A60A897-4FF1-4C0A-8DC5-936E0F44412F@twft.com> <4E299B29.8010801@fourthworld.com> Message-ID: <5C3B44F0-8183-4D3C-B9C2-90873312F89C@twft.com> oic I misread. Bob On Jul 22, 2011, at 8:45 AM, Richard Gaskin wrote: > Read-only is distinct from owner. Root can change the permissions to allow writing, and then write. > > But I wouldn't do it. If Apple feels those should be read-only, there's probably a good reason for it. > > -- > Richard Gaskin > Fourth World > LiveCode training and consulting: http://www.fourthworld.com > Webzine for LiveCode developers: http://www.LiveCodeJournal.com > LiveCode Journal blog: http://LiveCodejournal.com/blog.irv > > Bob Sneidar wrote: >> Wha??? Root is no longer the Super User?? That is what this amounts to. I am ready to issue a ban on Lion on this network until I can determine the impact this will have on usability. I think this is a step in the wrong direction for Apple. >> >> Bob >> >> On Jul 21, 2011, at 10:13 PM, Shao Sean wrote: >> >>> ? Permissions for /Network/Applications and /Network/Library are now mode 555 (unwritable even by root) instead of mode 755 (writable only by root). >> >> > > _______________________________________________ > use-livecode mailing list > use-livecode at lists.runrev.com > Please visit this url to subscribe, unsubscribe and manage your subscription preferences: > http://lists.runrev.com/mailman/listinfo/use-livecode From jacque at hyperactivesw.com Fri Jul 22 14:33:45 2011 From: jacque at hyperactivesw.com (J. Landman Gay) Date: Fri, 22 Jul 2011 13:33:45 -0500 Subject: Detecting Different Versions of Livecode In-Reply-To: <197101747093.20110721233855@ahsoftware.net> References: <1311288631267-3685390.post@n4.nabble.com> <3081311531.20110721175819@ahsoftware.net> <8797421046.20110721222649@ahsoftware.net> <197101747093.20110721233855@ahsoftware.net> Message-ID: <4E29C289.7090207@hyperactivesw.com> On 7/22/11 1:38 AM, Mark Wieder wrote: > Pete- > > Thursday, July 21, 2011, 11:11:30 PM, you wrote: > >> I guess your name is in uppercase to differentiate you from the other Mark >> "lowercase" Wieder (set the casesensitive to true). > > Doh! Of course! That guy's always making problems for me. > I thought it was capitalized because you're so IMPORTANT. -- Jacqueline Landman Gay | jacque at hyperactivesw.com HyperActive Software | http://www.hyperactivesw.com From andre at andregarzia.com Fri Jul 22 14:38:57 2011 From: andre at andregarzia.com (Andre Garzia) Date: Fri, 22 Jul 2011 15:38:57 -0300 Subject: [ANN] New plugin AAG|LayerComps In-Reply-To: References: Message-ID: Folks, I've just made a little update to this... v0.4 :-) On Fri, Jul 22, 2011 at 6:11 AM, Andre Garzia wrote: > Folks, > > It is with immense happiness that I announce a new plugin called > AAG|LayerComps. For those that know Adobe Photoshop, this is a recreation of > its Layer Comps palette for LiveCode. It allows you to record the visibility > and rects of controls in a given card and switch between those recorded > states (layer comps, aka, layer compositions). Basically it allows you to > layout your stack, press a button to save, change the layout, save it again, > and then switch between those states (or more). > > I made a web page and some videos available here: > > http://andregarzia.com/page/aaglayercomps > > You can use that for many things, but the most obvious usage scenarios are: > > * Create different prototype layouts to show to a client or friend and > switch between then easily. > * Create different layouts for the different mobile resolutions and > switch between them as needed. > * Create complex interfaces where you reveal or hide parts of it without > changing cards. For example: hiding and revealing different sidebars and > inspectors. > > The different recorded states can be activated using the plugin or by code > if you start using LibLayerComps stack. The plugin can copy this library > stack into your topstack if you want. > > This plugin is my first commercial plugin priced at USD 30. > > It is really useful for those doing mobile, revlets or complex interfaces. > > Hope you guys like it > > Andre Alves Garzia > > > -- > http://www.andregarzia.com All We Do Is Code. > -- http://www.andregarzia.com All We Do Is Code. From jacque at hyperactivesw.com Fri Jul 22 15:02:21 2011 From: jacque at hyperactivesw.com (J. Landman Gay) Date: Fri, 22 Jul 2011 14:02:21 -0500 Subject: OT: It's Android Jim, but not as we know it (what CPUs does Livecode compile to?) In-Reply-To: References: <4E289BD8.9080908@hyperactivesw.com> <4E28C855.5060902@hyperactivesw.com> Message-ID: <4E29C93D.8010303@hyperactivesw.com> On 7/22/11 6:58 AM, Roger Eller wrote: > I don't have anything on the Android Market. Unlike iOS, you can easily > install an APK that LiveCode has built, so I could email you a test app. > You simply check the box in your security settings to allow an install from > unknown sources, then go to the attachment in your email to install it, and > afterwards uncheck allow unknown sources. I've been very grateful for the ease of installation on Android. There are so many more ways to do an installation than Apple provides, and it's always very easy. The problem with email on a store device is that Bernard would have to set up the display model with his email info. Given the versatility of Android, I was thinking of using my public folder in Dropbox. Then he could install the free Dropbox on the store model and just click on my test app in there. Or if the store doesn't mind, he could copy the app to a thumb drive, attach it to the device, and install that way. At any rate, lots of options. The downside is that it is just as easy for someone to distribute malicious software. An Android user has to be more careful about that than iOS users. -- Jacqueline Landman Gay | jacque at hyperactivesw.com HyperActive Software | http://www.hyperactivesw.com From jacque at hyperactivesw.com Fri Jul 22 15:06:34 2011 From: jacque at hyperactivesw.com (J. Landman Gay) Date: Fri, 22 Jul 2011 14:06:34 -0500 Subject: OT: Go see Google today Message-ID: <4E29CA3A.2000807@hyperactivesw.com> Google has another neat logo today. -- Jacqueline Landman Gay | jacque at hyperactivesw.com HyperActive Software | http://www.hyperactivesw.com From rdimola at evergreeninfo.net Fri Jul 22 15:41:40 2011 From: rdimola at evergreeninfo.net (Ralph DiMola) Date: Fri, 22 Jul 2011 15:41:40 -0400 Subject: Report #9630 Android SQLite has stopped working in 4.6.3 Message-ID: <006501cc48a7$60ceddd0$226c9970$@net> SQLite has stopped working in 4.6.3 (dp1,2,3,RC1,Public release). If you do a "revOpenDatabase("sqlite", tDatabasePath, , , , )" the app just quits on the Android HW. And the origin of the of the crash has scrolled off the old android plug-in log viewer history. The IDE still works fine. 4.6.2 works fine. The IDE is on Win XP SP3 32 bit. Testing on a Droid X running 2.2 Froyo Ralph DiMola IT Director Evergreen Information Services Phone: 518-636-3998 Ex:11 Cell: 518-796-9332 From rdimola at evergreeninfo.net Fri Jul 22 15:48:47 2011 From: rdimola at evergreeninfo.net (Ralph DiMola) Date: Fri, 22 Jul 2011 15:48:47 -0400 Subject: Exit the app via a close stack on the Android platform Report# 9632 Message-ID: <006a01cc48a8$5f8180d0$1e848270$@net> Top of Form If you try to exit the app via a close stack on the Android HW the app freezes. If you start the app again you get a black screen. Then if you try to test a new version(via test button) the HW locks up and only a battery removal will get you running again. Running the IDE on Win XP SP3 32 bit. Testing on a Droid X running 2.2 Froyo. Bottom of Form _____ Ralph DiMola IT Director Evergreen Information Services Phone: 518-636-3998 Ex:11 Cell: 518-796-9332 From jacque at hyperactivesw.com Fri Jul 22 16:10:02 2011 From: jacque at hyperactivesw.com (J. Landman Gay) Date: Fri, 22 Jul 2011 15:10:02 -0500 Subject: Exit the app via a close stack on the Android platform Report# 9632 In-Reply-To: <006a01cc48a8$5f8180d0$1e848270$@net> References: <006a01cc48a8$5f8180d0$1e848270$@net> Message-ID: <4E29D91A.1070108@hyperactivesw.com> On 7/22/11 2:48 PM, Ralph DiMola wrote: > Top of Form > > If you try to exit the app via a close stack on the Android HW the app > freezes. > > If you start the app again you get a black screen. Then if you try to test a > > new version(via test button) the HW locks up and only a battery removal will > > get you running again. The engine closes the stack by itself, so you shouldn't handle that. If you have any cleanup you need to do, you can trap the shutdown message and do it, but don't close the stack. Let the engine do it. I see you've put in a bug report on it, that's good. -- Jacqueline Landman Gay | jacque at hyperactivesw.com HyperActive Software | http://www.hyperactivesw.com From rdimola at evergreeninfo.net Fri Jul 22 16:18:25 2011 From: rdimola at evergreeninfo.net (Ralph DiMola) Date: Fri, 22 Jul 2011 16:18:25 -0400 Subject: Bug Report# 9633 Android keystore file Message-ID: <007a01cc48ac$8378d250$8a6a76f0$@net> Top of Form In desktop shortcut for LC, the "Start in" folder points to "C:\Program Files\RunRev\LiveCode 4.6.x\". This seems normal enough. But if you specify a Keystore file in the Android standalone applications screen the IDE can't find it when building the app it unless you change the "Start in" folder to the folder where the .livecode file lives and restart LC. Also if you change the current directory to anything else in your scripts while testing, again the keystore file can't be found and LC has to be restarted. Also when doing a "Save as" the default folder is not where you opened the .livecode file from but is instead pointed to the LC shortcut "Start in" folder or the folder one last changed the directory to in a script. Running the IDE on Win XP SP3 32 bit. Testing on a Droid X running 2.2 Froyo Bottom of Form Ralph DiMola IT Director Evergreen Information Services Phone: 518-636-3998 Ex:11 Cell: 518-796-9332 From m.schonewille at economy-x-talk.com Fri Jul 22 16:19:15 2011 From: m.schonewille at economy-x-talk.com (Mark Schonewille) Date: Fri, 22 Jul 2011 22:19:15 +0200 Subject: Bug Report# 9633 Android keystore file In-Reply-To: <007a01cc48ac$8378d250$8a6a76f0$@net> References: <007a01cc48ac$8378d250$8a6a76f0$@net> Message-ID: Hi Ralph, Why are you posting this? Do you have a question? -- Best regards, Mark Schonewille Economy-x-Talk Consulting and Software Engineering Homepage: http://economy-x-talk.com Twitter: http://twitter.com/xtalkprogrammer KvK: 50277553 New: Download the Installer Maker Plugin 1.6 for LiveCode here http://qery.us/ce On 22 jul 2011, at 22:18, Ralph DiMola wrote: > > > Top of Form > > In desktop shortcut for LC, the "Start in" folder points to "C:\Program > > Files\RunRev\LiveCode 4.6.x\". This seems normal enough. But if you specify > a > From johnpatten at mac.com Fri Jul 22 16:34:12 2011 From: johnpatten at mac.com (John Patten) Date: Fri, 22 Jul 2011 13:34:12 -0700 Subject: Accessing mySQL DB from stack utilizing revserver/irev/on-rev? Message-ID: <3EEBBAA2-39BC-4DF8-85F2-20ED4E2925F9@mac.com> Hi All I'm trying to get a handle on using on-rev and irev files on the server in combination with post commands and mysql queries directly from within a stack. Here's what I have so far: 1. One card with two fields (username & password) and a button. 2. mySQL table with records consisting of a field for username and a field for password. 3. the beginning of an irev file in directory on on-rev server. My button contains the following script: on mouseup put URLEncode(cd fld "username")& URLEncode(cd fld "password") into tLoginVariables post tLoginVariables to URL ("http://server.on-rev.com/sandbox/login.irev ") end mouseUp irev file on the on-rev server: References: <007a01cc48ac$8378d250$8a6a76f0$@net> Message-ID: <008501cc48b0$9c8f9040$d5aeb0c0$@net> Yes I have a Question. Am I a bone-head or do I have a legitimate concern? If I'm a bone-head the tell what I did wrong and help me out. If it is a legitimate bug then this is an FYI on the bug report. I thought the users would like to know. Ralph DiMola IT Director Evergreen Information Services Phone: 518-636-3998 Ex:11 Cell: 518-796-9332 -----Original Message----- From: use-livecode-bounces at lists.runrev.com [mailto:use-livecode-bounces at lists.runrev.com] On Behalf Of Mark Schonewille Sent: Friday, July 22, 2011 4:19 PM To: How to use LiveCode Subject: Re: Bug Report# 9633 Android keystore file Hi Ralph, Why are you posting this? Do you have a question? -- Best regards, Mark Schonewille Economy-x-Talk Consulting and Software Engineering Homepage: http://economy-x-talk.com Twitter: http://twitter.com/xtalkprogrammer KvK: 50277553 New: Download the Installer Maker Plugin 1.6 for LiveCode here http://qery.us/ce On 22 jul 2011, at 22:18, Ralph DiMola wrote: > > > Top of Form > > In desktop shortcut for LC, the "Start in" folder points to "C:\Program > > Files\RunRev\LiveCode 4.6.x\". This seems normal enough. But if you specify > a > _______________________________________________ use-livecode mailing list use-livecode at lists.runrev.com Please visit this url to subscribe, unsubscribe and manage your subscription preferences: http://lists.runrev.com/mailman/listinfo/use-livecode From admin at FlexibleLearning.com Fri Jul 22 16:55:29 2011 From: admin at FlexibleLearning.com (FlexibleLearning) Date: Fri, 22 Jul 2011 21:55:29 +0100 Subject: Lion: 10 Things that Bug Me In-Reply-To: Message-ID: I read this and thought of all the posts here... http://www.tuaw.com/2011/07/20/lion-ten-things-that-bug-me/ Hugh Senior FLCo From lists.pete at haworths.org Fri Jul 22 17:04:14 2011 From: lists.pete at haworths.org (Pete Haworth) Date: Fri, 22 Jul 2011 14:04:14 -0700 Subject: [ANN] New plugin AAG|LayerComps In-Reply-To: References: Message-ID: That sounds great! If I'm understanding it correctly, it will get round one of my pet peeves about changing card layouts in the IDE - many of the changes cannot be undone and even if you close the card and say you don't want to save, the changes are already saved. Pete On Fri, Jul 22, 2011 at 2:11 AM, Andre Garzia wrote: > Folks, > > It is with immense happiness that I announce a new plugin called > AAG|LayerComps. For those that know Adobe Photoshop, this is a recreation > of > its Layer Comps palette for LiveCode. It allows you to record the > visibility > and rects of controls in a given card and switch between those recorded > states (layer comps, aka, layer compositions). Basically it allows you to > layout your stack, press a button to save, change the layout, save it > again, > and then switch between those states (or more). > > I made a web page and some videos available here: > > http://andregarzia.com/page/aaglayercomps > > You can use that for many things, but the most obvious usage scenarios are: > > * Create different prototype layouts to show to a client or friend and > switch between then easily. > * Create different layouts for the different mobile resolutions and > switch between them as needed. > * Create complex interfaces where you reveal or hide parts of it without > changing cards. For example: hiding and revealing different sidebars and > inspectors. > > The different recorded states can be activated using the plugin or by code > if you start using LibLayerComps stack. The plugin can copy this library > stack into your topstack if you want. > > This plugin is my first commercial plugin priced at USD 30. > > It is really useful for those doing mobile, revlets or complex interfaces. > > Hope you guys like it > > Andre Alves Garzia > > > -- > http://www.andregarzia.com All We Do Is Code. > _______________________________________________ > use-livecode mailing list > use-livecode at lists.runrev.com > Please visit this url to subscribe, unsubscribe and manage your > subscription preferences: > http://lists.runrev.com/mailman/listinfo/use-livecode > From lists.pete at haworths.org Fri Jul 22 17:10:08 2011 From: lists.pete at haworths.org (Pete Haworth) Date: Fri, 22 Jul 2011 14:10:08 -0700 Subject: Lion: 10 Things that Bug Me In-Reply-To: References: Message-ID: Wow! I think I want to see Lion running on someone else's computer before upgrading, at least until they EOL Snow Leopard. Pete On Fri, Jul 22, 2011 at 1:55 PM, FlexibleLearning < admin at flexiblelearning.com> wrote: > I read this and thought of all the posts here... > > http://www.tuaw.com/2011/07/20/lion-ten-things-that-bug-me/ > > > Hugh Senior > FLCo > > _______________________________________________ > use-livecode mailing list > use-livecode at lists.runrev.com > Please visit this url to subscribe, unsubscribe and manage your > subscription preferences: > http://lists.runrev.com/mailman/listinfo/use-livecode > From richmondmathewson at gmail.com Fri Jul 22 17:10:26 2011 From: richmondmathewson at gmail.com (Richmond Mathewson) Date: Sat, 23 Jul 2011 00:10:26 +0300 Subject: Lion: 10 Things that Bug Me In-Reply-To: References: Message-ID: <4E29E742.9040700@gmail.com> On 07/22/2011 11:55 PM, FlexibleLearning wrote: > I read this and thought of all the posts here... > > http://www.tuaw.com/2011/07/20/lion-ten-things-that-bug-me/ > > > Hugh Senior > FLCo > > Well that does confirm my suspicions that as Software giants get bigger they get more arrogant and care less about their end-users. I am beginning to feel the same about Ubuntu and its "Unity" desktop, and, similarly to Lion, its really disgusting new 'now-you-see-me-now-you-don't' ungrabble scroll bars. From scott at elementarysoftware.com Fri Jul 22 17:12:55 2011 From: scott at elementarysoftware.com (Scott Morrow) Date: Fri, 22 Jul 2011 14:12:55 -0700 Subject: Lion problem report and fix In-Reply-To: <4E298137.1060807@fourthworld.com> References: <4E298137.1060807@fourthworld.com> Message-ID: <64710D79-5F4F-4BAA-A432-7BDE22DACE6A@elementarysoftware.com> In the case of a "splash screen" application where the executable "appears" to have the ability to update itself by deleting the old stack files containing the UI / logic and downloading and running new stack file(s), where would be the best place to put the UI / Logic files? I had come to the conclusion that the user's Application Support folder was best, since a user with a managed account could still update the application without the need of a system admin. Scott Morrow Elementary Software (Now with 20% less chalk dust!) web http://elementarysoftware.com/ email scott at elementarysoftware.com ------------------------------------------------------ On Jul 22, 2011, at 6:55 AM, Richard Gaskin wrote: > Application Support: Apple says, "These are the files that your > application creates and manages on behalf of the user > and can include files that contain user data." > > The use of App Support is rather loosely defined, its distinction from Preferences not exactly clear. Given that many users are comfortable ditching the prefs for an app, I would use App Support for those files which are more central to the app than preferences, need to be writable, and are not documents. For example, an RSS reader might puts its URL DB and cache in there. > > But I must admit I've never yet had a need for it. It's rather specialized; the subset of things that aren't essential to running the app (which should be in the bundle) and aren't user-specific settings (which should be in Prefs) is slim. From francois.chaplais at mines-paristech.fr Fri Jul 22 17:18:54 2011 From: francois.chaplais at mines-paristech.fr (=?iso-8859-1?Q?Fran=E7ois_Chaplais?=) Date: Fri, 22 Jul 2011 23:18:54 +0200 Subject: Lion: 10 Things that Bug Me In-Reply-To: References: Message-ID: from the same blog http://www.tuaw.com/2011/07/21/the-perils-of-bashing-an-os-youve-never-used/ Le 22 juil. 2011 ? 22:55, FlexibleLearning a ?crit : > I read this and thought of all the posts here... > > http://www.tuaw.com/2011/07/20/lion-ten-things-that-bug-me/ > > > Hugh Senior > FLCo > > _______________________________________________ > use-livecode mailing list > use-livecode at lists.runrev.com > Please visit this url to subscribe, unsubscribe and manage your subscription preferences: > http://lists.runrev.com/mailman/listinfo/use-livecode From bobs at twft.com Fri Jul 22 17:21:48 2011 From: bobs at twft.com (Bob Sneidar) Date: Fri, 22 Jul 2011 14:21:48 -0700 Subject: Lion: 10 Things that Bug Me In-Reply-To: <4E29E742.9040700@gmail.com> References: <4E29E742.9040700@gmail.com> Message-ID: <8BAB9570-237C-469D-B944-8ECD0E8B14D4@twft.com> That is a big leap, from "I don't like the way they changed things" or "There seems to be a disconnect between the developers and the end users" to the purely moral judgement, "They are arrogant and don't care about end users". While I share some of your frustrations, I have to say, I would not like that judgement willy nilly made of me, especially by someone who doesn't know me from Adam. I think what is happening is that Apple believes all computing devices are going to become more like the iPad, and are developing towards that end, to get a jump on the market trend they foresee. In fact, they probably have lots of plans to hasten things along that path. I think it is a miscalculation on their part. But I was wrong once before, so it's possible it could happen again. ;-) Bob On Jul 22, 2011, at 2:10 PM, Richmond Mathewson wrote: > On 07/22/2011 11:55 PM, FlexibleLearning wrote: >> I read this and thought of all the posts here... >> >> http://www.tuaw.com/2011/07/20/lion-ten-things-that-bug-me/ >> >> >> Hugh Senior >> FLCo >> >> > Well that does confirm my suspicions that as Software giants get bigger they get more arrogant and care less about their end-users. > > I am beginning to feel the same about Ubuntu and its "Unity" desktop, and, similarly > to Lion, its really disgusting new 'now-you-see-me-now-you-don't' ungrabble scroll > bars. > > _______________________________________________ > use-livecode mailing list > use-livecode at lists.runrev.com > Please visit this url to subscribe, unsubscribe and manage your subscription preferences: > http://lists.runrev.com/mailman/listinfo/use-livecode From kray at sonsothunder.com Fri Jul 22 17:22:21 2011 From: kray at sonsothunder.com (Ken Ray) Date: Fri, 22 Jul 2011 16:22:21 -0500 Subject: Lion problem report and fix In-Reply-To: <64710D79-5F4F-4BAA-A432-7BDE22DACE6A@elementarysoftware.com> References: <4E298137.1060807@fourthworld.com> <64710D79-5F4F-4BAA-A432-7BDE22DACE6A@elementarysoftware.com> Message-ID: <4FE3E9E8-2566-42E5-B196-5280E293616E@sonsothunder.com> On Jul 22, 2011, at 4:12 PM, Scott Morrow wrote: > In the case of a "splash screen" application where the executable "appears" to have the ability to update itself by deleting the old stack files containing the UI / logic and downloading and running new stack file(s), where would be the best place to put the UI / Logic files? I had come to the conclusion that the user's Application Support folder was best, since a user with a managed account could still update the application without the need of a system admin. Yes, that's one place to keep it - the other is inside the application bundle. For *me* it comes down to this: will the location of these stacks need to be "user-accessible"? Meaning, will the user ever need to look at or manipulate them? If the answer is "yes", then I'd suggest ~/Application Support/. If not, then I'd suggest putting it inside the application bundle. Ken Ray Sons of Thunder Software, Inc. Email: kray at sonsothunder.com Web Site: http://www.sonsothunder.com/ From jacque at hyperactivesw.com Fri Jul 22 17:26:00 2011 From: jacque at hyperactivesw.com (J. Landman Gay) Date: Fri, 22 Jul 2011 16:26:00 -0500 Subject: [ANN] New plugin AAG|LayerComps In-Reply-To: References: Message-ID: <4E29EAE8.6050302@hyperactivesw.com> On 7/22/11 4:04 PM, Pete Haworth wrote: > That sounds great! If I'm understanding it correctly, it will get round one > of my pet peeves about changing card layouts in the IDE - many of the > changes cannot be undone and even if you close the card and say you don't > want to save, the changes are already saved. The changes aren't really saved, LiveCode never does that. Probably your stacks are using the default setting of the destroyStack property, which is not to remove the stack from RAM when it closes. In that case, closing the stack removes it from the message path and from view, but keeps the current copy in memory. The next time you open it, it opens the copy in RAM which does still contain your changes. It hasn't been saved to disk though, and if you quit LiveCode, or choose "Close and remove from memory" from the file menu, then you'll see it revert to its last-saved state. One of the first things I do when setting up preferences is to set the default behavior of destroystack to true, so that the situation never occurs. When I click the close box, the stack is removed completely so that when it re-opens, its actual last-saved state is active. Destroystack was intended to speed up the display 15 years ago when machines were much slower. It isn't really needed any more. The one advantage it does have is if you don't save a stack and then you're sorry, you can get your unsaved changes back by re-opening the stack before you quit LiveCode. So it's a trade-off. You can set the destroystack property for newly-created stacks in the Files and Memory section of prefs. This won't change stacks you already have created. For that, use the stack property inspector. -- Jacqueline Landman Gay | jacque at hyperactivesw.com HyperActive Software | http://www.hyperactivesw.com From francois.chaplais at mines-paristech.fr Fri Jul 22 17:30:21 2011 From: francois.chaplais at mines-paristech.fr (=?iso-8859-1?Q?Fran=E7ois_Chaplais?=) Date: Fri, 22 Jul 2011 23:30:21 +0200 Subject: Rejoice: I have received the DVD from the san jose conference Message-ID: <921FFC19-11B2-4A08-A75C-C913B5D47564@mines-paristech.fr> Shipped from Edimburgh to my Paris suburb home. The bindings inside the DVD box were broken, but the disks seems to be OK. Thanks, RunRev! From m.schonewille at economy-x-talk.com Fri Jul 22 17:48:55 2011 From: m.schonewille at economy-x-talk.com (Mark Schonewille) Date: Fri, 22 Jul 2011 23:48:55 +0200 Subject: Bug Report# 9633 Android keystore file In-Reply-To: <008501cc48b0$9c8f9040$d5aeb0c0$@net> References: <007a01cc48ac$8378d250$8a6a76f0$@net> <008501cc48b0$9c8f9040$d5aeb0c0$@net> Message-ID: Hi Ralph, If you are a bonehead, then it is only partly :-) I would expect to see the full filepath to the key file in the Android pane of the preferences window. Apparently, only the file name is stored in the preferences. I can see that this causes problems. In your case, you might want to keep the key file and your stack in the same folder and set the defaultFolder to the path to that folder before building a standalone. That should allow you to build standalones without changing the shortcut in the start menu. In my opinion, this is a relatively simple bug that needs to be fixed. -- Best regards, Mark Schonewille Economy-x-Talk Consulting and Software Engineering Homepage: http://economy-x-talk.com Twitter: http://twitter.com/xtalkprogrammer KvK: 50277553 New: Download the Installer Maker Plugin 1.6 for LiveCode here http://qery.us/ce On 22 jul 2011, at 22:47, Ralph DiMola wrote: > Yes I have a Question. Am I a bone-head or do I have a legitimate concern? > If I'm a bone-head the tell what I did wrong and help me out. If it is a > legitimate bug then this is an FYI on the bug report. I thought the users > would like to know. > > > Ralph DiMola > IT Director > Evergreen Information Services > Phone: 518-636-3998 Ex:11 > Cell: 518-796-9332 From keith at gulfbreezeortholab.com Fri Jul 22 17:59:12 2011 From: keith at gulfbreezeortholab.com (Keith (Gulf Breeze Ortho Lab)) Date: Fri, 22 Jul 2011 16:59:12 -0500 Subject: Writing to and from a binary file? Message-ID: Hello All, I am new to LiveCode and am continuing to explore the language. Quick question... Is it possible to write to and from a binary file utilizing LiveCode? For example, is it possible to take a graphic file (for example) and split it into several unequal parts, and then put the parts back together again into a single file? I have used another programming package to accomplish such. FYI: Reading in the Dictionary, I came across the byte and binfile entries. Can these be used to accomplish what I am looking for? If so, then are there any good examples out there? Thanks, and any help is most appreciated! Cheers from Florida! Keith "Boo" L. From scott at elementarysoftware.com Fri Jul 22 18:00:17 2011 From: scott at elementarysoftware.com (Scott Morrow) Date: Fri, 22 Jul 2011 15:00:17 -0700 Subject: Lion problem report and fix In-Reply-To: <4FE3E9E8-2566-42E5-B196-5280E293616E@sonsothunder.com> References: <4E298137.1060807@fourthworld.com> <64710D79-5F4F-4BAA-A432-7BDE22DACE6A@elementarysoftware.com> <4FE3E9E8-2566-42E5-B196-5280E293616E@sonsothunder.com> Message-ID: <1F7317E5-564C-4511-8DE2-CA7B7E08A7B6@elementarysoftware.com> ForIf it is inside the application bundle and that is inside the Applications folder then users with managed accounts won't be able to allow changes, as in the case of an update. Perhaps this is an unusual situation. On Jul 22, 2011, at 2:22 PM, Ken Ray wrote: > > On Jul 22, 2011, at 4:12 PM, Scott Morrow wrote: > >> In the case of a "splash screen" application where the executable "appears" to have the ability to update itself by deleting the old stack files containing the UI / logic and downloading and running new stack file(s), where would be the best place to put the UI / Logic files? I had come to the conclusion that the user's Application Support folder was best, since a user with a managed account could still update the application without the need of a system admin. > > Yes, that's one place to keep it - the other is inside the application bundle. For *me* it comes down to this: will the location of these stacks need to be "user-accessible"? Meaning, will the user ever need to look at or manipulate them? If the answer is "yes", then I'd suggest ~/Application Support/. If not, then I'd suggest putting it inside the application bundle. > > > Ken Ray > Sons of Thunder Software, Inc. > Email: kray at sonsothunder.com > Web Site: http://www.sonsothunder.com/ > > _______________________________________________ > use-livecode mailing list > use-livecode at lists.runrev.com > Please visit this url to subscribe, unsubscribe and manage your subscription preferences: > http://lists.runrev.com/mailman/listinfo/use-livecode From m.schonewille at economy-x-talk.com Fri Jul 22 18:05:58 2011 From: m.schonewille at economy-x-talk.com (Mark Schonewille) Date: Sat, 23 Jul 2011 00:05:58 +0200 Subject: Writing to and from a binary file? In-Reply-To: References: Message-ID: <073B71C7-EAE1-49BD-A907-777D68A9B7F7@economy-x-talk.com> Hi Keith, Unless I misuderstand you, you should be able to do what you want using byte and binfile (and the read and write commands or the put url command). -- Best regards, Mark Schonewille Economy-x-Talk Consulting and Software Engineering Homepage: http://economy-x-talk.com Twitter: http://twitter.com/xtalkprogrammer KvK: 50277553 New: Download the Installer Maker Plugin 1.6 for LiveCode here http://qery.us/ce On 22 jul 2011, at 23:59, Keith (Gulf Breeze Ortho Lab) wrote: > Hello All, > > I am new to LiveCode and am continuing to explore the language. > > Quick question... Is it possible to write to and from a binary file utilizing LiveCode? For example, is it possible to take a graphic file (for example) and split it into several unequal parts, and then put the parts back together again into a single file? I have used another programming package to accomplish such. > > FYI: Reading in the Dictionary, I came across the byte and binfile entries. Can these be used to accomplish what I am looking for? If so, then are there any good examples out there? > > Thanks, and any help is most appreciated! > > Cheers from Florida! > > Keith "Boo" L. From keith at gulfbreezeortholab.com Fri Jul 22 18:17:05 2011 From: keith at gulfbreezeortholab.com (Keith (Gulf Breeze Ortho Lab)) Date: Fri, 22 Jul 2011 17:17:05 -0500 Subject: Writing to and from a binary file? In-Reply-To: <073B71C7-EAE1-49BD-A907-777D68A9B7F7@economy-x-talk.com> References: <073B71C7-EAE1-49BD-A907-777D68A9B7F7@economy-x-talk.com> Message-ID: Thanks Mark. Would you happen to know of any examples out there especially for binary files? The greatest obstacle to discovery is not ignorance -- it is the illusion of knowledge. Daniel Boorstin ----- Original Message ----- From: "Mark Schonewille" To: "How to use LiveCode" Sent: Friday, July 22, 2011 5:05 PM Subject: Re: Writing to and from a binary file? > Hi Keith, > > Unless I misuderstand you, you should be able to do what you want using > byte and binfile (and the read and write commands or the put url command). > > -- > Best regards, > > Mark Schonewille > > Economy-x-Talk Consulting and Software Engineering > Homepage: http://economy-x-talk.com > Twitter: http://twitter.com/xtalkprogrammer > KvK: 50277553 > > New: Download the Installer Maker Plugin 1.6 for LiveCode here > http://qery.us/ce > > On 22 jul 2011, at 23:59, Keith (Gulf Breeze Ortho Lab) wrote: > >> Hello All, >> >> I am new to LiveCode and am continuing to explore the language. >> >> Quick question... Is it possible to write to and from a binary file >> utilizing LiveCode? For example, is it possible to take a graphic file >> (for example) and split it into several unequal parts, and then put the >> parts back together again into a single file? I have used another >> programming package to accomplish such. >> >> FYI: Reading in the Dictionary, I came across the byte and binfile >> entries. Can these be used to accomplish what I am looking for? If so, >> then are there any good examples out there? >> >> Thanks, and any help is most appreciated! >> >> Cheers from Florida! >> >> Keith "Boo" L. > > > _______________________________________________ > use-livecode mailing list > use-livecode at lists.runrev.com > Please visit this url to subscribe, unsubscribe and manage your > subscription preferences: > http://lists.runrev.com/mailman/listinfo/use-livecode > From m.schonewille at economy-x-talk.com Fri Jul 22 18:31:14 2011 From: m.schonewille at economy-x-talk.com (Mark Schonewille) Date: Sat, 23 Jul 2011 00:31:14 +0200 Subject: Writing to and from a binary file? In-Reply-To: References: <073B71C7-EAE1-49BD-A907-777D68A9B7F7@economy-x-talk.com> Message-ID: Hi Keith, This is standard stuff. on example answer file "Select a binary file..." if it is not empty then put it into myFile open file myFile for binary read read from file myFile until EOF put it into myData close file myFile // do something with the data below this line end if end example Instead of open, read, close you can also use: put url ("binfile:" & myFile) into myData Open/read/close gives you more control, but put url seems more straightforward (and less lines) -- Best regards, Mark Schonewille Economy-x-Talk Consulting and Software Engineering Homepage: http://economy-x-talk.com Twitter: http://twitter.com/xtalkprogrammer KvK: 50277553 New: Download the Installer Maker Plugin 1.6 for LiveCode here http://qery.us/ce On 23 jul 2011, at 00:17, Keith (Gulf Breeze Ortho Lab) wrote: > Thanks Mark. Would you happen to know of any examples out there especially for binary files? From lists.pete at haworths.org Fri Jul 22 18:45:10 2011 From: lists.pete at haworths.org (Pete Haworth) Date: Fri, 22 Jul 2011 15:45:10 -0700 Subject: [ANN] New plugin AAG|LayerComps In-Reply-To: <4E29EAE8.6050302@hyperactivesw.com> References: <4E29EAE8.6050302@hyperactivesw.com> Message-ID: OK, thanks for that info, that will definitely help. I tend to be wary of setting anything with "destroy" in its name to true! However Andre's plugin is still a hugely useful tool. I just watched the video on his web site and it appears that you can revert to previous layouts of any stack at any time. So I can make layout changes to multiple stacks (saving them in his plugin), save the whole stack file and when I open it again, I can back out changes to any individual stack. It seems that he has provided layout version control on a stack by (sub)stack basis. Pete On Fri, Jul 22, 2011 at 2:26 PM, J. Landman Gay wrote: > On 7/22/11 4:04 PM, Pete Haworth wrote: > >> That sounds great! If I'm understanding it correctly, it will get round >> one >> of my pet peeves about changing card layouts in the IDE - many of the >> changes cannot be undone and even if you close the card and say you don't >> want to save, the changes are already saved. >> > > The changes aren't really saved, LiveCode never does that. Probably your > stacks are using the default setting of the destroyStack property, which is > not to remove the stack from RAM when it closes. In that case, closing the > stack removes it from the message path and from view, but keeps the current > copy in memory. The next time you open it, it opens the copy in RAM which > does still contain your changes. It hasn't been saved to disk though, and if > you quit LiveCode, or choose "Close and remove from memory" from the file > menu, then you'll see it revert to its last-saved state. > > One of the first things I do when setting up preferences is to set the > default behavior of destroystack to true, so that the situation never > occurs. When I click the close box, the stack is removed completely so that > when it re-opens, its actual last-saved state is active. > > Destroystack was intended to speed up the display 15 years ago when > machines were much slower. It isn't really needed any more. The one > advantage it does have is if you don't save a stack and then you're sorry, > you can get your unsaved changes back by re-opening the stack before you > quit LiveCode. So it's a trade-off. > > You can set the destroystack property for newly-created stacks in the Files > and Memory section of prefs. This won't change stacks you already have > created. For that, use the stack property inspector. > > -- > Jacqueline Landman Gay | jacque at hyperactivesw.com > HyperActive Software | http://www.hyperactivesw.com > > > ______________________________**_________________ > use-livecode mailing list > use-livecode at lists.runrev.com > Please visit this url to subscribe, unsubscribe and manage your > subscription preferences: > http://lists.runrev.com/**mailman/listinfo/use-livecode > From andre at andregarzia.com Fri Jul 22 18:51:30 2011 From: andre at andregarzia.com (Andre Garzia) Date: Fri, 22 Jul 2011 19:51:30 -0300 Subject: [ANN] New plugin AAG|LayerComps In-Reply-To: References: <4E29EAE8.6050302@hyperactivesw.com> Message-ID: On Fri, Jul 22, 2011 at 7:45 PM, Pete Haworth wrote: > OK, thanks for that info, that will definitely help. I tend to be wary of > setting anything with "destroy" in its name to true! > > However Andre's plugin is still a hugely useful tool. I just watched the > video on his web site and it appears that you can revert to previous > layouts > of any stack at any time. So I can make layout changes to multiple stacks > (saving them in his plugin), save the whole stack file and when I open it > again, I can back out changes to any individual stack. It seems that he > has > provided layout version control on a stack by (sub)stack basis. > yes you can use it to backup different layouts provided that you create new layer comps for each backup. The objective is to be able to switch layouts and/or layout elements on the fly. It also has an API that allows you to active those recorded layouts by code. :-) > > Pete > > > > On Fri, Jul 22, 2011 at 2:26 PM, J. Landman Gay >wrote: > > > On 7/22/11 4:04 PM, Pete Haworth wrote: > > > >> That sounds great! If I'm understanding it correctly, it will get round > >> one > >> of my pet peeves about changing card layouts in the IDE - many of the > >> changes cannot be undone and even if you close the card and say you > don't > >> want to save, the changes are already saved. > >> > > > > The changes aren't really saved, LiveCode never does that. Probably your > > stacks are using the default setting of the destroyStack property, which > is > > not to remove the stack from RAM when it closes. In that case, closing > the > > stack removes it from the message path and from view, but keeps the > current > > copy in memory. The next time you open it, it opens the copy in RAM which > > does still contain your changes. It hasn't been saved to disk though, and > if > > you quit LiveCode, or choose "Close and remove from memory" from the file > > menu, then you'll see it revert to its last-saved state. > > > > One of the first things I do when setting up preferences is to set the > > default behavior of destroystack to true, so that the situation never > > occurs. When I click the close box, the stack is removed completely so > that > > when it re-opens, its actual last-saved state is active. > > > > Destroystack was intended to speed up the display 15 years ago when > > machines were much slower. It isn't really needed any more. The one > > advantage it does have is if you don't save a stack and then you're > sorry, > > you can get your unsaved changes back by re-opening the stack before you > > quit LiveCode. So it's a trade-off. > > > > You can set the destroystack property for newly-created stacks in the > Files > > and Memory section of prefs. This won't change stacks you already have > > created. For that, use the stack property inspector. > > > > -- > > Jacqueline Landman Gay | jacque at hyperactivesw.com > > HyperActive Software | http://www.hyperactivesw.com > > > > > > ______________________________**_________________ > > use-livecode mailing list > > use-livecode at lists.runrev.com > > Please visit this url to subscribe, unsubscribe and manage your > > subscription preferences: > > http://lists.runrev.com/**mailman/listinfo/use-livecode< > http://lists.runrev.com/mailman/listinfo/use-livecode> > > > _______________________________________________ > use-livecode mailing list > use-livecode at lists.runrev.com > Please visit this url to subscribe, unsubscribe and manage your > subscription preferences: > http://lists.runrev.com/mailman/listinfo/use-livecode > -- http://www.andregarzia.com All We Do Is Code. From lists.pete at haworths.org Fri Jul 22 19:20:19 2011 From: lists.pete at haworths.org (Pete Haworth) Date: Fri, 22 Jul 2011 16:20:19 -0700 Subject: [ANN] New plugin AAG|LayerComps In-Reply-To: References: <4E29EAE8.6050302@hyperactivesw.com> Message-ID: Thanks Andre, I just purchased it and so far it's working great! Pete On Fri, Jul 22, 2011 at 3:51 PM, Andre Garzia wrote: > On Fri, Jul 22, 2011 at 7:45 PM, Pete Haworth >wrote: > > > OK, thanks for that info, that will definitely help. I tend to be wary > of > > setting anything with "destroy" in its name to true! > > > > However Andre's plugin is still a hugely useful tool. I just watched the > > video on his web site and it appears that you can revert to previous > > layouts > > of any stack at any time. So I can make layout changes to multiple > stacks > > (saving them in his plugin), save the whole stack file and when I open it > > again, I can back out changes to any individual stack. It seems that he > > has > > provided layout version control on a stack by (sub)stack basis. > > > > yes you can use it to backup different layouts provided that you create new > layer comps for each backup. The objective is to be able to switch layouts > and/or layout elements on the fly. It also has an API that allows you to > active those recorded layouts by code. > > :-) > > > > > > > > Pete > > > > > > > > On Fri, Jul 22, 2011 at 2:26 PM, J. Landman Gay < > jacque at hyperactivesw.com > > >wrote: > > > > > On 7/22/11 4:04 PM, Pete Haworth wrote: > > > > > >> That sounds great! If I'm understanding it correctly, it will get > round > > >> one > > >> of my pet peeves about changing card layouts in the IDE - many of the > > >> changes cannot be undone and even if you close the card and say you > > don't > > >> want to save, the changes are already saved. > > >> > > > > > > The changes aren't really saved, LiveCode never does that. Probably > your > > > stacks are using the default setting of the destroyStack property, > which > > is > > > not to remove the stack from RAM when it closes. In that case, closing > > the > > > stack removes it from the message path and from view, but keeps the > > current > > > copy in memory. The next time you open it, it opens the copy in RAM > which > > > does still contain your changes. It hasn't been saved to disk though, > and > > if > > > you quit LiveCode, or choose "Close and remove from memory" from the > file > > > menu, then you'll see it revert to its last-saved state. > > > > > > One of the first things I do when setting up preferences is to set the > > > default behavior of destroystack to true, so that the situation never > > > occurs. When I click the close box, the stack is removed completely so > > that > > > when it re-opens, its actual last-saved state is active. > > > > > > Destroystack was intended to speed up the display 15 years ago when > > > machines were much slower. It isn't really needed any more. The one > > > advantage it does have is if you don't save a stack and then you're > > sorry, > > > you can get your unsaved changes back by re-opening the stack before > you > > > quit LiveCode. So it's a trade-off. > > > > > > You can set the destroystack property for newly-created stacks in the > > Files > > > and Memory section of prefs. This won't change stacks you already have > > > created. For that, use the stack property inspector. > > > > > > -- > > > Jacqueline Landman Gay | jacque at hyperactivesw.com > > > HyperActive Software | http://www.hyperactivesw.com > > > > > > > > > ______________________________**_________________ > > > use-livecode mailing list > > > use-livecode at lists.runrev.com > > > Please visit this url to subscribe, unsubscribe and manage your > > > subscription preferences: > > > http://lists.runrev.com/**mailman/listinfo/use-livecode< > > http://lists.runrev.com/mailman/listinfo/use-livecode> > > > > > _______________________________________________ > > use-livecode mailing list > > use-livecode at lists.runrev.com > > Please visit this url to subscribe, unsubscribe and manage your > > subscription preferences: > > http://lists.runrev.com/mailman/listinfo/use-livecode > > > > > > -- > http://www.andregarzia.com All We Do Is Code. > _______________________________________________ > use-livecode mailing list > use-livecode at lists.runrev.com > Please visit this url to subscribe, unsubscribe and manage your > subscription preferences: > http://lists.runrev.com/mailman/listinfo/use-livecode > From andre at andregarzia.com Fri Jul 22 19:29:48 2011 From: andre at andregarzia.com (Andre Garzia) Date: Fri, 22 Jul 2011 20:29:48 -0300 Subject: [ANN] New plugin AAG|LayerComps In-Reply-To: References: <4E29EAE8.6050302@hyperactivesw.com> Message-ID: Pete, Thanks for the support! :-) I hope you like it! This is just one of the many things I have in the oven :-) If you have any feedback or request, just mail me! Cheers andre On Fri, Jul 22, 2011 at 8:20 PM, Pete Haworth wrote: > Thanks Andre, I just purchased it and so far it's working great! > > Pete > > > > On Fri, Jul 22, 2011 at 3:51 PM, Andre Garzia > wrote: > > > On Fri, Jul 22, 2011 at 7:45 PM, Pete Haworth > >wrote: > > > > > OK, thanks for that info, that will definitely help. I tend to be wary > > of > > > setting anything with "destroy" in its name to true! > > > > > > However Andre's plugin is still a hugely useful tool. I just watched > the > > > video on his web site and it appears that you can revert to previous > > > layouts > > > of any stack at any time. So I can make layout changes to multiple > > stacks > > > (saving them in his plugin), save the whole stack file and when I open > it > > > again, I can back out changes to any individual stack. It seems that > he > > > has > > > provided layout version control on a stack by (sub)stack basis. > > > > > > > yes you can use it to backup different layouts provided that you create > new > > layer comps for each backup. The objective is to be able to switch > layouts > > and/or layout elements on the fly. It also has an API that allows you to > > active those recorded layouts by code. > > > > :-) > > > > > > > > > > > > > > Pete > > > > > > > > > > > > On Fri, Jul 22, 2011 at 2:26 PM, J. Landman Gay < > > jacque at hyperactivesw.com > > > >wrote: > > > > > > > On 7/22/11 4:04 PM, Pete Haworth wrote: > > > > > > > >> That sounds great! If I'm understanding it correctly, it will get > > round > > > >> one > > > >> of my pet peeves about changing card layouts in the IDE - many of > the > > > >> changes cannot be undone and even if you close the card and say you > > > don't > > > >> want to save, the changes are already saved. > > > >> > > > > > > > > The changes aren't really saved, LiveCode never does that. Probably > > your > > > > stacks are using the default setting of the destroyStack property, > > which > > > is > > > > not to remove the stack from RAM when it closes. In that case, > closing > > > the > > > > stack removes it from the message path and from view, but keeps the > > > current > > > > copy in memory. The next time you open it, it opens the copy in RAM > > which > > > > does still contain your changes. It hasn't been saved to disk though, > > and > > > if > > > > you quit LiveCode, or choose "Close and remove from memory" from the > > file > > > > menu, then you'll see it revert to its last-saved state. > > > > > > > > One of the first things I do when setting up preferences is to set > the > > > > default behavior of destroystack to true, so that the situation never > > > > occurs. When I click the close box, the stack is removed completely > so > > > that > > > > when it re-opens, its actual last-saved state is active. > > > > > > > > Destroystack was intended to speed up the display 15 years ago when > > > > machines were much slower. It isn't really needed any more. The one > > > > advantage it does have is if you don't save a stack and then you're > > > sorry, > > > > you can get your unsaved changes back by re-opening the stack before > > you > > > > quit LiveCode. So it's a trade-off. > > > > > > > > You can set the destroystack property for newly-created stacks in the > > > Files > > > > and Memory section of prefs. This won't change stacks you already > have > > > > created. For that, use the stack property inspector. > > > > > > > > -- > > > > Jacqueline Landman Gay | jacque at hyperactivesw.com > > > > HyperActive Software | http://www.hyperactivesw.com > > > > > > > > > > > > ______________________________**_________________ > > > > use-livecode mailing list > > > > use-livecode at lists.runrev.com > > > > Please visit this url to subscribe, unsubscribe and manage your > > > > subscription preferences: > > > > http://lists.runrev.com/**mailman/listinfo/use-livecode< > > > http://lists.runrev.com/mailman/listinfo/use-livecode> > > > > > > > _______________________________________________ > > > use-livecode mailing list > > > use-livecode at lists.runrev.com > > > Please visit this url to subscribe, unsubscribe and manage your > > > subscription preferences: > > > http://lists.runrev.com/mailman/listinfo/use-livecode > > > > > > > > > > > -- > > http://www.andregarzia.com All We Do Is Code. > > _______________________________________________ > > use-livecode mailing list > > use-livecode at lists.runrev.com > > Please visit this url to subscribe, unsubscribe and manage your > > subscription preferences: > > http://lists.runrev.com/mailman/listinfo/use-livecode > > > _______________________________________________ > use-livecode mailing list > use-livecode at lists.runrev.com > Please visit this url to subscribe, unsubscribe and manage your > subscription preferences: > http://lists.runrev.com/mailman/listinfo/use-livecode > -- http://www.andregarzia.com All We Do Is Code. From shaosean at wehostmacs.com Fri Jul 22 19:46:54 2011 From: shaosean at wehostmacs.com (Shao Sean) Date: Fri, 22 Jul 2011 19:46:54 -0400 Subject: Lion problem report and fix Message-ID: More information from Apple in regards to the ~/library/application support directory and what it is used for.. The Library directory is where applications and other code modules store their custom data files. Regardless of whether you are writing code for iOS or Mac OS X, understanding the structure of the Library directory is important. You use this directory to store data files, caches, resources, preferences, and even user data in some specific situations. There are several Library directories throughout the system but only a few that your code should ever need to access: ? Library in the current home directory?This is the version of the directory you use the most because it is the one that contains all user-specific files. In iOS, the home directory is the application?s sandbox directory. In Mac OS X, it is the application?s sandbox directory or the current user?s home directory (if the application is not in a sandbox). ? /Library (Mac OS X only)?Applications that share resources between users store those resources in this version of the Library directory. Sandboxed application are not permitted to use this directory. ? /System/Library (Mac OS X only)?This directory is reserved for use by Apple. Application Support Use this directory to store all application data files except those associated with the user?s documents. For example, you might use this directory to store application-created data files, configuration files, templates, or other fixed or modifiable resources that are managed by the application. An application might use this directory to store a modifiable copy of resources contained initially in the application?s bundle. A game might use this directory to store new levels purchased by the user and downloaded from a server. All content in this directory should be placed in a custom subdirectory whose name is that of your application?s bundle identifier or your company. http://developer.apple.com/library/mac/#documentation/FileManagement/ Conceptual/FileSystemProgrammingGUide/FileSystemOverview/ FileSystemOverview.html%23//apple_ref/doc/uid/TP40010672-CH2-SW1 From bobs at twft.com Fri Jul 22 19:47:42 2011 From: bobs at twft.com (Bob Sneidar) Date: Fri, 22 Jul 2011 16:47:42 -0700 Subject: Schema Admin Message-ID: <02276897-840E-4060-AFE8-5DC46E1FD1DC@twft.com> Hi all. A few days ago I stumbled upon a livecode developer's site which purported to have a kind of SQL Schema manager, which allowed you to build a database and it would create links between like named columns and such. I cannot for the life of me find that again. I sent the person an email tellign him the links to the files were broken but haven't heard anything back. Any ideas? Bob From bobs at twft.com Fri Jul 22 19:52:23 2011 From: bobs at twft.com (Bob Sneidar) Date: Fri, 22 Jul 2011 16:52:23 -0700 Subject: Schema Admin Message-ID: <5C574EAC-F684-4852-8153-7BCBFD3D3394@twft.com> Never mind I found it. It's Mark Wieder. Mark, are you out there? Hullooo? Give me a holler off list. I'd like to try and maybe buy your db Schema Generator. Bob > Hi all. > > A few days ago I stumbled upon a livecode developer's site which purported to have a kind of SQL Schema manager, which allowed you to build a database and it would create links between like named columns and such. I cannot for the life of me find that again. I sent the person an email tellign him the links to the files were broken but haven't heard anything back. > > Any ideas? > > Bob From mwieder at ahsoftware.net Fri Jul 22 20:46:05 2011 From: mwieder at ahsoftware.net (Mark Wieder) Date: Fri, 22 Jul 2011 17:46:05 -0700 Subject: Detecting Different Versions of Livecode In-Reply-To: <4E29C289.7090207@hyperactivesw.com> References: <1311288631267-3685390.post@n4.nabble.com> <3081311531.20110721175819@ahsoftware.net> <8797421046.20110721222649@ahsoftware.net> <197101747093.20110721233855@ahsoftware.net> <4E29C289.7090207@hyperactivesw.com> Message-ID: <99166977375.20110722174605@ahsoftware.net> Jacque- Friday, July 22, 2011, 11:33:45 AM, you wrote: > I thought it was capitalized because you're so IMPORTANT. I think it's more like when I was a kid and my parents used to call me in CAPITAL LETTERS. You could tell they were capital letters from a block away, which is usually where I was and why they had to do that in the first place. -- -Mark Wieder mwieder at ahsoftware.net From mwieder at ahsoftware.net Fri Jul 22 20:46:57 2011 From: mwieder at ahsoftware.net (Mark Wieder) Date: Fri, 22 Jul 2011 17:46:57 -0700 Subject: Detecting Different Versions of Livecode In-Reply-To: <2BFFCF15-A2BD-4AAC-9521-1A161F504D44@twft.com> References: <1311288631267-3685390.post@n4.nabble.com> <3081311531.20110721175819@ahsoftware.net> <8797421046.20110721222649@ahsoftware.net> <197101747093.20110721233855@ahsoftware.net> <2BFFCF15-A2BD-4AAC-9521-1A161F504D44@twft.com> Message-ID: <184167029234.20110722174657@ahsoftware.net> Bob- Friday, July 22, 2011, 8:49:53 AM, you wrote: > I also have an evil twin. Although I have never met him, my > associates assure me he exists. You can always tell it's him > however. Ask him to spell his first name. He always spells it > backwards. ;-) I've gotten emails from that guy. Don't worry - I just ignore them. -- -Mark Wieder mwieder at ahsoftware.net From mwieder at ahsoftware.net Fri Jul 22 20:49:48 2011 From: mwieder at ahsoftware.net (Mark Wieder) Date: Fri, 22 Jul 2011 17:49:48 -0700 Subject: [ANN] New plugin AAG|LayerComps In-Reply-To: <4E29EAE8.6050302@hyperactivesw.com> References: <4E29EAE8.6050302@hyperactivesw.com> Message-ID: <63167200328.20110722174948@ahsoftware.net> Jacque- Friday, July 22, 2011, 2:26:00 PM, you wrote: > You can set the destroystack property for newly-created stacks in the > Files and Memory section of prefs. Unless you create stacks by script, in which case the destroyStack preference is ignored... sad but true... -- -Mark Wieder mwieder at ahsoftware.net From bvg at mac.com Fri Jul 22 21:01:50 2011 From: bvg at mac.com (=?iso-8859-1?Q?Bj=F6rnke_von_Gierke?=) Date: Sat, 23 Jul 2011 03:01:50 +0200 Subject: [ANN] LiveCode.tv event #33 In-Reply-To: <0C1B6D89-1745-46DF-9C49-A5FE2E5B67B9@mac.com> References: <6B865405-BBC0-491C-AD25-BFB1C85A2D58@mac.com> <7EC6A85B-DD6B-4138-A597-512A194F4453@mac.com> <89782C22-C6E3-4245-95D1-A48F69423753@mac.com> <061D9182-B34B-4ABB-A7D6-9C2D09DA8755@mac.com> <73F84954-890B-4627-859A-702C8054F13B@mac.com> <093DEF5E-6C0F-4924-A262-F6DBE748ED45@mac.com> <3FB852C9-5675-4A19-9506-543733BB2547@mac.com> <914DF999-596E-4023-A213-9469C6A69FA0@mac.com> <2B105965-FB27-41D0-B95F-1F4C84ADD53B@mac.com> <163225D1-67D1-4CE7-8049-E85A8D94D177@mac.com> <225B0941-5D11-434A-BC0B-CD61B998E9F8@mac.com> <201756EB-9601-417D-856E-5128C5256EF1@mac.com> <081FD717-0A48-447D-90AA-A8F370B14F43@mac.com> <913B7E78-3052-4CF0-9883-C1CDD9F8BDFA@mac.com> <0C1B6D89-1745-46DF-9C49-A5FE2E5B67B9@mac.com> Message-ID: <497CC7C9-74BC-419C-9785-CEA07A8FE181@mac.com> I invite you heartily to join us tomorrow and watch Mario and David. Simply join ChatRev, and win an eBook about Mac OS X Lion: Go to http://bjoernke.com/chatrev/ or enter in the message box: go stack URL ?http://bjoernke.com/chatrev/chatrev1.3b3.rev? This Saturday at 20:00 in Monaco: Sat. 22:00 Moscow Sat. 14:00 New York Sat. 11:00 Los Angeles Sun. 2:00 Beijing Mario Miele will show us his style editing and storing palette. It allows him to grab the styles and visual properties of any object, and store them for later re-usal. http://livecode.tv/mario/ Between shows, the European HyperCard User Group (eHUG, http://www.ehug.info) will raffle off an e-book courtesy of TidBITS. This time around it?s ?Take Control of Using Lion?. You can find more info on the TC series here: http://www.takecontrolbooks.com David Bovill will wrap up the night with a look at his work on version control framework, which he aims to release soon, and makes collaboratively working on the same stack with several people automatically work. http://livecode.tv/david/ cheers Bj?rnke > On 15 Jul 2011, at 04:12, Bj?rnke von Gierke wrote: > >> Join us once again, when we show LiveCode at it's best. >> >> This Saturday, Pete Haworth will update us on his qd[SQL] development. qd[SQL] is a currently in development, and will allow you to easily create databases without using any SQL at all, by linking LiveCodes object/group/card metaphor to a backend Database automatically. >> >> See also his explanation on Google Docs: https://docs.google.com/document/d/1Rl7B3JJrnmOGctXqd-OL26a37jN10cVD43lujyF26uE/edit?hl=en_US >> >> Then, the European HyperCard User Group (eHUG, http://www.ehug.info) will raffle off an e-book courtesy of TidBITS. This time around it?s ?Take Control of TextExpander?. >> You can find more info on the TC series here: >> http://www.takecontrolbooks.com >> >> Finally, Bj?rnke von Gierke will take you on a tour trough some of the add-ons from the 3 for 2 special deal ( http://www.runrev.com/mailers/3for2/3for2.html ), currently offered on the RunRev store with the coupon code "THREEFORTWO". Some of the creator of those plugins will also be available in chat, so keep those questions ready. >> >> Don't forget that you can vote on which plug-in you are interested in: >> http://blog.livecode.tv/2011/07/poll-for-event-32/ >> >> >> All this and more on Saturdays Livecode.tv show: >> Sat. 22:00 Moscow >> Sat. 20:00 Paris >> Sat. 19:00h GMT >> Sat. 14:00 New York >> Sat. 11:00 Los Angeles >> Sun. 04:00 Sydney >> Sun. 03:00 Tokyo >> Sun. 2:00 Beijing >> >> Make sure to join ChatRev on the right time, otherwise you won?t know where to watch, because the correct streams will be announced in ChatRev. ChatRev is open 24/7 for your chatting needs. >> Go to >> http://bjoernke.com/chatrev/ >> or enter in the message box: >> go stack URL ?http://bjoernke.com/chatrev/chatrev1.3b3.rev? >> >> If you would like to show something in an event, now or somewhen, please head over to the participation page for ideas, or contact us: >> http://blog.livecode.tv/participate/ >> http://www3.economy-x-talk.com/file.php?node=contact >> >> cheers >> Bj?rnke >> _______________________________________________ >> use-livecode mailing list >> use-livecode at lists.runrev.com >> Please visit this url to subscribe, unsubscribe and manage your subscription preferences: >> http://lists.runrev.com/mailman/listinfo/use-livecode > > > _______________________________________________ > use-livecode mailing list > use-livecode at lists.runrev.com > Please visit this url to subscribe, unsubscribe and manage your subscription preferences: > http://lists.runrev.com/mailman/listinfo/use-livecode From jacque at hyperactivesw.com Fri Jul 22 22:49:28 2011 From: jacque at hyperactivesw.com (J. Landman Gay) Date: Fri, 22 Jul 2011 21:49:28 -0500 Subject: [ANN] New plugin AAG|LayerComps In-Reply-To: <63167200328.20110722174948@ahsoftware.net> References: <4E29EAE8.6050302@hyperactivesw.com> <63167200328.20110722174948@ahsoftware.net> Message-ID: <4E2A36B8.10906@hyperactivesw.com> On 7/22/11 7:49 PM, Mark Wieder wrote: > Jacque- > > Friday, July 22, 2011, 2:26:00 PM, you wrote: > >> You can set the destroystack property for newly-created stacks in the >> Files and Memory section of prefs. > > Unless you create stacks by script, in which case the destroyStack > preference is ignored... sad but true... > For real? But...I do that all the time. I guess I never actually checked on it, but I don't remember the files hanging around in memory either. Is that some new glitch? Does it also happen in standalones? (I wonder how you'd check that in a standalone.) -- Jacqueline Landman Gay | jacque at hyperactivesw.com HyperActive Software | http://www.hyperactivesw.com From jacque at hyperactivesw.com Fri Jul 22 22:51:51 2011 From: jacque at hyperactivesw.com (J. Landman Gay) Date: Fri, 22 Jul 2011 21:51:51 -0500 Subject: [ANN] New plugin AAG|LayerComps In-Reply-To: References: <4E29EAE8.6050302@hyperactivesw.com> Message-ID: <4E2A3747.8030101@hyperactivesw.com> On 7/22/11 5:45 PM, Pete Haworth wrote: > OK, thanks for that info, that will definitely help. I tend to be wary of > setting anything with "destroy" in its name to true! You aren't the only one. It was an unfortunate choice of terminology, and it's scared most everyone at some point. It should be called "purgeStack". > However Andre's plugin is still a hugely useful tool. Of that I'm sure. Andre's a genius. -- Jacqueline Landman Gay | jacque at hyperactivesw.com HyperActive Software | http://www.hyperactivesw.com From mwieder at ahsoftware.net Fri Jul 22 23:26:01 2011 From: mwieder at ahsoftware.net (Mark Wieder) Date: Fri, 22 Jul 2011 20:26:01 -0700 Subject: [ANN] New plugin AAG|LayerComps In-Reply-To: <4E2A36B8.10906@hyperactivesw.com> References: <4E29EAE8.6050302@hyperactivesw.com> <63167200328.20110722174948@ahsoftware.net> <4E2A36B8.10906@hyperactivesw.com> Message-ID: <104176573546.20110722202601@ahsoftware.net> Jacque- Friday, July 22, 2011, 7:49:28 PM, you wrote: > On 7/22/11 7:49 PM, Mark Wieder wrote: >> Jacque- >> >> Friday, July 22, 2011, 2:26:00 PM, you wrote: >> >>> You can set the destroystack property for newly-created stacks in the >>> Files and Memory section of prefs. >> >> Unless you create stacks by script, in which case the destroyStack >> preference is ignored... sad but true... >> > For real? But...I do that all the time. I guess I never actually checked > on it, but I don't remember the files hanging around in memory either. > Is that some new glitch? Does it also happen in standalones? (I wonder > how you'd check that in a standalone.) BZ #3190 (17 October 2005), but as Mark Waddingham attempts to explain "This isn't actually a bug." -- -Mark Wieder mwieder at ahsoftware.net From mwieder at ahsoftware.net Fri Jul 22 23:30:05 2011 From: mwieder at ahsoftware.net (Mark Wieder) Date: Fri, 22 Jul 2011 20:30:05 -0700 Subject: [ANN] New plugin AAG|LayerComps In-Reply-To: <4E2A3747.8030101@hyperactivesw.com> References: <4E29EAE8.6050302@hyperactivesw.com> <4E2A3747.8030101@hyperactivesw.com> Message-ID: <194176817265.20110722203005@ahsoftware.net> Jacque- Friday, July 22, 2011, 7:51:51 PM, you wrote: > On 7/22/11 5:45 PM, Pete Haworth wrote: >> OK, thanks for that info, that will definitely help. I tend to be wary of >> setting anything with "destroy" in its name to true! > You aren't the only one. It was an unfortunate choice of terminology, > and it's scared most everyone at some point. It should be called > "purgeStack". ...and that's BZ #1072 (18 December 2003), still listed as "Unconfirmed". -- -Mark Wieder mwieder at ahsoftware.net From lists.pete at haworths.org Sat Jul 23 01:21:02 2011 From: lists.pete at haworths.org (Pete Haworth) Date: Fri, 22 Jul 2011 22:21:02 -0700 Subject: [ANN] New plugin AAG|LayerComps In-Reply-To: <4E2A3747.8030101@hyperactivesw.com> References: <4E29EAE8.6050302@hyperactivesw.com> <4E2A3747.8030101@hyperactivesw.com> Message-ID: I just noticed on Andre's video, it's called something like "Purge this stack" in his inspector - maybe that's a 4.6.3 change. I guess "Purge" is marginally better than "destroy"! Pete On Fri, Jul 22, 2011 at 7:51 PM, J. Landman Gay wrote: > On 7/22/11 5:45 PM, Pete Haworth wrote: > >> OK, thanks for that info, that will definitely help. I tend to be wary of >> setting anything with "destroy" in its name to true! >> > > You aren't the only one. It was an unfortunate choice of terminology, and > it's scared most everyone at some point. It should be called "purgeStack". > > From revdev at pdslabs.net Sat Jul 23 01:22:19 2011 From: revdev at pdslabs.net (Phil Davis) Date: Fri, 22 Jul 2011 22:22:19 -0700 Subject: Writing to and from a binary file? In-Reply-To: References: <073B71C7-EAE1-49BD-A907-777D68A9B7F7@economy-x-talk.com> Message-ID: <4E2A5A8B.4050902@pdslabs.net> Hi Keith - welcome to the community! Here is a code snippet I shared on the use-revolution list almost 2 years ago. It shows how to cut an image up into 10 'frames': http://www.mail-archive.com/use-revolution at lists.runrev.com/msg124691.html HTH - Phil Davis On 7/22/11 3:17 PM, Keith (Gulf Breeze Ortho Lab) wrote: > Thanks Mark. Would you happen to know of any examples out there especially for > binary files? > > > The greatest obstacle to discovery is not ignorance -- > it is the illusion of knowledge. Daniel Boorstin > ----- Original Message ----- From: "Mark Schonewille" > > To: "How to use LiveCode" > Sent: Friday, July 22, 2011 5:05 PM > Subject: Re: Writing to and from a binary file? > > >> Hi Keith, >> >> Unless I misuderstand you, you should be able to do what you want using byte >> and binfile (and the read and write commands or the put url command). >> >> -- >> Best regards, >> >> Mark Schonewille >> >> Economy-x-Talk Consulting and Software Engineering >> Homepage: http://economy-x-talk.com >> Twitter: http://twitter.com/xtalkprogrammer >> KvK: 50277553 >> >> New: Download the Installer Maker Plugin 1.6 for LiveCode here http://qery.us/ce >> >> On 22 jul 2011, at 23:59, Keith (Gulf Breeze Ortho Lab) wrote: >> >>> Hello All, >>> >>> I am new to LiveCode and am continuing to explore the language. >>> >>> Quick question... Is it possible to write to and from a binary file >>> utilizing LiveCode? For example, is it possible to take a graphic file (for >>> example) and split it into several unequal parts, and then put the parts >>> back together again into a single file? I have used another programming >>> package to accomplish such. >>> >>> FYI: Reading in the Dictionary, I came across the byte and binfile entries. >>> Can these be used to accomplish what I am looking for? If so, then are there >>> any good examples out there? >>> >>> Thanks, and any help is most appreciated! >>> >>> Cheers from Florida! >>> >>> Keith "Boo" L. >> >> >> _______________________________________________ >> use-livecode mailing list >> use-livecode at lists.runrev.com >> Please visit this url to subscribe, unsubscribe and manage your subscription >> preferences: >> http://lists.runrev.com/mailman/listinfo/use-livecode >> > > > _______________________________________________ > use-livecode mailing list > use-livecode at lists.runrev.com > Please visit this url to subscribe, unsubscribe and manage your subscription > preferences: > http://lists.runrev.com/mailman/listinfo/use-livecode > -- Phil Davis PDS Labs Professional Software Development http://pdslabs.net From richmondmathewson at gmail.com Sat Jul 23 02:15:30 2011 From: richmondmathewson at gmail.com (Richmond Mathewson) Date: Sat, 23 Jul 2011 09:15:30 +0300 Subject: Lion: 10 Things that Bug Me In-Reply-To: <8BAB9570-237C-469D-B944-8ECD0E8B14D4@twft.com> References: <4E29E742.9040700@gmail.com> <8BAB9570-237C-469D-B944-8ECD0E8B14D4@twft.com> Message-ID: <4E2A6702.9090904@gmail.com> On 07/23/2011 12:21 AM, Bob Sneidar wrote: > That is a big leap, from "I don't like the way they changed things" or "There seems to be a disconnect between the developers and the end users" to the purely moral judgement, "They are arrogant and don't care about end users". While I share some of your frustrations, I have to say, I would not like that judgement willy nilly made of me, especially by someone who doesn't know me from Adam. Well, I may not know you from Adam; but I have been using Mac OS since 1993 (System 7). > > I think what is happening is that Apple believes all computing devices are going to become more like the iPad, and are developing towards that end, to get a jump on the market trend they foresee. In fact, they probably have lots of plans to hasten things along that path. I think it is a miscalculation on their part. But I was wrong once before, so it's possible it could happen again. ;-) An iPad is one thing, and a desktop computer is another, and a laptop is a third. I see Mac OS focussing on the iPad at the expense of the desktop, and I see Ubuntu focussing on the laptop at the expense of the desktop. This will, eventually, cheese-off desktop users to the extent that they look for other OSes that closer suit their needs. > > Bob > > > From bdrunrev at gmail.com Sat Jul 23 04:19:57 2011 From: bdrunrev at gmail.com (Bernard Devlin) Date: Sat, 23 Jul 2011 09:19:57 +0100 Subject: OT: It's Android Jim, but not as we know it (what CPUs does Livecode compile to?) In-Reply-To: <4E29C93D.8010303@hyperactivesw.com> References: <4E289BD8.9080908@hyperactivesw.com> <4E28C855.5060902@hyperactivesw.com> <4E29C93D.8010303@hyperactivesw.com> Message-ID: Jacque, that sounds like the easiest option in order for me to test the device in-store (but at this stage I'm feeling very confident it will work to run Livecode apps). If you could put a simple 'hello world' on to dropbox and send me the URL off list, then I'll give it a go. The various hurdles put in the way of app distribution/installation on the various iOS devices is the very thing that has kept me far away from those devices (and indeed from other Apple products) - not that any of that has stopped Apple's roaring profits :) Bernard On Fri, Jul 22, 2011 at 8:02 PM, J. Landman Gay wrote: > I've been very grateful for the ease of installation on Android. There are > so many more ways to do an installation than Apple provides, and it's always > very easy. > > The problem with email on a store device is that Bernard would have to set > up the display model with his email info. Given the versatility of Android, > I was thinking of using my public folder in Dropbox. Then he could install > the free Dropbox on the store model and just click on my test app in there. > Or if the store doesn't mind, he could copy the app to a thumb drive, attach > it to the device, and install that way. From sarah.reichelt at gmail.com Sat Jul 23 04:42:28 2011 From: sarah.reichelt at gmail.com (Sarah Reichelt) Date: Sat, 23 Jul 2011 18:42:28 +1000 Subject: Accessing mySQL DB from stack utilizing revserver/irev/on-rev? In-Reply-To: <3EEBBAA2-39BC-4DF8-85F2-20ED4E2925F9@mac.com> References: <3EEBBAA2-39BC-4DF8-85F2-20ED4E2925F9@mac.com> Message-ID: Hi John, When POSTing variables to the server, you have to identify them, so the script on the server can tell what is what. So your call to the server would need to be something like this: put URLEncode(fld "username") into tUserName put URLEncode(fld "password") into tPassword put "username=" & tUserName & "&password=" & tPassword into tLoginVariables post tLoginVariables to URL ("http://server.on-rev.com/sandbox/login.irev") Now when this gets to your login.irev file on the server, it can read the contents of the $_POST array to find what it needs: wrote: > Hi All > > ?I'm trying to get a handle on using on-rev and irev files on the server in > combination with post commands and mysql queries directly from within a > stack. > > ?Here's what I have so far: > > ?1. One card with two fields (username & password) and a button. > > ?2. mySQL table with records consisting of a field for username and a field > for password. > > ?3. the beginning of an irev file in directory on on-rev server. > > > > My button contains the following script: > > on mouseup > > put URLEncode(cd fld "username")& URLEncode(cd fld "password") into > tLoginVariables > > post tLoginVariables to URL ("http://server.on-rev.com/sandbox/login.irev") > > end mouseUp > > > > irev file on the on-rev server: > > > > > put "address-of-db" into tDatabaseAddress > > ?put "dbname" into tDatabaseName > > ?put "dbUsername" into tDatabaseUser > > ?put "dbpassword" into tDatabasePassword > > > > ?...and that's about as far as I've gotten so far. ?:-( > > > > Essentially I want to create the functionality I have working in my stack > now into a project that makes the database calls via an irev (on-rev) calls. > I would like to try making a mobile app out of what I have but my stack > relies heavily on calls to the mySQL database for data. > > ?I have very limited experience with irev and just need to know how to query > the database, get the resulting data from the query, do some stuff with in > the stack, and then put the data back into the database. > > ?Is there and example of this available? > > ?Thank you! > > John Patten > > SUSD From brami.serge at gmail.com Sat Jul 23 06:48:58 2011 From: brami.serge at gmail.com (Serge Brami) Date: Sat, 23 Jul 2011 12:48:58 +0200 Subject: pasting text from revbrowser Message-ID: <21BA7EEF-24B5-4F63-A0A5-4E3F5D919F44@gmail.com> Hello All I try to get the contents of a revbrowser window and paste it into a livecode field the web page is a banking chart first I select manually the text in the revbrowser window Then I Use revbrowserGet selected put the contents into myvariable and paste it into a field (using :set the htmltext of field myfield to myvariable) the problem is that all the return characters disappears Even if i copy manually the text and paste it directly in a field i get the same thing If I copy the text manually and paste it into a Word processor software it works fine the return characters don't disappear Have you any idea ?? best regards From mkoob at rogers.com Sat Jul 23 09:15:13 2011 From: mkoob at rogers.com (Martin Koob) Date: Sat, 23 Jul 2011 06:15:13 -0700 (PDT) Subject: on-Rev client software In-Reply-To: <4E280ED4.90502@crcom.net> References: <4E280ED4.90502@crcom.net> Message-ID: <1311426913309-3688862.post@n4.nabble.com> Hi Len I started with on-rev a while ago, I had no background in web development other than some basic HTML. The way I figured it out was I downloaded John Craig's .irev scripts and MySQL files from his Splash21 on-rev site that he developed and uploaded them to my own on-rev site as a starting point. You can find these at http://splash21.on-rev.com You have to login to see the downloads link. Then I started poking around the scripts learning how they worked and started modifying them to create my own site. John's scripts and handlers made it quite easy to create new pages and manage users and access to pages and interact with the mySQL database. Along the way I found other resources such as Sarah Reichelt's webpage http://www.troz.net/rev This has examples as well as links to other sites with on-rev info The other thing I used was the irev-engine-notes.txt file, which was dated 2009-06-02 I can't remember where I got that. That gives some basic explanations and documents some of the commands that were not in the dictionary. I set up my site before revigniter framework was developed http://revigniter.com/ I haven't yet tried revigniter yet but I want to but since the Splash21 based site is serving my needs right now It hasn't been a priority. Martin Koob -- View this message in context: http://runtime-revolution.278305.n4.nabble.com/on-Rev-client-software-tp3683452p3688862.html Sent from the Revolution - User mailing list archive at Nabble.com. From jimaultwins at yahoo.com Sat Jul 23 10:03:24 2011 From: jimaultwins at yahoo.com (Jim Ault) Date: Sat, 23 Jul 2011 07:03:24 -0700 Subject: pasting text from revbrowser In-Reply-To: <21BA7EEF-24B5-4F63-A0A5-4E3F5D919F44@gmail.com> References: <21BA7EEF-24B5-4F63-A0A5-4E3F5D919F44@gmail.com> Message-ID: <67BA5C1B-2ABD-4181-A8F6-DBC46D7A5536@yahoo.com> On Jul 23, 2011, at 3:48 AM, Serge Brami wrote: > I try to get the contents of a revbrowser window and paste it > into a livecode field > the web page is a banking chart > > first I select manually the text in the revbrowser window > > Then I Use revbrowserGet selected put the contents into > myvariable and paste it into a field (using :set the htmltext > of field myfield to myvariable) > the problem is that all the return characters disappears > > Even if i copy manually the text and paste it directly in a > field i get the same thing > If I copy the text manually and paste it into a Word processor > software it works fine the return characters don't disappear You have not specified which platform, but try this test on checkReturns if the clipboardData contains numtochar(13) then answer "Found char 13, a return on a Mac" end if if the clipboardData contains numtochar(10) then answer "Found char 10, a return in Rev/ Unix" end if if the clipboardData contains CRLF then answer "Found char 1310, a return-line feed in Windows" end if if the clipboardData contains " may be a tag that is used to create the effect of a return char on the web page. A word processor program is designed to do many conversions for the user's convenience, such as clipboard conversions of text and graphics. Hope this helps. > > Jim Ault Las Vegas From jimaultwins at yahoo.com Sat Jul 23 10:32:42 2011 From: jimaultwins at yahoo.com (Jim Ault) Date: Sat, 23 Jul 2011 07:32:42 -0700 Subject: on-Rev client software In-Reply-To: <1311426913309-3688862.post@n4.nabble.com> References: <4E280ED4.90502@crcom.net> <1311426913309-3688862.post@n4.nabble.com> Message-ID: <7BB47C2B-DFF8-4A3E-BD96-8E6394A6147D@yahoo.com> Just a note about Splash21 The author's name is Scott McDonald, and his web site http://splash21.on-rev.com/ John Craig is the author of MobGUI for LC mobile app programmers. By the way, John is a Scot, living in or near Edinburgh, and the Splash21 Scott lives in Australia. On Jul 23, 2011, at 6:15 AM, Martin Koob wrote: > Hi Len > I started with on-rev a while ago, I had no background in web > development > other than some basic HTML. The way I figured it out was I > downloaded John > Craig's .irev scripts and MySQL files from his Splash21 on-rev site > that he > developed and uploaded them to my own on-rev site as a starting > point. You > can find these at > http://splash21.on-rev.com > You have to login to see the downloads link. > Jim Ault Las Vegas From dixonja at hotmail.co.uk Sat Jul 23 10:45:22 2011 From: dixonja at hotmail.co.uk (John Dixon) Date: Sat, 23 Jul 2011 15:45:22 +0100 Subject: on-Rev client software In-Reply-To: <7BB47C2B-DFF8-4A3E-BD96-8E6394A6147D@yahoo.com> References: <4E280ED4.90502@crcom.net>, <1311426913309-3688862.post@n4.nabble.com>, <7BB47C2B-DFF8-4A3E-BD96-8E6394A6147D@yahoo.com> Message-ID: Hi Jim... I think that you are a little mixed up here..:-) splash21 is 'John Craig' of mobGUI fame and he is also responsible for the website http://splash21.on-rev.com/... He does indeed live fairly near Edinburgh, in Paisley which is closer to Glasgow... be well > Just a note about Splash21 > > The author's name is Scott McDonald, and his web site http://splash21.on-rev.com/ > John Craig is the author of MobGUI for LC mobile app programmers. > > By the way, John is a Scot, living in or near Edinburgh, > and the Splash21 Scott lives in Australia. > Jim Ault > Las Vegas From gwendalwood at cox.net Sat Jul 23 12:19:57 2011 From: gwendalwood at cox.net (gwendalwood at cox.net) Date: Sat, 23 Jul 2011 9:19:57 -0700 Subject: iOS files Message-ID: <20110723121957.RCJIP.1318946.imail@fed1rmwml33> Anyone know how to read and write files in an iOS application running on an iPad 2? Also, does anyone know how to print from within the iOS application? Any hints would be appreciated. My routines work in the simulator for reading and writing files, but don't work on the iPad 2. From bvg at mac.com Sat Jul 23 12:55:29 2011 From: bvg at mac.com (=?iso-8859-1?Q?Bj=F6rnke_von_Gierke?=) Date: Sat, 23 Jul 2011 18:55:29 +0200 Subject: [ANN] LiveCode.tv event #33 In-Reply-To: <497CC7C9-74BC-419C-9785-CEA07A8FE181@mac.com> References: <6B865405-BBC0-491C-AD25-BFB1C85A2D58@mac.com> <7EC6A85B-DD6B-4138-A597-512A194F4453@mac.com> <89782C22-C6E3-4245-95D1-A48F69423753@mac.com> <061D9182-B34B-4ABB-A7D6-9C2D09DA8755@mac.com> <73F84954-890B-4627-859A-702C8054F13B@mac.com> <093DEF5E-6C0F-4924-A262-F6DBE748ED45@mac.com> <3FB852C9-5675-4A19-9506-543733BB2547@mac.com> <914DF999-596E-4023-A213-9469C6A69FA0@mac.com> <2B105965-FB27-41D0-B95F-1F4C84ADD53B@mac.com> <163225D1-67D1-4CE7-8049-E85A8D94D177@mac.com> <225B0941-5D11-434A-BC0B-CD61B998E9F8@mac.com> <201756EB-9601-417D-856E-5128C5256EF1@mac.com> <081FD717-0A48-447D-90AA-A8F370B14F43@mac.com> <913B7E78-3052-4CF0-9883-C1CDD9F8BDFA@mac.com> <0C1B6D89-1745-46DF-9C49-A5FE2E5B67B9@mac.com> <497CC7C9-74BC-419C-9785-CEA07A8FE181@mac.com> Message-ID: Dearest friends there is but one hour to go will you watch? On 23 Jul 2011, at 03:01, Bj?rnke von Gierke wrote: > I invite you heartily to join us tomorrow and watch Mario and David. Simply join ChatRev, and win an eBook about Mac OS X Lion: > Go to > http://bjoernke.com/chatrev/ > or enter in the message box: > go stack URL ?http://bjoernke.com/chatrev/chatrev1.3b3.rev? > > This Saturday at 20:00 in Monaco: > Sat. 22:00 Moscow > Sat. 14:00 New York > Sat. 11:00 Los Angeles > Sun. 2:00 Beijing > > Mario Miele will show us his style editing and storing palette. It allows him to grab the styles and visual properties of any object, and store them for later re-usal. > http://livecode.tv/mario/ > > Between shows, the European HyperCard User Group (eHUG, http://www.ehug.info) will raffle off an e-book courtesy of TidBITS. This time around it?s ?Take Control of Using Lion?. > You can find more info on the TC series here: > http://www.takecontrolbooks.com > > David Bovill will wrap up the night with a look at his work on version control framework, which he aims to release soon, and makes collaboratively working on the same stack with several people automatically work. > http://livecode.tv/david/ > > cheers > Bj?rnke > > > >> On 15 Jul 2011, at 04:12, Bj?rnke von Gierke wrote: >> >>> Join us once again, when we show LiveCode at it's best. >>> >>> This Saturday, Pete Haworth will update us on his qd[SQL] development. qd[SQL] is a currently in development, and will allow you to easily create databases without using any SQL at all, by linking LiveCodes object/group/card metaphor to a backend Database automatically. >>> >>> See also his explanation on Google Docs: https://docs.google.com/document/d/1Rl7B3JJrnmOGctXqd-OL26a37jN10cVD43lujyF26uE/edit?hl=en_US >>> >>> Then, the European HyperCard User Group (eHUG, http://www.ehug.info) will raffle off an e-book courtesy of TidBITS. This time around it?s ?Take Control of TextExpander?. >>> You can find more info on the TC series here: >>> http://www.takecontrolbooks.com >>> >>> Finally, Bj?rnke von Gierke will take you on a tour trough some of the add-ons from the 3 for 2 special deal ( http://www.runrev.com/mailers/3for2/3for2.html ), currently offered on the RunRev store with the coupon code "THREEFORTWO". Some of the creator of those plugins will also be available in chat, so keep those questions ready. >>> >>> Don't forget that you can vote on which plug-in you are interested in: >>> http://blog.livecode.tv/2011/07/poll-for-event-32/ >>> >>> >>> All this and more on Saturdays Livecode.tv show: >>> Sat. 22:00 Moscow >>> Sat. 20:00 Paris >>> Sat. 19:00h GMT >>> Sat. 14:00 New York >>> Sat. 11:00 Los Angeles >>> Sun. 04:00 Sydney >>> Sun. 03:00 Tokyo >>> Sun. 2:00 Beijing >>> >>> Make sure to join ChatRev on the right time, otherwise you won?t know where to watch, because the correct streams will be announced in ChatRev. ChatRev is open 24/7 for your chatting needs. >>> Go to >>> http://bjoernke.com/chatrev/ >>> or enter in the message box: >>> go stack URL ?http://bjoernke.com/chatrev/chatrev1.3b3.rev? >>> >>> If you would like to show something in an event, now or somewhen, please head over to the participation page for ideas, or contact us: >>> http://blog.livecode.tv/participate/ >>> http://www3.economy-x-talk.com/file.php?node=contact >>> >>> cheers >>> Bj?rnke >>> _______________________________________________ >>> use-livecode mailing list >>> use-livecode at lists.runrev.com >>> Please visit this url to subscribe, unsubscribe and manage your subscription preferences: >>> http://lists.runrev.com/mailman/listinfo/use-livecode >> >> >> _______________________________________________ >> use-livecode mailing list >> use-livecode at lists.runrev.com >> Please visit this url to subscribe, unsubscribe and manage your subscription preferences: >> http://lists.runrev.com/mailman/listinfo/use-livecode > > > _______________________________________________ > use-livecode mailing list > use-livecode at lists.runrev.com > Please visit this url to subscribe, unsubscribe and manage your subscription preferences: > http://lists.runrev.com/mailman/listinfo/use-livecode From roger.e.eller at sealedair.com Sat Jul 23 13:02:18 2011 From: roger.e.eller at sealedair.com (Roger Eller) Date: Sat, 23 Jul 2011 13:02:18 -0400 Subject: [ANN] LiveCode.tv event #33 In-Reply-To: References: <6B865405-BBC0-491C-AD25-BFB1C85A2D58@mac.com> <7EC6A85B-DD6B-4138-A597-512A194F4453@mac.com> <89782C22-C6E3-4245-95D1-A48F69423753@mac.com> <061D9182-B34B-4ABB-A7D6-9C2D09DA8755@mac.com> <73F84954-890B-4627-859A-702C8054F13B@mac.com> <093DEF5E-6C0F-4924-A262-F6DBE748ED45@mac.com> <3FB852C9-5675-4A19-9506-543733BB2547@mac.com> <914DF999-596E-4023-A213-9469C6A69FA0@mac.com> <2B105965-FB27-41D0-B95F-1F4C84ADD53B@mac.com> <163225D1-67D1-4CE7-8049-E85A8D94D177@mac.com> <225B0941-5D11-434A-BC0B-CD61B998E9F8@mac.com> <201756EB-9601-417D-856E-5128C5256EF1@mac.com> <081FD717-0A48-447D-90AA-A8F370B14F43@mac.com> <913B7E78-3052-4CF0-9883-C1CDD9F8BDFA@mac.com> <0C1B6D89-1745-46DF-9C49-A5FE2E5B67B9@mac.com> <497CC7C9-74BC-419C-9785-CEA07A8FE181@mac.com> Message-ID: 2011/7/23 Bj?rnke von Gierke > Dearest friends > > there is but one hour to go > > will you watch? > > Perhaps. ... ... ... ... ... ... OK! Yes! :-D ?Roger From jacque at hyperactivesw.com Sat Jul 23 14:46:53 2011 From: jacque at hyperactivesw.com (J. Landman Gay) Date: Sat, 23 Jul 2011 13:46:53 -0500 Subject: [ANN] New plugin AAG|LayerComps In-Reply-To: <104176573546.20110722202601@ahsoftware.net> References: <4E29EAE8.6050302@hyperactivesw.com> <63167200328.20110722174948@ahsoftware.net> <4E2A36B8.10906@hyperactivesw.com> <104176573546.20110722202601@ahsoftware.net> Message-ID: <4E2B171D.8090402@hyperactivesw.com> On 7/22/11 10:26 PM, Mark Wieder wrote: > Jacque- > > Friday, July 22, 2011, 7:49:28 PM, you wrote: > >> On 7/22/11 7:49 PM, Mark Wieder wrote: >>> Jacque- >>> >>> Friday, July 22, 2011, 2:26:00 PM, you wrote: >>> >>>> You can set the destroystack property for newly-created stacks in the >>>> Files and Memory section of prefs. >>> >>> Unless you create stacks by script, in which case the destroyStack >>> preference is ignored... sad but true... >>> > >> For real? But...I do that all the time. I guess I never actually checked >> on it, but I don't remember the files hanging around in memory either. >> Is that some new glitch? Does it also happen in standalones? (I wonder >> how you'd check that in a standalone.) > > BZ #3190 (17 October 2005), but as Mark Waddingham attempts to explain > "This isn't actually a bug." > I see. I think I agree with Mark, but in any case it doesn't affect my code. I always script the destroystack property specifically after creating a new stack in code, and I almost always do it in a standalone (to make a user prefs stack usually.) It's good to know about it though. -- Jacqueline Landman Gay | jacque at hyperactivesw.com HyperActive Software | http://www.hyperactivesw.com From jacque at hyperactivesw.com Sat Jul 23 14:48:08 2011 From: jacque at hyperactivesw.com (J. Landman Gay) Date: Sat, 23 Jul 2011 13:48:08 -0500 Subject: OT: It's Android Jim, but not as we know it (what CPUs does Livecode compile to?) In-Reply-To: References: <4E289BD8.9080908@hyperactivesw.com> <4E28C855.5060902@hyperactivesw.com> <4E29C93D.8010303@hyperactivesw.com> Message-ID: <4E2B1768.4020207@hyperactivesw.com> On 7/23/11 3:19 AM, Bernard Devlin wrote: > Jacque, that sounds like the easiest option in order for me to test > the device in-store (but at this stage I'm feeling very confident it > will work to run Livecode apps). If you could put a simple 'hello > world' on to dropbox and send me the URL off list, then I'll give it a > go. Sure, no problem. Will write off-list. -- Jacqueline Landman Gay | jacque at hyperactivesw.com HyperActive Software | http://www.hyperactivesw.com From gerry.orkin at gmail.com Sat Jul 23 19:30:00 2011 From: gerry.orkin at gmail.com (Gerry Orkin) Date: Sun, 24 Jul 2011 09:30:00 +1000 Subject: iOS files In-Reply-To: <20110723121957.RCJIP.1318946.imail@fed1rmwml33> References: <20110723121957.RCJIP.1318946.imail@fed1rmwml33> Message-ID: Post your code... Gerry -- Sent from my iPhone. On 24/07/2011, at 2:19 AM, wrote: > Anyone know how to read and write files in an iOS application running on an iPad 2? > Also, does anyone know how to print from within the iOS application? > Any hints would be appreciated. My routines work in the simulator for reading and writing files, but don't work on the iPad 2. > > _______________________________________________ > use-livecode mailing list > use-livecode at lists.runrev.com > Please visit this url to subscribe, unsubscribe and manage your subscription preferences: > http://lists.runrev.com/mailman/listinfo/use-livecode From mwieder at ahsoftware.net Sat Jul 23 19:43:09 2011 From: mwieder at ahsoftware.net (Mark Wieder) Date: Sat, 23 Jul 2011 16:43:09 -0700 Subject: [ANN] New plugin AAG|LayerComps In-Reply-To: <4E2B171D.8090402@hyperactivesw.com> References: <4E29EAE8.6050302@hyperactivesw.com> <63167200328.20110722174948@ahsoftware.net> <4E2A36B8.10906@hyperactivesw.com> <104176573546.20110722202601@ahsoftware.net> <4E2B171D.8090402@hyperactivesw.com> Message-ID: <177249600890.20110723164309@ahsoftware.net> Jacque- Saturday, July 23, 2011, 11:46:53 AM, you wrote: > On 7/22/11 10:26 PM, Mark Wieder wrote: >> Jacque- >> >> Friday, July 22, 2011, 7:49:28 PM, you wrote: >> >>> On 7/22/11 7:49 PM, Mark Wieder wrote: >>>> Jacque- >>>> >>>> Friday, July 22, 2011, 2:26:00 PM, you wrote: >>>> >>>>> You can set the destroystack property for newly-created stacks in the >>>>> Files and Memory section of prefs. >>>> >>>> Unless you create stacks by script, in which case the destroyStack >>>> preference is ignored... sad but true... >>>> >> >>> For real? But...I do that all the time. I guess I never actually checked >>> on it, but I don't remember the files hanging around in memory either. >>> Is that some new glitch? Does it also happen in standalones? (I wonder >>> how you'd check that in a standalone.) >> >> BZ #3190 (17 October 2005), but as Mark Waddingham attempts to explain >> "This isn't actually a bug." >> > I see. I think I agree with Mark, but in any case it doesn't affect my > code. I always script the destroystack property specifically after > creating a new stack in code, and I almost always do it in a standalone > (to make a user prefs stack usually.) It's good to know about it though. You can also set the destroyStack of the templateStack before creating a script and get the same result. Still, I do wish stack creation would respect the IDE preferences setting so that we wouldn't end up with little surprises like this. -- -Mark Wieder mwieder at ahsoftware.net From jperryl at ecs.fullerton.edu Sat Jul 23 19:57:18 2011 From: jperryl at ecs.fullerton.edu (Judy Perry) Date: Sat, 23 Jul 2011 16:57:18 -0700 (PDT) Subject: Mac OS X Lion In-Reply-To: <0C9AA6AE-8F7B-4707-8367-261B415BC4F2@twft.com> References: <4E271A6B.7000509@fourthworld.com> <26629F3A-B9AB-4C69-95B2-53AB8782E68C@twft.com> <0C9AA6AE-8F7B-4707-8367-261B415BC4F2@twft.com> Message-ID: I hope so :-) Thanks! (Have been on vacation in Florida) Judy On Thu, 21 Jul 2011, Bob Sneidar wrote: > Let be burn you copies of what I have. Will .img files work for you? From ambassador at fourthworld.com Sat Jul 23 20:59:03 2011 From: ambassador at fourthworld.com (Richard Gaskin) Date: Sat, 23 Jul 2011 17:59:03 -0700 Subject: [ANN] New plugin AAG|LayerComps In-Reply-To: <177249600890.20110723164309@ahsoftware.net> References: <177249600890.20110723164309@ahsoftware.net> Message-ID: <4E2B6E57.2080907@fourthworld.com> Mark Wieder wrote: > You can also set the destroyStack of the templateStack before creating > a script and get the same result. Still, I do wish stack creation > would respect the IDE preferences setting so that we wouldn't end up > with little surprises like this. If your own scripts contain stacks that create other stacks, where would it find your LiveCode prefs when they become a standalone? If the IDE set the template objects rather than setting properties on the created stack, it would be way too easy for your own scripts to have different behaviors between development and runtime, opening up the possibility of confusion and error. As it is, if I understand this correctly the IDE leaves the engine defaults alone, which is what your script will be working with in a standalone. Like my t-shirt sez: Know the engine. Trust the engine. Use the engine. :) -- Richard Gaskin Fourth World LiveCode training and consulting: http://www.fourthworld.com Webzine for LiveCode developers: http://www.LiveCodeJournal.com LiveCode Journal blog: http://LiveCodejournal.com/blog.irv From mwieder at ahsoftware.net Sat Jul 23 21:16:09 2011 From: mwieder at ahsoftware.net (Mark Wieder) Date: Sat, 23 Jul 2011 18:16:09 -0700 Subject: [ANN] New plugin AAG|LayerComps In-Reply-To: <4E2B6E57.2080907@fourthworld.com> References: <177249600890.20110723164309@ahsoftware.net> <4E2B6E57.2080907@fourthworld.com> Message-ID: <173255180750.20110723181609@ahsoftware.net> Richard- Saturday, July 23, 2011, 5:59:03 PM, you wrote: > If your own scripts contain stacks that create other stacks, where would > it find your LiveCode prefs when they become a standalone? Uh oh... busted... good point. > Like my t-shirt sez: > Know the engine. > Trust the engine. > Use the engine. "My engine, right or wrong?" -- -Mark Wieder mwieder at ahsoftware.net From brami.serge at gmail.com Sun Jul 24 05:21:31 2011 From: brami.serge at gmail.com (Serge Brami) Date: Sun, 24 Jul 2011 11:21:31 +0200 Subject: pasting text from revbrowser In-Reply-To: <67BA5C1B-2ABD-4181-A8F6-DBC46D7A5536@yahoo.com> References: <21BA7EEF-24B5-4F63-A0A5-4E3F5D919F44@gmail.com> <67BA5C1B-2ABD-4181-A8F6-DBC46D7A5536@yahoo.com> Message-ID: <35B0C3DC-C2F2-479D-8025-0996E685BCB0@gmail.com> Thanks platform = Mac OSX snow leopard checkreturns return only Found char 10 a return in rev/unix but if I replace numtochar(10) with CRLF or numtochar(13) or return it doesnt change anything ... Le 23 juil. 2011 ? 16:03, Jim Ault a ?crit : > On Jul 23, 2011, at 3:48 AM, Serge Brami wrote: >> I try to get the contents of a revbrowser window and paste it into a livecode field >> the web page is a banking chart >> >> first I select manually the text in the revbrowser window >> >> Then I Use revbrowserGet selected put the contents into myvariable and paste it into a field (using :set the htmltext of field myfield to myvariable) >> the problem is that all the return characters disappears >> >> Even if i copy manually the text and paste it directly in a field i get the same thing >> If I copy the text manually and paste it into a Word processor software it works fine the return characters don't disappear > > You have not specified which platform, but try this test > > on checkReturns > if the clipboardData contains numtochar(13) then > answer "Found char 13, a return on a Mac" > end if > if the clipboardData contains numtochar(10) then > answer "Found char 10, a return in Rev/ Unix" > end if > if the clipboardData contains CRLF then > answer "Found char 1310, a return-line feed in Windows" > end if > if the clipboardData contains " answer "Found end if > if the clipboardData contains " answer "Found char end if > end checkReturns > > Also, look up CRLF in the dictionary. > Another factor in Html is that the
may be a tag that is used to create the effect of a return char on the web page. > > A word processor program is designed to do many conversions for the user's convenience, such as clipboard conversions of text and graphics. > > Hope this helps. > >> >> > > Jim Ault > Las Vegas > > > > _______________________________________________ > use-livecode mailing list > use-livecode at lists.runrev.com > Please visit this url to subscribe, unsubscribe and manage your subscription preferences: > http://lists.runrev.com/mailman/listinfo/use-livecode From klaus at major.on-rev.com Sun Jul 24 07:53:18 2011 From: klaus at major.on-rev.com (Klaus on-rev) Date: Sun, 24 Jul 2011 13:53:18 +0200 Subject: OT: on-rev account ftp server error 552 disk full??? Message-ID: <8650AA31-E936-49D5-A114-D50E82902462@major.on-rev.com> Hi friends, I tried to upload some files to my on-rev account via FTP a couple of minutes ago and got this strange server response: error 552 - Disk full, please upload later DISK FULL??? I updated my system to OS X 10.7 two days ago, but FTP did work all day yesterday! Checked my account via the cPanel, lots of disk space of course. Any hints very welcome! Best Klaus -- Klaus Major http://www.major-k.de klaus at major-k.de -- Klaus Major http://www.major-k.de klaus at major.on-rev.com From ambassador at fourthworld.com Sun Jul 24 10:07:14 2011 From: ambassador at fourthworld.com (Richard Gaskin) Date: Sun, 24 Jul 2011 07:07:14 -0700 Subject: [ANN] New plugin AAG|LayerComps In-Reply-To: <173255180750.20110723181609@ahsoftware.net> References: <173255180750.20110723181609@ahsoftware.net> Message-ID: <4E2C2712.60606@fourthworld.com> Mark Wieder: > Richard Gaskin wrote: >> Like my t-shirt sez: > >> Know the engine. >> Trust the engine. >> Use the engine. > > "My engine, right or wrong?" That'll be my new t-shirt if LiveCode goes open source and we can actually tailor the engine. ;) -- Richard Gaskin Fourth World LiveCode training and consulting: http://www.fourthworld.com Webzine for LiveCode developers: http://www.LiveCodeJournal.com LiveCode Journal blog: http://LiveCodejournal.com/blog.irv From slava at lexiconbridge.com Sun Jul 24 10:52:36 2011 From: slava at lexiconbridge.com (Slava Paperno) Date: Sun, 24 Jul 2011 10:52:36 -0400 Subject: pasting text from revbrowser In-Reply-To: <35B0C3DC-C2F2-479D-8025-0996E685BCB0@gmail.com> References: <21BA7EEF-24B5-4F63-A0A5-4E3F5D919F44@gmail.com> <67BA5C1B-2ABD-4181-A8F6-DBC46D7A5536@yahoo.com> <35B0C3DC-C2F2-479D-8025-0996E685BCB0@gmail.com> Message-ID: <002601cc4a11$57086850$051938f0$@lexiconbridge.com> What happens if you replace char(10) with the string "
"? Slava > -----Original Message----- > From: use-livecode-bounces at lists.runrev.com [mailto:use-livecode- > bounces at lists.runrev.com] On Behalf Of Serge Brami > Sent: Sunday, July 24, 2011 5:22 AM > To: How to use LiveCode > Subject: Re: pasting text from revbrowser > > Thanks > platform = Mac OSX snow leopard > > checkreturns return only Found char 10 a return in rev/unix > > but if I replace numtochar(10) with CRLF or numtochar(13) or return it doesnt > change anything ... > > Le 23 juil. 2011 ? 16:03, Jim Ault a ?crit : > > > On Jul 23, 2011, at 3:48 AM, Serge Brami wrote: > >> I try to get the contents of a revbrowser window and paste it into > >> a livecode field the web page is a banking chart > >> > >> first I select manually the text in the revbrowser window > >> > >> Then I Use revbrowserGet selected put the contents into myvariable and > paste it into a field (using :set the htmltext of field myfield to myvariable) > >> the problem is that all the return characters disappears > >> > >> Even if i copy manually the text and paste it directly in a field i get the same > thing > >> If I copy the text manually and paste it into a Word processor software it > works fine the return characters don't disappear > > > > You have not specified which platform, but try this test > > > > on checkReturns > > if the clipboardData contains numtochar(13) then > > answer "Found char 13, a return on a Mac" > > end if > > if the clipboardData contains numtochar(10) then > > answer "Found char 10, a return in Rev/ Unix" > > end if > > if the clipboardData contains CRLF then > > answer "Found char 1310, a return-line feed in Windows" > > end if > > if the clipboardData contains " > answer "Found > end if > > if the clipboardData contains " > answer "Found char > end if > > end checkReturns > > > > Also, look up CRLF in the dictionary. > > Another factor in Html is that the
may be a tag that is used to create the > effect of a return char on the web page. > > > > A word processor program is designed to do many conversions for the user's > convenience, such as clipboard conversions of text and graphics. > > > > Hope this helps. > > > >> > >> > > > > Jim Ault > > Las Vegas > > > > > > > > _______________________________________________ > > use-livecode mailing list > > use-livecode at lists.runrev.com > > Please visit this url to subscribe, unsubscribe and manage your subscription > preferences: > > http://lists.runrev.com/mailman/listinfo/use-livecode > > > _______________________________________________ > use-livecode mailing list > use-livecode at lists.runrev.com > Please visit this url to subscribe, unsubscribe and manage your subscription > preferences: > http://lists.runrev.com/mailman/listinfo/use-livecode From mikekann at yahoo.com Sun Jul 24 10:56:33 2011 From: mikekann at yahoo.com (Michael Kann) Date: Sun, 24 Jul 2011 07:56:33 -0700 (PDT) Subject: OT: on-rev account ftp server error 552 disk full??? In-Reply-To: <8650AA31-E936-49D5-A114-D50E82902462@major.on-rev.com> Message-ID: <1311519393.5965.YahooMailClassic@web161616.mail.bf1.yahoo.com> Klaus, I've been uploading using Filezilla the last few minutes. No problema. Mike --- On Sun, 7/24/11, Klaus on-rev wrote: From: Klaus on-rev Subject: OT: on-rev account ftp server error 552 disk full??? To: "How to use LiveCode" Date: Sunday, July 24, 2011, 6:53 AM Hi friends, I tried to upload some files to my on-rev account via FTP a couple of minutes ago and got this strange server response: error 552 - Disk full, please upload later DISK FULL??? I updated my system to OS X? 10.7 two days ago, but FTP did work all day yesterday! Checked my account via the cPanel, lots of disk space of course. Any hints very welcome! Best Klaus -- Klaus Major http://www.major-k.de klaus at major-k.de -- Klaus Major http://www.major-k.de klaus at major.on-rev.com _______________________________________________ use-livecode mailing list use-livecode at lists.runrev.com Please visit this url to subscribe, unsubscribe and manage your subscription preferences: http://lists.runrev.com/mailman/listinfo/use-livecode From klaus at major.on-rev.com Sun Jul 24 11:04:01 2011 From: klaus at major.on-rev.com (Klaus on-rev) Date: Sun, 24 Jul 2011 17:04:01 +0200 Subject: OT: on-rev account ftp server error 552 disk full??? In-Reply-To: <1311519393.5965.YahooMailClassic@web161616.mail.bf1.yahoo.com> References: <1311519393.5965.YahooMailClassic@web161616.mail.bf1.yahoo.com> Message-ID: Hi Michael Am 24.07.2011 um 16:56 schrieb Michael Kann: > Klaus, > I've been uploading using Filezilla the last few minutes. No problema. Good for you :-) Still the same problem here either using a FTP app like Captain FTP or a pure LiveCode one-liner! I can download things via FTP however, but always get the "Disk full" error when trying to upload even the smallest text file :-/ Too funky! I'm sure the error text is misleading (DISK FULL? Come on! :-D, but what can I do? I already wrote to support. > Mike Best Klaus -- Klaus Major http://www.major-k.de klaus at major.on-rev.com From matthias_livecode at me.com Sun Jul 24 11:57:14 2011 From: matthias_livecode at me.com (Matthias Rebbe) Date: Sun, 24 Jul 2011 17:57:14 +0200 Subject: OT: on-rev account ftp server error 552 disk full??? In-Reply-To: References: <1311519393.5965.YahooMailClassic@web161616.mail.bf1.yahoo.com> Message-ID: Hi Klaus, > I'm sure the error text is misleading (DISK FULL? Come on! :-D, but what can I do? > I already wrote to support. Did you put a URGENT at the beginning of the subject line? The word urgent ensures that your support request is handled with a higher priority. But keep in mind to use it only for problems, where the server seems to m?sbehave. And your problem seems to be part of such misbehave. Regards, Matthias From slava at lexiconbridge.com Sun Jul 24 12:10:18 2011 From: slava at lexiconbridge.com (Slava Paperno) Date: Sun, 24 Jul 2011 12:10:18 -0400 Subject: LC equivalent of the JS eval() function Message-ID: <002201cc4a1c$327240f0$9756c2d0$@com> I'm looking for the LC equivalent to the JavaScript eval() function. For example, if I have a variable called "tAddress" and I want an LC statement to assign a value to it, I'd like to say something like put "202 Main Street" into eval("tAddress"). That may sound silly because if I, the programmer, know the name of the variable, I should say put "202 Main Street" into tAddress. But I don't know the name of the variable because it will be generated by a handler. For example, if my handler reads a name-value pair from some file: LastKnownAddress="202 Main Street" then I want to create a variable called LastKnownAddress and set its value to "202 Main Street". In JS, I would use eval(). What can I use in LC? It's hard to find things like that in the docs when you don't know what they are called. Thanks! Slava From m.schonewille at economy-x-talk.com Sun Jul 24 12:20:10 2011 From: m.schonewille at economy-x-talk.com (Mark Schonewille) Date: Sun, 24 Jul 2011 18:20:10 +0200 Subject: LC equivalent of the JS eval() function In-Reply-To: <002201cc4a1c$327240f0$9756c2d0$@com> References: <002201cc4a1c$327240f0$9756c2d0$@com> Message-ID: <3A3A0FC9-F221-490A-8B2E-89981C5D3D98@economy-x-talk.com> Hi Slava, The do command and the value function could be what you are looking for. -- Best regards, Mark Schonewille Economy-x-Talk Consulting and Software Engineering Homepage: http://economy-x-talk.com Twitter: http://twitter.com/xtalkprogrammer KvK: 50277553 What does that error mean? Buy LiveCodeErrors for iPhone now http://qery.us/v4 A must-have for LiveCode programmers. On 24 jul 2011, at 18:10, Slava Paperno wrote: > I'm looking for the LC equivalent to the JavaScript eval() function. For > example, if I have a variable called "tAddress" and I want an LC statement > to assign a value to it, I'd like to say something like put "202 Main > Street" into eval("tAddress"). > > That may sound silly because if I, the programmer, know the name of the > variable, I should say put "202 Main Street" into tAddress. But I don't know > the name of the variable because it will be generated by a handler. For > example, if my handler reads a name-value pair from some file: > > LastKnownAddress="202 Main Street" > > then I want to create a variable called LastKnownAddress and set its value > to "202 Main Street". > > In JS, I would use eval(). What can I use in LC? It's hard to find things > like that in the docs when you don't know what they are called. > > Thanks! > > Slava > > From ambassador at fourthworld.com Sun Jul 24 12:25:51 2011 From: ambassador at fourthworld.com (Richard Gaskin) Date: Sun, 24 Jul 2011 09:25:51 -0700 Subject: LC equivalent of the JS eval() function In-Reply-To: <002201cc4a1c$327240f0$9756c2d0$@com> References: <002201cc4a1c$327240f0$9756c2d0$@com> Message-ID: <4E2C478F.6020400@fourthworld.com> Slava Paperno wrote: > I'm looking for the LC equivalent to the JavaScript eval() function. For > example, if I have a variable called "tAddress" and I want an LC statement > to assign a value to it, I'd like to say something like put "202 Main > Street" into eval("tAddress"). > > That may sound silly because if I, the programmer, know the name of the > variable, I should say put "202 Main Street" into tAddress. But I don't know > the name of the variable because it will be generated by a handler. As Mark suggested, the "do" command or the "value" function will do what you need. But this also seems a perfect fit for an array. Arrays are a great solution when you don't know the number of elements you'll need to store, or will need to access them by names you can't know in advance, e.g.: put "202 Main Street" into tMyData[tAddress] put "Slava Paperno" into tMyData[tName] -- Richard Gaskin Fourth World LiveCode training and consulting: http://www.fourthworld.com Webzine for LiveCode developers: http://www.LiveCodeJournal.com LiveCode Journal blog: http://LiveCodejournal.com/blog.irv From andre at andregarzia.com Sun Jul 24 12:38:09 2011 From: andre at andregarzia.com (Andre Garzia) Date: Sun, 24 Jul 2011 13:38:09 -0300 Subject: LC equivalent of the JS eval() function In-Reply-To: <4E2C478F.6020400@fourthworld.com> References: <002201cc4a1c$327240f0$9756c2d0$@com> <4E2C478F.6020400@fourthworld.com> Message-ID: don't forget merge... put the merge of "[[tAddress]]" into theContent On Sun, Jul 24, 2011 at 1:25 PM, Richard Gaskin wrote: > Slava Paperno wrote: > > I'm looking for the LC equivalent to the JavaScript eval() function. For >> example, if I have a variable called "tAddress" and I want an LC statement >> to assign a value to it, I'd like to say something like put "202 Main >> Street" into eval("tAddress"). >> >> That may sound silly because if I, the programmer, know the name of the >> variable, I should say put "202 Main Street" into tAddress. But I don't >> know >> the name of the variable because it will be generated by a handler. >> > > As Mark suggested, the "do" command or the "value" function will do what > you need. > > But this also seems a perfect fit for an array. > > Arrays are a great solution when you don't know the number of elements > you'll need to store, or will need to access them by names you can't know in > advance, e.g.: > > put "202 Main Street" into tMyData[tAddress] > put "Slava Paperno" into tMyData[tName] > > -- > Richard Gaskin > Fourth World > LiveCode training and consulting: http://www.fourthworld.com > Webzine for LiveCode developers: http://www.LiveCodeJournal.com > LiveCode Journal blog: http://LiveCodejournal.com/**blog.irv > > > ______________________________**_________________ > use-livecode mailing list > use-livecode at lists.runrev.com > Please visit this url to subscribe, unsubscribe and manage your > subscription preferences: > http://lists.runrev.com/**mailman/listinfo/use-livecode > -- http://www.andregarzia.com All We Do Is Code. From eriks at sisyph.us Sun Jul 24 12:39:33 2011 From: eriks at sisyph.us (Erik Schwartz) Date: Sun, 24 Jul 2011 09:39:33 -0700 Subject: on-rev client Message-ID: Has the on-rev client broken? I can edit .irev files directly on the server, but if I edit them with the local client and try to commit them from the client the changes are never uploaded. -- ========================================== eriks at sisyph.us? ? ? ? ? ? ? ? ? ? ? ? ? ? ? ? ? http://sisyph.us (530) 213-ERIK? ? ? ? ? ? ? ? ? ? ? ? ? http://twitter.com/eriks (530) 213-3745?? http://www.linkedin.com/in/erikschwartz ========================================== From kray at sonsothunder.com Sun Jul 24 12:50:46 2011 From: kray at sonsothunder.com (Ken Ray) Date: Sun, 24 Jul 2011 11:50:46 -0500 Subject: LC equivalent of the JS eval() function In-Reply-To: <3A3A0FC9-F221-490A-8B2E-89981C5D3D98@economy-x-talk.com> References: <002201cc4a1c$327240f0$9756c2d0$@com> <3A3A0FC9-F221-490A-8B2E-89981C5D3D98@economy-x-talk.com> Message-ID: On Jul 24, 2011, at 11:20 AM, Mark Schonewille wrote: > Hi Slava, > > The do command and the value function could be what you are looking for. And just to be clear on use, it would be like this: put "MyCustomVariable" into tVarToFill put "202 Main Street" into tStreet do "put tStreet into" && tVarToFill put MyCustomVariable --> 202 Main Street Hope this helps, Ken Ray Sons of Thunder Software, Inc. Email: kray at sonsothunder.com Web Site: http://www.sonsothunder.com/ > On 24 jul 2011, at 18:10, Slava Paperno wrote: > >> I'm looking for the LC equivalent to the JavaScript eval() function. For >> example, if I have a variable called "tAddress" and I want an LC statement >> to assign a value to it, I'd like to say something like put "202 Main >> Street" into eval("tAddress"). >> >> That may sound silly because if I, the programmer, know the name of the >> variable, I should say put "202 Main Street" into tAddress. But I don't know >> the name of the variable because it will be generated by a handler. For >> example, if my handler reads a name-value pair from some file: >> >> LastKnownAddress="202 Main Street" >> >> then I want to create a variable called LastKnownAddress and set its value >> to "202 Main Street". >> >> In JS, I would use eval(). What can I use in LC? It's hard to find things >> like that in the docs when you don't know what they are called. >> >> Thanks! >> >> Slava >> >> > > > _______________________________________________ > use-livecode mailing list > use-livecode at lists.runrev.com > Please visit this url to subscribe, unsubscribe and manage your subscription preferences: > http://lists.runrev.com/mailman/listinfo/use-livecode From kray at sonsothunder.com Sun Jul 24 12:52:29 2011 From: kray at sonsothunder.com (Ken Ray) Date: Sun, 24 Jul 2011 11:52:29 -0500 Subject: on-rev client In-Reply-To: References: Message-ID: <9F333144-824A-4707-B4EA-082288FBFDA7@sonsothunder.com> On Jul 24, 2011, at 11:39 AM, Erik Schwartz wrote: > Has the on-rev client broken? > > I can edit .irev files directly on the server, but if I edit them with > the local client and try to commit them from the client the changes > are never uploaded. I'm not sure, but I know someone said that RunRev would be upgrading the On-Rev server to use the latest LiveCode 4.6.3 Server release... if they are doing it this weekend that might be the reason? Ken Ray Sons of Thunder Software, Inc. Email: kray at sonsothunder.com Web Site: http://www.sonsothunder.com/ From mikekann at yahoo.com Sun Jul 24 12:55:20 2011 From: mikekann at yahoo.com (Michael Kann) Date: Sun, 24 Jul 2011 09:55:20 -0700 (PDT) Subject: on-rev client In-Reply-To: Message-ID: <1311526520.51358.YahooMailClassic@web161614.mail.bf1.yahoo.com> Erik, I use FileZilla to upload files to the on-rev server. One mistake I often make is that I upload a file to the server but I haven't selected the correct directory on the server. The file arrives in the wrong place but I don't realize it. When the webpage I've updated doesn't appear in my browser it looks like the server wasn't updated, but in reality it was updated in the wrong directory. Just something to watch out for. Mike --- On Sun, 7/24/11, Erik Schwartz wrote: From: Erik Schwartz Subject: on-rev client To: "How to use LiveCode" Date: Sunday, July 24, 2011, 11:39 AM Has the on-rev client broken? I can edit .irev files directly on the server, but if I edit them with the local client and try to commit them from the client the changes are never uploaded. -- ========================================== eriks at sisyph.us? ? ? ? ? ? ? ? ? ? ? ? ? ? ? ? ? http://sisyph.us (530) 213-ERIK? ? ? ? ? ? ? ? ? ? ? ? ? http://twitter.com/eriks (530) 213-3745?? http://www.linkedin.com/in/erikschwartz ========================================== _______________________________________________ use-livecode mailing list use-livecode at lists.runrev.com Please visit this url to subscribe, unsubscribe and manage your subscription preferences: http://lists.runrev.com/mailman/listinfo/use-livecode From eriks at sisyph.us Sun Jul 24 12:57:26 2011 From: eriks at sisyph.us (Erik Schwartz) Date: Sun, 24 Jul 2011 09:57:26 -0700 Subject: on-rev client In-Reply-To: <1311526520.51358.YahooMailClassic@web161614.mail.bf1.yahoo.com> References: <1311526520.51358.YahooMailClassic@web161614.mail.bf1.yahoo.com> Message-ID: I'll take a look around. I have not changed anything in my dev environment, but sometimes these things happen by themselves. Thanks, Erik On Sun, Jul 24, 2011 at 9:55 AM, Michael Kann wrote: > Erik, > > I use FileZilla to upload files to the on-rev server. One mistake I often make is that I upload a file to the server but I haven't selected the correct directory on the server. The file arrives in the wrong place but I don't realize it. When the webpage I've updated doesn't appear in my browser it looks like the server wasn't updated, but in reality it was updated in the wrong directory. Just something to watch out for. > -- ========================================== eriks at sisyph.us? ? ? ? ? ? ? ? ? ? ? ? ? ? ? ? ? http://sisyph.us (530) 213-ERIK? ? ? ? ? ? ? ? ? ? ? ? ? http://twitter.com/eriks (530) 213-3745?? http://www.linkedin.com/in/erikschwartz ========================================== From lists.pete at haworths.org Sun Jul 24 14:23:05 2011 From: lists.pete at haworths.org (Pete Haworth) Date: Sun, 24 Jul 2011 11:23:05 -0700 Subject: [ANN] New plugin AAG|LayerComps In-Reply-To: <177249600890.20110723164309@ahsoftware.net> References: <4E29EAE8.6050302@hyperactivesw.com> <63167200328.20110722174948@ahsoftware.net> <4E2A36B8.10906@hyperactivesw.com> <104176573546.20110722202601@ahsoftware.net> <4E2B171D.8090402@hyperactivesw.com> <177249600890.20110723164309@ahsoftware.net> Message-ID: I guess this isn't quite the same thing but why is it that when I create a field by script, the size of the field is different than when I just drag one to my stack from the Tools palette. I thought the templates were supposed to ensure consistency but maybe I'm misunderstanding their purpose Pete On Sat, Jul 23, 2011 at 4:43 PM, Mark Wieder wrote: > Jacque- > > Saturday, July 23, 2011, 11:46:53 AM, you wrote: > > > On 7/22/11 10:26 PM, Mark Wieder wrote: > >> Jacque- > >> > >> Friday, July 22, 2011, 7:49:28 PM, you wrote: > >> > >>> On 7/22/11 7:49 PM, Mark Wieder wrote: > >>>> Jacque- > >>>> > >>>> Friday, July 22, 2011, 2:26:00 PM, you wrote: > >>>> > >>>>> You can set the destroystack property for newly-created stacks in the > >>>>> Files and Memory section of prefs. > >>>> > >>>> Unless you create stacks by script, in which case the destroyStack > >>>> preference is ignored... sad but true... > >>>> > >> > >>> For real? But...I do that all the time. I guess I never actually > checked > >>> on it, but I don't remember the files hanging around in memory either. > >>> Is that some new glitch? Does it also happen in standalones? (I wonder > >>> how you'd check that in a standalone.) > >> > >> BZ #3190 (17 October 2005), but as Mark Waddingham attempts to explain > >> "This isn't actually a bug." > >> > > > I see. I think I agree with Mark, but in any case it doesn't affect my > > code. I always script the destroystack property specifically after > > creating a new stack in code, and I almost always do it in a standalone > > (to make a user prefs stack usually.) It's good to know about it though. > > You can also set the destroyStack of the templateStack before creating > a script and get the same result. Still, I do wish stack creation > would respect the IDE preferences setting so that we wouldn't end up > with little surprises like this. > > -- > -Mark Wieder > mwieder at ahsoftware.net > > > _______________________________________________ > use-livecode mailing list > use-livecode at lists.runrev.com > Please visit this url to subscribe, unsubscribe and manage your > subscription preferences: > http://lists.runrev.com/mailman/listinfo/use-livecode > From dsc at swcp.com Sun Jul 24 14:26:17 2011 From: dsc at swcp.com (Dar Scott) Date: Sun, 24 Jul 2011 12:26:17 -0600 Subject: LC equivalent of the JS eval() function In-Reply-To: <002201cc4a1c$327240f0$9756c2d0$@com> References: <002201cc4a1c$327240f0$9756c2d0$@com> Message-ID: <510B5C65-7208-4B42-8F3F-78C1A4D788F8@swcp.com> People have already mentioned 'do', 'merge()' and using arrays. I'll add these: Set the script of an object. Use a switch statement. Set a chunk of the value of a variable, perhaps a global. In limited situations, 'value()' might be useful. Pass the variable to the function rather than its value using @. Both 'do' and set scripts are limited to 10 lines in standalone applications. Perhaps 'do' is the most direct solution to what you want to do. (I have found 'merge' to be powerful in some cases.) I suspect what you want to do might be done well with a (perhaps global) variable containing an array. Like this: put "202 Main Street" into stuff[ tAddress ] However, maybe the @ fits into your scenario. Put an @ in front of a parameter in a hander (command or function) and when it is called with a variable the variable is passed not the value; use of the parameter is the same as the use of the original variable. This is much like pass-by-reference in Java. Like this: on configureLocation @pAddress, @pCity, @pState, pLongitude, pLatitude ... put "202 Main Street" into pAddress ... end configureLocation (Having--tongue-in-cheek--thrown candy at speakers at a RunRev conference who promoted their "morals" in using variable prefixes, I have to say that the use of such prefixes above does not constitute a promotion of such use, but only exists as an aid in communicating the example.) I wish you well in this. Dar Scott From slava at lexiconbridge.com Sun Jul 24 14:31:10 2011 From: slava at lexiconbridge.com (Slava Paperno) Date: Sun, 24 Jul 2011 14:31:10 -0400 Subject: LC equivalent of the JS eval() function In-Reply-To: References: <002201cc4a1c$327240f0$9756c2d0$@com> <3A3A0FC9-F221-490A-8B2E-89981C5D3D98@economy-x-talk.com> Message-ID: <003201cc4a2f$df649110$9e2db330$@com> Ah, "do"! Great. It's a much better word than "eval." JS should have called it "do" as well because the eval() function does more than return a value--it executes commands, just like the LC "do." Thanks, everyone, Slava > On Jul 24, 2011, at 11:20 AM, Mark Schonewille wrote: > > > Hi Slava, > > > > The do command and the value function could be what you are looking > for... From slava at lexiconbridge.com Sun Jul 24 14:35:38 2011 From: slava at lexiconbridge.com (Slava Paperno) Date: Sun, 24 Jul 2011 14:35:38 -0400 Subject: Preferences folder/files on mobile Message-ID: <003301cc4a30$7f88a910$7e99fb30$@com> Does the special folder "preferences" point to some location on mobile devices? If not, where is one supposed to place a preferences file for an application on a mobile? Is the Windows tradition of naming the preferences file with the "ini" extension upheld on mobiles? A related question: does special folder "home" have any meaning on a mobile? The concept of the "current user" does not exist on a mobile device, does it? Slava From jacque at hyperactivesw.com Sun Jul 24 15:04:43 2011 From: jacque at hyperactivesw.com (J. Landman Gay) Date: Sun, 24 Jul 2011 14:04:43 -0500 Subject: iOS files In-Reply-To: <20110723121957.RCJIP.1318946.imail@fed1rmwml33> References: <20110723121957.RCJIP.1318946.imail@fed1rmwml33> Message-ID: <4E2C6CCB.6050201@hyperactivesw.com> On 7/23/11 11:19 AM, gwendalwood at cox.net wrote: > Anyone know how to read and write files in an iOS application running > on an iPad 2? Also, does anyone know how to print from within the iOS > application? Any hints would be appreciated. My routines work in the > simulator for reading and writing files, but don't work on the iPad > 2. You can read/write to files in iOS but only to specific locations. You can't write to the engine folder, for example, but you can write to documents, cache, or other specific places. File read/write works the same way it does on the desktop as long as you use specialFolderPath() to build the correct path, and you restrict paths to the folders listed in the iOS release notes. If you still have trouble with it, do post your script and we can help. -- Jacqueline Landman Gay | jacque at hyperactivesw.com HyperActive Software | http://www.hyperactivesw.com From jacque at hyperactivesw.com Sun Jul 24 15:50:09 2011 From: jacque at hyperactivesw.com (J. Landman Gay) Date: Sun, 24 Jul 2011 14:50:09 -0500 Subject: Preferences folder/files on mobile In-Reply-To: <003301cc4a30$7f88a910$7e99fb30$@com> References: <003301cc4a30$7f88a910$7e99fb30$@com> Message-ID: <4E2C7771.6090100@hyperactivesw.com> On 7/24/11 1:35 PM, Slava Paperno wrote: > Does the special folder "preferences" point to some location on mobile > devices? No. The release notes on each mobile OS have a list of available folders you can access. > > If not, where is one supposed to place a preferences file for an application > on a mobile? Generally specialFolderPath("documents"), for a couple of reasons. Not only is it one of the folders backed up by iTunes on iOS (all app data is backed up on Android) but also since that folder exists and is writable on both mobile platforms, you don't have to branch your code. > > Is the Windows tradition of naming the preferences file with the "ini" > extension upheld on mobiles? As long as you store your file in one of the approved, writable folders, it doesn't matter what you name it. An "ini" file won't have any particular meaning on mobile however. Since it's difficult to see what other apps use for their file names because of the sandboxing, I don't know if Windows naming conventions are common. It wouldn't matter. > > A related question: does special folder "home" have any meaning on a mobile? > The concept of the "current user" does not exist on a mobile device, does > it? iOS has specialFolderPath("home") though I don't know if it is writable. Android doesn't have a "home" folder; at present it has only three special folder locations and only two are writable. I don't think mobile devices can have more than one user account (can they?) so there wouldn't be a specific "current user". The device itself would be the current user, I suppose. -- Jacqueline Landman Gay | jacque at hyperactivesw.com HyperActive Software | http://www.hyperactivesw.com From maarten.koopmans at gmail.com Sun Jul 24 16:30:10 2011 From: maarten.koopmans at gmail.com (Maarten Koopmans) Date: Sun, 24 Jul 2011 22:30:10 +0200 Subject: OT: gmail two factor authentication Message-ID: All, I just got a nice box from Google explaining that there was suspicious account activity from Mexico (I live in Amsterdam). I changed my password, enabled two-factor authentication using their iPhone app and use app-specific passwords. Also, I changed ALL passwords of all online services I use to new, unique ones. My guess: it took one malicious web page, even though I am up-to-fate with every component.... Anyway, if your a GMail user I strongly suggest switching to their two-factor authentication (meaning that you provide a uniquely generated number every time you login). I mean... if it can happen to a careful user like me who used to work on identity management - it literally can happen to anyone. I knew that - but I did change all my passwords and checked my credit card view (which is luckily "live") after the adrenalin rush. Just thought to share the experience. We all read the horror stories, I can confirm them now first-hand (though little damage has been done so far AFAIK). 1) use unique passwords per service 2) use two factor authN on Google - it's a great add-on to the service --Maarten From slava at lexiconbridge.com Sun Jul 24 16:47:56 2011 From: slava at lexiconbridge.com (Slava Paperno) Date: Sun, 24 Jul 2011 16:47:56 -0400 Subject: Preferences folder/files on mobile In-Reply-To: <4E2C7771.6090100@hyperactivesw.com> References: <003301cc4a30$7f88a910$7e99fb30$@com> <4E2C7771.6090100@hyperactivesw.com> Message-ID: <003b01cc4a42$fac14990$f043dcb0$@com> Thanks for the exhaustive coverage, Jacque! S. > -----Original Message----- > From: use-livecode-bounces at lists.runrev.com [mailto:use-livecode- > bounces at lists.runrev.com] On Behalf Of J. Landman Gay > Sent: Sunday, July 24, 2011 3:50 PM > To: How to use LiveCode > Subject: Re: Preferences folder/files on mobile > > On 7/24/11 1:35 PM, Slava Paperno wrote: > > Does the special folder "preferences" point to some location on > mobile > > devices? > > No. The release notes on each mobile OS have a list of available > folders > you can access. > > > > > If not, where is one supposed to place a preferences file for an > application > > on a mobile? > > Generally specialFolderPath("documents"), for a couple of reasons. Not > only is it one of the folders backed up by iTunes on iOS (all app data > is backed up on Android) but also since that folder exists and is > writable on both mobile platforms, you don't have to branch your code. > > > > > Is the Windows tradition of naming the preferences file with the > "ini" > > extension upheld on mobiles? > > As long as you store your file in one of the approved, writable > folders, > it doesn't matter what you name it. An "ini" file won't have any > particular meaning on mobile however. Since it's difficult to see what > other apps use for their file names because of the sandboxing, I don't > know if Windows naming conventions are common. It wouldn't matter. > > > > > A related question: does special folder "home" have any meaning on a > mobile? > > The concept of the "current user" does not exist on a mobile device, > does > > it? > > iOS has specialFolderPath("home") though I don't know if it is > writable. > Android doesn't have a "home" folder; at present it has only three > special folder locations and only two are writable. > > I don't think mobile devices can have more than one user account (can > they?) so there wouldn't be a specific "current user". The device > itself > would be the current user, I suppose. > > -- > Jacqueline Landman Gay | jacque at hyperactivesw.com > HyperActive Software | http://www.hyperactivesw.com > > _______________________________________________ > use-livecode mailing list > use-livecode at lists.runrev.com > Please visit this url to subscribe, unsubscribe and manage your > subscription preferences: > http://lists.runrev.com/mailman/listinfo/use-livecode From jiml at netrin.com Sun Jul 24 17:59:16 2011 From: jiml at netrin.com (Jim Lambert) Date: Sun, 24 Jul 2011 14:59:16 -0700 Subject: URGENT: RE:on-rev account ftp server error 552 disk full??? In-Reply-To: References: Message-ID: <121BBDE2-4473-45DF-89C9-3CC17752122B@netrin.com> Yes, Klaus I also get 552 on https://freyr.on-rev.com Jim Lambert From roger.e.eller at sealedair.com Sun Jul 24 18:09:16 2011 From: roger.e.eller at sealedair.com (Roger Eller) Date: Sun, 24 Jul 2011 18:09:16 -0400 Subject: OT: gmail two factor authentication In-Reply-To: References: Message-ID: On Sun, Jul 24, 2011 at 4:30 PM, Maarten Koopmans wrote: > All, > > I just got a nice box from Google explaining that there was suspicious > account activity from Mexico (I live in Amsterdam). I changed my > password, enabled two-factor authentication using their iPhone app and > use app-specific passwords. > > Also, I changed ALL passwords of all online services I use to new, > unique ones. My guess: it took one malicious web page, even though I > am up-to-fate with every component.... > > --Maarten > It's always up-to-fate, isn't it? ;-) ?Roger From matthias_livecode at me.com Sun Jul 24 18:26:27 2011 From: matthias_livecode at me.com (Matthias Rebbe) Date: Mon, 25 Jul 2011 00:26:27 +0200 Subject: URGENT: RE:on-rev account ftp server error 552 disk full??? In-Reply-To: <121BBDE2-4473-45DF-89C9-3CC17752122B@netrin.com> References: <121BBDE2-4473-45DF-89C9-3CC17752122B@netrin.com> Message-ID: <5DF145BB-93D9-4562-95B5-F12A900D7627@me.com> Jim, i think you got me wrong. I meant to include the word URGENT in the subject line of the email sent to the on-rev support. Regards, Matthias Am 24.07.2011 um 23:59 schrieb Jim Lambert: > Yes, Klaus I also get 552 on > https://freyr.on-rev.com > > Jim Lambert > > _______________________________________________ > use-livecode mailing list > use-livecode at lists.runrev.com > Please visit this url to subscribe, unsubscribe and manage your subscription preferences: > http://lists.runrev.com/mailman/listinfo/use-livecode From slava at lexiconbridge.com Sun Jul 24 18:32:13 2011 From: slava at lexiconbridge.com (Slava Paperno) Date: Sun, 24 Jul 2011 18:32:13 -0400 Subject: are all controls loaded? Message-ID: <004601cc4a51$8c4acae0$a4e060a0$@com> The openCard handler in the first card of my stack sends messages to various controls on the card, and I think some of these messages are sent before the control is fully loaded: I see error "no such object." If I use "send XXX to YYY in 800 milliseconds," then all is well. How do I solve this dilemma? I need something like an "afterAllControlsAreLoaded" message :) Slava From m.schonewille at economy-x-talk.com Sun Jul 24 18:44:22 2011 From: m.schonewille at economy-x-talk.com (Mark Schonewille) Date: Mon, 25 Jul 2011 00:44:22 +0200 Subject: are all controls loaded? In-Reply-To: <004601cc4a51$8c4acae0$a4e060a0$@com> References: <004601cc4a51$8c4acae0$a4e060a0$@com> Message-ID: <08F843D1-EB52-45DC-8355-3F279367DDE2@economy-x-talk.com> Hi Slava, I have similar problems once in a while. Often, I do this: on preOpenStack hide this stack // do some very important stuff here readPrefs -- (e.g. read registration data) // and send a command in x millisecs send "initialise" to me in 0 millisecs end preOpenStack on initialise put the seconds into mySecs go stack "Splash" in new window // the splash stack contains everything // to set itself up // now we can set up our controls send "foo" to btn 2 put url "http://bla.com" into fld 5 // has the user clicked Splash away? if "Splash" is among the lines of the openStacks then wait until (the seconds - theSecs > 5) or not \ ("Splash" is among the lines of the openStacks) with messages close stack "Splash" end if end initalise Of course, many variations are possible. -- Best regards, Mark Schonewille Economy-x-Talk Consulting and Software Engineering Homepage: http://economy-x-talk.com Twitter: http://twitter.com/xtalkprogrammer KvK: 50277553 What does that error mean? Buy LiveCodeErrors for iPhone now http://qery.us/v4 A must-have for LiveCode programmers. On 25 jul 2011, at 00:32, Slava Paperno wrote: > The openCard handler in the first card of my stack sends messages to various > controls on the card, and I think some of these messages are sent before the > control is fully loaded: I see error "no such object." If I use "send XXX to > YYY in 800 milliseconds," then all is well. > > How do I solve this dilemma? I need something like an > "afterAllControlsAreLoaded" message :) > > Slava > From sarah.reichelt at gmail.com Sun Jul 24 18:51:33 2011 From: sarah.reichelt at gmail.com (Sarah Reichelt) Date: Mon, 25 Jul 2011 08:51:33 +1000 Subject: OT: on-rev account ftp server error 552 disk full??? In-Reply-To: <8650AA31-E936-49D5-A114-D50E82902462@major.on-rev.com> References: <8650AA31-E936-49D5-A114-D50E82902462@major.on-rev.com> Message-ID: > I tried to upload some files to my on-rev account via FTP > a couple of minutes ago and got this strange server response: > error 552 - Disk full, please upload later > > DISK FULL??? > > I updated my system to OS X ?10.7 two days ago, but FTP did work all day yesterday! > Checked my account via the cPanel, lots of disk space of course. Which server are you on Klaus? I am on odin and was able to upload a file just now. Sarah From matthias_livecode at me.com Sun Jul 24 18:56:23 2011 From: matthias_livecode at me.com (Matthias Rebbe) Date: Mon, 25 Jul 2011 00:56:23 +0200 Subject: OT: on-rev account ftp server error 552 disk full??? In-Reply-To: References: <8650AA31-E936-49D5-A114-D50E82902462@major.on-rev.com> Message-ID: <2503F71E-6A97-4E13-8EAE-2215D9BCB8C7@me.com> Hi. I am on loki and i can also upload. Matthias Am 25.07.2011 um 00:51 schrieb Sarah Reichelt: >> I tried to upload some files to my on-rev account via FTP >> a couple of minutes ago and got this strange server response: >> error 552 - Disk full, please upload later >> >> DISK FULL??? >> >> I updated my system to OS X 10.7 two days ago, but FTP did work all day yesterday! >> Checked my account via the cPanel, lots of disk space of course. > > > Which server are you on Klaus? > I am on odin and was able to upload a file just now. > > Sarah > > _______________________________________________ > use-livecode mailing list > use-livecode at lists.runrev.com > Please visit this url to subscribe, unsubscribe and manage your subscription preferences: > http://lists.runrev.com/mailman/listinfo/use-livecode From jiml at netrin.com Sun Jul 24 19:59:54 2011 From: jiml at netrin.com (Jim Lambert) Date: Sun, 24 Jul 2011 16:59:54 -0700 Subject: OT: on-rev account ftp server error 552 disk full??? In-Reply-To: References: Message-ID: <2A8791BF-50A2-4C35-A734-3DD88A92DD87@netrin.com> I bet Klaus is on freyr since I see the same error on freyr. Jim Lambert From slava at lexiconbridge.com Sun Jul 24 20:39:44 2011 From: slava at lexiconbridge.com (Slava Paperno) Date: Sun, 24 Jul 2011 20:39:44 -0400 Subject: are all controls loaded? In-Reply-To: <08F843D1-EB52-45DC-8355-3F279367DDE2@economy-x-talk.com> References: <004601cc4a51$8c4acae0$a4e060a0$@com> <08F843D1-EB52-45DC-8355-3F279367DDE2@economy-x-talk.com> Message-ID: <004701cc4a63$5cabea50$1603bef0$@com> Yes, I can see how a splash screen can serve as a delay mechanism... Thanks, Mark. S. > -----Original Message----- > From: use-livecode-bounces at lists.runrev.com [mailto:use-livecode- > bounces at lists.runrev.com] On Behalf Of Mark Schonewille > Sent: Sunday, July 24, 2011 6:44 PM > To: How to use LiveCode > Subject: Re: are all controls loaded? > > Hi Slava, > > I have similar problems once in a while. Often, I do this: > > on preOpenStack > hide this stack > // do some very important stuff here > readPrefs -- (e.g. read registration data) > // and send a command in x millisecs > send "initialise" to me in 0 millisecs > end preOpenStack > > on initialise > put the seconds into mySecs > go stack "Splash" in new window > // the splash stack contains everything > // to set itself up > // now we can set up our controls > send "foo" to btn 2 > put url "http://bla.com" into fld 5 > // has the user clicked Splash away? > if "Splash" is among the lines of the openStacks then > wait until (the seconds - theSecs > 5) or not \ > ("Splash" is among the lines of the openStacks) with messages > close stack "Splash" > end if > end initalise > > Of course, many variations are possible. > > -- > Best regards, > > Mark Schonewille > > Economy-x-Talk Consulting and Software Engineering > Homepage: http://economy-x-talk.com > Twitter: http://twitter.com/xtalkprogrammer > KvK: 50277553 > > What does that error mean? Buy LiveCodeErrors for iPhone now > http://qery.us/v4 A must-have for LiveCode programmers. > > On 25 jul 2011, at 00:32, Slava Paperno wrote: > > > The openCard handler in the first card of my stack sends messages to > various > > controls on the card, and I think some of these messages are sent > before the > > control is fully loaded: I see error "no such object." If I use "send > XXX to > > YYY in 800 milliseconds," then all is well. > > > > How do I solve this dilemma? I need something like an > > "afterAllControlsAreLoaded" message :) > > > > Slava From keith at gulfbreezeortholab.com Sun Jul 24 20:59:47 2011 From: keith at gulfbreezeortholab.com (Keith (Gulf Breeze Ortho Lab)) Date: Sun, 24 Jul 2011 19:59:47 -0500 Subject: Writing to and from a binary file? In-Reply-To: <4E2A5A8B.4050902@pdslabs.net> References: <073B71C7-EAE1-49BD-A907-777D68A9B7F7@economy-x-talk.com> <4E2A5A8B.4050902@pdslabs.net> Message-ID: Thanks Phil! ----- Original Message ----- From: "Phil Davis" To: "How to use LiveCode" Sent: Saturday, July 23, 2011 12:22 AM Subject: Re: Writing to and from a binary file? > Hi Keith - welcome to the community! > > Here is a code snippet I shared on the use-revolution list almost 2 years > ago. It shows how to cut an image up into 10 'frames': > > http://www.mail-archive.com/use-revolution at lists.runrev.com/msg124691.html > > HTH - > Phil Davis > > > On 7/22/11 3:17 PM, Keith (Gulf Breeze Ortho Lab) wrote: >> Thanks Mark. Would you happen to know of any examples out there >> especially for binary files? >> >> >> The greatest obstacle to discovery is not ignorance -- >> it is the illusion of knowledge. Daniel Boorstin >> ----- Original Message ----- From: "Mark Schonewille" >> >> To: "How to use LiveCode" >> Sent: Friday, July 22, 2011 5:05 PM >> Subject: Re: Writing to and from a binary file? >> >> >>> Hi Keith, >>> >>> Unless I misuderstand you, you should be able to do what you want using >>> byte and binfile (and the read and write commands or the put url >>> command). >>> >>> -- >>> Best regards, >>> >>> Mark Schonewille >>> >>> Economy-x-Talk Consulting and Software Engineering >>> Homepage: http://economy-x-talk.com >>> Twitter: http://twitter.com/xtalkprogrammer >>> KvK: 50277553 >>> >>> New: Download the Installer Maker Plugin 1.6 for LiveCode here >>> http://qery.us/ce >>> >>> On 22 jul 2011, at 23:59, Keith (Gulf Breeze Ortho Lab) wrote: >>> >>>> Hello All, >>>> >>>> I am new to LiveCode and am continuing to explore the language. >>>> >>>> Quick question... Is it possible to write to and from a binary file >>>> utilizing LiveCode? For example, is it possible to take a graphic file >>>> (for example) and split it into several unequal parts, and then put the >>>> parts back together again into a single file? I have used another >>>> programming package to accomplish such. >>>> >>>> FYI: Reading in the Dictionary, I came across the byte and binfile >>>> entries. Can these be used to accomplish what I am looking for? If so, >>>> then are there any good examples out there? >>>> >>>> Thanks, and any help is most appreciated! >>>> >>>> Cheers from Florida! >>>> >>>> Keith "Boo" L. >>> >>> >>> _______________________________________________ >>> use-livecode mailing list >>> use-livecode at lists.runrev.com >>> Please visit this url to subscribe, unsubscribe and manage your >>> subscription preferences: >>> http://lists.runrev.com/mailman/listinfo/use-livecode >>> >> >> >> _______________________________________________ >> use-livecode mailing list >> use-livecode at lists.runrev.com >> Please visit this url to subscribe, unsubscribe and manage your >> subscription preferences: >> http://lists.runrev.com/mailman/listinfo/use-livecode >> > > -- > Phil Davis > > PDS Labs > Professional Software Development > http://pdslabs.net > > > _______________________________________________ > use-livecode mailing list > use-livecode at lists.runrev.com > Please visit this url to subscribe, unsubscribe and manage your > subscription preferences: > http://lists.runrev.com/mailman/listinfo/use-livecode > From gwendalwood at cox.net Sun Jul 24 22:32:37 2011 From: gwendalwood at cox.net (gwendalwood at cox.net) Date: Sun, 24 Jul 2011 19:32:37 -0700 Subject: ios files Message-ID: <20110724223237.FV717.1339251.imail@fed1rmwml38> I did figure out how to get to writing and reading files on ios, but on my second app it does not work. I still can't figure out how to print from inside ios on the iPad 2. I think maybe it can't be done yet. Following is my code snippets for files: ************** --write getPathsFromFile put specialfolderpath("documents") into temppath set defaultfolder to temppath put temppath&"/Defaults" into newtempath set defaultfolder to newtempath put the hilitedlines of cd fld "GroupThemeTitles" into grpCount put "StudentsGroup"&grpCount into temp1 open file temp1 for write write cd fld "Students" to file temp1 close file temp1 -- read put specialfolderpath("documents") into temppath set defaultfolder to temppath put temppath&"/Defaults" into newtempath set defaultfolder to newtempath put the hilitedlines of me into newgroup put "StudentsGroup"&newgroup into temp1 open file temp1 for read read from file temp1 until eof put it into cd fld "Students" put the number of lines in it into studentcount close file temp1 ************** Message: 4 Date: Sun, 24 Jul 2011 09:30:00 +1000 From: Gerry Orkin To: How to use LiveCode Subject: Re: iOS files Message-ID: Content-Type: text/plain; charset=us-ascii Post your code... Gerry -- Sent from my iPhone. On 24/07/2011, at 2:19 AM, wrote: > Anyone know how to read and write files in an iOS application running on an iPad 2? > Also, does anyone know how to print from within the iOS application? > Any hints would be appreciated. My routines work in the simulator for reading and writing files, but don't work on the iPad 2. From fmoyer at aol.com Sun Jul 24 22:32:46 2011 From: fmoyer at aol.com (Fred Moyer) Date: Sun, 24 Jul 2011 22:32:46 -0400 Subject: Exact zooming/resizing of stacks and groups Message-ID: <0E2E640C-A1D9-4CB8-9361-C7EAAA8E4771@aol.com> Has anyone done the heavy lifting to figure out how to do an exact resize/zoom of a stack or a group inside of a stack? What I am talking about is stuff like: - adjusting the margins of fields as they resize - adjusting the textsize and textheight of characters in fields as they resize - adjusting the linesize of lines and bordered graphics as they resize - keeping the same relative vscroll and hscroll as fields resizes etc. I think it would be a lot of work to figure this out; there are lots of factors to consider. But I assume someone has done it already. Of course, I am only talking about "proportional" resizing where the the x and y proportions remain the same. Thanks Fred Moyer From gwendalwood at cox.net Sun Jul 24 22:35:24 2011 From: gwendalwood at cox.net (gwendalwood at cox.net) Date: Sun, 24 Jul 2011 19:35:24 -0700 Subject: No subject Message-ID: <20110724223524.5TNV8.1339270.imail@fed1rmwml38> I did figure out how to get to writing and reading files on ios, but on my second app it does not work. I still can't figure out how to print from inside ios on the iPad 2. I think maybe it can't be done yet. I forgot to tell you my results. I get nothing in the variable it or the variable results Following is my code snippets for files: ************** --write getPathsFromFile put specialfolderpath("documents") into temppath set defaultfolder to temppath put temppath&"/Defaults" into newtempath set defaultfolder to newtempath put the hilitedlines of cd fld "GroupThemeTitles" into grpCount put "StudentsGroup"&grpCount into temp1 open file temp1 for write write cd fld "Students" to file temp1 close file temp1 -- read put specialfolderpath("documents") into temppath set defaultfolder to temppath put temppath&"/Defaults" into newtempath set defaultfolder to newtempath put the hilitedlines of me into newgroup put "StudentsGroup"&newgroup into temp1 open file temp1 for read read from file temp1 until eof put it into cd fld "Students" put the number of lines in it into studentcount close file temp1 ************** From niconiko at gmail.com Sun Jul 24 22:42:18 2011 From: niconiko at gmail.com (Nicolas Cueto) Date: Mon, 25 Jul 2011 11:42:18 +0900 Subject: stackfiles -- how to use? Message-ID: Hello, Apologies for the length of this, but I'd very much like list help to understand better a scripting practice I've been doing half-baked for a long time now. It has to do with stackFiles (I think?), the steps for implementing which I describe below, but first precede with an outline of what I'm after. Essentially, this is the structure of my standalone and stacks: AllMightyStack.exe --> Index-stack.livecode --> gamestacks1-8.livecode The role of AllMightyStack.exe: -- opens up Index-stack.livecode -- stays open but hidden in background -- (most importantly!) contains handlers/functions and global variables -- but no controls/objects -- that can be called by the index-stack and each of the gamestacks simply by name (ie, without having to specify a path) The role of Index-stack.rev: -- shows the user some buttons, clicking which opens up one of the various gamestacks -- stays open but hidden in background, so that, when a gamestack is done with, the user returns to Index-stack to either choose a different gamestack or quit -- does not contain handlers or controls referenced to by any other stack To do the above, here are my steps: STEP 1) place the following script in card 1 of AllMightyStack.exe: on openCard setUpFoundation end openCard on setUpFoundation put "Index-stack,Index-stack.rev" & cr & "game1,game1.rev" & cr & \ "game2,game2.rev" & cr & .... & cr & "animationEngine,animationEngine.rev" \ into tStackFiles set the stackFiles of stack "Mainstack.exe" to tStackFiles ... open stack "Index-stack" hide stack "Mainstack" end setUpFoundation STEP 2) in Livecode open the property-inspector for stack "AllMightyStack.rev" and then in the "Stack files" panel add Index-Stack and all the game-stacks. After which, I save as a standalone. STEP 3) repeat the above two steps for Index-stack.rev Anyway, I'm guessing I don't need to do STEP 3. But now I'm also guessing STEP 2 is superfluous. Or is it? Of course, I've read the documentation. But... Hope that all makes sense. Thank you -- for your help and forbearance. -- Nicolas Cueto From selander at tkf.att.ne.jp Sun Jul 24 23:48:33 2011 From: selander at tkf.att.ne.jp (Tim Selander) Date: Mon, 25 Jul 2011 12:48:33 +0900 Subject: OT: on-rev account ftp server error 552 disk full??? In-Reply-To: <2A8791BF-50A2-4C35-A734-3DD88A92DD87@netrin.com> References: <2A8791BF-50A2-4C35-A734-3DD88A92DD87@netrin.com> Message-ID: <4E2CE791.9050208@tkf.att.ne.jp> we are having the same problem on freyr... will be sending a support email asap. Tim Selander Tokyo, Japan Jim Lambert wrote: > I bet Klaus is on freyr since I see the same error on freyr. > > Jim Lambert > > _______________________________________________ > use-livecode mailing list > use-livecode at lists.runrev.com > Please visit this url to subscribe, unsubscribe and manage your subscription preferences: > http://lists.runrev.com/mailman/listinfo/use-livecode > From gerry.orkin at gmail.com Mon Jul 25 02:37:46 2011 From: gerry.orkin at gmail.com (Gerry Orkin) Date: Mon, 25 Jul 2011 16:37:46 +1000 Subject: No subject In-Reply-To: <20110724223524.5TNV8.1339270.imail@fed1rmwml38> References: <20110724223524.5TNV8.1339270.imail@fed1rmwml38> Message-ID: I keep things simple by using these two routines that might work for you: on writefile fileTowrite,dataTowrite set the defaultFolder to specialFolderPath("Documents") put dataTowrite into URL (("file:"&fileTowrite&".txt")) end writefile function readFromFile whatFile set the defaultFolder to specialFolderPath("Documents") return URL (("file:"&whatFile&".txt")) end readFromFile Gerry On 25/07/2011, at 12:35 PM, wrote: > Following is my code snippets for files: From niconiko at gmail.com Mon Jul 25 02:39:59 2011 From: niconiko at gmail.com (Nicolas Cueto) Date: Mon, 25 Jul 2011 15:39:59 +0900 Subject: standalone won't start on Win2K Message-ID: Hi, When a standalone built with LC4.6.1 starts on a Win2K PC, the result is a crash with "CreaterFiberEX" and "Kernel32.dll" in the crash message. Any workarounds for this (upgrading Win2K is not an option)? Thanks. -- Nicolas Cueto From tabithavogelaar at hotmail.com Mon Jul 25 03:47:56 2011 From: tabithavogelaar at hotmail.com (Tabitha Vogelaar) Date: Mon, 25 Jul 2011 09:47:56 +0200 Subject: I need to import photo's in an app? How? Message-ID: I'm just a beginner and this is the first time I use the mailinglist. I'm working on an app and I want to make it possible for the user to import photo's saved on his/her iPod/iPad. (I'm sorry, my english isn't very good. I'm dutch) My father said something about something called "specialFolderPath" or something but I can't test it because I have Lion and there's something wrong so I can't put the app's I made on my iPod to test it... What I need is that the user chooses a photo from one of his/her photo-album's on his/her iPod/iPad. That photo will be put into an image-field. Please explain it clearly, because my english isn't very good. I can understand most things but some are too difficult for me. Please help me.Tabitha. From gerry.orkin at gmail.com Mon Jul 25 04:07:59 2011 From: gerry.orkin at gmail.com (Gerry Orkin) Date: Mon, 25 Jul 2011 18:07:59 +1000 Subject: I need to import photo's in an app? How? In-Reply-To: References: Message-ID: <8897F751-838F-4FD3-80F3-498782F10E26@gmail.com> > I'm just a beginner and this is the first time I use the mailinglist. I'm working on an app and I want to make it possible for the user to import photo's saved on his/her iPod/iPad. Look at page 21 of the iOS User Guide. This code: iPhonePickPhoto "library", 300,400 ...will show the photo library. When the user picks a photo it will be shown on the current card, and it will be 300 pixels wide, and 400 pixels high. You can change those sizes if you need to. Gerry From mazzapaoloitaly at gmail.com Mon Jul 25 04:47:31 2011 From: mazzapaoloitaly at gmail.com (paolo mazza) Date: Mon, 25 Jul 2011 10:47:31 +0200 Subject: previous page button of browser opening on-rev server page Message-ID: How can I prevent users from opening an .irev page clicking the "previous page button" of the browser ? I mean. I have a on-rev.com/xxx.irev page resulting from the LiveCode cgi process . If the user move forward from this page, I do not want the user be able to go back to the previous page reloading the HTML code resulting from the cgi process. How can I do that? Any idea? Thanks a lot. Paolo Mazza From m.schonewille at economy-x-talk.com Mon Jul 25 04:54:36 2011 From: m.schonewille at economy-x-talk.com (Mark Schonewille) Date: Mon, 25 Jul 2011 10:54:36 +0200 Subject: standalone won't start on Win2K In-Reply-To: References: Message-ID: <8557A39C-7591-4D22-BFFF-6D7240E8FF35@economy-x-talk.com> Hi Nicolas, Do you have SP4 installed? -- Best regards, Mark Schonewille Economy-x-Talk Consulting and Software Engineering Homepage: http://economy-x-talk.com Twitter: http://twitter.com/xtalkprogrammer KvK: 50277553 What does that error mean? Buy LiveCodeErrors for iPhone now http://qery.us/v4 A must-have for LiveCode programmers. On 25 jul 2011, at 08:39, Nicolas Cueto wrote: > Hi, > > When a standalone built with LC4.6.1 starts on a Win2K PC, the result > is a crash with "CreaterFiberEX" and "Kernel32.dll" in the crash > message. > > Any workarounds for this (upgrading Win2K is not an option)? > > Thanks. > > -- > Nicolas Cueto From m.schonewille at economy-x-talk.com Mon Jul 25 05:00:49 2011 From: m.schonewille at economy-x-talk.com (Mark Schonewille) Date: Mon, 25 Jul 2011 11:00:49 +0200 Subject: previous page button of browser opening on-rev server page In-Reply-To: References: Message-ID: <2CA55DDD-D9D0-4C0F-8E57-BAF82E464649@economy-x-talk.com> Hi Paolo, You could set a cookie to true when the web form loads and to false when the cgi processes the web form. Whenever the cookie is false, the cgi should not be allowed to run. -- Best regards, Mark Schonewille Economy-x-Talk Consulting and Software Engineering Homepage: http://economy-x-talk.com Twitter: http://twitter.com/xtalkprogrammer KvK: 50277553 What does that error mean? Buy LiveCodeErrors for iPhone now http://qery.us/v4 A must-have for LiveCode programmers. On 25 jul 2011, at 10:47, paolo mazza wrote: > How can I prevent users from opening an .irev page clicking the > "previous page button" of the browser ? > > I mean. I have a on-rev.com/xxx.irev page resulting from the LiveCode > cgi process . If the user move forward from this page, I do not want > the user be able to go back to the previous page reloading the HTML > code resulting from the cgi process. > > How can I do that? Any idea? Thanks a lot. > > Paolo Mazza From maarten.koopmans at gmail.com Mon Jul 25 05:21:17 2011 From: maarten.koopmans at gmail.com (Maarten Koopmans) Date: Mon, 25 Jul 2011 11:21:17 +0200 Subject: OT: gmail two factor authentication In-Reply-To: References: Message-ID: Yep, a typo I decided was to spot-on to change :-) On Mon, Jul 25, 2011 at 12:09 AM, Roger Eller wrote: > On Sun, Jul 24, 2011 at 4:30 PM, Maarten Koopmans wrote: > >> All, >> >> I just got a nice box from Google explaining that there was suspicious >> account activity from Mexico (I live in Amsterdam). I changed my >> password, enabled two-factor authentication using their iPhone app and >> use app-specific passwords. >> >> Also, I changed ALL passwords of all online services I use to new, >> unique ones. My guess: it took one malicious web page, even though I >> am up-to-fate with every component.... > > > > > >> >> --Maarten >> > > It's always up-to-fate, isn't it? ?;-) > > ?Roger > _______________________________________________ > use-livecode mailing list > use-livecode at lists.runrev.com > Please visit this url to subscribe, unsubscribe and manage your subscription preferences: > http://lists.runrev.com/mailman/listinfo/use-livecode > From selander at tkf.att.ne.jp Mon Jul 25 05:26:24 2011 From: selander at tkf.att.ne.jp (Tim Selander) Date: Mon, 25 Jul 2011 18:26:24 +0900 Subject: OT: on-rev account ftp server error 552 disk full??? In-Reply-To: <4E2CE791.9050208@tkf.att.ne.jp> References: <2A8791BF-50A2-4C35-A734-3DD88A92DD87@netrin.com> <4E2CE791.9050208@tkf.att.ne.jp> Message-ID: <4E2D36C0.7090802@tkf.att.ne.jp> Problem solved. "Thank you" to Heather and the support team at on-rev.com Tim Selander Tokyo, Japan On 7/25/11 12:48 PM, Tim Selander wrote: > we are having the same problem on freyr... will be sending a support > email asap. > > Tim Selander > Tokyo, Japan > > Jim Lambert wrote: >> I bet Klaus is on freyr since I see the same error on freyr. >> >> Jim Lambert >>> From mazzapaolo at libero.it Mon Jul 25 05:30:20 2011 From: mazzapaolo at libero.it (paolo mazza) Date: Mon, 25 Jul 2011 11:30:20 +0200 Subject: previous page button of browser opening on-rev server page In-Reply-To: <2CA55DDD-D9D0-4C0F-8E57-BAF82E464649@economy-x-talk.com> References: <2CA55DDD-D9D0-4C0F-8E57-BAF82E464649@economy-x-talk.com> Message-ID: Hi Mark, when I go back with then "previous page" button of the browser, the cgi does not run again. The browser simply return the HTML recorded in the browser cache. I tried with Safary and with Firefox; both of them do the same thing. However, if I empty the cache before returning to the previous page, the browser runs the cgi again. Thanks Paolo From williamdesmet at gmail.com Mon Jul 25 05:36:02 2011 From: williamdesmet at gmail.com (William de Smet) Date: Mon, 25 Jul 2011 11:36:02 +0200 Subject: I need to import photo's in an app? How? In-Reply-To: References: Message-ID: <60F6C0FF-4288-467D-B7C7-5BB6513076F9@gmail.com> Hoi Tabitha, There is also a lesson about it on http://lesson.runrev.com Groeten, William ----- Verstuurd vanaf mijn iPhone! Op 25 jul. 2011 om 09:47 heeft Tabitha Vogelaar het volgende geschreven: > > I'm just a beginner and this is the first time I use the mailinglist. I'm working on an app and I want to make it possible for the user to import photo's saved on his/her iPod/iPad. (I'm sorry, my english isn't very good. I'm dutch) My father said something about something called "specialFolderPath" or something but I can't test it because I have Lion and there's something wrong so I can't put the app's I made on my iPod to test it... What I need is that the user chooses a photo from one of his/her photo-album's on his/her iPod/iPad. That photo will be put into an image-field. Please explain it clearly, because my english isn't very good. I can understand most things but some are too difficult for me. Please help me.Tabitha. > _______________________________________________ > use-livecode mailing list > use-livecode at lists.runrev.com > Please visit this url to subscribe, unsubscribe and manage your subscription preferences: > http://lists.runrev.com/mailman/listinfo/use-livecode From klaus at major.on-rev.com Mon Jul 25 05:56:23 2011 From: klaus at major.on-rev.com (Klaus on-rev) Date: Mon, 25 Jul 2011 11:56:23 +0200 Subject: OT: on-rev account ftp server error 552 disk full??? In-Reply-To: <2A8791BF-50A2-4C35-A734-3DD88A92DD87@netrin.com> References: <2A8791BF-50A2-4C35-A734-3DD88A92DD87@netrin.com> Message-ID: <538DC359-1E53-4E38-A0BB-6DD839FC908B@major.on-rev.com> Hi friends, Am 25.07.2011 um 01:59 schrieb Jim Lambert: > I bet Klaus is on freyr since I see the same error on freyr. Yep, its Freyr! But today it is working again as if nothing happened at all, go figure :-) Working for you, too, Jim? > Jim Lambert Best Klaus -- Klaus Major http://www.major-k.de klaus at major.on-rev.com From gerry.orkin at gmail.com Mon Jul 25 06:07:42 2011 From: gerry.orkin at gmail.com (Gerry Orkin) Date: Mon, 25 Jul 2011 20:07:42 +1000 Subject: I need to import photo's in an app? How? In-Reply-To: <60F6C0FF-4288-467D-B7C7-5BB6513076F9@gmail.com> References: <60F6C0FF-4288-467D-B7C7-5BB6513076F9@gmail.com> Message-ID: Er, it's http://lessons.runrev.com :) g On 25/07/2011, at 7:36 PM, William de Smet wrote: > http://lesson.runrev.com From m.schonewille at economy-x-talk.com Mon Jul 25 07:29:40 2011 From: m.schonewille at economy-x-talk.com (Mark Schonewille) Date: Mon, 25 Jul 2011 13:29:40 +0200 Subject: No subject In-Reply-To: <20110724223524.5TNV8.1339270.imail@fed1rmwml38> References: <20110724223524.5TNV8.1339270.imail@fed1rmwml38> Message-ID: <06D677E6-A264-4D33-9993-3F746F41B4CD@economy-x-talk.com> Hi ..., Your scripts looks correct, but you got some small mistakes in your script, which might make it fail. I also think your could improve your scripting style to make it more readable, which will help you to avoid mistakes. You set the defaultFolder for no apparent reason. References to card fields are a relic from the HyperCard days and I can't imagine you need that for iOS apps. The tendency of programmers to use non-human-like language for object names really drives me more than nuts. LiveCode allows for spaces in object and file names. Use them! -- write put specialFolderPath("documents") & "/Defaults" into myPath // I don't know whether you need the next line, just let's be sure it exists if not (there is a folder myPath) then create folder myPath // mind that LiveCode allows for readable object names. // The same applies to files names. Use spaces!!! put myPath & slash & "Students Group" && the hilitedLines of fld "Group Theme Titles" into myFile open file myFile for write write fld "Students" to file myFile close file myFile The read script is almost the same, using read from instead of write to. Note that I'm not using the defaultFolder because there is no need for it. -- Best regards, Mark Schonewille Economy-x-Talk Consulting and Software Engineering Homepage: http://economy-x-talk.com Twitter: http://twitter.com/xtalkprogrammer KvK: 50277553 What does that error mean? Buy LiveCodeErrors for iPhone now http://qery.us/v4 A must-have for LiveCode programmers. On 25 jul 2011, at 04:35, wrote: > I did figure out how to get to writing and reading files on ios, but on my > second app it does not work. I still can't figure out how to print from inside > ios on the iPad 2. I think maybe it can't be done yet. > I forgot to tell you my results. I get nothing in the variable it or the variable results > Following is my code snippets for files: > ************** > --write > getPathsFromFile > put specialfolderpath("documents") into temppath > set defaultfolder to temppath > put temppath&"/Defaults" into newtempath > set defaultfolder to newtempath > put the hilitedlines of cd fld "GroupThemeTitles" into grpCount > put "StudentsGroup"&grpCount into temp1 > open file temp1 for write > write cd fld "Students" to file temp1 > close file temp1 > > -- read > put specialfolderpath("documents") into temppath > set defaultfolder to temppath > put temppath&"/Defaults" into newtempath > set defaultfolder to newtempath > put the hilitedlines of me into newgroup > put "StudentsGroup"&newgroup into temp1 > open file temp1 for read > read from file temp1 until eof > put it into cd fld "Students" > put the number of lines in it into studentcount > close file temp1 > > > ************** > > _______________________________________________ > use-livecode mailing list > use-livecode at lists.runrev.com > Please visit this url to subscribe, unsubscribe and manage your subscription preferences: > http://lists.runrev.com/mailman/listinfo/use-livecode From shaosean at wehostmacs.com Mon Jul 25 08:51:39 2011 From: shaosean at wehostmacs.com (Shao Sean) Date: Mon, 25 Jul 2011 08:51:39 -0400 Subject: previous page button of browser opening on-rev server page Message-ID: <6F53F77D-1D26-4114-A7E5-06B5D641C9DC@wehostmacs.com> Jump down to example two and just do that in your .irev page http://php.net/manual/en/function.header.php From m.schonewille at economy-x-talk.com Mon Jul 25 09:04:00 2011 From: m.schonewille at economy-x-talk.com (Mark Schonewille) Date: Mon, 25 Jul 2011 15:04:00 +0200 Subject: previous page button of browser opening on-rev server page In-Reply-To: References: <2CA55DDD-D9D0-4C0F-8E57-BAF82E464649@economy-x-talk.com> Message-ID: <88F2DE00-13A2-4AD0-B9CE-961564DAB131@economy-x-talk.com> I see, Paolo, I misunderstood your questions. As always, do what Shao Sean says ;-) -- Best regards, Mark Schonewille Economy-x-Talk Consulting and Software Engineering Homepage: http://economy-x-talk.com Twitter: http://twitter.com/xtalkprogrammer KvK: 50277553 What does that error mean? Buy LiveCodeErrors for iPhone now http://qery.us/v4 A must-have for LiveCode programmers. On 25 jul 2011, at 11:30, paolo mazza wrote: > Hi Mark, > when I go back with then "previous page" button of the browser, the > cgi does not run again. The browser simply return the HTML recorded > in the browser cache. > I tried with Safary and with Firefox; both of them do the same thing. > However, if I empty the cache before returning to the previous page, > the browser runs the cgi again. > > Thanks > Paolo From kray at sonsothunder.com Mon Jul 25 09:15:48 2011 From: kray at sonsothunder.com (Ken Ray) Date: Mon, 25 Jul 2011 08:15:48 -0500 Subject: No subject In-Reply-To: <06D677E6-A264-4D33-9993-3F746F41B4CD@economy-x-talk.com> References: <20110724223524.5TNV8.1339270.imail@fed1rmwml38> <06D677E6-A264-4D33-9993-3F746F41B4CD@economy-x-talk.com> Message-ID: <99CF93B9-5C6B-48DB-9412-E02A5F03CE3D@sonsothunder.com> > The tendency of programmers to use non-human-like language for object names really drives me more than nuts. LiveCode allows for spaces in object and file names. Use them! Mark, although I can appreciate your feelings about this, there are legitimate reasons one might not want to use spaces in object names: 1) It is more efficient as it takes less characters to type 2) You can quickly select the name of the object by double-clicking it 3) You can more easily distinguish object names from text strings when glancing through your code 4) Most other languages don't let you do this, so If you use multiple languages it would be better to have a consistent style across languages. Just my 2 cents, Ken Ray Sons of Thunder Software, Inc. Email: kray at sonsothunder.com Web Site: http://www.sonsothunder.com > > -- write > put specialFolderPath("documents") & "/Defaults" into myPath > // I don't know whether you need the next line, just let's be sure it exists > if not (there is a folder myPath) then create folder myPath > // mind that LiveCode allows for readable object names. > // The same applies to files names. Use spaces!!! > put myPath & slash & "Students Group" && the hilitedLines of fld "Group Theme Titles" into myFile > open file myFile for write > write fld "Students" to file myFile > close file myFile > > The read script is almost the same, using read from instead of write to. Note that I'm not using the defaultFolder because there is no need for it. > > -- > Best regards, > > Mark Schonewille > > Economy-x-Talk Consulting and Software Engineering > Homepage: http://economy-x-talk.com > Twitter: http://twitter.com/xtalkprogrammer > KvK: 50277553 > > What does that error mean? Buy LiveCodeErrors for iPhone now http://qery.us/v4 A must-have for LiveCode programmers. > > > > > > On 25 jul 2011, at 04:35, wrote: > >> I did figure out how to get to writing and reading files on ios, but on my >> second app it does not work. I still can't figure out how to print from inside >> ios on the iPad 2. I think maybe it can't be done yet. >> I forgot to tell you my results. I get nothing in the variable it or the variable results >> Following is my code snippets for files: >> ************** >> --write >> getPathsFromFile >> put specialfolderpath("documents") into temppath >> set defaultfolder to temppath >> put temppath&"/Defaults" into newtempath >> set defaultfolder to newtempath >> put the hilitedlines of cd fld "GroupThemeTitles" into grpCount >> put "StudentsGroup"&grpCount into temp1 >> open file temp1 for write >> write cd fld "Students" to file temp1 >> close file temp1 >> >> -- read >> put specialfolderpath("documents") into temppath >> set defaultfolder to temppath >> put temppath&"/Defaults" into newtempath >> set defaultfolder to newtempath >> put the hilitedlines of me into newgroup >> put "StudentsGroup"&newgroup into temp1 >> open file temp1 for read >> read from file temp1 until eof >> put it into cd fld "Students" >> put the number of lines in it into studentcount >> close file temp1 >> >> >> ************** >> >> _______________________________________________ >> use-livecode mailing list >> use-livecode at lists.runrev.com >> Please visit this url to subscribe, unsubscribe and manage your subscription preferences: >> http://lists.runrev.com/mailman/listinfo/use-livecode > > > _______________________________________________ > use-livecode mailing list > use-livecode at lists.runrev.com > Please visit this url to subscribe, unsubscribe and manage your subscription preferences: > http://lists.runrev.com/mailman/listinfo/use-livecode From mazzapaoloitaly at gmail.com Mon Jul 25 11:08:33 2011 From: mazzapaoloitaly at gmail.com (paolo mazza) Date: Mon, 25 Jul 2011 17:08:33 +0200 Subject: previous page button of browser opening on-rev server page In-Reply-To: <88F2DE00-13A2-4AD0-B9CE-961564DAB131@economy-x-talk.com> References: <2CA55DDD-D9D0-4C0F-8E57-BAF82E464649@economy-x-talk.com> <88F2DE00-13A2-4AD0-B9CE-961564DAB131@economy-x-talk.com> Message-ID: Thank you all. Trying to get rid of the browser cache setting the proper headings (as Shao Sean suggested) , I came up with these 2 solutions: FIRST ONE WITH PHP Documento senza titolo THE SECOND ONE WITH LIVECODE ... much easyer ;-) HOWEVER, the first one (PHP) set the heathers properly but still I get the old time in the page (cache) when I move back and forth with the buttons of the browser. The second one (LiveCode) does NOT set the headers properly (is it a bug?) and the page remains in the browser cache. Any idea? All the best Paolo Mazza From bobs at twft.com Mon Jul 25 12:29:07 2011 From: bobs at twft.com (Bob Sneidar) Date: Mon, 25 Jul 2011 09:29:07 -0700 Subject: [ANN] New plugin AAG|LayerComps In-Reply-To: <4E2A3747.8030101@hyperactivesw.com> References: <4E29EAE8.6050302@hyperactivesw.com> <4E2A3747.8030101@hyperactivesw.com> Message-ID: <838F5162-F431-4423-8C1A-8329CC7EBEF0@twft.com> And he has yet to succumb to evil, the way most geniuses eventually do. Bob On Jul 22, 2011, at 7:51 PM, J. Landman Gay wrote: > Of that I'm sure. Andre's a genius. > > -- > Jacqueline Landman Gay | jacque at hyperactivesw.com > HyperActive Software | http://www.hyperactivesw.com From bobs at twft.com Mon Jul 25 12:35:09 2011 From: bobs at twft.com (Bob Sneidar) Date: Mon, 25 Jul 2011 09:35:09 -0700 Subject: OT: gmail two factor authentication In-Reply-To: References: Message-ID: Hah hah! I am going to have to work that into a conversation some time this week! Bob On Jul 24, 2011, at 3:09 PM, Roger Eller wrote: > On Sun, Jul 24, 2011 at 4:30 PM, Maarten Koopmans wrote: > >> All, >> >> I just got a nice box from Google explaining that there was suspicious >> account activity from Mexico (I live in Amsterdam). I changed my >> password, enabled two-factor authentication using their iPhone app and >> use app-specific passwords. >> >> Also, I changed ALL passwords of all online services I use to new, >> unique ones. My guess: it took one malicious web page, even though I >> am up-to-fate with every component.... > > > > > >> >> --Maarten >> > > It's always up-to-fate, isn't it? ;-) > > ?Roger > _______________________________________________ > use-livecode mailing list > use-livecode at lists.runrev.com > Please visit this url to subscribe, unsubscribe and manage your subscription preferences: > http://lists.runrev.com/mailman/listinfo/use-livecode From bobs at twft.com Mon Jul 25 12:40:16 2011 From: bobs at twft.com (Bob Sneidar) Date: Mon, 25 Jul 2011 09:40:16 -0700 Subject: No subject In-Reply-To: <99CF93B9-5C6B-48DB-9412-E02A5F03CE3D@sonsothunder.com> References: <20110724223524.5TNV8.1339270.imail@fed1rmwml38> <06D677E6-A264-4D33-9993-3F746F41B4CD@economy-x-talk.com> <99CF93B9-5C6B-48DB-9412-E02A5F03CE3D@sonsothunder.com> Message-ID: <5E2D3729-DFFD-4496-BC1B-201410832DCB@twft.com> I concur with Ken. So many environments do not allow the use of spaces. Some are case sensitive. The solution for me? Always use a-z lowercase names, never capitalize anything. Otherwise I WILL end up biting my own butt. Bob On Jul 25, 2011, at 6:15 AM, Ken Ray wrote: >> The tendency of programmers to use non-human-like language for object names really drives me more than nuts. LiveCode allows for spaces in object and file names. Use them! > > Mark, although I can appreciate your feelings about this, there are legitimate reasons one might not want to use spaces in object names: > > 1) It is more efficient as it takes less characters to type > 2) You can quickly select the name of the object by double-clicking it > 3) You can more easily distinguish object names from text strings when glancing through your code > 4) Most other languages don't let you do this, so If you use multiple languages it would be better to have a consistent style across languages. > > Just my 2 cents, > > > Ken Ray > Sons of Thunder Software, Inc. > Email: kray at sonsothunder.com > Web Site: http://www.sonsothunder.com > > From andre at andregarzia.com Mon Jul 25 12:57:59 2011 From: andre at andregarzia.com (Andre Garzia) Date: Mon, 25 Jul 2011 13:57:59 -0300 Subject: [ANN] New plugin AAG|LayerComps In-Reply-To: <838F5162-F431-4423-8C1A-8329CC7EBEF0@twft.com> References: <4E29EAE8.6050302@hyperactivesw.com> <4E2A3747.8030101@hyperactivesw.com> <838F5162-F431-4423-8C1A-8329CC7EBEF0@twft.com> Message-ID: I am not a genius, Mark Waddingham is a genius... I am Brazilian though, and since all Brazilians are Evil... I can qualify for evil now and try to reach my genius degree later... On Mon, Jul 25, 2011 at 1:29 PM, Bob Sneidar wrote: > And he has yet to succumb to evil, the way most geniuses eventually do. > > Bob > > > On Jul 22, 2011, at 7:51 PM, J. Landman Gay wrote: > > > Of that I'm sure. Andre's a genius. > > > > -- > > Jacqueline Landman Gay | jacque at hyperactivesw.com > > HyperActive Software | http://www.hyperactivesw.com > > > _______________________________________________ > use-livecode mailing list > use-livecode at lists.runrev.com > Please visit this url to subscribe, unsubscribe and manage your > subscription preferences: > http://lists.runrev.com/mailman/listinfo/use-livecode > -- http://www.andregarzia.com All We Do Is Code. From bobs at twft.com Mon Jul 25 13:48:33 2011 From: bobs at twft.com (Bob Sneidar) Date: Mon, 25 Jul 2011 10:48:33 -0700 Subject: [ANN] New plugin AAG|LayerComps In-Reply-To: References: <4E29EAE8.6050302@hyperactivesw.com> <4E2A3747.8030101@hyperactivesw.com> <838F5162-F431-4423-8C1A-8329CC7EBEF0@twft.com> Message-ID: hmmm... that is EXACTLY what an Evil Brazilian Genius might say to put us off his trail... Bob On Jul 25, 2011, at 9:57 AM, Andre Garzia wrote: > I am not a genius, Mark Waddingham is a genius... I am Brazilian though, and > since all Brazilians are Evil... I can qualify for evil now and try to reach > my genius degree later... > > On Mon, Jul 25, 2011 at 1:29 PM, Bob Sneidar wrote: > >> And he has yet to succumb to evil, the way most geniuses eventually do. >> >> Bob >> >> >> On Jul 22, 2011, at 7:51 PM, J. Landman Gay wrote: >> >>> Of that I'm sure. Andre's a genius. >>> >>> -- >>> Jacqueline Landman Gay | jacque at hyperactivesw.com >>> HyperActive Software | http://www.hyperactivesw.com >> >> >> _______________________________________________ >> use-livecode mailing list >> use-livecode at lists.runrev.com >> Please visit this url to subscribe, unsubscribe and manage your >> subscription preferences: >> http://lists.runrev.com/mailman/listinfo/use-livecode >> > > > > -- > http://www.andregarzia.com All We Do Is Code. > _______________________________________________ > use-livecode mailing list > use-livecode at lists.runrev.com > Please visit this url to subscribe, unsubscribe and manage your subscription preferences: > http://lists.runrev.com/mailman/listinfo/use-livecode From terryhass at tdheng.com Mon Jul 25 14:22:36 2011 From: terryhass at tdheng.com (TERRY HASS) Date: Mon, 25 Jul 2011 13:22:36 -0500 Subject: iPhone/iPad App FINALLY Resleased Message-ID: Hi All, I am proud to announce that my GoldTrader101 iPhone/iPad app has FINALLY been released and is available at the Apple App Store. It is written entirely in LiveCode. Below is a link to the app page. View In iTunes Being new to LiveCode and to programming in general, this experience has required something of a learning curve. I will outline my newbee lessons in another letter at a later time. Thank you all for being such an open and generous community. Terry From jacque at hyperactivesw.com Mon Jul 25 14:28:36 2011 From: jacque at hyperactivesw.com (J. Landman Gay) Date: Mon, 25 Jul 2011 13:28:36 -0500 Subject: stackfiles -- how to use? In-Reply-To: References: Message-ID: <4E2DB5D4.7020000@hyperactivesw.com> On 7/24/11 9:42 PM, Nicolas Cueto wrote: > To do the above, here are my steps: > > STEP 1) place the following script in card 1 of AllMightyStack.exe: > > on openCard > setUpFoundation > end openCard > > on setUpFoundation > put "Index-stack,Index-stack.rev"& cr& "game1,game1.rev"& cr& \ > "game2,game2.rev"& cr& ....& cr& "animationEngine,animationEngine.rev" \ > into tStackFiles > set the stackFiles of stack "Mainstack.exe" to tStackFiles > ... > open stack "Index-stack" > hide stack "Mainstack" > end setUpFoundation > > > STEP 2) in Livecode open the property-inspector for stack > "AllMightyStack.rev" and then in the "Stack files" panel add > Index-Stack and all the game-stacks. After which, I save as a > standalone. > > STEP 3) repeat the above two steps for Index-stack.rev > > Anyway, I'm guessing I don't need to do STEP 3. But now I'm also > guessing STEP 2 is superfluous. Or is it? Stackfiles are a permanent property of a stack, so you don't need to set them more than once. If they don't change, just set them up in the inspector and save the stack. I think I'd only set them by script if they were dynamically determined at launch; for example, if a script reads a folder to determine which files are available. I'm not sure whether the inheritance hierachy applies to stackfiles, I don't use them too much. It seems like it should. But if all your navigation is done in the mainstack, then that would be the only one that needs to have stackfiles set. -- Jacqueline Landman Gay | jacque at hyperactivesw.com HyperActive Software | http://www.hyperactivesw.com From roger.e.eller at sealedair.com Mon Jul 25 14:32:50 2011 From: roger.e.eller at sealedair.com (Roger Eller) Date: Mon, 25 Jul 2011 14:32:50 -0400 Subject: iPhone/iPad App FINALLY Resleased In-Reply-To: References: Message-ID: On Mon, Jul 25, 2011 at 2:22 PM, TERRY HASS wrote: > Hi All, > > I am proud to announce that my GoldTrader101 iPhone/iPad app has FINALLY > been released and is available at the Apple App Store. It is written > entirely in LiveCode. Below is a link to the app page. > > View In iTunes > > Being new to LiveCode and to programming in general, this experience has > required something of a learning curve. I will outline my newbee lessons in > another letter at a later time. Thank you all for being such an open and > generous community. > > Terry > Congrats! Now about that link... ~Roger From lists.pete at haworths.org Mon Jul 25 14:43:27 2011 From: lists.pete at haworths.org (Pete Haworth) Date: Mon, 25 Jul 2011 11:43:27 -0700 Subject: stackfiles -- how to use? In-Reply-To: <4E2DB5D4.7020000@hyperactivesw.com> References: <4E2DB5D4.7020000@hyperactivesw.com> Message-ID: I think I missed the original post of this thread, but perhaps a "start using" command would take care of all this? Pete On Mon, Jul 25, 2011 at 11:28 AM, J. Landman Gay wrote: > On 7/24/11 9:42 PM, Nicolas Cueto wrote: > > >> To do the above, here are my steps: >> >> STEP 1) place the following script in card 1 of AllMightyStack.exe: >> >> on openCard >> setUpFoundation >> end openCard >> >> on setUpFoundation >> put "Index-stack,Index-stack.rev"& cr& "game1,game1.rev"& cr& \ >> "game2,game2.rev"& cr& ....& cr& "animationEngine,**animationEngine.rev" >> \ >> into tStackFiles >> set the stackFiles of stack "Mainstack.exe" to tStackFiles >> ... >> open stack "Index-stack" >> hide stack "Mainstack" >> end setUpFoundation >> >> >> STEP 2) in Livecode open the property-inspector for stack >> "AllMightyStack.rev" and then in the "Stack files" panel add >> Index-Stack and all the game-stacks. After which, I save as a >> standalone. >> >> STEP 3) repeat the above two steps for Index-stack.rev >> >> Anyway, I'm guessing I don't need to do STEP 3. But now I'm also >> guessing STEP 2 is superfluous. Or is it? >> > > Stackfiles are a permanent property of a stack, so you don't need to set > them more than once. If they don't change, just set them up in the inspector > and save the stack. I think I'd only set them by script if they were > dynamically determined at launch; for example, if a script reads a folder to > determine which files are available. > > I'm not sure whether the inheritance hierachy applies to stackfiles, I > don't use them too much. It seems like it should. But if all your navigation > is done in the mainstack, then that would be the only one that needs to have > stackfiles set. > > -- > Jacqueline Landman Gay | jacque at hyperactivesw.com > HyperActive Software | http://www.hyperactivesw.com > > ______________________________**_________________ > use-livecode mailing list > use-livecode at lists.runrev.com > Please visit this url to subscribe, unsubscribe and manage your > subscription preferences: > http://lists.runrev.com/**mailman/listinfo/use-livecode > From stephenREVOLUTION2 at barncard.com Mon Jul 25 15:06:09 2011 From: stephenREVOLUTION2 at barncard.com (stephen barncard) Date: Mon, 25 Jul 2011 12:06:09 -0700 Subject: No subject In-Reply-To: <5E2D3729-DFFD-4496-BC1B-201410832DCB@twft.com> References: <20110724223524.5TNV8.1339270.imail@fed1rmwml38> <06D677E6-A264-4D33-9993-3F746F41B4CD@economy-x-talk.com> <99CF93B9-5C6B-48DB-9412-E02A5F03CE3D@sonsothunder.com> <5E2D3729-DFFD-4496-BC1B-201410832DCB@twft.com> Message-ID: Another good piece of advice is watch namespaces. The parser looks at all tokens, so avoid the use of names that match language functions, commands or properties. A handler named answerDialog for instance, is bound for trouble. This usually can be avoided by using the 'standard' prefixes as explained here clearly by Mr. Gaskin. http://www.fourthworld.com/embassy/articles/scriptstyle.html All this info is good, re this topic - scroll down to 'naming conventions'. On 25 July 2011 09:40, Bob Sneidar wrote: > I concur with Ken. So many environments do not allow the use of spaces. > Some are case sensitive. The solution for me? Always use a-z lowercase > names, never capitalize anything. Otherwise I WILL end up biting my own > butt. > > Bob > > > On Jul 25, 2011, at 6:15 AM, Ken Ray wrote: > > >> The tendency of programmers to use non-human-like language for object > names really drives me more than nuts. LiveCode allows for spaces in object > and file names. Use them! > > > > Mark, although I can appreciate your feelings about this, there are > legitimate reasons one might not want to use spaces in object names: > > > > 1) It is more efficient as it takes less characters to type > > 2) You can quickly select the name of the object by double-clicking it > > 3) You can more easily distinguish object names from text strings when > glancing through your code > > 4) Most other languages don't let you do this, so If you use multiple > languages it would be better to have a consistent style across languages. > > > > Just my 2 cents, > > > > > > Ken Ray > > Sons of Thunder Software, Inc. > > Email: kray at sonsothunder.com > > Web Site: http://www.sonsothunder.com > > > > > > > _______________________________________________ > use-livecode mailing list > use-livecode at lists.runrev.com > Please visit this url to subscribe, unsubscribe and manage your > subscription preferences: > http://lists.runrev.com/mailman/listinfo/use-livecode > -- Stephen Barncard San Francisco Ca. USA more about sqb From keith at gulfbreezeortholab.com Mon Jul 25 15:08:53 2011 From: keith at gulfbreezeortholab.com (Keith (Gulf Breeze Ortho Lab)) Date: Mon, 25 Jul 2011 14:08:53 -0500 Subject: A Question about Geometry Manager in LiveCode... In-Reply-To: References: <4E2DB5D4.7020000@hyperactivesw.com> Message-ID: <8161200CB44C4368AC45EB7ECFE12F33@KeithPC> Hello All! I am still playing with LiveCode and have a question regarding the Geometry Manager. When I am working on a stack, I set the Geometry Manager to resize several objects. The thing is, after implementing the settings, the objects do not resize until compiled (i.e., into a standalone application). Is this normal? To clarify, the resizing works correctly in my compiled program, but it does not resize while coding in the LiveCode IDE. (Even when clicking on the Run/Browse tool.) Also, does anyone have any feedback regarding the NativeGeometry Addon in the RunRev store? Thanks! - Boo From jacque at hyperactivesw.com Mon Jul 25 15:14:38 2011 From: jacque at hyperactivesw.com (J. Landman Gay) Date: Mon, 25 Jul 2011 14:14:38 -0500 Subject: Field Sizes [was: [ANN] New plugin AAG|LayerComps] In-Reply-To: References: <4E29EAE8.6050302@hyperactivesw.com> <63167200328.20110722174948@ahsoftware.net> <4E2A36B8.10906@hyperactivesw.com> <104176573546.20110722202601@ahsoftware.net> <4E2B171D.8090402@hyperactivesw.com> <177249600890.20110723164309@ahsoftware.net> Message-ID: <4E2DC09E.2090208@hyperactivesw.com> On 7/24/11 1:23 PM, Pete Haworth wrote: > I guess this isn't quite the same thing but why is it that when I create a > field by script, the size of the field is different than when I just drag > one to my stack from the Tools palette. I thought the templates were > supposed to ensure consistency but maybe I'm misunderstanding their purpose When you create by script, the size of the template object is used. When you create from the tool palette, the IDE intercepts and adjusts the sizes to those specified in your preferences. -- Jacqueline Landman Gay | jacque at hyperactivesw.com HyperActive Software | http://www.hyperactivesw.com From jacque at hyperactivesw.com Mon Jul 25 15:16:23 2011 From: jacque at hyperactivesw.com (J. Landman Gay) Date: Mon, 25 Jul 2011 14:16:23 -0500 Subject: stackfiles -- how to use? In-Reply-To: References: <4E2DB5D4.7020000@hyperactivesw.com> Message-ID: <4E2DC107.9030609@hyperactivesw.com> On 7/25/11 1:43 PM, Pete Haworth wrote: > I think I missed the original post of this thread, but perhaps a "start > using" command would take care of all this? If the stackfiles property is included in the inheritance hierarchy, which is what I'm not sure of, then yes. Although in this case, the stack in question is the main standalone stack, so it's always in use anyway. -- Jacqueline Landman Gay | jacque at hyperactivesw.com HyperActive Software | http://www.hyperactivesw.com From andre at andregarzia.com Mon Jul 25 15:32:31 2011 From: andre at andregarzia.com (Andre Garzia) Date: Mon, 25 Jul 2011 16:32:31 -0300 Subject: Field Sizes [was: [ANN] New plugin AAG|LayerComps] In-Reply-To: <4E2DC09E.2090208@hyperactivesw.com> References: <4E29EAE8.6050302@hyperactivesw.com> <63167200328.20110722174948@ahsoftware.net> <4E2A36B8.10906@hyperactivesw.com> <104176573546.20110722202601@ahsoftware.net> <4E2B171D.8090402@hyperactivesw.com> <177249600890.20110723164309@ahsoftware.net> <4E2DC09E.2090208@hyperactivesw.com> Message-ID: On Mon, Jul 25, 2011 at 4:14 PM, J. Landman Gay wrote: > On 7/24/11 1:23 PM, Pete Haworth wrote: > >> I guess this isn't quite the same thing but why is it that when I create a >> field by script, the size of the field is different than when I just drag >> one to my stack from the Tools palette. I thought the templates were >> supposed to ensure consistency but maybe I'm misunderstanding their >> purpose >> > > When you create by script, the size of the template object is used. When > you create from the tool palette, the IDE intercepts and adjusts the sizes > to those specified in your preferences. > > just out of curiosity, what does MC IDE do, any clue? > -- > Jacqueline Landman Gay | jacque at hyperactivesw.com > HyperActive Software | http://www.hyperactivesw.com > > ______________________________**_________________ > use-livecode mailing list > use-livecode at lists.runrev.com > Please visit this url to subscribe, unsubscribe and manage your > subscription preferences: > http://lists.runrev.com/**mailman/listinfo/use-livecode > -- http://www.andregarzia.com All We Do Is Code. From lists.pete at haworths.org Mon Jul 25 15:51:15 2011 From: lists.pete at haworths.org (Pete Haworth) Date: Mon, 25 Jul 2011 12:51:15 -0700 Subject: Field Sizes [was: [ANN] New plugin AAG|LayerComps] In-Reply-To: <4E2DC09E.2090208@hyperactivesw.com> References: <4E29EAE8.6050302@hyperactivesw.com> <63167200328.20110722174948@ahsoftware.net> <4E2A36B8.10906@hyperactivesw.com> <104176573546.20110722202601@ahsoftware.net> <4E2B171D.8090402@hyperactivesw.com> <177249600890.20110723164309@ahsoftware.net> <4E2DC09E.2090208@hyperactivesw.com> Message-ID: Cool, thanks Jacque - I need to look at the references more carefully! Still think it would be nice if the templates obeyed the preference settings though, or at least have a preference setting to achieve that. Pete On Mon, Jul 25, 2011 at 12:14 PM, J. Landman Gay wrote: > On 7/24/11 1:23 PM, Pete Haworth wrote: > >> I guess this isn't quite the same thing but why is it that when I create a >> field by script, the size of the field is different than when I just drag >> one to my stack from the Tools palette. I thought the templates were >> supposed to ensure consistency but maybe I'm misunderstanding their >> purpose >> > > When you create by script, the size of the template object is used. When > you create from the tool palette, the IDE intercepts and adjusts the sizes > to those specified in your preferences. > > -- > Jacqueline Landman Gay | jacque at hyperactivesw.com > HyperActive Software | http://www.hyperactivesw.com > > ______________________________**_________________ > use-livecode mailing list > use-livecode at lists.runrev.com > Please visit this url to subscribe, unsubscribe and manage your > subscription preferences: > http://lists.runrev.com/**mailman/listinfo/use-livecode > From mwieder at ahsoftware.net Mon Jul 25 15:53:36 2011 From: mwieder at ahsoftware.net (Mark Wieder) Date: Mon, 25 Jul 2011 12:53:36 -0700 Subject: Field Sizes [was: [ANN] New plugin AAG|LayerComps] In-Reply-To: References: <4E29EAE8.6050302@hyperactivesw.com> <63167200328.20110722174948@ahsoftware.net> <4E2A36B8.10906@hyperactivesw.com> <104176573546.20110722202601@ahsoftware.net> <4E2B171D.8090402@hyperactivesw.com> <177249600890.20110723164309@ahsoftware.net> <4E2DC09E.2090208@hyperactivesw.com> Message-ID: <161408628625.20110725125336@ahsoftware.net> Andre- Monday, July 25, 2011, 12:32:31 PM, you wrote: > just out of curiosity, what does MC IDE do, any clue? With the MC IDE you draw the object onto the stack rather than dragging it, rather like the RR tool palette of old, so the default size doesn't really come into play. -- -Mark Wieder mwieder at ahsoftware.net From klaus at major.on-rev.com Mon Jul 25 15:56:16 2011 From: klaus at major.on-rev.com (Klaus on-rev) Date: Mon, 25 Jul 2011 21:56:16 +0200 Subject: Field Sizes [was: [ANN] New plugin AAG|LayerComps] In-Reply-To: References: <4E29EAE8.6050302@hyperactivesw.com> <63167200328.20110722174948@ahsoftware.net> <4E2A36B8.10906@hyperactivesw.com> <104176573546.20110722202601@ahsoftware.net> <4E2B171D.8090402@hyperactivesw.com> <177249600890.20110723164309@ahsoftware.net> <4E2DC09E.2090208@hyperactivesw.com> Message-ID: <2424443D-F3CF-478A-83EF-BEEA86F27597@major.on-rev.com> Hi Andre, Am 25.07.2011 um 21:32 schrieb Andre Garzia: > On Mon, Jul 25, 2011 at 4:14 PM, J. Landman Gay wrote: > >> On 7/24/11 1:23 PM, Pete Haworth wrote: >> >>> I guess this isn't quite the same thing but why is it that when I create a >>> field by script, the size of the field is different than when I just drag >>> one to my stack from the Tools palette. I thought the templates were >>> supposed to ensure consistency but maybe I'm misunderstanding their >>> purpose >> When you create by script, the size of the template object is used. When >> you create from the tool palette, the IDE intercepts and adjusts the sizes >> to those specified in your preferences. > just out of curiosity, what does MC IDE do, any clue? no predefined sizes, just select a tool and drag/create an object. Best Klaus -- Klaus Major http://www.major-k.de klaus at major.on-rev.com From andre at andregarzia.com Mon Jul 25 16:01:06 2011 From: andre at andregarzia.com (Andre Garzia) Date: Mon, 25 Jul 2011 17:01:06 -0300 Subject: Field Sizes [was: [ANN] New plugin AAG|LayerComps] In-Reply-To: <2424443D-F3CF-478A-83EF-BEEA86F27597@major.on-rev.com> References: <4E29EAE8.6050302@hyperactivesw.com> <63167200328.20110722174948@ahsoftware.net> <4E2A36B8.10906@hyperactivesw.com> <104176573546.20110722202601@ahsoftware.net> <4E2B171D.8090402@hyperactivesw.com> <177249600890.20110723164309@ahsoftware.net> <4E2DC09E.2090208@hyperactivesw.com> <2424443D-F3CF-478A-83EF-BEEA86F27597@major.on-rev.com> Message-ID: > > > > no predefined sizes, just select a tool and drag/create an object. > Oh I see!!! Thanks guys! > > > Best > > Klaus > > -- > Klaus Major > http://www.major-k.de > klaus at major.on-rev.com > > > _______________________________________________ > use-livecode mailing list > use-livecode at lists.runrev.com > Please visit this url to subscribe, unsubscribe and manage your > subscription preferences: > http://lists.runrev.com/mailman/listinfo/use-livecode > -- http://www.andregarzia.com All We Do Is Code. From bobs at twft.com Mon Jul 25 16:23:54 2011 From: bobs at twft.com (Bob Sneidar) Date: Mon, 25 Jul 2011 13:23:54 -0700 Subject: Field Sizes [was: [ANN] New plugin AAG|LayerComps] In-Reply-To: References: <4E29EAE8.6050302@hyperactivesw.com> <63167200328.20110722174948@ahsoftware.net> <4E2A36B8.10906@hyperactivesw.com> <104176573546.20110722202601@ahsoftware.net> <4E2B171D.8090402@hyperactivesw.com> <177249600890.20110723164309@ahsoftware.net> <4E2DC09E.2090208@hyperactivesw.com> <2424443D-F3CF-478A-83EF-BEEA86F27597@major.on-rev.com> Message-ID: <325D2850-FC83-4DBF-9075-8678FE96B7A0@twft.com> One of the things I was working on was a dropField stack, in which you set up some minimum and maximum sizes, the font and size you want, and then you can pick a column in an SQL table, and the dropfield stack will drop the field onto your main stack card, no smaller than x, no larger than y, capable of containing the SQL data, with font and size set, complete with a label. It was handy for me creating forms, but it has a lot of extraneous stuff in it because I was trying to make my own relational system, so I was futzing with a bunch of properties. If I ever get back to it I think I will trim it down to something universally functional, and maybe submit it as a droptool. Bob On Jul 25, 2011, at 1:01 PM, Andre Garzia wrote: >> >> >> >> no predefined sizes, just select a tool and drag/create an object. >> > > > Oh I see!!! > > Thanks guys! > > >> >> >> Best >> >> Klaus >> >> -- >> Klaus Major >> http://www.major-k.de >> klaus at major.on-rev.com >> >> >> _______________________________________________ >> use-livecode mailing list >> use-livecode at lists.runrev.com >> Please visit this url to subscribe, unsubscribe and manage your >> subscription preferences: >> http://lists.runrev.com/mailman/listinfo/use-livecode >> > > > > -- > http://www.andregarzia.com All We Do Is Code. > _______________________________________________ > use-livecode mailing list > use-livecode at lists.runrev.com > Please visit this url to subscribe, unsubscribe and manage your subscription preferences: > http://lists.runrev.com/mailman/listinfo/use-livecode From brami.serge at gmail.com Mon Jul 25 16:32:39 2011 From: brami.serge at gmail.com (Serge Brami) Date: Mon, 25 Jul 2011 22:32:39 +0200 Subject: pasting text from revbrowser In-Reply-To: <002601cc4a11$57086850$051938f0$@lexiconbridge.com> References: <21BA7EEF-24B5-4F63-A0A5-4E3F5D919F44@gmail.com> <67BA5C1B-2ABD-4181-A8F6-DBC46D7A5536@yahoo.com> <35B0C3DC-C2F2-479D-8025-0996E685BCB0@gmail.com> <002601cc4a11$57086850$051938f0$@lexiconbridge.com> Message-ID: <76E1AF38-5FB2-4574-AD07-046DB976A37F@gmail.com> Hi it happens nothing ... always no returns Le 24 juil. 2011 ? 16:52, Slava Paperno a ?crit : > What happens if you replace char(10) with the string "
"? > > Slava > >> -----Original Message----- >> From: use-livecode-bounces at lists.runrev.com [mailto:use-livecode- >> bounces at lists.runrev.com] On Behalf Of Serge Brami >> Sent: Sunday, July 24, 2011 5:22 AM >> To: How to use LiveCode >> Subject: Re: pasting text from revbrowser >> >> Thanks >> platform = Mac OSX snow leopard >> >> checkreturns return only Found char 10 a return in rev/unix >> >> but if I replace numtochar(10) with CRLF or numtochar(13) or return it > doesnt >> change anything ... >> >> Le 23 juil. 2011 ? 16:03, Jim Ault a ?crit : >> >>> On Jul 23, 2011, at 3:48 AM, Serge Brami wrote: >>>> I try to get the contents of a revbrowser window and paste it into >>>> a livecode field the web page is a banking chart >>>> >>>> first I select manually the text in the revbrowser window >>>> >>>> Then I Use revbrowserGet selected put the contents into myvariable > and >> paste it into a field (using :set the htmltext of field myfield to > myvariable) >>>> the problem is that all the return characters disappears >>>> >>>> Even if i copy manually the text and paste it directly in a field i > get the same >> thing >>>> If I copy the text manually and paste it into a Word processor software > it >> works fine the return characters don't disappear >>> >>> You have not specified which platform, but try this test >>> >>> on checkReturns >>> if the clipboardData contains numtochar(13) then >>> answer "Found char 13, a return on a Mac" >>> end if >>> if the clipboardData contains numtochar(10) then >>> answer "Found char 10, a return in Rev/ Unix" >>> end if >>> if the clipboardData contains CRLF then >>> answer "Found char 1310, a return-line feed in Windows" >>> end if >>> if the clipboardData contains ">> answer "Found >> end if >>> if the clipboardData contains ">> answer "Found char >> end if >>> end checkReturns >>> >>> Also, look up CRLF in the dictionary. >>> Another factor in Html is that the
may be a tag that is used to > create the >> effect of a return char on the web page. >>> >>> A word processor program is designed to do many conversions for the > user's >> convenience, such as clipboard conversions of text and graphics. >>> >>> Hope this helps. >>> >>>> >>>> >>> >>> Jim Ault >>> Las Vegas >>> >>> >>> >>> _______________________________________________ >>> use-livecode mailing list >>> use-livecode at lists.runrev.com >>> Please visit this url to subscribe, unsubscribe and manage your > subscription >> preferences: >>> http://lists.runrev.com/mailman/listinfo/use-livecode >> >> >> _______________________________________________ >> use-livecode mailing list >> use-livecode at lists.runrev.com >> Please visit this url to subscribe, unsubscribe and manage your > subscription >> preferences: >> http://lists.runrev.com/mailman/listinfo/use-livecode > > > _______________________________________________ > use-livecode mailing list > use-livecode at lists.runrev.com > Please visit this url to subscribe, unsubscribe and manage your subscription preferences: > http://lists.runrev.com/mailman/listinfo/use-livecode From jacque at hyperactivesw.com Mon Jul 25 16:40:13 2011 From: jacque at hyperactivesw.com (J. Landman Gay) Date: Mon, 25 Jul 2011 15:40:13 -0500 Subject: A Question about Geometry Manager in LiveCode... In-Reply-To: <8161200CB44C4368AC45EB7ECFE12F33@KeithPC> References: <4E2DB5D4.7020000@hyperactivesw.com> <8161200CB44C4368AC45EB7ECFE12F33@KeithPC> Message-ID: <4E2DD4AD.2000805@hyperactivesw.com> On 7/25/11 2:08 PM, Keith (Gulf Breeze Ortho Lab) wrote: > Hello All! > > I am still playing with LiveCode and have a question regarding the > Geometry Manager. When I am working on a stack, I set the Geometry > Manager to resize several objects. The thing is, after implementing the > settings, the objects do not resize until compiled (i.e., into a > standalone application). Is this normal? To clarify, the resizing works > correctly in my compiled program, but it does not resize while coding in > the LiveCode IDE. (Even when clicking on the Run/Browse tool.) It should work in the IDE. Make sure the Messages button in the toolbar is not bold. If it is, your stack won't get any messages. -- Jacqueline Landman Gay | jacque at hyperactivesw.com HyperActive Software | http://www.hyperactivesw.com From keith at gulfbreezeortholab.com Mon Jul 25 16:43:35 2011 From: keith at gulfbreezeortholab.com (Keith (Gulf Breeze Ortho Lab)) Date: Mon, 25 Jul 2011 15:43:35 -0500 Subject: Sorry: A Question about Geometry Manager in LiveCode... Message-ID: <3BE61BDA06174F59AB148C3B752B0B21@KeithPC> Sorry folks! New to this... I think my last post went into the wrong thread (because I replied to a message--my bad)... 8-( Here it is again as a separate post: Hello All! I am still playing with LiveCode and have a question regarding the Geometry Manager. When I am working on a stack, I set the Geometry Manager to resize several objects. The thing is, after implementing the settings, the objects do not resize until compiled (i.e., into a standalone application). Is this normal? To clarify, the resizing works correctly in my compiled program, but it does not resize while coding in the LiveCode IDE. (Even when clicking on the Run/Browse tool.) Also, does anyone have any feedback regarding the NativeGeometry Addon in the RunRev store? Thanks! - Boo From keith at gulfbreezeortholab.com Mon Jul 25 16:51:48 2011 From: keith at gulfbreezeortholab.com (Keith (Gulf Breeze Ortho Lab)) Date: Mon, 25 Jul 2011 15:51:48 -0500 Subject: A Question about Geometry Manager in LiveCode... In-Reply-To: <4E2DD4AD.2000805@hyperactivesw.com> References: <4E2DB5D4.7020000@hyperactivesw.com> <8161200CB44C4368AC45EB7ECFE12F33@KeithPC> <4E2DD4AD.2000805@hyperactivesw.com> Message-ID: Hi Jacqueline, Thanks for your reply. (FYI: I just reposted this original message because my previous message used a reply message and was therefore placed in the wrong thread within the message archive.) Actually, the Messages button is not selected/bold. And it is still not working in the IDE; only when compiled as a standalone. I am using LiveCode for Windows. Any other ideas? Any other folks out there notice this issue? Thanks, - Boo ----- Original Message ----- From: "J. Landman Gay" To: "How to use LiveCode" Sent: Monday, July 25, 2011 3:40 PM Subject: Re: A Question about Geometry Manager in LiveCode... > On 7/25/11 2:08 PM, Keith (Gulf Breeze Ortho Lab) wrote: >> Hello All! >> >> I am still playing with LiveCode and have a question regarding the >> Geometry Manager. When I am working on a stack, I set the Geometry >> Manager to resize several objects. The thing is, after implementing the >> settings, the objects do not resize until compiled (i.e., into a >> standalone application). Is this normal? To clarify, the resizing works >> correctly in my compiled program, but it does not resize while coding in >> the LiveCode IDE. (Even when clicking on the Run/Browse tool.) > > It should work in the IDE. Make sure the Messages button in the toolbar is > not bold. If it is, your stack won't get any messages. > > -- > Jacqueline Landman Gay | jacque at hyperactivesw.com > HyperActive Software | http://www.hyperactivesw.com > > _______________________________________________ > use-livecode mailing list > use-livecode at lists.runrev.com > Please visit this url to subscribe, unsubscribe and manage your > subscription preferences: > http://lists.runrev.com/mailman/listinfo/use-livecode > From keith at gulfbreezeortholab.com Mon Jul 25 17:02:59 2011 From: keith at gulfbreezeortholab.com (Keith (Gulf Breeze Ortho Lab)) Date: Mon, 25 Jul 2011 16:02:59 -0500 Subject: Sorry: A Question about Geometry Manager in LiveCode... In-Reply-To: <3BE61BDA06174F59AB148C3B752B0B21@KeithPC> References: <3BE61BDA06174F59AB148C3B752B0B21@KeithPC> Message-ID: <6819375CE4FD48FC9DC9A4A8369D2047@KeithPC> Solved! Okay; here was the problem. ;-) I downloaded and installed the trial version of NativeGeometry the other day... The plugin was loading each time I started LiveCode and was obviously causing some kind of conflict. I went into the LiveCode Plugin Settings dialog and set the plugin so it would not open when LiveCode starts. Problem fixed. 8-) ----- Original Message ----- From: "Keith (Gulf Breeze Ortho Lab)" To: "How to use LiveCode" Sent: Monday, July 25, 2011 3:43 PM Subject: Sorry: A Question about Geometry Manager in LiveCode... > Sorry folks! New to this... I think my last post went into the wrong > thread (because I replied to a message--my bad)... 8-( > > Here it is again as a separate post: > Hello All! > > I am still playing with LiveCode and have a question regarding the > Geometry Manager. When I am working on a stack, I set the Geometry Manager > to resize several objects. The thing is, after implementing the settings, > the objects > do not resize until compiled (i.e., into a standalone application). Is > this normal? To clarify, the resizing works correctly in my compiled > program, but it does not resize while coding in the LiveCode IDE. (Even > when clicking on > the Run/Browse tool.) > > Also, does anyone have any feedback regarding the NativeGeometry Addon in > the RunRev store? > > Thanks! > > - Boo > _______________________________________________ > use-livecode mailing list > use-livecode at lists.runrev.com > Please visit this url to subscribe, unsubscribe and manage your > subscription preferences: > http://lists.runrev.com/mailman/listinfo/use-livecode > From bobs at twft.com Mon Jul 25 17:26:27 2011 From: bobs at twft.com (Bob Sneidar) Date: Mon, 25 Jul 2011 14:26:27 -0700 Subject: A Question about Geometry Manager in LiveCode... In-Reply-To: References: <4E2DB5D4.7020000@hyperactivesw.com> <8161200CB44C4368AC45EB7ECFE12F33@KeithPC> <4E2DD4AD.2000805@hyperactivesw.com> Message-ID: <00DA0FB3-8107-4720-AFB9-8B95F7078ECD@twft.com> Are you intercepting any messages and not passing them in an attempt to control your object geometry? If so, pass the messages after you handle them and see what happens. Just a guess. Bob On Jul 25, 2011, at 1:51 PM, Keith (Gulf Breeze Ortho Lab) wrote: > Hi Jacqueline, > > Thanks for your reply. (FYI: I just reposted this original message because my previous message used a reply message and was therefore placed in the wrong thread within the message archive.) > > Actually, the Messages button is not selected/bold. And it is still not working in the IDE; only when compiled as a standalone. I am using LiveCode for Windows. Any other ideas? Any other folks out there notice this issue? > > Thanks, > > - Boo > > ----- Original Message ----- From: "J. Landman Gay" > To: "How to use LiveCode" > Sent: Monday, July 25, 2011 3:40 PM > Subject: Re: A Question about Geometry Manager in LiveCode... > > >> On 7/25/11 2:08 PM, Keith (Gulf Breeze Ortho Lab) wrote: >>> Hello All! >>> >>> I am still playing with LiveCode and have a question regarding the >>> Geometry Manager. When I am working on a stack, I set the Geometry >>> Manager to resize several objects. The thing is, after implementing the >>> settings, the objects do not resize until compiled (i.e., into a >>> standalone application). Is this normal? To clarify, the resizing works >>> correctly in my compiled program, but it does not resize while coding in >>> the LiveCode IDE. (Even when clicking on the Run/Browse tool.) >> >> It should work in the IDE. Make sure the Messages button in the toolbar is not bold. If it is, your stack won't get any messages. >> >> -- >> Jacqueline Landman Gay | jacque at hyperactivesw.com >> HyperActive Software | http://www.hyperactivesw.com >> >> _______________________________________________ >> use-livecode mailing list >> use-livecode at lists.runrev.com >> Please visit this url to subscribe, unsubscribe and manage your subscription preferences: >> http://lists.runrev.com/mailman/listinfo/use-livecode > > > _______________________________________________ > use-livecode mailing list > use-livecode at lists.runrev.com > Please visit this url to subscribe, unsubscribe and manage your subscription preferences: > http://lists.runrev.com/mailman/listinfo/use-livecode From niconiko at gmail.com Mon Jul 25 17:32:23 2011 From: niconiko at gmail.com (Nicolas Cueto) Date: Tue, 26 Jul 2011 06:32:23 +0900 Subject: standalone won't start on Win2K In-Reply-To: <8557A39C-7591-4D22-BFFF-6D7240E8FF35@economy-x-talk.com> References: <8557A39C-7591-4D22-BFFF-6D7240E8FF35@economy-x-talk.com> Message-ID: > Do you have SP4 installed? I didn't. But now I do. And, problem gone. Thanks Mark! -- Nicolas Cueto From kray at sonsothunder.com Mon Jul 25 17:33:02 2011 From: kray at sonsothunder.com (Ken Ray) Date: Mon, 25 Jul 2011 16:33:02 -0500 Subject: Field Sizes [was: [ANN] New plugin AAG|LayerComps] In-Reply-To: References: <4E29EAE8.6050302@hyperactivesw.com> <63167200328.20110722174948@ahsoftware.net> <4E2A36B8.10906@hyperactivesw.com> <104176573546.20110722202601@ahsoftware.net> <4E2B171D.8090402@hyperactivesw.com> <177249600890.20110723164309@ahsoftware.net> <4E2DC09E.2090208@hyperactivesw.com> Message-ID: <52352F4F-6B70-43D2-BA50-7F70E18B02A9@sonsothunder.com> On Jul 25, 2011, at 2:32 PM, Andre Garzia wrote: > On Mon, Jul 25, 2011 at 4:14 PM, J. Landman Gay wrote: > >> On 7/24/11 1:23 PM, Pete Haworth wrote: >> >>> I guess this isn't quite the same thing but why is it that when I create a >>> field by script, the size of the field is different than when I just drag >>> one to my stack from the Tools palette. I thought the templates were >>> supposed to ensure consistency but maybe I'm misunderstanding their >>> purpose >>> >> >> When you create by script, the size of the template object is used. When >> you create from the tool palette, the IDE intercepts and adjusts the sizes >> to those specified in your preferences. >> >> > just out of curiosity, what does MC IDE do, any clue? It doesn't do anything... since you aren't dragging a fully formed object onto your card and you are just given the tool to create the object you want, it will be whatever size you make it. That's on the list of things to add to the MC IDE - being able to set the "HIG size" of an object... Ken Ray Sons of Thunder Software, Inc. Email: kray at sonsothunder.com Web Site: http://www.sonsothunder.com/ From kray at sonsothunder.com Mon Jul 25 17:34:06 2011 From: kray at sonsothunder.com (Ken Ray) Date: Mon, 25 Jul 2011 16:34:06 -0500 Subject: Field Sizes [was: [ANN] New plugin AAG|LayerComps] In-Reply-To: <325D2850-FC83-4DBF-9075-8678FE96B7A0@twft.com> References: <4E29EAE8.6050302@hyperactivesw.com> <63167200328.20110722174948@ahsoftware.net> <4E2A36B8.10906@hyperactivesw.com> <104176573546.20110722202601@ahsoftware.net> <4E2B171D.8090402@hyperactivesw.com> <177249600890.20110723164309@ahsoftware.net> <4E2DC09E.2090208@hyperactivesw.com> <2424443D-F3CF-478A-83EF-BEEA86F27597@major.on-rev.com> <325D2850-FC83-4DBF-9075-8678FE96B7A0@twft.com> Message-ID: <5A0E3DA4-44E1-47FB-A982-5540B2272B0E@sonsothunder.com> On Jul 25, 2011, at 3:23 PM, Bob Sneidar wrote: > One of the things I was working on was a dropField stack, in which you set up some minimum and maximum sizes, the font and size you want, and then you can pick a column in an SQL table, and the dropfield stack will drop the field onto your main stack card, no smaller than x, no larger than y, capable of containing the SQL data, with font and size set, complete with a label. It was handy for me creating forms, but it has a lot of extraneous stuff in it because I was trying to make my own relational system, so I was futzing with a bunch of properties. If I ever get back to it I think I will trim it down to something universally functional, and maybe submit it as a droptool. Sounds great, Bob! Looking forward to it... Ken Ray Sons of Thunder Software, Inc. Email: kray at sonsothunder.com Web Site: http://www.sonsothunder.com/ From keith at gulfbreezeortholab.com Mon Jul 25 17:38:40 2011 From: keith at gulfbreezeortholab.com (Keith (Gulf Breeze Ortho Lab)) Date: Mon, 25 Jul 2011 16:38:40 -0500 Subject: A Question about Geometry Manager in LiveCode... In-Reply-To: <00DA0FB3-8107-4720-AFB9-8B95F7078ECD@twft.com> References: <4E2DB5D4.7020000@hyperactivesw.com> <8161200CB44C4368AC45EB7ECFE12F33@KeithPC><4E2DD4AD.2000805@hyperactivesw.com> <00DA0FB3-8107-4720-AFB9-8B95F7078ECD@twft.com> Message-ID: Hi Bob, The other day I installed the trial version of the NativeGeometry plugin, and apparently it was causing a conflict. I disabled it and now everything works okay. 8-) Thanks! - Boo ----- Original Message ----- From: "Bob Sneidar" To: "How to use LiveCode" Sent: Monday, July 25, 2011 4:26 PM Subject: Re: A Question about Geometry Manager in LiveCode... > Are you intercepting any messages and not passing them in an attempt to > control your object geometry? If so, pass the messages after you handle > them and see what happens. Just a guess. > > Bob > > > On Jul 25, 2011, at 1:51 PM, Keith (Gulf Breeze Ortho Lab) wrote: > >> Hi Jacqueline, >> >> Thanks for your reply. (FYI: I just reposted this original message >> because my previous message used a reply message and was therefore placed >> in the wrong thread within the message archive.) >> >> Actually, the Messages button is not selected/bold. And it is still not >> working in the IDE; only when compiled as a standalone. I am using >> LiveCode for Windows. Any other ideas? Any other folks out there notice >> this issue? >> >> Thanks, >> >> - Boo >> >> ----- Original Message ----- From: "J. Landman Gay" >> >> To: "How to use LiveCode" >> Sent: Monday, July 25, 2011 3:40 PM >> Subject: Re: A Question about Geometry Manager in LiveCode... >> >> >>> On 7/25/11 2:08 PM, Keith (Gulf Breeze Ortho Lab) wrote: >>>> Hello All! >>>> >>>> I am still playing with LiveCode and have a question regarding the >>>> Geometry Manager. When I am working on a stack, I set the Geometry >>>> Manager to resize several objects. The thing is, after implementing the >>>> settings, the objects do not resize until compiled (i.e., into a >>>> standalone application). Is this normal? To clarify, the resizing works >>>> correctly in my compiled program, but it does not resize while coding >>>> in >>>> the LiveCode IDE. (Even when clicking on the Run/Browse tool.) >>> >>> It should work in the IDE. Make sure the Messages button in the toolbar >>> is not bold. If it is, your stack won't get any messages. >>> >>> -- >>> Jacqueline Landman Gay | jacque at hyperactivesw.com >>> HyperActive Software | http://www.hyperactivesw.com >>> >>> _______________________________________________ >>> use-livecode mailing list >>> use-livecode at lists.runrev.com >>> Please visit this url to subscribe, unsubscribe and manage your >>> subscription preferences: >>> http://lists.runrev.com/mailman/listinfo/use-livecode >> >> >> _______________________________________________ >> use-livecode mailing list >> use-livecode at lists.runrev.com >> Please visit this url to subscribe, unsubscribe and manage your >> subscription preferences: >> http://lists.runrev.com/mailman/listinfo/use-livecode > > > _______________________________________________ > use-livecode mailing list > use-livecode at lists.runrev.com > Please visit this url to subscribe, unsubscribe and manage your > subscription preferences: > http://lists.runrev.com/mailman/listinfo/use-livecode > From keith at gulfbreezeortholab.com Mon Jul 25 17:51:00 2011 From: keith at gulfbreezeortholab.com (Keith (Gulf Breeze Ortho Lab)) Date: Mon, 25 Jul 2011 16:51:00 -0500 Subject: What is the fastest database? Message-ID: Hello, I have approximately 25 MB of data (plain text) that consists of approximately 140,000 different entries (one per line--pipe delimited). I am working on an application to quickly search through the data not unlike the LiveCode Dictionary application. However, it will be necessary for my users to be able to edit any entry at will. For a desktop application, what database would you recommend for the job? I can use a flat file database, but the problem is that all 25 MB of data must be loaded into memory... What about SQLite? Valencia? Speed is of the essence. FYI: In the past I used an Access database, but it was too sluggish for my purposes. Any recommendations/feedback is most welcome. Thanks. LiveCode is awesome! Sincerely, - Boo From massung at gmail.com Mon Jul 25 18:00:29 2011 From: massung at gmail.com (Jeff Massung) Date: Mon, 25 Jul 2011 16:00:29 -0600 Subject: What is the fastest database? In-Reply-To: References: Message-ID: That amount of data is ridiculously small. If you don't care about many thousands of concurrent connections and having to transform the data set periodically over time, then SQLite3 is probably the easiest to use, but really, for that tiny amount of data, *any* database application will work. I've used Access (from 10+ years ago) with > 1 GB data sets and never had a performance problem. You just to to make sure you setup the right indexes, foreign keys, and turn off uniqueness where they really don't matter. Jeff M. From keith at gulfbreezeortholab.com Mon Jul 25 18:07:36 2011 From: keith at gulfbreezeortholab.com (Keith (Gulf Breeze Ortho Lab)) Date: Mon, 25 Jul 2011 17:07:36 -0500 Subject: What is the fastest database? In-Reply-To: References: Message-ID: <8DA04F7BE2114D50B3A6B2D473033047@KeithPC> Hi Jeff, The thing is is that the database will grow significantly over time. Also, I might want to eventually move it online. Therefore, I need the best solution to cover all bases for the present and the future... Thanks, - Boo ----- Original Message ----- From: "Jeff Massung" To: "How to use LiveCode" Sent: Monday, July 25, 2011 5:00 PM Subject: Re: What is the fastest database? > That amount of data is ridiculously small. If you don't care about many > thousands of concurrent connections and having to transform the data set > periodically over time, then SQLite3 is probably the easiest to use, but > really, for that tiny amount of data, *any* database application will > work. > I've used Access (from 10+ years ago) with > 1 GB data sets and never had > a > performance problem. You just to to make sure you setup the right indexes, > foreign keys, and turn off uniqueness where they really don't matter. > > Jeff M. > _______________________________________________ > use-livecode mailing list > use-livecode at lists.runrev.com > Please visit this url to subscribe, unsubscribe and manage your > subscription preferences: > http://lists.runrev.com/mailman/listinfo/use-livecode > From massung at gmail.com Mon Jul 25 18:13:21 2011 From: massung at gmail.com (Jeff Massung) Date: Mon, 25 Jul 2011 16:13:21 -0600 Subject: What is the fastest database? In-Reply-To: <8DA04F7BE2114D50B3A6B2D473033047@KeithPC> References: <8DA04F7BE2114D50B3A6B2D473033047@KeithPC> Message-ID: If you are thinking web and growing for the future, then it's hard to beat PostgreSQL, which also comes with some nice web-admin tools for viewing the database, etc. MySQL - I hear - is equally nice, but I haven't used it. You may just want to quickly test your work with SQLite3, which is near-identical to MySQL as far as query syntax goes, and if you think you like it, then migrate to using MySQL. I also recommend downloading Navcat Lite as a tool for viewing whatever database you decide to use. It's very handy. Jeff M. On Mon, Jul 25, 2011 at 4:07 PM, Keith (Gulf Breeze Ortho Lab) < keith at gulfbreezeortholab.com> wrote: > Hi Jeff, > > The thing is is that the database will grow significantly over time. Also, > I might want to eventually move it online. Therefore, I need the best > solution to cover all bases for the present and the future... > > Thanks, > > - Boo > > ----- Original Message ----- From: "Jeff Massung" > To: "How to use LiveCode" > Sent: Monday, July 25, 2011 5:00 PM > Subject: Re: What is the fastest database? > > > That amount of data is ridiculously small. If you don't care about many >> thousands of concurrent connections and having to transform the data set >> periodically over time, then SQLite3 is probably the easiest to use, but >> really, for that tiny amount of data, *any* database application will >> work. >> I've used Access (from 10+ years ago) with > 1 GB data sets and never had >> a >> performance problem. You just to to make sure you setup the right indexes, >> foreign keys, and turn off uniqueness where they really don't matter. >> >> Jeff M. >> ______________________________**_________________ >> use-livecode mailing list >> use-livecode at lists.runrev.com >> Please visit this url to subscribe, unsubscribe and manage your >> subscription preferences: >> http://lists.runrev.com/**mailman/listinfo/use-livecode >> >> > > ______________________________**_________________ > use-livecode mailing list > use-livecode at lists.runrev.com > Please visit this url to subscribe, unsubscribe and manage your > subscription preferences: > http://lists.runrev.com/**mailman/listinfo/use-livecode > From lists.pete at haworths.org Mon Jul 25 18:29:56 2011 From: lists.pete at haworths.org (Pete Haworth) Date: Mon, 25 Jul 2011 15:29:56 -0700 Subject: What is the fastest database? In-Reply-To: References: <8DA04F7BE2114D50B3A6B2D473033047@KeithPC> Message-ID: All good stuff. SQLite is great for prototyping applications and in production use for single user applications, but it will have problems dealing with networked file access and/or a high volume of multi-user updating activity (due to locking issues). Pete On Mon, Jul 25, 2011 at 3:13 PM, Jeff Massung wrote: > If you are thinking web and growing for the future, then it's hard to beat > PostgreSQL, which also comes with some nice web-admin tools for viewing the > database, etc. MySQL - I hear - is equally nice, but I haven't used it. > > You may just want to quickly test your work with SQLite3, which is > near-identical to MySQL as far as query syntax goes, and if you think you > like it, then migrate to using MySQL. > > I also recommend downloading Navcat Lite as a tool for viewing whatever > database you decide to use. It's very handy. > > Jeff M. > > On Mon, Jul 25, 2011 at 4:07 PM, Keith (Gulf Breeze Ortho Lab) < > keith at gulfbreezeortholab.com> wrote: > > > Hi Jeff, > > > > The thing is is that the database will grow significantly over time. > Also, > > I might want to eventually move it online. Therefore, I need the best > > solution to cover all bases for the present and the future... > > > > Thanks, > > > > - Boo > > > > ----- Original Message ----- From: "Jeff Massung" > > To: "How to use LiveCode" > > Sent: Monday, July 25, 2011 5:00 PM > > Subject: Re: What is the fastest database? > > > > > > That amount of data is ridiculously small. If you don't care about many > >> thousands of concurrent connections and having to transform the data set > >> periodically over time, then SQLite3 is probably the easiest to use, but > >> really, for that tiny amount of data, *any* database application will > >> work. > >> I've used Access (from 10+ years ago) with > 1 GB data sets and never > had > >> a > >> performance problem. You just to to make sure you setup the right > indexes, > >> foreign keys, and turn off uniqueness where they really don't matter. > >> > >> Jeff M. > >> ______________________________**_________________ > >> use-livecode mailing list > >> use-livecode at lists.runrev.com > >> Please visit this url to subscribe, unsubscribe and manage your > >> subscription preferences: > >> http://lists.runrev.com/**mailman/listinfo/use-livecode< > http://lists.runrev.com/mailman/listinfo/use-livecode> > >> > >> > > > > ______________________________**_________________ > > use-livecode mailing list > > use-livecode at lists.runrev.com > > Please visit this url to subscribe, unsubscribe and manage your > > subscription preferences: > > http://lists.runrev.com/**mailman/listinfo/use-livecode< > http://lists.runrev.com/mailman/listinfo/use-livecode> > > > _______________________________________________ > use-livecode mailing list > use-livecode at lists.runrev.com > Please visit this url to subscribe, unsubscribe and manage your > subscription preferences: > http://lists.runrev.com/mailman/listinfo/use-livecode > From terryhass at tdheng.com Mon Jul 25 18:40:54 2011 From: terryhass at tdheng.com (TERRY HASS) Date: Mon, 25 Jul 2011 17:40:54 -0500 Subject: GoldTrader101 Released Post #2 Message-ID: Hi All, I don't know why my previous post did not come through with a clickable link, so here is another try. www.tdheng.com I know this link is not clickable, but copying it into your browser will take you to the app's page. There is a link on that page which will take you to the App Store. Thanks for looking. Terry From andre at andregarzia.com Mon Jul 25 18:47:15 2011 From: andre at andregarzia.com (Andre Garzia) Date: Mon, 25 Jul 2011 19:47:15 -0300 Subject: GoldTrader101 Released Post #2 In-Reply-To: References: Message-ID: Cool Terry! Congratulations! On Mon, Jul 25, 2011 at 7:40 PM, TERRY HASS wrote: > Hi All, > > I don't know why my previous post did not come through with a clickable > link, so here is another try. > > www.tdheng.com > > I know this link is not clickable, but copying it into your browser will > take you to the app's page. There is a link on that page which will take you > to the App Store. > > Thanks for looking. > Terry > > > > _______________________________________________ > use-livecode mailing list > use-livecode at lists.runrev.com > Please visit this url to subscribe, unsubscribe and manage your > subscription preferences: > http://lists.runrev.com/mailman/listinfo/use-livecode > -- http://www.andregarzia.com All We Do Is Code. From bobs at twft.com Mon Jul 25 18:49:41 2011 From: bobs at twft.com (Bob Sneidar) Date: Mon, 25 Jul 2011 15:49:41 -0700 Subject: What is the fastest database? In-Reply-To: References: Message-ID: <5245936C-4DA4-4998-8E9D-DEB9F5A5F325@twft.com> If you plan on moving this to an external server, then you should not allow direct access to your SQL server (although I do so myself in development using On-Rev). You will probably want to access it through a web based script of some sort to firewall your SQL data. This will prevent things like SQL injection attacks and such. If this is your eventual goal, I would advise developing your application using this methodology right from the start. my 2? Bob On Jul 25, 2011, at 2:51 PM, Keith (Gulf Breeze Ortho Lab) wrote: > Hello, > > I have approximately 25 MB of data (plain text) that consists of approximately 140,000 different entries (one per line--pipe delimited). I am working on an application to quickly search through the data not unlike the LiveCode Dictionary application. However, it will be necessary for my users to be able to edit any entry at will. > > For a desktop application, what database would you recommend for the job? I can use a flat file database, but the problem is that all 25 MB of data must be loaded into memory... What about SQLite? Valencia? Speed is of the essence. > > FYI: In the past I used an Access database, but it was too sluggish for my purposes. > > Any recommendations/feedback is most welcome. Thanks. > > LiveCode is awesome! > > Sincerely, > > - Boo > _______________________________________________ > use-livecode mailing list > use-livecode at lists.runrev.com > Please visit this url to subscribe, unsubscribe and manage your subscription preferences: > http://lists.runrev.com/mailman/listinfo/use-livecode From bobs at twft.com Mon Jul 25 18:50:40 2011 From: bobs at twft.com (Bob Sneidar) Date: Mon, 25 Jul 2011 15:50:40 -0700 Subject: GoldTrader101 Released Post #2 In-Reply-To: References: Message-ID: <29728616-557D-4ED7-8F41-7C355D9447B0@twft.com> Works for me, but I use Apple Mail which converts anything that looks like it might be a clickable link into a real one. Bob On Jul 25, 2011, at 3:40 PM, TERRY HASS wrote: > Hi All, > > I don't know why my previous post did not come through with a clickable link, so here is another try. > > www.tdheng.com > > I know this link is not clickable, but copying it into your browser will take you to the app's page. There is a link on that page which will take you to the App Store. > > Thanks for looking. > Terry > > > > _______________________________________________ > use-livecode mailing list > use-livecode at lists.runrev.com > Please visit this url to subscribe, unsubscribe and manage your subscription preferences: > http://lists.runrev.com/mailman/listinfo/use-livecode From ruslan_zasukhin at valentina-db.com Mon Jul 25 18:57:56 2011 From: ruslan_zasukhin at valentina-db.com (Ruslan Zasukhin) Date: Tue, 26 Jul 2011 01:57:56 +0300 Subject: What is the fastest database? In-Reply-To: Message-ID: On 7/26/11 12:51 AM, "Keith (Gulf Breeze Ortho Lab)" wrote: > Hello, > > I have approximately 25 MB of data (plain text) that consists of approximately > 140,000 different entries (one per line--pipe delimited). I am working on an > application to quickly search through the data not unlike the LiveCode > Dictionary application. However, it will be necessary for my users to be able > to edit any entry at will. > > For a desktop application, what database would you recommend for the job? I > can use a flat file database, but the problem is that all 25 MB of data must > be loaded into memory... What about SQLite? Valencia? Speed is of the essence. > > FYI: In the past I used an Access database, but it was too sluggish for my > purposes. > > Any recommendations/feedback is most welcome. Thanks. Hi Keith, Our Valentina database :-) 1) It is faster in 50-100 times (usually) of any RDBMS as SqlLite, mySQL, Postgre, MS SQL, Access. Because Valentina DB is columnar, and because of many other things. 2) Vertica is good but they say much much more expensive. If you need more details let me know. May be you can find useful this page http://www.paradigmasoft.com/en/testimonials 3) in fact, 25MB and 140k records is quite small database :-) Why you think you need super-speed? Our users did have e.g. 40 million records, 30-40GB and more... -- Best regards, Ruslan Zasukhin VP Engineering and New Technology Paradigma Software, Inc Valentina - Joining Worlds of Information http://www.paradigmasoft.com [I feel the need: the need for speed] From capellan2000 at gmail.com Mon Jul 25 19:57:31 2011 From: capellan2000 at gmail.com (Alejandro Tejada) Date: Mon, 25 Jul 2011 19:57:31 -0400 Subject: Exact zooming/resizing of stacks and groups Message-ID: Hi Fred, on Sun Jul 24 2011 Fred Moyer wrote: > Has anyone done the heavy lifting to figure out > how to do an exact resize/zoom of a stack or a > group inside of a stack? [snip] Actually, I know about specific pieces of code to resize images and graphics, but have not seen a complete solution. Do you want to collaborate to create this library? :-) Al From capellan2000 at gmail.com Mon Jul 25 20:12:32 2011 From: capellan2000 at gmail.com (Alejandro Tejada) Date: Mon, 25 Jul 2011 17:12:32 -0700 (PDT) Subject: What is the fastest database? In-Reply-To: References: Message-ID: <1311639152642-3694491.post@n4.nabble.com> Hi Keith, Keith (Gulf Breeze Ortho Lab) wrote: > > I have approximately 25 MB of data (plain text) that consists of > approximately 140,000 different entries (one per line--pipe delimited). I > am working on an application to quickly search through the data not unlike > the LiveCode Dictionary application. However, it will be necessary for my > users to be able to edit any entry at will. > I understand you perfectly, when you say that you do not want to load the complete database in memory. Could you compress the data??? This could reduce significantly it's memory footprint. Are you going to port this application to mobile devices like Android and iPad???? This is vital to know beforehand. Have a nice day! Al -- View this message in context: http://runtime-revolution.278305.n4.nabble.com/What-is-the-fastest-database-tp3694246p3694491.html Sent from the Revolution - User mailing list archive at Nabble.com. From lists.pete at haworths.org Mon Jul 25 20:53:20 2011 From: lists.pete at haworths.org (Pete Haworth) Date: Mon, 25 Jul 2011 17:53:20 -0700 Subject: Importing an image Message-ID: I have a png file that I want to get into a stack using the import command. I issue the command from the message box: "import eps from file "" as soon as I hit return, Livecode quits. This is a png file that was created by anLC export snapshot command and the resized in Photoshop. Any know problems with importing PNG files? Pete From tsj at unimelb.edu.au Mon Jul 25 21:08:22 2011 From: tsj at unimelb.edu.au (Terry Judd) Date: Tue, 26 Jul 2011 11:08:22 +1000 Subject: Importing an image In-Reply-To: Message-ID: On 26/07/2011 10:53 AM, "Pete Haworth" wrote: > I have a png file that I want to get into a stack using the import command. > I issue the command from the message box: > > "import eps from file "" Pete - I think you should be using 'paint' rather than 'eps' import paint from file Using 'eps' crashes Livecode for me as well with a PNG file - 'paint' works fine however. Terry... > > as soon as I hit return, Livecode quits. This is a png file that was > created by anLC export snapshot command and the resized in Photoshop. > > Any know problems with importing PNG files? > > > Pete > _______________________________________________ > use-livecode mailing list > use-livecode at lists.runrev.com > Please visit this url to subscribe, unsubscribe and manage your subscription > preferences: > http://lists.runrev.com/mailman/listinfo/use-livecode > -- Dr Terry Judd | Senior Lecturer in Medical Education Medical Education Unit Melbourne Medical School The University of Melbourne From capellan2000 at gmail.com Mon Jul 25 21:38:10 2011 From: capellan2000 at gmail.com (Alejandro Tejada) Date: Mon, 25 Jul 2011 18:38:10 -0700 (PDT) Subject: Writing to and from a binary file? In-Reply-To: <4E2A5A8B.4050902@pdslabs.net> References: <073B71C7-EAE1-49BD-A907-777D68A9B7F7@economy-x-talk.com> <4E2A5A8B.4050902@pdslabs.net> Message-ID: <1311644290259-3694622.post@n4.nabble.com> Hi Phil, Phil Davis-5 wrote: > >> Here is a code snippet I shared on the use-revolution list almost 2 years >> ago. >> It shows how to cut an image up into 10 'frames': >> http://www.mail-archive.com/use-revolution%40lists.runrev.com/msg124691.html > Great work! Now, just out of curiosity How could I modify your code, so it cuts an image based in an specific characteristic? For example, suppose that I want to cut an image of the alphabet in their single letters: ABCDEFGHIJKLMNOPQRSTUVWXYZ The image is horizontal and I want to split it by script in their individual letters. Thanks in advance! Al -- View this message in context: http://runtime-revolution.278305.n4.nabble.com/Writing-to-and-from-a-binary-file-tp3688010p3694622.html Sent from the Revolution - User mailing list archive at Nabble.com. From keith at gulfbreezeortholab.com Mon Jul 25 21:44:44 2011 From: keith at gulfbreezeortholab.com (Keith (Gulf Breeze Ortho Lab)) Date: Mon, 25 Jul 2011 20:44:44 -0500 Subject: What is the fastest database? In-Reply-To: <1311639152642-3694491.post@n4.nabble.com> References: <1311639152642-3694491.post@n4.nabble.com> Message-ID: Hi Al, The data is as compressed as possible right now (I believe, anyway). And yes, I hope to eventually port the system to mobile. Thanks, - Boo ----- Original Message ----- From: "Alejandro Tejada" To: Sent: Monday, July 25, 2011 7:12 PM Subject: Re: What is the fastest database? > Hi Keith, > > > Keith (Gulf Breeze Ortho Lab) wrote: >> >> I have approximately 25 MB of data (plain text) that consists of >> approximately 140,000 different entries (one per line--pipe delimited). I >> am working on an application to quickly search through the data not >> unlike >> the LiveCode Dictionary application. However, it will be necessary for my >> users to be able to edit any entry at will. >> > > I understand you perfectly, when you say that you do not want to > load the complete database in memory. > > Could you compress the data??? This could reduce significantly > it's memory footprint. > > Are you going to port this application to mobile devices > like Android and iPad???? This is vital to know beforehand. > > Have a nice day! > > Al > > -- > View this message in context: > http://runtime-revolution.278305.n4.nabble.com/What-is-the-fastest-database-tp3694246p3694491.html > Sent from the Revolution - User mailing list archive at Nabble.com. > > _______________________________________________ > use-livecode mailing list > use-livecode at lists.runrev.com > Please visit this url to subscribe, unsubscribe and manage your > subscription preferences: > http://lists.runrev.com/mailman/listinfo/use-livecode > From andre at andregarzia.com Mon Jul 25 21:49:21 2011 From: andre at andregarzia.com (Andre Garzia) Date: Mon, 25 Jul 2011 22:49:21 -0300 Subject: What is the fastest database? In-Reply-To: <5245936C-4DA4-4998-8E9D-DEB9F5A5F325@twft.com> References: <5245936C-4DA4-4998-8E9D-DEB9F5A5F325@twft.com> Message-ID: On Mon, Jul 25, 2011 at 7:49 PM, Bob Sneidar wrote: > If you plan on moving this to an external server, then you should not allow > direct access to your SQL server (although I do so myself in development > using On-Rev). You will probably want to access it through a web based > script of some sort to firewall your SQL data. This will prevent things like > SQL injection attacks and such. > > If this is your eventual goal, I would advise developing your application > using this methodology right from the start. > > my 2? > what he said! > > Bob > > > On Jul 25, 2011, at 2:51 PM, Keith (Gulf Breeze Ortho Lab) wrote: > > > Hello, > > > > I have approximately 25 MB of data (plain text) that consists of > approximately 140,000 different entries (one per line--pipe delimited). I am > working on an application to quickly search through the data not unlike the > LiveCode Dictionary application. However, it will be necessary for my users > to be able to edit any entry at will. > > > > For a desktop application, what database would you recommend for the job? > I can use a flat file database, but the problem is that all 25 MB of data > must be loaded into memory... What about SQLite? Valencia? Speed is of the > essence. > > > > FYI: In the past I used an Access database, but it was too sluggish for > my purposes. > > > > Any recommendations/feedback is most welcome. Thanks. > > > > LiveCode is awesome! > > > > Sincerely, > > > > - Boo > > _______________________________________________ > > use-livecode mailing list > > use-livecode at lists.runrev.com > > Please visit this url to subscribe, unsubscribe and manage your > subscription preferences: > > http://lists.runrev.com/mailman/listinfo/use-livecode > > > _______________________________________________ > use-livecode mailing list > use-livecode at lists.runrev.com > Please visit this url to subscribe, unsubscribe and manage your > subscription preferences: > http://lists.runrev.com/mailman/listinfo/use-livecode > -- http://www.andregarzia.com All We Do Is Code. From lists.pete at haworths.org Mon Jul 25 22:14:07 2011 From: lists.pete at haworths.org (Pete Haworth) Date: Mon, 25 Jul 2011 19:14:07 -0700 Subject: Importing an image In-Reply-To: References: Message-ID: Excellent, thanks! Pete On Mon, Jul 25, 2011 at 6:08 PM, Terry Judd wrote: > > On 26/07/2011 10:53 AM, "Pete Haworth" wrote: > > > I have a png file that I want to get into a stack using the import > command. > > I issue the command from the message box: > > > > "import eps from file "" > > Pete - I think you should be using 'paint' rather than 'eps' > > import paint from file > > Using 'eps' crashes Livecode for me as well with a PNG file - 'paint' works > fine however. > > Terry... > > > > as soon as I hit return, Livecode quits. This is a png file that was > > created by anLC export snapshot command and the resized in Photoshop. > > > > Any know problems with importing PNG files? > > > > > > Pete > > _______________________________________________ > > use-livecode mailing list > > use-livecode at lists.runrev.com > > Please visit this url to subscribe, unsubscribe and manage your > subscription > > preferences: > > http://lists.runrev.com/mailman/listinfo/use-livecode > > > > -- > Dr Terry Judd | Senior Lecturer in Medical Education > Medical Education Unit > Melbourne Medical School > The University of Melbourne > > > > > _______________________________________________ > use-livecode mailing list > use-livecode at lists.runrev.com > Please visit this url to subscribe, unsubscribe and manage your > subscription preferences: > http://lists.runrev.com/mailman/listinfo/use-livecode > From kray at sonsothunder.com Tue Jul 26 00:24:03 2011 From: kray at sonsothunder.com (Ken Ray) Date: Mon, 25 Jul 2011 23:24:03 -0500 Subject: Importing an image In-Reply-To: References: Message-ID: <52297415-BF30-4B1A-B625-58C2ECB66801@sonsothunder.com> > Using 'eps' crashes Livecode for me as well with a PNG file - 'paint' works > fine however. EPS is a *very old* import method, dating back to when MetaCard (the precursor to Revolution which was the precursor to LIveCode) was only on Unix; it was for systems that used Display Postscript (like NeXT boxes)... so I'm not too surprised that it crashes LiveCode when fed the wrong filetype... ;-) Ken Ray Sons of Thunder Software, Inc. Email: kray at sonsothunder.com Web Site: http://www.sonsothunder.com/ From lists.pete at haworths.org Tue Jul 26 01:32:09 2011 From: lists.pete at haworths.org (Pete Haworth) Date: Mon, 25 Jul 2011 22:32:09 -0700 Subject: Importing an image In-Reply-To: <52297415-BF30-4B1A-B625-58C2ECB66801@sonsothunder.com> References: <52297415-BF30-4B1A-B625-58C2ECB66801@sonsothunder.com> Message-ID: I totally missed the "paint" keyword in the dictionary but all is well now. The reason I was doing this is because took a PNG snapshot of a group control into an image control, then tried to resize the image control, but I found that the image lost definition very quickly. My solution was to export to a file, resize it in Photoshop, then import it. Maybe there's a way to do the resizing in LC without losing definition? Pete On Mon, Jul 25, 2011 at 9:24 PM, Ken Ray wrote: > > Using 'eps' crashes Livecode for me as well with a PNG file - 'paint' > works > > fine however. > > EPS is a *very old* import method, dating back to when MetaCard (the > precursor to Revolution which was the precursor to LIveCode) was only on > Unix; it was for systems that used Display Postscript (like NeXT boxes)... > so I'm not too surprised that it crashes LiveCode when fed the wrong > filetype... > > ;-) > > Ken Ray > Sons of Thunder Software, Inc. > Email: kray at sonsothunder.com > Web Site: http://www.sonsothunder.com/ > > _______________________________________________ > use-livecode mailing list > use-livecode at lists.runrev.com > Please visit this url to subscribe, unsubscribe and manage your > subscription preferences: > http://lists.runrev.com/mailman/listinfo/use-livecode > From scott at tactilemedia.com Tue Jul 26 01:52:11 2011 From: scott at tactilemedia.com (Scott Rossi) Date: Mon, 25 Jul 2011 22:52:11 -0700 Subject: Importing an image In-Reply-To: References: <52297415-BF30-4B1A-B625-58C2ECB66801@sonsothunder.com> Message-ID: <3810E19D-68C4-44FC-AE3A-2103675C2E39@tactilemedia.com> One thing that might help is to set the resizeQuality property of the image to "best" before resizing (default is "normal"). Regards, Scott Rossi Creative Director Tactile Media, UX Design On Jul 25, 2011, at 10:32 PM, Pete Haworth wrote: > I totally missed the "paint" keyword in the dictionary but all is well now. > > The reason I was doing this is because took a PNG snapshot of a group > control into an image control, then tried to resize the image control, but I > found that the image lost definition very quickly. My solution was to > export to a file, resize it in Photoshop, then import it. Maybe there's a > way to do the resizing in LC without losing definition? > > Pete > > > > On Mon, Jul 25, 2011 at 9:24 PM, Ken Ray wrote: > >>> Using 'eps' crashes Livecode for me as well with a PNG file - 'paint' >> works >>> fine however. >> >> EPS is a *very old* import method, dating back to when MetaCard (the >> precursor to Revolution which was the precursor to LIveCode) was only on >> Unix; it was for systems that used Display Postscript (like NeXT boxes)... >> so I'm not too surprised that it crashes LiveCode when fed the wrong >> filetype... >> >> ;-) >> >> Ken Ray >> Sons of Thunder Software, Inc. >> Email: kray at sonsothunder.com >> Web Site: http://www.sonsothunder.com/ >> >> _______________________________________________ >> use-livecode mailing list >> use-livecode at lists.runrev.com >> Please visit this url to subscribe, unsubscribe and manage your >> subscription preferences: >> http://lists.runrev.com/mailman/listinfo/use-livecode >> > _______________________________________________ > use-livecode mailing list > use-livecode at lists.runrev.com > Please visit this url to subscribe, unsubscribe and manage your subscription preferences: > http://lists.runrev.com/mailman/listinfo/use-livecode > From mazzapaoloitaly at gmail.com Tue Jul 26 02:40:41 2011 From: mazzapaoloitaly at gmail.com (paolo mazza) Date: Tue, 26 Jul 2011 08:40:41 +0200 Subject: previous page button of browser opening on-rev server page In-Reply-To: <88F2DE00-13A2-4AD0-B9CE-961564DAB131@economy-x-talk.com> References: <2CA55DDD-D9D0-4C0F-8E57-BAF82E464649@economy-x-talk.com> <88F2DE00-13A2-4AD0-B9CE-961564DAB131@economy-x-talk.com> Message-ID: Thank you all. Trying to get rid of the browser cache setting the proper headings (as Shao Sean suggested) , I came up with these 2 solutions: PHP: LIVECODE ?... much easyer ?;-) HOWEVER, the first solution (PHP) sets the heathers properly but still when I move back and forth with the buttons of the browser I get the old time (cache). The second one (LiveCode) does NOT set ?the headers properly (is it a bug?) and the page remains in the browser cache. Any idea? All the best Paolo Mazza From janschenkel at yahoo.com Tue Jul 26 05:57:46 2011 From: janschenkel at yahoo.com (Jan Schenkel) Date: Tue, 26 Jul 2011 02:57:46 -0700 (PDT) Subject: previous page button of browser opening on-rev server page In-Reply-To: References: <2CA55DDD-D9D0-4C0F-8E57-BAF82E464649@economy-x-talk.com> <88F2DE00-13A2-4AD0-B9CE-961564DAB131@economy-x-talk.com> Message-ID: <1311674266.51201.YahooMailNeo@web65408.mail.ac4.yahoo.com> I don't think setting the 'httpHeaders' will do what you want - that's meant for getting a URL or posting to a it from within your LC script. Try this instead (untested, but should do the trick, I think): HTH, Jan Schenkel. ===== Quartam Reports & PDF Library for LiveCode www.quartam.com ===== "As we grow older, we grow both wiser and more foolish at the same time." (La Rochefoucauld) ----- Original Message ----- From: paolo mazza To: How to use LiveCode Subject: Re: previous page button of browser opening on-rev server page Thank you all. Trying to get rid of the browser cache setting the proper headings (as Shao Sean suggested) , I came up with these 2 solutions: PHP: LIVECODE ?... much easyer ?;-) HOWEVER, the first solution (PHP) sets the heathers properly but still when I move back and forth with the buttons of the browser I get the old time? (cache). The second one (LiveCode) does NOT set ?the headers properly (is it a bug?) and the page remains in the browser cache. Any idea? All the best Paolo Mazza _______________________________________________ use-livecode mailing list use-livecode at lists.runrev.com Please visit this url to subscribe, unsubscribe and manage your subscription preferences: http://lists.runrev.com/mailman/listinfo/use-livecode From matthias_livecode at me.com Tue Jul 26 06:23:44 2011 From: matthias_livecode at me.com (Matthias Rebbe) Date: Tue, 26 Jul 2011 12:23:44 +0200 Subject: How to add something to a group Message-ID: Hi, maybe a dumb question, but how do i add for example a button to an already existing group? Matthias From klaus at major.on-rev.com Tue Jul 26 06:28:30 2011 From: klaus at major.on-rev.com (Klaus on-rev) Date: Tue, 26 Jul 2011 12:28:30 +0200 Subject: How to add something to a group In-Reply-To: References: Message-ID: Hi Matthias, Am 26.07.2011 um 12:23 schrieb Matthias Rebbe: > Hi, > > maybe a dumb question, but how do i add for example a button to an already existing group? manually: "edit background", add objects via script: copy btn x to grp Y or create button in group Y > Matthias Best Klaus -- Klaus Major http://www.major-k.de klaus at major.on-rev.com From dave.cragg at lacscentre.co.uk Tue Jul 26 07:03:01 2011 From: dave.cragg at lacscentre.co.uk (Dave Cragg) Date: Tue, 26 Jul 2011 12:03:01 +0100 Subject: previous page button of browser opening on-rev server page In-Reply-To: References: <2CA55DDD-D9D0-4C0F-8E57-BAF82E464649@economy-x-talk.com> <88F2DE00-13A2-4AD0-B9CE-961564DAB131@economy-x-talk.com> Message-ID: <6132513B-F65D-48F3-8428-FCE4424CCE55@lacscentre.co.uk> On 26 Jul 2011, at 07:40, paolo mazza wrote: > Thank you all. Trying to get rid of the browser cache setting the > proper headings (as Shao Sean suggested) , I came up with these 2 > solutions: > > PHP: > > header("Cache-Control: no-cache, must-revalidate"); > header("Expires: Fri, 30 oct 1998 14:18:41 GMT"); > ?> > > $dateTime = date("Y:m:d-H:i:s"); > function getDateTime() { > global $dateTime; > return $dateTime; > } > ?> > > > > > > > > > > LIVECODE ... much easyer ;-) Not really a fair comparison. The following php script will do the same as your Livecode script: Not so different. :-) Dave From mazzapaolo at libero.it Tue Jul 26 09:54:57 2011 From: mazzapaolo at libero.it (paolo mazza) Date: Tue, 26 Jul 2011 15:54:57 +0200 Subject: previous page button of browser opening on-rev server page In-Reply-To: <6132513B-F65D-48F3-8428-FCE4424CCE55@lacscentre.co.uk> References: <2CA55DDD-D9D0-4C0F-8E57-BAF82E464649@economy-x-talk.com> <88F2DE00-13A2-4AD0-B9CE-961564DAB131@economy-x-talk.com> <6132513B-F65D-48F3-8428-FCE4424CCE55@lacscentre.co.uk> Message-ID: Thank you all. So, there is a property ( header ) only for the on-rev server environment? In fact I can not use this property in the IDE (I get a compilation error) and it does not exists in the LC documentation (version 4.6.0). Are there other undocumented properties suitable only to the server environment? Actually the script suggested by Jan works, and I get a page with 2 new headers. However, for some reasons, the browser do not reload the page. Dave, you are right, I was unfair with PHP... still, at least "put the long date" is pretty elegant. All the best. Paolo Mazza From mwieder at ahsoftware.net Tue Jul 26 11:18:21 2011 From: mwieder at ahsoftware.net (Mark Wieder) Date: Tue, 26 Jul 2011 08:18:21 -0700 Subject: Importing an image In-Reply-To: References: <52297415-BF30-4B1A-B625-58C2ECB66801@sonsothunder.com> Message-ID: <62478513125.20110726081821@ahsoftware.net> Pete- Monday, July 25, 2011, 10:32:09 PM, you wrote: > I totally missed the "paint" keyword in the dictionary but all is well now. > The reason I was doing this is because took a PNG snapshot of a group > control into an image control, then tried to resize the image control, but I > found that the image lost definition very quickly. My solution was to > export to a file, resize it in Photoshop, then import it. Maybe there's a > way to do the resizing in LC without losing definition? Here's how I do group control snapshots in PowerTools. Obviously this could be simplified a lot if you're working in the same stack and with a fixed height and width: private function CreateThumbnail pWidgetGroup, pStack, pThumbRect import snapshot from control id pWidgetGroup of stack pStack set the resizequality of the last image to "best" set the width of the last image to item 1 of pThumbRect set the height of the last image to item 2 of pThumbRect return the id of the last image end CreateThumbnail -- -Mark Wieder mwieder at ahsoftware.net From lfredricks at proactive-intl.com Tue Jul 26 11:52:58 2011 From: lfredricks at proactive-intl.com (Lynn Fredricks) Date: Tue, 26 Jul 2011 08:52:58 -0700 Subject: What is the fastest database? In-Reply-To: References: Message-ID: <12008DBA3B0645EDBD626347C66550A4@GATEWAY> > For a desktop application, what database would you recommend > for the job? I can use a flat file database, but the problem > is that all 25 MB of data must be loaded into memory... What > about SQLite? Valencia? Speed is of the essence. Ill try to avoid a tag team with Ruslan on this ;-) In terms of Valentina, your database is positively tiny compared with many of our users, though the number of records you have are reaching the point where you'll begin to see some real differences between database engines. Ruslan has already mentioned the super speed. A few other points which I think you'll find important in evaluating Valentina: - Load it into Memory. Yes, you can do this - this can be done with the local version of Valentina (Valentina ADK) or on the server implementations. - Valentina Plays Well With LiveCode. One differentiator of Valentina is that we have the means to support many development tools ourselves. Paradigma is a long time partner of LiveCode. We compete against many of the big boy databases (esp on the server side), but show of hands the last time IBM, Microsoft or Oracle went out of their way to do anything to adapt to the needs of the LiveCode community? - 12 Months of Updates. If you like all that's a part of Valentina DB now, it will get even better down the road. You get 12 months of updates, which includes new features, fixes and performance enhancements. - We Support a Lot of Platforms. We've been at this now for quite some time ( well into the "plus" side after the first decade), and we've been able to support a broad range of development tools and operating systems (together forming "platforms"). For this reason, you have a lot of flexibility in planning your implementations - for example, how you might handle building a "heavy" client and what might be on the server (PHP, Ruby on Rails, etc). - We Got Tools That Do the Job. Valentina Studio Pro is a solid tool for administering and developing your databases - and it supports SQLite, too, as well as provide a visual builder for reports if you also get Valentina Reports. There are also free admin tools for Valentina Server for iPad and iPhone. - We are Going to Be Here Tomorrow. The last thing I want to happen when I invest in infrastructure tools is the vendor to either A) die or B) go VC-Postal, get huge investments, then either get bought out by an unloving god or do a bizzare business model change that messes with my business. We haven't gone VC-Postal, and we are, for the most part, healthier today than we were yesterday (well, a little hair loss and expanding beltline too). I hope this is useful information for you. Best regards, Lynn Fredricks President Paradigma Software http://www.paradigmasoft.com Valentina SQL Server: The Ultra-fast, Royalty Free Database Server From bobs at twft.com Tue Jul 26 12:25:28 2011 From: bobs at twft.com (Bob Sneidar) Date: Tue, 26 Jul 2011 09:25:28 -0700 Subject: How to add something to a group In-Reply-To: References: Message-ID: Graphically: 1. Click group to be edited 2. Click Edit Group on your button bar, or select it in the Object menu 3. Add your button (or paste it if you cut it previously) 4. (very important) Click the Edit Group button (or menu) again or everything else you do from here on out will be done to the group! (ask me how I know) Bob On Jul 26, 2011, at 3:23 AM, Matthias Rebbe wrote: > Hi, > > maybe a dumb question, but how do i add for example a button to an already existing group? > > Matthias > > > _______________________________________________ > use-livecode mailing list > use-livecode at lists.runrev.com > Please visit this url to subscribe, unsubscribe and manage your subscription preferences: > http://lists.runrev.com/mailman/listinfo/use-livecode From bobs at twft.com Tue Jul 26 12:31:18 2011 From: bobs at twft.com (Bob Sneidar) Date: Tue, 26 Jul 2011 09:31:18 -0700 Subject: Importing an image In-Reply-To: References: <52297415-BF30-4B1A-B625-58C2ECB66801@sonsothunder.com> Message-ID: <68097985-8876-4F71-B927-2F787A8C9D74@twft.com> I will say this about resizing images in Livecode: Don't. The Preview app that comes with Apple OS X does a much better job of resizing an image than Livecode does "on the fly" by which I mean setting the size of the image using the height and width properties. This is not a criticism, but an observation. Also, resizing in LC doesn't stick unless you lock the size and position of both the image and the object using the image (in my experience at least). Quitting and relaunching Livecode may reveal that the images have reverted to their original sizes. So now I create images the size I need them to be in the application. Scott Rossi will tell you the best way of scaling the images once inside LC. Bob On Jul 25, 2011, at 10:32 PM, Pete Haworth wrote: > I totally missed the "paint" keyword in the dictionary but all is well now. > > The reason I was doing this is because took a PNG snapshot of a group > control into an image control, then tried to resize the image control, but I > found that the image lost definition very quickly. My solution was to > export to a file, resize it in Photoshop, then import it. Maybe there's a > way to do the resizing in LC without losing definition? > > Pete > > > > On Mon, Jul 25, 2011 at 9:24 PM, Ken Ray wrote: > >>> Using 'eps' crashes Livecode for me as well with a PNG file - 'paint' >> works >>> fine however. >> >> EPS is a *very old* import method, dating back to when MetaCard (the >> precursor to Revolution which was the precursor to LIveCode) was only on >> Unix; it was for systems that used Display Postscript (like NeXT boxes)... >> so I'm not too surprised that it crashes LiveCode when fed the wrong >> filetype... >> >> ;-) >> >> Ken Ray >> Sons of Thunder Software, Inc. >> Email: kray at sonsothunder.com >> Web Site: http://www.sonsothunder.com/ >> >> _______________________________________________ >> use-livecode mailing list >> use-livecode at lists.runrev.com >> Please visit this url to subscribe, unsubscribe and manage your >> subscription preferences: >> http://lists.runrev.com/mailman/listinfo/use-livecode >> > _______________________________________________ > use-livecode mailing list > use-livecode at lists.runrev.com > Please visit this url to subscribe, unsubscribe and manage your subscription preferences: > http://lists.runrev.com/mailman/listinfo/use-livecode From bobs at twft.com Tue Jul 26 12:32:41 2011 From: bobs at twft.com (Bob Sneidar) Date: Tue, 26 Jul 2011 09:32:41 -0700 Subject: Importing an image In-Reply-To: <3810E19D-68C4-44FC-AE3A-2103675C2E39@tactilemedia.com> References: <52297415-BF30-4B1A-B625-58C2ECB66801@sonsothunder.com> <3810E19D-68C4-44FC-AE3A-2103675C2E39@tactilemedia.com> Message-ID: <6BA9B2CC-A4DD-42D8-81B2-105E19750FEB@twft.com> Hmmm... that may modify what I posted. I will have to compare Livecode resizing with pre-resizing before import again. Bob On Jul 25, 2011, at 10:52 PM, Scott Rossi wrote: > One thing that might help is to set the resizeQuality property of the image to "best" before resizing (default is "normal"). > > Regards, > > Scott Rossi > Creative Director > Tactile Media, UX Design > > > On Jul 25, 2011, at 10:32 PM, Pete Haworth wrote: > >> I totally missed the "paint" keyword in the dictionary but all is well now. >> >> The reason I was doing this is because took a PNG snapshot of a group >> control into an image control, then tried to resize the image control, but I >> found that the image lost definition very quickly. My solution was to >> export to a file, resize it in Photoshop, then import it. Maybe there's a >> way to do the resizing in LC without losing definition? >> >> Pete >> >> >> >> On Mon, Jul 25, 2011 at 9:24 PM, Ken Ray wrote: >> >>>> Using 'eps' crashes Livecode for me as well with a PNG file - 'paint' >>> works >>>> fine however. >>> >>> EPS is a *very old* import method, dating back to when MetaCard (the >>> precursor to Revolution which was the precursor to LIveCode) was only on >>> Unix; it was for systems that used Display Postscript (like NeXT boxes)... >>> so I'm not too surprised that it crashes LiveCode when fed the wrong >>> filetype... >>> >>> ;-) >>> >>> Ken Ray >>> Sons of Thunder Software, Inc. >>> Email: kray at sonsothunder.com >>> Web Site: http://www.sonsothunder.com/ >>> >>> _______________________________________________ >>> use-livecode mailing list >>> use-livecode at lists.runrev.com >>> Please visit this url to subscribe, unsubscribe and manage your >>> subscription preferences: >>> http://lists.runrev.com/mailman/listinfo/use-livecode >>> >> _______________________________________________ >> use-livecode mailing list >> use-livecode at lists.runrev.com >> Please visit this url to subscribe, unsubscribe and manage your subscription preferences: >> http://lists.runrev.com/mailman/listinfo/use-livecode >> > > _______________________________________________ > use-livecode mailing list > use-livecode at lists.runrev.com > Please visit this url to subscribe, unsubscribe and manage your subscription preferences: > http://lists.runrev.com/mailman/listinfo/use-livecode From lists.pete at haworths.org Tue Jul 26 12:41:38 2011 From: lists.pete at haworths.org (Pete Haworth) Date: Tue, 26 Jul 2011 09:41:38 -0700 Subject: Importing an image In-Reply-To: <62478513125.20110726081821@ahsoftware.net> References: <52297415-BF30-4B1A-B625-58C2ECB66801@sonsothunder.com> <62478513125.20110726081821@ahsoftware.net> Message-ID: Thanks everyone - so many properties, so little time..... Pete On Tue, Jul 26, 2011 at 8:18 AM, Mark Wieder wrote: > Pete- > > Monday, July 25, 2011, 10:32:09 PM, you wrote: > > > I totally missed the "paint" keyword in the dictionary but all is well > now. > > > The reason I was doing this is because took a PNG snapshot of a group > > control into an image control, then tried to resize the image control, > but I > > found that the image lost definition very quickly. My solution was to > > export to a file, resize it in Photoshop, then import it. Maybe there's > a > > way to do the resizing in LC without losing definition? > > Here's how I do group control snapshots in PowerTools. Obviously this > could be simplified a lot if you're working in the same stack and with > a fixed height and width: > > private function CreateThumbnail pWidgetGroup, pStack, pThumbRect > import snapshot from control id pWidgetGroup of stack pStack > set the resizequality of the last image to "best" > set the width of the last image to item 1 of pThumbRect > set the height of the last image to item 2 of pThumbRect > return the id of the last image > end CreateThumbnail > > -- > -Mark Wieder > mwieder at ahsoftware.net > > > _______________________________________________ > use-livecode mailing list > use-livecode at lists.runrev.com > Please visit this url to subscribe, unsubscribe and manage your > subscription preferences: > http://lists.runrev.com/mailman/listinfo/use-livecode > From lists.pete at haworths.org Tue Jul 26 13:03:25 2011 From: lists.pete at haworths.org (Pete Haworth) Date: Tue, 26 Jul 2011 10:03:25 -0700 Subject: How to add something to a group In-Reply-To: References: Message-ID: Another way is: - Place the button where you want it. - Select the group - Click Ungroup - Shift click the new button so it's selected along with all the original members of the group - Click Group LC remembers all the properties of the group and reinstates them (as long as you don't close the card before grouping). I believe there's a plugin available somewhere that provides ways of adding controls to a group by re-layering but I can't remember what it's called. Pete On Tue, Jul 26, 2011 at 9:25 AM, Bob Sneidar wrote: > Graphically: > > 1. Click group to be edited > 2. Click Edit Group on your button bar, or select it in the Object menu > 3. Add your button (or paste it if you cut it previously) > 4. (very important) Click the Edit Group button (or menu) again or > everything else you do from here on out will be done to the group! > > (ask me how I know) > > Bob > > > On Jul 26, 2011, at 3:23 AM, Matthias Rebbe wrote: > > > Hi, > > > > maybe a dumb question, but how do i add for example a button to an > already existing group? > > > > Matthias > > > > > > _______________________________________________ > > use-livecode mailing list > > use-livecode at lists.runrev.com > > Please visit this url to subscribe, unsubscribe and manage your > subscription preferences: > > http://lists.runrev.com/mailman/listinfo/use-livecode > > > _______________________________________________ > use-livecode mailing list > use-livecode at lists.runrev.com > Please visit this url to subscribe, unsubscribe and manage your > subscription preferences: > http://lists.runrev.com/mailman/listinfo/use-livecode > From capellan2000 at gmail.com Tue Jul 26 14:30:10 2011 From: capellan2000 at gmail.com (Alejandro Tejada) Date: Tue, 26 Jul 2011 14:30:10 -0400 Subject: Writing to and from a binary file? Message-ID: Hi All, Many Thanks to Bernd for sending an example stack! I will start experimenting using this code as starting point. :-D Have a great good day! Al On Mon, Jul 25, 2011 Alejandro Tejada wrote: > Great work! Now, just out of curiosity > How could I modify your code, so it cuts > an image based in an specific characteristic? > For example, suppose that I want to cut an image > of the alphabet in their single letters: > ABCDEFGHIJKLMNOPQRSTUVWXYZ > The image is horizontal and I want to split it > by script in their individual letters. > Thanks in advance! From gcanyon+rev at gmail.com Tue Jul 26 14:37:24 2011 From: gcanyon+rev at gmail.com (Geoff Canyon Rev) Date: Tue, 26 Jul 2011 13:37:24 -0500 Subject: How to add something to a group In-Reply-To: References: Message-ID: There are several; Navigator (mine) is one, but it is ancient, unmaintained and somewhat buggy at this point. I believe I already said this, but anyone who still wants to use it can do so for free at this point. An even more ancient version of navigator comes with your standard installation; you can get an updated (but still ancient) version here: http://inspiredlogic.com/navigator/Navigator.html gc On Tue, Jul 26, 2011 at 12:03 PM, Pete Haworth wrote: > Another way is: > > - Place the button where you want it. > - Select the group > - Click Ungroup > - Shift click the new button so it's selected along with all the original > members of the group > - Click Group > > LC remembers all the properties of the group and reinstates them (as long > as > you don't close the card before grouping). > > I believe there's a plugin available somewhere that provides ways of adding > controls to a group by re-layering but I can't remember what it's called. > > Pete > > > > On Tue, Jul 26, 2011 at 9:25 AM, Bob Sneidar wrote: > > > Graphically: > > > > 1. Click group to be edited > > 2. Click Edit Group on your button bar, or select it in the Object menu > > 3. Add your button (or paste it if you cut it previously) > > 4. (very important) Click the Edit Group button (or menu) again or > > everything else you do from here on out will be done to the group! > > > > (ask me how I know) > > > > Bob > > > > > > On Jul 26, 2011, at 3:23 AM, Matthias Rebbe wrote: > > > > > Hi, > > > > > > maybe a dumb question, but how do i add for example a button to an > > already existing group? > > > > > > Matthias > > > > > > > > > _______________________________________________ > > > use-livecode mailing list > > > use-livecode at lists.runrev.com > > > Please visit this url to subscribe, unsubscribe and manage your > > subscription preferences: > > > http://lists.runrev.com/mailman/listinfo/use-livecode > > > > > > _______________________________________________ > > use-livecode mailing list > > use-livecode at lists.runrev.com > > Please visit this url to subscribe, unsubscribe and manage your > > subscription preferences: > > http://lists.runrev.com/mailman/listinfo/use-livecode > > > _______________________________________________ > use-livecode mailing list > use-livecode at lists.runrev.com > Please visit this url to subscribe, unsubscribe and manage your > subscription preferences: > http://lists.runrev.com/mailman/listinfo/use-livecode > From capellan2000 at gmail.com Tue Jul 26 15:03:07 2011 From: capellan2000 at gmail.com (Alejandro Tejada) Date: Tue, 26 Jul 2011 12:03:07 -0700 (PDT) Subject: Where does survive the inventive user? Message-ID: <1311706987455-3696711.post@n4.nabble.com> Hi all, Today, I read again this article by Dan Shafer: http://www.danshaferblog.com/inventive-users-need-help-on-the-ithings Many obvious questions arise from this article: Does latest versions of Livecode fill this niche? Is programming for mobile so easy (using Livecode) that anyone that wants to, could do it? Did anyone here knows someone who actually started learning programming after buying one of the mobile platforms? Today, I woke with an strange idea: Mobile computing will displace desktop computing for most everyday computing tasks in a really short time (5 to 10 years). Tell me if this idea has a real basis or is just an echo of the hype that surrounds the latest products. Thanks in advance! Al -- View this message in context: http://runtime-revolution.278305.n4.nabble.com/Where-does-survive-the-inventive-user-tp3696711p3696711.html Sent from the Revolution - User mailing list archive at Nabble.com. From andre at andregarzia.com Tue Jul 26 15:08:54 2011 From: andre at andregarzia.com (Andre Garzia) Date: Tue, 26 Jul 2011 16:08:54 -0300 Subject: Where does survive the inventive user? In-Reply-To: <1311706987455-3696711.post@n4.nabble.com> References: <1311706987455-3696711.post@n4.nabble.com> Message-ID: mobile computing is consumer computing. Developers and inventive users will keep on platforms that allow them to develop stuff. It means that slowly, those users will move towards freedom so even though mobile computing will be ubiquitous, you will find the developers and inventive users using something else where they can actually develop stuff unrestricted. They will probably be on linux... On Tue, Jul 26, 2011 at 4:03 PM, Alejandro Tejada wrote: > Hi all, > > Today, I read again this article by Dan Shafer: > http://www.danshaferblog.com/inventive-users-need-help-on-the-ithings > > Many obvious questions arise from this article: > Does latest versions of Livecode fill this niche? > > Is programming for mobile so easy (using Livecode) > that anyone that wants to, could do it? > > Did anyone here knows someone who actually started > learning programming after buying one of the > mobile platforms? > > Today, I woke with an strange idea: Mobile computing > will displace desktop computing for most everyday > computing tasks in a really short time (5 to 10 years). > > Tell me if this idea has a real basis or is just an echo of > the hype that surrounds the latest products. > > Thanks in advance! > > Al > > -- > View this message in context: > http://runtime-revolution.278305.n4.nabble.com/Where-does-survive-the-inventive-user-tp3696711p3696711.html > Sent from the Revolution - User mailing list archive at Nabble.com. > > _______________________________________________ > use-livecode mailing list > use-livecode at lists.runrev.com > Please visit this url to subscribe, unsubscribe and manage your > subscription preferences: > http://lists.runrev.com/mailman/listinfo/use-livecode > -- http://www.andregarzia.com All We Do Is Code. From lists.pete at haworths.org Tue Jul 26 15:18:27 2011 From: lists.pete at haworths.org (Pete Haworth) Date: Tue, 26 Jul 2011 12:18:27 -0700 Subject: How to add something to a group In-Reply-To: References: Message-ID: Thanks Geoff. Yes I remember seeing your note about that. Sorry its' no longer maintained, I've always found the maintenance of groups to be awkward in LC, especially when you get into nested group situations. Pete On Tue, Jul 26, 2011 at 11:37 AM, Geoff Canyon Rev wrote: > There are several; Navigator (mine) is one, but it is ancient, unmaintained > and somewhat buggy at this point. I believe I already said this, but anyone > who still wants to use it can do so for free at this point. An even more > ancient version of navigator comes with your standard installation; you can > get an updated (but still ancient) version here: > http://inspiredlogic.com/navigator/Navigator.html > > gc > > On Tue, Jul 26, 2011 at 12:03 PM, Pete Haworth >wrote: > > > Another way is: > > > > - Place the button where you want it. > > - Select the group > > - Click Ungroup > > - Shift click the new button so it's selected along with all the original > > members of the group > > - Click Group > > > > LC remembers all the properties of the group and reinstates them (as long > > as > > you don't close the card before grouping). > > > > I believe there's a plugin available somewhere that provides ways of > adding > > controls to a group by re-layering but I can't remember what it's called. > > > > Pete > > > > > > > > On Tue, Jul 26, 2011 at 9:25 AM, Bob Sneidar wrote: > > > > > Graphically: > > > > > > 1. Click group to be edited > > > 2. Click Edit Group on your button bar, or select it in the Object menu > > > 3. Add your button (or paste it if you cut it previously) > > > 4. (very important) Click the Edit Group button (or menu) again or > > > everything else you do from here on out will be done to the group! > > > > > > (ask me how I know) > > > > > > Bob > > > > > > > > > On Jul 26, 2011, at 3:23 AM, Matthias Rebbe wrote: > > > > > > > Hi, > > > > > > > > maybe a dumb question, but how do i add for example a button to an > > > already existing group? > > > > > > > > Matthias > > > > > > > > > > > > _______________________________________________ > > > > use-livecode mailing list > > > > use-livecode at lists.runrev.com > > > > Please visit this url to subscribe, unsubscribe and manage your > > > subscription preferences: > > > > http://lists.runrev.com/mailman/listinfo/use-livecode > > > > > > > > > _______________________________________________ > > > use-livecode mailing list > > > use-livecode at lists.runrev.com > > > Please visit this url to subscribe, unsubscribe and manage your > > > subscription preferences: > > > http://lists.runrev.com/mailman/listinfo/use-livecode > > > > > _______________________________________________ > > use-livecode mailing list > > use-livecode at lists.runrev.com > > Please visit this url to subscribe, unsubscribe and manage your > > subscription preferences: > > http://lists.runrev.com/mailman/listinfo/use-livecode > > > _______________________________________________ > use-livecode mailing list > use-livecode at lists.runrev.com > Please visit this url to subscribe, unsubscribe and manage your > subscription preferences: > http://lists.runrev.com/mailman/listinfo/use-livecode > From matthias_livecode at me.com Tue Jul 26 15:33:54 2011 From: matthias_livecode at me.com (Matthias Rebbe) Date: Tue, 26 Jul 2011 21:33:54 +0200 Subject: How to add something to a group In-Reply-To: References: Message-ID: <3B9F3086-E8B9-4B32-841F-B4A9168F90AE@me.com> Hi, thanks for all your suggestions. I never used "Edit Group" so far. I even was not sure for what it was. Now i know.... Regards, Matthias Am 26.07.2011 um 21:18 schrieb Pete Haworth: > Thanks Geoff. Yes I remember seeing your note about that. Sorry its' no > longer maintained, I've always found the maintenance of groups to be awkward > in LC, especially when you get into nested group situations. > > Pete > > > > On Tue, Jul 26, 2011 at 11:37 AM, Geoff Canyon Rev wrote: > >> There are several; Navigator (mine) is one, but it is ancient, unmaintained >> and somewhat buggy at this point. I believe I already said this, but anyone >> who still wants to use it can do so for free at this point. An even more >> ancient version of navigator comes with your standard installation; you can >> get an updated (but still ancient) version here: >> http://inspiredlogic.com/navigator/Navigator.html >> >> gc >> >> On Tue, Jul 26, 2011 at 12:03 PM, Pete Haworth >> wrote: >> >>> Another way is: >>> >>> - Place the button where you want it. >>> - Select the group >>> - Click Ungroup >>> - Shift click the new button so it's selected along with all the original >>> members of the group >>> - Click Group >>> >>> LC remembers all the properties of the group and reinstates them (as long >>> as >>> you don't close the card before grouping). >>> >>> I believe there's a plugin available somewhere that provides ways of >> adding >>> controls to a group by re-layering but I can't remember what it's called. >>> >>> Pete >>> >>> >>> >>> On Tue, Jul 26, 2011 at 9:25 AM, Bob Sneidar wrote: >>> >>>> Graphically: >>>> >>>> 1. Click group to be edited >>>> 2. Click Edit Group on your button bar, or select it in the Object menu >>>> 3. Add your button (or paste it if you cut it previously) >>>> 4. (very important) Click the Edit Group button (or menu) again or >>>> everything else you do from here on out will be done to the group! >>>> >>>> (ask me how I know) >>>> >>>> Bob >>>> >>>> >>>> On Jul 26, 2011, at 3:23 AM, Matthias Rebbe wrote: >>>> >>>>> Hi, >>>>> >>>>> maybe a dumb question, but how do i add for example a button to an >>>> already existing group? >>>>> >>>>> Matthias >>>>> >>>>> >>>>> _______________________________________________ >>>>> use-livecode mailing list >>>>> use-livecode at lists.runrev.com >>>>> Please visit this url to subscribe, unsubscribe and manage your >>>> subscription preferences: >>>>> http://lists.runrev.com/mailman/listinfo/use-livecode >>>> >>>> >>>> _______________________________________________ >>>> use-livecode mailing list >>>> use-livecode at lists.runrev.com >>>> Please visit this url to subscribe, unsubscribe and manage your >>>> subscription preferences: >>>> http://lists.runrev.com/mailman/listinfo/use-livecode >>>> >>> _______________________________________________ >>> use-livecode mailing list >>> use-livecode at lists.runrev.com >>> Please visit this url to subscribe, unsubscribe and manage your >>> subscription preferences: >>> http://lists.runrev.com/mailman/listinfo/use-livecode >>> >> _______________________________________________ >> use-livecode mailing list >> use-livecode at lists.runrev.com >> Please visit this url to subscribe, unsubscribe and manage your >> subscription preferences: >> http://lists.runrev.com/mailman/listinfo/use-livecode >> > _______________________________________________ > use-livecode mailing list > use-livecode at lists.runrev.com > Please visit this url to subscribe, unsubscribe and manage your subscription preferences: > http://lists.runrev.com/mailman/listinfo/use-livecode From Camm29 at tesco.net Tue Jul 26 16:30:01 2011 From: Camm29 at tesco.net (Camm) Date: Tue, 26 Jul 2011 21:30:01 +0100 Subject: Tahoma Font Message-ID: <000001cc4bd2$cd1e4470$675acd50$@net> Tahoma font will not work in standalone unless its run in Windows 95 or 98 compatibility mode? I am using Windows XP...... Any ideas ? Best Regards Camm From dave.cragg at lacscentre.co.uk Tue Jul 26 16:38:48 2011 From: dave.cragg at lacscentre.co.uk (Dave Cragg) Date: Tue, 26 Jul 2011 21:38:48 +0100 Subject: previous page button of browser opening on-rev server page In-Reply-To: References: <2CA55DDD-D9D0-4C0F-8E57-BAF82E464649@economy-x-talk.com> <88F2DE00-13A2-4AD0-B9CE-961564DAB131@economy-x-talk.com> <6132513B-F65D-48F3-8428-FCE4424CCE55@lacscentre.co.uk> Message-ID: On 26 Jul 2011, at 14:54, paolo mazza wrote: > Actually the script suggested by Jan works, and I get a page with 2 > new headers. However, for some reasons, the browser do not reload the > page. > > Dave, you are right, I was unfair with PHP... still, at least "put > the long date" is pretty elegant. I should apologize as I butted in without following the earlier mails. But your problem has got me interested as I didn't know the back button doesn't respect the cache settings. I found this solution using "no-store" that appears to work for Firefox but not Safari. Can't test IE right now. For PHP... header("Cache-Control: no-cache, no-store, must-revalidate"); header("Expires: Fri, 30 oct 1998 14:18:41 GMT"); echo date("Y:m:d-H:i:s"); For Livecode (not tested) put header "Cache-Control: no-cache, no-store, must-revalidate" put header "Expires: Fri, 30 oct 1998 14:18:41 GMT" put the long time Cheers Dave From bvg at mac.com Tue Jul 26 17:28:26 2011 From: bvg at mac.com (=?iso-8859-1?Q?Bj=F6rnke_von_Gierke?=) Date: Tue, 26 Jul 2011 23:28:26 +0200 Subject: [ANN] LiveCode.tv event #33 In-Reply-To: <497CC7C9-74BC-419C-9785-CEA07A8FE181@mac.com> References: <6B865405-BBC0-491C-AD25-BFB1C85A2D58@mac.com> <7EC6A85B-DD6B-4138-A597-512A194F4453@mac.com> <89782C22-C6E3-4245-95D1-A48F69423753@mac.com> <061D9182-B34B-4ABB-A7D6-9C2D09DA8755@mac.com> <73F84954-890B-4627-859A-702C8054F13B@mac.com> <093DEF5E-6C0F-4924-A262-F6DBE748ED45@mac.com> <3FB852C9-5675-4A19-9506-543733BB2547@mac.com> <914DF999-596E-4023-A213-9469C6A69FA0@mac.com> <2B105965-FB27-41D0-B95F-1F4C84ADD53B@mac.com> <163225D1-67D1-4CE7-8049-E85A8D94D177@mac.com> <225B0941-5D11-434A-BC0B-CD61B998E9F8@mac.com> <201756EB-9601-417D-856E-5128C5256EF1@mac.com> <081FD717-0A48-447D-90AA-A8F370B14F43@mac.com> <913B7E78-3052-4CF0-9883-C1CDD9F8BDFA@mac.com> <0C1B6D89-1745-46DF-9C49-A5FE2E5B67B9@mac.com> <497CC7C9-74BC-419C-9785-CEA07A8FE181@mac.com> Message-ID: Everyone had a great time, you should join in as soon as you have time. Mario showed a tool he has in development, that allows him to create custom buttons with blending and 3d effect very easily. We do have a recording for you. http://www.ustream.tv/recorded/16197148 David showed his almost finished collaboration platform, which he wants to release as public beta very soon. As usual, he did not record his presentation. David was also the one who won the "take control of upgrading to os x" eBook. See you dearies next weekend Bj?rnke On 23 Jul 2011, at 03:01, Bj?rnke von Gierke wrote: > I invite you heartily to join us tomorrow and watch Mario and David. Simply join ChatRev, and win an eBook about Mac OS X Lion: > Go to > http://bjoernke.com/chatrev/ > or enter in the message box: > go stack URL ?http://bjoernke.com/chatrev/chatrev1.3b3.rev? > > This Saturday at 20:00 in Monaco: > Sat. 22:00 Moscow > Sat. 14:00 New York > Sat. 11:00 Los Angeles > Sun. 2:00 Beijing > > Mario Miele will show us his style editing and storing palette. It allows him to grab the styles and visual properties of any object, and store them for later re-usal. > http://livecode.tv/mario/ > > Between shows, the European HyperCard User Group (eHUG, http://www.ehug.info) will raffle off an e-book courtesy of TidBITS. This time around it?s ?Take Control of Using Lion?. > You can find more info on the TC series here: > http://www.takecontrolbooks.com > > David Bovill will wrap up the night with a look at his work on version control framework, which he aims to release soon, and makes collaboratively working on the same stack with several people automatically work. > http://livecode.tv/david/ > > cheers > Bj?rnke > > > >> On 15 Jul 2011, at 04:12, Bj?rnke von Gierke wrote: >> >>> Join us once again, when we show LiveCode at it's best. >>> >>> This Saturday, Pete Haworth will update us on his qd[SQL] development. qd[SQL] is a currently in development, and will allow you to easily create databases without using any SQL at all, by linking LiveCodes object/group/card metaphor to a backend Database automatically. >>> >>> See also his explanation on Google Docs: https://docs.google.com/document/d/1Rl7B3JJrnmOGctXqd-OL26a37jN10cVD43lujyF26uE/edit?hl=en_US >>> >>> Then, the European HyperCard User Group (eHUG, http://www.ehug.info) will raffle off an e-book courtesy of TidBITS. This time around it?s ?Take Control of TextExpander?. >>> You can find more info on the TC series here: >>> http://www.takecontrolbooks.com >>> >>> Finally, Bj?rnke von Gierke will take you on a tour trough some of the add-ons from the 3 for 2 special deal ( http://www.runrev.com/mailers/3for2/3for2.html ), currently offered on the RunRev store with the coupon code "THREEFORTWO". Some of the creator of those plugins will also be available in chat, so keep those questions ready. >>> >>> Don't forget that you can vote on which plug-in you are interested in: >>> http://blog.livecode.tv/2011/07/poll-for-event-32/ >>> >>> >>> All this and more on Saturdays Livecode.tv show: >>> Sat. 22:00 Moscow >>> Sat. 20:00 Paris >>> Sat. 19:00h GMT >>> Sat. 14:00 New York >>> Sat. 11:00 Los Angeles >>> Sun. 04:00 Sydney >>> Sun. 03:00 Tokyo >>> Sun. 2:00 Beijing >>> >>> Make sure to join ChatRev on the right time, otherwise you won?t know where to watch, because the correct streams will be announced in ChatRev. ChatRev is open 24/7 for your chatting needs. >>> Go to >>> http://bjoernke.com/chatrev/ >>> or enter in the message box: >>> go stack URL ?http://bjoernke.com/chatrev/chatrev1.3b3.rev? >>> >>> If you would like to show something in an event, now or somewhen, please head over to the participation page for ideas, or contact us: >>> http://blog.livecode.tv/participate/ >>> http://www3.economy-x-talk.com/file.php?node=contact >>> >>> cheers >>> Bj?rnke >>> _______________________________________________ >>> use-livecode mailing list >>> use-livecode at lists.runrev.com >>> Please visit this url to subscribe, unsubscribe and manage your subscription preferences: >>> http://lists.runrev.com/mailman/listinfo/use-livecode >> >> >> _______________________________________________ >> use-livecode mailing list >> use-livecode at lists.runrev.com >> Please visit this url to subscribe, unsubscribe and manage your subscription preferences: >> http://lists.runrev.com/mailman/listinfo/use-livecode > > > _______________________________________________ > use-livecode mailing list > use-livecode at lists.runrev.com > Please visit this url to subscribe, unsubscribe and manage your subscription preferences: > http://lists.runrev.com/mailman/listinfo/use-livecode From scott at tactilemedia.com Tue Jul 26 17:35:09 2011 From: scott at tactilemedia.com (Scott Rossi) Date: Tue, 26 Jul 2011 14:35:09 -0700 Subject: Tahoma Font In-Reply-To: <000001cc4bd2$cd1e4470$675acd50$@net> Message-ID: I personally have been fighting a lot with cross platform font issues and find it a chore. When using stacks' non-default fonts/sizes, I've found no other solution than to brute-force set these upon opening stacks. Maybe someone else knows the secrets for handling this better. Have you made certain that Tahoma is the only problem font and tested other fonts in your current setup? If you haven't already, you might try explicitly applying your text settings at startup in the standalone. Regards, Scott Rossi Creative Director Tactile Media, UX Design Recently, Camm wrote: > Tahoma font will not work in standalone unless its run in Windows 95 or 98 > compatibility mode? > > > > I am using Windows XP...... Any ideas ? > > > > Best Regards > > Camm > > _______________________________________________ > use-livecode mailing list > use-livecode at lists.runrev.com > Please visit this url to subscribe, unsubscribe and manage your subscription > preferences: > http://lists.runrev.com/mailman/listinfo/use-livecode From jacque at hyperactivesw.com Tue Jul 26 17:36:19 2011 From: jacque at hyperactivesw.com (J. Landman Gay) Date: Tue, 26 Jul 2011 16:36:19 -0500 Subject: [OT] PHP scripter Message-ID: <4E2F3353.1010307@hyperactivesw.com> The daughter of a friend has a PHP script that is throwing errors, and she needs someone to take a look and fix it. Since I know some of you know more than one language, I said I'd ask here to see if anyone wants an afternoon's work. Drop me a note off-list if you're interested, she has a modest budget. -- Jacqueline Landman Gay | jacque at hyperactivesw.com HyperActive Software | http://www.hyperactivesw.com From bobs at twft.com Tue Jul 26 17:56:42 2011 From: bobs at twft.com (Bob Sneidar) Date: Tue, 26 Jul 2011 14:56:42 -0700 Subject: The proper place to put things Message-ID: <11080EDA-699A-4119-8134-752DCF7CF5E7@twft.com> Hi all. In 4.6.2 I had some 3rd party stuff in the /Applications/Revolution Studio/Third Party/ folder. sqlYoga was one of them. 4.6.3 does not seem to see them anymore. My User Extensions is set to /Users/Shared/Livecode/ so that all user logins gain access to all my LiveCode plugins. So where should I now put the Third Party stuff? In the Plugins folder? I wonder why 4.6.2 looked in /Applications/Revolution Studio/Third Party/ but 4.6.3 does not? Bob From bobs at twft.com Tue Jul 26 18:01:36 2011 From: bobs at twft.com (Bob Sneidar) Date: Tue, 26 Jul 2011 15:01:36 -0700 Subject: The proper place to put things Message-ID: NVM I remember now that I had to make libSQLYoga a substack of my main application. I will do that. But if there is a place to "put it" that is proper so that Start Using can find it, I'd like to put it there. Bob > Hi all. > > In 4.6.2 I had some 3rd party stuff in the /Applications/Revolution Studio/Third Party/ folder. sqlYoga was one of them. 4.6.3 does not seem to see them anymore. My User Extensions is set to /Users/Shared/Livecode/ so that all user logins gain access to all my LiveCode plugins. So where should I now put the Third Party stuff? In the Plugins folder? I wonder why 4.6.2 looked in /Applications/Revolution Studio/Third Party/ but 4.6.3 does not? > > Bob > > From jacque at hyperactivesw.com Tue Jul 26 18:11:58 2011 From: jacque at hyperactivesw.com (J. Landman Gay) Date: Tue, 26 Jul 2011 17:11:58 -0500 Subject: Tahoma Font In-Reply-To: References: Message-ID: <4E2F3BAE.2010606@hyperactivesw.com> On 7/26/11 4:35 PM, Scott Rossi wrote: > I personally have been fighting a lot with cross platform font issues and > find it a chore. When using stacks' non-default fonts/sizes, I've found no > other solution than to brute-force set these upon opening stacks. Maybe > someone else knows the secrets for handling this better. I do the same thing and it's a pain. I think this is something that profiles are supposed to be good for, but I haven't actually tried them. -- Jacqueline Landman Gay | jacque at hyperactivesw.com HyperActive Software | http://www.hyperactivesw.com From bobs at twft.com Tue Jul 26 18:37:14 2011 From: bobs at twft.com (Bob Sneidar) Date: Tue, 26 Jul 2011 15:37:14 -0700 Subject: Tahoma Font In-Reply-To: <4E2F3BAE.2010606@hyperactivesw.com> References: <4E2F3BAE.2010606@hyperactivesw.com> Message-ID: It's been an assumption of mine that if planning for standalone application cross platform support, there are three approaches: 1. ONLY use fonts common to all systems. This is fairly easy with Windows and Apple, but becomes problematic with other flavors. 2. Create an installer that includes the truetype fonts you use, and install them via a shell. This of course may mean licensing the fonts you want to use, unless you choose public domain fonts. 3. Script for the detected operating system and as Scott said, brute force it. This is just ugly, but is probably the most flexible way to go about it. my 2?. Not saying anything new I suppose. I had an issue years ago where Microsoft had a font they used in Windows which Foxpro made much use of, and the app I developed for was built around it. I was porting a Mac version of the app, so I contacted Microsoft to see if I could get a copy of the font for use on Apples, and was willing to license it. They took the line that all parts of the Microsoft operating system were part and parcel of it, and would not allow me to obtain or use it under any circumstance, even though I was using Microsoft Foxpro for the Mac. Is it lawyers that make companies anal, or vis versa? Bob On Jul 26, 2011, at 3:11 PM, J. Landman Gay wrote: > On 7/26/11 4:35 PM, Scott Rossi wrote: >> I personally have been fighting a lot with cross platform font issues and >> find it a chore. When using stacks' non-default fonts/sizes, I've found no >> other solution than to brute-force set these upon opening stacks. Maybe >> someone else knows the secrets for handling this better. > > I do the same thing and it's a pain. I think this is something that profiles are supposed to be good for, but I haven't actually tried them. > > -- > Jacqueline Landman Gay | jacque at hyperactivesw.com > HyperActive Software | http://www.hyperactivesw.com > > _______________________________________________ > use-livecode mailing list > use-livecode at lists.runrev.com > Please visit this url to subscribe, unsubscribe and manage your subscription preferences: > http://lists.runrev.com/mailman/listinfo/use-livecode From warren at warrensweb.us Tue Jul 26 19:09:49 2011 From: warren at warrensweb.us (Warren Samples) Date: Tue, 26 Jul 2011 18:09:49 -0500 Subject: Tahoma Font In-Reply-To: References: <4E2F3BAE.2010606@hyperactivesw.com> Message-ID: <201107261809.49508.warren@warrensweb.us> On Tuesday, July 26, 2011 05:37:14 PM Bob Sneidar wrote: > 1. ONLY use fonts common to all systems. This is fairly easy with Windows > and Apple, but becomes problematic with other flavors. 2. Create an > installer that includes the truetype fonts you use, and install them via a > shell. This of course may mean licensing the fonts you want to use, unless > you choose public domain fonts. 3. Script for the detected operating > system and as Scott said, brute force it. This is just ugly, but is > probably the most flexible way to go about it. I have seen font surveys and lists which suggest that even the popular web-safe font list is not as bullet- proof as most sources imply. I Don't understand why detecting the system and setting the fonts accordingly should seem ugly to anyone. It seems like a perfectly natural thing to do. It's certainly not something that needs to be rediscovered every time one starts a project. Play with it a little one day, and store your script snippet. You can do it! It doesn't strike me as philosophically any different from bundling a font and forcing the use of that font, btw; just forces you to jump a different (low) hurdle :) Best, Warren From lists.pete at haworths.org Tue Jul 26 19:28:49 2011 From: lists.pete at haworths.org (Pete Haworth) Date: Tue, 26 Jul 2011 16:28:49 -0700 Subject: Getting a list of non-Rev main stacks Message-ID: I have a need to get a list of all open application stacks. The mainstacks property includes all the IDE stacks so I filter them out by looking for the ones that begin with "rev". That gets most of them but still leaves a few that don't start with "rev", e.g "Home", "Answer Dialog", "AskDialog", "Message Box". I keep adding to the list of specific names to exclude, but new ones crop up every now and again. Does anyone know of a fail safe method of detecting that a stack belongs to the IDE? Maybe some custom property or other? Pete From bonnmike at gmail.com Tue Jul 26 19:50:13 2011 From: bonnmike at gmail.com (Mike Bonner) Date: Tue, 26 Jul 2011 17:50:13 -0600 Subject: Getting a list of non-Rev main stacks In-Reply-To: References: Message-ID: if you want to get stacks that are not ide, you can use revloadedstacks(application) put revloadedstacks(application) into tStackList On Tue, Jul 26, 2011 at 5:28 PM, Pete Haworth wrote: > I have a need to get a list of all open application stacks. The mainstacks > property includes all the IDE stacks so I filter them out by looking for > the > ones that begin with "rev". That gets most of them but still leaves a few > that don't start with "rev", e.g "Home", "Answer Dialog", "AskDialog", > "Message Box". I keep adding to the list of specific names to exclude, but > new ones crop up every now and again. > > Does anyone know of a fail safe method of detecting that a stack belongs to > the IDE? Maybe some custom property or other? > > > Pete > _______________________________________________ > use-livecode mailing list > use-livecode at lists.runrev.com > Please visit this url to subscribe, unsubscribe and manage your > subscription preferences: > http://lists.runrev.com/mailman/listinfo/use-livecode > From bobs at twft.com Tue Jul 26 19:52:12 2011 From: bobs at twft.com (Bob Sneidar) Date: Tue, 26 Jul 2011 16:52:12 -0700 Subject: Tahoma Font In-Reply-To: <201107261809.49508.warren@warrensweb.us> References: <4E2F3BAE.2010606@hyperactivesw.com> <201107261809.49508.warren@warrensweb.us> Message-ID: I should have prefaced it with, "In the spirit of rapid application development...". :-) On Jul 26, 2011, at 4:09 PM, Warren Samples wrote: > On Tuesday, July 26, 2011 05:37:14 PM Bob Sneidar wrote: > > >> 1. ONLY use fonts common to all systems. This is fairly easy with Windows >> and Apple, but becomes problematic with other flavors. 2. Create an >> installer that includes the truetype fonts you use, and install them via a >> shell. This of course may mean licensing the fonts you want to use, unless >> you choose public domain fonts. 3. Script for the detected operating >> system and as Scott said, brute force it. This is just ugly, but is >> probably the most flexible way to go about it. > > > I have seen font surveys and lists which suggest that even the popular web-safe font list is not as bullet- > proof as most sources imply. > > I Don't understand why detecting the system and setting the fonts accordingly should seem ugly to anyone. It > seems like a perfectly natural thing to do. It's certainly not something that needs to be rediscovered every > time one starts a project. Play with it a little one day, and store your script snippet. You can do it! It > doesn't strike me as philosophically any different from bundling a font and forcing the use of that font, btw; > just forces you to jump a different (low) hurdle :) > > Best, > > Warren > > _______________________________________________ > use-livecode mailing list > use-livecode at lists.runrev.com > Please visit this url to subscribe, unsubscribe and manage your subscription preferences: > http://lists.runrev.com/mailman/listinfo/use-livecode From gwendalwood at cox.net Tue Jul 26 20:01:08 2011 From: gwendalwood at cox.net (gwendalwood at cox.net) Date: Tue, 26 Jul 2011 17:01:08 -0700 Subject: iPad file format Message-ID: <20110726200108.77L2A.957958.imail@fed1rmwml4201> I have an app that on the Mac, opens, read the files in a folder called "Songs" and put the names of the files into a field. When the user clicks on the name in the listfield, the player plays that song. Works great! However, ......when I compile and run on the iPad, nothing I can find allows me to add those songs to the iPad using iTunes or whatever and then find those songs on the iPad. I assumed that adding songs to the Music folder in iTunes would put the song files into a folder called Music and then I could list the files in that folder and put them into the fld in my application and play them. Nothing I do works. I have tried writing a text file, and that is successful. Any suggestions? Thanks From bruceap at comcast.net Tue Jul 26 20:20:21 2011 From: bruceap at comcast.net (Bruce Pokras) Date: Tue, 26 Jul 2011 20:20:21 -0400 Subject: on-rev client In-Reply-To: References: <1311526520.51358.YahooMailClassic@web161614.mail.bf1.yahoo.com> Message-ID: I had the same problem this past weekend. I ended up doing my edits with a text editor. Then on Sunday (after a reboot, but that might have been a coincidence) the client started uploading again. It was weird. On Jul 24, 2011, at 12:57 PM, Erik Schwartz wrote: > I'll take a look around. I have not changed anything in my dev > environment, but sometimes these things happen by themselves. > > Thanks, > > Erik > > On Sun, Jul 24, 2011 at 9:55 AM, Michael Kann wrote: >> Erik, >> >> I use FileZilla to upload files to the on-rev server. One mistake I often make is that I upload a file to the server but I haven't selected the correct directory on the server. The file arrives in the wrong place but I don't realize it. When the webpage I've updated doesn't appear in my browser it looks like the server wasn't updated, but in reality it was updated in the wrong directory. Just something to watch out for. >> > -- > ========================================== > eriks at sisyph.us http://sisyph.us > (530) 213-ERIK http://twitter.com/eriks > (530) 213-3745 http://www.linkedin.com/in/erikschwartz > ========================================== > > _______________________________________________ > use-livecode mailing list > use-livecode at lists.runrev.com > Please visit this url to subscribe, unsubscribe and manage your subscription preferences: > http://lists.runrev.com/mailman/listinfo/use-livecode From tsj at unimelb.edu.au Tue Jul 26 20:21:08 2011 From: tsj at unimelb.edu.au (Terry Judd) Date: Wed, 27 Jul 2011 10:21:08 +1000 Subject: iPad file format In-Reply-To: <20110726200108.77L2A.957958.imail@fed1rmwml4201> Message-ID: Did you import the songs into iTunes as well? Once you've added your folder of songs to your 'iTunes music' folder you need to fire up iTunes and import them (Select 'Add to Library...' from the file menu). From there you should be able to sync them to your iPad. Not sure how you go about accessing them from your iPad app once you've done that. Terry... On 27/07/2011 10:01 AM, "gwendalwood at cox.net" wrote: > I have an app that on the Mac, opens, read the files in a folder called > "Songs" and put the names of the files into a field. When the user clicks on > the name in the listfield, the player plays that song. Works great! > However, > ......when I compile and run on the iPad, nothing I can find allows me to add > those songs to the iPad using iTunes or whatever and then find those songs on > the iPad. I assumed that adding songs to the Music folder in iTunes would put > the song files into a folder called Music and then I could list the files in > that folder and put them into the fld in my application and play them. > Nothing I do works. I have tried writing a text file, and that is successful. > Any suggestions? > Thanks > > _______________________________________________ > use-livecode mailing list > use-livecode at lists.runrev.com > Please visit this url to subscribe, unsubscribe and manage your subscription > preferences: > http://lists.runrev.com/mailman/listinfo/use-livecode > -- Dr Terry Judd | Senior Lecturer in Medical Education Medical Education Unit Melbourne Medical School The University of Melbourne From gerry.orkin at gmail.com Tue Jul 26 20:41:26 2011 From: gerry.orkin at gmail.com (Gerry Orkin) Date: Wed, 27 Jul 2011 10:41:26 +1000 Subject: iPad file format In-Reply-To: <20110726200108.77L2A.957958.imail@fed1rmwml4201> References: <20110726200108.77L2A.957958.imail@fed1rmwml4201> Message-ID: <933C5EF0-57DD-42C7-BAB5-933F732765BF@gmail.com> We don't (yet) have the ability to access music files that are synced via iTunes on computers to the iPad/iPhone iTunes app on iOS devices. However we can access music files that we copy to one of the folders owned by our apps on those devices. For example, my app comes with 3 mp3s already installed, and the user can download more. I include the default mp3s in the Copy Files pane of the standalone settings, and copy them to the Document folder on the device with a script the first time the user opens the app. Is that possible in your case? Gerry On 27/07/2011, at 10:01 AM, wrote: > I have an app that on the Mac, opens, read the files in a folder called "Songs" and put the names of the files into a field. When the user clicks on the name in the listfield, the player plays that song. Works great! > However, > ......when I compile and run on the iPad, nothing I can find allows me to add those songs to the iPad using iTunes or whatever and then find those songs on the iPad. I assumed that adding songs to the Music folder in iTunes would put the song files into a folder called Music and then I could list the files in that folder and put them into the fld in my application and play them. > Nothing I do works. I have tried writing a text file, and that is successful. > Any suggestions? From scott at tactilemedia.com Tue Jul 26 20:47:12 2011 From: scott at tactilemedia.com (Scott Rossi) Date: Tue, 26 Jul 2011 17:47:12 -0700 Subject: iPad file format In-Reply-To: Message-ID: I don't believe the engine provides access to the device music library yet, though this feature has been (strongly) requested: http://quality.runrev.com/show_bug.cgi?id=9339 Regards, Scott Rossi Creative Director Tactile Media, UX Design Recently, Terry Judd wrote: > Did you import the songs into iTunes as well? Once you've added your folder > of songs to your 'iTunes music' folder you need to fire up iTunes and import > them (Select 'Add to Library...' from the file menu). From there you should > be able to sync them to your iPad. Not sure how you go about accessing them > from your iPad app once you've done that. > > Terry... > > > On 27/07/2011 10:01 AM, "gwendalwood at cox.net" wrote: > >> I have an app that on the Mac, opens, read the files in a folder called >> "Songs" and put the names of the files into a field. When the user clicks on >> the name in the listfield, the player plays that song. Works great! >> However, >> ......when I compile and run on the iPad, nothing I can find allows me to add >> those songs to the iPad using iTunes or whatever and then find those songs on >> the iPad. I assumed that adding songs to the Music folder in iTunes would put >> the song files into a folder called Music and then I could list the files in >> that folder and put them into the fld in my application and play them. >> Nothing I do works. I have tried writing a text file, and that is successful. >> Any suggestions? >> Thanks >> >> _______________________________________________ >> use-livecode mailing list >> use-livecode at lists.runrev.com >> Please visit this url to subscribe, unsubscribe and manage your subscription >> preferences: >> http://lists.runrev.com/mailman/listinfo/use-livecode >> > > -- > Dr Terry Judd | Senior Lecturer in Medical Education > Medical Education Unit > Melbourne Medical School > The University of Melbourne From lists.pete at haworths.org Tue Jul 26 21:30:10 2011 From: lists.pete at haworths.org (Pete Haworth) Date: Tue, 26 Jul 2011 18:30:10 -0700 Subject: Getting a list of non-Rev main stacks In-Reply-To: References: Message-ID: Perfect! Thank you. Pete On Tue, Jul 26, 2011 at 4:50 PM, Mike Bonner wrote: > if you want to get stacks that are not ide, you can use > revloadedstacks(application) > > put revloadedstacks(application) into tStackList > > > On Tue, Jul 26, 2011 at 5:28 PM, Pete Haworth >wrote: > > > I have a need to get a list of all open application stacks. The > mainstacks > > property includes all the IDE stacks so I filter them out by looking for > > the > > ones that begin with "rev". That gets most of them but still leaves a > few > > that don't start with "rev", e.g "Home", "Answer Dialog", "AskDialog", > > "Message Box". I keep adding to the list of specific names to exclude, > but > > new ones crop up every now and again. > > > > Does anyone know of a fail safe method of detecting that a stack belongs > to > > the IDE? Maybe some custom property or other? > > > > > > Pete > > _______________________________________________ > > use-livecode mailing list > > use-livecode at lists.runrev.com > > Please visit this url to subscribe, unsubscribe and manage your > > subscription preferences: > > http://lists.runrev.com/mailman/listinfo/use-livecode > > > _______________________________________________ > use-livecode mailing list > use-livecode at lists.runrev.com > Please visit this url to subscribe, unsubscribe and manage your > subscription preferences: > http://lists.runrev.com/mailman/listinfo/use-livecode > From lists.pete at haworths.org Tue Jul 26 21:34:17 2011 From: lists.pete at haworths.org (Pete Haworth) Date: Tue, 26 Jul 2011 18:34:17 -0700 Subject: Getting a list of non-Rev main stacks In-Reply-To: References: Message-ID: Oops, spoke too soon. That gets all stacks, not just main stacks, but from there I can go through and weed out the substacks so still OK. Pete On Tue, Jul 26, 2011 at 4:50 PM, Mike Bonner wrote: > if you want to get stacks that are not ide, you can use > revloadedstacks(application) > > put revloadedstacks(application) into tStackList > > > On Tue, Jul 26, 2011 at 5:28 PM, Pete Haworth >wrote: > > > I have a need to get a list of all open application stacks. The > mainstacks > > property includes all the IDE stacks so I filter them out by looking for > > the > > ones that begin with "rev". That gets most of them but still leaves a > few > > that don't start with "rev", e.g "Home", "Answer Dialog", "AskDialog", > > "Message Box". I keep adding to the list of specific names to exclude, > but > > new ones crop up every now and again. > > > > Does anyone know of a fail safe method of detecting that a stack belongs > to > > the IDE? Maybe some custom property or other? > > > > > > Pete > > _______________________________________________ > > use-livecode mailing list > > use-livecode at lists.runrev.com > > Please visit this url to subscribe, unsubscribe and manage your > > subscription preferences: > > http://lists.runrev.com/mailman/listinfo/use-livecode > > > _______________________________________________ > use-livecode mailing list > use-livecode at lists.runrev.com > Please visit this url to subscribe, unsubscribe and manage your > subscription preferences: > http://lists.runrev.com/mailman/listinfo/use-livecode > From martyknapp at comcast.net Tue Jul 26 21:57:39 2011 From: martyknapp at comcast.net (Marty Knapp) Date: Tue, 26 Jul 2011 18:57:39 -0700 Subject: Tahoma Font In-Reply-To: References: <4E2F3BAE.2010606@hyperactivesw.com> Message-ID: <4E2F7093.7020600@comcast.net> I've not tinkered with shell commands - what is the shell command for installing fonts on Mac? Marty K > It's been an assumption of mine that if planning for standalone application cross platform support, there are three approaches: > > 1. ONLY use fonts common to all systems. This is fairly easy with Windows and Apple, but becomes problematic with other flavors. > 2. Create an installer that includes the truetype fonts you use, and install them via a shell. This of course may mean licensing the fonts you want to use, unless you choose public domain fonts. > 3. Script for the detected operating system and as Scott said, brute force it. This is just ugly, but is probably the most flexible way to go about it. > > my 2?. Not saying anything new I suppose. I had an issue years ago where Microsoft had a font they used in Windows which Foxpro made much use of, and the app I developed for was built around it. I was porting a Mac version of the app, so I contacted Microsoft to see if I could get a copy of the font for use on Apples, and was willing to license it. They took the line that all parts of the Microsoft operating system were part and parcel of it, and would not allow me to obtain or use it under any circumstance, even though I was using Microsoft Foxpro for the Mac. > > Is it lawyers that make companies anal, or vis versa? > > Bob > > > On Jul 26, 2011, at 3:11 PM, J. Landman Gay wrote: > >> On 7/26/11 4:35 PM, Scott Rossi wrote: >>> I personally have been fighting a lot with cross platform font issues and >>> find it a chore. When using stacks' non-default fonts/sizes, I've found no >>> other solution than to brute-force set these upon opening stacks. Maybe >>> someone else knows the secrets for handling this better. >> I do the same thing and it's a pain. I think this is something that profiles are supposed to be good for, but I haven't actually tried them. >> >> -- >> Jacqueline Landman Gay | jacque at hyperactivesw.com >> HyperActive Software | http://www.hyperactivesw.com >> From gwendalwood at cox.net Tue Jul 26 22:39:00 2011 From: gwendalwood at cox.net (gwendalwood at cox.net) Date: Tue, 26 Jul 2011 19:39:00 -0700 Subject: iPad directory format Message-ID: <20110726223900.NQ8NB.1358778.imail@fed1rmwml33> On July 26, Gerry wrote: "We don't (yet) have the ability to access music files that are synced via iTunes on computers to the iPad/iPhone iTunes app on iOS devices. However we can access music files that we copy to one of the folders owned by our apps on those devices. For example, my app comes with 3 mp3s already installed, and the user can download more. I include the default mp3s in the Copy Files pane of the standalone settings, and copy them to the Document folder on the device with a script the first time the user opens the app. Is that possible in your case? " Gerry: Could you send a short script to show me how to copy the mp3 file sent via the Copy files pane into the Document folder on the device? I have tried sending the mp3 file, but I can't find how to access it to use it. Thanks gwendalwood at cox.net From toolbook at kestner.de Wed Jul 27 05:04:35 2011 From: toolbook at kestner.de (Tiemo Hollmann TB) Date: Wed, 27 Jul 2011 11:04:35 +0200 Subject: AW: Tahoma Font In-Reply-To: <000001cc4bd2$cd1e4470$675acd50$@net> References: <000001cc4bd2$cd1e4470$675acd50$@net> Message-ID: <000f01cc4c3c$3b747b60$b25d7220$@de> When I started with revolution Tahoma was the default font of all objects and I didn't cared about it at that time. I am now using Tahoma since years without any problems on windows and Mac, though I have learned that Tahoma isn't a standard font on Mac and is supposed to be substituted (what never made any problems in my apps). On windows systems Tahoma is a standard font up to today as far as I know, or did I miss anything? Tiemo > -----Urspr?ngliche Nachricht----- > Von: use-livecode-bounces at lists.runrev.com [mailto:use-livecode- > bounces at lists.runrev.com] Im Auftrag von Camm > Gesendet: Dienstag, 26. Juli 2011 22:30 > An: use-livecode at lists.runrev.com > Betreff: Tahoma Font > > Tahoma font will not work in standalone unless its run in Windows 95 or 98 > compatibility mode? > > > > I am using Windows XP...... Any ideas ? > > > > Best Regards > > Camm > > _______________________________________________ > use-livecode mailing list > use-livecode at lists.runrev.com > Please visit this url to subscribe, unsubscribe and manage your subscription > preferences: > http://lists.runrev.com/mailman/listinfo/use-livecode From effendi at wanadoo.fr Wed Jul 27 06:40:28 2011 From: effendi at wanadoo.fr (Francis Nugent Dixon) Date: Wed, 27 Jul 2011 12:40:28 +0200 Subject: Where does survive the inventive user ? Message-ID: Alejandro asks some potent questions ..... Question 1 - Is programming so easy ..... ? I think we should ask the question "Is programming a niche occupation ? During my early career, practically everybody I knew was a programmer (birds of a feather flock together ?) Now I am out of the industry (retired), outside of the LiveCode forum, I don't know anybody who programs (although most of my acquaintances have computers and Google all day, they don't WRITE programs.) Programming is a mentality, and there aren't many of us who have this mentality (even to make money from it). Although LiveCode is a great incentive for non-programmers to "have a go" , programming is limiited to a strange mind-form which even I cannot define ! The question should be - "What is the VISIBILTY of LiveCode to the man in the street who has never even thought about programming ?" And the answer is "NONE". The chances of anybody "moving in" to programming are about the same as being struck by lightning. Question 2 Did anyone know someone ..... ? I would think that buying a mobile platform (iPhone, iPad), and learning to program are two ideas so far apart, as to be unlikely. I do not know anyone who has started programming because of this "mobile technology" and the platforms thereon. In a recent thread on the forum I voiced my "No Way Baby" intent to NOT go to mobile computing, although I wrote my first program more than 50 years ago, and I still program EVERY day. The cost is far too high, and the returns are doubtful ! I am a rare bird who now programs for fun. Question 3 Will mobile computing displace desktop computing .... ? This reminds me of the 1980's question "Will Desktop computing ever displace Mainframe Computing ?" A lot of people said no in them days. Industry has been talking about "cloud computing" for years, and IS slowly moving towards it. But, although I may accept having my data in a cloud, I will always want my apps to be in my hand, so I can have control over them. Renting application use out of a cloud would be the same has handing over your wallet to a stranger. You can see which direction Apple is going. They want to charge you for the use of YOUR OWN computer, and then for storing your data in their cloud, and then for using their applications from their cloud. That could cost you an arm and a leg. All my communication in the hands of a stranger ? It's bad enough already! God help us all in the future ! The problem is - it's not hype - it's tomorrows computing, and I don't like the way the wind is blowing ..... The days when you rented an application, and you got the computer for free may return. When computers become so dirt cheap that there is no big profit to make, those guys "up there" have to think of a new way to get your money. We will soon be paying more for communication facilities than we are spending on food (si ce n'est deja fait !, as they say here)! "Nothing should ever be done for the first time !" -Francis PS. How about the question "When will we be grafting micro-chips into the brain to allow us instant and global communication, and complex problem solving and decision making ? From mazzapaolo at libero.it Wed Jul 27 06:59:30 2011 From: mazzapaolo at libero.it (paolo mazza) Date: Wed, 27 Jul 2011 12:59:30 +0200 Subject: previous page button of browser opening on-rev server page In-Reply-To: References: <2CA55DDD-D9D0-4C0F-8E57-BAF82E464649@economy-x-talk.com> <88F2DE00-13A2-4AD0-B9CE-961564DAB131@economy-x-talk.com> <6132513B-F65D-48F3-8428-FCE4424CCE55@lacscentre.co.uk> Message-ID: Thank you Dave. I found out that Safary needs an additional attribute in the body tag : to reload the page. So, here it is the code working for Firefox, Safari and IE, both with PHP and LiveCode server. PHP Documento senza titolo LIVECODE Documento senza titolo All the best. Paolo Mazza From andre.bisseret at wanadoo.fr Wed Jul 27 07:41:26 2011 From: andre.bisseret at wanadoo.fr (=?iso-8859-1?Q?Andr=E9_Bisseret?=) Date: Wed, 27 Jul 2011 13:41:26 +0200 Subject: A case of assigned behavior not taken into account Message-ID: <9EC1084F-AC69-492B-A2C6-1658129FF052@wanadoo.fr> Bonjour, On an app. I am developing on Mac, I have a main stack a substack of which is a model used for creating new stacks which are cloned from the model and saved as "independent" stacks (not substacks). The scripts of the card 1 of this model and of all objects on this card are all together in a behavior button which is assigned to this card 1. This behavior is on card 2 of the main stack. All is working well on Mac. But when I load the standalone for Windows on a PC (by means of a USB key) then a newly created stack from the model is inert. Meanwhile, I verified that the behavior is actually assigned to the card 1 of the new stack, but all behaves like this was not the case! If, by script, I reassign the behavior to card 1 of the model before cloning it, then the new stack is working as expected. So I could stay with this reassignment but? Is it normal (seems not to me!), or am I missing something ? Any hint much appreciated Best regards from Grenoble Andr? From index at kenjikojima.com Wed Jul 27 07:49:27 2011 From: index at kenjikojima.com (Kenji Kojima) Date: Wed, 27 Jul 2011 07:49:27 -0400 Subject: Safari 5.1 cannot show revlets Message-ID: <8F1E5783-524F-498D-AC95-2E99E0ED5E92@kenjikojima.com> Hi, Safari 5.1 cannot show my revlet stacks. It shows only white stack. I updated a plugin to LiveCode Player(R9), but did not work. Safari 5.0.5 and firefox can show them. I made the revlet stacks by LiveCode 4.5.3. Does anybody have same problem? URLs: http://kenjikojima.com/newsmusic/rgbRevlet.html http://kenjikojima.com/rgbmusiclab/ http://kenjikojima.com/rgbmusicrenga/nycsubway/ Thanks, -- Kenji Kojima / ???? http://www.kenjikojima.com/ From dunbarx at aol.com Wed Jul 27 08:35:49 2011 From: dunbarx at aol.com (dunbarx at aol.com) Date: Wed, 27 Jul 2011 08:35:49 -0400 (EDT) Subject: Where does survive the inventive user ? In-Reply-To: References: Message-ID: <8CE1A70D9625EA6-2550-214DE@web-mmc-d04.sysops.aol.com> In the old days, Hypercard was. like a viral pandemic, infected the world because it was bundled with every Macintosh. It was offered like a promotion, a possibly valuable coupon one gets in the mail, which you will at least read before throwing out, And it became a nerd fad, with many hundreds of thousands of people trying it out. Certainly only a small fraction became enamored; many of those are reading this post. Without that once in a lifetime vehicle, it is an uphill battle to engage people who might fall in love with LC if they only were simply exposed to it. Worse, these days, the mindset is that everything comes in small ready-to-go packages, complete and compact. I have three kids who just don't think about building stuff, especially from raw materials. I used to, though. Thank the iMac, iPhone, iPod, etc., for creating that expectation, a far cry from reading a bank of eight lights telling you what byte was currently passing by. LC should be taught in the ninth grade in every school in the world. Craig Newman -----Original Message----- From: Francis Nugent Dixon To: use-livecode Sent: Wed, Jul 27, 2011 2:42 am Subject: Where does survive the inventive user ? Alejandro asks some potent questions ..... Question 1 - Is programming so easy ..... ? I think we should ask the question "Is programming a niche occupation ? During my early career, practically everybody I knew was a programmer (birds of a feather flock together ?) Now I am out of the industry (retired), outside of the LiveCode forum, I don't know anybody who programs (although most of my acquaintances have computers and Google all day, they don't WRITE programs.) Programming is a mentality, and there aren't many of us who have this mentality (even to make money from it). Although LiveCode is a great incentive for non-programmers to "have a go" , programming is limiited to a strange mind-form which even I cannot define ! The question should be - "What is the VISIBILTY of LiveCode to the man in the street who has never even thought about programming ?" And the answer is "NONE". The chances of anybody "moving in" to programming are about the same as being struck by lightning. Question 2 Did anyone know someone ..... ? I would think that buying a mobile platform (iPhone, iPad), and learning to program are two ideas so far apart, as to be unlikely. I do not know anyone who has started programming because of this "mobile technology" and the platforms thereon. In a recent thread on the forum I voiced my "No Way Baby" intent to NOT go to mobile computing, although I wrote my first program more than 50 years ago, and I still program EVERY day. The cost is far too high, and the returns are doubtful ! I am a rare bird who now programs for fun. Question 3 Will mobile computing displace desktop computing .... ? This reminds me of the 1980's question "Will Desktop computing ever displace Mainframe Computing ?" A lot of people said no in them days. Industry has been talking about "cloud computing" for years, and IS slowly moving towards it. But, although I may accept having my data in a cloud, I will always want my apps to be in my hand, so I can have control over them. Renting application use out of a cloud would be the same has handing over your wallet to a stranger. You can see which direction Apple is going. They want to charge you for the use of YOUR OWN computer, and then for storing your data in their cloud, and then for using their applications from their cloud. That could cost you an arm and a leg. All my communication in the hands of a stranger ? It's bad enough already! God help us all in the future ! The problem is - it's not hype - it's tomorrows computing, and I don't like the way the wind is blowing ..... The days when you rented an application, and you got the computer for free may return. When computers become so dirt cheap that there is no big profit to make, those guys "up there" have to think of a new way to get your money. We will soon be paying more for communication facilities than we are spending on food (si ce n'est deja fait !, as they say here)! "Nothing should ever be done for the first time !" -Francis PS. How about the question "When will we be grafting micro-chips into the brain to allow us instant and global communication, and complex problem solving and decision making ? _______________________________________________ use-livecode mailing list use-livecode at lists.runrev.com Please visit this url to subscribe, unsubscribe and manage your subscription preferences: http://lists.runrev.com/mailman/listinfo/use-livecode From klaus at major.on-rev.com Wed Jul 27 08:41:50 2011 From: klaus at major.on-rev.com (Klaus on-rev) Date: Wed, 27 Jul 2011 14:41:50 +0200 Subject: Safari 5.1 cannot show revlets In-Reply-To: <8F1E5783-524F-498D-AC95-2E99E0ED5E92@kenjikojima.com> References: <8F1E5783-524F-498D-AC95-2E99E0ED5E92@kenjikojima.com> Message-ID: <4D71FAFB-2F1C-428F-8B70-F2C5ECA9AF35@major.on-rev.com> Hi Kenji, Am 27.07.2011 um 13:49 schrieb Kenji Kojima: > Hi, > > Safari 5.1 cannot show my revlet stacks. > It shows only white stack. I updated a plugin to LiveCode Player(R9), but did not work. > Safari 5.0.5 and firefox can show them. > I made the revlet stacks by LiveCode 4.5.3. > Does anybody have same problem? > > URLs: > http://kenjikojima.com/newsmusic/rgbRevlet.html > http://kenjikojima.com/rgbmusiclab/ > http://kenjikojima.com/rgbmusicrenga/nycsubway/ Yep, I also get a white stack here. Safari 5.1 on OS X 10.7 > Thanks, > -- > Kenji Kojima / ???? > http://www.kenjikojima.com/ Best Klaus -- Klaus Major http://www.major-k.de klaus at major.on-rev.com From capellan2000 at gmail.com Wed Jul 27 09:45:45 2011 From: capellan2000 at gmail.com (Alejandro Tejada) Date: Wed, 27 Jul 2011 06:45:45 -0700 (PDT) Subject: Where does survive the inventive user? In-Reply-To: References: <1311706987455-3696711.post@n4.nabble.com> Message-ID: <1311774345363-3698545.post@n4.nabble.com> Hi Andre, Andre Garzia-3 wrote: > > mobile computing is consumer computing. Developers and inventive users > will > keep on platforms that allow them to develop stuff. It means that slowly, > those users will move towards freedom so even though mobile computing will > be ubiquitous, you will find the developers and inventive users using > something else where they can actually develop stuff unrestricted. They > will > probably be on linux... > I agree that mobile computing is consumer computing. Now I am scared... Just imagine the kind of malware that will expand like fire, shutting down large parts of the network. By definition, consumers are NOT the most careful users. In fact, I foresee a future where a cellphone starts malfunctioning and the consumer extract the chip and without thinking about repair it, inmediatly trash the cellphone (with the battery still working) I understand that many of you already know some people who acts like these consumers, but It's not the standard behavior, at least where I live. Andre, from my limited contact with Linux communities, I have collected the impression that they would send you to "READ THE --- MANUAL" with more frequency than Mac or Windows communities. From my personal experience, that do not reflect, in any way, others people experience... :-) Have a nice day! Al -- View this message in context: http://runtime-revolution.278305.n4.nabble.com/Where-does-survive-the-inventive-user-tp3696711p3698545.html Sent from the Revolution - User mailing list archive at Nabble.com. From lists.pete at haworths.org Wed Jul 27 10:02:18 2011 From: lists.pete at haworths.org (Pete Haworth) Date: Wed, 27 Jul 2011 07:02:18 -0700 Subject: Android Info Message-ID: This link might be of interest to those of you developing for Android: http://www.devx.com/DailyNews/Article/47096?trk=DXRSS_WEBDEV Pete From bonnmike at gmail.com Wed Jul 27 10:34:32 2011 From: bonnmike at gmail.com (Mike Bonner) Date: Wed, 27 Jul 2011 08:34:32 -0600 Subject: windows revserver stack support Message-ID: Has anyone managed to get a stack to load using livecode-server for windows? All I get is "livecode-server exited due to fatal signal 11" This happens when trying to library the stack as well as using go stack Otherwise livecode-server is working great, just can't get the stack support to work. I've even checked it from the command line to make sure that I don't have some strange apache misconfiguration going on, but the result is the same. Any suggestions? From index at kenjikojima.com Wed Jul 27 10:39:18 2011 From: index at kenjikojima.com (Kenji Kojima) Date: Wed, 27 Jul 2011 10:39:18 -0400 Subject: Safari 5.1 cannot show revlets In-Reply-To: <4D71FAFB-2F1C-428F-8B70-F2C5ECA9AF35@major.on-rev.com> References: <8F1E5783-524F-498D-AC95-2E99E0ED5E92@kenjikojima.com> <4D71FAFB-2F1C-428F-8B70-F2C5ECA9AF35@major.on-rev.com> Message-ID: <2B373C21-D677-4BDA-9E7B-43A39C7E27A7@kenjikojima.com> Klaus, I put a JavaScript note on Safari pages until it's resolved. Thanks, -- Kenji Kojima / ???? http://www.kenjikojima.com/ On Jul 27, 2011, at 8:41 AM, Klaus on-rev wrote: > Hi Kenji, > > Am 27.07.2011 um 13:49 schrieb Kenji Kojima: > >> Hi, >> >> Safari 5.1 cannot show my revlet stacks. >> It shows only white stack. I updated a plugin to LiveCode Player(R9), but did not work. >> Safari 5.0.5 and firefox can show them. >> I made the revlet stacks by LiveCode 4.5.3. >> Does anybody have same problem? >> >> URLs: >> http://kenjikojima.com/newsmusic/rgbRevlet.html >> http://kenjikojima.com/rgbmusiclab/ >> http://kenjikojima.com/rgbmusicrenga/nycsubway/ > > Yep, I also get a white stack here. > Safari 5.1 on OS X 10.7 > >> Thanks, >> -- >> Kenji Kojima / ???? >> http://www.kenjikojima.com/ > > Best > > Klaus > > -- > Klaus Major > http://www.major-k.de > klaus at major.on-rev.com > > > _______________________________________________ > use-livecode mailing list > use-livecode at lists.runrev.com > Please visit this url to subscribe, unsubscribe and manage your subscription preferences: > http://lists.runrev.com/mailman/listinfo/use-livecode From barryb at libero.it Wed Jul 27 10:49:39 2011 From: barryb at libero.it (barryb at libero.it) Date: Wed, 27 Jul 2011 16:49:39 +0200 (CEST) Subject: Clicks along a line Message-ID: <32728323.2635921311778179991.JavaMail.defaultUser@defaultHost> I am trying to make the mouse click at fixed intervals along a straight line. So far, I have the following which 'compiles' without errors: on RollEm click at "40,300" -- first point on line repeat 5 times move the mouseLoc relative "40,0" without waiting click at the mouseLoc end repeat end RollEm Only the first statement works. At line 4 it throws the error: Type move: can't find object Object card id 1002 Line move the mouseLoc relative "40,0" without waiting Hint RollEm How should I script that repeat structure? With many thanks in advance. Barry Barber From m.schonewille at economy-x-talk.com Wed Jul 27 10:56:21 2011 From: m.schonewille at economy-x-talk.com (Mark Schonewille) Date: Wed, 27 Jul 2011 16:56:21 +0200 Subject: Clicks along a line In-Reply-To: <32728323.2635921311778179991.JavaMail.defaultUser@defaultHost> References: <32728323.2635921311778179991.JavaMail.defaultUser@defaultHost> Message-ID: Hi Barry, Have a look at the screenMouseLoc. You can't move the mouseLoc, but you can set the screenMouseLoc. -- Best regards, Mark Schonewille Economy-x-Talk Consulting and Software Engineering Homepage: http://economy-x-talk.com Twitter: http://twitter.com/xtalkprogrammer KvK: 50277553 What does that error mean? Buy LiveCodeErrors for iPhone now http://qery.us/v4 A must-have for LiveCode programmers. On 27 jul 2011, at 16:49, barryb at libero.it wrote: > > I am trying to make the mouse click at fixed intervals along a straight line. So far, I have the following which 'compiles' without errors: > > on RollEm > click at "40,300" -- first point on line > repeat 5 times > move the mouseLoc relative "40,0" without waiting > click at the mouseLoc > end repeat > end RollEm > > Only the first statement works. At line 4 it throws the error: > > Type move: can't find object > Object card id 1002 > Line move the mouseLoc relative "40,0" without waiting > Hint RollEm > > How should I script that repeat structure? > With many thanks in advance. > Barry Barber From bonnmike at gmail.com Wed Jul 27 11:04:59 2011 From: bonnmike at gmail.com (Mike Bonner) Date: Wed, 27 Jul 2011 09:04:59 -0600 Subject: windows revserver stack support In-Reply-To: References: Message-ID: More information. On a different machine, *ubuntu 1, apache2, livecode-server 4-6-3* there is also a signal 11, but quite a bit more info (that I don't understand.) livecode-server exited due to fatal signal 11 in __kernel_sigreturn @ 0 in @ 0x8158f85 in @ 0x809e596 in @ 0x809ea13 in @ 0x809eccc in @ 0x809177d in @ 0x8091fed in @ 0x81cd09e in @ 0x81c96d3 in @ 0x81c9b40 in __libc_start_main @ 231 in fwrite @ 305 I'm going to re-download the livecode server distributions on the off chance theres been a sneaky fix or update that I haven't seen, but at this point i'm without clue. Which is of course my usual state of being! *grin* Will send a note to support in a day or so if nobody can point me to whatever mistake I made. Thanks in advance for any help On Wed, Jul 27, 2011 at 8:34 AM, Mike Bonner wrote: > Has anyone managed to get a stack to load using livecode-server for > windows? All I get is "livecode-server exited due to fatal signal 11" > > This happens when trying to library the stack as well as using go stack > > > Otherwise livecode-server is working great, just can't get the stack > support to work. > > I've even checked it from the command line to make sure that I don't have > some strange apache misconfiguration going on, but the result is the same. > > > Any suggestions? > From barryb at libero.it Wed Jul 27 11:34:32 2011 From: barryb at libero.it (barryb at libero.it) Date: Wed, 27 Jul 2011 17:34:32 +0200 (CEST) Subject: Clicks along a line Message-ID: <6471148.2661021311780872148.JavaMail.defaultUser@defaultHost> Wed Jul 27 09:56:21 CDT 2011 Mark Schonewille wrote >Hi Barry, >Have a look at the screenMouseLoc. >You can't move the mouseLoc, but you can set the screenMouseLoc. Hello Mark, Thanks for another prompt reply, however I have tried everything in the book connected with mouseloc. Infact, screenMouseLoc worked well but 'clicked' along a line on my desktop! My objects are in the card ;),and also received the clicks but, as I want to return the color, nothing happened because my desktop is all black. The mouseLoc works in the first line and defining each point absolute slows things down. Anyway I feel I'm getting closer. Gratefully Barry From lfredricks at proactive-intl.com Wed Jul 27 11:43:22 2011 From: lfredricks at proactive-intl.com (Lynn Fredricks) Date: Wed, 27 Jul 2011 08:43:22 -0700 Subject: Where does survive the inventive user ? In-Reply-To: <8CE1A70D9625EA6-2550-214DE@web-mmc-d04.sysops.aol.com> References: <8CE1A70D9625EA6-2550-214DE@web-mmc-d04.sysops.aol.com> Message-ID: <84B6FD13FF704AA99037249FF171A9B2@GATEWAY> > Without that once in a lifetime vehicle, it is an uphill > battle to engage people who might fall in love with LC if > they only were simply exposed to it. Worse, these days, the > mindset is that everything comes in small ready-to-go > packages, complete and compact. I have three kids who just > don't think about building stuff, especially from raw > materials. I used to, though. > > > Thank the iMac, iPhone, iPod, etc., for creating that > expectation, a far cry from reading a bank of eight lights > telling you what byte was currently passing by. I think that began long before the iDevices. There are still creative kids out there, just sadly, far fewer of them. I also see an equal dwindling of interest in the adults that engage them (Im not implying anything about you, just look around at the next parent-involved event at your children's school to see what I am talking about). Products are easier to sell if there is instant gradification. In software for example, you should design your product so that it rewards your user no less than every five minutes when its in its demo mode. This reward can be subtle or not, but it should deliver a sense of satisfaction. By no means is this sort of thing relegated to software. It is a lot of hard work to light the fires of competitiveness and creativity in kids in this consumerist society. I know many intelligent, well educated parents who for whatever reason end up with a bunch of high school drop out Lotus Eaters living in their basement. But I also know a much, much smaller number of families that raise competitive, creative and intellectually engaged kids too. They exist - just in much smaller numbers. It is a lot easier to sell to the Lotus Eaters though. Best regards, Lynn Fredricks President Paradigma Software http://www.paradigmasoft.com Valentina SQL Server: The Ultra-fast, Royalty Free Database Server From m.schonewille at economy-x-talk.com Wed Jul 27 11:49:11 2011 From: m.schonewille at economy-x-talk.com (Mark Schonewille) Date: Wed, 27 Jul 2011 17:49:11 +0200 Subject: Clicks along a line In-Reply-To: <6471148.2661021311780872148.JavaMail.defaultUser@defaultHost> References: <6471148.2661021311780872148.JavaMail.defaultUser@defaultHost> Message-ID: Barry, Why exactly does it click on your desktop? If you re-calculate the locations, it should click on the card. There's a function globalLoc(). -- Best regards, Mark Schonewille Economy-x-Talk Consulting and Software Engineering Homepage: http://economy-x-talk.com Twitter: http://twitter.com/xtalkprogrammer KvK: 50277553 What does that error mean? Buy LiveCodeErrors for iPhone now http://qery.us/v4 A must-have for LiveCode programmers. On 27 jul 2011, at 17:34, barryb at libero.it wrote: > > Hello Mark, > Thanks for another prompt reply, however I have tried everything > in the book connected with mouseloc. > Infact, screenMouseLoc worked well but 'clicked' along a line > on my desktop! > My objects are in the card ;),and also received the clicks but, > as I want to return the color, nothing happened because my desktop > is all black. The mouseLoc works in the first line and defining each > point absolute slows things down. > > Anyway I feel I'm getting closer. > Gratefully Barry From bobs at twft.com Wed Jul 27 12:24:23 2011 From: bobs at twft.com (Bob Sneidar) Date: Wed, 27 Jul 2011 09:24:23 -0700 Subject: Tahoma Font In-Reply-To: <4E2F7093.7020600@comcast.net> References: <4E2F3BAE.2010606@hyperactivesw.com> <4E2F7093.7020600@comcast.net> Message-ID: <8F399A93-0D40-4574-926F-0997445DA62E@twft.com> You just put the font file into /System/Library/Fonts. You have to shell it if the user is not an admin because of permissions. The shell command is simply the one to copy, but you may have to sudo it because it is in the System package. Alternately you could put the font into the user's font folder ~/Library/Fonts (surprisingly this is how Microsoft does it when they run their Office Install the first time you run an Office product in a profile). This way you would not have to sudo or shell because the user (presumably) has write permission to his own library. For Windows however, I am not sure how you would go about doing that. Lemme google... AH YES! Here you go. http://www.sevenforums.com/general-discussion/28817-installing-fonts-via-command-line-script.html Bob ps. Google is your friend. On Jul 26, 2011, at 6:57 PM, Marty Knapp wrote: > I've not tinkered with shell commands - what is the shell command for installing fonts on Mac? > > Marty K >> It's been an assumption of mine that if planning for standalone application cross platform support, there are three approaches: >> >> 1. ONLY use fonts common to all systems. This is fairly easy with Windows and Apple, but becomes problematic with other flavors. >> 2. Create an installer that includes the truetype fonts you use, and install them via a shell. This of course may mean licensing the fonts you want to use, unless you choose public domain fonts. >> 3. Script for the detected operating system and as Scott said, brute force it. This is just ugly, but is probably the most flexible way to go about it. >> >> my 2?. Not saying anything new I suppose. I had an issue years ago where Microsoft had a font they used in Windows which Foxpro made much use of, and the app I developed for was built around it. I was porting a Mac version of the app, so I contacted Microsoft to see if I could get a copy of the font for use on Apples, and was willing to license it. They took the line that all parts of the Microsoft operating system were part and parcel of it, and would not allow me to obtain or use it under any circumstance, even though I was using Microsoft Foxpro for the Mac. >> >> Is it lawyers that make companies anal, or vis versa? >> >> Bob >> >> >> On Jul 26, 2011, at 3:11 PM, J. Landman Gay wrote: >> >>> On 7/26/11 4:35 PM, Scott Rossi wrote: >>>> I personally have been fighting a lot with cross platform font issues and >>>> find it a chore. When using stacks' non-default fonts/sizes, I've found no >>>> other solution than to brute-force set these upon opening stacks. Maybe >>>> someone else knows the secrets for handling this better. >>> I do the same thing and it's a pain. I think this is something that profiles are supposed to be good for, but I haven't actually tried them. >>> >>> -- >>> Jacqueline Landman Gay | jacque at hyperactivesw.com >>> HyperActive Software | http://www.hyperactivesw.com >>> > > _______________________________________________ > use-livecode mailing list > use-livecode at lists.runrev.com > Please visit this url to subscribe, unsubscribe and manage your subscription preferences: > http://lists.runrev.com/mailman/listinfo/use-livecode From bobs at twft.com Wed Jul 27 12:26:21 2011 From: bobs at twft.com (Bob Sneidar) Date: Wed, 27 Jul 2011 09:26:21 -0700 Subject: Tahoma Font In-Reply-To: <4E2F7093.7020600@comcast.net> References: <4E2F3BAE.2010606@hyperactivesw.com> <4E2F7093.7020600@comcast.net> Message-ID: <2E12D9B8-54C0-41ED-B149-9874801984B7@twft.com> I also found this. Haven't tried it though. http://corefonts.sourceforge.net/ Bob On Jul 26, 2011, at 6:57 PM, Marty Knapp wrote: > I've not tinkered with shell commands - what is the shell command for installing fonts on Mac? > > Marty K >> It's been an assumption of mine that if planning for standalone application cross platform support, there are three approaches: >> >> 1. ONLY use fonts common to all systems. This is fairly easy with Windows and Apple, but becomes problematic with other flavors. >> 2. Create an installer that includes the truetype fonts you use, and install them via a shell. This of course may mean licensing the fonts you want to use, unless you choose public domain fonts. >> 3. Script for the detected operating system and as Scott said, brute force it. This is just ugly, but is probably the most flexible way to go about it. >> >> my 2?. Not saying anything new I suppose. I had an issue years ago where Microsoft had a font they used in Windows which Foxpro made much use of, and the app I developed for was built around it. I was porting a Mac version of the app, so I contacted Microsoft to see if I could get a copy of the font for use on Apples, and was willing to license it. They took the line that all parts of the Microsoft operating system were part and parcel of it, and would not allow me to obtain or use it under any circumstance, even though I was using Microsoft Foxpro for the Mac. >> >> Is it lawyers that make companies anal, or vis versa? >> >> Bob >> >> >> On Jul 26, 2011, at 3:11 PM, J. Landman Gay wrote: >> >>> On 7/26/11 4:35 PM, Scott Rossi wrote: >>>> I personally have been fighting a lot with cross platform font issues and >>>> find it a chore. When using stacks' non-default fonts/sizes, I've found no >>>> other solution than to brute-force set these upon opening stacks. Maybe >>>> someone else knows the secrets for handling this better. >>> I do the same thing and it's a pain. I think this is something that profiles are supposed to be good for, but I haven't actually tried them. >>> >>> -- >>> Jacqueline Landman Gay | jacque at hyperactivesw.com >>> HyperActive Software | http://www.hyperactivesw.com >>> > > _______________________________________________ > use-livecode mailing list > use-livecode at lists.runrev.com > Please visit this url to subscribe, unsubscribe and manage your subscription preferences: > http://lists.runrev.com/mailman/listinfo/use-livecode From bobs at twft.com Wed Jul 27 12:37:41 2011 From: bobs at twft.com (Bob Sneidar) Date: Wed, 27 Jul 2011 09:37:41 -0700 Subject: A case of assigned behavior not taken into account In-Reply-To: <9EC1084F-AC69-492B-A2C6-1658129FF052@wanadoo.fr> References: <9EC1084F-AC69-492B-A2C6-1658129FF052@wanadoo.fr> Message-ID: I will take a shot at this. Behaviors are actually the script of a button, referenced as it's long ID. The long ID (as you could see if you got the long ID of any object) references not just the card it is on but the stack itself. When you clone a stack with behaviors, I suspect that the behaviors are still using the reference to the long ID of the button in the template stack. You will have to change that by script as you suspect. The reason this is like that is because you would want a modified behavior script to affect all object that use it in your entire application. It's just a little bit like OOP for programming. If you wanted the behavior to be altered a bit for certain objects, you could either intercept the message in the object's script, do what is different, then optionally pass it, or you could create a new button which was a copy of the behavior button and assign the object's behavior to that. But obviously you would lose the "one edit fixes all" for that button. Bob On Jul 27, 2011, at 4:41 AM, Andr? Bisseret wrote: > Bonjour, > > On an app. I am developing on Mac, I have a main stack a substack of which is a model used for creating new stacks which are cloned from the model and saved as "independent" stacks (not substacks). > > The scripts of the card 1 of this model and of all objects on this card are all together in a behavior button which is assigned to this card 1. > This behavior is on card 2 of the main stack. > > All is working well on Mac. > > But when I load the standalone for Windows on a PC (by means of a USB key) then a newly created stack from the model is inert. > Meanwhile, I verified that the behavior is actually assigned to the card 1 of the new stack, but all behaves like this was not the case! > > If, by script, I reassign the behavior to card 1 of the model before cloning it, then the new stack is working as expected. > > So I could stay with this reassignment but? > > Is it normal (seems not to me!), or am I missing something ? > > Any hint much appreciated > > Best regards from Grenoble > > Andr? > > > _______________________________________________ > use-livecode mailing list > use-livecode at lists.runrev.com > Please visit this url to subscribe, unsubscribe and manage your subscription preferences: > http://lists.runrev.com/mailman/listinfo/use-livecode From toolbook at kestner.de Wed Jul 27 13:11:33 2011 From: toolbook at kestner.de (Tiemo Hollmann TB) Date: Wed, 27 Jul 2011 19:11:33 +0200 Subject: OT: I want to buy Lion Message-ID: <001801cc4c80$41078b00$c316a100$@de> Probably I should address this to the apple support, but I know here are a lot of kind guys, willing to help also OT much faster as apple. Today I wanted to buy lion to make a dual boot install on my mac, because I had first customer calls telling me about trouble with my program on lion. Because of a slow internet connection I went to the next computer shop and heard them telling that lion won't be available on CD, only download. Ok I returned home, heading several nights of download, opened apple.com, opened the lion announcement and it pointed me first to itunes and then to the macappstore, which couldn't be opened by my safari. Safari tells me that it can't open pages, beginning with "macappstore://". So I begun to read and learned that snow leopard is a requirement to open the macappstore and so to buy lion!??? (I still have leopard on my mac and didn't wanted to upgrade) Trying to buy lion in a non apple online shop, they tell me I have to buy it at apple. Never using my mac, beside of testing my programs, the restrictive apple world is still a big secret for me, even after years. Can anybody shed some light on how I can buy a full version of lion (to create a dual boot) without this silly macappstore? Or do I really have to go again to a computer shop to buy snow leopard, install it to get ready for buying lion? I think I will never love apple. Thanks for any hints Tiemo From dunbarx at aol.com Wed Jul 27 13:14:10 2011 From: dunbarx at aol.com (dunbarx at aol.com) Date: Wed, 27 Jul 2011 13:14:10 -0400 (EDT) Subject: Clicks along a line In-Reply-To: <32728323.2635921311778179991.JavaMail.defaultUser@defaultHost> References: <32728323.2635921311778179991.JavaMail.defaultUser@defaultHost> Message-ID: <8CE1A97BC1C583A-2550-25C60@web-mmc-d04.sysops.aol.com> Wouldn't it be more straightforward to: on RollEm put "40,300" into tCllick-- first point on line click at tClick repeat 5 times --or 4? add 40 to item 1 of tClick click at tClick end repeat end RollEm -----Original Message----- From: barryb To: Use List Revolution Sent: Wed, Jul 27, 2011 6:52 am Subject: Clicks along a line I am trying to make the mouse click at fixed intervals along a straight line. So far, I have the following which 'compiles' without errors: on RollEm click at "40,300" -- first point on line repeat 5 times move the mouseLoc relative "40,0" without waiting click at the mouseLoc end repeat end RollEm Only the first statement works. At line 4 it throws the error: Type move: can't find object Object card id 1002 Line move the mouseLoc relative "40,0" without waiting Hint RollEm How should I script that repeat structure? With many thanks in advance. Barry Barber _______________________________________________ use-livecode mailing list use-livecode at lists.runrev.com Please visit this url to subscribe, unsubscribe and manage your subscription preferences: http://lists.runrev.com/mailman/listinfo/use-livecode From bobs at twft.com Wed Jul 27 13:29:30 2011 From: bobs at twft.com (Bob Sneidar) Date: Wed, 27 Jul 2011 10:29:30 -0700 Subject: Where does survive the inventive user ? In-Reply-To: <8CE1A70D9625EA6-2550-214DE@web-mmc-d04.sysops.aol.com> References: <8CE1A70D9625EA6-2550-214DE@web-mmc-d04.sysops.aol.com> Message-ID: Personally I spend a whole lot of time futzing with little things in Livecode, where I ought to be focused on creating the user interface or writing the code. I will give you an example: I am creating an interface with buttons that have graphic icons. In order to use the icons I have to first import the image into Livecode, note the ID, and then set the icon of the button to that property. Ok so far so good. But I ALSO want the button to look different when the user clicks. Okay, open image in an editor, modify it, save it as something else, import, wash, rinse repeat. Now I want the button to look different when disabled. Ok, blah blah yadda yadda wash rinse repeat. There are six of these I could potentially use. No complaints so far. I get it. Now I see that my 12 buttons are all too big! Hmmm... says I to myself, sizing the button may scale the icon too! Alas, no way. Maybe sizing the icon will work. But damn I hid the images! Okay show hidden objects, (there are 36 and they are cluttering the page) scale image, HEY the button icon scales too. Great! (wash rinse repeat 36 times). Save stack quit LC go have lunch come back open project DAMN! All the icons on my interface have reverted to their original size!!! Spend another hour at the laundry washing rinsing and repeating. Yes I know there are "ways" to do things which can minimize this impact, but the casual user/programmer is not going to know that. There are lots of examples I can give which puts unsuspecting newbies in this quandary. The reason we who remain "put up with it" is because we all know that in the long run, no matter how tedious it can get, LC is still almost infinitely easier to use to develop, debug and distribute cross platform with than C++ or Java. I learned Pascual up to the point of working with a GUI, and staring up at what appeared to me to be an insurmountable cliff (for a hobbyist developer) with the assurance that the next OS that came out would require scaling at least a part of that mountain over again, I said to myself, "No thank you." Hypercard (and now Livecode) have restored my faith that I CAN produce a useful app in a reasonable time and still have a life. That being said, when you subject a "casual user" to the nuances of Livecode, the frustrations can be enough to put them off, maybe forever. My first attempt to make a database kind of app in Revolution involved using the database connection stuff built into the old fields. After days of frustrating inability to get the daggum thing to work right, I posted on this list and got the response, "Yeah, that has never worked very well. You should probably script it." My suggestion to my first example would be having the option to set the icon of a button to a file on the hard drive, and then be able to scale the icon in the button itself, and have that scaling stick. My suggestion to the second example would be to have a real database connection interface in each stack where, once the connection settings to the database were entered and I was connected, I could refer to the data as objects with properties like any other object in Livecode. Put cellData(theRow, theColumn, theTable, theDBConnection) into field "fldFirstname" Put TableData(theColumnList, theRowcount, theStartingRow) into field "fldTableField" Wouldn't THAT be lovely? (Yes Trevor's sqlYoga goes a long way towards this but not quite all the way, great as it is.) I guess I am asking for most of the hard work to be already done for me eh? But isn't that the nature of a Rapid Application Development environment? The question being posed in this thread, seems to me to be, how much of most of the work should RunRev do in order to woo the casual developer? Or have they gone far enough, indeed quite a long way already, and already have the market base they were shooting for, i.e.. us? In other words, it's not a question of principle or implementation, but of degree. Bob On Jul 27, 2011, at 5:35 AM, DunbarX at aol.com wrote: > In the old days, Hypercard was. like a viral pandemic, infected the world because it was bundled with every Macintosh. It was offered like a promotion, a possibly valuable coupon one gets in the mail, which you will at least read before throwing out, And it became a nerd fad, with many hundreds of thousands of people trying it out. Certainly only a small fraction became enamored; many of those are reading this post. > > > Without that once in a lifetime vehicle, it is an uphill battle to engage people who might fall in love with LC if they only were simply exposed to it. Worse, these days, the mindset is that everything comes in small ready-to-go packages, complete and compact. I have three kids who just don't think about building stuff, especially from raw materials. I used to, though. > > > Thank the iMac, iPhone, iPod, etc., for creating that expectation, a far cry from reading a bank of eight lights telling you what byte was currently passing by. > > > LC should be taught in the ninth grade in every school in the world. > > > Craig Newman From martyknapp at comcast.net Wed Jul 27 13:32:20 2011 From: martyknapp at comcast.net (Marty Knapp) Date: Wed, 27 Jul 2011 10:32:20 -0700 Subject: Tahoma Font In-Reply-To: <8F399A93-0D40-4574-926F-0997445DA62E@twft.com> References: <4E2F3BAE.2010606@hyperactivesw.com> <4E2F7093.7020600@comcast.net> <8F399A93-0D40-4574-926F-0997445DA62E@twft.com> Message-ID: <4E304BA4.1090309@comcast.net> The reason I ask is that about 1% of my customers are finding that my app (which uses revFontLoad) does not load the font, which is tucked inside the application bundle. The result returns a not very helpful "unable to load font" message. When the application starts up it looks in the fontnames to see if the font is present. If not and revFontLoad doesn't work, the app copies the font out to disk and instructs them to manually install the font in their user font folder and restart. For some, even this does not work. Though one person said the font worked in their email program but not in my app . . . and sure enough it was. The next step is that I have them clear the font cache by doing a safe boot and then a normal boot, this seems to make things right - with the manually installed font. I haven't experimented with clearing the font cache and then trying revFontLoad. I can't replicate the problem, which makes it hard to trouble shoot. When you mentioned using shell to install fonts, I just wondered if going that route was a more reliable way of installing fonts. Marty K > You just put the font file into /System/Library/Fonts. You have to shell it if the user is not an admin because of permissions. The shell command is simply the one to copy, but you may have to sudo it because it is in the System package. Alternately you could put the font into the user's font folder ~/Library/Fonts (surprisingly this is how Microsoft does it when they run their Office Install the first time you run an Office product in a profile). This way you would not have to sudo or shell because the user (presumably) has write permission to his own library. > > For Windows however, I am not sure how you would go about doing that. Lemme google... AH YES! Here you go. > http://www.sevenforums.com/general-discussion/28817-installing-fonts-via-command-line-script.html > > Bob > > ps. Google is your friend. > > On Jul 26, 2011, at 6:57 PM, Marty Knapp wrote: > >> I've not tinkered with shell commands - what is the shell command for installing fonts on Mac? >> >> Marty K >>> It's been an assumption of mine that if planning for standalone application cross platform support, there are three approaches: >>> >>> 1. ONLY use fonts common to all systems. This is fairly easy with Windows and Apple, but becomes problematic with other flavors. >>> 2. Create an installer that includes the truetype fonts you use, and install them via a shell. This of course may mean licensing the fonts you want to use, unless you choose public domain fonts. >>> 3. Script for the detected operating system and as Scott said, brute force it. This is just ugly, but is probably the most flexible way to go about it. >>> >>> my 2?. Not saying anything new I suppose. I had an issue years ago where Microsoft had a font they used in Windows which Foxpro made much use of, and the app I developed for was built around it. I was porting a Mac version of the app, so I contacted Microsoft to see if I could get a copy of the font for use on Apples, and was willing to license it. They took the line that all parts of the Microsoft operating system were part and parcel of it, and would not allow me to obtain or use it under any circumstance, even though I was using Microsoft Foxpro for the Mac. >>> >>> Is it lawyers that make companies anal, or vis versa? >>> >>> Bob >>> >>> >>> On Jul 26, 2011, at 3:11 PM, J. Landman Gay wrote: >>> >>>> On 7/26/11 4:35 PM, Scott Rossi wrote: >>>>> I personally have been fighting a lot with cross platform font issues and >>>>> find it a chore. When using stacks' non-default fonts/sizes, I've found no >>>>> other solution than to brute-force set these upon opening stacks. Maybe >>>>> someone else knows the secrets for handling this better. >>>> I do the same thing and it's a pain. I think this is something that profiles are supposed to be good for, but I haven't actually tried them. >>>> >>>> -- >>>> Jacqueline Landman Gay | jacque at hyperactivesw.com >>>> HyperActive Software | http://www.hyperactivesw.com From kee at kagi.com Wed Jul 27 13:34:00 2011 From: kee at kagi.com (Kee Nethery) Date: Wed, 27 Jul 2011 10:34:00 -0700 Subject: OT: I want to buy Lion In-Reply-To: <001801cc4c80$41078b00$c316a100$@de> References: <001801cc4c80$41078b00$c316a100$@de> Message-ID: <454B1F26-AC82-47A8-B41B-B090346A2E0C@kagi.com> The Mac App Store is kind of a Snow Leopard thing. Apple decided to distribute Lion initially via the Mac App Store so ... if you do not have Snow Leopard installed, you cannot buy and install it. It is my understanding that there will be a version of Lion that comes on physical media that you will be able to buy from Apple. Once it was done and ready to ship, it took them a couple of minutes to start shipping Lion via the Mac App Store. It will take several weeks to create enough physical media to allow people to buy it in an Apple store. What you are seeing is not that they are making you jump through hoops to buy it. Instead what you are seeing is that under certain circumstances you were able to buy it a couple of minutes after it was declared done and final. In the past everyone would have to wait several weeks for physical distribution. Because you do not have the specific setup for immediate distribution, you'll have to wait for the physical media distribution. Kee Nethery From bobs at twft.com Wed Jul 27 13:34:52 2011 From: bobs at twft.com (Bob Sneidar) Date: Wed, 27 Jul 2011 10:34:52 -0700 Subject: OT: I want to buy Lion In-Reply-To: <001801cc4c80$41078b00$c316a100$@de> References: <001801cc4c80$41078b00$c316a100$@de> Message-ID: <8C7BA5AA-EDE0-4817-BB42-E8C01C7D4FDE@twft.com> Hi Tiemo. You are not alone. A lot of raised eyebrows are only now relaxing after learning of Apple's method of delivery. If you upgrade to Snow Leopard and then do all the updates, you would see in your Apple Menu under Software Updates a new item called App Store. If you went there and set up a new Apple Store account (or used an existing one) you could search for and purchase Lion. Once you did that, Lion would install on your computer. Some people are surmising that soon Apple will sell Lion on a USB stick (or else how are all the Leopard users going to get it)? I tend to believe these rumors simply because it is unreasonable to require Leopard users to upgrade to Snow Leopard before upgrading to Lion (unless they make Snow Leopard upgrade free). Bob On Jul 27, 2011, at 10:11 AM, Tiemo Hollmann TB wrote: > Probably I should address this to the apple support, but I know here are a > lot of kind guys, willing to help also OT much faster as apple. > > Today I wanted to buy lion to make a dual boot install on my mac, because I > had first customer calls telling me about trouble with my program on lion. > > Because of a slow internet connection I went to the next computer shop and > heard them telling that lion won't be available on CD, only download. Ok I > returned home, heading several nights of download, opened apple.com, opened > the lion announcement and it pointed me first to itunes and then to the > macappstore, which couldn't be opened by my safari. Safari tells me that it > can't open pages, beginning with "macappstore://". So I begun to read and > learned that snow leopard is a requirement to open the macappstore and so to > buy lion!??? (I still have leopard on my mac and didn't wanted to upgrade) > Trying to buy lion in a non apple online shop, they tell me I have to buy it > at apple. > > Never using my mac, beside of testing my programs, the restrictive apple > world is still a big secret for me, even after years. Can anybody shed some > light on how I can buy a full version of lion (to create a dual boot) > without this silly macappstore? Or do I really have to go again to a > computer shop to buy snow leopard, install it to get ready for buying lion? > I think I will never love apple. > > Thanks for any hints > > Tiemo > > > > > > _______________________________________________ > use-livecode mailing list > use-livecode at lists.runrev.com > Please visit this url to subscribe, unsubscribe and manage your subscription preferences: > http://lists.runrev.com/mailman/listinfo/use-livecode From bobs at twft.com Wed Jul 27 13:37:29 2011 From: bobs at twft.com (Bob Sneidar) Date: Wed, 27 Jul 2011 10:37:29 -0700 Subject: OT: I want to buy Lion In-Reply-To: <001801cc4c80$41078b00$c316a100$@de> References: <001801cc4c80$41078b00$c316a100$@de> Message-ID: Oh, by the way I hope you mean Dual Partition. Dual boot *can* mean to install two operating systems on one partition (you can actually do this with Windows but no one recommends it). You will want to install Lion on a second partition or hard drive, because Lion is going to overwrite your existing OS. It may also do some nonsense with the boot sector. Bob On Jul 27, 2011, at 10:11 AM, Tiemo Hollmann TB wrote: > Probably I should address this to the apple support, but I know here are a > lot of kind guys, willing to help also OT much faster as apple. > > Today I wanted to buy lion to make a dual boot install on my mac, because I > had first customer calls telling me about trouble with my program on lion. > > Because of a slow internet connection I went to the next computer shop and > heard them telling that lion won't be available on CD, only download. Ok I > returned home, heading several nights of download, opened apple.com, opened > the lion announcement and it pointed me first to itunes and then to the > macappstore, which couldn't be opened by my safari. Safari tells me that it > can't open pages, beginning with "macappstore://". So I begun to read and > learned that snow leopard is a requirement to open the macappstore and so to > buy lion!??? (I still have leopard on my mac and didn't wanted to upgrade) > Trying to buy lion in a non apple online shop, they tell me I have to buy it > at apple. > > Never using my mac, beside of testing my programs, the restrictive apple > world is still a big secret for me, even after years. Can anybody shed some > light on how I can buy a full version of lion (to create a dual boot) > without this silly macappstore? Or do I really have to go again to a > computer shop to buy snow leopard, install it to get ready for buying lion? > I think I will never love apple. > > Thanks for any hints > > Tiemo > > > > > > _______________________________________________ > use-livecode mailing list > use-livecode at lists.runrev.com > Please visit this url to subscribe, unsubscribe and manage your subscription preferences: > http://lists.runrev.com/mailman/listinfo/use-livecode From admin at FlexibleLearning.com Wed Jul 27 13:44:15 2011 From: admin at FlexibleLearning.com (FlexibleLearning) Date: Wed, 27 Jul 2011 18:44:15 +0100 Subject: AW: Tahoma Font In-Reply-To: Message-ID: I use Tahoma on Macs and Win and have never had a problem. Tends to be the default unless missing; failing Tahoma I use Verdana 11pt. Hugh Senior FLCo Tiemo Hollmann wrote: When I started with revolution Tahoma was the default font of all objects and I didn't cared about it at that time. I am now using Tahoma since years without any problems on windows and Mac, though I have learned that Tahoma isn't a standard font on Mac and is supposed to be substituted (what never made any problems in my apps). On windows systems Tahoma is a standard font up to today as far as I know, or did I miss anything? Tiemo > -----Urspr?ngliche Nachricht----- > Von: use-livecode-bounces at lists.runrev.com [mailto:use-livecode- > bounces at lists.runrev.com] Im Auftrag von Camm > Gesendet: Dienstag, 26. Juli 2011 22:30 > An: use-livecode at lists.runrev.com > Betreff: Tahoma Font > > Tahoma font will not work in standalone unless its run in Windows 95 or 98 > compatibility mode? From Camm29 at tesco.net Wed Jul 27 13:50:35 2011 From: Camm29 at tesco.net (Camm) Date: Wed, 27 Jul 2011 18:50:35 +0100 Subject: Tahoma Font In-Reply-To: <000f01cc4c3c$3b747b60$b25d7220$@de> References: <000001cc4bd2$cd1e4470$675acd50$@net> <000f01cc4c3c$3b747b60$b25d7220$@de> Message-ID: <000c01cc4c85$b0b7ab10$12270130$@net> Well , my issue is just with windows ? The Tahoma font is their during development mode but vanishes in Standalone on XP. If I set the properties of the standalone.exe in Windows to 95 or 98 compatibility it returns ? How do I force load a font on startup ? Best Regards Camm -----Original Message----- From: use-livecode-bounces at lists.runrev.com [mailto:use-livecode-bounces at lists.runrev.com] On Behalf Of Tiemo Hollmann TB Sent: 27 July 2011 10:05 To: 'How to use LiveCode' Subject: AW: Tahoma Font When I started with revolution Tahoma was the default font of all objects and I didn't cared about it at that time. I am now using Tahoma since years without any problems on windows and Mac, though I have learned that Tahoma isn't a standard font on Mac and is supposed to be substituted (what never made any problems in my apps). On windows systems Tahoma is a standard font up to today as far as I know, or did I miss anything? Tiemo > -----Urspr?ngliche Nachricht----- > Von: use-livecode-bounces at lists.runrev.com [mailto:use-livecode- > bounces at lists.runrev.com] Im Auftrag von Camm > Gesendet: Dienstag, 26. Juli 2011 22:30 > An: use-livecode at lists.runrev.com > Betreff: Tahoma Font > > Tahoma font will not work in standalone unless its run in Windows 95 or 98 > compatibility mode? > > > > I am using Windows XP...... Any ideas ? > > > > Best Regards > > Camm > > _______________________________________________ > use-livecode mailing list > use-livecode at lists.runrev.com > Please visit this url to subscribe, unsubscribe and manage your subscription > preferences: > http://lists.runrev.com/mailman/listinfo/use-livecode _______________________________________________ use-livecode mailing list use-livecode at lists.runrev.com Please visit this url to subscribe, unsubscribe and manage your subscription preferences: http://lists.runrev.com/mailman/listinfo/use-livecode ----- No virus found in this message. Checked by AVG - www.avg.com Version: 10.0.1390 / Virus Database: 1518/3789 - Release Date: 07/26/11 From barryb at libero.it Wed Jul 27 13:51:56 2011 From: barryb at libero.it (barryb at libero.it) Date: Wed, 27 Jul 2011 19:51:56 +0200 (CEST) Subject: Clicks along a line Message-ID: <807182.2723101311789116268.JavaMail.defaultUser@defaultHost> >Barry, >Why exactly does it click on your desktop? If you re-calculate the locations, it should click on the card. There's a function globalLoc(). Damned if I know, but I saw the mouse icon do it to the left of my stack and when it's path passed over the Rev toolpalette the relative objs on the card reacted to the change in colour, as per my script! That was the nearest I've ever got to what I want! (FWIW -- Using Rev 3.5 on MS Vista) -- >Best regards, >Mark Schonewille Grateful and puzzled Kind regards Barry Barber From toolbook at kestner.de Wed Jul 27 14:00:18 2011 From: toolbook at kestner.de (Tiemo Hollmann TB) Date: Wed, 27 Jul 2011 20:00:18 +0200 Subject: AW: OT: I want to buy Lion In-Reply-To: <454B1F26-AC82-47A8-B41B-B090346A2E0C@kagi.com> References: <001801cc4c80$41078b00$c316a100$@de> <454B1F26-AC82-47A8-B41B-B090346A2E0C@kagi.com> Message-ID: <002001cc4c87$1120f410$3362dc30$@de> Hi Kee, thanks for your apple friendly explanation :) I will try to see it this way and will explain it to my customers that they will have to wait for my support until apple is ready to ship it on CD/stick. Thanks Tiemo > -----Urspr?ngliche Nachricht----- > Von: use-livecode-bounces at lists.runrev.com [mailto:use-livecode- > bounces at lists.runrev.com] Im Auftrag von Kee Nethery > Gesendet: Mittwoch, 27. Juli 2011 19:34 > An: How to use LiveCode > Betreff: Re: OT: I want to buy Lion > > The Mac App Store is kind of a Snow Leopard thing. Apple decided to distribute > Lion initially via the Mac App Store so ... if you do not have Snow Leopard > installed, you cannot buy and install it. > > It is my understanding that there will be a version of Lion that comes on > physical media that you will be able to buy from Apple. Once it was done and > ready to ship, it took them a couple of minutes to start shipping Lion via the > Mac App Store. It will take several weeks to create enough physical media to > allow people to buy it in an Apple store. > > What you are seeing is not that they are making you jump through hoops to buy > it. Instead what you are seeing is that under certain circumstances you were > able to buy it a couple of minutes after it was declared done and final. In > the past everyone would have to wait several weeks for physical distribution. > > Because you do not have the specific setup for immediate distribution, you'll > have to wait for the physical media distribution. > > Kee Nethery > _______________________________________________ > use-livecode mailing list > use-livecode at lists.runrev.com > Please visit this url to subscribe, unsubscribe and manage your subscription > preferences: > http://lists.runrev.com/mailman/listinfo/use-livecode From toolbook at kestner.de Wed Jul 27 14:02:05 2011 From: toolbook at kestner.de (Tiemo Hollmann TB) Date: Wed, 27 Jul 2011 20:02:05 +0200 Subject: AW: OT: I want to buy Lion In-Reply-To: <8C7BA5AA-EDE0-4817-BB42-E8C01C7D4FDE@twft.com> References: <001801cc4c80$41078b00$c316a100$@de> <8C7BA5AA-EDE0-4817-BB42-E8C01C7D4FDE@twft.com> Message-ID: <002101cc4c87$4f71dd60$ee559820$@de> Hi Bob, thank you for showing me that I am not the only one! That helps (at least for my mind). And yes I meant dual partition! Thanks for reminding me Tiemo > -----Urspr?ngliche Nachricht----- > Von: use-livecode-bounces at lists.runrev.com [mailto:use-livecode- > bounces at lists.runrev.com] Im Auftrag von Bob Sneidar > Gesendet: Mittwoch, 27. Juli 2011 19:35 > An: How to use LiveCode > Betreff: Re: OT: I want to buy Lion > > Hi Tiemo. You are not alone. A lot of raised eyebrows are only now relaxing > after learning of Apple's method of delivery. > > If you upgrade to Snow Leopard and then do all the updates, you would see in > your Apple Menu under Software Updates a new item called App Store. If you > went there and set up a new Apple Store account (or used an existing one) you > could search for and purchase Lion. Once you did that, Lion would install on > your computer. > > Some people are surmising that soon Apple will sell Lion on a USB stick (or > else how are all the Leopard users going to get it)? I tend to believe these > rumors simply because it is unreasonable to require Leopard users to upgrade > to Snow Leopard before upgrading to Lion (unless they make Snow Leopard > upgrade free). > > Bob > > > On Jul 27, 2011, at 10:11 AM, Tiemo Hollmann TB wrote: > > > Probably I should address this to the apple support, but I know here are a > > lot of kind guys, willing to help also OT much faster as apple. > > > > Today I wanted to buy lion to make a dual boot install on my mac, because I > > had first customer calls telling me about trouble with my program on lion. > > > > Because of a slow internet connection I went to the next computer shop and > > heard them telling that lion won't be available on CD, only download. Ok I > > returned home, heading several nights of download, opened apple.com, opened > > the lion announcement and it pointed me first to itunes and then to the > > macappstore, which couldn't be opened by my safari. Safari tells me that it > > can't open pages, beginning with "macappstore://". So I begun to read and > > learned that snow leopard is a requirement to open the macappstore and so to > > buy lion!??? (I still have leopard on my mac and didn't wanted to upgrade) > > Trying to buy lion in a non apple online shop, they tell me I have to buy it > > at apple. > > > > Never using my mac, beside of testing my programs, the restrictive apple > > world is still a big secret for me, even after years. Can anybody shed some > > light on how I can buy a full version of lion (to create a dual boot) > > without this silly macappstore? Or do I really have to go again to a > > computer shop to buy snow leopard, install it to get ready for buying lion? > > I think I will never love apple. > > > > Thanks for any hints > > > > Tiemo > > > > > > > > > > > > _______________________________________________ > > use-livecode mailing list > > use-livecode at lists.runrev.com > > Please visit this url to subscribe, unsubscribe and manage your subscription > preferences: > > http://lists.runrev.com/mailman/listinfo/use-livecode > > > _______________________________________________ > use-livecode mailing list > use-livecode at lists.runrev.com > Please visit this url to subscribe, unsubscribe and manage your subscription > preferences: > http://lists.runrev.com/mailman/listinfo/use-livecode From bobs at twft.com Wed Jul 27 13:58:51 2011 From: bobs at twft.com (Bob Sneidar) Date: Wed, 27 Jul 2011 10:58:51 -0700 Subject: Tahoma Font In-Reply-To: <4E304BA4.1090309@comcast.net> References: <4E2F3BAE.2010606@hyperactivesw.com> <4E2F7093.7020600@comcast.net> <8F399A93-0D40-4574-926F-0997445DA62E@twft.com> <4E304BA4.1090309@comcast.net> Message-ID: <6DD225CA-36DB-491C-A605-0E4BD4BD4DA7@twft.com> IC. I mentioned the shell because if the end user is not an admin, and you want the font installed universally, then the only way I know to do that is to use sudo in a terminal for Mac (and presumably unix/linux), and Run As in a command line for Windows. It sure sounds like a font conflict. The name may not conflict, but if ID's do, you can have problems. I have known Apple systems to lock up or apps to unexpectedly quit when a corrupt font is accessed. Aside from having them troubleshoot their own fonts with a utility, I am not sure there is much you can do. Bob On Jul 27, 2011, at 10:32 AM, Marty Knapp wrote: > The reason I ask is that about 1% of my customers are finding that my app (which uses revFontLoad) does not load the font, which is tucked inside the application bundle. The result returns a not very helpful "unable to load font" message. When the application starts up it looks in the fontnames to see if the font is present. If not and revFontLoad doesn't work, the app copies the font out to disk and instructs them to manually install the font in their user font folder and restart. For some, even this does not work. Though one person said the font worked in their email program but not in my app . . . and sure enough it was. > > The next step is that I have them clear the font cache by doing a safe boot and then a normal boot, this seems to make things right - with the manually installed font. I haven't experimented with clearing the font cache and then trying revFontLoad. I can't replicate the problem, which makes it hard to trouble shoot. When you mentioned using shell to install fonts, I just wondered if going that route was a more reliable way of installing fonts. > > Marty K From bobs at twft.com Wed Jul 27 14:03:41 2011 From: bobs at twft.com (Bob Sneidar) Date: Wed, 27 Jul 2011 11:03:41 -0700 Subject: Tahoma Font In-Reply-To: <000c01cc4c85$b0b7ab10$12270130$@net> References: <000001cc4bd2$cd1e4470$675acd50$@net> <000f01cc4c3c$3b747b60$b25d7220$@de> <000c01cc4c85$b0b7ab10$12270130$@net> Message-ID: <5312DC63-3162-4FD0-B853-D84235CE9C0D@twft.com> This may help: http://support.microsoft.com/kb/131943 Bob On Jul 27, 2011, at 10:50 AM, Camm wrote: > Well , my issue is just with windows ? > The Tahoma font is their during development mode but vanishes in Standalone > on XP. > If I set the properties of the standalone.exe in Windows to 95 or 98 > compatibility it returns ? > > How do I force load a font on startup ? > > Best Regards > Camm > > > -----Original Message----- > From: use-livecode-bounces at lists.runrev.com > [mailto:use-livecode-bounces at lists.runrev.com] On Behalf Of Tiemo Hollmann > TB > Sent: 27 July 2011 10:05 > To: 'How to use LiveCode' > Subject: AW: Tahoma Font > > When I started with revolution Tahoma was the default font of all objects > and I didn't cared about it at that time. I am now using Tahoma since years > without any problems on windows and Mac, though I have learned that Tahoma > isn't a standard font on Mac and is supposed to be substituted (what never > made any problems in my apps). On windows systems Tahoma is a standard font > up to today as far as I know, or did I miss anything? > Tiemo > >> -----Urspr?ngliche Nachricht----- >> Von: use-livecode-bounces at lists.runrev.com [mailto:use-livecode- >> bounces at lists.runrev.com] Im Auftrag von Camm >> Gesendet: Dienstag, 26. Juli 2011 22:30 >> An: use-livecode at lists.runrev.com >> Betreff: Tahoma Font >> >> Tahoma font will not work in standalone unless its run in Windows 95 or 98 >> compatibility mode? >> >> >> >> I am using Windows XP...... Any ideas ? >> >> >> >> Best Regards >> >> Camm >> >> _______________________________________________ >> use-livecode mailing list >> use-livecode at lists.runrev.com >> Please visit this url to subscribe, unsubscribe and manage your > subscription >> preferences: >> http://lists.runrev.com/mailman/listinfo/use-livecode > > > _______________________________________________ > use-livecode mailing list > use-livecode at lists.runrev.com > Please visit this url to subscribe, unsubscribe and manage your subscription > preferences: > http://lists.runrev.com/mailman/listinfo/use-livecode > ----- > No virus found in this message. > Checked by AVG - www.avg.com > Version: 10.0.1390 / Virus Database: 1518/3789 - Release Date: 07/26/11 > > > _______________________________________________ > use-livecode mailing list > use-livecode at lists.runrev.com > Please visit this url to subscribe, unsubscribe and manage your subscription preferences: > http://lists.runrev.com/mailman/listinfo/use-livecode From arietext at mac.com Wed Jul 27 14:20:05 2011 From: arietext at mac.com (Arie van der Ent) Date: Wed, 27 Jul 2011 20:20:05 +0200 Subject: OT: I want to buy Lion In-Reply-To: <002101cc4c87$4f71dd60$ee559820$@de> References: <001801cc4c80$41078b00$c316a100$@de> <8C7BA5AA-EDE0-4817-BB42-E8C01C7D4FDE@twft.com> <002101cc4c87$4f71dd60$ee559820$@de> Message-ID: <9308DB19-829D-41A0-A887-B2F59C1C4CF4@mac.com> Hi Tiemo, It is not necessary to wait for Lion on a USB-stick. Don McAllister from screencastsonline did the following; 1. buy Lion from App store; 2. make a copy from the installer (in your application folder); 3. open up the copy of the installer package, then open the folder Shared Support in it; 4. there you'll find InstallEDG.dmg; 5. copy this file to an USB-stick 8 gb. That's all. Arie Op 27 jul. 2011, om 20:02 heeft Tiemo Hollmann TB het volgende geschreven: > Hi Bob, > > thank you for showing me that I am not the only one! That helps (at least > for my mind). > And yes I meant dual partition! Thanks for reminding me > Tiemo > >> -----Urspr?ngliche Nachricht----- >> Von: use-livecode-bounces at lists.runrev.com [mailto:use-livecode- >> bounces at lists.runrev.com] Im Auftrag von Bob Sneidar >> Gesendet: Mittwoch, 27. Juli 2011 19:35 >> An: How to use LiveCode >> Betreff: Re: OT: I want to buy Lion >> >> Hi Tiemo. You are not alone. A lot of raised eyebrows are only now > relaxing >> after learning of Apple's method of delivery. >> >> If you upgrade to Snow Leopard and then do all the updates, you would see > in >> your Apple Menu under Software Updates a new item called App Store. If you >> went there and set up a new Apple Store account (or used an existing one) > you >> could search for and purchase Lion. Once you did that, Lion would install > on >> your computer. >> >> Some people are surmising that soon Apple will sell Lion on a USB stick > (or >> else how are all the Leopard users going to get it)? I tend to believe > these >> rumors simply because it is unreasonable to require Leopard users to > upgrade >> to Snow Leopard before upgrading to Lion (unless they make Snow Leopard >> upgrade free). >> >> Bob >> >> >> On Jul 27, 2011, at 10:11 AM, Tiemo Hollmann TB wrote: >> >>> Probably I should address this to the apple support, but I know here are > a >>> lot of kind guys, willing to help also OT much faster as apple. >>> >>> Today I wanted to buy lion to make a dual boot install on my mac, > because I >>> had first customer calls telling me about trouble with my program on > lion. >>> >>> Because of a slow internet connection I went to the next computer shop > and >>> heard them telling that lion won't be available on CD, only download. Ok > I >>> returned home, heading several nights of download, opened apple.com, > opened >>> the lion announcement and it pointed me first to itunes and then to the >>> macappstore, which couldn't be opened by my safari. Safari tells me that > it >>> can't open pages, beginning with "macappstore://". So I begun to read > and >>> learned that snow leopard is a requirement to open the macappstore and > so to >>> buy lion!??? (I still have leopard on my mac and didn't wanted to > upgrade) >>> Trying to buy lion in a non apple online shop, they tell me I have to > buy it >>> at apple. >>> >>> Never using my mac, beside of testing my programs, the restrictive apple >>> world is still a big secret for me, even after years. Can anybody shed > some >>> light on how I can buy a full version of lion (to create a dual boot) >>> without this silly macappstore? Or do I really have to go again to a >>> computer shop to buy snow leopard, install it to get ready for buying > lion? >>> I think I will never love apple. >>> >>> Thanks for any hints >>> >>> Tiemo >>> >>> >>> >>> >>> >>> _______________________________________________ >>> use-livecode mailing list >>> use-livecode at lists.runrev.com >>> Please visit this url to subscribe, unsubscribe and manage your > subscription >> preferences: >>> http://lists.runrev.com/mailman/listinfo/use-livecode >> >> >> _______________________________________________ >> use-livecode mailing list >> use-livecode at lists.runrev.com >> Please visit this url to subscribe, unsubscribe and manage your > subscription >> preferences: >> http://lists.runrev.com/mailman/listinfo/use-livecode > > > _______________________________________________ > use-livecode mailing list > use-livecode at lists.runrev.com > Please visit this url to subscribe, unsubscribe and manage your subscription preferences: > http://lists.runrev.com/mailman/listinfo/use-livecode From richmondmathewson at gmail.com Wed Jul 27 14:25:49 2011 From: richmondmathewson at gmail.com (Richmond Mathewson) Date: Wed, 27 Jul 2011 21:25:49 +0300 Subject: Creative Inventors Message-ID: <4E30582D.8000709@gmail.com> Well, I'm one, and I have been being desperately creative over many years; both for my in-house software for my language school and for my Devawriter. -------------------------------------------------------------------------------- My in-house stuff works, insofar as kids seem to learn both English and cognitive skills from it. Frankly, if my school did not have 4 PCs running Linux with those standalones, while the children would have a more boring time, the parents who pay wouldn't give a damn. -------------------------------------------------------------------------------- Devawriter is most instructive; it is so bl**dy creative, so bl**dy inventive that people react against it; it is too clever by half; people would rather continue working with the half-cock methods of doing what Devawriter does far better because . . . . . . well, because people stick with Windows XP here in Bulgaria, rather than moving over to Linux, even though their systems are virused to the hilt, crash like mad, and so on and so forth . . . So far I have made the princely sum of $10 from my Devawriter: it sits there, mocking me, in my PayPal account. ---------------------------------------------------------------------------------- So, probably better NOT to be very creative, or very inventive, but churn out rather more pedestrian stuff . . . :) From bonnmike at gmail.com Wed Jul 27 14:54:46 2011 From: bonnmike at gmail.com (Mike Bonner) Date: Wed, 27 Jul 2011 12:54:46 -0600 Subject: Clicks along a line In-Reply-To: <807182.2723101311789116268.JavaMail.defaultUser@defaultHost> References: <807182.2723101311789116268.JavaMail.defaultUser@defaultHost> Message-ID: As mentioned, the globalloc() function takes a relative location and turns it into a screen relative numbers. localloc() takes a screen relative location and converts it to card relative. So if you want to move the mouse to a location in your stack then have it click, you can first set the screenmouseloc to globalloc(40,300) which will move it to your first click location. Since screenmouseloc is screen based your numbers have to be based on that, which is why the globalloc() is necessary to convert your card relative 40,300 location. At this point, since the mouse is where you want it you can 'click at the mouseloc' Then modify your card relative number, again feed it to globalloc() and proceed. Something like this (untested) will probably work. on mouseUp put globalLoc("40,300") into tMouseLoc repeat 6 times set the screenMouseLoc to tMouseLoc click at the mouseLoc add 40 to item 1 of tMouseLoc wait 500 milliseconds with messages -- I just put that in to watch the mouse pointer move end repeat end mouseUp If you just want to have the clicks done, you don't need to set the screenmouseloc necessarily, you can just 'click at tMouseLoc I think. Will have to test to make sure On Wed, Jul 27, 2011 at 11:51 AM, barryb at libero.it wrote: > >Barry, > > >Why exactly does it click on your desktop? If you re-calculate the > locations, it should click on the card. There's a function globalLoc(). > > > Damned if I know, but I saw the mouse icon do it to the left of my stack > and when it's path passed over the Rev toolpalette the relative objs on the > card reacted to the change in colour, as per my script! That was the nearest > I've ever got to what I want! (FWIW -- Using Rev 3.5 on MS Vista) > -- > >Best regards, > >Mark Schonewille > > Grateful and puzzled > Kind regards > Barry Barber > > > _______________________________________________ > use-livecode mailing list > use-livecode at lists.runrev.com > Please visit this url to subscribe, unsubscribe and manage your > subscription preferences: > http://lists.runrev.com/mailman/listinfo/use-livecode > From richmondmathewson at gmail.com Wed Jul 27 15:04:09 2011 From: richmondmathewson at gmail.com (Richmond Mathewson) Date: Wed, 27 Jul 2011 22:04:09 +0300 Subject: 30 day trial Message-ID: <4E306129.7060305@gmail.com> Oddly enough when I enter my email and password into a Livcode 4.6.3 install it does not allow me a 30 day trial: it prompts me to set up an account, which I already have. From m.schonewille at economy-x-talk.com Wed Jul 27 15:08:29 2011 From: m.schonewille at economy-x-talk.com (Mark Schonewille) Date: Wed, 27 Jul 2011 21:08:29 +0200 Subject: 30 day trial In-Reply-To: <4E306129.7060305@gmail.com> References: <4E306129.7060305@gmail.com> Message-ID: <6BCD4FEE-B3DF-4FD3-A805-F536344EDF3B@economy-x-talk.com> Yeah, that whole trial thing is annoying. Whenever I want to test the latest version, I have to e-mail support. That's only additional work for support and for me. -- Best regards, Mark Schonewille Economy-x-Talk Consulting and Software Engineering Homepage: http://economy-x-talk.com Twitter: http://twitter.com/xtalkprogrammer KvK: 50277553 What does that error mean? Buy LiveCodeErrors for iPhone now http://qery.us/v4 A must-have for LiveCode programmers. On 27 jul 2011, at 21:04, Richmond Mathewson wrote: > Oddly enough when I enter my email and password into a Livcode 4.6.3 install it does not allow me a 30 day trial: it prompts me to set up an account, which I already have. From richmondmathewson at gmail.com Wed Jul 27 15:11:03 2011 From: richmondmathewson at gmail.com (Richmond Mathewson) Date: Wed, 27 Jul 2011 22:11:03 +0300 Subject: 30 day trial In-Reply-To: <6BCD4FEE-B3DF-4FD3-A805-F536344EDF3B@economy-x-talk.com> References: <4E306129.7060305@gmail.com> <6BCD4FEE-B3DF-4FD3-A805-F536344EDF3B@economy-x-talk.com> Message-ID: <4E3062C7.3070503@gmail.com> On 07/27/2011 10:08 PM, Mark Schonewille wrote: > Yeah, that whole trial thing is annoying. Whenever I want to test the latest version, I have to e-mail support. That's only additional work for support and for me. Thanks for the info. > -- > Best regards, > > Mark Schonewille > > Economy-x-Talk Consulting and Software Engineering > Homepage: http://economy-x-talk.com > Twitter: http://twitter.com/xtalkprogrammer > KvK: 50277553 > > What does that error mean? Buy LiveCodeErrors for iPhone now http://qery.us/v4 A must-have for LiveCode programmers. > > On 27 jul 2011, at 21:04, Richmond Mathewson wrote: > >> Oddly enough when I enter my email and password into a Livcode 4.6.3 install it does not allow me a 30 day trial: it prompts me to set up an account, which I already have. > > > _______________________________________________ > use-livecode mailing list > use-livecode at lists.runrev.com > Please visit this url to subscribe, unsubscribe and manage your subscription preferences: > http://lists.runrev.com/mailman/listinfo/use-livecode From warren at warrensweb.us Wed Jul 27 15:14:36 2011 From: warren at warrensweb.us (Warren Samples) Date: Wed, 27 Jul 2011 14:14:36 -0500 Subject: OT: I want to buy Lion In-Reply-To: <9308DB19-829D-41A0-A887-B2F59C1C4CF4@mac.com> References: <001801cc4c80$41078b00$c316a100$@de> <002101cc4c87$4f71dd60$ee559820$@de> <9308DB19-829D-41A0-A887-B2F59C1C4CF4@mac.com> Message-ID: <201107271414.36804.warren@warrensweb.us> On Wednesday, July 27, 2011 01:20:05 PM Arie van der Ent wrote: > Hi Tiemo, > > It is not necessary to wait for Lion on a USB-stick. Don McAllister from > screencastsonline did the following; 1. buy Lion from App store; > 2. make a copy from the installer (in your application folder); > 3. open up the copy of the installer package, then open the folder Shared > Support in it; 4. there you'll find InstallEDG.dmg; > 5. copy this file to an USB-stick 8 gb. > > That's all. > > Arie Arie, He's explained why he can't buy 10.7 at the moment. Unless you're silently implying Tiemo enlist the aid of someone who had purchased it and is willing to share, this is not useful information. There seems to be conflicting information regarding a clean install of 10.7 with some saying they've done it and others saying there's got to be 10.6.8 avaible at one point or another to run the installer. It does not seem likely that Apple will ignore forever those users who need or would simply prefer to be able to purchase physical install media of some sort or another, though. I wonder if it's possible to clone a drive and just install that drive in another machine. Best, Warren From richmondmathewson at gmail.com Wed Jul 27 15:14:54 2011 From: richmondmathewson at gmail.com (Richmond Mathewson) Date: Wed, 27 Jul 2011 22:14:54 +0300 Subject: 30 day trial In-Reply-To: <6BCD4FEE-B3DF-4FD3-A805-F536344EDF3B@economy-x-talk.com> References: <4E306129.7060305@gmail.com> <6BCD4FEE-B3DF-4FD3-A805-F536344EDF3B@economy-x-talk.com> Message-ID: <4E3063AE.9040908@gmail.com> On 07/27/2011 10:08 PM, Mark Schonewille wrote: > Yeah, that whole trial thing is annoying. Whenever I want to test the latest version, I have to e-mail support. That's only additional work for support and for me. > > Wrote to support and have just got an automated response that finishes: "Warm Regards The RunRev Robot" Calculated to irritate methinks. From dwares at mac.com Wed Jul 27 15:48:17 2011 From: dwares at mac.com (Dave Wares) Date: Wed, 27 Jul 2011 14:48:17 -0500 Subject: Sending Text Messages Message-ID: <7D97AF38-30DC-4EE2-91C4-DEE3314E8C35@mac.com> Is it possible to send a text message directly from LiveCode, even if the user still needs to hit the "send" button? From m.schonewille at economy-x-talk.com Wed Jul 27 15:52:38 2011 From: m.schonewille at economy-x-talk.com (Mark Schonewille) Date: Wed, 27 Jul 2011 21:52:38 +0200 Subject: Sending Text Messages In-Reply-To: <7D97AF38-30DC-4EE2-91C4-DEE3314E8C35@mac.com> References: <7D97AF38-30DC-4EE2-91C4-DEE3314E8C35@mac.com> Message-ID: <2FF2DA76-933D-4D8A-889D-86B0E8B28F81@economy-x-talk.com> To where? -- Best regards, Mark Schonewille Economy-x-Talk Consulting and Software Engineering Homepage: http://economy-x-talk.com Twitter: http://twitter.com/xtalkprogrammer KvK: 50277553 What does that error mean? Buy LiveCodeErrors for iPhone now http://qery.us/v4 A must-have for LiveCode programmers. On 27 jul 2011, at 21:48, Dave Wares wrote: > Is it possible to send a text message directly from LiveCode, even if the user still needs to hit the "send" button? From bonnmike at gmail.com Wed Jul 27 15:56:12 2011 From: bonnmike at gmail.com (Mike Bonner) Date: Wed, 27 Jul 2011 13:56:12 -0600 Subject: windows revserver stack support In-Reply-To: References: Message-ID: Got it to work. If there is a button in the stack it causes a sig 11. remove them and things start to work dandy. On Wed, Jul 27, 2011 at 9:04 AM, Mike Bonner wrote: > More information. On a different machine, *ubuntu 1, apache2, > livecode-server 4-6-3* there is also a signal 11, but quite a bit more info > (that I don't understand.) > > > livecode-server exited due to fatal signal 11 > in __kernel_sigreturn @ 0 > in @ 0x8158f85 > in @ 0x809e596 > in @ 0x809ea13 > in @ 0x809eccc > in @ 0x809177d > in @ 0x8091fed > in @ 0x81cd09e > in @ 0x81c96d3 > in @ 0x81c9b40 > in __libc_start_main @ 231 > in fwrite @ 305 > > I'm going to re-download the livecode server distributions on the off > chance theres been a sneaky fix or update that I haven't seen, but at this > point i'm without clue. Which is of course my usual state of being! *grin* > > Will send a note to support in a day or so if nobody can point me to > whatever mistake I made. > > Thanks in advance for any help > > > On Wed, Jul 27, 2011 at 8:34 AM, Mike Bonner wrote: > >> Has anyone managed to get a stack to load using livecode-server for >> windows? All I get is "livecode-server exited due to fatal signal 11" >> >> This happens when trying to library the stack as well as using go stack >> >> >> Otherwise livecode-server is working great, just can't get the stack >> support to work. >> >> I've even checked it from the command line to make sure that I don't have >> some strange apache misconfiguration going on, but the result is the same. >> >> >> Any suggestions? >> > > From stephenREVOLUTION2 at barncard.com Wed Jul 27 16:04:27 2011 From: stephenREVOLUTION2 at barncard.com (stephen barncard) Date: Wed, 27 Jul 2011 13:04:27 -0700 Subject: windows revserver stack support In-Reply-To: References: Message-ID: yes you can have multiple cards and as many custom props as you like but no other objects like fields or buttons. Fail for sure. It's strictly an intelligent storage device. On 27 July 2011 12:56, Mike Bonner wrote: > Got it to work. If there is a button in the stack it causes a sig 11. > remove > them and things start to work dandy. > > Stephen Barncard San Francisco Ca. USA more about sqb From stephenREVOLUTION2 at barncard.com Wed Jul 27 16:23:41 2011 From: stephenREVOLUTION2 at barncard.com (stephen barncard) Date: Wed, 27 Jul 2011 13:23:41 -0700 Subject: Livecode Server 4.6.3 - First impressions pt1 - Getting it working at Dreamhost Message-ID: After a bit of trial and error - many errors, Andre Garzia helped me get my new Livecode server running at Dreamhost*. Twice. I hope I am not 'stealing his thunder' here because I owe him a great deal of gratitude for getting my sites working with LCS.( I know he wants to talk about it - when this very busy gentleman gets a second of time I am sure he will have plenty of informative things to tell us.) First tried the ".htaccess" method on my shared account. Couldn't make it work, just displayed the binary of the executable into the browser. Garbage. Andre tried too. so then I got a VPS account with access to "root" - well sort of. Andre made some changes in the Apache config file which was now accessible (as per the instructions) got LCS working everywhere in the server with one installation. We were all celebrating when I tried to bring over more domains using the control panel. The site move trounced the Apache config file. hey, this was just too delicate to be useful. Besides - I really don't need LC on all my domains. Sheepishly, I cried "ANDRE" !! Andre went back in and finally came up with simple changes to the suggested ".htaccess" file and it works! working file structure: (site root) .htaccess (htaccess1 file at site root) *(cgi-bin)* (directory) .htaccess (htaccess2 file inside cgi-bin) *(livecode-server)* livecode-server (application binary - set to 755!) *(externals)* *(drivers)* .... other site files script of .htaccess #1 Options ExecCGI FollowSymLinks AddHandler livecode-script .lc .irev DirectoryIndex index.irev index.lc index.php index.html Action livecode-script /cgi-bin/livecode-server/livecode-server script of .htaccess #2 Options ExecCGI SetHandler cgi-script Wonderfulness! Livecode running on all my own sites at last. *next: actually using the new version 4.6.3 - and an editor surprise.* * disclaimer - On-Rev is a great service and most people would be happy just hosting there and be done with it, rather than hosting elsewhere and setting up their own server. Half my sites being at Dreamhost (and many email addresses and namesake domain) and a long history with the Dreamhost had a lot to do with me trying to do this. I am sort of a control freak. But I never found anything objectionable in ON-Rev's quality of service. -- -- Stephen Barncard San Francisco Ca. USA more about sqb From cszasz at mac.com Wed Jul 27 17:07:37 2011 From: cszasz at mac.com (Charles Szasz) Date: Wed, 27 Jul 2011 17:07:37 -0400 Subject: Mac Apps made with Rev run on Lion? Message-ID: <3B16BE7C-9E36-44A0-9F9A-79D4D0781D37@mac.com> Will Mac apps made with Rev 4.0 run on Lion? Or do I need to made my Mac apps with LiveCode 4.6.3? I have purchased LiveCode 4.6.3 but have yet use it to make any apps Mac or Windows. Sent from my iPad From effendi at wanadoo.fr Wed Jul 27 17:09:27 2011 From: effendi at wanadoo.fr (Francis Nugent Dixon) Date: Wed, 27 Jul 2011 23:09:27 +0200 Subject: Where does survive the inventive user ? Message-ID: <13527D10-FDE9-4F42-99EE-D7B83B34D5B7@wanadoo.fr> Hi, Craig said : > who might fall in love with LC if they only were simply > exposed to it Amen to that ! This was my point - Who IS exposed to LiveCode ? Maybe we can get some input from the LiveCode Commercial Department. How do you go about tickling a non-LiveCode-User ? Certainly not on an upturned dustbin in Trafalgar Square .... -Francis "Nothing should ever be done for the first time !" From barryb at libero.it Wed Jul 27 17:46:31 2011 From: barryb at libero.it (barryb at libero.it) Date: Wed, 27 Jul 2011 23:46:31 +0200 (CEST) Subject: Click along a line Message-ID: <24341282.2869931311803191746.JavaMail.root@wmail44> Hello CraigYes, that's the sort of thing I was looking for and it works fine as far as moving the clicks is concerned. However now the visible cursor remains on the button that triggers it all and I don't get the right colours returned, as I did from the desktop using screenMouseLoc.We shall overcome .....someday.Thanks for the help Barry From bobs at twft.com Wed Jul 27 18:19:31 2011 From: bobs at twft.com (Bob Sneidar) Date: Wed, 27 Jul 2011 15:19:31 -0700 Subject: OT: I want to buy Lion In-Reply-To: <9308DB19-829D-41A0-A887-B2F59C1C4CF4@mac.com> References: <001801cc4c80$41078b00$c316a100$@de> <8C7BA5AA-EDE0-4817-BB42-E8C01C7D4FDE@twft.com> <002101cc4c87$4f71dd60$ee559820$@de> <9308DB19-829D-41A0-A887-B2F59C1C4CF4@mac.com> Message-ID: <5797EA4F-46FF-4ABC-A3CB-1EB0EC7F5FAB@twft.com> So how do you download lion from the App store when all you have is Leopard or even Tiger? I do not think (perhaps I am wrong) that the App Store shows up in Leopard. Only Snow Leopard. And are you saying that Lion will install over top of an OS that is NOT Snow Leopard? This seems to contradict what I have been hearing about it, but I cannot test all this as I am already running Snow Leopard and don't want to upgrade to Lion, at least not yet. Bob On Jul 27, 2011, at 11:20 AM, Arie van der Ent wrote: > Hi Tiemo, > > It is not necessary to wait for Lion on a USB-stick. Don McAllister from screencastsonline did the following; > 1. buy Lion from App store; > 2. make a copy from the installer (in your application folder); > 3. open up the copy of the installer package, then open the folder Shared Support in it; > 4. there you'll find InstallEDG.dmg; > 5. copy this file to an USB-stick 8 gb. > > That's all. > > Arie From bobs at twft.com Wed Jul 27 18:22:24 2011 From: bobs at twft.com (Bob Sneidar) Date: Wed, 27 Jul 2011 15:22:24 -0700 Subject: OT: I want to buy Lion In-Reply-To: <201107271414.36804.warren@warrensweb.us> References: <001801cc4c80$41078b00$c316a100$@de> <002101cc4c87$4f71dd60$ee559820$@de> <9308DB19-829D-41A0-A887-B2F59C1C4CF4@mac.com> <201107271414.36804.warren@warrensweb.us> Message-ID: Yes it is. In fact I have done exactly that with the RC1 of Lion. I used Carbon Copy Cloner to copy my internal drive system and all to my Time Machine drive. If I want to run Lion, I reboot holding the option key and select the external drive. Bob On Jul 27, 2011, at 12:14 PM, Warren Samples wrote: > It does not seem likely that > Apple will ignore forever those users who need or would simply prefer to be able to purchase physical install > media of some sort or another, though. I wonder if it's possible to clone a drive and just install that drive > in another machine. > > Best, > > Warren From bobs at twft.com Wed Jul 27 18:37:01 2011 From: bobs at twft.com (Bob Sneidar) Date: Wed, 27 Jul 2011 15:37:01 -0700 Subject: Where does survive the inventive user ? In-Reply-To: <13527D10-FDE9-4F42-99EE-D7B83B34D5B7@wanadoo.fr> References: <13527D10-FDE9-4F42-99EE-D7B83B34D5B7@wanadoo.fr> Message-ID: <83F85A66-017E-433C-9FF3-B82907D243CD@twft.com> It's hard to get people who know non-scripted languages to even look at Livecode, because they think that something simpler than what they already know has to suffer from an equal lack of speed, usability, features, security etc. Even still, if you can convince them that they can produce professional quality apps in a fraction of the time, they remember how long it took to learn their present flavor of whatever, and wrongly project the same time and difficulty to learning "another language". I would say the best way is to actually produce polished apps and then showcase them, say in an online magazine that has that target audience. Talk about how much time it took to develop the app. Put example scripts in the article. Give links to some downloadable iPhone/iPad apps as examples of mobile apps. They need to get their hands on polished apps that do something well, and then be told "This took me 2 weeks to produce". Otherwise it's just another article in another publication among thousands every day. And I wouldn't mention Hypercard or the other supersets of that much. That alone is enough to give the wrong impression that Livecode is not very robust. Oh yes, and perhaps have development set a couple weeks aside to deal with some of the quirkier Livecode issues that have been around for a while. No one wants to burn away their 30 day trial futzing with IDE eccentricities. Bob On Jul 27, 2011, at 2:09 PM, Francis Nugent Dixon wrote: > Hi, > > Craig said : > >> who might fall in love with LC if they only were simply > >> exposed to it > > Amen to that ! > > This was my point - Who IS exposed to LiveCode ? > > Maybe we can get some input from the LiveCode Commercial > Department. How do you go about tickling a non-LiveCode-User ? > > Certainly not on an upturned dustbin in Trafalgar Square .... > > -Francis > > "Nothing should ever be done for the first time !" > > _______________________________________________ > use-livecode mailing list > use-livecode at lists.runrev.com > Please visit this url to subscribe, unsubscribe and manage your subscription preferences: > http://lists.runrev.com/mailman/listinfo/use-livecode From stephenREVOLUTION2 at barncard.com Wed Jul 27 18:38:02 2011 From: stephenREVOLUTION2 at barncard.com (stephen barncard) Date: Wed, 27 Jul 2011 15:38:02 -0700 Subject: Livecode Server 4.6.3 - First impressions pt2 - using the new version at Dreamhost Message-ID: *LIVECODE SERVER 4.6.3* Wow. Haven't done any speed benchmarks, and I've just started using this version of LCS, but it offers what was missing before, and a lot more. Stacks are full fledged citizens now . One can 'start using stack x' - one's tested land-based libraries can be used at sea. This is a ****** big deal! One can create, delete and open stacks stacks mean One can set and get custom properties of any dimension One can add and delete cards for more dimensions One can store and obfuscate binary files in props many dimensions /stack/card/propertyset/property/property/property ... etc caveats: no objects are allowed 'on' stacks. This will crash your code. (expected) stacks must be saved immediately after changes, before the handler quits. uses: sitewide database tasks - saving visitor data in an array libraries can be thoroughly debugged on the desktop and run intact in the server environment And here's my little editing surprise. I had the IDE open for the dictionary, and had accidentally clicked on the test *testprops.rev* file in the list in Transmit (the excellent FTP client from Panic Software). I expected to see the jumble of binary that usually shows in text editors. Instead, it loaded into the open Livecode IDE! I looked at the stack - it had all the changes I did and saved online. Then I made some updates in the stack in the IDE, and saved it back to the site, and they stuck! I don't remember everything working like this before. Round trip - edit compile save run - the ultimate speed design setup. Stacks are now on a par with text files for online editing ease. Another thing - the "inline" error messages are much better than before - more informative and right on the money. Great work REv team. -- Stephen Barncard San Francisco Ca. USA more about sqb From bonnmike at gmail.com Wed Jul 27 18:53:45 2011 From: bonnmike at gmail.com (Mike Bonner) Date: Wed, 27 Jul 2011 16:53:45 -0600 Subject: windows revserver stack support In-Reply-To: References: Message-ID: Fields work, you can place pretty much every object EXCEPT a button on there and it works. Its too bad. It is so much easier to develop a complete app in livecode that it would be nice to be able to place a completed working stack next to an lc script that supplies only interface elements. Can still do that though by doing the same thing on the desktop side, separate out the interface, and have all real script elements in another stack. Will just have to retrain myself, too used to making stacks to do what I need done right this moment, quick and dirty style. The new way will be better practice in the long run anyway. However, I still think failing with a seg fault is probably not a good thing. On Wed, Jul 27, 2011 at 2:04 PM, stephen barncard < stephenREVOLUTION2 at barncard.com> wrote: > yes you can have multiple cards and as many custom props as you like but > no > other objects like fields or buttons. Fail for sure. It's strictly an > intelligent storage device. > > On 27 July 2011 12:56, Mike Bonner wrote: > > > Got it to work. If there is a button in the stack it causes a sig 11. > > remove > > them and things start to work dandy. > > > > > > > Stephen Barncard > San Francisco Ca. USA > > more about sqb > _______________________________________________ > use-livecode mailing list > use-livecode at lists.runrev.com > Please visit this url to subscribe, unsubscribe and manage your > subscription preferences: > http://lists.runrev.com/mailman/listinfo/use-livecode > From kray at sonsothunder.com Wed Jul 27 19:18:48 2011 From: kray at sonsothunder.com (Ken Ray) Date: Wed, 27 Jul 2011 18:18:48 -0500 Subject: Mac Apps made with Rev run on Lion? In-Reply-To: <3B16BE7C-9E36-44A0-9F9A-79D4D0781D37@mac.com> References: <3B16BE7C-9E36-44A0-9F9A-79D4D0781D37@mac.com> Message-ID: On Jul 27, 2011, at 4:07 PM, Charles Szasz wrote: > Will Mac apps made with Rev 4.0 run on Lion? Or do I need to made my Mac apps with LiveCode 4.6.3? I have purchased LiveCode 4.6.3 but have yet use it to make any apps Mac or Windows. 4.0 apps work just fine on Lion... Ken Ray Sons of Thunder Software, Inc. Email: kray at sonsothunder.com Web Site: http://www.sonsothunder.com/ From barryb at libero.it Wed Jul 27 19:42:13 2011 From: barryb at libero.it (barryb at libero.it) Date: Thu, 28 Jul 2011 01:42:13 +0200 (CEST) Subject: Clicks along a line Message-ID: <16420647.2880471311810133094.JavaMail.root@wmail44> Hello Mike,THANKS, YES! Your explanation was really useful and the scriptlet worked first time, including all the dependent handlers.That's a project I gave up on months ago and only recently decided to have another try due to some new intuitions. Now I can go to bed happy! 01.30 Thanks again and also to Mark and Craig.Barry From bvlahos at mac.com Wed Jul 27 19:46:02 2011 From: bvlahos at mac.com (Bill Vlahos) Date: Wed, 27 Jul 2011 16:46:02 -0700 Subject: Mac Apps made with Rev run on Lion? In-Reply-To: <3B16BE7C-9E36-44A0-9F9A-79D4D0781D37@mac.com> References: <3B16BE7C-9E36-44A0-9F9A-79D4D0781D37@mac.com> Message-ID: <29E6F0A1-CA8C-40C4-9ED5-2B211BA03E33@mac.com> Yes apps built in 4.0 will work. Bill Vlahos _________________ InfoWallet (http://www.infowallet.com) is about keeping your important life information with you, accessible, and secure. On Jul 27, 2011, at 2:07 PM, Charles Szasz wrote: > Will Mac apps made with Rev 4.0 run on Lion? Or do I need to made my Mac apps with LiveCode 4.6.3? I have purchased LiveCode 4.6.3 but have yet use it to make any apps Mac or Windows. > > Sent from my iPad > > _______________________________________________ > use-livecode mailing list > use-livecode at lists.runrev.com > Please visit this url to subscribe, unsubscribe and manage your subscription preferences: > http://lists.runrev.com/mailman/listinfo/use-livecode From andre at andregarzia.com Wed Jul 27 20:03:47 2011 From: andre at andregarzia.com (Andre Garzia) Date: Wed, 27 Jul 2011 21:03:47 -0300 Subject: Livecode Server 4.6.3 - First impressions pt2 - using the new version at Dreamhost In-Reply-To: References: Message-ID: Stephen, be aware that loading large stacks into memory will get you to the memory protection police! If your database stack is too big, and lots of copies are loaded for all the requests, Apache or whoever does the purging might kill your engine to save resources. This is hardcoded into On-Rev accounts. The engine on dreamhost should not have these limits but still, dreamhost is a shared server and they will probably kill processess that starte the server of its resources. Cheers andre On Wed, Jul 27, 2011 at 7:38 PM, stephen barncard < stephenREVOLUTION2 at barncard.com> wrote: > *LIVECODE SERVER 4.6.3* > > Wow. Haven't done any speed benchmarks, and I've just started using this > version of LCS, but it offers what was missing before, and a lot more. > > Stacks are full fledged citizens now . One can 'start using stack x' - > one's tested land-based libraries can be used at sea. > > This is a ****** big deal! > > One can create, delete and open stacks > > stacks mean > > One can set and get custom properties of any dimension > One can add and delete cards for more dimensions > One can store and obfuscate binary files in props > many dimensions /stack/card/propertyset/property/property/property ... etc > > caveats: > > no objects are allowed 'on' stacks. This will crash your code. (expected) > stacks must be saved immediately after changes, before the handler quits. > > uses: > > sitewide database tasks - saving visitor data in an array > libraries can be thoroughly debugged on the desktop and run intact in the > server environment > > > > And here's my little editing surprise. > > I had the IDE open for the dictionary, and had accidentally clicked on the > test *testprops.rev* file in the list in Transmit (the excellent FTP client > from Panic Software). I expected to see the jumble of binary that usually > shows in text editors. Instead, it loaded into the open Livecode IDE! > > I looked at the stack - it had all the changes I did and saved online. Then > I made some updates in the stack in the IDE, and saved it back to the site, > and they stuck! > > I don't remember everything working like this before. > > Round trip - edit compile save run - the ultimate speed design setup. > Stacks are now on a par with text files for online editing ease. > > Another thing - the "inline" error messages are much better than before - > more informative and right on the money. > > Great work REv team. > > > > -- > > > > Stephen Barncard > San Francisco Ca. USA > > more about sqb > _______________________________________________ > use-livecode mailing list > use-livecode at lists.runrev.com > Please visit this url to subscribe, unsubscribe and manage your > subscription preferences: > http://lists.runrev.com/mailman/listinfo/use-livecode > -- http://www.andregarzia.com All We Do Is Code. From chipp at altuit.com Wed Jul 27 20:51:54 2011 From: chipp at altuit.com (Chipp Walters) Date: Wed, 27 Jul 2011 19:51:54 -0500 Subject: [OT] More Apple Foolishness Message-ID: <6F918963-BD07-4351-83D1-41E1A78D06FD@altuit.com> Maybe I'm not the biggest Apple fan in the world, but my family does own three iPhones, three iPads, three Apple TV's and a MacBook Air. So, in some sense, I do support them, no matter how ridiculous their 1984-like Big Brother policies. And I feel it gives the right to gripe about some stuff as well. Warning: Those for which Apple can do no wrong should press delete and move on to the next thread. You've been warned! So today, I update my apps, and it turns out my Kindle app, (and the B&N Nook) has an update which remove per Apple's latest policy, the "Shop for a book" button in the app-- which basically just launched Safari and went to the Amazon.com Kindle section. So now, I have to manually go to Amazon.com to buy a book-- requiring me to go an extra step and launch the browser. Smartly, Amazon figured this out and as soon as I arrive at their Kindle store in Safari, a little JavaScript popup said to "click this button and we'll put a nice Kindle shop icon on the Home screen." Which I did-- and now I have a Kindle shop icon next to my Kindle icon, thus cluttering up my home screens further. Score 1 for Amazon, 0 for Apple and -1 for the customer. Nice to see Apple looking out for their customers. Damn, if only there was ANY decent alternative to iOS. Fact is, there are so many interface problems with iOS, I don't even know were to start. Oh yes I do-- somewhere after four or five screens worth of icons, you forget where stuff is. Finding an app in "folders" is like playing that kids game of concentration, is it here? No, how about here? No. Where in tarnation is it? Oh, let's search for it, which finds and launches it, but still doesn't give you a clue about where it's at. Please give me a list or outline view, which can sort by name, or date installed, or date last used. Smartly someone at the AppStore has figured some of this out as they AT LEAST show your updates and purchased apps in a list, though searching for an app to purchase there is a supreme exercise in frustration. Thankfully, others have jumped in to help. For instance there's a great free app called Discovr (yes that's the correct spelling) which creates a mind map diagram linking an app you know with others like it. Then there's the free website Quixey, which does a great job indexing apps of all kinds. From chipp at altuit.com Wed Jul 27 20:59:45 2011 From: chipp at altuit.com (Chipp Walters) Date: Wed, 27 Jul 2011 19:59:45 -0500 Subject: Tahoma Font In-Reply-To: <000c01cc4c85$b0b7ab10$12270130$@net> References: <000001cc4bd2$cd1e4470$675acd50$@net> <000f01cc4c3c$3b747b60$b25d7220$@de> <000c01cc4c85$b0b7ab10$12270130$@net> Message-ID: <7718C7D1-010C-412B-B60C-9C94D32E893D@altuit.com> Camm, Open up your stack in Rev as Topstack and before you make a standalone, type this in the message box: set the textfont of the topstack to "Tahoma" Then press enter and save your stack. Now make a standalone and see if it doesn't work. If Tahoma is installed on a Windows users machine, it will be used for all the "effective" textfont styles. Chipp Walters CEO, Shafer Walters Group, Inc On Jul 27, 2011, at 12:50 PM, "Camm" wrote: > Well , my issue is just with windows ? > The Tahoma font is their during development mode but vanishes in Standalone > on XP. > If I set the properties of the standalone.exe in Windows to 95 or 98 > compatibility it returns ? > > How do I force load a font on startup ? > > Best Regards > Camm > > > -----Original Message----- > From: use-livecode-bounces at lists.runrev.com > [mailto:use-livecode-bounces at lists.runrev.com] On Behalf Of Tiemo Hollmann > TB > Sent: 27 July 2011 10:05 > To: 'How to use LiveCode' > Subject: AW: Tahoma Font > > When I started with revolution Tahoma was the default font of all objects > and I didn't cared about it at that time. I am now using Tahoma since years > without any problems on windows and Mac, though I have learned that Tahoma > isn't a standard font on Mac and is supposed to be substituted (what never > made any problems in my apps). On windows systems Tahoma is a standard font > up to today as far as I know, or did I miss anything? > Tiemo > >> -----Urspr?ngliche Nachricht----- >> Von: use-livecode-bounces at lists.runrev.com [mailto:use-livecode- >> bounces at lists.runrev.com] Im Auftrag von Camm >> Gesendet: Dienstag, 26. Juli 2011 22:30 >> An: use-livecode at lists.runrev.com >> Betreff: Tahoma Font >> >> Tahoma font will not work in standalone unless its run in Windows 95 or 98 >> compatibility mode? >> >> >> >> I am using Windows XP...... Any ideas ? >> >> >> >> Best Regards >> >> Camm >> >> _______________________________________________ >> use-livecode mailing list >> use-livecode at lists.runrev.com >> Please visit this url to subscribe, unsubscribe and manage your > subscription >> preferences: >> http://lists.runrev.com/mailman/listinfo/use-livecode > > > _______________________________________________ > use-livecode mailing list > use-livecode at lists.runrev.com > Please visit this url to subscribe, unsubscribe and manage your subscription > preferences: > http://lists.runrev.com/mailman/listinfo/use-livecode > ----- > No virus found in this message. > Checked by AVG - www.avg.com > Version: 10.0.1390 / Virus Database: 1518/3789 - Release Date: 07/26/11 > > > _______________________________________________ > use-livecode mailing list > use-livecode at lists.runrev.com > Please visit this url to subscribe, unsubscribe and manage your subscription preferences: > http://lists.runrev.com/mailman/listinfo/use-livecode From cszasz at mac.com Wed Jul 27 21:03:21 2011 From: cszasz at mac.com (Charles Szasz) Date: Wed, 27 Jul 2011 21:03:21 -0400 Subject: Mac Apps made with Rev run on Lion? Message-ID: Thanks Ken and Bill! One more question. For Lion only, should I compile only for Intel for Lion? Or will universal run on Lion? Sent from my iPad Sent from my iPad From slava at lexiconbridge.com Wed Jul 27 21:13:02 2011 From: slava at lexiconbridge.com (Slava Paperno) Date: Wed, 27 Jul 2011 21:13:02 -0400 Subject: [OT] More Apple Foolishness In-Reply-To: <6F918963-BD07-4351-83D1-41E1A78D06FD@altuit.com> References: <6F918963-BD07-4351-83D1-41E1A78D06FD@altuit.com> Message-ID: <00af01cc4cc3$83a64f30$8af2ed90$@com> I like the way these consecutive statements go together: > Damn, if only there was ANY decent alternative to iOS. > > Fact is, there are so many interface problems with iOS, I don't even > know where to start... S. From gandalf at doctorTimothyMiller.com Wed Jul 27 21:25:48 2011 From: gandalf at doctorTimothyMiller.com (Timothy Miller) Date: Wed, 27 Jul 2011 18:25:48 -0700 Subject: Where does survive the inventive user ? In-Reply-To: <8CE1A70D9625EA6-2550-214DE@web-mmc-d04.sysops.aol.com> References: <8CE1A70D9625EA6-2550-214DE@web-mmc-d04.sysops.aol.com> Message-ID: <44857038-F42F-4374-8DA7-F6BCDF16629C@doctorTimothyMiller.com> I've never been a computer professional. Not even close. I taught myself to write Basic programs for my Atari 64, mostly out of curiosity. Around 1984 I taught myself HyperCard. I wrote a variety of applications for home and business use. A pretty good phonics tutorial for my kids, among others. And I gradually cobbled together a complex set of HyperCard stacks, which I use every day to run my business. They're crude, kludgy and ugly, but they work. Many thousands of lines of script, no idea how many thousands. When I need a new feature, I write it. Often, it's working reliably in less than an hour. It's been years since I saw an error message. Sometimes I "discover" clever and useful features I wrote and forgot about. When HyperCard became obsolete, I moved over to Runtime Revolution, which was rather an ordeal, because RR was far more complex. It's comfortable now. And now it's LiveCode, but I've barely noticed the change. LiveCode does many things I don't understand, but that's not a problem. I still write stacks for my own use. I recently written a stack to help me study and identify photos and songs of birds. Also, I'm taking notes for a book and I've written a stack to help me organize the notes. I will eventually use the same stack to help me develop the book -- probably some kind of a one-paragraph-per-card arrangement with many summarizing, indexing, re-sequencing, search and notation features. I tweak old features and invent new ones as I go along. None of this seems very difficult. It's a gradual transition from HyperCard. I'm not really a nerd -- computers don't fascinate me all that much. I write a stack when the time invested justifies the functionality of the final product. It's gradually dawning on me that "programmers" like me have become rather rare. Fewer and fewer non-professionals on this list, as far as I can tell. I don't understand why, seems like a shame. Many people have use for the kind of functionality an amateur and dabbler can get out of LiveCode, and it isn't that hard to do. Admittedly, HyperCard was easier, simply because it was less complex. I've wondered if LiveCode might be more approachable if it had some kind of "dumb mode," sort of like the old userLevel system in HyperCard. Probably won't happen though. FWIW... Tim On Jul 27, 2011, at 5:35 AM, dunbarx at aol.com wrote: > In the old days, Hypercard was. like a viral pandemic, infected the world because it was bundled with every Macintosh. It was offered like a promotion, a possibly valuable coupon one gets in the mail, which you will at least read before throwing out, And it became a nerd fad, with many hundreds of thousands of people trying it out. Certainly only a small fraction became enamored; many of those are reading this post. > > > Without that once in a lifetime vehicle, it is an uphill battle to engage people who might fall in love with LC if they only were simply exposed to it. Worse, these days, the mindset is that everything comes in small ready-to-go packages, complete and compact. I have three kids who just don't think about building stuff, especially from raw materials. I used to, though. > > > Thank the iMac, iPhone, iPod, etc., for creating that expectation, a far cry from reading a bank of eight lights telling you what byte was currently passing by. > > > LC should be taught in the ninth grade in every school in the world. > > > Craig Newman > > > > > > > > > > -----Original Message----- > From: Francis Nugent Dixon > To: use-livecode > Sent: Wed, Jul 27, 2011 2:42 am > Subject: Where does survive the inventive user ? > > > Alejandro asks some potent questions ..... > > Question 1 - Is programming so easy ..... ? > > I think we should ask the question "Is programming a niche occupation ? > > During my early career, practically everybody I knew was a programmer > (birds of a feather flock together ?) > > Now I am out of the industry (retired), outside of the LiveCode forum, > I don't know anybody who programs (although most of my acquaintances > have computers and Google all day, they don't WRITE programs.) > Programming is a mentality, and there aren't many of us who have > this mentality (even to make money from it). Although LiveCode is > a great incentive for non-programmers to "have a go" , programming > is limiited to a strange mind-form which even I cannot define ! > The question should be - "What is the VISIBILTY of LiveCode to the man > in the street who has never even thought about programming ?" > And the answer is "NONE". The chances of anybody "moving in" to > programming are about the same as being struck by lightning. > > Question 2 Did anyone know someone ..... ? > > I would think that buying a mobile platform (iPhone, iPad), and > learning to program are two ideas so far apart, as to be unlikely. > I do not know anyone who has started programming because of this > "mobile technology" and the platforms thereon. In a recent thread > on the forum I voiced my "No Way Baby" intent to NOT go to mobile > computing, although I wrote my first program more than 50 years > ago, and I still program EVERY day. The cost is far too high, and > the returns are doubtful ! I am a rare bird who now programs for fun. > > Question 3 Will mobile computing displace desktop computing .... ? > > This reminds me of the 1980's question "Will Desktop computing ever > displace Mainframe Computing ?" A lot of people said no in them days. > > Industry has been talking about "cloud computing" for years, and > IS slowly moving towards it. But, although I may accept having my > data in a cloud, I will always want my apps to be in my hand, so > I can have control over them. Renting application use out of a > cloud would be the same has handing over your wallet to a stranger. > You can see which direction Apple is going. They want to charge > you for the use of YOUR OWN computer, and then for storing your > data in their cloud, and then for using their applications from > their cloud. That could cost you an arm and a leg. All my > communication in the hands of a stranger ? It's bad enough already! > God help us all in the future ! > > The problem is - it's not hype - it's tomorrows computing, and > I don't like the way the wind is blowing ..... > > The days when you rented an application, and you got the computer > for free may return. When computers become so dirt cheap that there > is no big profit to make, those guys "up there" have to think of a > new way to get your money. We will soon be paying more for > communication facilities than we are spending on food > (si ce n'est deja fait !, as they say here)! > > "Nothing should ever be done for the first time !" > > -Francis > > PS. How about the question "When will we be grafting micro-chips > into the brain to allow us instant and global communication, and > complex problem solving and decision making ? > > > _______________________________________________ > use-livecode mailing list > use-livecode at lists.runrev.com > Please visit this url to subscribe, unsubscribe and manage your subscription > preferences: > http://lists.runrev.com/mailman/listinfo/use-livecode > > > > _______________________________________________ > use-livecode mailing list > use-livecode at lists.runrev.com > Please visit this url to subscribe, unsubscribe and manage your subscription preferences: > http://lists.runrev.com/mailman/listinfo/use-livecode From kee at kagi.com Wed Jul 27 21:26:27 2011 From: kee at kagi.com (Kee Nethery) Date: Wed, 27 Jul 2011 18:26:27 -0700 Subject: OT: I want to buy Lion In-Reply-To: <201107271414.36804.warren@warrensweb.us> References: <001801cc4c80$41078b00$c316a100$@de> <002101cc4c87$4f71dd60$ee559820$@de> <9308DB19-829D-41A0-A887-B2F59C1C4CF4@mac.com> <201107271414.36804.warren@warrensweb.us> Message-ID: <49DD8E13-C41D-4570-801F-AC5654D1F6DD@kagi.com> Oregon physical Apple stores appear to be letting people come into the store to do the install. Perhaps physical Apple stores in your location offer the same service. Kee From slava at lexiconbridge.com Wed Jul 27 21:54:10 2011 From: slava at lexiconbridge.com (Slava Paperno) Date: Wed, 27 Jul 2011 21:54:10 -0400 Subject: horizontal scrollbar in revBrowser Message-ID: <00b001cc4cc9$41afa7b0$c50ef710$@com> Rather like any Web browser window, my revBrowser windows sometimes has a horizontal scroll bar and sometimes not. Because the display in my revBrowser window is wrappable text, this horizontal scroll bar is never useful. Even when it appears, it only scrolls for about five or six pixels. Unnecessarily. My revBrowser window is resized as the stack window is resized, and as I drag the window borders to make it wider or narrower, I can always find a width at which the horizontal scroll bar is not there. Is there a clever way to get rid of this pesky scroll bar? I have complete control over the Web pages that are displayed in this revBrowser and can add any style tricks to it. Slava From chipp at chipp.com Wed Jul 27 21:57:51 2011 From: chipp at chipp.com (Chipp Walters) Date: Wed, 27 Jul 2011 20:57:51 -0500 Subject: [OT] More Apple Foolishness In-Reply-To: <00af01cc4cc3$83a64f30$8af2ed90$@com> References: <6F918963-BD07-4351-83D1-41E1A78D06FD@altuit.com> <00af01cc4cc3$83a64f30$8af2ed90$@com> Message-ID: Yep, and that's the rub. Google has too much money and not enough discipline to put together any decent offering. And then there's Microsoft-- so sad. As long as Balmer's at the helm, they are doomed to mediocrity. And once Apple was the high priest of UIX, and now how the mighty have fallen. I wonder what Tog thinks? On Wed, Jul 27, 2011 at 8:13 PM, Slava Paperno wrote: > I like the way these consecutive statements go together: > > > Damn, if only there was ANY decent alternative to iOS. > > > > Fact is, there are so many interface problems with iOS, I don't even > > know where to start... > > S. > > > > > _______________________________________________ > use-livecode mailing list > use-livecode at lists.runrev.com > Please visit this url to subscribe, unsubscribe and manage your > subscription preferences: > http://lists.runrev.com/mailman/listinfo/use-livecode > -- Chipp Walters CEO, Shafer Walters Group, Inc. From chipp at chipp.com Wed Jul 27 22:03:19 2011 From: chipp at chipp.com (Chipp Walters) Date: Wed, 27 Jul 2011 21:03:19 -0500 Subject: [OT] More Apple Foolishness In-Reply-To: References: <6F918963-BD07-4351-83D1-41E1A78D06FD@altuit.com> <00af01cc4cc3$83a64f30$8af2ed90$@com> Message-ID: Well, turns out with a little digital sleuthing, we can find out exactly what Tog thinks, and he's pretty much thinking the same thing: "In Part 1, I discussed the ill-effects to the Macintosh of Apple's Flatland aesthetic, a visual simplicity that threatens to bury Apple's users with unnecessary clutter and complexity." and at: http://www.asktog.com/columns/076AppleFlatlandPart2.html iPhone/iPod Touch Home Screen You'll see the now familiar Flatland ethic on the home screen of the iPhone and iPod Touch, where it's impossible to, for example, drag all your games into a folder. Instead, you are offered eight distinct, flat, unlabeled pages that you must spin back and forth through in search of that one elusive program. Apple has thus created eight faithful copies of the Windows ?95 desktop, where new icons cling to the uppermost, leftmost corner of the screen like helium balloons in a leftward-blowing wind. At least with Windows ?95, you could also have folders, not just Apps and documents, on the desktop?not where you wanted them, true, but at least they were there. However, the iPhone/iPod Touch desktop doesn?t even allow folders, part and parcel of the endless prairie of Flatland. This flatness mania is damaging Apple developers. When the App Store first opened, I was buying everything. I?ve now stopped buying. I no longer have anything I want to throw away and nowhere to put anything new. I?m an early (and compulsive) adopter, but millions of others will soon reach this same point. The gold rush is going to suddenly be over, and it has nothing to do with people getting bored or the Apps becoming less interesting. (See, "Right Study, Wrong Prediction," below.) It?s just that Apple has failed to give people a means of storing what they might buy. To copy such a poor example of design as the Windows ?95 desktop, ten-plus years later, is almost unbelievable, and I?m quite confident that, as with George Harrison copying ?She?s So Fine? when composing ?My Sweet Lord,? it was inadvertent. However, it was also naive and misplaced, and it needs to be corrected before Apps sales collapse on their developers. From chipp at chipp.com Wed Jul 27 22:08:22 2011 From: chipp at chipp.com (Chipp Walters) Date: Wed, 27 Jul 2011 21:08:22 -0500 Subject: horizontal scrollbar in revBrowser In-Reply-To: <00b001cc4cc9$41afa7b0$c50ef710$@com> References: <00b001cc4cc9$41afa7b0$c50ef710$@com> Message-ID: html {overflow-x:hidden;} On Wed, Jul 27, 2011 at 8:54 PM, Slava Paperno wrote: > Rather like any Web browser window, my revBrowser windows sometimes has a > horizontal scroll bar and sometimes not. Because the display in my > revBrowser window is wrappable text, this horizontal scroll bar is never > useful. Even when it appears, it only scrolls for about five or six pixels. > Unnecessarily. > > My revBrowser window is resized as the stack window is resized, and as I > drag the window borders to make it wider or narrower, I can always find a > width at which the horizontal scroll bar is not there. > > Is there a clever way to get rid of this pesky scroll bar? I have complete > control over the Web pages that are displayed in this revBrowser and can > add > any style tricks to it. > > Slava > > > > _______________________________________________ > use-livecode mailing list > use-livecode at lists.runrev.com > Please visit this url to subscribe, unsubscribe and manage your > subscription preferences: > http://lists.runrev.com/mailman/listinfo/use-livecode > -- Chipp Walters CEO, Shafer Walters Group, Inc. From eriks at sisyph.us Wed Jul 27 22:19:12 2011 From: eriks at sisyph.us (Erik Schwartz) Date: Wed, 27 Jul 2011 19:19:12 -0700 Subject: Sending Text Messages In-Reply-To: <2FF2DA76-933D-4D8A-889D-86B0E8B28F81@economy-x-talk.com> References: <7D97AF38-30DC-4EE2-91C4-DEE3314E8C35@mac.com> <2FF2DA76-933D-4D8A-889D-86B0E8B28F81@economy-x-talk.com> Message-ID: You can use something like Twilio, or if you're talking low volume you can often use the carrier's email gateways. Erik On Wed, Jul 27, 2011 at 12:52 PM, Mark Schonewille wrote: > To where? > > -- > Best regards, > > Mark Schonewille > > Economy-x-Talk Consulting and Software Engineering > Homepage: http://economy-x-talk.com > Twitter: http://twitter.com/xtalkprogrammer > KvK: 50277553 > > What does that error mean? Buy LiveCodeErrors for iPhone now http://qery.us/v4 A must-have for LiveCode programmers. > > On 27 jul 2011, at 21:48, Dave Wares wrote: > >> Is it possible to send a text message directly from LiveCode, even if the user still needs to hit the "send" button? > > > _______________________________________________ > use-livecode mailing list > use-livecode at lists.runrev.com > Please visit this url to subscribe, unsubscribe and manage your subscription preferences: > http://lists.runrev.com/mailman/listinfo/use-livecode > -- ========================================== eriks at sisyph.us? ? ? ? ? ? ? ? ? ? ? ? ? ? ? ? ? http://sisyph.us (530) 213-ERIK? ? ? ? ? ? ? ? ? ? ? ? ? http://twitter.com/eriks (530) 213-3745?? http://www.linkedin.com/in/erikschwartz ========================================== From pmbrig at gmail.com Wed Jul 27 22:24:01 2011 From: pmbrig at gmail.com (Peter Brigham MD) Date: Wed, 27 Jul 2011 22:24:01 -0400 Subject: Where does survive the inventive user ? In-Reply-To: <44857038-F42F-4374-8DA7-F6BCDF16629C@doctorTimothyMiller.com> References: <8CE1A70D9625EA6-2550-214DE@web-mmc-d04.sysops.aol.com> <44857038-F42F-4374-8DA7-F6BCDF16629C@doctorTimothyMiller.com> Message-ID: <9F9F49D9-9AF5-45B6-AF72-B2B239E55388@gmail.com> Just to let you know that you're not alone -- I'm a similar LC user, started with HC (actually bought and read through Danny Goodman's book even before I bought my first Mac), developed a set of stacks to manage my clinical notes, incorporated more and more features, moved it over to LC a number of years ago, at which point with all of LC's capability the feature set grew even larger, now a full-fledged practice management tool, with 45 substacks, over 32,000 lines of script, couldn't manage without it. I'm not a programmer, just picked it up on the side. I have no idea how you market to people like me, but I suspect there are lots of us scattered around. -- Peter Peter M. Brigham pmbrig at gmail.com http://home.comcast.net/~pmbrig On Jul 27, 2011, at 9:25 PM, Timothy Miller wrote: > I've never been a computer professional. Not even close. I taught myself to write Basic programs for my Atari 64, mostly out of curiosity. Around 1984 I taught myself HyperCard. I wrote a variety of applications for home and business use. A pretty good phonics tutorial for my kids, among others. And I gradually cobbled together a complex set of HyperCard stacks, which I use every day to run my business. They're crude, kludgy and ugly, but they work. Many thousands of lines of script, no idea how many thousands. When I need a new feature, I write it. Often, it's working reliably in less than an hour. It's been years since I saw an error message. Sometimes I "discover" clever and useful features I wrote and forgot about. > > When HyperCard became obsolete, I moved over to Runtime Revolution, which was rather an ordeal, because RR was far more complex. It's comfortable now. And now it's LiveCode, but I've barely noticed the change. LiveCode does many things I don't understand, but that's not a problem. > > I still write stacks for my own use. I recently written a stack to help me study and identify photos and songs of birds. Also, I'm taking notes for a book and I've written a stack to help me organize the notes. I will eventually use the same stack to help me develop the book -- probably some kind of a one-paragraph-per-card arrangement with many summarizing, indexing, re-sequencing, search and notation features. I tweak old features and invent new ones as I go along. > > None of this seems very difficult. It's a gradual transition from HyperCard. I'm not really a nerd -- computers don't fascinate me all that much. I write a stack when the time invested justifies the functionality of the final product. > > It's gradually dawning on me that "programmers" like me have become rather rare. Fewer and fewer non-professionals on this list, as far as I can tell. I don't understand why, seems like a shame. > > Many people have use for the kind of functionality an amateur and dabbler can get out of LiveCode, and it isn't that hard to do. > > Admittedly, HyperCard was easier, simply because it was less complex. I've wondered if LiveCode might be more approachable if it had some kind of "dumb mode," sort of like the old userLevel system in HyperCard. Probably won't happen though. > > FWIW... > > Tim > > > > On Jul 27, 2011, at 5:35 AM, dunbarx at aol.com wrote: > >> In the old days, Hypercard was. like a viral pandemic, infected the world because it was bundled with every Macintosh. It was offered like a promotion, a possibly valuable coupon one gets in the mail, which you will at least read before throwing out, And it became a nerd fad, with many hundreds of thousands of people trying it out. Certainly only a small fraction became enamored; many of those are reading this post. >> >> >> Without that once in a lifetime vehicle, it is an uphill battle to engage people who might fall in love with LC if they only were simply exposed to it. Worse, these days, the mindset is that everything comes in small ready-to-go packages, complete and compact. I have three kids who just don't think about building stuff, especially from raw materials. I used to, though. >> >> >> Thank the iMac, iPhone, iPod, etc., for creating that expectation, a far cry from reading a bank of eight lights telling you what byte was currently passing by. >> >> >> LC should be taught in the ninth grade in every school in the world. >> >> >> Craig Newman >> >> >> >> >> >> >> >> >> >> -----Original Message----- >> From: Francis Nugent Dixon >> To: use-livecode >> Sent: Wed, Jul 27, 2011 2:42 am >> Subject: Where does survive the inventive user ? >> >> >> Alejandro asks some potent questions ..... >> >> Question 1 - Is programming so easy ..... ? >> >> I think we should ask the question "Is programming a niche occupation ? >> >> During my early career, practically everybody I knew was a programmer >> (birds of a feather flock together ?) >> >> Now I am out of the industry (retired), outside of the LiveCode forum, >> I don't know anybody who programs (although most of my acquaintances >> have computers and Google all day, they don't WRITE programs.) >> Programming is a mentality, and there aren't many of us who have >> this mentality (even to make money from it). Although LiveCode is >> a great incentive for non-programmers to "have a go" , programming >> is limiited to a strange mind-form which even I cannot define ! >> The question should be - "What is the VISIBILTY of LiveCode to the man >> in the street who has never even thought about programming ?" >> And the answer is "NONE". The chances of anybody "moving in" to >> programming are about the same as being struck by lightning. >> >> Question 2 Did anyone know someone ..... ? >> >> I would think that buying a mobile platform (iPhone, iPad), and >> learning to program are two ideas so far apart, as to be unlikely. >> I do not know anyone who has started programming because of this >> "mobile technology" and the platforms thereon. In a recent thread >> on the forum I voiced my "No Way Baby" intent to NOT go to mobile >> computing, although I wrote my first program more than 50 years >> ago, and I still program EVERY day. The cost is far too high, and >> the returns are doubtful ! I am a rare bird who now programs for fun. >> >> Question 3 Will mobile computing displace desktop computing .... ? >> >> This reminds me of the 1980's question "Will Desktop computing ever >> displace Mainframe Computing ?" A lot of people said no in them days. >> >> Industry has been talking about "cloud computing" for years, and >> IS slowly moving towards it. But, although I may accept having my >> data in a cloud, I will always want my apps to be in my hand, so >> I can have control over them. Renting application use out of a >> cloud would be the same has handing over your wallet to a stranger. >> You can see which direction Apple is going. They want to charge >> you for the use of YOUR OWN computer, and then for storing your >> data in their cloud, and then for using their applications from >> their cloud. That could cost you an arm and a leg. All my >> communication in the hands of a stranger ? It's bad enough already! >> God help us all in the future ! >> >> The problem is - it's not hype - it's tomorrows computing, and >> I don't like the way the wind is blowing ..... >> >> The days when you rented an application, and you got the computer >> for free may return. When computers become so dirt cheap that there >> is no big profit to make, those guys "up there" have to think of a >> new way to get your money. We will soon be paying more for >> communication facilities than we are spending on food >> (si ce n'est deja fait !, as they say here)! >> >> "Nothing should ever be done for the first time !" >> >> -Francis >> >> PS. How about the question "When will we be grafting micro-chips >> into the brain to allow us instant and global communication, and >> complex problem solving and decision making ? >> >> >> _______________________________________________ >> use-livecode mailing list >> use-livecode at lists.runrev.com >> Please visit this url to subscribe, unsubscribe and manage your subscription >> preferences: >> http://lists.runrev.com/mailman/listinfo/use-livecode >> >> >> >> _______________________________________________ >> use-livecode mailing list >> use-livecode at lists.runrev.com >> Please visit this url to subscribe, unsubscribe and manage your subscription preferences: >> http://lists.runrev.com/mailman/listinfo/use-livecode > > > _______________________________________________ > use-livecode mailing list > use-livecode at lists.runrev.com > Please visit this url to subscribe, unsubscribe and manage your subscription preferences: > http://lists.runrev.com/mailman/listinfo/use-livecode From jacque at hyperactivesw.com Wed Jul 27 22:25:43 2011 From: jacque at hyperactivesw.com (J. Landman Gay) Date: Wed, 27 Jul 2011 21:25:43 -0500 Subject: [OT] More Apple Foolishness In-Reply-To: <6F918963-BD07-4351-83D1-41E1A78D06FD@altuit.com> References: <6F918963-BD07-4351-83D1-41E1A78D06FD@altuit.com> Message-ID: <4E30C8A7.40405@hyperactivesw.com> On 7/27/11 7:51 PM, Chipp Walters wrote: > > Warning: Those for which Apple can do no wrong should press delete > and move on to the next thread. You've been warned! Well, that's pretty much me. For the desktop OS anyway. I'm getting more broadminded in my old age though. > > Fact is, there are so many interface problems with iOS, I don't even > know were to start. Yeah. I feel like a traitor, but I am less than enchanted with iOS. Apple, who used to be the master of intuitive interface, has dumbed down iOS so much that I can't find anything. Last night I wanted to add a bookmark to Safari and I had to look up how to do it in the user manual -- there was no intuitive way. If you want to add a bookmark, wouldn't you naturally tap on the bookmark icon and look for a command in there? No, to add a bookmark you tap on the Export icon, the same one that lets you save stuff or email a link. Silly me, expecting bookmark functions to be under a Bookmark icon. What I miss most are menus. All Android apps have a Home, Menu, and Back button, and they always do the same things. iOS apps are capricious and arbitrary with no consistency. The iOS Calendar has almost no interface at all -- or if it does, Apple has managed to hide it successfully from me. So it's back to the user manual. Aha. To link to an online calendar, you do not open the button named "Calendars." No, no, that would be too obvious. You exit the app and go to system settings, hunt for Calendar in the list, work your way through a few screens, and set it there. I have read more Apple documentation in the last 2 months than I have in the last ten years. Maybe I'm just too entrenched in old habits, maybe it's intuitive to new users. My iPad is a gorgeous machine with breathtakingly beautiful apps, it is faster (though much heavier) than my cheapie Android tablet and works more reliably in iffy wi-fi situations. But in spite of my Android's hiccups and performance issues, I still choose it when I want to actually use a tablet for something other than testing. About the ebook restrictions: the online media has been jumping all over that the last couple of days. Everybody is about as PO'ed as you are. -- Jacqueline Landman Gay | jacque at hyperactivesw.com HyperActive Software | http://www.hyperactivesw.com From bvlahos at mac.com Wed Jul 27 22:35:17 2011 From: bvlahos at mac.com (Bill Vlahos) Date: Wed, 27 Jul 2011 19:35:17 -0700 Subject: Mac Apps made with Rev run on Lion? In-Reply-To: References: Message-ID: Either work on Lion. If you only care about Lion then use the Intel build as Lion won't run on PPC Macs anyway. This is also true for Snow Leopard. Unless you don't care about older Macs use the Universal build and you will be fine. Bill Vlahos _________________ InfoWallet (http://www.infowallet.com) is about keeping your important life information with you, accessible, and secure. On Jul 27, 2011, at 6:03 PM, Charles Szasz wrote: > Thanks Ken and Bill! > > One more question. For Lion only, should I compile only for Intel for Lion? Or will universal run on Lion? > > Sent from my iPad > > Sent from my iPad > _______________________________________________ > use-livecode mailing list > use-livecode at lists.runrev.com > Please visit this url to subscribe, unsubscribe and manage your subscription preferences: > http://lists.runrev.com/mailman/listinfo/use-livecode From bvlahos at mac.com Wed Jul 27 22:43:18 2011 From: bvlahos at mac.com (Bill Vlahos) Date: Wed, 27 Jul 2011 19:43:18 -0700 Subject: [OT] More Apple Foolishness In-Reply-To: References: <6F918963-BD07-4351-83D1-41E1A78D06FD@altuit.com> <00af01cc4cc3$83a64f30$8af2ed90$@com> Message-ID: <80001DA7-522D-4AB6-A812-501492B660BF@mac.com> Actually you can have folders. 1. Hold your finger on an icon until they all quiver 2. Drag an icon(s) on top of another one and it will make a folder of them. Bill Vlahos _________________ InfoWallet (http://www.infowallet.com) is about keeping your important life information with you, accessible, and secure. On Jul 27, 2011, at 7:03 PM, Chipp Walters wrote: > At least with Windows ?95, you could also have folders, not just Apps and > documents, on the desktop?not where you wanted them, true, but at least they > were there. However, the iPhone/iPod Touch desktop doesn?t even allow > folders, part and parcel of the endless prairie of Flatland. From chipp at chipp.com Wed Jul 27 22:49:19 2011 From: chipp at chipp.com (Chipp Walters) Date: Wed, 27 Jul 2011 21:49:19 -0500 Subject: [OT] More Apple Foolishness In-Reply-To: <80001DA7-522D-4AB6-A812-501492B660BF@mac.com> References: <6F918963-BD07-4351-83D1-41E1A78D06FD@altuit.com> <00af01cc4cc3$83a64f30$8af2ed90$@com> <80001DA7-522D-4AB6-A812-501492B660BF@mac.com> Message-ID: Yep, Bill. I know. In fact, I mentioned in my first post that trying to find an app in one of those folders is like playing a kids game. That quote you reference was from Bruce, and said the bit about folders BEFORE iPad came out with folders, though it really hasn't changed much as far as ease-of-use goes. Try dragging an icon across 7 pages of OTHER icons to deposit in a folder on page 1. Ugh. If that's the best they can figure out then they need to rehire Tog. On Wed, Jul 27, 2011 at 9:43 PM, Bill Vlahos wrote: > Actually you can have folders. > > 1. Hold your finger on an icon until they all quiver > 2. Drag an icon(s) on top of another one and it will make a folder of them. > > Bill Vlahos > _________________ > InfoWallet (http://www.infowallet.com) is about keeping your important > life information with you, accessible, and secure. > > From chipp at chipp.com Wed Jul 27 22:57:28 2011 From: chipp at chipp.com (Chipp Walters) Date: Wed, 27 Jul 2011 21:57:28 -0500 Subject: [OT] More Apple Foolishness In-Reply-To: <4E30C8A7.40405@hyperactivesw.com> References: <6F918963-BD07-4351-83D1-41E1A78D06FD@altuit.com> <4E30C8A7.40405@hyperactivesw.com> Message-ID: On Wed, Jul 27, 2011 at 9:25 PM, J. Landman Gay wrote: > > > I have read more Apple documentation in the last 2 months than I have in > the last ten years. Maybe I'm just too entrenched in old habits, maybe it's > intuitive to new users. Yep, agreed. You should see the gymnastics I have to go through just to send a Keynote presentation from my iPad to another person. You can't typically send it via email because it's too large and iMail just barfs, so then you need to deposit it in a public folder in your DropBox account, but of course Apple doesn't support third party 'file managers' (even though for some damn reason they've decided NOT to have one of their own-- OOPS, I guess some actually think MobileMe was the answer!) In order to get it to DropBox you need to set up the pricey WebDav product: DropDav, then send it there, then get the URL from your DropBox account and finally send THAT to the person you're trying to share the Keynote presentation with. Oh, the other way is to 'tether' your 'Post PC iPad' to an 'old generation' PC, wait the 45 minutes or so that it forces a sync on you, then grab it from iTunes (the single most frustrating and annoying app interface-wise on the whole planet on any OS), and then email it from your PC. From capellan2000 at gmail.com Wed Jul 27 23:01:11 2011 From: capellan2000 at gmail.com (Alejandro Tejada) Date: Wed, 27 Jul 2011 20:01:11 -0700 (PDT) Subject: Where does survive the inventive user ? In-Reply-To: References: Message-ID: <1311822071516-3700292.post@n4.nabble.com> Hi Francis, Francis Nugent Dixon wrote: > > Question 1 - Is programming so easy ..... ? > I think we should ask the question "Is programming a niche occupation ? > I understand programming as an exercise in Logical thinking. Yes, I know, trust me, I know. Everyday we see so many counterexamples, that we actually doubt that "Logical" and "Thinking" are used in the same phrase anymore. My personal take is that programming IS NOT a niche occupation, given the ubiquity of computers in modern society. If for any reason, computers stop functioning in the future, the outcome would be obvious... Francis Nugent Dixon wrote: > > Programming is a mentality, and there aren't many of us who have > this mentality (even to make money from it). Although LiveCode is > a great incentive for non-programmers to "have a go" , programming > is limited to a strange mind-form which even I cannot define! > Programming should be associated with problem-solving. Just another tool for solving everyday tasks. Francis Nugent Dixon wrote: > > The question should be - "What is the VISIBILTY of LiveCode to the man > in the street who has never even thought about programming ?" > And the answer is "NONE". The chances of anybody "moving in" to > programming are about the same as being struck by lightning. > Actually this is a good visual metaphor. Instead of a lightning bulb, struck by lightning... :-D Francis Nugent Dixon wrote: > > Question 3 Will mobile computing displace desktop computing .... ? > [snip] > You can see which direction Apple is going. They want to charge > you for the use of YOUR OWN computer, and then for storing your > data in their cloud, and then for using their applications from > their cloud. That could cost you an arm and a leg. All my > communication in the hands of a stranger ? It's bad enough already! > God help us all in the future ! > > The problem is - it's not hype - it's tomorrows computing, and > I don't like the way the wind is blowing ..... > Me neither. In the name of who knows what, some "bright bulbs" would decide who, how, when and how much each one could use their "allowed" computer time... Francis Nugent Dixon wrote: > > The days when you rented an application, and you got the computer > for free may return. When computers become so dirt cheap that there > is no big profit to make, those guys "up there" have to think of a > new way to get your money. We will soon be paying more for > communication facilities than we are spending on food > (si ce n'est deja fait !, as they say here)! > Well, in some places, communications are more heavily taxed than food: 28% vs 16% Francis Nugent Dixon wrote: > > PS. How about the question "When will we be grafting micro-chips > into the brain to allow us instant and global communication, and > complex problem solving and decision making ? > Like "Neuromancer"? http://en.wikipedia.org/wiki/Sprawl_trilogy or "Ghost in the Shell"? ;-) http://en.wikipedia.org/wiki/Ghost_in_the_Shell Hopefully Not! :-D Al -- View this message in context: http://runtime-revolution.278305.n4.nabble.com/Where-does-survive-the-inventive-user-tp3698117p3700292.html Sent from the Revolution - User mailing list archive at Nabble.com. From slava at lexiconbridge.com Wed Jul 27 23:03:22 2011 From: slava at lexiconbridge.com (Slava Paperno) Date: Wed, 27 Jul 2011 23:03:22 -0400 Subject: horizontal scrollbar in revBrowser In-Reply-To: References: <00b001cc4cc9$41afa7b0$c50ef710$@com> Message-ID: <00b101cc4cd2$eca85640$c5f902c0$@com> An instant fix. Wow. Thank you, Chipp! Slava > -----Original Message----- > From: use-livecode-bounces at lists.runrev.com [mailto:use-livecode- > bounces at lists.runrev.com] On Behalf Of Chipp Walters > Sent: Wednesday, July 27, 2011 10:08 PM > To: How to use LiveCode > Subject: Re: horizontal scrollbar in revBrowser > > html {overflow-x:hidden;} > > On Wed, Jul 27, 2011 at 8:54 PM, Slava Paperno > wrote: > > > Rather like any Web browser window, my revBrowser windows sometimes > has a > > horizontal scroll bar and sometimes not. Because the display in my > > revBrowser window is wrappable text, this horizontal scroll bar is > never > > useful. Even when it appears, it only scrolls for about five or six > pixels. > > Unnecessarily. > > > > My revBrowser window is resized as the stack window is resized, and > as I > > drag the window borders to make it wider or narrower, I can always > find a > > width at which the horizontal scroll bar is not there. > > > > Is there a clever way to get rid of this pesky scroll bar? I have > complete > > control over the Web pages that are displayed in this revBrowser and > can > > add > > any style tricks to it. > > > > Slava From chipp at chipp.com Wed Jul 27 23:11:11 2011 From: chipp at chipp.com (Chipp Walters) Date: Wed, 27 Jul 2011 22:11:11 -0500 Subject: Where does survive the inventive user ? In-Reply-To: <8CE1A70D9625EA6-2550-214DE@web-mmc-d04.sysops.aol.com> References: <8CE1A70D9625EA6-2550-214DE@web-mmc-d04.sysops.aol.com> Message-ID: Craig, You make some excellent points, most all I agree with. I was one of the few who had a little experience in Fortran and Basic, but jumped at the HC opportunity-- because it was there. Today, I believe there is so much more 'there' for folks. Tremendous interactive gaming consumes some. Others see a future in learning all things web. More serious folks jump into XCode and other serious frameworks. There just isn't enough exposure to the wonders of xTalk languages except by those already converted. You are right, too, about LC being taught in every school-- but it's a huge tough sell. Perhaps if LC was open sourced it would have more of a chance? But then, how would RR get paid? Of course some Open Source apps have figured out how to have a 'free' and 'commercial' version. Still, by Open Sourcing LC, would we be dooming our favorite dev environment or guaranteeing it's success? On Wed, Jul 27, 2011 at 7:35 AM, wrote: > In the old days, Hypercard was. like a viral pandemic, infected the world > because it was bundled with every Macintosh. It was offered like a > promotion, a possibly valuable coupon one gets in the mail, which you will > at least read before throwing out, And it became a nerd fad, with many > hundreds of thousands of people trying it out. Certainly only a small > fraction became enamored; many of those are reading this post. > > > Without that once in a lifetime vehicle, it is an uphill battle to engage > people who might fall in love with LC if they only were simply exposed to > it. Worse, these days, the mindset is that everything comes in small > ready-to-go packages, complete and compact. I have three kids who just don't > think about building stuff, especially from raw materials. I used to, > though. > > > Thank the iMac, iPhone, iPod, etc., for creating that expectation, a far > cry from reading a bank of eight lights telling you what byte was currently > passing by. > > > LC should be taught in the ninth grade in every school in the world. > > > Craig Newman > > > > > > > > > > -----Original Message----- > From: Francis Nugent Dixon > To: use-livecode > Sent: Wed, Jul 27, 2011 2:42 am > Subject: Where does survive the inventive user ? > > > Alejandro asks some potent questions ..... > > Question 1 - Is programming so easy ..... ? > > I think we should ask the question "Is programming a niche occupation ? > > During my early career, practically everybody I knew was a programmer > (birds of a feather flock together ?) > > Now I am out of the industry (retired), outside of the LiveCode forum, > I don't know anybody who programs (although most of my acquaintances > have computers and Google all day, they don't WRITE programs.) > Programming is a mentality, and there aren't many of us who have > this mentality (even to make money from it). Although LiveCode is > a great incentive for non-programmers to "have a go" , programming > is limiited to a strange mind-form which even I cannot define ! > The question should be - "What is the VISIBILTY of LiveCode to the man > in the street who has never even thought about programming ?" > And the answer is "NONE". The chances of anybody "moving in" to > programming are about the same as being struck by lightning. > > Question 2 Did anyone know someone ..... ? > > I would think that buying a mobile platform (iPhone, iPad), and > learning to program are two ideas so far apart, as to be unlikely. > I do not know anyone who has started programming because of this > "mobile technology" and the platforms thereon. In a recent thread > on the forum I voiced my "No Way Baby" intent to NOT go to mobile > computing, although I wrote my first program more than 50 years > ago, and I still program EVERY day. The cost is far too high, and > the returns are doubtful ! I am a rare bird who now programs for fun. > > Question 3 Will mobile computing displace desktop computing .... ? > > This reminds me of the 1980's question "Will Desktop computing ever > displace Mainframe Computing ?" A lot of people said no in them days. > > Industry has been talking about "cloud computing" for years, and > IS slowly moving towards it. But, although I may accept having my > data in a cloud, I will always want my apps to be in my hand, so > I can have control over them. Renting application use out of a > cloud would be the same has handing over your wallet to a stranger. > You can see which direction Apple is going. They want to charge > you for the use of YOUR OWN computer, and then for storing your > data in their cloud, and then for using their applications from > their cloud. That could cost you an arm and a leg. All my > communication in the hands of a stranger ? It's bad enough already! > God help us all in the future ! > > The problem is - it's not hype - it's tomorrows computing, and > I don't like the way the wind is blowing ..... > > The days when you rented an application, and you got the computer > for free may return. When computers become so dirt cheap that there > is no big profit to make, those guys "up there" have to think of a > new way to get your money. We will soon be paying more for > communication facilities than we are spending on food > (si ce n'est deja fait !, as they say here)! > > "Nothing should ever be done for the first time !" > > -Francis > > PS. How about the question "When will we be grafting micro-chips > into the brain to allow us instant and global communication, and > complex problem solving and decision making ? > > > _______________________________________________ > use-livecode mailing list > use-livecode at lists.runrev.com > Please visit this url to subscribe, unsubscribe and manage your > subscription > preferences: > http://lists.runrev.com/mailman/listinfo/use-livecode > > > > _______________________________________________ > use-livecode mailing list > use-livecode at lists.runrev.com > Please visit this url to subscribe, unsubscribe and manage your > subscription preferences: > http://lists.runrev.com/mailman/listinfo/use-livecode > -- Chipp Walters CEO, Shafer Walters Group, Inc. From bornstein at designeq.com Wed Jul 27 23:40:46 2011 From: bornstein at designeq.com (Howard Bornstein) Date: Wed, 27 Jul 2011 20:40:46 -0700 Subject: [OT] More Apple Foolishness In-Reply-To: References: <6F918963-BD07-4351-83D1-41E1A78D06FD@altuit.com> <00af01cc4cc3$83a64f30$8af2ed90$@com> <80001DA7-522D-4AB6-A812-501492B660BF@mac.com> Message-ID: There are a couple of apps that deal with these deficiencies very intuitively: Multi-icon mover, where you put the apps in wiggle mode and then tap which ever apps you want to move, even if they are on different home screens. Then just go to the page or folder where you want them moved to and hit the home button and they are all moved there at once. Sprintboard, which lets you drag your fingers across the home page dots to quickly scroll through the home pages. These apps make it very easy to move icons around different pages and into folders. The rub is that you have to have a jailbroken iphone/ipad to use them. Let's hope that Apple continues their habit of (slowly) copying jailbreak innovation and eventually folding them into the IOS. On Wed, Jul 27, 2011 at 7:49 PM, Chipp Walters wrote: > Yep, Bill. I know. In fact, I mentioned in my first post that trying to > find > an app in one of those folders is like playing a kids game. That quote you > reference was from Bruce, and said the bit about folders BEFORE iPad came > out with folders, though it really hasn't changed much as far as > ease-of-use > goes. Try dragging an icon across 7 pages of OTHER icons to deposit in a > folder on page 1. Ugh. If that's the best they can figure out then they > need > to rehire Tog. > > On Wed, Jul 27, 2011 at 9:43 PM, Bill Vlahos wrote: > > > Actually you can have folders. > > > > 1. Hold your finger on an icon until they all quiver > > 2. Drag an icon(s) on top of another one and it will make a folder of > them. > > > > Bill Vlahos > > _________________ > > InfoWallet (http://www.infowallet.com) is about keeping your important > > life information with you, accessible, and secure. > > > > > _______________________________________________ > use-livecode mailing list > use-livecode at lists.runrev.com > Please visit this url to subscribe, unsubscribe and manage your > subscription preferences: > http://lists.runrev.com/mailman/listinfo/use-livecode > -- Regards, Howard Bornstein ----------------------- www.designeq.com From capellan2000 at gmail.com Wed Jul 27 23:41:15 2011 From: capellan2000 at gmail.com (Alejandro Tejada) Date: Wed, 27 Jul 2011 20:41:15 -0700 (PDT) Subject: [OT] More Apple Foolishness In-Reply-To: References: <6F918963-BD07-4351-83D1-41E1A78D06FD@altuit.com> <00af01cc4cc3$83a64f30$8af2ed90$@com> Message-ID: <1311824475377-3700339.post@n4.nabble.com> Hi Chipp, When I started to use Macintosh, back in the nineties, consistency in User interface conventions was a real godsend. But, after a while, Apple started looking in the mirror and started to comparing themselves with Windows: Magazines, Promotional Brochures, Ads... Wherever you see Apple, there was Windows too. Invited by Apple themselves. As a result of these detailed comparisions, I learned to use Windows 95 and 98 more efficiently than new Windows users. After all that unexpected training, transition to Windows was easy, thanks to an insane Apple desire to compare themselves (and win) against Windows. When Apple starts to compare themselves with Android, you will know that the writing is in the wall. Al -- View this message in context: http://runtime-revolution.278305.n4.nabble.com/OT-More-Apple-Foolishness-tp3700154p3700339.html Sent from the Revolution - User mailing list archive at Nabble.com. From tsj at unimelb.edu.au Wed Jul 27 23:46:22 2011 From: tsj at unimelb.edu.au (Terry Judd) Date: Thu, 28 Jul 2011 03:46:22 +0000 Subject: Unzipping Mac executable files with revZip - revisited Message-ID: <2DBD6DE8-CDED-4DCE-98A9-1D1A3902D5BB@unimelb.edu.au> About 3 weeks ago I posted about an issue I had with certain Mac files (those that are recognised by the MacOS as 'unix executable files') losing their executable status when they are unpacked using revZip. A couple of solutions were offered and I'm currently using 'chmod +x [filepath]' to 'reset' these files. However, I still have a problem in that I'm creating a cross-platform installer/updater that will be writing out (from a zip archive) both Win and Mac apps/externals/etc at the same time - our software is delivered on a USB memory stick and includes cross-platform versions of a number of educational apps. If the user installs/updates on a Mac then the chmod solution works perfectly. If however they are installing/updating under Windows then there doesn't seem to be any way for me to 'reset' any Mac executable files that have been updated so that they will function correctly the next time the user uses the software on a Mac (most of our users have PCs but most of our computer labs have Macs). Or is there a way that I'm unaware of (a Win equivalent of chmod)? The only idea I have at the moment is to temporarily store a list of files that need to be reset when the user does their install/update on a PC and then use this (and clear it afterwards) to do some housekeeping the next time they start up the software on a Mac. Any other (better) suggestions out there? Regards, Terry... From tsj at unimelb.edu.au Thu Jul 28 00:32:31 2011 From: tsj at unimelb.edu.au (Terry Judd) Date: Thu, 28 Jul 2011 04:32:31 +0000 Subject: Unzipping Mac executable files with revZip - revisited In-Reply-To: <2DBD6DE8-CDED-4DCE-98A9-1D1A3902D5BB@unimelb.edu.au> References: <2DBD6DE8-CDED-4DCE-98A9-1D1A3902D5BB@unimelb.edu.au> Message-ID: <016257E4-3151-4E93-B521-FD92CC810064@unimelb.edu.au> OK - scratch that quey. It looks like using revZip to unzip Mac apps on a PC is ok after all. From what I can tell (following admittedly limited testing) any file without an extension that is unzipped on a PC ends up being recognised as a unix executable file on the Mac. Bottom line is that Mac apps unzipped on a PC using revZip still work on a Mac. Kinda ironic when Mac apps unzipped on a Mac using revZip don't. Terry... On 28/07/2011, at 01:46 PM, Terry Judd wrote: > About 3 weeks ago I posted about an issue I had with certain Mac files (those that are recognised by the MacOS as 'unix executable files') losing their executable status when they are unpacked using revZip. A couple of solutions were offered and I'm currently using 'chmod +x [filepath]' to 'reset' these files. > > However, I still have a problem in that I'm creating a cross-platform installer/updater that will be writing out (from a zip archive) both Win and Mac apps/externals/etc at the same time - our software is delivered on a USB memory stick and includes cross-platform versions of a number of educational apps. > > If the user installs/updates on a Mac then the chmod solution works perfectly. If however they are installing/updating under Windows then there doesn't seem to be any way for me to 'reset' any Mac executable files that have been updated so that they will function correctly the next time the user uses the software on a Mac (most of our users have PCs but most of our computer labs have Macs). Or is there a way that I'm unaware of (a Win equivalent of chmod)? > > The only idea I have at the moment is to temporarily store a list of files that need to be reset when the user does their install/update on a PC and then use this (and clear it afterwards) to do some housekeeping the next time they start up the software on a Mac. Any other (better) suggestions out there? > > Regards, > > Terry... > > _______________________________________________ > use-livecode mailing list > use-livecode at lists.runrev.com > Please visit this url to subscribe, unsubscribe and manage your subscription preferences: > http://lists.runrev.com/mailman/listinfo/use-livecode > From stephenREVOLUTION2 at barncard.com Thu Jul 28 00:38:57 2011 From: stephenREVOLUTION2 at barncard.com (stephen barncard) Date: Wed, 27 Jul 2011 21:38:57 -0700 Subject: windows revserver stack support In-Reply-To: References: Message-ID: Yeah, fields - tried adding one - and then I couldn't access the stack unless I deleted the field in the IDE. But granted the only reason I might want a field in an online stack is to do some text conversion that requires a 'field'. sqb On 27 July 2011 15:53, Mike Bonner wrote: > Fields work, you can place pretty much every object EXCEPT a button on > there > and it works. > Stephen Barncard San Francisco Ca. USA more about sqb From stephenREVOLUTION2 at barncard.com Thu Jul 28 00:49:32 2011 From: stephenREVOLUTION2 at barncard.com (stephen barncard) Date: Wed, 27 Jul 2011 21:49:32 -0700 Subject: Livecode Server 4.6.3 - First impressions pt2 - using the new version at Dreamhost In-Reply-To: References: Message-ID: I had no intention of using CPs in stacks that way as a large db -I really thinking about snippets - boilerplate text, etc. and using the CP hierarchy to accomplish that. Databases are still king when it comes to large amounts of data, but I was thinking more in terms of 'business logic' and keeping configuration data from prying eyes. But the thought of going from a CP to an array - or vice versa - easily and quickly appeals to me. What size, Andre, would constitute a 'large stack' that would cause a problem? On 27 July 2011 17:03, Andre Garzia wrote: > Stephen, > > be aware that loading large stacks into memory will get you to the memory > protection police! If your database stack is too big, and lots of copies > are > loaded for all the requests, Apache or whoever does the purging might kill > your engine to save resources. This is hardcoded into On-Rev accounts. The > engine on dreamhost should not have these limits but still, dreamhost is a > shared server and they will probably kill processess that starte the server > of its resources. > > Cheers > andre > > On Wed, Jul 27, 2011 at 7:38 PM, stephen barncard < > stephenREVOLUTION2 at barncard.com> wrote: > > > *LIVECODE SERVER 4.6.3* > > > > Wow. Haven't done any speed benchmarks, and I've just started using this > > version of LCS, but it offers what was missing before, and a lot more. > Stephen Barncard San Francisco Ca. USA more about sqb From shaosean at wehostmacs.com Thu Jul 28 00:58:41 2011 From: shaosean at wehostmacs.com (Shao Sean) Date: Thu, 28 Jul 2011 00:58:41 -0400 Subject: windows revserver stack support Message-ID: <519A309C-1919-45FA-B67A-BFAD4FDAB3F7@wehostmacs.com> In some old stack libraries I would use the templateField to do my field trickery without having to have a field object present, perhaps that will help you as well.. From lists.pete at haworths.org Thu Jul 28 02:08:17 2011 From: lists.pete at haworths.org (Pete Haworth) Date: Wed, 27 Jul 2011 23:08:17 -0700 Subject: Unzipping Mac executable files with revZip - revisited In-Reply-To: <016257E4-3151-4E93-B521-FD92CC810064@unimelb.edu.au> References: <2DBD6DE8-CDED-4DCE-98A9-1D1A3902D5BB@unimelb.edu.au> <016257E4-3151-4E93-B521-FD92CC810064@unimelb.edu.au> Message-ID: I ran into this once with Time Machine. I restored an iPages file from a backup and it brought it back without an extension and treated it as a unix executable. Even more ironic that Time Machine can't restore a file from an Apple application properly! Pete On Wed, Jul 27, 2011 at 9:32 PM, Terry Judd wrote: > OK - scratch that quey. It looks like using revZip to unzip Mac apps on a > PC is ok after all. From what I can tell (following admittedly limited > testing) any file without an extension that is unzipped on a PC ends up > being recognised as a unix executable file on the Mac. Bottom line is that > Mac apps unzipped on a PC using revZip still work on a Mac. Kinda ironic > when Mac apps unzipped on a Mac using revZip don't. > > Terry... > > > On 28/07/2011, at 01:46 PM, Terry Judd wrote: > > > About 3 weeks ago I posted about an issue I had with certain Mac files > (those that are recognised by the MacOS as 'unix executable files') losing > their executable status when they are unpacked using revZip. A couple of > solutions were offered and I'm currently using 'chmod +x [filepath]' to > 'reset' these files. > > > > However, I still have a problem in that I'm creating a cross-platform > installer/updater that will be writing out (from a zip archive) both Win and > Mac apps/externals/etc at the same time - our software is delivered on a > USB memory stick and includes cross-platform versions of a number of > educational apps. > > > > If the user installs/updates on a Mac then the chmod solution works > perfectly. If however they are installing/updating under Windows then there > doesn't seem to be any way for me to 'reset' any Mac executable files that > have been updated so that they will function correctly the next time the > user uses the software on a Mac (most of our users have PCs but most of our > computer labs have Macs). Or is there a way that I'm unaware of (a Win > equivalent of chmod)? > > > > The only idea I have at the moment is to temporarily store a list of > files that need to be reset when the user does their install/update on a PC > and then use this (and clear it afterwards) to do some housekeeping the next > time they start up the software on a Mac. Any other (better) suggestions out > there? > > > > Regards, > > > > Terry... > > > > _______________________________________________ > > use-livecode mailing list > > use-livecode at lists.runrev.com > > Please visit this url to subscribe, unsubscribe and manage your > subscription preferences: > > http://lists.runrev.com/mailman/listinfo/use-livecode > > > > > > _______________________________________________ > use-livecode mailing list > use-livecode at lists.runrev.com > Please visit this url to subscribe, unsubscribe and manage your > subscription preferences: > http://lists.runrev.com/mailman/listinfo/use-livecode > From Camm29 at tesco.net Thu Jul 28 02:30:46 2011 From: Camm29 at tesco.net (Camm) Date: Thu, 28 Jul 2011 07:30:46 +0100 Subject: Tahoma Font In-Reply-To: <7718C7D1-010C-412B-B60C-9C94D32E893D@altuit.com> References: <000001cc4bd2$cd1e4470$675acd50$@net> <000f01cc4c3c$3b747b60$b25d7220$@de> <000c01cc4c85$b0b7ab10$12270130$@net> <7718C7D1-010C-412B-B60C-9C94D32E893D@altuit.com> Message-ID: <000c01cc4cef$e294f790$a7bee6b0$@net> Chipp , Perfect ........................ Many thanks ! Regards Camm -----Original Message----- From: use-livecode-bounces at lists.runrev.com [mailto:use-livecode-bounces at lists.runrev.com] On Behalf Of Chipp Walters Sent: 28 July 2011 02:00 To: How to use LiveCode Subject: Re: Tahoma Font Camm, Open up your stack in Rev as Topstack and before you make a standalone, type this in the message box: set the textfont of the topstack to "Tahoma" Then press enter and save your stack. Now make a standalone and see if it doesn't work. If Tahoma is installed on a Windows users machine, it will be used for all the "effective" textfont styles. Chipp Walters CEO, Shafer Walters Group, Inc On Jul 27, 2011, at 12:50 PM, "Camm" wrote: > Well , my issue is just with windows ? > The Tahoma font is their during development mode but vanishes in Standalone > on XP. > If I set the properties of the standalone.exe in Windows to 95 or 98 > compatibility it returns ? > > How do I force load a font on startup ? > > Best Regards > Camm > > > -----Original Message----- > From: use-livecode-bounces at lists.runrev.com > [mailto:use-livecode-bounces at lists.runrev.com] On Behalf Of Tiemo Hollmann > TB > Sent: 27 July 2011 10:05 > To: 'How to use LiveCode' > Subject: AW: Tahoma Font > > When I started with revolution Tahoma was the default font of all objects > and I didn't cared about it at that time. I am now using Tahoma since years > without any problems on windows and Mac, though I have learned that Tahoma > isn't a standard font on Mac and is supposed to be substituted (what never > made any problems in my apps). On windows systems Tahoma is a standard font > up to today as far as I know, or did I miss anything? > Tiemo > >> -----Urspr?ngliche Nachricht----- >> Von: use-livecode-bounces at lists.runrev.com [mailto:use-livecode- >> bounces at lists.runrev.com] Im Auftrag von Camm >> Gesendet: Dienstag, 26. Juli 2011 22:30 >> An: use-livecode at lists.runrev.com >> Betreff: Tahoma Font >> >> Tahoma font will not work in standalone unless its run in Windows 95 or 98 >> compatibility mode? >> >> >> >> I am using Windows XP...... Any ideas ? >> >> >> >> Best Regards >> >> Camm >> >> _______________________________________________ >> use-livecode mailing list >> use-livecode at lists.runrev.com >> Please visit this url to subscribe, unsubscribe and manage your > subscription >> preferences: >> http://lists.runrev.com/mailman/listinfo/use-livecode > > > _______________________________________________ > use-livecode mailing list > use-livecode at lists.runrev.com > Please visit this url to subscribe, unsubscribe and manage your subscription > preferences: > http://lists.runrev.com/mailman/listinfo/use-livecode > ----- > No virus found in this message. > Checked by AVG - www.avg.com > Version: 10.0.1390 / Virus Database: 1518/3789 - Release Date: 07/26/11 > > > _______________________________________________ > use-livecode mailing list > use-livecode at lists.runrev.com > Please visit this url to subscribe, unsubscribe and manage your subscription preferences: > http://lists.runrev.com/mailman/listinfo/use-livecode _______________________________________________ use-livecode mailing list use-livecode at lists.runrev.com Please visit this url to subscribe, unsubscribe and manage your subscription preferences: http://lists.runrev.com/mailman/listinfo/use-livecode ----- No virus found in this message. Checked by AVG - www.avg.com Version: 10.0.1390 / Virus Database: 1518/3791 - Release Date: 07/27/11 From toolbook at kestner.de Thu Jul 28 02:41:57 2011 From: toolbook at kestner.de (Tiemo Hollmann TB) Date: Thu, 28 Jul 2011 08:41:57 +0200 Subject: AW: OT: I want to buy Lion In-Reply-To: <9308DB19-829D-41A0-A887-B2F59C1C4CF4@mac.com> References: <001801cc4c80$41078b00$c316a100$@de> <8C7BA5AA-EDE0-4817-BB42-E8C01C7D4FDE@twft.com> <002101cc4c87$4f71dd60$ee559820$@de> <9308DB19-829D-41A0-A887-B2F59C1C4CF4@mac.com> Message-ID: <001101cc4cf1$75d822b0$61886810$@de> Hi Arie, noop, that's just the annoying thing. I can't even open the MacAppStore on Leopard, no chance. Even if Apple wants to switch to this new MacApp Store thing for the future, I don't understand why they don't offer anymore any "classic download" option for backward compatibility for users who are not up to date every day like me. Tiemo > -----Urspr?ngliche Nachricht----- > Von: use-livecode-bounces at lists.runrev.com [mailto:use-livecode- > bounces at lists.runrev.com] Im Auftrag von Arie van der Ent > Gesendet: Mittwoch, 27. Juli 2011 20:20 > An: How to use LiveCode > Betreff: Re: OT: I want to buy Lion > > Hi Tiemo, > > It is not necessary to wait for Lion on a USB-stick. Don McAllister from > screencastsonline did the following; > 1. buy Lion from App store; > 2. make a copy from the installer (in your application folder); > 3. open up the copy of the installer package, then open the folder Shared > Support in it; > 4. there you'll find InstallEDG.dmg; > 5. copy this file to an USB-stick 8 gb. > > That's all. > > Arie > From gcanyon+rev at gmail.com Thu Jul 28 02:51:09 2011 From: gcanyon+rev at gmail.com (Geoff Canyon Rev) Date: Thu, 28 Jul 2011 01:51:09 -0500 Subject: [OT] More Apple Foolishness In-Reply-To: References: <6F918963-BD07-4351-83D1-41E1A78D06FD@altuit.com> <00af01cc4cc3$83a64f30$8af2ed90$@com> <80001DA7-522D-4AB6-A812-501492B660BF@mac.com> Message-ID: I used to do this. Then I realized that it was quicker (if more complex and less intuitive) to: 1. Drag something off the shelf at the bottom of the screen if necessary to make room 2. Drag the item I want to move onto the shelf 3. Go to the home screen, either by swiping (easier than dragging) or by dropping out of rearrange mode, clicking the home button, and going back into rearrange mode 4. Drag the item I want to move onto screen 1 5. If step 1 was necessary, go back to the original screen and put the item I dragged off the shelf back onto the shelf. I agree, this aspect of the UI is not optimal. On Wed, Jul 27, 2011 at 9:49 PM, Chipp Walters wrote: > Try dragging an icon across 7 pages of OTHER icons to deposit in a > folder on page 1. From warren at warrensweb.us Thu Jul 28 03:26:10 2011 From: warren at warrensweb.us (Warren Samples) Date: Thu, 28 Jul 2011 02:26:10 -0500 Subject: AW: OT: I want to buy Lion In-Reply-To: <001101cc4cf1$75d822b0$61886810$@de> References: <001801cc4c80$41078b00$c316a100$@de> <9308DB19-829D-41A0-A887-B2F59C1C4CF4@mac.com> <001101cc4cf1$75d822b0$61886810$@de> Message-ID: <201107280226.10849.warren@warrensweb.us> On Thursday, July 28, 2011 01:41:57 AM Tiemo Hollmann TB wrote: > I don't understand why they don't offer anymore any > "classic download" option for backward compatibility for users who are not > up to date every day like me. > Tiemo They're not interested in helping you to maintain your "backward" status. Apple wants its users up to date, with the program, and feeding them money through iTunes and the App Store. Best, Warren From toolbook at kestner.de Thu Jul 28 03:45:23 2011 From: toolbook at kestner.de (Tiemo Hollmann TB) Date: Thu, 28 Jul 2011 09:45:23 +0200 Subject: AW: AW: OT: I want to buy Lion In-Reply-To: <201107280226.10849.warren@warrensweb.us> References: <001801cc4c80$41078b00$c316a100$@de> <9308DB19-829D-41A0-A887-B2F59C1C4CF4@mac.com> <001101cc4cf1$75d822b0$61886810$@de> <201107280226.10849.warren@warrensweb.us> Message-ID: <001a01cc4cfa$5347c850$f9d758f0$@de> Yep, thats obviously their mind Tiemo > > > They're not interested in helping you to maintain your "backward" status. > Apple wants its users up to date, > with the program, and feeding them money through iTunes and the App Store. > > Best, > > Warren > From andre.bisseret at wanadoo.fr Thu Jul 28 03:41:11 2011 From: andre.bisseret at wanadoo.fr (=?iso-8859-1?Q?Andr=E9_Bisseret?=) Date: Thu, 28 Jul 2011 09:41:11 +0200 Subject: A case of assigned behavior not taken into account In-Reply-To: References: <9EC1084F-AC69-492B-A2C6-1658129FF052@wanadoo.fr> Message-ID: Hi Bob, You are right ; i verified that when loading the application from one computer to another (from Mac to PC and vice versa) I had to reassign the behavior button So I am keeping doing this reassignment. Thank you very much for your attention and explanation. Andr? Le 27 juil. 2011 ? 18:37, Bob Sneidar a ?crit : > I will take a shot at this. Behaviors are actually the script of a button, referenced as it's long ID. The long ID (as you could see if you got the long ID of any object) references not just the card it is on but the stack itself. When you clone a stack with behaviors, I suspect that the behaviors are still using the reference to the long ID of the button in the template stack. You will have to change that by script as you suspect. > > The reason this is like that is because you would want a modified behavior script to affect all object that use it in your entire application. It's just a little bit like OOP for programming. If you wanted the behavior to be altered a bit for certain objects, you could either intercept the message in the object's script, do what is different, then optionally pass it, or you could create a new button which was a copy of the behavior button and assign the object's behavior to that. But obviously you would lose the "one edit fixes all" for that button. > > Bob > > > On Jul 27, 2011, at 4:41 AM, Andr? Bisseret wrote: > >> Bonjour, >> >> On an app. I am developing on Mac, I have a main stack a substack of which is a model used for creating new stacks which are cloned from the model and saved as "independent" stacks (not substacks). >> >> The scripts of the card 1 of this model and of all objects on this card are all together in a behavior button which is assigned to this card 1. >> This behavior is on card 2 of the main stack. >> >> All is working well on Mac. >> >> But when I load the standalone for Windows on a PC (by means of a USB key) then a newly created stack from the model is inert. >> Meanwhile, I verified that the behavior is actually assigned to the card 1 of the new stack, but all behaves like this was not the case! >> >> If, by script, I reassign the behavior to card 1 of the model before cloning it, then the new stack is working as expected. >> >> So I could stay with this reassignment but? >> >> Is it normal (seems not to me!), or am I missing something ? >> >> Any hint much appreciated >> >> Best regards from Grenoble >> >> Andr? >> >> >> _______________________________________________ >> use-livecode mailing list >> use-livecode at lists.runrev.com >> Please visit this url to subscribe, unsubscribe and manage your subscription preferences: >> http://lists.runrev.com/mailman/listinfo/use-livecode > > > _______________________________________________ > use-livecode mailing list > use-livecode at lists.runrev.com > Please visit this url to subscribe, unsubscribe and manage your subscription preferences: > http://lists.runrev.com/mailman/listinfo/use-livecode From coiin at verizon.net Thu Jul 28 05:01:25 2011 From: coiin at verizon.net (Colin Holgate) Date: Thu, 28 Jul 2011 10:01:25 +0100 Subject: [OT] More Apple Foolishness In-Reply-To: References: <6F918963-BD07-4351-83D1-41E1A78D06FD@altuit.com> <00af01cc4cc3$83a64f30$8af2ed90$@com> <80001DA7-522D-4AB6-A812-501492B660BF@mac.com> Message-ID: <76870884-CAEE-4396-B191-B2C540ACA93B@verizon.net> I hardly ever use iTunes for getting things onto my devices, but if you are an iTunes user you can move app icons from any of the screens to any other of the screes. Needs a bit of scrolling if you have 11 screen's worth like I do, but at least you can get everything in the order you want it, then sync your device. From m.schonewille at economy-x-talk.com Thu Jul 28 05:01:59 2011 From: m.schonewille at economy-x-talk.com (Mark Schonewille) Date: Thu, 28 Jul 2011 11:01:59 +0200 Subject: Unzipping Mac executable files with revZip - revisited In-Reply-To: <2DBD6DE8-CDED-4DCE-98A9-1D1A3902D5BB@unimelb.edu.au> References: <2DBD6DE8-CDED-4DCE-98A9-1D1A3902D5BB@unimelb.edu.au> Message-ID: <0C519216-2678-45BE-A40B-CA1421002A69@economy-x-talk.com> Hi Terry, Do you know about Economy-x-Talk's installer? That should solve all those problems at once, although I wouldn't install Mac software under Windows. You can find it here http://qery.us/ce -- Best regards, Mark Schonewille Economy-x-Talk Consulting and Software Engineering Homepage: http://economy-x-talk.com Twitter: http://twitter.com/xtalkprogrammer KvK: 50277553 What does that error mean? Buy LiveCodeErrors for iPhone now http://qery.us/v4 A must-have for LiveCode programmers. On 28 jul 2011, at 05:46, Terry Judd wrote: > About 3 weeks ago I posted about an issue I had with certain Mac files (those that are recognised by the MacOS as 'unix executable files') losing their executable status when they are unpacked using revZip. A couple of solutions were offered and I'm currently using 'chmod +x [filepath]' to 'reset' these files. > > However, I still have a problem in that I'm creating a cross-platform installer/updater that will be writing out (from a zip archive) both Win and Mac apps/externals/etc at the same time - our software is delivered on a USB memory stick and includes cross-platform versions of a number of educational apps. > > If the user installs/updates on a Mac then the chmod solution works perfectly. If however they are installing/updating under Windows then there doesn't seem to be any way for me to 'reset' any Mac executable files that have been updated so that they will function correctly the next time the user uses the software on a Mac (most of our users have PCs but most of our computer labs have Macs). Or is there a way that I'm unaware of (a Win equivalent of chmod)? > > The only idea I have at the moment is to temporarily store a list of files that need to be reset when the user does their install/update on a PC and then use this (and clear it afterwards) to do some housekeeping the next time they start up the software on a Mac. Any other (better) suggestions out there? > > Regards, > > Terry... > > _______________________________________________ > use-livecode mailing list > use-livecode at lists.runrev.com > Please visit this url to subscribe, unsubscribe and manage your subscription preferences: > http://lists.runrev.com/mailman/listinfo/use-livecode From capellan2000 at gmail.com Thu Jul 28 06:46:48 2011 From: capellan2000 at gmail.com (Alejandro Tejada) Date: Thu, 28 Jul 2011 03:46:48 -0700 (PDT) Subject: Where does survive the inventive user ? In-Reply-To: References: <8CE1A70D9625EA6-2550-214DE@web-mmc-d04.sysops.aol.com> Message-ID: <1311850008331-3701012.post@n4.nabble.com> Hi Chipp, Could help if you could use Livecode inside OpenOffice as scripting language, just like they use Python? Al -- View this message in context: http://runtime-revolution.278305.n4.nabble.com/Where-does-survive-the-inventive-user-tp3698117p3701012.html Sent from the Revolution - User mailing list archive at Nabble.com. From tsj at unimelb.edu.au Thu Jul 28 06:56:29 2011 From: tsj at unimelb.edu.au (Terry Judd) Date: Thu, 28 Jul 2011 10:56:29 +0000 Subject: Unzipping Mac executable files with revZip - revisited In-Reply-To: <0C519216-2678-45BE-A40B-CA1421002A69@economy-x-talk.com> References: <2DBD6DE8-CDED-4DCE-98A9-1D1A3902D5BB@unimelb.edu.au> <0C519216-2678-45BE-A40B-CA1421002A69@economy-x-talk.com> Message-ID: On 28/07/2011, at 07:01 PM, Mark Schonewille wrote: > Hi Terry, > > Do you know about Economy-x-Talk's installer? That should solve all those problems at once, although I wouldn't install Mac software under Windows. You can find it here http://qery.us/ce > Thanks Mark - I think we're OK with our bespoke solution for our current project but I'll keep your installer in mind for future projects. Regards, Terry... > -- > Best regards, > > Mark Schonewille > > Economy-x-Talk Consulting and Software Engineering > Homepage: http://economy-x-talk.com > Twitter: http://twitter.com/xtalkprogrammer > KvK: 50277553 > > What does that error mean? Buy LiveCodeErrors for iPhone now http://qery.us/v4 A must-have for LiveCode programmers. > > On 28 jul 2011, at 05:46, Terry Judd wrote: > >> About 3 weeks ago I posted about an issue I had with certain Mac files (those that are recognised by the MacOS as 'unix executable files') losing their executable status when they are unpacked using revZip. A couple of solutions were offered and I'm currently using 'chmod +x [filepath]' to 'reset' these files. >> >> However, I still have a problem in that I'm creating a cross-platform installer/updater that will be writing out (from a zip archive) both Win and Mac apps/externals/etc at the same time - our software is delivered on a USB memory stick and includes cross-platform versions of a number of educational apps. >> >> If the user installs/updates on a Mac then the chmod solution works perfectly. If however they are installing/updating under Windows then there doesn't seem to be any way for me to 'reset' any Mac executable files that have been updated so that they will function correctly the next time the user uses the software on a Mac (most of our users have PCs but most of our computer labs have Macs). Or is there a way that I'm unaware of (a Win equivalent of chmod)? >> >> The only idea I have at the moment is to temporarily store a list of files that need to be reset when the user does their install/update on a PC and then use this (and clear it afterwards) to do some housekeeping the next time they start up the software on a Mac. Any other (better) suggestions out there? >> >> Regards, >> >> Terry... >> >> _______________________________________________ >> use-livecode mailing list >> use-livecode at lists.runrev.com >> Please visit this url to subscribe, unsubscribe and manage your subscription preferences: >> http://lists.runrev.com/mailman/listinfo/use-livecode > > > _______________________________________________ > use-livecode mailing list > use-livecode at lists.runrev.com > Please visit this url to subscribe, unsubscribe and manage your subscription preferences: > http://lists.runrev.com/mailman/listinfo/use-livecode > From psahores at free.fr Thu Jul 28 07:30:40 2011 From: psahores at free.fr (Pierre Sahores) Date: Thu, 28 Jul 2011 13:30:40 +0200 Subject: Where does survive the inventive user? In-Reply-To: References: <1311706987455-3696711.post@n4.nabble.com> Message-ID: <08F3632C-76A9-402E-97B4-C82090DC8D05@free.fr> Dear All, Computer programming is born with the talent of a few designers capable of creating "paterns" accessible to smaller processors. That was more than fifty years ago and any computer program was running in the pure logic of a Turing machine. In the late 80's, Apple, Oracle, IBM and others were afraid that the increased performance of hardware and ultra high level programming languages ??gives rise to a breed of ultra creative designers and developers capable to break their market control and distribution software rules. They fight since then to manipulate the developers and lock them into the roles of pure technical performers. They fear that developers become production program artists and "writers", free of their intentions and fear the creative quality and performance results of their achievements. I sincerely believe that their desire to control the freedom of initiative and economy of means - that is dear to all those who have already understood that the technique is a simple tool for creativity - will fail like all the idiot strategies whose, before them, thought they would manipulate for their own interests only the market of painting, literature and music by controlling the manufacture of brushes, paper production and ownership of concert halls. They sought to turn away the best functional programming ??and procedural languages in trying to intoxicant us with the supposed superiority of the sterile logic of the UML and object-oriented programming methodologies. I truly believe they will soon fail and that cloud computing is one of the last rounds they seize to lock consumers and developers in their net monopolistic business as bankers try to lock in sheep production the yoke of the proletarianization of the agricultural world. They force us to realize that the ways we through the web and are sharing our knowledge with each other helps us all to become designers and artists of the information age. There is too much to be done for supporters of the lowest common denominator of market control by the monopolistic structure of supply to prevent us do as we please in the interest of any particular customer. The cloud is just a hollow phishing marketing idea for lambda. Saas and Web development is fortunately too rich and no one needs to prevent us from making it the largest territory of conceptual and creative freedom. We have to be proud to position ourselves away from all attempts at market manipulation as the writers of the information economy. The global economic crisis is our ally. Customers also reflect and begin to understand what we can offer them by selling or renting them the information systems they need rather than selling their prices and software that stretch their budgets without ever reaching their needs for the next five years. Programming for the Web with LiveCode desktop, LiveCode server and the LiveCode web plugin, with SunnYperl (Unicode, SSL,Oracle,...), with RevIgniter, with the open-source DB, demons, etc... and all those wonderful libraries and methodologies that we share since the first steps of the xTalk programming birth makes us very special birds. Thank you All. Thank you for continuing to work and act in a spirit very similar which animates our colleagues of the open source community. Apple, Oracle, IBM or Microsoft don't have any interest to oppose to those who, through their ideas and generosity, are more than ever, working to develop the economic models of the post-crisis information age. Friendly yours, Pierre PS : When you says, Andre, that "mobile computing is consumer computing." i can just applaud and i hope that our sweet mothership will invest and become stronger and stronger over the years because the LC desktop and server products line + associated services. Le 26 juil. 2011 ? 21:08, Andre Garzia a ?crit : > mobile computing is consumer computing. Developers and inventive users will > keep on platforms that allow them to develop stuff. It means that slowly, > those users will move towards freedom so even though mobile computing will > be ubiquitous, you will find the developers and inventive users using > something else where they can actually develop stuff unrestricted. They will > probably be on linux... > > On Tue, Jul 26, 2011 at 4:03 PM, Alejandro Tejada wrote: > >> Hi all, >> >> Today, I read again this article by Dan Shafer: >> http://www.danshaferblog.com/inventive-users-need-help-on-the-ithings >> >> Many obvious questions arise from this article: >> Does latest versions of Livecode fill this niche? >> >> Is programming for mobile so easy (using Livecode) >> that anyone that wants to, could do it? >> >> Did anyone here knows someone who actually started >> learning programming after buying one of the >> mobile platforms? >> >> Today, I woke with an strange idea: Mobile computing >> will displace desktop computing for most everyday >> computing tasks in a really short time (5 to 10 years). >> >> Tell me if this idea has a real basis or is just an echo of >> the hype that surrounds the latest products. >> >> Thanks in advance! >> >> Al >> >> -- >> View this message in context: >> http://runtime-revolution.278305.n4.nabble.com/Where-does-survive-the-inventive-user-tp3696711p3696711.html >> Sent from the Revolution - User mailing list archive at Nabble.com. >> >> _______________________________________________ >> use-livecode mailing list >> use-livecode at lists.runrev.com >> Please visit this url to subscribe, unsubscribe and manage your >> subscription preferences: >> http://lists.runrev.com/mailman/listinfo/use-livecode >> > > > > -- > http://www.andregarzia.com All We Do Is Code. > _______________________________________________ > use-livecode mailing list > use-livecode at lists.runrev.com > Please visit this url to subscribe, unsubscribe and manage your subscription preferences: > http://lists.runrev.com/mailman/listinfo/use-livecode > -- Pierre Sahores mobile : (33) 6 03 95 77 70 www.woooooooords.com www.sahores-conseil.com From mikekann at yahoo.com Thu Jul 28 08:28:20 2011 From: mikekann at yahoo.com (Michael Kann) Date: Thu, 28 Jul 2011 05:28:20 -0700 (PDT) Subject: on-rev livecodeServer 4.6.3 .irev .lc In-Reply-To: Message-ID: <1311856100.56144.YahooMailClassic@web161604.mail.bf1.yahoo.com> Fellow webmeisters, I just read the release notes for the livecodeServer 4.6.3 and was wondering if the on-rev server has been changed over from revServer to livecodeServer and what that means for the users.? I tried using the syntax and it worked. Are "lc" and "irev" interchangeable now? Or does "lc" get you the new server and "irev" the old one? I'm especially interested in the UNICODE functions. Any gotchas get anyone yet? Thanks as always. Mike From andre at andregarzia.com Thu Jul 28 10:37:54 2011 From: andre at andregarzia.com (Andre Garzia) Date: Thu, 28 Jul 2011 11:37:54 -0300 Subject: [OT] More Apple Foolishness In-Reply-To: <76870884-CAEE-4396-B191-B2C540ACA93B@verizon.net> References: <6F918963-BD07-4351-83D1-41E1A78D06FD@altuit.com> <00af01cc4cc3$83a64f30$8af2ed90$@com> <80001DA7-522D-4AB6-A812-501492B660BF@mac.com> <76870884-CAEE-4396-B191-B2C540ACA93B@verizon.net> Message-ID: Ever since the little license fiasco that Apple hit developers a while ago, I the urge to leave the platform. I trust Steve Jobs to do what he thinks is best for Apple and Apple-only customers. I don't trust cross-platform software has a place in his plan. I have six macintosh computers, one iPad and one iPhone. I am slowly moving back to linux, the only thing that prevents me from moving everything to linux at this time is that I invested too much in apps that are only available for Mac OS X, so on the Desktop, I will migrate slowly as I find replacements. Now, about iOS, I will not repeat anything said before because we all heard all the opinions. I am now using an Android Nexus S and a Palm Pre 2 as my main phone (I alternate between them because I simply can't choose). The WebOS phone has the best usability and features, the Nexus S is more powerful and I can code in LC for it... so I use one phone for a week, then the other one for a while... both are synchronized to the same Google Accounts so I have the same contacts and emails. All over the air... The thing is, I'd rather use 2009 Palm Pre2 that has no LiveCode support than go back to iOS. I feel that iOS is a consumer platform, like a microwave oven, it does one simple thing, if you try to do something else, you end up with a burned appliance. Chipp, there are lots of Android phones out there in different levels of crappiness (<-- I invented this word). If you decide to get one, I recommend the Nexus S because that is the Google Vanilla phone, no vendor stuff, just plain android with no customizations, it is better than the stuff that HTC and Motorola are shipping. (as a joke we can all wait for MeeGo) From andre at andregarzia.com Thu Jul 28 10:38:37 2011 From: andre at andregarzia.com (Andre Garzia) Date: Thu, 28 Jul 2011 11:38:37 -0300 Subject: on-rev livecodeServer 4.6.3 .irev .lc In-Reply-To: <1311856100.56144.YahooMailClassic@web161604.mail.bf1.yahoo.com> References: <1311856100.56144.YahooMailClassic@web161604.mail.bf1.yahoo.com> Message-ID: Michael, I think on-rev might be updated, do a: and see if it is something like 4.6.3... On Thu, Jul 28, 2011 at 9:28 AM, Michael Kann wrote: > Fellow webmeisters, > > I just read the release notes for the livecodeServer 4.6.3 and was > wondering if the on-rev server has been changed over from revServer to > livecodeServer and what that means for the users. I tried using the ?> syntax and it worked. Are "lc" and "irev" interchangeable now? Or does > "lc" get you the new server and "irev" the old one? I'm especially > interested in the UNICODE functions. Any gotchas get anyone yet? Thanks as > always. > > Mike > > > _______________________________________________ > use-livecode mailing list > use-livecode at lists.runrev.com > Please visit this url to subscribe, unsubscribe and manage your > subscription preferences: > http://lists.runrev.com/mailman/listinfo/use-livecode > -- http://www.andregarzia.com All We Do Is Code. From bobs at twft.com Thu Jul 28 11:54:24 2011 From: bobs at twft.com (Bob Sneidar) Date: Thu, 28 Jul 2011 08:54:24 -0700 Subject: horizontal scrollbar in revBrowser In-Reply-To: References: <00b001cc4cc9$41afa7b0$c50ef710$@com> Message-ID: <35ACBE58-4C86-4858-B824-23AD2D7E28EC@twft.com> Wha?? Now I have to learn HTML?? ;-) Bob On Jul 27, 2011, at 7:08 PM, Chipp Walters wrote: > html {overflow-x:hidden;} > > On Wed, Jul 27, 2011 at 8:54 PM, Slava Paperno wrote: > >> Rather like any Web browser window, my revBrowser windows sometimes has a >> horizontal scroll bar and sometimes not. Because the display in my >> revBrowser window is wrappable text, this horizontal scroll bar is never >> useful. Even when it appears, it only scrolls for about five or six pixels. >> Unnecessarily. >> >> My revBrowser window is resized as the stack window is resized, and as I >> drag the window borders to make it wider or narrower, I can always find a >> width at which the horizontal scroll bar is not there. >> >> Is there a clever way to get rid of this pesky scroll bar? I have complete >> control over the Web pages that are displayed in this revBrowser and can >> add >> any style tricks to it. >> >> Slava >> >> >> >> _______________________________________________ >> use-livecode mailing list >> use-livecode at lists.runrev.com >> Please visit this url to subscribe, unsubscribe and manage your >> subscription preferences: >> http://lists.runrev.com/mailman/listinfo/use-livecode >> > > > > -- > Chipp Walters > CEO, Shafer Walters Group, Inc. > _______________________________________________ > use-livecode mailing list > use-livecode at lists.runrev.com > Please visit this url to subscribe, unsubscribe and manage your subscription preferences: > http://lists.runrev.com/mailman/listinfo/use-livecode From bobs at twft.com Thu Jul 28 11:55:49 2011 From: bobs at twft.com (Bob Sneidar) Date: Thu, 28 Jul 2011 08:55:49 -0700 Subject: Where does survive the inventive user ? In-Reply-To: <44857038-F42F-4374-8DA7-F6BCDF16629C@doctorTimothyMiller.com> References: <8CE1A70D9625EA6-2550-214DE@web-mmc-d04.sysops.aol.com> <44857038-F42F-4374-8DA7-F6BCDF16629C@doctorTimothyMiller.com> Message-ID: <6A3D1E15-1F5A-4C0F-86BF-828128EBAC9A@twft.com> Count me. On Jul 27, 2011, at 6:25 PM, Timothy Miller wrote: > It's gradually dawning on me that "programmers" like me have become rather rare. Fewer and fewer non-professionals on this list, as far as I can tell. I don't understand why, seems like a shame. From bobs at twft.com Thu Jul 28 12:05:34 2011 From: bobs at twft.com (Bob Sneidar) Date: Thu, 28 Jul 2011 09:05:34 -0700 Subject: Where does survive the inventive user ? In-Reply-To: References: <8CE1A70D9625EA6-2550-214DE@web-mmc-d04.sysops.aol.com> Message-ID: I'm going to say doom. I purchased the lifetime On-Rev and the 5 year license when it was offered, partly because I want to see these guys thrive. If they do not, then sooner or later Livecode is destined to fail. So I invested in them when they needed capital to grow. If they had stock I would probably by some. What if they had faltered back in the Revolution 2.0 days? I hate to think of having to do things without a datagrid, without behaviors that make things like sqlYoga possible. That was HUGE! Also, it's the focus on making Livecode a particular thing, and not what a lot of other developers want it to be that lends itself to continued innovation along "the right lines" and I think Open Source would not maintain that vision. RunRev takes great care to prevent making other people's past projects obsolete by ensuring the way things currently work will work tomorrow (sometimes to my disappointment). I do not think that Open Sourcing Livecode would preserve that consideration for backwards compatibility. Bob On Jul 27, 2011, at 8:11 PM, Chipp Walters wrote: > Perhaps if LC was open sourced it would have more of a chance? But then, how > would RR get paid? Of course some Open Source apps have figured out how to > have a 'free' and 'commercial' version. Still, by Open Sourcing LC, would we > be dooming our favorite dev environment or guaranteeing it's success? From bobs at twft.com Thu Jul 28 12:08:56 2011 From: bobs at twft.com (Bob Sneidar) Date: Thu, 28 Jul 2011 09:08:56 -0700 Subject: Tahoma Font In-Reply-To: <7718C7D1-010C-412B-B60C-9C94D32E893D@altuit.com> References: <000001cc4bd2$cd1e4470$675acd50$@net> <000f01cc4c3c$3b747b60$b25d7220$@de> <000c01cc4c85$b0b7ab10$12270130$@net> <7718C7D1-010C-412B-B60C-9C94D32E893D@altuit.com> Message-ID: <2A444E59-0146-4C9D-B366-D6E8179B3DCF@twft.com> This one's going into my LiveCode Keepers Folder! Nice fix. Bob On Jul 27, 2011, at 5:59 PM, Chipp Walters wrote: > Camm, > > Open up your stack in Rev as Topstack and before you make a standalone, type this in the message box: > > set the textfont of the topstack to "Tahoma" > > Then press enter and save your stack. Now make a standalone and see if it doesn't work. If Tahoma is installed on a Windows users machine, it will be used for all the "effective" textfont styles. > > Chipp Walters > CEO, Shafer Walters Group, Inc From JRARICK at hpsk12.net Thu Jul 28 12:09:42 2011 From: JRARICK at hpsk12.net (JACK RARICK) Date: Thu, 28 Jul 2011 12:09:42 -0400 Subject: FTP Error Message-ID: I am using an on-rev server. I request the contents of the public_html directory (FTP) - and I get it. I now have a list of directories inside of my public_html folder. (strA) That list is put inside of a scrolling field - and I allow the user to choose a directory. (strB) I use that request to make another list of the FILES inside of that directory. (Use FTP) And that works. (That list is put inside of a scrolling field.) The user is now looking at a list of FILES. The idea is that when the user chooses a file, it will then be downloaded to his/her computer. (strC) I put together a path to that file name by using pieces from above. strA&"/"&strB&"/"&strC - and let's call it strFINAL When I use libURLDownloadToFile with the correct parameters it returns "error" - without much else. When I simply PUT the URL strFINAL into a location on my computer it returns "Can't open /public_html/strA/strB_/Media/strC_" - no such file or directory. BUT PLEASE NOTICE that somewhere along the line underscores were added at the end of individual parts of my path. I THINK that is my problem. I evaluate the string inside of LC and I do NOT see the underscores, but when the error is returned, the underscores are there. 1) I do not know if the underscores are the problem, but could they be? 2) Any other reason this might not be working? Thank you! I will hang up now and listen to your answers and suggestions! As always, many, many thanks in advance. Jack Rarick Freshmen Center Earth Science Technology Coodinator Varsity Men's Track & Field Varsity Football - Offensive Line Coach, Peanut Butter Coordinator From bobs at twft.com Thu Jul 28 12:13:23 2011 From: bobs at twft.com (Bob Sneidar) Date: Thu, 28 Jul 2011 09:13:23 -0700 Subject: AW: OT: I want to buy Lion In-Reply-To: <201107280226.10849.warren@warrensweb.us> References: <001801cc4c80$41078b00$c316a100$@de> <9308DB19-829D-41A0-A887-B2F59C1C4CF4@mac.com> <001101cc4cf1$75d822b0$61886810$@de> <201107280226.10849.warren@warrensweb.us> Message-ID: <49A067C7-D033-4068-B7A1-5035CAA7FD7B@twft.com> At the risk of getting flamed by some as an Apple lover, I think that may be harsh. Remember one of Microsoft's excuses for not producing a really modern OS for so long was that they had to maintain backwards compatibility for all their users. At some point this becomes a ball and chain for the vendor. Everyone gets less so that some won't be left behind. To a point that is all well and good. Beyond that point, I say leave them behind. What they have works for them. No one is taking anything away from people who do not upgrade. The situation is rather, if you want to upgrade, you will have to pay. Is that unfair? Bob On Jul 28, 2011, at 12:26 AM, Warren Samples wrote: > On Thursday, July 28, 2011 01:41:57 AM Tiemo Hollmann TB wrote: >> I don't understand why they don't offer anymore any >> "classic download" option for backward compatibility for users who are not >> up to date every day like me. >> Tiemo > > > They're not interested in helping you to maintain your "backward" status. Apple wants its users up to date, > with the program, and feeding them money through iTunes and the App Store. > > Best, > > Warren > > _______________________________________________ > use-livecode mailing list > use-livecode at lists.runrev.com > Please visit this url to subscribe, unsubscribe and manage your subscription preferences: > http://lists.runrev.com/mailman/listinfo/use-livecode From bobs at twft.com Thu Jul 28 12:16:05 2011 From: bobs at twft.com (Bob Sneidar) Date: Thu, 28 Jul 2011 09:16:05 -0700 Subject: A case of assigned behavior not taken into account In-Reply-To: References: <9EC1084F-AC69-492B-A2C6-1658129FF052@wanadoo.fr> Message-ID: <31540456-786D-4297-A832-708142849BA5@twft.com> No problem. Also remember that if you are creating a standalone application, the mainstack cannot be altered. It is read only. If you have to change it every time, might this be the cause? Bob On Jul 28, 2011, at 12:41 AM, Andr? Bisseret wrote: > Hi Bob, > > You are right ; i verified that when loading the application from one computer to another (from Mac to PC and vice versa) I had to reassign the behavior button > So I am keeping doing this reassignment. > > Thank you very much for your attention and explanation. > > Andr? From bobs at twft.com Thu Jul 28 12:18:20 2011 From: bobs at twft.com (Bob Sneidar) Date: Thu, 28 Jul 2011 09:18:20 -0700 Subject: Unzipping Mac executable files with revZip - revisited In-Reply-To: References: <2DBD6DE8-CDED-4DCE-98A9-1D1A3902D5BB@unimelb.edu.au> <016257E4-3151-4E93-B521-FD92CC810064@unimelb.edu.au> Message-ID: <5439D6E8-2124-4F4F-9E39-D6166C6E16F0@twft.com> mmm... not sure that is true. Otherwise, how could a full Time Machine restore work? Bob On Jul 27, 2011, at 11:08 PM, Pete Haworth wrote: > I ran into this once with Time Machine. I restored an iPages file from a > backup and it brought it back without an extension and treated it as a unix > executable. Even more ironic that Time Machine can't restore a file from an > Apple application properly! > > Pete From keith.clarke at clarkeandclarke.co.uk Thu Jul 28 12:19:15 2011 From: keith.clarke at clarkeandclarke.co.uk (Keith Clarke) Date: Thu, 28 Jul 2011 17:19:15 +0100 Subject: FTP Error In-Reply-To: References: Message-ID: <39DB829E-749E-4758-A754-82AE6880A329@clarkeandclarke.co.uk> ...maybe try put urlencode(strFINAL) to manage those pesky non-text characters? Best, Keith.. On 28 Jul 2011, at 17:09, JACK RARICK wrote: > I am using an on-rev server. I request the contents of the public_html directory (FTP) - and I get it. I now have a list of directories inside of my public_html folder. (strA) > > That list is put inside of a scrolling field - and I allow the user to choose a directory. (strB) > > I use that request to make another list of the FILES inside of that directory. (Use FTP) And that works. (That list is put inside of a scrolling field.) > > The user is now looking at a list of FILES. The idea is that when the user chooses a file, it will then be downloaded to his/her computer. (strC) > > I put together a path to that file name by using pieces from above. strA&"/"&strB&"/"&strC - and let's call it strFINAL > > When I use libURLDownloadToFile with the correct parameters it returns "error" - without much else. > > When I simply PUT the URL strFINAL into a location on my computer it returns "Can't open /public_html/strA/strB_/Media/strC_" - no such file or directory. > > BUT PLEASE NOTICE that somewhere along the line underscores were added at the end of individual parts of my path. I THINK that is my problem. I evaluate the string inside of LC and I do NOT see the underscores, but when the error is returned, the underscores are there. > > 1) I do not know if the underscores are the problem, but could they be? > > 2) Any other reason this might not be working? > > > Thank you! I will hang up now and listen to your answers and suggestions! > As always, many, many thanks in advance. > > > Jack Rarick > Freshmen Center > Earth Science > Technology Coodinator > Varsity Men's Track & Field > Varsity Football - Offensive Line Coach, Peanut Butter Coordinator > _______________________________________________ > use-livecode mailing list > use-livecode at lists.runrev.com > Please visit this url to subscribe, unsubscribe and manage your subscription preferences: > http://lists.runrev.com/mailman/listinfo/use-livecode From jacque at hyperactivesw.com Thu Jul 28 12:21:55 2011 From: jacque at hyperactivesw.com (J. Landman Gay) Date: Thu, 28 Jul 2011 11:21:55 -0500 Subject: [OT] More Apple Foolishness In-Reply-To: <76870884-CAEE-4396-B191-B2C540ACA93B@verizon.net> References: <6F918963-BD07-4351-83D1-41E1A78D06FD@altuit.com> <00af01cc4cc3$83a64f30$8af2ed90$@com> <80001DA7-522D-4AB6-A812-501492B660BF@mac.com> <76870884-CAEE-4396-B191-B2C540ACA93B@verizon.net> Message-ID: <4E318CA3.2030106@hyperactivesw.com> On 7/28/11 4:01 AM, Colin Holgate wrote: > I hardly ever use iTunes for getting things onto my devices, What do you use? I'm tired of hooking up to iTunes for everything. In particular, I want to move test standalones to the device. -- Jacqueline Landman Gay | jacque at hyperactivesw.com HyperActive Software | http://www.hyperactivesw.com From JRARICK at hpsk12.net Thu Jul 28 12:21:39 2011 From: JRARICK at hpsk12.net (JACK RARICK) Date: Thu, 28 Jul 2011 12:21:39 -0400 Subject: FTP Error In-Reply-To: <39DB829E-749E-4758-A754-82AE6880A329@clarkeandclarke.co.uk> References: , <39DB829E-749E-4758-A754-82AE6880A329@clarkeandclarke.co.uk> Message-ID: Keith said: ...maybe try put urlencode(strFINAL) to manage those pesky non-text characters? Thank you - I did - and it didn't! Dang. Any chance it might be a DIRECTORY issue? Should I be writing this code for the public_ftp folder? I will try that. On 28 Jul 2011, at 17:09, JACK RARICK wrote: > I am using an on-rev server. I request the contents of the public_html directory (FTP) - and I get it. I now have a list of directories inside of my public_html folder. (strA) > > That list is put inside of a scrolling field - and I allow the user to choose a directory. (strB) > > I use that request to make another list of the FILES inside of that directory. (Use FTP) And that works. (That list is put inside of a scrolling field.) > > The user is now looking at a list of FILES. The idea is that when the user chooses a file, it will then be downloaded to his/her computer. (strC) > > I put together a path to that file name by using pieces from above. strA&"/"&strB&"/"&strC - and let's call it strFINAL > > When I use libURLDownloadToFile with the correct parameters it returns "error" - without much else. > > When I simply PUT the URL strFINAL into a location on my computer it returns "Can't open /public_html/strA/strB_/Media/strC_" - no such file or directory. > > BUT PLEASE NOTICE that somewhere along the line underscores were added at the end of individual parts of my path. I THINK that is my problem. I evaluate the string inside of LC and I do NOT see the underscores, but when the error is returned, the underscores are there. > > 1) I do not know if the underscores are the problem, but could they be? > > 2) Any other reason this might not be working? > > > Thank you! I will hang up now and listen to your answers and suggestions! > As always, many, many thanks in advance. > > > Jack Rarick > Freshmen Center > Earth Science > Technology Coodinator > Varsity Men's Track & Field > Varsity Football - Offensive Line Coach, Peanut Butter Coordinator > _______________________________________________ > use-livecode mailing list > use-livecode at lists.runrev.com > Please visit this url to subscribe, unsubscribe and manage your subscription preferences: > http://lists.runrev.com/mailman/listinfo/use-livecode _______________________________________________ use-livecode mailing list use-livecode at lists.runrev.com Please visit this url to subscribe, unsubscribe and manage your subscription preferences: http://lists.runrev.com/mailman/listinfo/use-livecode From bobs at twft.com Thu Jul 28 12:26:42 2011 From: bobs at twft.com (Bob Sneidar) Date: Thu, 28 Jul 2011 09:26:42 -0700 Subject: Where does survive the inventive user? In-Reply-To: <08F3632C-76A9-402E-97B4-C82090DC8D05@free.fr> References: <1311706987455-3696711.post@n4.nabble.com> <08F3632C-76A9-402E-97B4-C82090DC8D05@free.fr> Message-ID: <465EFD52-8532-4945-A2F0-C2CE01371329@twft.com> That is a way of looking at things I suppose. But as I have said before, the purpose of a business is to make money. That may offend some, but if Apple does not succeed, then someone else will. They will be the bogie then. If RunRev had not succeeded then we would not have our beloved Livecode. The Mothership (by which I think you mean RunRev) did succeeded, and so we have the opportunity to succeed. Large business enterprises always seem to some to be evil giants, until we as individuals succeed fairly nicely, and find that we are now working for one, or better yet are the CEO of one. Bob On Jul 28, 2011, at 4:30 AM, Pierre Sahores wrote: > I sincerely believe that their desire to control the freedom of initiative and economy of means - that is dear to all those who have already understood that the technique is a simple tool for creativity - will fail like all the idiot strategies whose, before them, thought they would manipulate for their own interests only the market of painting, literature and music by controlling the manufacture of brushes, paper production and ownership of concert halls. > > They sought to turn away the best functional programming ??and procedural languages in trying to intoxicant us with the supposed superiority of the sterile logic of the UML and object-oriented programming methodologies. From klaus at major.on-rev.com Thu Jul 28 12:29:17 2011 From: klaus at major.on-rev.com (Klaus on-rev) Date: Thu, 28 Jul 2011 18:29:17 +0200 Subject: [OT] More Apple Foolishness In-Reply-To: <4E318CA3.2030106@hyperactivesw.com> References: <6F918963-BD07-4351-83D1-41E1A78D06FD@altuit.com> <00af01cc4cc3$83a64f30$8af2ed90$@com> <80001DA7-522D-4AB6-A812-501492B660BF@mac.com> <76870884-CAEE-4396-B191-B2C540ACA93B@verizon.net> <4E318CA3.2030106@hyperactivesw.com> Message-ID: <3818E517-4384-41E3-AD1E-75058CB5C5EF@major.on-rev.com> Hi Jacqueline, Am 28.07.2011 um 18:21 schrieb J. Landman Gay: > On 7/28/11 4:01 AM, Colin Holgate wrote: >> I hardly ever use iTunes for getting things onto my devices, > What do you use? I'm tired of hooking up to iTunes for everything. In particular, I want to move test standalones to the device. I use the "Organizer" in XCode for that. > -- > Jacqueline Landman Gay Best Klaus -- Klaus Major http://www.major-k.de klaus at major.on-rev.com From keith.clarke at clarkeandclarke.co.uk Thu Jul 28 12:39:27 2011 From: keith.clarke at clarkeandclarke.co.uk (Keith Clarke) Date: Thu, 28 Jul 2011 17:39:27 +0100 Subject: FTP Error In-Reply-To: References: , <39DB829E-749E-4758-A754-82AE6880A329@clarkeandclarke.co.uk> Message-ID: Jack, have you considered keeping the returned lists stored in variables or custom properties and just presenting them in fields for user selection? That way, you can ensure that the list items stay as plain text (as sometimes fields will 'interpret' and hide some characters). For debugging web and server stuff, I tend to avoid fields for storage and set custom properties for anything I need to validate as plain text - before getting the properties back into a concatenation variable as the final URL. Best, Keith.. On 28 Jul 2011, at 17:21, JACK RARICK wrote: > Keith said: ...maybe try put urlencode(strFINAL) to manage those pesky non-text characters? > > Thank you - I did - and it didn't! Dang. Any chance it might be a DIRECTORY issue? Should I be writing this code for the public_ftp folder? I will try that. > > On 28 Jul 2011, at 17:09, JACK RARICK wrote: > >> I am using an on-rev server. I request the contents of the public_html directory (FTP) - and I get it. I now have a list of directories inside of my public_html folder. (strA) >> >> That list is put inside of a scrolling field - and I allow the user to choose a directory. (strB) >> >> I use that request to make another list of the FILES inside of that directory. (Use FTP) And that works. (That list is put inside of a scrolling field.) >> >> The user is now looking at a list of FILES. The idea is that when the user chooses a file, it will then be downloaded to his/her computer. (strC) >> >> I put together a path to that file name by using pieces from above. strA&"/"&strB&"/"&strC - and let's call it strFINAL >> >> When I use libURLDownloadToFile with the correct parameters it returns "error" - without much else. >> >> When I simply PUT the URL strFINAL into a location on my computer it returns "Can't open /public_html/strA/strB_/Media/strC_" - no such file or directory. >> >> BUT PLEASE NOTICE that somewhere along the line underscores were added at the end of individual parts of my path. I THINK that is my problem. I evaluate the string inside of LC and I do NOT see the underscores, but when the error is returned, the underscores are there. >> >> 1) I do not know if the underscores are the problem, but could they be? >> >> 2) Any other reason this might not be working? >> >> >> Thank you! I will hang up now and listen to your answers and suggestions! >> As always, many, many thanks in advance. >> >> >> Jack Rarick >> Freshmen Center >> Earth Science >> Technology Coodinator >> Varsity Men's Track & Field >> Varsity Football - Offensive Line Coach, Peanut Butter Coordinator From lists.pete at haworths.org Thu Jul 28 12:52:09 2011 From: lists.pete at haworths.org (Pete Haworth) Date: Thu, 28 Jul 2011 09:52:09 -0700 Subject: Unzipping Mac executable files with revZip - revisited In-Reply-To: <5439D6E8-2124-4F4F-9E39-D6166C6E16F0@twft.com> References: <2DBD6DE8-CDED-4DCE-98A9-1D1A3902D5BB@unimelb.edu.au> <016257E4-3151-4E93-B521-FD92CC810064@unimelb.edu.au> <5439D6E8-2124-4F4F-9E39-D6166C6E16F0@twft.com> Message-ID: Oh, it's true - I didn't dream it. I'm referring specifically to iPages file (possibly other iWorks files), not all file types. Why it should happen only with those files, I have no idea. Pete On Thu, Jul 28, 2011 at 9:18 AM, Bob Sneidar wrote: > mmm... not sure that is true. Otherwise, how could a full Time Machine > restore work? > > Bob > > > On Jul 27, 2011, at 11:08 PM, Pete Haworth wrote: > > > I ran into this once with Time Machine. I restored an iPages file from a > > backup and it brought it back without an extension and treated it as a > unix > > executable. Even more ironic that Time Machine can't restore a file from > an > > Apple application properly! > > > > Pete > > > _______________________________________________ > use-livecode mailing list > use-livecode at lists.runrev.com > Please visit this url to subscribe, unsubscribe and manage your > subscription preferences: > http://lists.runrev.com/mailman/listinfo/use-livecode > From lists.pete at haworths.org Thu Jul 28 12:55:39 2011 From: lists.pete at haworths.org (Pete Haworth) Date: Thu, 28 Jul 2011 09:55:39 -0700 Subject: A case of assigned behavior not taken into account In-Reply-To: References: <9EC1084F-AC69-492B-A2C6-1658129FF052@wanadoo.fr> Message-ID: The dictionary does say that behaviors are in the form of long IDs but I have found that they don't have to be. A behavior of the form 'button ID 1234 of stack "xyz" works just fine. I have no idea if this is a loophole or whether it is intended to work that way. I do know that it save a huge amount of work reassigning behaviors every time you move a stack form one location to another. Pete On Thu, Jul 28, 2011 at 12:41 AM, Andr? Bisseret wrote: > Hi Bob, > > You are right ; i verified that when loading the application from one > computer to another (from Mac to PC and vice versa) I had to reassign the > behavior button > So I am keeping doing this reassignment. > > Thank you very much for your attention and explanation. > > Andr? > > > Le 27 juil. 2011 ? 18:37, Bob Sneidar a ?crit : > > > I will take a shot at this. Behaviors are actually the script of a > button, referenced as it's long ID. The long ID (as you could see if you got > the long ID of any object) references not just the card it is on but the > stack itself. When you clone a stack with behaviors, I suspect that the > behaviors are still using the reference to the long ID of the button in the > template stack. You will have to change that by script as you suspect. > > > > The reason this is like that is because you would want a modified > behavior script to affect all object that use it in your entire application. > It's just a little bit like OOP for programming. If you wanted the behavior > to be altered a bit for certain objects, you could either intercept the > message in the object's script, do what is different, then optionally pass > it, or you could create a new button which was a copy of the behavior button > and assign the object's behavior to that. But obviously you would lose the > "one edit fixes all" for that button. > > > > Bob > > > > > > On Jul 27, 2011, at 4:41 AM, Andr? Bisseret wrote: > > > >> Bonjour, > >> > >> On an app. I am developing on Mac, I have a main stack a substack of > which is a model used for creating new stacks which are cloned from the > model and saved as "independent" stacks (not substacks). > >> > >> The scripts of the card 1 of this model and of all objects on this card > are all together in a behavior button which is assigned to this card 1. > >> This behavior is on card 2 of the main stack. > >> > >> All is working well on Mac. > >> > >> But when I load the standalone for Windows on a PC (by means of a USB > key) then a newly created stack from the model is inert. > >> Meanwhile, I verified that the behavior is actually assigned to the card > 1 of the new stack, but all behaves like this was not the case! > >> > >> If, by script, I reassign the behavior to card 1 of the model before > cloning it, then the new stack is working as expected. > >> > >> So I could stay with this reassignment but? > >> > >> Is it normal (seems not to me!), or am I missing something ? > >> > >> Any hint much appreciated > >> > >> Best regards from Grenoble > >> > >> Andr? > >> > >> > >> _______________________________________________ > >> use-livecode mailing list > >> use-livecode at lists.runrev.com > >> Please visit this url to subscribe, unsubscribe and manage your > subscription preferences: > >> http://lists.runrev.com/mailman/listinfo/use-livecode > > > > > > _______________________________________________ > > use-livecode mailing list > > use-livecode at lists.runrev.com > > Please visit this url to subscribe, unsubscribe and manage your > subscription preferences: > > http://lists.runrev.com/mailman/listinfo/use-livecode > > > _______________________________________________ > use-livecode mailing list > use-livecode at lists.runrev.com > Please visit this url to subscribe, unsubscribe and manage your > subscription preferences: > http://lists.runrev.com/mailman/listinfo/use-livecode > From coiin at verizon.net Thu Jul 28 12:56:50 2011 From: coiin at verizon.net (Colin Holgate) Date: Thu, 28 Jul 2011 17:56:50 +0100 Subject: [OT] More Apple Foolishness In-Reply-To: <3818E517-4384-41E3-AD1E-75058CB5C5EF@major.on-rev.com> References: <6F918963-BD07-4351-83D1-41E1A78D06FD@altuit.com> <00af01cc4cc3$83a64f30$8af2ed90$@com> <80001DA7-522D-4AB6-A812-501492B660BF@mac.com> <76870884-CAEE-4396-B191-B2C540ACA93B@verizon.net> <4E318CA3.2030106@hyperactivesw.com> <3818E517-4384-41E3-AD1E-75058CB5C5EF@major.on-rev.com> Message-ID: <3657691C-1A95-486F-A38B-B3C60C0E30F1@verizon.net> Me too, and I tend to update App Store apps straight to my iPad or iPhone. I also bought this Mac App Store product: http://itunes.apple.com/us/app/betabuilder-for-ios-apps/id415348946?mt=12 It's the same trick that TestFlight uses, except you can post your app to your own server. I sometimes do that instead of using the Organizer window. On Jul 28, 2011, at 5:29 PM, Klaus on-rev wrote: > I use the "Organizer" in XCode for that. From andre at andregarzia.com Thu Jul 28 12:58:31 2011 From: andre at andregarzia.com (Andre Garzia) Date: Thu, 28 Jul 2011 13:58:31 -0300 Subject: FTP Error In-Reply-To: References: Message-ID: Jack, create a field and use liburlsetlogfield this will log all the transactions so that you can see where the problem lies. My guess is that you are using a "/" in the front of your path, this will fail because there is not such thing as /public_html but there is /home//public_html... Try removing the first slash to make it relative to the home folder instead of in the root level. my 2 BRL cents... On Thu, Jul 28, 2011 at 1:09 PM, JACK RARICK wrote: > I am using an on-rev server. I request the contents of the public_html > directory (FTP) - and I get it. I now have a list of directories inside of > my public_html folder. (strA) > > That list is put inside of a scrolling field - and I allow the user to > choose a directory. (strB) > > I use that request to make another list of the FILES inside of that > directory. (Use FTP) And that works. (That list is put inside of a > scrolling field.) > > The user is now looking at a list of FILES. The idea is that when the user > chooses a file, it will then be downloaded to his/her computer. (strC) > > I put together a path to that file name by using pieces from above. > strA&"/"&strB&"/"&strC - and let's call it strFINAL > > When I use libURLDownloadToFile with the correct parameters it returns > "error" - without much else. > > When I simply PUT the URL strFINAL into a location on my computer it > returns "Can't open /public_html/strA/strB_/Media/strC_" - no such file or > directory. > > BUT PLEASE NOTICE that somewhere along the line underscores were added at > the end of individual parts of my path. I THINK that is my problem. I > evaluate the string inside of LC and I do NOT see the underscores, but when > the error is returned, the underscores are there. > > 1) I do not know if the underscores are the problem, but could they be? > > 2) Any other reason this might not be working? > > > Thank you! I will hang up now and listen to your answers and suggestions! > As always, many, many thanks in advance. > > > Jack Rarick > Freshmen Center > Earth Science > Technology Coodinator > Varsity Men's Track & Field > Varsity Football - Offensive Line Coach, Peanut Butter Coordinator > _______________________________________________ > use-livecode mailing list > use-livecode at lists.runrev.com > Please visit this url to subscribe, unsubscribe and manage your > subscription preferences: > http://lists.runrev.com/mailman/listinfo/use-livecode > -- http://www.andregarzia.com All We Do Is Code. From warren at warrensweb.us Thu Jul 28 13:00:00 2011 From: warren at warrensweb.us (Warren Samples) Date: Thu, 28 Jul 2011 12:00:00 -0500 Subject: AW: OT: I want to buy Lion In-Reply-To: <49A067C7-D033-4068-B7A1-5035CAA7FD7B@twft.com> References: <001801cc4c80$41078b00$c316a100$@de> <201107280226.10849.warren@warrensweb.us> <49A067C7-D033-4068-B7A1-5035CAA7FD7B@twft.com> Message-ID: <201107281200.00594.warren@warrensweb.us> On Thursday, July 28, 2011 11:13:23 AM Bob Sneidar wrote: > I think that may be harsh. Remember one of Microsoft's excuses for not > producing a really modern OS for so long was that they had to maintain > backwards compatibility for all their users. At some point this becomes a > ball and chain for the vendor No, Bob, not harsh at all. I don't see much judgement being passed, really. There is only slight and mostly implied chacterization of Apple's obvious marketring strategy. I'm not going to flame you, but your apologia strikes me as being almost irrelevant to the issue of directing traffic in a gratuitous fashion through Apple's various money-making apparatus. Not always convenient for the consumer, but certainly convenient for Apple. Warren From lists.pete at haworths.org Thu Jul 28 13:02:13 2011 From: lists.pete at haworths.org (Pete Haworth) Date: Thu, 28 Jul 2011 10:02:13 -0700 Subject: AW: OT: I want to buy Lion In-Reply-To: <49A067C7-D033-4068-B7A1-5035CAA7FD7B@twft.com> References: <001801cc4c80$41078b00$c316a100$@de> <9308DB19-829D-41A0-A887-B2F59C1C4CF4@mac.com> <001101cc4cf1$75d822b0$61886810$@de> <201107280226.10849.warren@warrensweb.us> <49A067C7-D033-4068-B7A1-5035CAA7FD7B@twft.com> Message-ID: This is not about people who do not want to upgrade, it's about people who do want to upgrade, are willing to pay, need to support the users of their applications, but cannot because of Apple's upgrade methodology. Sounds like they will provide a "hard copy" method to upgrade eventually. Pete On Thu, Jul 28, 2011 at 9:13 AM, Bob Sneidar wrote: > At the risk of getting flamed by some as an Apple lover, I think that may > be harsh. Remember one of Microsoft's excuses for not producing a really > modern OS for so long was that they had to maintain backwards compatibility > for all their users. At some point this becomes a ball and chain for the > vendor. Everyone gets less so that some won't be left behind. To a point > that is all well and good. Beyond that point, I say leave them behind. What > they have works for them. No one is taking anything away from people who do > not upgrade. The situation is rather, if you want to upgrade, you will have > to pay. Is that unfair? > > Bob > > > On Jul 28, 2011, at 12:26 AM, Warren Samples wrote: > > > On Thursday, July 28, 2011 01:41:57 AM Tiemo Hollmann TB wrote: > >> I don't understand why they don't offer anymore any > >> "classic download" option for backward compatibility for users who are > not > >> up to date every day like me. > >> Tiemo > > > > > > They're not interested in helping you to maintain your "backward" status. > Apple wants its users up to date, > > with the program, and feeding them money through iTunes and the App > Store. > > > > Best, > > > > Warren > > > > _______________________________________________ > > use-livecode mailing list > > use-livecode at lists.runrev.com > > Please visit this url to subscribe, unsubscribe and manage your > subscription preferences: > > http://lists.runrev.com/mailman/listinfo/use-livecode > > > _______________________________________________ > use-livecode mailing list > use-livecode at lists.runrev.com > Please visit this url to subscribe, unsubscribe and manage your > subscription preferences: > http://lists.runrev.com/mailman/listinfo/use-livecode > From stephenREVOLUTION2 at barncard.com Thu Jul 28 13:03:27 2011 From: stephenREVOLUTION2 at barncard.com (stephen barncard) Date: Thu, 28 Jul 2011 10:03:27 -0700 Subject: Unzipping Mac executable files with revZip - revisited In-Reply-To: References: <2DBD6DE8-CDED-4DCE-98A9-1D1A3902D5BB@unimelb.edu.au> <016257E4-3151-4E93-B521-FD92CC810064@unimelb.edu.au> <5439D6E8-2124-4F4F-9E39-D6166C6E16F0@twft.com> Message-ID: Note that those "files" you mentioned are actually "bundles" - folders with a special plist. Like Livecode.app itself. On 28 July 2011 09:52, Pete Haworth wrote: > Oh, it's true - I didn't dream it. I'm referring specifically to iPages > file (possibly other iWorks files), not all file types. Why it should > happen only with those files, I have no idea. > > Pete > > > > On Thu, Jul 28, 2011 at 9:18 AM, Bob Sneidar wrote: > > > mmm... not sure that is true. Otherwise, how could a full Time Machine > > restore work? > > > > Bob > > > > > > On Jul 27, 2011, at 11:08 PM, Pete Haworth wrote: > > > > > I ran into this once with Time Machine. I restored an iPages file from > a > > > backup and it brought it back without an extension and treated it as a > > unix > > > executable. Even more ironic that Time Machine can't restore a file > from > > an > > > Apple application properly! > > > > > > Pete > > > > > > _______________________________________________ > > use-livecode mailing list > > use-livecode at lists.runrev.com > > Please visit this url to subscribe, unsubscribe and manage your > > subscription preferences: > > http://lists.runrev.com/mailman/listinfo/use-livecode > > > _______________________________________________ > use-livecode mailing list > use-livecode at lists.runrev.com > Please visit this url to subscribe, unsubscribe and manage your > subscription preferences: > http://lists.runrev.com/mailman/listinfo/use-livecode > -- Stephen Barncard San Francisco Ca. USA more about sqb From jacque at hyperactivesw.com Thu Jul 28 13:17:41 2011 From: jacque at hyperactivesw.com (J. Landman Gay) Date: Thu, 28 Jul 2011 12:17:41 -0500 Subject: [OT] More Apple Foolishness In-Reply-To: <3657691C-1A95-486F-A38B-B3C60C0E30F1@verizon.net> References: <6F918963-BD07-4351-83D1-41E1A78D06FD@altuit.com> <00af01cc4cc3$83a64f30$8af2ed90$@com> <80001DA7-522D-4AB6-A812-501492B660BF@mac.com> <76870884-CAEE-4396-B191-B2C540ACA93B@verizon.net> <4E318CA3.2030106@hyperactivesw.com> <3818E517-4384-41E3-AD1E-75058CB5C5EF@major.on-rev.com> <3657691C-1A95-486F-A38B-B3C60C0E30F1@verizon.net> Message-ID: <4E3199B5.2010601@hyperactivesw.com> On 7/28/11 11:56 AM, Colin Holgate wrote: > Me too, and I tend to update App Store apps straight to my iPad or > iPhone. I also bought this Mac App Store product: > > http://itunes.apple.com/us/app/betabuilder-for-ios-apps/id415348946?mt=12 > > It's the same trick that TestFlight uses, except you can post your > app to your own server. I sometimes do that instead of using the > Organizer window. > > > On Jul 28, 2011, at 5:29 PM, Klaus on-rev wrote: > >> I use the "Organizer" in XCode for that. Thanks guys. I was hoping for a non-physical connection method, so maybe I'll look into the app you mention. I wish Dropbox worked, that's mostly how I move my Android builds. -- Jacqueline Landman Gay | jacque at hyperactivesw.com HyperActive Software | http://www.hyperactivesw.com From JRARICK at hpsk12.net Thu Jul 28 13:16:11 2011 From: JRARICK at hpsk12.net (JACK RARICK) Date: Thu, 28 Jul 2011 13:16:11 -0400 Subject: FTP Error In-Reply-To: References: , Message-ID: Andre! Thanks for the tip to use liburlsetlogfield! If anything I will learn a lot more a lot faster. Here is my line of code: put "ftp://" & FTPUSER & ":" & FTPPASS & "@" & "ftp.cva.on-rev.com/public_ftp/holtfootball"&"/"&gamefilm&"/Media/"&tFilename into xStr I am a football coach. (American) holtfootball is the school name. gamefilm is the name of the specific game. tFilename is the name of the particular "clip" of the game. The log field says that the path is getting their "correctly". (At least everything is spelled right etc.) Although the path itself may be wrong. But on reading the log field, those pesky underscores are returned. I tried the public_html directory as well. No go. Thanks - I really appreciate all the help. What am I forgetting? Jack Rarick Freshmen Center Earth Science Technology Coodinator Varsity Men's Track & Field Varsity Football - Offensive Line Coach, Peanut Butter Coordinator From bobs at twft.com Thu Jul 28 13:21:09 2011 From: bobs at twft.com (Bob Sneidar) Date: Thu, 28 Jul 2011 10:21:09 -0700 Subject: Livecode Lockups Message-ID: <291762D3-0EF6-40C3-B3F5-47DE06A6E67A@twft.com> Hi all. I am using 4.6.3. It has been beach-balling quite a lot since I upgraded from 4.6.2. I just suppressed messages so I could go from card to card without executing any scripts. I went back a few cards and it beach-balled again. Correct me if I am wrong, but since suppress messages was turned on, this CANNOT BE a problem with any of my scripts, right? Bob From bobs at twft.com Thu Jul 28 13:27:07 2011 From: bobs at twft.com (Bob Sneidar) Date: Thu, 28 Jul 2011 10:27:07 -0700 Subject: Unzipping Mac executable files with revZip - revisited In-Reply-To: References: <2DBD6DE8-CDED-4DCE-98A9-1D1A3902D5BB@unimelb.edu.au> <016257E4-3151-4E93-B521-FD92CC810064@unimelb.edu.au> <5439D6E8-2124-4F4F-9E39-D6166C6E16F0@twft.com> Message-ID: <8BCCDDC5-2B4D-4C4F-A936-07F3133CDE05@twft.com> Oh sorry, I read your post as, "Time Machine won't properly restore an application." Bob On Jul 28, 2011, at 9:52 AM, Pete Haworth wrote: > Oh, it's true - I didn't dream it. I'm referring specifically to iPages > file (possibly other iWorks files), not all file types. Why it should > happen only with those files, I have no idea. > > Pete > > > > On Thu, Jul 28, 2011 at 9:18 AM, Bob Sneidar wrote: > >> mmm... not sure that is true. Otherwise, how could a full Time Machine >> restore work? >> >> Bob >> >> >> On Jul 27, 2011, at 11:08 PM, Pete Haworth wrote: >> >>> I ran into this once with Time Machine. I restored an iPages file from a >>> backup and it brought it back without an extension and treated it as a >> unix >>> executable. Even more ironic that Time Machine can't restore a file from >> an >>> Apple application properly! >>> >>> Pete >> >> >> _______________________________________________ >> use-livecode mailing list >> use-livecode at lists.runrev.com >> Please visit this url to subscribe, unsubscribe and manage your >> subscription preferences: >> http://lists.runrev.com/mailman/listinfo/use-livecode >> > _______________________________________________ > use-livecode mailing list > use-livecode at lists.runrev.com > Please visit this url to subscribe, unsubscribe and manage your subscription preferences: > http://lists.runrev.com/mailman/listinfo/use-livecode From jacque at hyperactivesw.com Thu Jul 28 13:34:16 2011 From: jacque at hyperactivesw.com (J. Landman Gay) Date: Thu, 28 Jul 2011 12:34:16 -0500 Subject: [OT] More Apple Foolishness In-Reply-To: <3657691C-1A95-486F-A38B-B3C60C0E30F1@verizon.net> References: <6F918963-BD07-4351-83D1-41E1A78D06FD@altuit.com> <00af01cc4cc3$83a64f30$8af2ed90$@com> <80001DA7-522D-4AB6-A812-501492B660BF@mac.com> <76870884-CAEE-4396-B191-B2C540ACA93B@verizon.net> <4E318CA3.2030106@hyperactivesw.com> <3818E517-4384-41E3-AD1E-75058CB5C5EF@major.on-rev.com> <3657691C-1A95-486F-A38B-B3C60C0E30F1@verizon.net> Message-ID: <4E319D98.2030909@hyperactivesw.com> On 7/28/11 11:56 AM, Colin Holgate wrote: > Me too, and I tend to update App Store apps straight to my iPad or > iPhone. I also bought this Mac App Store product: I just found this: Looks like it might work, and it's a free solution that uses Dropbox. -- Jacqueline Landman Gay | jacque at hyperactivesw.com HyperActive Software | http://www.hyperactivesw.com From lists.pete at haworths.org Thu Jul 28 13:38:44 2011 From: lists.pete at haworths.org (Pete Haworth) Date: Thu, 28 Jul 2011 10:38:44 -0700 Subject: Unzipping Mac executable files with revZip - revisited In-Reply-To: References: <2DBD6DE8-CDED-4DCE-98A9-1D1A3902D5BB@unimelb.edu.au> <016257E4-3151-4E93-B521-FD92CC810064@unimelb.edu.au> <5439D6E8-2124-4F4F-9E39-D6166C6E16F0@twft.com> Message-ID: Yes, I discovered that when finding the solution to the problem (via the iPages Discussion Forum). Pete On Thu, Jul 28, 2011 at 10:03 AM, stephen barncard < stephenREVOLUTION2 at barncard.com> wrote: > Note that those "files" you mentioned are actually "bundles" - folders with > a special plist. Like Livecode.app itself. > > On 28 July 2011 09:52, Pete Haworth wrote: > > > Oh, it's true - I didn't dream it. I'm referring specifically to iPages > > file (possibly other iWorks files), not all file types. Why it should > > happen only with those files, I have no idea. > > > > Pete > > > > > > > > On Thu, Jul 28, 2011 at 9:18 AM, Bob Sneidar wrote: > > > > > mmm... not sure that is true. Otherwise, how could a full Time Machine > > > restore work? > > > > > > Bob > > > > > > > > > On Jul 27, 2011, at 11:08 PM, Pete Haworth wrote: > > > > > > > I ran into this once with Time Machine. I restored an iPages file > from > > a > > > > backup and it brought it back without an extension and treated it as > a > > > unix > > > > executable. Even more ironic that Time Machine can't restore a file > > from > > > an > > > > Apple application properly! > > > > > > > > Pete > > > > > > > > > _______________________________________________ > > > use-livecode mailing list > > > use-livecode at lists.runrev.com > > > Please visit this url to subscribe, unsubscribe and manage your > > > subscription preferences: > > > http://lists.runrev.com/mailman/listinfo/use-livecode > > > > > _______________________________________________ > > use-livecode mailing list > > use-livecode at lists.runrev.com > > Please visit this url to subscribe, unsubscribe and manage your > > subscription preferences: > > http://lists.runrev.com/mailman/listinfo/use-livecode > > > > > > -- > > > > Stephen Barncard > San Francisco Ca. USA > > more about sqb > _______________________________________________ > use-livecode mailing list > use-livecode at lists.runrev.com > Please visit this url to subscribe, unsubscribe and manage your > subscription preferences: > http://lists.runrev.com/mailman/listinfo/use-livecode > From andre at andregarzia.com Thu Jul 28 13:39:18 2011 From: andre at andregarzia.com (Andre Garzia) Date: Thu, 28 Jul 2011 14:39:18 -0300 Subject: Livecode Lockups In-Reply-To: <291762D3-0EF6-40C3-B3F5-47DE06A6E67A@twft.com> References: <291762D3-0EF6-40C3-B3F5-47DE06A6E67A@twft.com> Message-ID: Bob, check the pending messages for something odd... :-/ On Thu, Jul 28, 2011 at 2:21 PM, Bob Sneidar wrote: > Hi all. > > I am using 4.6.3. It has been beach-balling quite a lot since I upgraded > from 4.6.2. I just suppressed messages so I could go from card to card > without executing any scripts. I went back a few cards and it beach-balled > again. Correct me if I am wrong, but since suppress messages was turned on, > this CANNOT BE a problem with any of my scripts, right? > > Bob > > > _______________________________________________ > use-livecode mailing list > use-livecode at lists.runrev.com > Please visit this url to subscribe, unsubscribe and manage your > subscription preferences: > http://lists.runrev.com/mailman/listinfo/use-livecode > -- http://www.andregarzia.com All We Do Is Code. From bobs at twft.com Thu Jul 28 13:40:33 2011 From: bobs at twft.com (Bob Sneidar) Date: Thu, 28 Jul 2011 10:40:33 -0700 Subject: Behavior not working?? Message-ID: <4B2D3742-521C-4DA5-916B-1EEB88C876D7@twft.com> Hi all. I set the behavior of a card to the long id of a button on the first card of a stack. In that button I have an openCard handler. At first it worked famously, but now when I call openCard from the message box on the card with the behavior set, I get an error that it cannot find the handler openCard. I put the behavior of the card in the message box and it is indeed set to the long ID of that button, but for whatever reason the behavior is not getting triggered. I set the behavior of the card to empty and then back to the behavior of the button, but no joy. Any ideas? Bob From bobs at twft.com Thu Jul 28 13:52:08 2011 From: bobs at twft.com (Bob Sneidar) Date: Thu, 28 Jul 2011 10:52:08 -0700 Subject: Livecode Lockups In-Reply-To: References: <291762D3-0EF6-40C3-B3F5-47DE06A6E67A@twft.com> Message-ID: <44C67AD7-8E54-40A7-A527-A7B645579583@twft.com> Can't the app is locked up. Bob On Jul 28, 2011, at 10:39 AM, Andre Garzia wrote: > Bob, > > check the pending messages for something odd... > > > :-/ > > On Thu, Jul 28, 2011 at 2:21 PM, Bob Sneidar wrote: > >> Hi all. >> >> I am using 4.6.3. It has been beach-balling quite a lot since I upgraded >> from 4.6.2. I just suppressed messages so I could go from card to card >> without executing any scripts. I went back a few cards and it beach-balled >> again. Correct me if I am wrong, but since suppress messages was turned on, >> this CANNOT BE a problem with any of my scripts, right? >> >> Bob >> >> >> _______________________________________________ >> use-livecode mailing list >> use-livecode at lists.runrev.com >> Please visit this url to subscribe, unsubscribe and manage your >> subscription preferences: >> http://lists.runrev.com/mailman/listinfo/use-livecode >> > > > > -- > http://www.andregarzia.com All We Do Is Code. > _______________________________________________ > use-livecode mailing list > use-livecode at lists.runrev.com > Please visit this url to subscribe, unsubscribe and manage your subscription preferences: > http://lists.runrev.com/mailman/listinfo/use-livecode From bobs at twft.com Thu Jul 28 13:53:06 2011 From: bobs at twft.com (Bob Sneidar) Date: Thu, 28 Jul 2011 10:53:06 -0700 Subject: Behavior not working?? Message-ID: <425AEF8D-88FF-4FAB-B9FB-ED68D5D68620@twft.com> NVM I removed a suspected plugin which shall remain nameless and behaviors seem to be working again. Bob > Hi all. > > I set the behavior of a card to the long id of a button on the first card of a stack. In that button I have an openCard handler. At first it worked famously, but now when I call openCard from the message box on the card with the behavior set, I get an error that it cannot find the handler openCard. I put the behavior of the card in the message box and it is indeed set to the long ID of that button, but for whatever reason the behavior is not getting triggered. > > I set the behavior of the card to empty and then back to the behavior of the button, but no joy. Any ideas? > > Bob > > From bobs at twft.com Thu Jul 28 13:56:23 2011 From: bobs at twft.com (Bob Sneidar) Date: Thu, 28 Jul 2011 10:56:23 -0700 Subject: Behavior not working?? Message-ID: I think I know what happened. I may have has a window open from the unmentioned plugin when I issued the command to set the behavior. I may have inadvertently set the behavior of the plugin's open card to that of my own button! ACK! I can simply reinstall a copy of the plugin. Note to everyone then, be sure you have selected what you think you have selected before willy nilly assigning behaviors. Bob > NVM I removed a suspected plugin which shall remain nameless and behaviors seem to be working again. > > Bob > > >> Hi all. >> >> I set the behavior of a card to the long id of a button on the first card of a stack. In that button I have an openCard handler. At first it worked famously, but now when I call openCard from the message box on the card with the behavior set, I get an error that it cannot find the handler openCard. I put the behavior of the card in the message box and it is indeed set to the long ID of that button, but for whatever reason the behavior is not getting triggered. >> >> I set the behavior of the card to empty and then back to the behavior of the button, but no joy. Any ideas? >> >> Bob >> > From andre at andregarzia.com Thu Jul 28 14:00:18 2011 From: andre at andregarzia.com (Andre Garzia) Date: Thu, 28 Jul 2011 15:00:18 -0300 Subject: FTP Error In-Reply-To: References: Message-ID: Jack, can you post the content of the log field here? cheers andre On Thu, Jul 28, 2011 at 2:16 PM, JACK RARICK wrote: > Andre! Thanks for the tip to use liburlsetlogfield! If anything I will > learn a lot more a lot faster. > > Here is my line of code: > put "ftp://" & FTPUSER & ":" & FTPPASS & "@" & " > ftp.cva.on-rev.com/public_ftp/holtfootball"&"/"&gamefilm&"/Media/"&tFilename > into xStr > > I am a football coach. (American) > holtfootball is the school name. > gamefilm is the name of the specific game. > tFilename is the name of the particular "clip" of the game. > > The log field says that the path is getting their "correctly". (At least > everything is spelled right etc.) Although the path itself may be wrong. > > But on reading the log field, those pesky underscores are returned. > > I tried the public_html directory as well. No go. > > Thanks - I really appreciate all the help. What am I forgetting? > > Jack Rarick > Freshmen Center > Earth Science > Technology Coodinator > Varsity Men's Track & Field > Varsity Football - Offensive Line Coach, Peanut Butter Coordinator > _______________________________________________ > use-livecode mailing list > use-livecode at lists.runrev.com > Please visit this url to subscribe, unsubscribe and manage your > subscription preferences: > http://lists.runrev.com/mailman/listinfo/use-livecode > -- http://www.andregarzia.com All We Do Is Code. From JRARICK at hpsk12.net Thu Jul 28 14:02:14 2011 From: JRARICK at hpsk12.net (JACK RARICK) Date: Thu, 28 Jul 2011 14:02:14 -0400 Subject: FTP Error In-Reply-To: References: , Message-ID: Here is the contents of the log field. THANKS! socket selected: 74.54.153.71:21|6973 220---------- Welcome to Pure-FTPd [privsep] [TLS] ---------- 220-You are user number 3 of 50 allowed. 220-Local time is now 12:54. Server port: 21. 220-IPv6 connections are also welcome on this server. 220 You will be disconnected after 15 minutes of inactivity. USER cva at cva.on-rev.com 331 User cva at cva.on-rev.com OK. Password required PASS 230 OK. Current restricted directory is / 257 "/" is your current location TYPE I 200 TYPE is now 8-bit binary SIZE /public_ftp/Hartland.iMovieProject /Media/Clip 009.mov 550 Can't check for file existence PASV 227 Entering Passive Mode (74,54,153,71,8,155) RETR /public_ftp/Hartland.iMovieProject /Media/Clip 009.mov 550 Can't open /public_ftp/Hartland.iMovieProject_/Media/Clip 009.mov_: No such file or directory QUIT 221-Goodbye. You uploaded 0 and downloaded 0 kbytes. 221 Logout. Jack Rarick Freshmen Center Earth Science Technology Coodinator Varsity Men's Track & Field Varsity Football - Offensive Line Coach, Peanut Butter Coordinator From andre at andregarzia.com Thu Jul 28 14:13:51 2011 From: andre at andregarzia.com (Andre Garzia) Date: Thu, 28 Jul 2011 15:13:51 -0300 Subject: FTP Error In-Reply-To: References: Message-ID: >From that log file, you are trying to reach a file that does not exist. I believe you are pointing to the wrong folder. It is probably not "/public_html" but "something/public_html"... On Thu, Jul 28, 2011 at 3:02 PM, JACK RARICK wrote: > Here is the contents of the log field. > THANKS! > > socket selected: 74.54.153.71:21|6973 > 220---------- Welcome to Pure-FTPd [privsep] [TLS] ---------- > > 220-You are user number 3 of 50 allowed. > > 220-Local time is now 12:54. Server port: 21. > > 220-IPv6 connections are also welcome on this server. > > 220 You will be disconnected after 15 minutes of inactivity. > > USER cva at cva.on-rev.com > 331 User cva at cva.on-rev.com OK. Password required > > PASS > 230 OK. Current restricted directory is / > > 257 "/" is your current location > > TYPE I > 200 TYPE is now 8-bit binary > > SIZE /public_ftp/Hartland.iMovieProject > /Media/Clip 009.mov > > 550 Can't check for file existence > > PASV > 227 Entering Passive Mode (74,54,153,71,8,155) > > RETR /public_ftp/Hartland.iMovieProject > /Media/Clip 009.mov > > 550 Can't open /public_ftp/Hartland.iMovieProject_/Media/Clip 009.mov_: No > such file or directory > > QUIT > 221-Goodbye. You uploaded 0 and downloaded 0 kbytes. > > 221 Logout. > > > > Jack Rarick > Freshmen Center > Earth Science > Technology Coodinator > Varsity Men's Track & Field > Varsity Football - Offensive Line Coach, Peanut Butter Coordinator > _______________________________________________ > use-livecode mailing list > use-livecode at lists.runrev.com > Please visit this url to subscribe, unsubscribe and manage your > subscription preferences: > http://lists.runrev.com/mailman/listinfo/use-livecode > -- http://www.andregarzia.com All We Do Is Code. From rjb at robelko.com Thu Jul 28 14:29:47 2011 From: rjb at robelko.com (Robert Brenstein) Date: Thu, 28 Jul 2011 20:29:47 +0200 Subject: FTP Error In-Reply-To: References: , Message-ID: On 28.07.2011 at 14:02 Uhr -0400 JACK RARICK apparently wrote: > >RETR /public_ftp/Hartland.iMovieProject >/Media/Clip 009.mov > >550 Can't open /public_ftp/Hartland.iMovieProject_/Media/Clip >009.mov_: No such file or directory > There seem to be a space in the file name which terminates that path prematurely, causing the file not found error, I believe. Robert From JRARICK at hpsk12.net Thu Jul 28 14:37:32 2011 From: JRARICK at hpsk12.net (JACK RARICK) Date: Thu, 28 Jul 2011 14:37:32 -0400 Subject: FTP Error In-Reply-To: References: , , Message-ID: >From Robert: There seem to be a space in the file name which terminates that path prematurely, causing the file not found error, I believe. YES! There is a space - how do I get rid of it? Jack Rarick Freshmen Center Earth Science Technology Coodinator Varsity Men's Track & Field Varsity Football - Offensive Line Coach, Peanut Butter Coordinator ________________________________________ From: use-livecode-bounces at lists.runrev.com [use-livecode-bounces at lists.runrev.com] On Behalf Of Robert Brenstein [rjb at robelko.com] Sent: Thursday, July 28, 2011 2:29 PM To: How to use LiveCode Subject: RE: FTP Error On 28.07.2011 at 14:02 Uhr -0400 JACK RARICK apparently wrote: > >RETR /public_ftp/Hartland.iMovieProject >/Media/Clip 009.mov > >550 Can't open /public_ftp/Hartland.iMovieProject_/Media/Clip >009.mov_: No such file or directory > Robert _______________________________________________ use-livecode mailing list use-livecode at lists.runrev.com Please visit this url to subscribe, unsubscribe and manage your subscription preferences: http://lists.runrev.com/mailman/listinfo/use-livecode From mcgrath3 at mac.com Thu Jul 28 15:06:26 2011 From: mcgrath3 at mac.com (Thomas McGrath III) Date: Thu, 28 Jul 2011 15:06:26 -0400 Subject: [OT] More Apple Foolishness In-Reply-To: <76870884-CAEE-4396-B191-B2C540ACA93B@verizon.net> References: <6F918963-BD07-4351-83D1-41E1A78D06FD@altuit.com> <00af01cc4cc3$83a64f30$8af2ed90$@com> <80001DA7-522D-4AB6-A812-501492B660BF@mac.com> <76870884-CAEE-4396-B191-B2C540ACA93B@verizon.net> Message-ID: <0FDF1317-2F02-4E8A-9AF1-69AC5E11E7E3@mac.com> I use Printopia which then allows me to 'print' any file on the iPad or iPhone to Dropbox. Works great with Keynote presentations. -- Tom McGrath III http://lazyriver.on-rev.com 3mcgrath at comcast.net On Jul 28, 2011, at 5:01 AM, Colin Holgate wrote: > I hardly ever use iTunes for getting things onto my devices, but if you are an iTunes user you can move app icons from any of the screens to any other of the screes. Needs a bit of scrolling if you have 11 screen's worth like I do, but at least you can get everything in the order you want it, then sync your device. > > > > _______________________________________________ > use-livecode mailing list > use-livecode at lists.runrev.com > Please visit this url to subscribe, unsubscribe and manage your subscription preferences: > http://lists.runrev.com/mailman/listinfo/use-livecode From chipp at chipp.com Thu Jul 28 15:11:45 2011 From: chipp at chipp.com (Chipp Walters) Date: Thu, 28 Jul 2011 14:11:45 -0500 Subject: [OT] More Apple Foolishness In-Reply-To: <0FDF1317-2F02-4E8A-9AF1-69AC5E11E7E3@mac.com> References: <6F918963-BD07-4351-83D1-41E1A78D06FD@altuit.com> <00af01cc4cc3$83a64f30$8af2ed90$@com> <80001DA7-522D-4AB6-A812-501492B660BF@mac.com> <76870884-CAEE-4396-B191-B2C540ACA93B@verizon.net> <0FDF1317-2F02-4E8A-9AF1-69AC5E11E7E3@mac.com> Message-ID: Thanks Tom, I'll check it out. As I mentioned the DropDav is fairly expensive for doing just one thing-- and I hate subscription auto-billed software. On Thu, Jul 28, 2011 at 2:06 PM, Thomas McGrath III wrote: > I use Printopia which then allows me to 'print' any file on the iPad or > iPhone to Dropbox. > > Works great with Keynote presentations. > From richmondmathewson at gmail.com Thu Jul 28 15:12:33 2011 From: richmondmathewson at gmail.com (Richmond Mathewson) Date: Thu, 28 Jul 2011 22:12:33 +0300 Subject: LC 4.6.3 on Linux Message-ID: <4E31B4A1.2070708@gmail.com> Heather very kindly sorted out things so i could download a 30-day "go" with 4.6.3. So; downloaded the Linux version, and after install on Ubuntu 11.04 got a "setting defaults" message that made me, finally, kill the thing, having gone away, made supper, eaten supper, talked extensively (and pompously) on various sententious topics to my children (both of whom are counting the days until they can leave home), and generally misbehaved as befits a middle-aged f**t. No real joy there. Will "have a go" doing the same on my PPC running "Tiger" tomorrow evening. From chipp at chipp.com Thu Jul 28 15:14:12 2011 From: chipp at chipp.com (Chipp Walters) Date: Thu, 28 Jul 2011 14:14:12 -0500 Subject: [OT] More Apple Foolishness In-Reply-To: References: <6F918963-BD07-4351-83D1-41E1A78D06FD@altuit.com> <00af01cc4cc3$83a64f30$8af2ed90$@com> <80001DA7-522D-4AB6-A812-501492B660BF@mac.com> <76870884-CAEE-4396-B191-B2C540ACA93B@verizon.net> Message-ID: Thanks Andre for the advice. One of the reasons I don't use Android is the lack of software. On the iPad, I depend heavily on Keynote and Notify, two must have apps and neither are on Android. FlipBoard is nice too. On Thu, Jul 28, 2011 at 9:37 AM, Andre Garzia wrote: > Chipp, there are lots of Android phones out there in different levels of > crappiness (<-- I invented this word). If you decide to get one, I > recommend > the Nexus S because that is the Google Vanilla phone, no vendor stuff, just > plain android with no customizations, it is better than the stuff that HTC > and Motorola are shipping. > > From chipp at chipp.com Thu Jul 28 15:16:12 2011 From: chipp at chipp.com (Chipp Walters) Date: Thu, 28 Jul 2011 14:16:12 -0500 Subject: horizontal scrollbar in revBrowser In-Reply-To: <35ACBE58-4C86-4858-B824-23AD2D7E28EC@twft.com> References: <00b001cc4cc9$41afa7b0$c50ef710$@com> <35ACBE58-4C86-4858-B824-23AD2D7E28EC@twft.com> Message-ID: Nope, that's CSS ;-) On Thu, Jul 28, 2011 at 10:54 AM, Bob Sneidar wrote: > Wha?? Now I have to learn HTML?? ;-) > > From richmondmathewson at gmail.com Thu Jul 28 15:23:03 2011 From: richmondmathewson at gmail.com (Richmond Mathewson) Date: Thu, 28 Jul 2011 22:23:03 +0300 Subject: AW: OT: I want to buy Lion In-Reply-To: References: <001801cc4c80$41078b00$c316a100$@de> <9308DB19-829D-41A0-A887-B2F59C1C4CF4@mac.com> <001101cc4cf1$75d822b0$61886810$@de> <201107280226.10849.warren@warrensweb.us> <49A067C7-D033-4068-B7A1-5035CAA7FD7B@twft.com> Message-ID: <4E31B717.2020001@gmail.com> On 07/28/2011 08:02 PM, Pete Haworth wrote: > This is not about people who do not want to upgrade, it's about people who > do want to upgrade, are willing to pay, need to support the users of their > applications, but cannot because of Apple's upgrade methodology. Sounds > like they will provide a "hard copy" method to upgrade eventually. > > Pete > > > I don't know what is wrong with me; but reading this thread I really do want to get back to a very nice All-In-One Performa, running Mac OS 8.6 that currently is "resting" in my attic in my house in Scotland. I think of Mac OS 8.5 - 9.2.2, and 10.3 - 10.5 as really rather good, insofar as one could do a myriad of things without too much bother (especially when I remember I did my whole Windows-based MSc course work on 10.3 with Windows XP running in Virtual PC). I am well aware that 10.0 to 10.2 were intermediate builds, which apple, naughtily charged people for; let's hope "Lion" is the same sort of thing, and they don't manage to lose loads of customers before they produce some sort of mature "Mac OS XI". I think that Apple are digging themselves a hole. Microsoft, for all their many, manifest sins, have tried their best re backwards compatibility so that an OS upgrade doesn't necessarily entail thousands of bucks, quid, euros, ????, ????? or whatever in ancilliary upgrades. Iff I buy a new mac Mini it will ONLY be for one reason; to run LiveCode (face it, the Linux version still has some hiccups); all other software would be open source, which on the whole runs better on systems running Linux. From mikekann at yahoo.com Thu Jul 28 15:40:08 2011 From: mikekann at yahoo.com (Michael Kann) Date: Thu, 28 Jul 2011 12:40:08 -0700 (PDT) Subject: on-rev livecodeServer 4.6.3 .irev .lc In-Reply-To: Message-ID: <1311882008.75385.YahooMailClassic@web161612.mail.bf1.yahoo.com> Andre, Thanks for the info. It looks like you can use either the old irev server or the new liveCodeServer server, depending on the file extension you use. ----------------------------- file: get_version.irev output: 3.5 -------------------------- -------------------------- file: get_version.lc output: 4.6.3 -------------------------- -------------------------- file: get_version.irev output: JUNK as expected -------------------------- -------------------------- file: get_version.lc output: 4.6.3 --- the file extension decides which version you get, --- you can use either wrote: From: Andre Garzia Subject: Re: on-rev livecodeServer 4.6.3 .irev .lc To: "How to use LiveCode" Date: Thursday, July 28, 2011, 9:38 AM Michael, I think on-rev might be updated, do a: and see if it is something like 4.6.3... On Thu, Jul 28, 2011 at 9:28 AM, Michael Kann wrote: > Fellow webmeisters, > > I just read the release notes for the livecodeServer 4.6.3 and was > wondering if the on-rev server has been changed over from revServer to > livecodeServer and what that means for the users.? I tried using the ?> syntax and it worked. Are "lc" and "irev" interchangeable now? Or does > "lc" get you the new server and "irev" the old one? I'm especially > interested in the UNICODE functions. Any gotchas get anyone yet? Thanks as > always. > > Mike > > > _______________________________________________ > use-livecode mailing list > use-livecode at lists.runrev.com > Please visit this url to subscribe, unsubscribe and manage your > subscription preferences: > http://lists.runrev.com/mailman/listinfo/use-livecode > -- http://www.andregarzia.com All We Do Is Code. _______________________________________________ use-livecode mailing list use-livecode at lists.runrev.com Please visit this url to subscribe, unsubscribe and manage your subscription preferences: http://lists.runrev.com/mailman/listinfo/use-livecode From gandalf at doctorTimothyMiller.com Thu Jul 28 16:40:29 2011 From: gandalf at doctorTimothyMiller.com (Timothy Miller) Date: Thu, 28 Jul 2011 13:40:29 -0700 Subject: Where does survive the inventive user ? In-Reply-To: References: <8CE1A70D9625EA6-2550-214DE@web-mmc-d04.sysops.aol.com> Message-ID: A one-seat, one-platform version of LiveCode is quite affordable. Here's a scheme that might draw new, untrained users. Make LC into some kind of a game. It starts with most of LC's features crippled or hidden. To unlock features you have to solve challenges. Step one, obviously -- Make a field, a button, and a script that puts "hello world" into the field. That unlocks one or two more commands, properties, objects, or whatever. Then you've got to do something a little harder, and so on. Lots of hints, prompts and mini-tutorials along the way. A moderately intelligent user who goofs around with it now and then could get pretty skilled within six months, or less. I'm not in favor of Open Source for LC, for the usual reasons. It might be interesting to see what happens if a crippled form of LC is sold as a game, as described above, at a rather low price, perhaps free, with copyrights protected. It could catch, on virally perhaps. Those who become skilled and remain interested could upgrade to the full-featured version. There's no obvious reason the "HyperCard revolution" could not happen again. I'd love to see it. I meet lots of young people who want to learn to "program." Most of them don't even know what that means, or they think running a malware-dection app and reinstalling the OS is "programming." When I was first learning hyperCard, I had a HyperCard stack that taught you how to use HyperCard. That's how I started. Don't remember much about it. Cheers, Tim On Jul 28, 2011, at 9:05 AM, Bob Sneidar wrote: > I'm going to say doom. I purchased the lifetime On-Rev and the 5 year license when it was offered, partly because I want to see these guys thrive. If they do not, then sooner or later Livecode is destined to fail. So I invested in them when they needed capital to grow. If they had stock I would probably by some. What if they had faltered back in the Revolution 2.0 days? I hate to think of having to do things without a datagrid, without behaviors that make things like sqlYoga possible. That was HUGE! > > Also, it's the focus on making Livecode a particular thing, and not what a lot of other developers want it to be that lends itself to continued innovation along "the right lines" and I think Open Source would not maintain that vision. RunRev takes great care to prevent making other people's past projects obsolete by ensuring the way things currently work will work tomorrow (sometimes to my disappointment). I do not think that Open Sourcing Livecode would preserve that consideration for backwards compatibility. > > Bob > From kray at sonsothunder.com Thu Jul 28 17:14:50 2011 From: kray at sonsothunder.com (Ken Ray) Date: Thu, 28 Jul 2011 16:14:50 -0500 Subject: on-rev livecodeServer 4.6.3 .irev .lc In-Reply-To: <1311882008.75385.YahooMailClassic@web161612.mail.bf1.yahoo.com> References: <1311882008.75385.YahooMailClassic@web161612.mail.bf1.yahoo.com> Message-ID: <16D770C6-81DE-4969-BEA1-0835240E01AB@sonsothunder.com> On Jul 28, 2011, at 2:40 PM, Michael Kann wrote: > Andre, > > Thanks for the info. It looks like you can use either the old irev server or the new liveCodeServer server, depending on the file extension you use. Cool! I've contacted the developer who makes Textastic (http://www.textasticapp.com/) an awesome iOS source-code editor for the iPad, to update his syntax colorization to support LiveCode files with .lc and .irev extensions. It *currently* supports LiveCode with a ".lcs" extension, but that was way back at the beginning when there wasn't an official extension for text-based LC files. I'll let everyone know when the update to Textastic accepts the new extensions... Ken Ray Sons of Thunder Software, Inc. Email: kray at sonsothunder.com Web Site: http://www.sonsothunder.com/ From gwendalwood at cox.net Thu Jul 28 17:37:30 2011 From: gwendalwood at cox.net (gwendalwood at cox.net) Date: Thu, 28 Jul 2011 14:37:30 -0700 Subject: iPad directory format Message-ID: <20110728173730.PMU6C.1366459.imail@fed1rmwml34> On a previous post, I wrote: I have an app that on the Mac, opens, read the files in a folder called "Songs" and put the names of the files into a field. When the user clicks on the name in the listfield, the player plays that song. Works great! However, ......when I compile and run on the iPad, nothing I can find allows me to add those songs to the iPad using iTunes or whatever and then find those songs on the iPad. I assumed that adding songs to the Music folder in iTunes would put the song files into a folder called Music and then I could list the files in that folder and put them into the fld in my application and play them. Nothing I do works. I have tried writing a text file, and that is successful. Any suggestions? Thanks John responded with a sample stack that works in the Simulator, but not in the iPad. The method was to copy the files sent via the Copy files panel in the standalone setup from the engine folder to the documents folder. As I said, this works for the Simulator, but not for the standalone. Anyone else have another solution? Thanks, George Wood gwendalwood at cox.net From hyberson at gmail.com Thu Jul 28 17:43:03 2011 From: hyberson at gmail.com (Hyberson Pereira) Date: Thu, 28 Jul 2011 18:43:03 -0300 Subject: Installing iOS SDK and Xcode on Windows 7. Message-ID: Hi everyone. I'm new here. Has anyone read http://ipodtoucher55.blogspot.com/2010/12/installing-ios-sdk-and-xcode-on-windows.html? Unfortunately I can't test the procedure. The important question to me would be: if it is possible to use iOS SDK on Windows, will it be possible to use LiveCode for iOS on Windows? Thank you for your attention. Best regards, Hyberson From chipp at chipp.com Thu Jul 28 18:00:07 2011 From: chipp at chipp.com (Chipp Walters) Date: Thu, 28 Jul 2011 17:00:07 -0500 Subject: Where does survive the inventive user ? In-Reply-To: References: <8CE1A70D9625EA6-2550-214DE@web-mmc-d04.sysops.aol.com> Message-ID: Timothy, I'm not sure I agree with this statement. Apple, with all it's marketing prowess, and free version of HC, and included on every Mac, with no competition from the Internet, and seriously hyped by all, still couldn't make it work. Let's not forget, HC was a TCP/IP stack away from BEING a first browser ( http://www.isegoria.net/2008/05/hypercard-what-could-have-been/), so I'm don't think it could happen again-- though of course I would be rooting for it! On Thu, Jul 28, 2011 at 3:40 PM, Timothy Miller < gandalf at doctortimothymiller.com> wrote: > > There's no obvious reason the "HyperCard revolution" could not happen > again. I'd love to see it. > > From pepetoo at cox.net Thu Jul 28 18:49:22 2011 From: pepetoo at cox.net (Joe Lewis Wilkins) Date: Thu, 28 Jul 2011 15:49:22 -0700 Subject: Home Stack Error Message-ID: <00A66F97-268A-48A5-9222-ECBCE3AE3E52@cox.net> The following error appeared when I opened one of the sub-stacks in my most recent project. There is a Splash Mainstack with 6 sub-stacks. The one that was being opened is an 8 MB stack. All the others are much smaller. Opening it from a Menu option does so without the error, but opening it using the CommandKey equivalent is when the error appears. "There was an error executing a script in stack Home. No more information is available because the stack is password protected." If I ignore this, will it eventually come up to bite me in the hind-quarters or should I be doing lsomething to protect the future integrity of the project? TIA, Joe Wilkins From bobs at twft.com Thu Jul 28 18:50:37 2011 From: bobs at twft.com (Bob Sneidar) Date: Thu, 28 Jul 2011 15:50:37 -0700 Subject: Where does survive the inventive user ? In-Reply-To: References: <8CE1A70D9625EA6-2550-214DE@web-mmc-d04.sysops.aol.com> Message-ID: <54729309-D39D-4D70-A239-BC3BA825F5E9@twft.com> I don't agree that Hypercard didn't work. It worked amazingly! Just not as a mainstream development environment, but it was never marketed or presented as such. A lot of people wrote Xcmd's for it. One guy wrote an Xcmd that allowed you to access a dBase database file and read and write to it. It wasn't very good though, very buggy, but the concept was sound. I think Hypercard happened too early, and lacked so many things for so long that people eventually went elsewhere. It took them forever to include color support, and then it wasn't very good, and Apple had already been trying to dump it for some time. It was a half hearted effort on Apple's part that really spelled the doom of Hypercard, and who can blame them? It wasn't exactly a profit center! Bob On Jul 28, 2011, at 3:00 PM, Chipp Walters wrote: > Timothy, > > I'm not sure I agree with this statement. Apple, with all it's marketing > prowess, and free version of HC, and included on every Mac, with no > competition from the Internet, and seriously hyped by all, still couldn't > make it work. > > Let's not forget, HC was a TCP/IP stack away from BEING a first browser ( > http://www.isegoria.net/2008/05/hypercard-what-could-have-been/), so I'm > don't think it could happen again-- though of course I would be rooting for > it! > > On Thu, Jul 28, 2011 at 3:40 PM, Timothy Miller < > gandalf at doctortimothymiller.com> wrote: > >> >> There's no obvious reason the "HyperCard revolution" could not happen >> again. I'd love to see it. >> >> > _______________________________________________ > use-livecode mailing list > use-livecode at lists.runrev.com > Please visit this url to subscribe, unsubscribe and manage your subscription preferences: > http://lists.runrev.com/mailman/listinfo/use-livecode From stephenREVOLUTION2 at barncard.com Thu Jul 28 18:51:13 2011 From: stephenREVOLUTION2 at barncard.com (stephen barncard) Date: Thu, 28 Jul 2011 15:51:13 -0700 Subject: Where does survive the inventive user ? In-Reply-To: References: <8CE1A70D9625EA6-2550-214DE@web-mmc-d04.sysops.aol.com> Message-ID: I know, I tried to do it myself in the early 90s. TCP/IP on the mac was quite unreliable. On 28 July 2011 15:00, Chipp Walters wrote: > > > Let's not forget, HC was a TCP/IP stack away from BEING a first browser ( > http://www.isegoria.net/2008/05/hypercard-what-could-have-been/), so I'm > don't think it could happen again-- though of course I would be rooting for > it! > Stephen Barncard San Francisco Ca. USA more about sqb From bobs at twft.com Thu Jul 28 18:53:27 2011 From: bobs at twft.com (Bob Sneidar) Date: Thu, 28 Jul 2011 15:53:27 -0700 Subject: FTP Error In-Reply-To: References: , , Message-ID: Never use spaces for web files? Bob On Jul 28, 2011, at 11:37 AM, JACK RARICK wrote: > From Robert: > > There seem to be a space in the file name which terminates that path > prematurely, causing the file not found error, I believe. > > YES! There is a space - how do I get rid of it? > > > Jack Rarick > Freshmen Center > Earth Science > Technology Coodinator > Varsity Men's Track & Field > Varsity Football - Offensive Line Coach, Peanut Butter Coordinator > ________________________________________ > From: use-livecode-bounces at lists.runrev.com [use-livecode-bounces at lists.runrev.com] On Behalf Of Robert Brenstein [rjb at robelko.com] > Sent: Thursday, July 28, 2011 2:29 PM > To: How to use LiveCode > Subject: RE: FTP Error > > On 28.07.2011 at 14:02 Uhr -0400 JACK RARICK apparently wrote: >> >> RETR /public_ftp/Hartland.iMovieProject >> /Media/Clip 009.mov >> >> 550 Can't open /public_ftp/Hartland.iMovieProject_/Media/Clip >> 009.mov_: No such file or directory >> > > > Robert > > _______________________________________________ > use-livecode mailing list > use-livecode at lists.runrev.com > Please visit this url to subscribe, unsubscribe and manage your subscription preferences: > http://lists.runrev.com/mailman/listinfo/use-livecode > > _______________________________________________ > use-livecode mailing list > use-livecode at lists.runrev.com > Please visit this url to subscribe, unsubscribe and manage your subscription preferences: > http://lists.runrev.com/mailman/listinfo/use-livecode From stephenREVOLUTION2 at barncard.com Thu Jul 28 18:56:55 2011 From: stephenREVOLUTION2 at barncard.com (stephen barncard) Date: Thu, 28 Jul 2011 15:56:55 -0700 Subject: FTP Error In-Reply-To: References: Message-ID: URLEncode On 28 July 2011 11:37, JACK RARICK wrote: > From Robert: > > There seem to be a space in the file name which terminates that path > prematurely, causing the file not found error, I believe. > > YES! There is a space - how do I get rid of it? > > > Jack RarickStephen Barncard > San Francisco Ca. USA more about sqb From matthias_livecode at me.com Thu Jul 28 19:03:05 2011 From: matthias_livecode at me.com (Matthias Rebbe) Date: Fri, 29 Jul 2011 01:03:05 +0200 Subject: FTP Error In-Reply-To: References: Message-ID: <40F5DFBF-13BA-49E8-B7A3-01698BE601A8@me.com> Hi Jack, if i am not wrong just replace the space with %20 in the filename. So Clip 009.mov will get Clip%20009.mov That should doing it. Or just try to avoid spaces in filenames for the web. Regards, Matthias Am 28.07.2011 um 20:37 schrieb JACK RARICK: > From Robert: > > There seem to be a space in the file name which terminates that path > prematurely, causing the file not found error, I believe. > > YES! There is a space - how do I get rid of it? > > > Jack Rarick > Freshmen Center > Earth Science > Technology Coodinator > Varsity Men's Track & Field > Varsity Football - Offensive Line Coach, Peanut Butter Coordinator > ________________________________________ > From: use-livecode-bounces at lists.runrev.com [use-livecode-bounces at lists.runrev.com] On Behalf Of Robert Brenstein [rjb at robelko.com] > Sent: Thursday, July 28, 2011 2:29 PM > To: How to use LiveCode > Subject: RE: FTP Error > > On 28.07.2011 at 14:02 Uhr -0400 JACK RARICK apparently wrote: >> >> RETR /public_ftp/Hartland.iMovieProject >> /Media/Clip 009.mov >> >> 550 Can't open /public_ftp/Hartland.iMovieProject_/Media/Clip >> 009.mov_: No such file or directory >> > > > Robert > > _______________________________________________ > use-livecode mailing list > use-livecode at lists.runrev.com > Please visit this url to subscribe, unsubscribe and manage your subscription preferences: > http://lists.runrev.com/mailman/listinfo/use-livecode > > _______________________________________________ > use-livecode mailing list > use-livecode at lists.runrev.com > Please visit this url to subscribe, unsubscribe and manage your subscription preferences: > http://lists.runrev.com/mailman/listinfo/use-livecode From niconiko at gmail.com Thu Jul 28 19:14:35 2011 From: niconiko at gmail.com (Nicolas Cueto) Date: Fri, 29 Jul 2011 08:14:35 +0900 Subject: stackfiles -- how to use? In-Reply-To: <4E2DC107.9030609@hyperactivesw.com> References: <4E2DB5D4.7020000@hyperactivesw.com> <4E2DC107.9030609@hyperactivesw.com> Message-ID: > Although in this case, the stack in > question is the main standalone stack, so it's always in use anyway. Interesting (and come to think of it, obvious). So does this mean that I do not need to include a "start using this stack" in my standalone to allow other stacks to call functions and handlers that are in the standalone? P.S. Thank you to those who've responded so far. Using your explanations, I've been experimenting with my stacks and learning how to unclutter them a bit. -- Nicolas Cueto From scott at tactilemedia.com Thu Jul 28 21:18:35 2011 From: scott at tactilemedia.com (Scott Rossi) Date: Thu, 28 Jul 2011 18:18:35 -0700 Subject: [OT] HTML5 Amazingness Message-ID: Requires Google Chrome: http://www.allisnotlo.st/ (Note: shut down all other tabs in Chrome for a better experience.) I think I actually said "OMG" out loud when I first saw this. Regards, Scott Rossi Creative Director Tactile Media, UX Design From gandalf at doctorTimothyMiller.com Thu Jul 28 22:24:56 2011 From: gandalf at doctorTimothyMiller.com (Timothy Miller) Date: Thu, 28 Jul 2011 19:24:56 -0700 Subject: Where does survive the inventive user ? In-Reply-To: <54729309-D39D-4D70-A239-BC3BA825F5E9@twft.com> References: <8CE1A70D9625EA6-2550-214DE@web-mmc-d04.sysops.aol.com> <54729309-D39D-4D70-A239-BC3BA825F5E9@twft.com> Message-ID: <5BA719DE-3AC9-403A-ABDF-CA8009CF8D43@doctorTimothyMiller.com> What Bob said. Tim On Jul 28, 2011, at 3:50 PM, Bob Sneidar wrote: > I don't agree that Hypercard didn't work. It worked amazingly! Just not as a mainstream development environment, but it was never marketed or presented as such. A lot of people wrote Xcmd's for it. One guy wrote an Xcmd that allowed you to access a dBase database file and read and write to it. It wasn't very good though, very buggy, but the concept was sound. > > I think Hypercard happened too early, and lacked so many things for so long that people eventually went elsewhere. It took them forever to include color support, and then it wasn't very good, and Apple had already been trying to dump it for some time. It was a half hearted effort on Apple's part that really spelled the doom of Hypercard, and who can blame them? It wasn't exactly a profit center! > > Bob From rjb at robelko.com Thu Jul 28 22:40:20 2011 From: rjb at robelko.com (Robert Brenstein) Date: Fri, 29 Jul 2011 04:40:20 +0200 Subject: Where does survive the inventive user ? In-Reply-To: <54729309-D39D-4D70-A239-BC3BA825F5E9@twft.com> References: <8CE1A70D9625EA6-2550-214DE@web-mmc-d04.sysops.aol.com> <54729309-D39D-4D70-A239-BC3BA825F5E9@twft.com> Message-ID: On 28.07.2011 at 15:50 Uhr -0700 Bob Sneidar apparently wrote: >I don't agree that Hypercard didn't work. It worked amazingly! Just >not as a mainstream development environment, but it was never >marketed or presented as such. A lot of people wrote Xcmd's for it. >One guy wrote an Xcmd that allowed you to access a dBase database >file and read and write to it. It wasn't very good though, very >buggy, but the concept was sound. > If I recall, HyperCard was called an "erector set" for Mac users, not necessarily programmers, and indeed used mostly by non-professional programmers. There was also an Xcmd for Valentina -- yes, I started using Valentina database with HyperCard -- and it worked really well but Valentina was not network based then. Robert From pepetoo at cox.net Fri Jul 29 01:05:06 2011 From: pepetoo at cox.net (Joe Lewis Wilkins) Date: Thu, 28 Jul 2011 22:05:06 -0700 Subject: A way to get LC out of the way Message-ID: Hi everyone, I'm struggling; maybe I should say "fighting" with LC. I have an app in which I do some things that create new cards in one of the sub-stacks. In the process of refining things I want to swap the original stack with the one that has the new cards, but LC always remembers the larger stack. After a while, I'm usually able to have the smaller one used, but it's a real effort. Have I missed some simple way of doing this. In most other programs, I would be able to just replace the large one with the small one. I can do it in this case too, but the small one is never seen. TIA, Joe Wilkins From niconiko at gmail.com Fri Jul 29 01:27:40 2011 From: niconiko at gmail.com (Nicolas Cueto) Date: Fri, 29 Jul 2011 14:27:40 +0900 Subject: memory, buffered images and closed stacks (and groups?) Message-ID: Hello. Actually, this is about a crash caused by the command "group". Which I explain further down. But first... How can I make sure that a closed stack releases all memory, i.e., that the stack is "really" closed? I'm prompted to ask cause of some odd memory results I'm encountering. More or less, here's the memory usage as shown by Windows' Task Manager: 1) Standalone and a central stack open = 38,000 K 2) Click a button on central stack so it hides itself and then opens a game stack = 40,000 K 3) Game stack does "stuff" to itself (cf. below) = 43,000 K 4) Close game stack and unhide central stack (standalone of course still open) = 45,000 K [Note that at (4), I confirm with "put the openstacks" that the standalone and the central stack are indeed the only open stacks.] The memory state at (4) seems significantly greater than at (1), is it not? Or is this normal for LC? About the "stuff" my game stack does to itself (!). When it opens, it clones up to 36 group objects, each bearing buffered images. Plus there's over 300 other buffered images, each of whose size property ranges at about 2000. Also, the stack resizes itself as the game goes along. Finally, about that "group" crash. Which is what's really prompting me to write this. Everything's ok the first time that game stack of mine runs. But, when I close it, return to the central stack, and then go back to the same game stack, kaboom! Took uninterrupted hours of trial-and-error till I isolated the problematic script line in the game stack. Here's the naughty bit: repeat for each item tTile in tGroupingData if there is not a group tTile then put "WTF!" into tMsg answer tMsg exit to top end if set the selected of group tTile to true end repeat group select empty The line "group" is when the crash happens, after "end repeat" and never getting to "select empty" (nor ever seeing a "WTF!"). I've already tried adding a "select empty" before the "repeat for", and a "wait xxx milliseconds with messages" after the line "group". So, now I'm trying the list :-) Thanks. -- Nicolas Cueto From kray at sonsothunder.com Fri Jul 29 01:40:40 2011 From: kray at sonsothunder.com (Ken Ray) Date: Fri, 29 Jul 2011 00:40:40 -0500 Subject: memory, buffered images and closed stacks (and groups?) In-Reply-To: References: Message-ID: On Jul 29, 2011, at 12:27 AM, Nicolas Cueto wrote: > Hello. > > Actually, this is about a crash caused by the command "group". Which I > explain further down. But first... > > How can I make sure that a closed stack releases all memory, i.e., > that the stack is "really" closed? You can do one of the following: 1) Set the 'destroyStack' property of the stack to true. WHen it closes, it should be purged from memory. 2) Issue the "delete stack " command, but BE CAREFUL!!! - if is a *substack*, the stack will ACTUALLY BE DELETED; if the stack is a *mainstack*, it will close the stack and remove it from memory. (I tend to use something like "if the mainstack of = the short name of then delete stack ".) The way you can test is to look at "the mainstacks".... if the name of the stack is no longer in "the mainstacks", then it has been closed and purged from memory. Ken Ray Sons of Thunder Software, Inc. Email: kray at sonsothunder.com Web Site: http://www.sonsothunder.com/ From revdev at pdslabs.net Fri Jul 29 01:44:46 2011 From: revdev at pdslabs.net (Phil Davis) Date: Thu, 28 Jul 2011 22:44:46 -0700 Subject: [OT] HTML5 Amazingness In-Reply-To: References: Message-ID: <4E3248CE.1030200@pdslabs.net> It sort of reaches the "magic" level for me! At some point I quit thinking, "How is it doing that?" and just took it in. Grinning and shaking my head. Phil Davis On 7/28/11 6:18 PM, Scott Rossi wrote: > Requires Google Chrome: > > http://www.allisnotlo.st/ > > (Note: shut down all other tabs in Chrome for a better experience.) > > I think I actually said "OMG" out loud when I first saw this. > > Regards, > > Scott Rossi > Creative Director > Tactile Media, UX Design > > > > _______________________________________________ > use-livecode mailing list > use-livecode at lists.runrev.com > Please visit this url to subscribe, unsubscribe and manage your subscription preferences: > http://lists.runrev.com/mailman/listinfo/use-livecode > -- Phil Davis PDS Labs Professional Software Development http://pdslabs.net From stephenREVOLUTION2 at barncard.com Fri Jul 29 01:52:10 2011 From: stephenREVOLUTION2 at barncard.com (stephen barncard) Date: Thu, 28 Jul 2011 22:52:10 -0700 Subject: [OT] HTML5 Amazingness In-Reply-To: References: Message-ID: I love this band. And the video presentation rocks. But why did they have to screw up the audio? 50% over-saturated peaks. Even louder than livesteam commercials. Video guys just don't care about audio. On 28 July 2011 18:18, Scott Rossi wrote: > Requires Google Chrome: > > http://www.allisnotlo.st/ > > (Note: shut down all other tabs in Chrome for a better experience.) > > I think I actually said "OMG" out loud when I first saw this. > > Regards, > > Scott Rossi > Creative Director > Tactile Media, UX Design > > > > _______________________________________________ > use-livecode mailing list > use-livecode at lists.runrev.com > Please visit this url to subscribe, unsubscribe and manage your > subscription preferences: > http://lists.runrev.com/mailman/listinfo/use-livecode > -- Stephen Barncard San Francisco Ca. USA more about sqb From stephenREVOLUTION2 at barncard.com Fri Jul 29 01:59:33 2011 From: stephenREVOLUTION2 at barncard.com (stephen barncard) Date: Thu, 28 Jul 2011 22:59:33 -0700 Subject: Where does survive the inventive user ? In-Reply-To: References: <8CE1A70D9625EA6-2550-214DE@web-mmc-d04.sysops.aol.com> <54729309-D39D-4D70-A239-BC3BA825F5E9@twft.com> Message-ID: Well I used it in a professional environment (AM Studios) for several years and nobody seemed to mention that it might seem non-professional. I have worked in 6502 and Z80 assembly for quite a while before the Mac and hypercard. HC was quite a libration from the tedium of just getting a window with a button on the screen in ZBasic and everybody was quite surprised and happy that the project got done on schedule and happily in use by the staff - so much I got a big bonus when Herb left the company. On 28 July 2011 19:40, Robert Brenstein wrote: > >> >> > If I recall, HyperCard was called an "erector set" for Mac users, not > necessarily programmers, and indeed used mostly by non-professional > programmers. There was also an Xcmd for Valentina -- yes, I started using > Valentina database with HyperCard -- and it worked really well but Valentina > was not network based then. > > Robert > > > Stephen Barncard San Francisco Ca. USA more about sqb From capellan2000 at gmail.com Fri Jul 29 02:10:26 2011 From: capellan2000 at gmail.com (Alejandro Tejada) Date: Thu, 28 Jul 2011 23:10:26 -0700 (PDT) Subject: Where does survive the inventive user? In-Reply-To: <465EFD52-8532-4945-A2F0-C2CE01371329@twft.com> References: <1311706987455-3696711.post@n4.nabble.com> <08F3632C-76A9-402E-97B4-C82090DC8D05@free.fr> <465EFD52-8532-4945-A2F0-C2CE01371329@twft.com> Message-ID: <1311919826020-3703159.post@n4.nabble.com> Thinking about inventive users and open source software... When I first read about the Open Source movement, I though that it was a group of developers that wanted to create software in the same way that an artist creates his work: A lasting work of art that trascend time because of his many outstanding and unique qualities... Hmmm, looks like I was wrong in my first impression about Open source, but just in case: Could anyone show me these outstanding artistic qualities in Open Source software? Al -- View this message in context: http://runtime-revolution.278305.n4.nabble.com/Where-does-survive-the-inventive-user-tp3696711p3703159.html Sent from the Revolution - User mailing list archive at Nabble.com. From capellan2000 at gmail.com Fri Jul 29 02:23:21 2011 From: capellan2000 at gmail.com (Alejandro Tejada) Date: Thu, 28 Jul 2011 23:23:21 -0700 (PDT) Subject: Where does survive the inventive user ? In-Reply-To: References: <8CE1A70D9625EA6-2550-214DE@web-mmc-d04.sysops.aol.com> <54729309-D39D-4D70-A239-BC3BA825F5E9@twft.com> Message-ID: <1311920601481-3703174.post@n4.nabble.com> Hi Stephen, Stephen Barncard-4 wrote: > > Well I used it in a professional environment (AM Studios) for several > years > and nobody seemed to mention that it might seem non-professional. I have > worked in 6502 and Z80 assembly for quite a while before the Mac and > hypercard. HC was quite a libration from the tedium of just getting a > window > with a button on the screen in ZBasic and everybody was quite surprised > and > happy that the project got done on schedule and happily in use by the > staff > - so much I got a big bonus when Herb left the company. > Nice history! Apple should have featured your use of HyperCard, back then. By the way, Herb is Herb Alpert? Al -- View this message in context: http://runtime-revolution.278305.n4.nabble.com/Where-does-survive-the-inventive-user-tp3698117p3703174.html Sent from the Revolution - User mailing list archive at Nabble.com. From scott at tactilemedia.com Fri Jul 29 02:41:02 2011 From: scott at tactilemedia.com (Scott Rossi) Date: Thu, 28 Jul 2011 23:41:02 -0700 Subject: [OT] HTML5 Amazingness In-Reply-To: <4E3248CE.1030200@pdslabs.net> Message-ID: No doubt. Years and years ago when I was using SuperCard, I was developing an app called CyberRave that was designed to mimic (as best as one could at that time) the experience of being at club, hearing dance music, and seeing some simple effects on your screen (background image changes, simulated laser lights, etc). I was also doing a stint with the HI group in Apple's eWorld, and we concocted a way to hook CyberRave up to eWorld's live chat services, where special chat commands would trigger the effects on connected systems. We set up our first rave in house, got the music going, and started sending chat messages back and forth. It lasted maybe a minute before crashing. But it was awesome. One of the features in CyberRave was a video wall, where a small QuickTime movie could be made to play cloned across a 16x16 grid, or the single movie occupying the entire grid. Seeing the OKGO multi-window experience brought back fond memories of doing almost exactly the same thing around 15 years ago... Regards, Scott Rossi Creative Director Tactile Media, UX Design Recently, Phil Davis wrote: > It sort of reaches the "magic" level for me! At some point I quit thinking, > "How > is it doing that?" and just took it in. Grinning and shaking my head. > > Phil Davis > > > On 7/28/11 6:18 PM, Scott Rossi wrote: >> Requires Google Chrome: >> >> http://www.allisnotlo.st/ >> >> (Note: shut down all other tabs in Chrome for a better experience.) >> >> I think I actually said "OMG" out loud when I first saw this. >> >> Regards, >> >> Scott Rossi >> Creative Director >> Tactile Media, UX Design >> >> >> >> _______________________________________________ >> use-livecode mailing list >> use-livecode at lists.runrev.com >> Please visit this url to subscribe, unsubscribe and manage your subscription >> preferences: >> http://lists.runrev.com/mailman/listinfo/use-livecode From slava at lexiconbridge.com Fri Jul 29 03:00:31 2011 From: slava at lexiconbridge.com (Slava Paperno) Date: Fri, 29 Jul 2011 03:00:31 -0400 Subject: browserBeforeNavigate isn't sent (now and then) Message-ID: <00ec01cc4dbd$38941bd0$a9bc5370$@com> The Web page in a revBrowser instance on the card has this: div onclick="this.className='highlightedEntry'; window.location='#?RecNum=450'" /div When the user clicks the div, the window.location clause triggers the browserBeforeNavigate message to be sent to the stack; the className change highlights the div. This works 95% of the time. When it fails, I know from the "on browserBeforeNavigate" handler that the message hasn't been sent--the handler is not called. The handler is in the card script. The failure happens in the IDE and in standalones in Windows and Mac OS X. It seems entirely random: I launch the standalone twenty times, and the click produces the expected results about 19 times out of 20. My only recourse is to close the app and re-launch it. Since the browserBeforeNavigate message isn't sent, and since I know from the highlighting that the click has registered, I don't know where I can insert any diagnostic check. My only hope is that someone who has used revBrowser a lot has seen this and has a suggestion. Thanks! Slava From andrew at rjdfarm.com Fri Jul 29 03:31:07 2011 From: andrew at rjdfarm.com (Andrew Kluthe) Date: Fri, 29 Jul 2011 00:31:07 -0700 (PDT) Subject: Installing iOS SDK and Xcode on Windows 7. In-Reply-To: References: Message-ID: <1311924667391-3703277.post@n4.nabble.com> I may be wrong, but I am pretty sure iOS dev is limited to Mac Only. :\ -- View this message in context: http://runtime-revolution.278305.n4.nabble.com/Installing-iOS-SDK-and-Xcode-on-Windows-7-tp3702599p3703277.html Sent from the Revolution - User mailing list archive at Nabble.com. From andre.bisseret at wanadoo.fr Fri Jul 29 03:37:34 2011 From: andre.bisseret at wanadoo.fr (=?iso-8859-1?Q?Andr=E9_Bisseret?=) Date: Fri, 29 Jul 2011 09:37:34 +0200 Subject: Behavior not working?? In-Reply-To: References: Message-ID: <6757E17A-6010-4F9B-8FF5-F4DD3BA8CEF5@wanadoo.fr> Bonjour Bob, Good advice! That happened to me several times when assigning behaviors from the message box, because generally I set the behavior of THIS cd to the long id etc. Not always but sometimes I think to set the behavior of cd "n" of stack "suchstack" which avoids the error ;-) Andr? Le 28 juil. 2011 ? 19:56, Bob Sneidar a ?crit : > I think I know what happened. I may have has a window open from the unmentioned plugin when I issued the command to set the behavior. I may have inadvertently set the behavior of the plugin's open card to that of my own button! ACK! I can simply reinstall a copy of the plugin. > or set the behavior of cd n of the plugin to empty > Note to everyone then, be sure you have selected what you think you have selected before willy nilly assigning behaviors. > > Bob > > > >> NVM I removed a suspected plugin which shall remain nameless and behaviors seem to be working again. >> >> Bob >> >> >>> Hi all. >>> >>> I set the behavior of a card to the long id of a button on the first card of a stack. In that button I have an openCard handler. At first it worked famously, but now when I call openCard from the message box on the card with the behavior set, I get an error that it cannot find the handler openCard. I put the behavior of the card in the message box and it is indeed set to the long ID of that button, but for whatever reason the behavior is not getting triggered. >>> >>> I set the behavior of the card to empty and then back to the behavior of the button, but no joy. Any ideas? >>> >>> Bob >>> >> > > _______________________________________________ > use-livecode mailing list > use-livecode at lists.runrev.com > Please visit this url to subscribe, unsubscribe and manage your subscription preferences: > http://lists.runrev.com/mailman/listinfo/use-livecode From niconiko at gmail.com Fri Jul 29 04:03:14 2011 From: niconiko at gmail.com (Nicolas Cueto) Date: Fri, 29 Jul 2011 17:03:14 +0900 Subject: memory, buffered images and closed stacks (and groups?) In-Reply-To: References: Message-ID: > 1) Set the 'destroyStack' property of the stack to true. WHen it closes, it should be purged from memory. > 2) Issue the "delete stack " command, but BE CAREFUL!!! - if is a *substack*, the stack will ACTUALLY BE DELETED; if the stack is a *mainstack*, it will close the stack and remove it from memory. (I tend to use something like "if the mainstack of = the short name of then delete stack ".) > > The way you can test is to look at "the mainstacks".... if the name of the stack is no longer in "the mainstacks", then it has been closed and purged from memory. Thanks Ken. Tried your suggestions. The game stack doesn't appear in the mainstacks, its destroyStack is true, and I do a "delete stack" even though mainstacks doesn't list the game stack. And yet, memory values after the game stack has been confirmed as closed are 2000 to 3000 K larger than when the game stack was first opened. So, I'm still wondering if this memory discrepancy is connected to my stack crashing immediately before using the command "group" after the game stack has been re-opened. -- Nicolas Cueto From shaosean at wehostmacs.com Fri Jul 29 04:14:20 2011 From: shaosean at wehostmacs.com (Shao Sean) Date: Fri, 29 Jul 2011 04:14:20 -0400 Subject: Installing iOS SDK and Xcode on Windows 7. Message-ID: > I may be wrong, but I am pretty sure iOS dev is limited to Mac > Only. :\ They are running the tools in a virtual machine so as far as the software is aware it is running on a Mac.. From chipp at chipp.com Fri Jul 29 06:09:41 2011 From: chipp at chipp.com (Chipp Walters) Date: Fri, 29 Jul 2011 05:09:41 -0500 Subject: Where does survive the inventive user ? In-Reply-To: References: <8CE1A70D9625EA6-2550-214DE@web-mmc-d04.sysops.aol.com> Message-ID: Yep. We wrote the first XCMD/Xtra for TCP/IP for Director. It was called XtraNet. Macromedia tried to buy it from us. On Thu, Jul 28, 2011 at 5:51 PM, stephen barncard < stephenREVOLUTION2 at barncard.com> wrote: > I know, I tried to do it myself in the early 90s. TCP/IP on the mac was > quite unreliable. > > On 28 July 2011 15:00, Chipp Walters wrote: > > > > > > > Let's not forget, HC was a TCP/IP stack away from BEING a first browser ( > > http://www.isegoria.net/2008/05/hypercard-what-could-have-been/), so I'm > > don't think it could happen again-- though of course I would be rooting > for > > it! > > > > Stephen Barncard > San Francisco Ca. USA > > more about sqb > _______________________________________________ > use-livecode mailing list > use-livecode at lists.runrev.com > Please visit this url to subscribe, unsubscribe and manage your > subscription preferences: > http://lists.runrev.com/mailman/listinfo/use-livecode > -- Chipp Walters CEO, Shafer Walters Group, Inc. From toolbook at kestner.de Fri Jul 29 07:00:17 2011 From: toolbook at kestner.de (Tiemo Hollmann TB) Date: Fri, 29 Jul 2011 13:00:17 +0200 Subject: AW: AW: OT: I want to buy Lion In-Reply-To: References: <001801cc4c80$41078b00$c316a100$@de> <9308DB19-829D-41A0-A887-B2F59C1C4CF4@mac.com> <001101cc4cf1$75d822b0$61886810$@de> <201107280226.10849.warren@warrensweb.us> <49A067C7-D033-4068-B7A1-5035CAA7FD7B@twft.com> Message-ID: <003401cc4dde$b7ad2d00$27078700$@de> Thanks Pete, that's what I tried to say Tiemo > -----Urspr?ngliche Nachricht----- > Von: use-livecode-bounces at lists.runrev.com [mailto:use-livecode- > bounces at lists.runrev.com] Im Auftrag von Pete Haworth > Gesendet: Donnerstag, 28. Juli 2011 19:02 > An: How to use LiveCode > Betreff: Re: AW: OT: I want to buy Lion > > This is not about people who do not want to upgrade, it's about people who > do want to upgrade, are willing to pay, need to support the users of their > applications, but cannot because of Apple's upgrade methodology. Sounds > like they will provide a "hard copy" method to upgrade eventually. > > Pete > From hyberson at gmail.com Fri Jul 29 07:39:06 2011 From: hyberson at gmail.com (Hyberson Pereira) Date: Fri, 29 Jul 2011 08:39:06 -0300 Subject: Installing iOS SDK and Xcode on Windows 7. In-Reply-To: References: Message-ID: Would LiveCode for iOS work in such an environment? On Fri, Jul 29, 2011 at 05:14, Shao Sean wrote: > I may be wrong, but I am pretty sure iOS dev is limited to Mac Only. :\ >> > > They are running the tools in a virtual machine so as far as the software > is aware it is running on a Mac.. > > > ______________________________**_________________ > use-livecode mailing list > use-livecode at lists.runrev.com > Please visit this url to subscribe, unsubscribe and manage your > subscription preferences: > http://lists.runrev.com/**mailman/listinfo/use-livecode > From williamdesmet at gmail.com Fri Jul 29 07:57:29 2011 From: williamdesmet at gmail.com (William de Smet) Date: Fri, 29 Jul 2011 13:57:29 +0200 Subject: Is Apple going to allow this? Message-ID: <5F0FAE3D-985F-4B2C-9CF0-97EAB4409E9E@gmail.com> Hi there, I am thinking of using an iMac (with no Apple logo) vector image in the splash screen of my first iOS app. Is Apple going to allow this? Or is it better to you use a more 'standard' monitor? Greetings, William ----- Verstuurd vanaf mijn iPhone! From m.schonewille at economy-x-talk.com Fri Jul 29 08:06:08 2011 From: m.schonewille at economy-x-talk.com (Mark Schonewille) Date: Fri, 29 Jul 2011 14:06:08 +0200 Subject: Is Apple going to allow this? In-Reply-To: <5F0FAE3D-985F-4B2C-9CF0-97EAB4409E9E@gmail.com> References: <5F0FAE3D-985F-4B2C-9CF0-97EAB4409E9E@gmail.com> Message-ID: Hi William, I've heard about apps that have been rejected because pictures of iPhones were included. Apple may not accept any picture of an existing device, be it phone, computer, monitor, or bicycle. I'd use a picture that's clearly drawn rather than photographed and doesn't resemble a device of any existing brand. Then again... with Apple you never know. Just try it, the worst case scenario is only an additional week's delay due to a rejection. -- Best regards, Mark Schonewille Economy-x-Talk Consulting and Software Engineering Homepage: http://economy-x-talk.com Twitter: http://twitter.com/xtalkprogrammer KvK: 50277553 What does that error mean? Buy LiveCodeErrors for iPhone now http://qery.us/v4 A must-have for LiveCode programmers. On 29 jul 2011, at 13:57, William de Smet wrote: > Hi there, > > I am thinking of using an iMac (with no Apple logo) vector image in the splash screen of my first iOS app. Is Apple going to allow this? Or is it better to you use a more 'standard' monitor? > > Greetings, > > William From andre at andregarzia.com Fri Jul 29 08:51:40 2011 From: andre at andregarzia.com (Andre Garzia) Date: Fri, 29 Jul 2011 09:51:40 -0300 Subject: [OT] HTML5 Amazingness In-Reply-To: References: <4E3248CE.1030200@pdslabs.net> Message-ID: OK Go has the best music videos... for those not familiar check out http://www.youtube.com/watch?v=qybUFnY7Y8w :-) this google chrome experience with them is incredible.... :-) On Fri, Jul 29, 2011 at 3:41 AM, Scott Rossi wrote: > No doubt. > > Years and years ago when I was using SuperCard, I was developing an app > called CyberRave that was designed to mimic (as best as one could at that > time) the experience of being at club, hearing dance music, and seeing some > simple effects on your screen (background image changes, simulated laser > lights, etc). I was also doing a stint with the HI group in Apple's > eWorld, > and we concocted a way to hook CyberRave up to eWorld's live chat services, > where special chat commands would trigger the effects on connected systems. > We set up our first rave in house, got the music going, and started sending > chat messages back and forth. It lasted maybe a minute before crashing. > But > it was awesome. > > One of the features in CyberRave was a video wall, where a small QuickTime > movie could be made to play cloned across a 16x16 grid, or the single movie > occupying the entire grid. Seeing the OKGO multi-window experience brought > back fond memories of doing almost exactly the same thing around 15 years > ago... > > Regards, > > Scott Rossi > Creative Director > Tactile Media, UX Design > > > > > Recently, Phil Davis wrote: > > > It sort of reaches the "magic" level for me! At some point I quit > thinking, > > "How > > is it doing that?" and just took it in. Grinning and shaking my head. > > > > Phil Davis > > > > > > On 7/28/11 6:18 PM, Scott Rossi wrote: > >> Requires Google Chrome: > >> > >> http://www.allisnotlo.st/ > >> > >> (Note: shut down all other tabs in Chrome for a better experience.) > >> > >> I think I actually said "OMG" out loud when I first saw this. > >> > >> Regards, > >> > >> Scott Rossi > >> Creative Director > >> Tactile Media, UX Design > >> > >> > >> > >> _______________________________________________ > >> use-livecode mailing list > >> use-livecode at lists.runrev.com > >> Please visit this url to subscribe, unsubscribe and manage your > subscription > >> preferences: > >> http://lists.runrev.com/mailman/listinfo/use-livecode > > > > _______________________________________________ > use-livecode mailing list > use-livecode at lists.runrev.com > Please visit this url to subscribe, unsubscribe and manage your > subscription preferences: > http://lists.runrev.com/mailman/listinfo/use-livecode > -- http://www.andregarzia.com All We Do Is Code. From bvg at mac.com Fri Jul 29 08:57:13 2011 From: bvg at mac.com (=?iso-8859-1?Q?Bj=F6rnke_von_Gierke?=) Date: Fri, 29 Jul 2011 14:57:13 +0200 Subject: [ANN] LiveCode.tv event #34 In-Reply-To: References: <6B865405-BBC0-491C-AD25-BFB1C85A2D58@mac.com> <7EC6A85B-DD6B-4138-A597-512A194F4453@mac.com> <89782C22-C6E3-4245-95D1-A48F69423753@mac.com> <061D9182-B34B-4ABB-A7D6-9C2D09DA8755@mac.com> <73F84954-890B-4627-859A-702C8054F13B@mac.com> <093DEF5E-6C0F-4924-A262-F6DBE748ED45@mac.com> <3FB852C9-5675-4A19-9506-543733BB2547@mac.com> <914DF999-596E-4023-A213-9469C6A69FA0@mac.com> <2B105965-FB27-41D0-B95F-1F4C84ADD53B@mac.com> <163225D1-67D1-4CE7-8049-E85A8D94D177@mac.com> <225B0941-5D11-434A-BC0B-CD61B998E9F8@mac.com> <201756EB-9601-417D-856E-5128C5256EF1@mac.com> <081FD717-0A48-447D-90AA-A8F370B14F43@mac.com> <913B7E78-3052-4CF0-9883-C1CDD9F8BDFA@mac.com> <0C1B6D89-1745-46DF-9C49-A5FE2E5B67B9@mac.com> <497CC7C9-74BC-419C-9785-CEA07A8FE181@mac.com> Message-ID: Tomorrows event will be of reduced scope due to a lack of volunteers. All applicants welcome. I will introduce the release of BvG Docu 2. Join chatrev at 20:00 CET. Go to http://bjoernke.com/chatrev/ or enter in the message box: go stack URL ?http://bjoernke.com/chatrev/chatrev1.3b3.rev? Sat. 22:00 Moscow Sat. 14:00 New York Sat. 11:00 Los Angeles Sun. 2:00 Beijing See http://livecode.tv/ for more info. Mail me to do your presentation, it doesn't matter if you do one in 4 months or right now. Bj?rnke From john at splash21.com Fri Jul 29 09:54:38 2011 From: john at splash21.com (John Craig) Date: Fri, 29 Jul 2011 14:54:38 +0100 Subject: [ANN] LiveCode.tv event #34 In-Reply-To: References: <6B865405-BBC0-491C-AD25-BFB1C85A2D58@mac.com> <2B105965-FB27-41D0-B95F-1F4C84ADD53B@mac.com> <163225D1-67D1-4CE7-8049-E85A8D94D177@mac.com> <225B0941-5D11-434A-BC0B-CD61B998E9F8@mac.com> <201756EB-9601-417D-856E-5128C5256EF1@mac.com> <081FD717-0A48-447D-90AA-A8F370B14F43@mac.com> <913B7E78-3052-4CF0-9883-C1CDD9F8BDFA@mac.com> <0C1B6D89-1745-46DF-9C49-A5FE2E5B67B9@mac.com> <497CC7C9-74BC-419C-9785-CEA07A8FE181@mac.com> Message-ID: <4E32BB9E.6020104@splash21.com> Hi, Bj?rnke. I sent a mail from my phone earlier, but it was acting a bit wierd, so I don't know if it really sent. I can present tomorrow if still required. John. On 29/07/2011 13:57, Bj?rnke von Gierke wrote: > Tomorrows event will be of reduced scope due to a lack of volunteers. All applicants welcome. > > I will introduce the release of BvG Docu 2. > > Join chatrev at 20:00 CET. Go to > http://bjoernke.com/chatrev/ > or enter in the message box: > go stack URL ?http://bjoernke.com/chatrev/chatrev1.3b3.rev? > > Sat. 22:00 Moscow > Sat. 14:00 New York > Sat. 11:00 Los Angeles > Sun. 2:00 Beijing > > See http://livecode.tv/ for more info. > > Mail me to do your presentation, it doesn't matter if you do one in 4 months or right now. > Bj?rnke > > > > _______________________________________________ > use-livecode mailing list > use-livecode at lists.runrev.com > Please visit this url to subscribe, unsubscribe and manage your subscription preferences: > http://lists.runrev.com/mailman/listinfo/use-livecode > From bvg at mac.com Fri Jul 29 09:57:41 2011 From: bvg at mac.com (=?iso-8859-1?Q?Bj=F6rnke_von_Gierke?=) Date: Fri, 29 Jul 2011 15:57:41 +0200 Subject: [ANN] LiveCode.tv event #34 In-Reply-To: <4E32BB9E.6020104@splash21.com> References: <6B865405-BBC0-491C-AD25-BFB1C85A2D58@mac.com> <2B105965-FB27-41D0-B95F-1F4C84ADD53B@mac.com> <163225D1-67D1-4CE7-8049-E85A8D94D177@mac.com> <225B0941-5D11-434A-BC0B-CD61B998E9F8@mac.com> <201756EB-9601-417D-856E-5128C5256EF1@mac.com> <081FD717-0A48-447D-90AA-A8F370B14F43@mac.com> <913B7E78-3052-4CF0-9883-C1CDD9F8BDFA@mac.com> <0C1B6D89-1745-46DF-9C49-A5FE2E5B67B9@mac.com> <497CC7C9-74BC-419C-9785-CEA07A8FE181@mac.com> <4E32BB9E.6020104@splash21.com> Message-ID: <1EFA85EA-C8D8-4B91-BF22-B2B7ABE016CE@mac.com> yes, i got it, answered offlist. On 29 Jul 2011, at 15:54, John Craig wrote: > Hi, Bj?rnke. I sent a mail from my phone earlier, but it was acting a bit wierd, so I don't know if it really sent. > I can present tomorrow if still required. > > > John. > > > On 29/07/2011 13:57, Bj?rnke von Gierke wrote: >> Tomorrows event will be of reduced scope due to a lack of volunteers. All applicants welcome. >> >> I will introduce the release of BvG Docu 2. >> >> Join chatrev at 20:00 CET. Go to >> http://bjoernke.com/chatrev/ >> or enter in the message box: >> go stack URL ?http://bjoernke.com/chatrev/chatrev1.3b3.rev? >> >> Sat. 22:00 Moscow >> Sat. 14:00 New York >> Sat. 11:00 Los Angeles >> Sun. 2:00 Beijing >> >> See http://livecode.tv/ for more info. >> >> Mail me to do your presentation, it doesn't matter if you do one in 4 months or right now. >> Bj?rnke >> >> >> >> _______________________________________________ >> use-livecode mailing list >> use-livecode at lists.runrev.com >> Please visit this url to subscribe, unsubscribe and manage your subscription preferences: >> http://lists.runrev.com/mailman/listinfo/use-livecode >> > > _______________________________________________ > use-livecode mailing list > use-livecode at lists.runrev.com > Please visit this url to subscribe, unsubscribe and manage your subscription preferences: > http://lists.runrev.com/mailman/listinfo/use-livecode From ambassador at fourthworld.com Fri Jul 29 10:24:44 2011 From: ambassador at fourthworld.com (Richard Gaskin) Date: Fri, 29 Jul 2011 07:24:44 -0700 Subject: [OT] HTML5 Amazingness In-Reply-To: References: Message-ID: <4E32C2AC.8070608@fourthworld.com> Andre Garzia wrote: > OK Go has the best music videos... for those not familiar check out > http://www.youtube.com/watch?v=qybUFnY7Y8w :-) My friend and neighbor worked on the crew for that video, one of the happy cheering faces in the loft above the band near the end. The pile of smashed TVs in the background barely tells the story of how difficult it was to make; more telling is the exhausted look on the band members' faces. IIRC he told me that the final take we see in that video happened around 3:30AM after many dozens of takes, each one requiring all of the elements to be reset (note the amount of paint on their splash suits at the start of the video). As much as I appreciate the technical flourish of the Chrome app, I'm even more impressed by the low-tech Rube Goldberg machine they set up for that video. Now *that's* ingenuity. -- Richard Gaskin Fourth World LiveCode training and consulting: http://www.fourthworld.com Webzine for LiveCode developers: http://www.LiveCodeJournal.com LiveCode Journal blog: http://LiveCodejournal.com/blog.irv From roger.e.eller at sealedair.com Fri Jul 29 11:09:13 2011 From: roger.e.eller at sealedair.com (Roger Eller) Date: Fri, 29 Jul 2011 11:09:13 -0400 Subject: Installing iOS SDK and Xcode on Windows 7. In-Reply-To: References: Message-ID: On Fri, Jul 29, 2011 at 4:14 AM, Shao Sean wrote: > I may be wrong, but I am pretty sure iOS dev is limited to Mac Only. :\ >> > > They are running the tools in a virtual machine so as far as the software > is aware it is running on a Mac.. > > True, the software would not know the difference, but the user definitely would because it would be prohibitively slow "in a vm". I googled "tonymac", and found that people apparently are running directly on i5 and i7 machines with the OS installed directly from purchased retail install media. Due to Apple licensing restrictions, one can't use such a system in a business, but it is quite interesting what is technically possible. From psahores at free.fr Fri Jul 29 11:30:04 2011 From: psahores at free.fr (Pierre Sahores) Date: Fri, 29 Jul 2011 17:30:04 +0200 Subject: Where does survive the inventive user ? In-Reply-To: References: <8CE1A70D9625EA6-2550-214DE@web-mmc-d04.sysops.aol.com> Message-ID: The way was to handle TCP/IP trough MacOS 8 + WebSTAR 3 <--TCP AppleEvent sockets translator --> HC 2.4.1 hand made standalone server app (via Resedit, if i right remember...). Worked in single thread mode only because the OS and HC architectures ! ... Went to Linux, Apache and Metacard cgi because those limitations and because i did't trust in the WebObjects, ColdFusion and Visual Cafe first public versions i brought... ;-) Le 29 juil. 2011 ? 00:51, stephen barncard a ?crit : > I know, I tried to do it myself in the early 90s. TCP/IP on the mac was > quite unreliable. > > On 28 July 2011 15:00, Chipp Walters wrote: > >> >> >> Let's not forget, HC was a TCP/IP stack away from BEING a first browser ( >> http://www.isegoria.net/2008/05/hypercard-what-could-have-been/), so I'm >> don't think it could happen again-- though of course I would be rooting for >> it! >> > > Stephen Barncard > San Francisco Ca. USA > > more about sqb > _______________________________________________ > use-livecode mailing list > use-livecode at lists.runrev.com > Please visit this url to subscribe, unsubscribe and manage your subscription preferences: > http://lists.runrev.com/mailman/listinfo/use-livecode > -- Pierre Sahores mobile : (33) 6 03 95 77 70 www.woooooooords.com www.sahores-conseil.com From lfredricks at proactive-intl.com Fri Jul 29 11:35:15 2011 From: lfredricks at proactive-intl.com (Lynn Fredricks) Date: Fri, 29 Jul 2011 08:35:15 -0700 Subject: Where does survive the inventive user ? In-Reply-To: References: <8CE1A70D9625EA6-2550-214DE@web-mmc-d04.sysops.aol.com><54729309-D39D-4D70-A239-BC3BA825F5E9@twft.com> Message-ID: <51F779AA08EC41DABF060BF07BBD819A@GATEWAY> > If I recall, HyperCard was called an "erector set" for Mac users, not > necessarily programmers, and indeed used mostly by non-professional > programmers. There was also an Xcmd for Valentina -- yes, I started > using Valentina database with HyperCard -- and it worked really well > but Valentina was not network based then. The depths of time.... :-) Valentina DB started out originally as a C++ library for Mac OS back in 1998, after several years of advanced R&D by Ruslan (ooooo, CodeWarrior!). The XCMD format was so well established then, and several products on the market supported that. There were a lot of really cool externals around back then, some of which were made by folks who are here yet today. Best regards, Lynn Fredricks President Paradigma Software http://www.paradigmasoft.com Valentina SQL Server: The Ultra-fast, Royalty Free Database Server From psahores at free.fr Fri Jul 29 11:37:12 2011 From: psahores at free.fr (Pierre Sahores) Date: Fri, 29 Jul 2011 17:37:12 +0200 Subject: [OT] HTML5 Amazingness In-Reply-To: References: Message-ID: Extra ! Thanks for the link, Scott :-) Le 29 juil. 2011 ? 03:18, Scott Rossi a ?crit : > Requires Google Chrome: > > http://www.allisnotlo.st/ > > (Note: shut down all other tabs in Chrome for a better experience.) > > I think I actually said "OMG" out loud when I first saw this. > > Regards, > > Scott Rossi > Creative Director > Tactile Media, UX Design > > > > _______________________________________________ > use-livecode mailing list > use-livecode at lists.runrev.com > Please visit this url to subscribe, unsubscribe and manage your subscription preferences: > http://lists.runrev.com/mailman/listinfo/use-livecode > -- Pierre Sahores mobile : (33) 6 03 95 77 70 www.woooooooords.com www.sahores-conseil.com From ambassador at fourthworld.com Fri Jul 29 11:51:30 2011 From: ambassador at fourthworld.com (Richard Gaskin) Date: Fri, 29 Jul 2011 08:51:30 -0700 Subject: Where does survive the inventive user? In-Reply-To: <1311919826020-3703159.post@n4.nabble.com> References: <1311919826020-3703159.post@n4.nabble.com> Message-ID: <4E32D702.5000003@fourthworld.com> Alejandro Tejada wrote: > Thinking about inventive users and open source software... > > When I first read about the Open Source movement, > I though that it was a group of developers that > wanted to create software in the same way that an > artist creates his work: > > A lasting work of art that trascend time > because of his many outstanding and > unique qualities... > > Hmmm, looks like I was wrong in my first impression > about Open source, but just in case: > > Could anyone show me these outstanding artistic > qualities in Open Source software? That's a very thoughtful question, Alejandro. I'm certainly no FOSS expert, and perhaps Mark Weider or David Bovill can contribute more concretely here, but that didn't stop me from writing my own offhand thoughts on this. ;) As sometimes happens during my morning coffee, that got a bit lengthy for this list, so out of respect for the readers' bandwidth here in this OT thread I posted it to the LiveCode Journal blog: -- Richard Gaskin Fourth World LiveCode training and consulting: http://www.fourthworld.com Webzine for LiveCode developers: http://www.LiveCodeJournal.com LiveCode Journal blog: http://LiveCodejournal.com/blog.irv From psahores at free.fr Fri Jul 29 12:03:06 2011 From: psahores at free.fr (Pierre Sahores) Date: Fri, 29 Jul 2011 18:03:06 +0200 Subject: Where does survive the inventive user? In-Reply-To: <1311919826020-3703159.post@n4.nabble.com> References: <1311706987455-3696711.post@n4.nabble.com> <08F3632C-76A9-402E-97B4-C82090DC8D05@free.fr> <465EFD52-8532-4945-A2F0-C2CE01371329@twft.com> <1311919826020-3703159.post@n4.nabble.com> Message-ID: If we believe that technical design can be arts relevant (my case,...), sailboats, cars or information's systems can be arts relevant ;-) it's probably why the Alan's Turing works gave the binary coding paradigm to computers, because why the Linus Torwalds initiative gave us Linux, because why John Mc Carty LISP, because the elegance of PostgresQL where Oracle is just a big sad truck..., because technical skills mainly serves visions and not the inverse... Even if xtalk is not open-source, the non technical guys whose invented xtalk, Metacard and LiveCode and we, the xtalk dev community are dependent from the open-source tools we are binding to our LC solutions. It's at least my case and i'm every day graceful about this. Le 29 juil. 2011 ? 08:10, Alejandro Tejada a ?crit : > Could anyone show me these outstanding artistic > qualities in Open Source software? -- Pierre Sahores mobile : (33) 6 03 95 77 70 www.woooooooords.com www.sahores-conseil.com From warren at warrensweb.us Fri Jul 29 12:12:59 2011 From: warren at warrensweb.us (Warren Samples) Date: Fri, 29 Jul 2011 11:12:59 -0500 Subject: Installing iOS SDK and Xcode on Windows 7. In-Reply-To: References: Message-ID: <201107291112.59468.warren@warrensweb.us> On Friday, July 29, 2011 10:09:13 AM Roger Eller wrote: > it would be prohibitively slow "in a vm" Roger, Have you some disappointing experience running vms? My own experience running VirtualBox is that the guest OSs run quite snappily. I cannot recommend that one rely on a virtualized OS for development purposes because certain OS features are bound to be tied to hardware features that the vm won't provide. That said, for many purposes, including some kinds of testing during development, they are very usable and useful indeed. Best, Warren From capellan2000 at gmail.com Fri Jul 29 12:19:50 2011 From: capellan2000 at gmail.com (Alejandro Tejada) Date: Fri, 29 Jul 2011 09:19:50 -0700 (PDT) Subject: Where does survive the inventive user ? In-Reply-To: References: <8CE1A70D9625EA6-2550-214DE@web-mmc-d04.sysops.aol.com> Message-ID: <1311956390066-3704546.post@n4.nabble.com> >From the archives: http://www.thefreelibrary.com/Human+Code+Unveils+XtraNet+Technology+Enabling+Shockwave+Applications...-a018567832 Chipp Walters wrote: > > Yep. We wrote the first XCMD/Xtra for TCP/IP for Director. It was called > XtraNet. Macromedia tried to buy it from us. > -- View this message in context: http://runtime-revolution.278305.n4.nabble.com/Where-does-survive-the-inventive-user-tp3698117p3704546.html Sent from the Revolution - User mailing list archive at Nabble.com. From bobs at twft.com Fri Jul 29 12:25:57 2011 From: bobs at twft.com (Bob Sneidar) Date: Fri, 29 Jul 2011 09:25:57 -0700 Subject: A way to get LC out of the way In-Reply-To: References: Message-ID: I do not understand exactly what you mean, but I will give it a go. It sounds like you work with a copy of a stack to update it, then replace the older copy with the newer one, but LC is remembering the older one. There is a property of a stack called destroyStack, an unfortunate name because it only destroys the copy in memory. I believe the corresponding checkbox is Purge Stack On Close. If this is unchecked, closing the stack will still leave it in memory, so that any future reference to it will reopen it. I am unsure if my understanding of what you are doing is correct, but I would reconsider how you are going about things if you are replacing one stack with another just like it. In any case, check that checkbox and see if that doesn't help. Bob On Jul 28, 2011, at 10:05 PM, Joe Lewis Wilkins wrote: > Hi everyone, > > I'm struggling; maybe I should say "fighting" with LC. I have an app in which I do some things that create new cards in one of the sub-stacks. In the process of refining things I want to swap the original stack with the one that has the new cards, but LC always remembers the larger stack. After a while, I'm usually able to have the smaller one used, but it's a real effort. Have I missed some simple way of doing this. In most other programs, I would be able to just replace the large one with the small one. I can do it in this case too, but the small one is never seen. > > TIA, > > Joe Wilkins > > > _______________________________________________ > use-livecode mailing list > use-livecode at lists.runrev.com > Please visit this url to subscribe, unsubscribe and manage your subscription preferences: > http://lists.runrev.com/mailman/listinfo/use-livecode From bobs at twft.com Fri Jul 29 12:28:19 2011 From: bobs at twft.com (Bob Sneidar) Date: Fri, 29 Jul 2011 09:28:19 -0700 Subject: Behavior not working?? In-Reply-To: <6757E17A-6010-4F9B-8FF5-F4DD3BA8CEF5@wanadoo.fr> References: <6757E17A-6010-4F9B-8FF5-F4DD3BA8CEF5@wanadoo.fr> Message-ID: unless the plugin itself uses behaviors, which since I don't know, replacing is probably the best strategy. This is why I always copy downloads to their appropriate location instead of moving them. Bob On Jul 29, 2011, at 12:37 AM, Andr? Bisseret wrote: > or set the behavior of cd n of the plugin to empty From psahores at free.fr Fri Jul 29 12:37:27 2011 From: psahores at free.fr (Pierre Sahores) Date: Fri, 29 Jul 2011 18:37:27 +0200 Subject: [OT] HTML5 Amazingness In-Reply-To: <4E32C2AC.8070608@fourthworld.com> References: <4E32C2AC.8070608@fourthworld.com> Message-ID: <685C1EDD-AD84-4AD2-BE79-5213A1971EB8@free.fr> Le 29 juil. 2011 ? 16:24, Richard Gaskin a ?crit : > Andre Garzia wrote: > >> OK Go has the best music videos... for those not familiar check out >> http://www.youtube.com/watch?v=qybUFnY7Y8w :-) > > My friend and neighbor worked on the crew for that video, one of the happy cheering faces in the loft above the band near the end. The pile of smashed TVs in the background barely tells the story of how difficult it was to make; more telling is the exhausted look on the band members' faces. IIRC he told me that the final take we see in that video happened around 3:30AM after many dozens of takes, each one requiring all of the elements to be reset (note the amount of paint on their splash suits at the start of the video). > > As much as I appreciate the technical flourish of the Chrome app, I'm even more impressed by the low-tech Rube Goldberg machine they set up for that video. Now *that's* ingenuity. and still so refreshing ;-) > > -- > Richard Gaskin > Fourth World > LiveCode training and consulting: http://www.fourthworld.com > Webzine for LiveCode developers: http://www.LiveCodeJournal.com > LiveCode Journal blog: http://LiveCodejournal.com/blog.irv > > _______________________________________________ > use-livecode mailing list > use-livecode at lists.runrev.com > Please visit this url to subscribe, unsubscribe and manage your subscription preferences: > http://lists.runrev.com/mailman/listinfo/use-livecode > -- Pierre Sahores mobile : (33) 6 03 95 77 70 www.woooooooords.com www.sahores-conseil.com From bobs at twft.com Fri Jul 29 12:38:02 2011 From: bobs at twft.com (Bob Sneidar) Date: Fri, 29 Jul 2011 09:38:02 -0700 Subject: [OT] HTML5 Amazingness In-Reply-To: References: Message-ID: <36D2F845-2EDA-4BD0-9040-B43A156E1BD2@twft.com> Mine is stuck at 97%. PAR! Bob On Jul 28, 2011, at 6:18 PM, Scott Rossi wrote: > Requires Google Chrome: > > http://www.allisnotlo.st/ > > (Note: shut down all other tabs in Chrome for a better experience.) > > I think I actually said "OMG" out loud when I first saw this. > > Regards, > > Scott Rossi > Creative Director > Tactile Media, UX Design > > > > _______________________________________________ > use-livecode mailing list > use-livecode at lists.runrev.com > Please visit this url to subscribe, unsubscribe and manage your subscription preferences: > http://lists.runrev.com/mailman/listinfo/use-livecode From bobs at twft.com Fri Jul 29 12:42:12 2011 From: bobs at twft.com (Bob Sneidar) Date: Fri, 29 Jul 2011 09:42:12 -0700 Subject: [OT] HTML5 Amazingness In-Reply-To: References: Message-ID: Finally worked! Cool. On Jul 28, 2011, at 6:18 PM, Scott Rossi wrote: > Requires Google Chrome: > > http://www.allisnotlo.st/ > > (Note: shut down all other tabs in Chrome for a better experience.) > > I think I actually said "OMG" out loud when I first saw this. > > Regards, > > Scott Rossi > Creative Director > Tactile Media, UX Design > > > > _______________________________________________ > use-livecode mailing list > use-livecode at lists.runrev.com > Please visit this url to subscribe, unsubscribe and manage your subscription preferences: > http://lists.runrev.com/mailman/listinfo/use-livecode From pepetoo at cox.net Fri Jul 29 12:42:42 2011 From: pepetoo at cox.net (Joe Lewis Wilkins) Date: Fri, 29 Jul 2011 09:42:42 -0700 Subject: A way to get LC out of the way In-Reply-To: References: Message-ID: <939643FA-9BBF-4433-B736-C2D510D8BFEF@cox.net> Thanks Bob. I had been diddling around with exactly those features when I have managed to make the substitution, but felt a little uneasy that the Destroy Stack might be messing me up in ways that I didn't realize. The name IS unfortunate as you mentioned. Actually, I have an even more exasperating "trial" to deal with. I want to reassemble my program using the existing stacks, but there appears to be no way to severe LC's strangle-hold on things. I'd love to be able to create a new program by dragging copies of the existing stacks over to a new folder and establishing "new" relationships amongst them, but LC is insistent on maintaining the "old" relationships. I'd like to change the standalone application settings of this "new" relationship. So far I have been unsuccessful; just spinning my wheels doing things over and over again. Regardless, Thanks. If anyone has a clue as to how this last approach may be accomplished I would love to hear about it. Joe Wilkins On Jul 29, 2011, at 9:25 AM, Bob Sneidar wrote: > I do not understand exactly what you mean, but I will give it a go. It sounds like you work with a copy of a stack to update it, then replace the older copy with the newer one, but LC is remembering the older one. > > There is a property of a stack called destroyStack, an unfortunate name because it only destroys the copy in memory. I believe the corresponding checkbox is Purge Stack On Close. If this is unchecked, closing the stack will still leave it in memory, so that any future reference to it will reopen it. > > I am unsure if my understanding of what you are doing is correct, but I would reconsider how you are going about things if you are replacing one stack with another just like it. In any case, check that checkbox and see if that doesn't help. > > Bob > > > On Jul 28, 2011, at 10:05 PM, Joe Lewis Wilkins wrote: > >> Hi everyone, >> >> I'm struggling; maybe I should say "fighting" with LC. I have an app in which I do some things that create new cards in one of the sub-stacks. In the process of refining things I want to swap the original stack with the one that has the new cards, but LC always remembers the larger stack. After a while, I'm usually able to have the smaller one used, but it's a real effort. Have I missed some simple way of doing this. In most other programs, I would be able to just replace the large one with the small one. I can do it in this case too, but the small one is never seen. >> >> TIA, >> >> Joe Wilkins From bobs at twft.com Fri Jul 29 12:43:51 2011 From: bobs at twft.com (Bob Sneidar) Date: Fri, 29 Jul 2011 09:43:51 -0700 Subject: Where does survive the inventive user? In-Reply-To: References: <1311706987455-3696711.post@n4.nabble.com> <08F3632C-76A9-402E-97B4-C82090DC8D05@free.fr> <465EFD52-8532-4945-A2F0-C2CE01371329@twft.com> <1311919826020-3703159.post@n4.nabble.com> Message-ID: <9689CBE7-9912-456B-9AF6-1141749A9E47@twft.com> I have a saying: If everyone can do it, it's not art. And even if only a few can do it, it's still not art! Bob On Jul 29, 2011, at 9:03 AM, Pierre Sahores wrote: > If we believe that technical design can be arts relevant (my case,...), sailboats, cars or information's systems can be arts relevant ;-) it's probably why the Alan's Turing works gave the binary coding paradigm to computers, because why the Linus Torwalds initiative gave us Linux, because why John Mc Carty LISP, because the elegance of PostgresQL where Oracle is just a big sad truck..., because technical skills mainly serves visions and not the inverse... > > Even if xtalk is not open-source, the non technical guys whose invented xtalk, Metacard and LiveCode and we, the xtalk dev community are dependent from the open-source tools we are binding to our LC solutions. It's at least my case and i'm every day graceful about this. > > Le 29 juil. 2011 ? 08:10, Alejandro Tejada a ?crit : > >> Could anyone show me these outstanding artistic >> qualities in Open Source software? > > -- > Pierre Sahores > mobile : (33) 6 03 95 77 70 > > www.woooooooords.com > www.sahores-conseil.com > > > > > > _______________________________________________ > use-livecode mailing list > use-livecode at lists.runrev.com > Please visit this url to subscribe, unsubscribe and manage your subscription preferences: > http://lists.runrev.com/mailman/listinfo/use-livecode From davidocoker at gmail.com Fri Jul 29 12:44:11 2011 From: davidocoker at gmail.com (David C.) Date: Fri, 29 Jul 2011 11:44:11 -0500 Subject: [OT] More Apple Foolishness In-Reply-To: References: <6F918963-BD07-4351-83D1-41E1A78D06FD@altuit.com> <4E30C8A7.40405@hyperactivesw.com> Message-ID: I've been trying to refrain from some of these posts with the thought that maybe I've been to negative sounding recently... However, in light of this: ...from iTunes (the single most frustrating and annoying app interface-wise on the whole planet on any OS) Absolutely on the mark! To be tied at the hip with that one piece of software is absolutely awful and I'm doing just about all that I can to remedy that. I am so glad to know that I'm not the only one that feels that way. Best regards, David C. From ambassador at fourthworld.com Fri Jul 29 12:55:53 2011 From: ambassador at fourthworld.com (Richard Gaskin) Date: Fri, 29 Jul 2011 09:55:53 -0700 Subject: [OT] More Apple Foolishness In-Reply-To: References: Message-ID: <4E32E619.5010908@fourthworld.com> David C. wrote: > ...from iTunes (the single most frustrating and annoying app > interface-wise on the whole planet on any OS) > > Absolutely on the mark! To be tied at the hip with that one piece of > software is absolutely awful and I'm doing just about all that I can > to remedy that. > > I am so glad to know that I'm not the only one that feels that way. The irony here is that Apple themselves feels the same way: while iTunes may be where Apple requires you to get your apps, the iTunes app itself doesn't conform to Apple's own HIG substantially enough to allow it to be submitted to the App Store. ;) -- Richard Gaskin Fourth World LiveCode training and consulting: http://www.fourthworld.com Webzine for LiveCode developers: http://www.LiveCodeJournal.com LiveCode Journal blog: http://LiveCodejournal.com/blog.irv From psahores at free.fr Fri Jul 29 13:00:26 2011 From: psahores at free.fr (Pierre Sahores) Date: Fri, 29 Jul 2011 19:00:26 +0200 Subject: Where does survive the inventive user? In-Reply-To: <4E32D702.5000003@fourthworld.com> References: <1311919826020-3703159.post@n4.nabble.com> <4E32D702.5000003@fourthworld.com> Message-ID: <9BB81CB5-0E57-4802-92E3-448005FFC1B6@free.fr> "The artistry of open source may be subtle, but it's pervasive." - Sir Richard Gaskin - ;-) Le 29 juil. 2011 ? 17:51, Richard Gaskin a ?crit : > Alejandro Tejada wrote: > >> Thinking about inventive users and open source software... >> >> When I first read about the Open Source movement, >> I though that it was a group of developers that >> wanted to create software in the same way that an >> artist creates his work: >> >> A lasting work of art that trascend time >> because of his many outstanding and >> unique qualities... >> >> Hmmm, looks like I was wrong in my first impression >> about Open source, but just in case: >> >> Could anyone show me these outstanding artistic >> qualities in Open Source software? > > That's a very thoughtful question, Alejandro. I'm certainly no FOSS expert, and perhaps Mark Weider or David Bovill can contribute more concretely here, but that didn't stop me from writing my own offhand thoughts on this. ;) > > As sometimes happens during my morning coffee, that got a bit lengthy for this list, so out of respect for the readers' bandwidth here in this OT thread I posted it to the LiveCode Journal blog: > > > > -- > Richard Gaskin > Fourth World > LiveCode training and consulting: http://www.fourthworld.com > Webzine for LiveCode developers: http://www.LiveCodeJournal.com > LiveCode Journal blog: http://LiveCodejournal.com/blog.irv > > _______________________________________________ > use-livecode mailing list > use-livecode at lists.runrev.com > Please visit this url to subscribe, unsubscribe and manage your subscription preferences: > http://lists.runrev.com/mailman/listinfo/use-livecode > -- Pierre Sahores mobile : (33) 6 03 95 77 70 www.woooooooords.com www.sahores-conseil.com From roger.e.eller at sealedair.com Fri Jul 29 13:04:49 2011 From: roger.e.eller at sealedair.com (Roger Eller) Date: Fri, 29 Jul 2011 13:04:49 -0400 Subject: Installing iOS SDK and Xcode on Windows 7. In-Reply-To: <201107291112.59468.warren@warrensweb.us> References: <201107291112.59468.warren@warrensweb.us> Message-ID: On Fri, Jul 29, 2011 at 12:12 PM, Warren Samples wrote: > On Friday, July 29, 2011 10:09:13 AM Roger Eller wrote: > > it would be prohibitively slow "in a vm" > > Roger, > > Have you some disappointing experience running vms? My own experience > running VirtualBox is that the guest OSs > run quite snappily. I cannot recommend that one rely on a virtualized OS > for development purposes because > certain OS features are bound to be tied to hardware features that the vm > won't provide. That said, for many > purposes, including some kinds of testing during development, they are very > usable and useful indeed. > > Best, > > Warren > > A few years ago, I remember seeing a video of Leopard running in a vm, maybe even Snow Leopard. It could not utilise quartz/core graphics, and the hard-drive was not a real partition, there was no 3D acceleration or access to the video card, so it made the overall system appear slow. But according to what the tonymac people are saying, it doesn't have to be this way. They seem to now have it running natively on brand new i7 machines with high-end video cards. From warren at warrensweb.us Fri Jul 29 13:21:26 2011 From: warren at warrensweb.us (Warren Samples) Date: Fri, 29 Jul 2011 12:21:26 -0500 Subject: [OT] More Apple Foolishness In-Reply-To: References: <6F918963-BD07-4351-83D1-41E1A78D06FD@altuit.com> Message-ID: <201107291221.26414.warren@warrensweb.us> On Friday, July 29, 2011 11:44:11 AM David C. wrote: > ...from iTunes (the single most frustrating and annoying app > interface-wise on the whole planet on any OS) > > Absolutely on the mark! To be tied at the hip with that one piece of > software is absolutely awful and I'm doing just about all that I can > to remedy that. > > I am so glad to know that I'm not the only one that feels that way. > > Best regards, > David C. I almost never used iTunes when I ran OS X. It wasn't my preferred application for anything and I was never interested in the store. Since the vast majority of the requent updates to iTunes focused on the store, I eventually tried to get software update to ignore those updates. To no avail. Software update refused to honor that request. It was the only update that refused to be ignored. Best, Warren From warren at warrensweb.us Fri Jul 29 13:53:10 2011 From: warren at warrensweb.us (Warren Samples) Date: Fri, 29 Jul 2011 12:53:10 -0500 Subject: Installing iOS SDK and Xcode on Windows 7. In-Reply-To: References: <201107291112.59468.warren@warrensweb.us> Message-ID: <201107291253.11006.warren@warrensweb.us> On Friday, July 29, 2011 12:04:49 PM Roger Eller wrote: > running natively on brand new i7 machines with high-end > video cards. The Hackintosh is not a great secret, and indeed there is no reason for OS X not to run beautifully on non- Apple hardware once the issues of booting and certain drivers are overcome, but the question was regarding performance within a virtual machine. I do not suffer any serious degree of slowness running any OS inside a virtual machine here. I don't particularly encourage the venture proposed by the OP, and am very disturbed by the preoccupation of the author of the article he references with avoided paying what are legitimate fees, but my experience does not support your prediction. I think the sort of statement you make presents unwarranted discouragement to those who may otherwise find virtualization a very useful and convenient tool for certain purposes. What I find most curious about the original post, is how does the OP propose to implement this if he cannot also test it? It makes me think of a post I saw the other day in a forum, "Here fishy fishy..." Best, Warren From hyberson at gmail.com Fri Jul 29 14:47:17 2011 From: hyberson at gmail.com (Hyberson Pereira) Date: Fri, 29 Jul 2011 15:47:17 -0300 Subject: Installing iOS SDK and Xcode on Windows 7. In-Reply-To: <201107291253.11006.warren@warrensweb.us> References: <201107291112.59468.warren@warrensweb.us> <201107291253.11006.warren@warrensweb.us> Message-ID: Warren, *"What I find most curious about the original post, is how does the OP propose to implement this if he cannot also test it?"* Unfortunately at the moment I cannot meet the requirementsto test the setup, for I don't have the hardware or the software to do that. That's exactly why I posted at this forum. If I come to the conclusion that the setup works, I will invest some money in hardware and software. Please notice that I did not propose to implement anything. Best regards, Hyberson On Fri, Jul 29, 2011 at 14:53, Warren Samples wrote: > On Friday, July 29, 2011 12:04:49 PM Roger Eller wrote: > > running natively on brand new i7 machines with high-end > > video cards. > > > The Hackintosh is not a great secret, and indeed there is no reason for OS > X not to run beautifully on non- > Apple hardware once the issues of booting and certain drivers are overcome, > but the question was regarding > performance within a virtual machine. I do not suffer any serious degree of > slowness running any OS inside a > virtual machine here. I don't particularly encourage the venture proposed > by the OP, and am very disturbed by > the preoccupation of the author of the article he references with avoided > paying what are legitimate fees, but > my experience does not support your prediction. I think the sort of > statement you make presents unwarranted > discouragement to those who may otherwise find virtualization a very useful > and convenient tool for certain > purposes. > > What I find most curious about the original post, is how does the OP > propose to implement this if he cannot > also test it? It makes me think of a post I saw the other day in a forum, > "Here fishy fishy..." > > > Best, > > Warren > > _______________________________________________ > use-livecode mailing list > use-livecode at lists.runrev.com > Please visit this url to subscribe, unsubscribe and manage your > subscription preferences: > http://lists.runrev.com/mailman/listinfo/use-livecode > From andre.bisseret at wanadoo.fr Fri Jul 29 14:52:19 2011 From: andre.bisseret at wanadoo.fr (=?iso-8859-1?Q?Andr=E9_Bisseret?=) Date: Fri, 29 Jul 2011 20:52:19 +0200 Subject: Behavior not working?? In-Reply-To: References: <6757E17A-6010-4F9B-8FF5-F4DD3BA8CEF5@wanadoo.fr> Message-ID: <4D317F7E-BE8E-437F-AC84-6CDB5AE771DE@wanadoo.fr> Le 29 juil. 2011 ? 18:28, Bob Sneidar a ?crit : > unless the plugin itself uses behaviors, OOuuups ! yes! your are right indeed!I did not think to this possibilitty Andr? > which since I don't know, replacing is probably the best strategy. This is why I always copy downloads to their appropriate location instead of moving them. > > Bob > > > On Jul 29, 2011, at 12:37 AM, Andr? Bisseret wrote: > >> or set the behavior of cd n of the plugin to empty > > > _______________________________________________ > use-livecode mailing list > use-livecode at lists.runrev.com > Please visit this url to subscribe, unsubscribe and manage your subscription preferences: > http://lists.runrev.com/mailman/listinfo/use-livecode From davidocoker at gmail.com Fri Jul 29 14:58:40 2011 From: davidocoker at gmail.com (David C.) Date: Fri, 29 Jul 2011 13:58:40 -0500 Subject: [OT] More Apple Foolishness In-Reply-To: <201107291221.26414.warren@warrensweb.us> References: <6F918963-BD07-4351-83D1-41E1A78D06FD@altuit.com> <201107291221.26414.warren@warrensweb.us> Message-ID: > I almost never used iTunes when I ran OS X. It wasn't my preferred application for anything and I was never > interested in the store. Since the vast majority of the requent updates to iTunes focused on the store, I > eventually tried to get software update to ignore those updates. To no avail. Software update refused to honor > that request. It was the only update that refused to be ignored. > > Best, > > Warren After trying iTunes early on, I hardly ever used it at all.. and in fact avoided it up until I purchased the original iPhone, which I still use. Now I have a ton of songs and media that I'm stuck with for the most part. Been buying nothing but MP3's from Amazon for the last year or so, but still don't want to give up all that I've invested in from Apple. Best regards, David C. From roger.e.eller at sealedair.com Fri Jul 29 14:59:53 2011 From: roger.e.eller at sealedair.com (Roger Eller) Date: Fri, 29 Jul 2011 14:59:53 -0400 Subject: Installing iOS SDK and Xcode on Windows 7. In-Reply-To: <201107291253.11006.warren@warrensweb.us> References: <201107291112.59468.warren@warrensweb.us> <201107291253.11006.warren@warrensweb.us> Message-ID: On Fri, Jul 29, 2011 at 1:53 PM, Warren Samples wrote: > On Friday, July 29, 2011 12:04:49 PM Roger Eller wrote: > > running natively on brand new i7 machines with high-end > > video cards. > > > The Hackintosh is not a great secret, and indeed there is no reason for OS > X not to run beautifully on non- > Apple hardware once the issues of booting and certain drivers are > overcome... Agreed. > I think the sort of statement you make presents unwarranted > discouragement to those who may otherwise find virtualization a very useful > and convenient tool for certain > purposes. > As I stated, the only vm of OS X that I have seen was 'years' ago. If it has been improved, and you find it useful, that is great news! I sincerely hope Apple someday will encourage everyone to buy a copy. I think eventually, they will. I only discourage using without paying. > > Best, > > Warren > > ~Roger From stephenREVOLUTION2 at barncard.com Fri Jul 29 15:08:54 2011 From: stephenREVOLUTION2 at barncard.com (stephen barncard) Date: Fri, 29 Jul 2011 12:08:54 -0700 Subject: [OT] BBC: Apple holding more cash than USA Message-ID: http://www.bbc.co.uk/news/technology-14340470 In light of this, imagine Steve Jobs as your benevolent dictator! Maybe he could buy the USA and save us all. yeah, right. George Orwell (Eric Arthur Blair) would have loved this. -- Stephen Barncard San Francisco Ca. USA more about sqb From stephenREVOLUTION2 at barncard.com Fri Jul 29 16:02:35 2011 From: stephenREVOLUTION2 at barncard.com (stephen barncard) Date: Fri, 29 Jul 2011 13:02:35 -0700 Subject: A way to get LC out of the way In-Reply-To: <939643FA-9BBF-4433-B736-C2D510D8BFEF@cox.net> References: <939643FA-9BBF-4433-B736-C2D510D8BFEF@cox.net> Message-ID: Joe, you have run into the namespace issue. One way around it is to run *multiple instances* of Livecode. If you have ever seen any of Trevor's presentations, you will see three or four Livecode app icons at the bottom of his screens. And he has recommended this method. It was mentioned on this forum that it is possible to cut and paste between instances. The hardest thing to do would be to keep the different instances straight. You can rename the app package of some of the secondary instances but not sure if that shows up in the dock icons. On 29 July 2011 09:42, Joe Lewis Wilkins wrote: > Thanks Bob. I had been diddling around with exactly those features when I > have managed to make the substitution, but felt a little uneasy that the > Destroy Stack might be messing me up in ways that I didn't realize. The name > IS unfortunate as you mentioned. Actually, I have an even more exasperating > "trial" to deal with. I want to reassemble my program using the existing > stacks, but there appears to be no way to severe LC's strangle-hold on > things. I'd love to be able to create a new program by dragging copies of > the existing stacks over to a new folder and establishing "new" > relationships amongst them, but LC is insistent on maintaining the "old" > relationships. I'd like to change the standalone application settings of > this "new" relationship. So far I have been unsuccessful; just spinning my > wheels doing things over and over again. > > Regardless, Thanks. If anyone has a clue as to how this last approach may > be accomplished I would love to hear about it. > > Joe Wilkins > > On Jul 29, 2011, at 9:25 AM, Bob Sneidar wrote: > > > I do not understand exactly what you mean, but I will give it a go. It > sounds like you work with a copy of a stack to update it, then replace the > older copy with the newer one, but LC is remembering the older one. > > > > There is a property of a stack called destroyStack, an unfortunate name > because it only destroys the copy in memory. I believe the corresponding > checkbox is Purge Stack On Close. If this is unchecked, closing the stack > will still leave it in memory, so that any future reference to it will > reopen it. > > > > I am unsure if my understanding of what you are doing is correct, but I > would reconsider how you are going about things if you are replacing one > stack with another just like it. In any case, check that checkbox and see if > that doesn't help. > > > > Bob > > > > > > On Jul 28, 2011, at 10:05 PM, Joe Lewis Wilkins wrote: > > > >> Hi everyone, > >> > >> I'm struggling; maybe I should say "fighting" with LC. I have an app in > which I do some things that create new cards in one of the sub-stacks. In > the process of refining things I want to swap the original stack with the > one that has the new cards, but LC always remembers the larger stack. After > a while, I'm usually able to have the smaller one used, but it's a real > effort. Have I missed some simple way of doing this. In most other programs, > I would be able to just replace the large one with the small one. I can do > it in this case too, but the small one is never seen. > >> > >> TIA, > >> > >> Joe Wilkins > > > _______________________________________________ > use-livecode mailing list > use-livecode at lists.runrev.com > Please visit this url to subscribe, unsubscribe and manage your > subscription preferences: > http://lists.runrev.com/mailman/listinfo/use-livecode > -- Stephen Barncard San Francisco Ca. USA more about sqb From francois.chaplais at mines-paristech.fr Fri Jul 29 16:24:37 2011 From: francois.chaplais at mines-paristech.fr (=?iso-8859-1?Q?Fran=E7ois_Chaplais?=) Date: Fri, 29 Jul 2011 22:24:37 +0200 Subject: [OT] More Apple Foolishness In-Reply-To: <4E318CA3.2030106@hyperactivesw.com> References: <6F918963-BD07-4351-83D1-41E1A78D06FD@altuit.com> <00af01cc4cc3$83a64f30$8af2ed90$@com> <80001DA7-522D-4AB6-A812-501492B660BF@mac.com> <76870884-CAEE-4396-B191-B2C540ACA93B@verizon.net> <4E318CA3.2030106@hyperactivesw.com> Message-ID: <18E8E444-EE80-4D1D-8D1C-611515F969B7@mines-paristech.fr> If I understand well, you will not need iTunes (or even a computer) on your iOS devices when IOS 5 will be (soon) released. After that, your mac (I am not sure about other platforms) will be at the same level as the IOS devices as far as data storage, synchronization is concerned. It was announced in the last Apple keynote, I think. Best regards, Fran?ois Le 28 juil. 2011 ? 18:21, J. Landman Gay a ?crit : > On 7/28/11 4:01 AM, Colin Holgate wrote: >> I hardly ever use iTunes for getting things onto my devices, > > What do you use? I'm tired of hooking up to iTunes for everything. In particular, I want to move test standalones to the device. > > -- > Jacqueline Landman Gay | jacque at hyperactivesw.com > HyperActive Software | http://www.hyperactivesw.com > > _______________________________________________ > use-livecode mailing list > use-livecode at lists.runrev.com > Please visit this url to subscribe, unsubscribe and manage your subscription preferences: > http://lists.runrev.com/mailman/listinfo/use-livecode From francois.chaplais at mines-paristech.fr Fri Jul 29 16:32:34 2011 From: francois.chaplais at mines-paristech.fr (=?iso-8859-1?Q?Fran=E7ois_Chaplais?=) Date: Fri, 29 Jul 2011 22:32:34 +0200 Subject: [OT] BBC: Apple holding more cash than USA In-Reply-To: References: Message-ID: wrong metrics. You'd better look at China's reserves in dollars. Le 29 juil. 2011 ? 21:08, stephen barncard a ?crit : > http://www.bbc.co.uk/news/technology-14340470 > > In light of this, imagine Steve Jobs as your benevolent dictator! > Maybe he could buy the USA and save us all. yeah, right. > > George Orwell (Eric Arthur Blair) would have loved this. > > -- > > > > Stephen Barncard > San Francisco Ca. USA > > more about sqb > _______________________________________________ > use-livecode mailing list > use-livecode at lists.runrev.com > Please visit this url to subscribe, unsubscribe and manage your subscription preferences: > http://lists.runrev.com/mailman/listinfo/use-livecode From bobs at twft.com Fri Jul 29 16:49:56 2011 From: bobs at twft.com (Bob Sneidar) Date: Fri, 29 Jul 2011 13:49:56 -0700 Subject: [OT] BBC: Apple holding more cash than USA In-Reply-To: References: Message-ID: <369A31CC-6B29-4966-917C-9AD447E73317@twft.com> Who's throwing the sledge hammer now Steve?? LOL ;-) As all benevolent dictators learn, they have to hurt some people to get everyone to see how great their way really is. You have to silence the detractors, especially the more successful they are. Bob On Jul 29, 2011, at 12:08 PM, stephen barncard wrote: > http://www.bbc.co.uk/news/technology-14340470 > > In light of this, imagine Steve Jobs as your benevolent dictator! > Maybe he could buy the USA and save us all. yeah, right. > > George Orwell (Eric Arthur Blair) would have loved this. > > -- > > > > Stephen Barncard > San Francisco Ca. USA > > more about sqb > _______________________________________________ > use-livecode mailing list > use-livecode at lists.runrev.com > Please visit this url to subscribe, unsubscribe and manage your subscription preferences: > http://lists.runrev.com/mailman/listinfo/use-livecode From jacque at hyperactivesw.com Fri Jul 29 16:58:48 2011 From: jacque at hyperactivesw.com (J. Landman Gay) Date: Fri, 29 Jul 2011 15:58:48 -0500 Subject: [OT] More Apple Foolishness In-Reply-To: <18E8E444-EE80-4D1D-8D1C-611515F969B7@mines-paristech.fr> References: <6F918963-BD07-4351-83D1-41E1A78D06FD@altuit.com> <00af01cc4cc3$83a64f30$8af2ed90$@com> <80001DA7-522D-4AB6-A812-501492B660BF@mac.com> <76870884-CAEE-4396-B191-B2C540ACA93B@verizon.net> <4E318CA3.2030106@hyperactivesw.com> <18E8E444-EE80-4D1D-8D1C-611515F969B7@mines-paristech.fr> Message-ID: <4E331F08.40408@hyperactivesw.com> On 7/29/11 3:24 PM, Fran?ois Chaplais wrote: > If I understand well, you will not need iTunes (or even a computer) > on your iOS devices when IOS 5 will be (soon) released. After that, > your mac (I am not sure about other platforms) will be at the same > level as the IOS devices as far as data storage, synchronization is > concerned. I have mixed feelings about putting my data in the cloud. But it doesn't matter, I've already solved it. It turns out to be pretty easy to set up your own server/wi-fi solution and I've automated it for use as a LiveCode plugin. When it is tested and prettied up a little bit I'll release it to the community. It works with any server and with Dropbox, and you don't even need to leave LiveCode to use it. Now that I've done it, I'm not surprised that there are so many similar solutions in the App Store. It took me an hour to write, anyone could do it. -- Jacqueline Landman Gay | jacque at hyperactivesw.com HyperActive Software | http://www.hyperactivesw.com From revdev at pdslabs.net Fri Jul 29 18:46:37 2011 From: revdev at pdslabs.net (Phil Davis) Date: Fri, 29 Jul 2011 15:46:37 -0700 Subject: stackfiles -- how to use? In-Reply-To: References: Message-ID: <4E33384D.60600@pdslabs.net> Hi Nicolas, Coming late to the party... For an example of what 'the stackfiles' look like, open the IDE message box and type" put the stackfiles of stack "home" Phil Davis On 7/24/11 7:42 PM, Nicolas Cueto wrote: > Hello, > > Apologies for the length of this, but I'd very much like list help to > understand better a scripting practice I've been doing half-baked for > a long time now. > > It has to do with stackFiles (I think?), the steps for implementing > which I describe below, but first precede with an outline of what I'm > after. > > > Essentially, this is the structure of my standalone and stacks: > > AllMightyStack.exe --> Index-stack.livecode --> gamestacks1-8.livecode > > > The role of AllMightyStack.exe: > -- opens up Index-stack.livecode > -- stays open but hidden in background > -- (most importantly!) contains handlers/functions and global > variables -- but no controls/objects -- that can be called by the > index-stack and each of the gamestacks simply by name (ie, without > having to specify a path) > > The role of Index-stack.rev: > -- shows the user some buttons, clicking which opens up one of the > various gamestacks > -- stays open but hidden in background, so that, when a gamestack is > done with, the user returns to Index-stack to either choose a > different gamestack or quit > -- does not contain handlers or controls referenced to by any other stack > > > To do the above, here are my steps: > > STEP 1) place the following script in card 1 of AllMightyStack.exe: > > on openCard > setUpFoundation > end openCard > > on setUpFoundation > put "Index-stack,Index-stack.rev"& cr& "game1,game1.rev"& cr& \ > "game2,game2.rev"& cr& ....& cr& "animationEngine,animationEngine.rev" \ > into tStackFiles > set the stackFiles of stack "Mainstack.exe" to tStackFiles > ... > open stack "Index-stack" > hide stack "Mainstack" > end setUpFoundation > > > > STEP 2) in Livecode open the property-inspector for stack > "AllMightyStack.rev" and then in the "Stack files" panel add > Index-Stack and all the game-stacks. After which, I save as a > standalone. > > > STEP 3) repeat the above two steps for Index-stack.rev > > > Anyway, I'm guessing I don't need to do STEP 3. But now I'm also > guessing STEP 2 is superfluous. Or is it? > > Of course, I've read the documentation. But... > > > Hope that all makes sense. > > Thank you -- for your help and forbearance. > > -- > Nicolas Cueto > > _______________________________________________ > use-livecode mailing list > use-livecode at lists.runrev.com > Please visit this url to subscribe, unsubscribe and manage your subscription preferences: > http://lists.runrev.com/mailman/listinfo/use-livecode > -- Phil Davis PDS Labs Professional Software Development http://pdslabs.net From johnpatten at mac.com Fri Jul 29 19:28:04 2011 From: johnpatten at mac.com (John Patten) Date: Fri, 29 Jul 2011 16:28:04 -0700 Subject: iRev mySQL and a Revdberr? Message-ID: <0C2C1E12-33AB-40A1-AC57-4376708BBACF@mac.com> Hi All! I have a basic insert mysql query in an irev file. It manages to insert the new record but it returns a revdberr too. The table consists of 5 fields with the last field auto-updated incrementally. put "INSERT INTO opentrades (sendtoid, traderid, traderitem, tradeeitem) VALUES (" & "'" & tTradeeID & "'" & "," & "'" & tPlayerID & "'" & "," & "'" & tMyItem & "'" & "," & "'" & tTheirItem & "'" & ")" into tSQL It works but still returns the short "revdberr" message... I have another irev file, and update, that does the same kind of thing. It works, but returns an error. put "UPDATE location SET xcoord='" & txcoord & "', ycoord='" & tycoord & "' WHERE id='" & tTargetID & "'" into tSQL Anything jump out a miss? Wondering why it would be a successful query either creating a new record or updating and existing record but still return an error??? Thank you! John Patten SUSD From admin at mfelkerco.com Fri Jul 29 19:35:19 2011 From: admin at mfelkerco.com (Admin) Date: Fri, 29 Jul 2011 18:35:19 -0500 Subject: EMERGENCY - File Browser not showing up in IE and other idiosyncracies with the Revlet Player Message-ID: <64b38b2ab12a50508a379c9c308a329d@mfelkerco.com> In my cards and scripts in Live Code, loading files work fine by pressing the BROWSE button (I made a button for each upload and labelled it BROWSE). You click it and a file box comes up with the proper filters. Everything works great! However, when using the same card on the internet (a revlet), pressing the BROWSE button does nothing. Is there some special procedure or syntax I have to follow to use file requesters from a revlet? Here is my code: ============================= # Start by getting the file to upload local tFileForUpload, tFileName answer file "Select a file to upload" with filter "Video Files,*.avi;*.mpg;*.mp4;*.mov;*.wmv" put it into tFileForUpload # Get the name of the file for upload set the itemdel to "/" put the last item of tFileForUpload into tFileName put empty into field "fldVidLink" # Connect the start the upload local tDestination put "http://www." & "travelandworkusa.com" &FilePath &tFileName into tFieldDest put "ftp://" & FTPUSER & ":" & FTPPASS & "@" & "ftp.travelandworkusa.com" &FilePath & tFileName into tDestination libURLSetStatusCallback "uploadProgress", the long ID of me put the long ID of me into progressmeter libURLftpUploadFile tFileForUpload, tDestination, "uploadComplete" put tFieldDest into field "fldVidLink" end mouseUp on uploadComplete pURL, pStatus put "Status Update:" && pStatus && return after field "lblProgress" end uploadComplete on uploadProgress pURL, pStatus put "Status Update:" && pStatus && return before field "lblProgress" --set the thumbpos of sb "prgScrollBar" to progressmeter end uploadProgress ============================================= Anything wrong with this as it pertains to FTPing to the server from a local file - from the context of a revlet online? After going through it, it's the FILE REQUESTER that is not working for me. I can FTP fine. I included ALL the libraries in the stand alone settings, just in case. So, does anyone know why the file requester would not come up in a revlet? How can I fix this - it's MAJOR for me to fix this. Thanks. Mike Felker P.S. Ken, a very nice gentleman from Sons of Thunder Software helped me out on the phone and was able to make some of this work on the Mac, but pretty much everything I experienced he did as well. From mwieder at ahsoftware.net Fri Jul 29 20:25:56 2011 From: mwieder at ahsoftware.net (Mark Wieder) Date: Fri, 29 Jul 2011 17:25:56 -0700 Subject: [OT] More Apple Foolishness In-Reply-To: <4E331F08.40408@hyperactivesw.com> References: <6F918963-BD07-4351-83D1-41E1A78D06FD@altuit.com> <00af01cc4cc3$83a64f30$8af2ed90$@com> <80001DA7-522D-4AB6-A812-501492B660BF@mac.com> <76870884-CAEE-4396-B191-B2C540ACA93B@verizon.net> <4E318CA3.2030106@hyperactivesw.com> <18E8E444-EE80-4D1D-8D1C-611515F969B7@mines-paristech.fr> <4E331F08.40408@hyperactivesw.com> Message-ID: <191770568234.20110729172556@ahsoftware.net> Jacque- Friday, July 29, 2011, 1:58:48 PM, you wrote: > It works with any server and with Dropbox, > and you don't even need to leave LiveCode to use it. Sweet! Want. -- -Mark Wieder mwieder at ahsoftware.net From mwieder at ahsoftware.net Fri Jul 29 20:31:47 2011 From: mwieder at ahsoftware.net (Mark Wieder) Date: Fri, 29 Jul 2011 17:31:47 -0700 Subject: LC 4.6.3 on Linux In-Reply-To: <4E31B4A1.2070708@gmail.com> References: <4E31B4A1.2070708@gmail.com> Message-ID: <141770919343.20110729173147@ahsoftware.net> Richmond- Thursday, July 28, 2011, 12:12:33 PM, you wrote: > So; downloaded the Linux version, and after install on Ubuntu 11.04 got a > "setting defaults" message that made me, finally, kill the thing, having > gone away, Don't know about Unbuntu, but on Fedora Core 13 4.6.3 just installed the usual way and is happily being a part of my system. -- -Mark Wieder mwieder at ahsoftware.net From dave.cragg at lacscentre.co.uk Fri Jul 29 20:40:55 2011 From: dave.cragg at lacscentre.co.uk (Dave Cragg) Date: Sat, 30 Jul 2011 01:40:55 +0100 Subject: iRev mySQL and a Revdberr? In-Reply-To: <0C2C1E12-33AB-40A1-AC57-4376708BBACF@mac.com> References: <0C2C1E12-33AB-40A1-AC57-4376708BBACF@mac.com> Message-ID: <79484B34-0631-44C0-B9DD-3A9C2A0FEF96@lacscentre.co.uk> On 30 Jul 2011, at 00:28, John Patten wrote: > Hi All! > > I have a basic insert mysql query in an irev file. It manages to insert the new record but it returns a revdberr too. > > The table consists of 5 fields with the last field auto-updated incrementally. > > put "INSERT INTO opentrades (sendtoid, traderid, traderitem, tradeeitem) VALUES (" & "'" & tTradeeID & "'" & "," & "'" & tPlayerID & "'" & "," & "'" & tMyItem & "'" & "," & "'" & tTheirItem & "'" & ")" into tSQL Just a guess, but are the fields "sendtoid" and "traderid" integer fields. If so, the values shouldn't be quoted. Dave From massung at gmail.com Fri Jul 29 20:41:25 2011 From: massung at gmail.com (Jeff Massung) Date: Fri, 29 Jul 2011 18:41:25 -0600 Subject: LC 4.6.3 on Linux In-Reply-To: <141770919343.20110729173147@ahsoftware.net> References: <4E31B4A1.2070708@gmail.com> <141770919343.20110729173147@ahsoftware.net> Message-ID: I'm running Fedora 15 and it appears to be just fine. But I also haven't tried doing much with it lately. Jeff M. From jacque at hyperactivesw.com Fri Jul 29 20:47:53 2011 From: jacque at hyperactivesw.com (J. Landman Gay) Date: Fri, 29 Jul 2011 19:47:53 -0500 Subject: [OT] More Apple Foolishness In-Reply-To: <191770568234.20110729172556@ahsoftware.net> References: <6F918963-BD07-4351-83D1-41E1A78D06FD@altuit.com> <00af01cc4cc3$83a64f30$8af2ed90$@com> <80001DA7-522D-4AB6-A812-501492B660BF@mac.com> <76870884-CAEE-4396-B191-B2C540ACA93B@verizon.net> <4E318CA3.2030106@hyperactivesw.com> <18E8E444-EE80-4D1D-8D1C-611515F969B7@mines-paristech.fr> <4E331F08.40408@hyperactivesw.com> <191770568234.20110729172556@ahsoftware.net> Message-ID: <4E3354B9.8040504@hyperactivesw.com> On 7/29/11 7:25 PM, Mark Wieder wrote: > Jacque- > > Friday, July 29, 2011, 1:58:48 PM, you wrote: > >> It works with any server and with Dropbox, >> and you don't even need to leave LiveCode to use it. > > Sweet! Want. > No sweat. Give me the weekend to fix it up. -- Jacqueline Landman Gay | jacque at hyperactivesw.com HyperActive Software | http://www.hyperactivesw.com From kray at sonsothunder.com Fri Jul 29 21:04:49 2011 From: kray at sonsothunder.com (Ken Ray) Date: Fri, 29 Jul 2011 20:04:49 -0500 Subject: EMERGENCY - File Browser not showing up in IE and other idiosyncracies with the Revlet Player In-Reply-To: <64b38b2ab12a50508a379c9c308a329d@mfelkerco.com> References: <64b38b2ab12a50508a379c9c308a329d@mfelkerco.com> Message-ID: <776378A2-F1B5-44AD-A487-1A3375C14177@sonsothunder.com> > In my cards and scripts in Live Code, loading files work fine by > pressing the BROWSE button (I made a button for each upload and labelled > it BROWSE). You click it and a file box comes up with the proper > filters. Everything works great! However, when using the same card on > the internet (a revlet), pressing the BROWSE button does nothing. > > Is > there some special procedure or syntax I have to follow to use file > requesters from a revlet? Actually let me clarify - when I was working with Mike it *worked*, but only on certain browsers; for example it worked fine in Safari/Mac, but didn't work at all in Firefox/Windows or Internet Explorer/Windows. It's a simple "answer file", so it *should* work for everything, but I don't have much experience with revlets... does anyone know if "answer file" only works with certain browsers? If so, are there any workarounds? Ken Ray Sons of Thunder Software, Inc. Email: kray at sonsothunder.com Web Site: http://www.sonsothunder.com/ From bruceap at comcast.net Fri Jul 29 21:41:02 2011 From: bruceap at comcast.net (Bruce Pokras) Date: Fri, 29 Jul 2011 21:41:02 -0400 Subject: Path to start using a stack in On-Rev? In-Reply-To: <70265F34-9EA0-4FF5-9E91-6F6DC68E5AAB@comcast.net> References: <4E188107.7070404@hyperactivesw.com> <70265F34-9EA0-4FF5-9E91-6F6DC68E5AAB@comcast.net> Message-ID: If the stack and irev HTML page are in the same folder, what is the path to the stack for the "start using" command? Is it simply: start using stack "Test_Stack.livecode" Or is it something more complicated than that? I am testing by putting a simple function called "respond" in the stack file's stack script, but so far no response in the .irev page when it is supposed to run the command "put respond()". Am I missing something completely? Regards, Bruce Pokras Blazing Dawn Software From lan.kc.macmail at gmail.com Fri Jul 29 22:51:24 2011 From: lan.kc.macmail at gmail.com (Kay C Lan) Date: Sat, 30 Jul 2011 10:51:24 +0800 Subject: Where does survive the inventive user ? In-Reply-To: References: Message-ID: On Wed, Jul 27, 2011 at 6:40 PM, Francis Nugent Dixon wrote: > Renting application use out of a > cloud would be the same has handing over your wallet to a stranger. > Which you already happily do right. I look in my wallet an there are a couple of notes and a couple of plastic cards. The notes represent about 0.01% of iMoney I have in my account. I can use those plastic cards to access the BankCloud and if the strangers at the Bank are willing, the machine will give me more real money on a 1 to 1 reduction of my iMoney. Sometimes I don't even have to change iMoney into real money, I just go to the shop and transfer iMoney from my account to their iMoney account which all resides in the same BankCloud. Of course I have to pay a 'rental' fee for the privilege of being able to access my iMoney at virtually any time or shop. But then again, if a stranger at the bank goes all Nick Leeson on me, the bank will collapse and my iMoney in the BankCloud will disappear like the early morning stratocumulus, leaving me without any real money. Strange what we think we'd never do. Oh, and Tim Miller, I'm just another 'no professional' still with LC, I just don't visit the LC List as often as I'd like because Life is getting more and more complicated, Time is getting shorter and shorter, and between Life, LiveCoding and reading the LiveCode Use List, reading the List is the one I've had to sacrifice. Actually two brand new brainwaves with associated stacks built in the last two weeks and I've got another problem to solve today which undoubtedly will involve a brand new stack. Love LC :-) From johnpatten at me.com Fri Jul 29 23:09:40 2011 From: johnpatten at me.com (John Patten) Date: Fri, 29 Jul 2011 20:09:40 -0700 Subject: iRev mySQL and a Revdberr? In-Reply-To: <79484B34-0631-44C0-B9DD-3A9C2A0FEF96@lacscentre.co.uk> References: <0C2C1E12-33AB-40A1-AC57-4376708BBACF@mac.com> <79484B34-0631-44C0-B9DD-3A9C2A0FEF96@lacscentre.co.uk> Message-ID: Ah! Thanks Dave! They are. I'll make the change and see if that helps. John Patten SUSD Sent from my iPad On Jul 29, 2011, at 5:40 PM, Dave Cragg wrote: > > On 30 Jul 2011, at 00:28, John Patten wrote: > >> Hi All! >> >> I have a basic insert mysql query in an irev file. It manages to insert the new record but it returns a revdberr too. >> >> The table consists of 5 fields with the last field auto-updated incrementally. >> >> put "INSERT INTO opentrades (sendtoid, traderid, traderitem, tradeeitem) VALUES (" & "'" & tTradeeID & "'" & "," & "'" & tPlayerID & "'" & "," & "'" & tMyItem & "'" & "," & "'" & tTheirItem & "'" & ")" into tSQL > > Just a guess, but are the fields "sendtoid" and "traderid" integer fields. If so, the values shouldn't be quoted. > > Dave > _______________________________________________ > use-livecode mailing list > use-livecode at lists.runrev.com > Please visit this url to subscribe, unsubscribe and manage your subscription preferences: > http://lists.runrev.com/mailman/listinfo/use-livecode From jacque at hyperactivesw.com Sat Jul 30 00:50:53 2011 From: jacque at hyperactivesw.com (J. Landman Gay) Date: Fri, 29 Jul 2011 23:50:53 -0500 Subject: EMERGENCY - File Browser not showing up in IE and other idiosyncracies with the Revlet Player In-Reply-To: <776378A2-F1B5-44AD-A487-1A3375C14177@sonsothunder.com> References: <64b38b2ab12a50508a379c9c308a329d@mfelkerco.com> <776378A2-F1B5-44AD-A487-1A3375C14177@sonsothunder.com> Message-ID: <4E338DAD.3010306@hyperactivesw.com> On 7/29/11 8:04 PM, Ken Ray wrote: >> In my cards and scripts in Live Code, loading files work fine by >> pressing the BROWSE button (I made a button for each upload and >> labelled it BROWSE). You click it and a file box comes up with the >> proper filters. Everything works great! However, when using the >> same card on the internet (a revlet), pressing the BROWSE button >> does nothing. >> >> Is there some special procedure or syntax I have to follow to use >> file requesters from a revlet? > > Actually let me clarify - when I was working with Mike it *worked*, > but only on certain browsers; for example it worked fine in > Safari/Mac, but didn't work at all in Firefox/Windows or Internet > Explorer/Windows. > > It's a simple "answer file", so it *should* work for everything, but > I don't have much experience with revlets... does anyone know if > "answer file" only works with certain browsers? If so, are there any > workarounds? I haven't done much with revlets recently, but the two browsers you say don't work are the same two that don't work with any revlets at all for me. When they went to 64-bit, all my existing revlets refused to load. I think "answer file" used to work everywhere back when we were all 32-bit. There's an html fix for the loading problem, but that wouldn't help if "answer file" needs 32-bit. If you have older, 32-bit versions of those browsers, one of you could test. -- Jacqueline Landman Gay | jacque at hyperactivesw.com HyperActive Software | http://www.hyperactivesw.com From martinblackman at gmail.com Sat Jul 30 00:54:10 2011 From: martinblackman at gmail.com (Martin Blackman) Date: Sat, 30 Jul 2011 12:54:10 +0800 Subject: Where does survive the inventive user ? In-Reply-To: References: Message-ID: Kay echoes my sentiments, both with regards to time and appreciation of LC! FWIW I'm a non-pro who found Rev when version 1 was given away on the front of a computer mag, and once I got past "where's the equivalent of a writeline command??" and managed hello world I found the learning curve to be pretty manageable. On 30 July 2011 10:51, Kay C Lan wrote: > O > Oh, and Tim Miller, I'm just another 'no professional' still with LC, I > just > don't visit the LC List as often as I'd like because Life is getting more > and more complicated, Time is getting shorter and shorter, and between > Life, > LiveCoding and reading the LiveCode Use List, reading the List is the one > I've had to sacrifice. > > Actually two brand new brainwaves with associated stacks built in the last > two weeks and I've got another problem to solve today which undoubtedly > will > involve a brand new stack. > > Love LC :-) > _______________________________________________ > use-livecode mailing list > use-livecode at lists.runrev.com > Please visit this url to subscribe, unsubscribe and manage your > subscription preferences: > http://lists.runrev.com/mailman/listinfo/use-livecode > From admin at mfelkerco.com Sat Jul 30 01:01:44 2011 From: admin at mfelkerco.com (Admin) Date: Sat, 30 Jul 2011 00:01:44 -0500 Subject: EMERGENCY - File Browser not showing up in IE and other idiosyncracies with the Revlet Player In-Reply-To: <4E338DAD.3010306@hyperactivesw.com> References: <64b38b2ab12a50508a379c9c308a329d@mfelkerco.com> <776378A2-F1B5-44AD-A487-1A3375C14177@sonsothunder.com> <4E338DAD.3010306@hyperactivesw.com> Message-ID: Yes, I was able to change the html auto-generated code to get IE to load revlets. That's not the problem. I have random problems - some can use it on Mac running Safari and others cannot (they can't even click on a button) Random problems in IE and reliable problems with attempting to open a file browser, which works on other browsers, but the ftp commands don't seem to work at all. The stand-alone versions work 100%. This is infuriating. Mike On Fri, 29 Jul 2011 23:50:53 -0500, J. Landman Gay wrote: > On 7/29/11 8:04 PM, Ken Ray wrote: > >>> In my cards and scripts in Live Code, loading files work fine by pressing the BROWSE button (I made a button for each upload and labelled it BROWSE). You click it and a file box comes up with the proper filters. Everything works great! However, when using the same card on the internet (a revlet), pressing the BROWSE button does nothing. Is there some special procedure or syntax I have to follow to use file requesters from a revlet? >> Actually let me clarify - when I was working with Mike it *worked*, but only on certain browsers; for example it worked fine in Safari/Mac, but didn't work at all in Firefox/Windows or Internet Explorer/Windows. It's a simple "answer file", so it *should* work for everything, but I don't have much experience with revlets... does anyone know if "answer file" only works with certain browsers? If so, are there any workarounds? > > I haven't done much with revlets recently, but the two browsers you say > don't work are the same two that don't work with any revlets at all for > me. When they went to 64-bit, all my existing revlets refused to load. > > I think "answer file" used to work everywhere back when we were all > 32-bit. There's an html fix for the loading problem, but that wouldn't > help if "answer file" needs 32-bit. If you have older, 32-bit versions > of those browsers, one of you could test. From henshaw at me.com Sat Jul 30 06:51:13 2011 From: henshaw at me.com (Andy Henshaw) Date: Sat, 30 Jul 2011 11:51:13 +0100 Subject: [OT] More Apple Foolishness In-Reply-To: <4E3354B9.8040504@hyperactivesw.com> References: <6F918963-BD07-4351-83D1-41E1A78D06FD@altuit.com> <00af01cc4cc3$83a64f30$8af2ed90$@com> <80001DA7-522D-4AB6-A812-501492B660BF@mac.com> <76870884-CAEE-4396-B191-B2C540ACA93B@verizon.net> <4E318CA3.2030106@hyperactivesw.com> <18E8E444-EE80-4D1D-8D1C-611515F969B7@mines-paristech.fr> <4E331F08.40408@hyperactivesw.com> <191770568234.20110729172556@ahsoftware.net> <4E3354B9.8040504@hyperactivesw.com> Message-ID: <05303ABD-84C0-4BEF-8918-0EB729B3B419@me.com> Yes, its sound great!!! I think it would take me slightly more than an hour!!!! Andy On 30 Jul 2011, at 01:47, J. Landman Gay wrote: > On 7/29/11 7:25 PM, Mark Wieder wrote: >> Jacque- >> >> Friday, July 29, 2011, 1:58:48 PM, you wrote: >> >>> It works with any server and with Dropbox, >>> and you don't even need to leave LiveCode to use it. >> >> Sweet! Want. >> > > No sweat. Give me the weekend to fix it up. > > -- > Jacqueline Landman Gay | jacque at hyperactivesw.com > HyperActive Software | http://www.hyperactivesw.com > > _______________________________________________ > use-livecode mailing list > use-livecode at lists.runrev.com > Please visit this url to subscribe, unsubscribe and manage your subscription preferences: > http://lists.runrev.com/mailman/listinfo/use-livecode From ambassador at fourthworld.com Sat Jul 30 10:42:38 2011 From: ambassador at fourthworld.com (Richard Gaskin) Date: Sat, 30 Jul 2011 07:42:38 -0700 Subject: Where does survive the inventive user ? In-Reply-To: References: Message-ID: <4E34185E.9070405@fourthworld.com> Kay C Lan wrote: > I look in my wallet an there are a couple of notes and a couple of plastic > cards. The notes represent about 0.01% of iMoney I have in my account. I can > use those plastic cards to access the BankCloud and if the strangers at the > Bank are willing, the machine will give me more real money on a 1 to 1 > reduction of my iMoney. Sometimes I don't even have to change iMoney into > real money, I just go to the shop and transfer iMoney from my account to > their iMoney account which all resides in the same BankCloud. Of course I > have to pay a 'rental' fee for the privilege of being able to access my > iMoney at virtually any time or shop. But then again, if a stranger at the > bank goes all Nick Leeson on me, the bank will collapse and my iMoney in the > BankCloud will disappear like the early morning stratocumulus, leaving me > without any real money. > > Strange what we think we'd never do. Or even stranger, we could trust our money to so-called professional money managers who mishandle it so badly that the world economy is brought to the edge of collapse. ;) For all of our concerns about online security, one of the most common methods is still the most old-school: Every day millions of us go to restaurants where we hand our credit card to a stranger who takes it out of the room for several minutes. Ostensibly they're merely processing the transaction, but of course we have no way to know exactly what happens while the card is out of the room. I do a lot of online banking, but the only instance of identity theft I've experienced was from numbers stolen off a card I only use in the physical world. Further irony: most old-school modems used at restaurants and retail stores transmit without encryption over standard phone lines. While those lines aren't exposed to the Internet, they are vulnerable to any physical interception of traffic, such as tapping the local trunk. And here in the States we have a growing problem with fake or modified ATMs that skim card data. Remember: just because you're paranoid doesn't mean they aren't out to get you. :) -- Richard Gaskin Fourth World LiveCode training and consulting: http://www.fourthworld.com Webzine for LiveCode developers: http://www.LiveCodeJournal.com LiveCode Journal blog: http://LiveCodejournal.com/blog.irv From mikekann at yahoo.com Sat Jul 30 11:16:43 2011 From: mikekann at yahoo.com (Michael Kann) Date: Sat, 30 Jul 2011 08:16:43 -0700 (PDT) Subject: Path to start using a stack in On-Rev? In-Reply-To: Message-ID: <1312039003.39634.YahooMailClassic@web161612.mail.bf1.yahoo.com> Bruce, Not sure about the path, but there is one other item to consider. If your web page is good_page.irev then the old 3.5 engine will be used, so you aren't going to connect to the stack no matter where it is. Try file: good_page.lc within that page you can use either or The file name seems to be the determining factor. Mike --- On Fri, 7/29/11, Bruce Pokras wrote: From: Bruce Pokras Subject: Path to start using a stack in On-Rev? To: "How to use LiveCode" Date: Friday, July 29, 2011, 8:41 PM If the stack and irev HTML page are in the same folder, what is the path to the stack for the "start using" command? Is it simply: start using stack "Test_Stack.livecode" Or is it something more complicated than that? I am testing by putting a simple function called "respond" in the stack file's stack script, but so far no response in the .irev page when it is supposed to run the command "put respond()". Am I missing something completely? Regards, Bruce Pokras Blazing Dawn Software _______________________________________________ use-livecode mailing list use-livecode at lists.runrev.com Please visit this url to subscribe, unsubscribe and manage your subscription preferences: http://lists.runrev.com/mailman/listinfo/use-livecode From pmbrig at gmail.com Sat Jul 30 11:20:38 2011 From: pmbrig at gmail.com (Peter Brigham MD) Date: Sat, 30 Jul 2011 11:20:38 -0400 Subject: Where does survive the inventive user ? In-Reply-To: <4E34185E.9070405@fourthworld.com> References: <4E34185E.9070405@fourthworld.com> Message-ID: On Jul 30, 2011, at 10:42 AM, Richard Gaskin wrote: > Kay C Lan wrote: > >> I look in my wallet an there are a couple of notes and a couple of plastic >> cards. The notes represent about 0.01% of iMoney I have in my account. I can >> use those plastic cards to access the BankCloud and if the strangers at the >> Bank are willing, the machine will give me more real money on a 1 to 1 >> reduction of my iMoney. Sometimes I don't even have to change iMoney into >> real money, I just go to the shop and transfer iMoney from my account to >> their iMoney account which all resides in the same BankCloud. Of course I >> have to pay a 'rental' fee for the privilege of being able to access my >> iMoney at virtually any time or shop. But then again, if a stranger at the >> bank goes all Nick Leeson on me, the bank will collapse and my iMoney in the >> BankCloud will disappear like the early morning stratocumulus, leaving me >> without any real money. >> >> Strange what we think we'd never do. > > Or even stranger, we could trust our money to so-called professional money managers who mishandle it so badly that the world economy is brought to the edge of collapse. ;) I've always wondered why they're called "brokers" ... I think if I were in that line of work I'd find another way of describing it. -- Peter Peter M. Brigham pmbrig at gmail.com http://home.comcast.net/~pmbrig From mikekann at yahoo.com Sat Jul 30 11:21:08 2011 From: mikekann at yahoo.com (Michael Kann) Date: Sat, 30 Jul 2011 08:21:08 -0700 (PDT) Subject: Path to start using a stack in On-Rev? In-Reply-To: <1312039003.39634.YahooMailClassic@web161612.mail.bf1.yahoo.com> Message-ID: <1312039268.48913.YahooMailClassic@web161606.mail.bf1.yahoo.com> A little mistake in the last post: wrote: From: Michael Kann Subject: Re: Path to start using a stack in On-Rev? To: "How to use LiveCode" Date: Saturday, July 30, 2011, 10:16 AM Bruce, Not sure about the path, but there is one other item to consider. If your web page is good_page.irev then the old 3.5 engine will be used, so you aren't going to connect to the stack no matter where it is. Try file: good_page.lc within that page you can use either or The file name seems to be the determining factor. Mike --- On Fri, 7/29/11, Bruce Pokras wrote: From: Bruce Pokras Subject: Path to start using a stack in On-Rev? To: "How to use LiveCode" Date: Friday, July 29, 2011, 8:41 PM If the stack and irev HTML page are in the same folder, what is the path to the stack for the "start using" command? Is it simply: start using stack "Test_Stack.livecode" Or is it something more complicated than that? I am testing by putting a simple function called "respond" in the stack file's stack script, but so far no response in the .irev page when it is supposed to run the command "put respond()". Am I missing something completely? Regards, Bruce Pokras Blazing Dawn Software _______________________________________________ use-livecode mailing list use-livecode at lists.runrev.com Please visit this url to subscribe, unsubscribe and manage your subscription preferences: http://lists.runrev.com/mailman/listinfo/use-livecode _______________________________________________ use-livecode mailing list use-livecode at lists.runrev.com Please visit this url to subscribe, unsubscribe and manage your subscription preferences: http://lists.runrev.com/mailman/listinfo/use-livecode From dixonja at hotmail.co.uk Sat Jul 30 11:27:20 2011 From: dixonja at hotmail.co.uk (John Dixon) Date: Sat, 30 Jul 2011 16:27:20 +0100 Subject: Where does survive the inventive user ? In-Reply-To: References: , <4E34185E.9070405@fourthworld.com>, Message-ID: :-) > > I've always wondered why they're called "brokers" ... I think if I were in that line of work I'd find another way of describing it. > > -- Peter > > Peter M. Brigham > pmbrig at gmail.com > http://home.comcast.net/~pmbrig From roger.e.eller at sealedair.com Sat Jul 30 11:31:40 2011 From: roger.e.eller at sealedair.com (Roger Eller) Date: Sat, 30 Jul 2011 11:31:40 -0400 Subject: Where does survive the inventive user ? In-Reply-To: <4E34185E.9070405@fourthworld.com> References: <4E34185E.9070405@fourthworld.com> Message-ID: On Sat, Jul 30, 2011 at 10:42 AM, Richard Gaskin wrote: > Kay C Lan wrote: > > I look in my wallet an there are a couple of notes and a couple of plastic >> cards. The notes represent about 0.01% of iMoney I have in my account. I >> can >> use those plastic cards to access the BankCloud and if the strangers at >> the >> Bank are willing, the machine will give me more real money on a 1 to 1 >> reduction of my iMoney. Sometimes I don't even have to change iMoney into >> real money, I just go to the shop and transfer iMoney from my account to >> their iMoney account which all resides in the same BankCloud. Of course I >> have to pay a 'rental' fee for the privilege of being able to access my >> iMoney at virtually any time or shop. But then again, if a stranger at the >> bank goes all Nick Leeson on me, the bank will collapse and my iMoney in >> the >> BankCloud will disappear like the early morning stratocumulus, leaving me >> without any real money. >> >> Strange what we think we'd never do. >> > > Or even stranger, we could trust our money to so-called professional money > managers who mishandle it so badly that the world economy is brought to the > edge of collapse. ;) > This video is a very informative explanation of where money 'actually' comes from. It's shocking if you've not seen it before. http://video.google.com/videoplay?docid=-2550156453790090544# ?Roger From Mark_Smith at cpe.umanitoba.ca Sat Jul 30 11:56:55 2011 From: Mark_Smith at cpe.umanitoba.ca (Mark Smith) Date: Sat, 30 Jul 2011 10:56:55 -0500 Subject: LC Summer Academy In-Reply-To: <5BF66A2B-7911-441E-A49D-1BC3E7713A25@verizon.net> References: <4DB099A6.70 90403@fourthworld.com>< 8CDCE8898FE3AFB-EF0-86AB@webmail-d015.sysops.aol.com><48002F50-AB4D-499C-A5AF-2BC50D2C1F54@twft.com> , <5BF66A2B-7911-441E-A49D-1BC3E7713A25@verizon.net> Message-ID: <3F95480D5F0DA64EA0785711087864DEAA47E704DC@exch01.mchpe.cpe.umanitoba.ca> The Summer Academy was incredibly useful (and hats off to Elanor for being such a great presenter). A ton of information was presented in the 7 weeks, so much so that I had trouble keeping track of it all and definitely wanted a way to be able to go back and review it later. So I created the Summer Academy '11 Index... over 600 topical entries covering all of the material presented, including references to many livecode commands and tutorial links, and all keyword searchable. I've posted it to the RevOnline user samples (I think you can find it by searching for Index or Academy). Here are a couple of tips to help get you started: if you want to search for everything presented in a given week type Wk followed by the week number. If you want to know just what was presented in the Webinar portion, type Wk?w and replace ? with the week number you are interested in. This gives a kind of minute by minute overview of the webinar presentation. Interested in knowing what links where discussed? search for http. For those of you who attended, and for those of you interested in knowing what was covered I hope this tool is useful. Cheers, -- Mark ________________________________________ From: use-livecode-bounces at lists.runrev.com [use-livecode-bounces at lists.runrev.com] On Behalf Of Colin Holgate [coiin at verizon.net] Sent: Thursday, April 21, 2011 7:19 PM To: How to use LiveCode Subject: Re: LC Summer Academy On Apr 21, 2011, at 7:36 PM, Andre Garzia wrote: > I don't know what we're talking about, anyone can provide a link? http://www.runrev.com/newsletter/april/issue109/newsletter1.php _______________________________________________ use-livecode mailing list use-livecode at lists.runrev.com Please visit this url to subscribe, unsubscribe and manage your subscription preferences: http://lists.runrev.com/mailman/listinfo/use-livecode From bvg at mac.com Sat Jul 30 13:01:57 2011 From: bvg at mac.com (=?iso-8859-1?Q?Bj=F6rnke_von_Gierke?=) Date: Sat, 30 Jul 2011 19:01:57 +0200 Subject: [ANN] LiveCode.tv event #34 In-Reply-To: References: <6B865405-BBC0-491C-AD25-BFB1C85A2D58@mac.com> <7EC6A85B-DD6B-4138-A597-512A194F4453@mac.com> <89782C22-C6E3-4245-95D1-A48F69423753@mac.com> <061D9182-B34B-4ABB-A7D6-9C2D09DA8755@mac.com> <73F84954-890B-4627-859A-702C8054F13B@mac.com> <093DEF5E-6C0F-4924-A262-F6DBE748ED45@mac.com> <3FB852C9-5675-4A19-9506-543733BB2547@mac.com> <914DF999-596E-4023-A213-9469C6A69FA0@mac.com> <2B105965-FB27-41D0-B95F-1F4C84ADD53B@mac.com> <163225D1-67D1-4CE7-8049-E85A8D94D177@mac.com> <225B0941-5D11-434A-BC0B-CD61B998E9F8@mac.com> <201756EB-9601-417D-856E-5128C5256EF1@mac.com> <081FD717-0A48-447D-90AA-A8F370B14F43@mac.com> <913B7E78-3052-4CF0-9883-C1CDD9F8BDFA@mac.com> <0C1B6D89-1745-46DF-9C49-A5FE2E5B67B9@mac.com> <497CC7C9-74BC-419C-9785-CEA07A8FE181@mac.com> Message-ID: One hour to go... approximately. Remember that John Caig was so kind to step up as second presenter. see you in chatrev. On 29 Jul 2011, at 14:57, Bj?rnke von Gierke wrote: > Tomorrows event will be of reduced scope due to a lack of volunteers. All applicants welcome. > > I will introduce the release of BvG Docu 2. > > Join chatrev at 20:00 CET. Go to > http://bjoernke.com/chatrev/ > or enter in the message box: > go stack URL ?http://bjoernke.com/chatrev/chatrev1.3b3.rev? > > Sat. 22:00 Moscow > Sat. 14:00 New York > Sat. 11:00 Los Angeles > Sun. 2:00 Beijing > > See http://livecode.tv/ for more info. > > Mail me to do your presentation, it doesn't matter if you do one in 4 months or right now. > Bj?rnke > > > > _______________________________________________ > use-livecode mailing list > use-livecode at lists.runrev.com > Please visit this url to subscribe, unsubscribe and manage your subscription preferences: > http://lists.runrev.com/mailman/listinfo/use-livecode From bruceap at comcast.net Sat Jul 30 13:22:45 2011 From: bruceap at comcast.net (Bruce Pokras) Date: Sat, 30 Jul 2011 13:22:45 -0400 Subject: Path to start using a stack in On-Rev? In-Reply-To: <1312039268.48913.YahooMailClassic@web161606.mail.bf1.yahoo.com> References: <1312039268.48913.YahooMailClassic@web161606.mail.bf1.yahoo.com> Message-ID: <85CC2950-EBF0-4083-9DE2-8C24F27D9033@comcast.net> Mike, thanks very much. Changing the HTML file's extension from .irev to .lc worked. The web page is now displaying the result of the function that is in the stack script of my test stack file. I had not seen anything about the file extension being critical to functionality on On-Rev. However, here is another mystery. Now that my HTML file has the .lc extension, the On-Rev app does not recognize it! I have to use a text editor to edit it, or keep it as .irev and change it to .lc at deployment. Do you know why that should be? Regards, Bruce On Jul 30, 2011, at 11:21 AM, Michael Kann wrote: > A little mistake in the last post: > > > > > --- On Sat, 7/30/11, Michael Kann wrote: > > From: Michael Kann > Subject: Re: Path to start using a stack in On-Rev? > To: "How to use LiveCode" > Date: Saturday, July 30, 2011, 10:16 AM > > Bruce, > > Not sure about the path, but there is one other item to consider. If your web page is good_page.irev then the old 3.5 engine will be used, so you aren't going to connect to the stack no matter where it is. > > Try > > file: good_page.lc > > within that page you can use either > > put "good stuff here" > ?> > > or > > > put "good stuff here" > > ?> > > The file name seems to be the determining factor. > > Mike > > > > > > --- On Fri, 7/29/11, Bruce Pokras wrote: > > From: Bruce Pokras > Subject: Path to start using a stack in On-Rev? > To: "How to use LiveCode" > Date: Friday, July 29, 2011, 8:41 PM > > If the stack and irev HTML page are in the same folder, what is the path to the stack for the "start using" command? Is it simply: > > start using stack "Test_Stack.livecode" > > Or is it something more complicated than that? I am testing by putting a simple function called "respond" in the stack file's stack script, but so far no response in the .irev page when it is supposed to run the command "put respond()". > > Am I missing something completely? > > Regards, > > Bruce Pokras > Blazing Dawn Software > _______________________________________________ > use-livecode mailing list > use-livecode at lists.runrev.com > Please visit this url to subscribe, unsubscribe and manage your subscription preferences: > http://lists.runrev.com/mailman/listinfo/use-livecode > _______________________________________________ > use-livecode mailing list > use-livecode at lists.runrev.com > Please visit this url to subscribe, unsubscribe and manage your subscription preferences: > http://lists.runrev.com/mailman/listinfo/use-livecode > _______________________________________________ > use-livecode mailing list > use-livecode at lists.runrev.com > Please visit this url to subscribe, unsubscribe and manage your subscription preferences: > http://lists.runrev.com/mailman/listinfo/use-livecode From mikekann at yahoo.com Sat Jul 30 14:34:19 2011 From: mikekann at yahoo.com (Michael Kann) Date: Sat, 30 Jul 2011 11:34:19 -0700 (PDT) Subject: Path to start using a stack in On-Rev? In-Reply-To: <85CC2950-EBF0-4083-9DE2-8C24F27D9033@comcast.net> Message-ID: <1312050859.62628.YahooMailClassic@web161610.mail.bf1.yahoo.com> Bruce, I'm not sure why the on-rev app doesn't approve of .lc yet. There is probably a line in a preferences file somewhere that hasn't been changed yet. Mike --- On Sat, 7/30/11, Bruce Pokras wrote: From: Bruce Pokras Subject: Re: Path to start using a stack in On-Rev? To: "How to use LiveCode" Date: Saturday, July 30, 2011, 12:22 PM Mike, thanks very much. Changing the HTML file's extension from .irev to .lc worked. The web page is now displaying the result of the function that is in the stack script of my test stack file. I had not seen anything about the file extension being critical to functionality on On-Rev. However, here is another mystery. Now that my HTML file has the .lc extension, the On-Rev app does not recognize it! I have to use a text editor to edit it, or keep it as .irev and change it to .lc at deployment. Do you know why that should be? Regards, Bruce On Jul 30, 2011, at 11:21 AM, Michael Kann wrote: > A little mistake in the last post: > > > > > --- On Sat, 7/30/11, Michael Kann wrote: > > From: Michael Kann > Subject: Re: Path to start using a stack in On-Rev? > To: "How to use LiveCode" > Date: Saturday, July 30, 2011, 10:16 AM > > Bruce, > > Not sure about the path, but there is one other item to consider. If your web page is good_page.irev then the old 3.5 engine will be used, so you aren't going to connect to the stack no matter where it is. > > Try > > file: good_page.lc > > within that page you can use either > > put "good stuff here" > ?> > > or > > > put "good stuff here" > > ?> > > The file name seems to be the determining factor. > > Mike > > > > > > --- On Fri, 7/29/11, Bruce Pokras wrote: > > From: Bruce Pokras > Subject: Path to start using a stack in On-Rev? > To: "How to use LiveCode" > Date: Friday, July 29, 2011, 8:41 PM > > If the stack and irev HTML page are in the same folder, what is the path to the stack for the "start using" command? Is it simply: > > start using stack "Test_Stack.livecode" > > Or is it something more complicated than that? I am testing by putting a simple function called "respond" in the stack file's stack script, but so far no response in the .irev page when it is supposed to run the command "put respond()". > > Am I missing something completely? > > Regards, > > Bruce Pokras > Blazing Dawn Software > _______________________________________________ > use-livecode mailing list > use-livecode at lists.runrev.com > Please visit this url to subscribe, unsubscribe and manage your subscription preferences: > http://lists.runrev.com/mailman/listinfo/use-livecode > _______________________________________________ > use-livecode mailing list > use-livecode at lists.runrev.com > Please visit this url to subscribe, unsubscribe and manage your subscription preferences: > http://lists.runrev.com/mailman/listinfo/use-livecode > _______________________________________________ > use-livecode mailing list > use-livecode at lists.runrev.com > Please visit this url to subscribe, unsubscribe and manage your subscription preferences: > http://lists.runrev.com/mailman/listinfo/use-livecode _______________________________________________ use-livecode mailing list use-livecode at lists.runrev.com Please visit this url to subscribe, unsubscribe and manage your subscription preferences: http://lists.runrev.com/mailman/listinfo/use-livecode From pete at mollysrevenge.com Sat Jul 30 14:42:04 2011 From: pete at mollysrevenge.com (Pete) Date: Sat, 30 Jul 2011 11:42:04 -0700 Subject: User Interface Question Message-ID: Wondering if anyone has any thoughts on a couple of user interface questions. I'm providing a popup contextual menu when the user right-clicks on a datagrid. I'm also providing the ability to re-order rows in a datagrid by dragging and dropping them. The common thread between these two functions is that there is no on-screen indication that they exist. Popup menus are invisible until they are invoked and drag and drop is equally invisible until someone initiates it. Are there any common methods for indicating to users that features like this are available to them? Pete Molly's Revenge From massung at gmail.com Sat Jul 30 14:48:33 2011 From: massung at gmail.com (Jeff Massung) Date: Sat, 30 Jul 2011 12:48:33 -0600 Subject: User Interface Question In-Reply-To: References: Message-ID: For dragging, I'd go with "handles" on the left side of the datagrid that have hand icon when you hover over them. They've become pretty ubiquitous on webpages. For an example of what I mean: http://jqueryui.com/demos/sortable/ As for a context menu, you be okay with just a tool-tip. People generally try and right-click a lot for things they expect to be there. Otherwise, again, go with something common to show that actions are available. For example, and disclosure triangle on the far right of a UI widget is typically clickable and will bring up a menu. Left-click will bring up your context menu, and r-click anywhere will bring it up as well. Jeff M. On Sat, Jul 30, 2011 at 12:42 PM, Pete wrote: > Wondering if anyone has any thoughts on a couple of user interface > questions. > > I'm providing a popup contextual menu when the user right-clicks on a > datagrid. I'm also providing the ability to re-order rows in a datagrid by > dragging and dropping them. The common thread between these two functions > is that there is no on-screen indication that they exist. Popup menus are > invisible until they are invoked and drag and drop is equally invisible > until someone initiates it. > > Are there any common methods for indicating to users that features like > this > are available to them? > > Pete > Molly's Revenge > _______________________________________________ > use-livecode mailing list > use-livecode at lists.runrev.com > Please visit this url to subscribe, unsubscribe and manage your > subscription preferences: > http://lists.runrev.com/mailman/listinfo/use-livecode > From pete at mollysrevenge.com Sat Jul 30 15:58:10 2011 From: pete at mollysrevenge.com (Pete) Date: Sat, 30 Jul 2011 12:58:10 -0700 Subject: User Interface Question In-Reply-To: References: Message-ID: Thanks Jeff. I'm trying to implement the cursor change and running into some weirdness. In the script of the datagrid, I have these two handlers: *on* mouseEnter *lock* cursor *set* the cursor to hand *end* mouseEnter *on* mouseLeave *unlock* cursor *end* mouseLeave This changes the cursor as expected and most of the time, it returns to the correct cursor when I move the mouse away from the datagrid. However, if I move the cursor above the datagrid, the cursor stays as the hand and does not return to any other image no matter where I move the mouse to. This is the first time I've attempted to control the cursor so no doubt I'm doing something wrong! Pete Molly's Revenge On Sat, Jul 30, 2011 at 11:48 AM, Jeff Massung wrote: > For dragging, I'd go with "handles" on the left side of the datagrid that > have hand icon when you hover over them. They've become pretty ubiquitous > on > webpages. For an example of what I mean: > > http://jqueryui.com/demos/sortable/ > > As for a context menu, you be okay with just a tool-tip. People generally > try and right-click a lot for things they expect to be there. Otherwise, > again, go with something common to show that actions are available. For > example, and disclosure triangle on the far right of a UI widget is > typically clickable and will bring up a menu. Left-click will bring up your > context menu, and r-click anywhere will bring it up as well. > > Jeff M. > > On Sat, Jul 30, 2011 at 12:42 PM, Pete wrote: > > > Wondering if anyone has any thoughts on a couple of user interface > > questions. > > > > I'm providing a popup contextual menu when the user right-clicks on a > > datagrid. I'm also providing the ability to re-order rows in a datagrid > by > > dragging and dropping them. The common thread between these two > functions > > is that there is no on-screen indication that they exist. Popup menus > are > > invisible until they are invoked and drag and drop is equally invisible > > until someone initiates it. > > > > Are there any common methods for indicating to users that features like > > this > > are available to them? > > > > Pete > > Molly's Revenge > > _______________________________________________ > > use-livecode mailing list > > use-livecode at lists.runrev.com > > Please visit this url to subscribe, unsubscribe and manage your > > subscription preferences: > > http://lists.runrev.com/mailman/listinfo/use-livecode > > > _______________________________________________ > use-livecode mailing list > use-livecode at lists.runrev.com > Please visit this url to subscribe, unsubscribe and manage your > subscription preferences: > http://lists.runrev.com/mailman/listinfo/use-livecode > > From williamdesmet at gmail.com Sat Jul 30 16:05:36 2011 From: williamdesmet at gmail.com (William de Smet) Date: Sat, 30 Jul 2011 22:05:36 +0200 Subject: User Interface Question In-Reply-To: References: Message-ID: <6139A280-1392-4060-9A4C-587AAFF85C64@gmail.com> Hi Pete, After unlock you have to set the cursor to arrow, Greetings, William ----- Verstuurd vanaf mijn iPhone! Op 30 jul. 2011 om 21:58 heeft Pete het volgende geschreven: > Thanks Jeff. I'm trying to implement the cursor change and running into > some weirdness. > > In the script of the datagrid, I have these two handlers: > > *on* mouseEnter > > *lock* cursor > > *set* the cursor to hand > > *end* mouseEnter > > > *on* mouseLeave > > *unlock* cursor > > *end* mouseLeave > > > This changes the cursor as expected and most of the time, it returns to the > correct cursor when I move the mouse away from the datagrid. However, if I > move the cursor above the datagrid, the cursor stays as the hand and does > not return to any other image no matter where I move the mouse to. > > > This is the first time I've attempted to control the cursor so no doubt I'm > doing something wrong! > > > Pete > Molly's Revenge > > > > > On Sat, Jul 30, 2011 at 11:48 AM, Jeff Massung wrote: > >> For dragging, I'd go with "handles" on the left side of the datagrid that >> have hand icon when you hover over them. They've become pretty ubiquitous >> on >> webpages. For an example of what I mean: >> >> http://jqueryui.com/demos/sortable/ >> >> As for a context menu, you be okay with just a tool-tip. People generally >> try and right-click a lot for things they expect to be there. Otherwise, >> again, go with something common to show that actions are available. For >> example, and disclosure triangle on the far right of a UI widget is >> typically clickable and will bring up a menu. Left-click will bring up your >> context menu, and r-click anywhere will bring it up as well. >> >> Jeff M. >> >> On Sat, Jul 30, 2011 at 12:42 PM, Pete wrote: >> >>> Wondering if anyone has any thoughts on a couple of user interface >>> questions. >>> >>> I'm providing a popup contextual menu when the user right-clicks on a >>> datagrid. I'm also providing the ability to re-order rows in a datagrid >> by >>> dragging and dropping them. The common thread between these two >> functions >>> is that there is no on-screen indication that they exist. Popup menus >> are >>> invisible until they are invoked and drag and drop is equally invisible >>> until someone initiates it. >>> >>> Are there any common methods for indicating to users that features like >>> this >>> are available to them? >>> >>> Pete >>> Molly's Revenge >>> _______________________________________________ >>> use-livecode mailing list >>> use-livecode at lists.runrev.com >>> Please visit this url to subscribe, unsubscribe and manage your >>> subscription preferences: >>> http://lists.runrev.com/mailman/listinfo/use-livecode >>> >> _______________________________________________ >> use-livecode mailing list >> use-livecode at lists.runrev.com >> Please visit this url to subscribe, unsubscribe and manage your >> subscription preferences: >> http://lists.runrev.com/mailman/listinfo/use-livecode >> >> > _______________________________________________ > use-livecode mailing list > use-livecode at lists.runrev.com > Please visit this url to subscribe, unsubscribe and manage your subscription preferences: > http://lists.runrev.com/mailman/listinfo/use-livecode From pete at mollysrevenge.com Sat Jul 30 16:12:47 2011 From: pete at mollysrevenge.com (Pete) Date: Sat, 30 Jul 2011 13:12:47 -0700 Subject: User Interface Question In-Reply-To: <6139A280-1392-4060-9A4C-587AAFF85C64@gmail.com> References: <6139A280-1392-4060-9A4C-587AAFF85C64@gmail.com> Message-ID: I tried that - didn't make any difference. The cursor returns to arrow automatically with the code I have, except when I move the mouse above the datagrid. Pete Molly's Revenge On Sat, Jul 30, 2011 at 1:05 PM, William de Smet wrote: > Hi Pete, > > After unlock you have to set the cursor to arrow, > > Greetings, > > William > > ----- > Verstuurd vanaf mijn iPhone! > > Op 30 jul. 2011 om 21:58 heeft Pete het volgende > geschreven: > > > Thanks Jeff. I'm trying to implement the cursor change and running into > > some weirdness. > > > > In the script of the datagrid, I have these two handlers: > > > > *on* mouseEnter > > > > *lock* cursor > > > > *set* the cursor to hand > > > > *end* mouseEnter > > > > > > *on* mouseLeave > > > > *unlock* cursor > > > > *end* mouseLeave > > > > > > This changes the cursor as expected and most of the time, it returns to > the > > correct cursor when I move the mouse away from the datagrid. However, if > I > > move the cursor above the datagrid, the cursor stays as the hand and does > > not return to any other image no matter where I move the mouse to. > > > > > > This is the first time I've attempted to control the cursor so no doubt > I'm > > doing something wrong! > > > > > > Pete > > Molly's Revenge > > > > > > > > > > On Sat, Jul 30, 2011 at 11:48 AM, Jeff Massung > wrote: > > > >> For dragging, I'd go with "handles" on the left side of the datagrid > that > >> have hand icon when you hover over them. They've become pretty > ubiquitous > >> on > >> webpages. For an example of what I mean: > >> > >> http://jqueryui.com/demos/sortable/ > >> > >> As for a context menu, you be okay with just a tool-tip. People > generally > >> try and right-click a lot for things they expect to be there. Otherwise, > >> again, go with something common to show that actions are available. For > >> example, and disclosure triangle on the far right of a UI widget is > >> typically clickable and will bring up a menu. Left-click will bring up > your > >> context menu, and r-click anywhere will bring it up as well. > >> > >> Jeff M. > >> > >> On Sat, Jul 30, 2011 at 12:42 PM, Pete wrote: > >> > >>> Wondering if anyone has any thoughts on a couple of user interface > >>> questions. > >>> > >>> I'm providing a popup contextual menu when the user right-clicks on a > >>> datagrid. I'm also providing the ability to re-order rows in a > datagrid > >> by > >>> dragging and dropping them. The common thread between these two > >> functions > >>> is that there is no on-screen indication that they exist. Popup menus > >> are > >>> invisible until they are invoked and drag and drop is equally invisible > >>> until someone initiates it. > >>> > >>> Are there any common methods for indicating to users that features like > >>> this > >>> are available to them? > >>> > >>> Pete > >>> Molly's Revenge > >>> _______________________________________________ > >>> use-livecode mailing list > >>> use-livecode at lists.runrev.com > >>> Please visit this url to subscribe, unsubscribe and manage your > >>> subscription preferences: > >>> http://lists.runrev.com/mailman/listinfo/use-livecode > >>> > >> _______________________________________________ > >> use-livecode mailing list > >> use-livecode at lists.runrev.com > >> Please visit this url to subscribe, unsubscribe and manage your > >> subscription preferences: > >> http://lists.runrev.com/mailman/listinfo/use-livecode > >> > >> > > _______________________________________________ > > use-livecode mailing list > > use-livecode at lists.runrev.com > > Please visit this url to subscribe, unsubscribe and manage your > subscription preferences: > > http://lists.runrev.com/mailman/listinfo/use-livecode > > _______________________________________________ > use-livecode mailing list > use-livecode at lists.runrev.com > Please visit this url to subscribe, unsubscribe and manage your > subscription preferences: > http://lists.runrev.com/mailman/listinfo/use-livecode > > From m.schonewille at economy-x-talk.com Sat Jul 30 16:13:47 2011 From: m.schonewille at economy-x-talk.com (Mark Schonewille) Date: Sat, 30 Jul 2011 22:13:47 +0200 Subject: User Interface Question In-Reply-To: References: Message-ID: Hi Pete, Never offer a popup menu as the only way to access a feature. Put your popup menu into the menubar and enable the menu items in those situations when the user might as well click on the objects to show the popup menu. Show the same menu, or the essential menu items of it, in the optional popup menu. -- Best regards, Mark Schonewille Economy-x-Talk Consulting and Software Engineering Homepage: http://economy-x-talk.com Twitter: http://twitter.com/xtalkprogrammer KvK: 50277553 What does that error mean? Buy LiveCodeErrors for iPhone now http://qery.us/v4 A must-have for LiveCode programmers. On 30 jul 2011, at 20:42, Pete wrote: > Wondering if anyone has any thoughts on a couple of user interface > questions. > > I'm providing a popup contextual menu when the user right-clicks on a > datagrid. I'm also providing the ability to re-order rows in a datagrid by > dragging and dropping them. The common thread between these two functions > is that there is no on-screen indication that they exist. Popup menus are > invisible until they are invoked and drag and drop is equally invisible > until someone initiates it. > > Are there any common methods for indicating to users that features like this > are available to them? > > Pete > Molly's Revenge From pete at mollysrevenge.com Sat Jul 30 16:45:47 2011 From: pete at mollysrevenge.com (Pete) Date: Sat, 30 Jul 2011 13:45:47 -0700 Subject: User Interface Question In-Reply-To: References: Message-ID: Hi Mark, Yes, that sound like good advice. In fact, I just read the Apple HIG about contextual menus and they recommend the same approach, except they recommend the use of the standard Apple Action menu rather than a menubar entry. Pete Molly's Revenge On Sat, Jul 30, 2011 at 1:13 PM, Mark Schonewille < m.schonewille at economy-x-talk.com> wrote: > Hi Pete, > > Never offer a popup menu as the only way to access a feature. Put your > popup menu into the menubar and enable the menu items in those situations > when the user might as well click on the objects to show the popup menu. > Show the same menu, or the essential menu items of it, in the optional popup > menu. > > -- > Best regards, > > Mark Schonewille > > Economy-x-Talk Consulting and Software Engineering > Homepage: http://economy-x-talk.com > Twitter: http://twitter.com/xtalkprogrammer > KvK: 50277553 > > What does that error mean? Buy LiveCodeErrors for iPhone now > http://qery.us/v4 A must-have for LiveCode programmers. > > On 30 jul 2011, at 20:42, Pete wrote: > > > Wondering if anyone has any thoughts on a couple of user interface > > questions. > > > > I'm providing a popup contextual menu when the user right-clicks on a > > datagrid. I'm also providing the ability to re-order rows in a datagrid > by > > dragging and dropping them. The common thread between these two > functions > > is that there is no on-screen indication that they exist. Popup menus > are > > invisible until they are invoked and drag and drop is equally invisible > > until someone initiates it. > > > > Are there any common methods for indicating to users that features like > this > > are available to them? > > > > Pete > > Molly's Revenge > > > > _______________________________________________ > use-livecode mailing list > use-livecode at lists.runrev.com > Please visit this url to subscribe, unsubscribe and manage your > subscription preferences: > http://lists.runrev.com/mailman/listinfo/use-livecode > > From zryip.theslug at gmail.com Sat Jul 30 17:03:53 2011 From: zryip.theslug at gmail.com (zryip theSlug) Date: Sat, 30 Jul 2011 23:03:53 +0200 Subject: [ANN] Data Grid Helper 1.3.5 Update Message-ID: Dear LC and DGH users, A new update for DGH is available. This new version has the following fix and changes: Fix: After opening the behaviors or the columns topics, some others properties values could appear locked and with an inappropriated tooltip. Changes: 1. All the DGH's substacks are now prefixed by "DGH_" to insure compatibility with possible existing projects. 2. A new window allows you to send us any unexpected errors. 3. Navigation and selection of a property value is now possible by using the keyboard: - Hit "tabkey" to edit the next text property. Use the shiftkey modifier to edit the previous text property. - Hit "return" or "enter" for editing a property, or for opening the content of a menu. - Hit "esc" to cancel the edition. 4. Column builder preview: selecting a column will now select the column header and it's content. If you have not tried DGH yet, a trial version free for 30 days can be downloaded here: http://www.aslugontheroad.com/index.php?option=com_phocadownload&view=category&id=12:trial&Itemid=63 You can also reach the Data Grid Helper plugin on the LiveCode Marketplace: http://www.runrev.com/store/product/data-grid-helper-1-2-0/ Remember, LiveCode has a special THREEFORTWO offer, which will be closed very shortly: http://www.runrev.com/newsletter/july/issue115/newsletter2.php?a=NWS115 Best regards, -- -Zryip TheSlug- wish you the best! 8) http://www.aslugontheroad.co.cc From mkoob at rogers.com Sat Jul 30 17:08:54 2011 From: mkoob at rogers.com (Martin Koob) Date: Sat, 30 Jul 2011 14:08:54 -0700 (PDT) Subject: using stacks with on-rev Message-ID: <1312060134258-3706864.post@n4.nabble.com> Now that on-rev's engine is upgraded to 4.6.3 I thought I would try the 'start using stack' command. I created a simple stack with a couple of functions in it and I uploaded to the directory where my index.irev page is. I put the following command in my index.irev file start using stack "libstackmain.rev" When this page loads it stops at the above line, it displays no error message. I also tried using a different extension for the stack file start using stack "libstackmain.livecode" I got the same result. Are stacks supported on on-rev? If so do I have wrong? What is it relative to? Any other ideas how to get this working? Thanks Martin -- View this message in context: http://runtime-revolution.278305.n4.nabble.com/using-stacks-with-on-rev-tp3706864p3706864.html Sent from the Revolution - User mailing list archive at Nabble.com. From admin at mfelkerco.com Sat Jul 30 17:20:53 2011 From: admin at mfelkerco.com (Admin) Date: Sat, 30 Jul 2011 17:20:53 -0400 Subject: Getting someone's age from dob and current year In-Reply-To: <1312060134258-3706864.post@n4.nabble.com> References: <1312060134258-3706864.post@n4.nabble.com> Message-ID: Hello all, I need to figure out someone's age. I have their date of birth and I have today's date. How do I get the end result. My math skills SUCK. Here's my code (loaded in from the database with a SELECT statement): put item 7 of myLine into dobMonth put item 8 of myLine into dobDay put item 9 of myLine into dobYear put chartonum("dobYear") into dobYearNum put the long system date into field "TodayDate" I figured I would subtract this year from the dobyear and get the age - am I right? How would I do this if I am? Mike From admin at mfelkerco.com Sat Jul 30 17:25:01 2011 From: admin at mfelkerco.com (Admin) Date: Sat, 30 Jul 2011 17:25:01 -0400 Subject: carrying a global variable from stack to sub stack In-Reply-To: <1312060134258-3706864.post@n4.nabble.com> References: <1312060134258-3706864.post@n4.nabble.com> Message-ID: <8632aaae54a6aecd62fdd741c3a1578a@mfelkerco.com> Hello all, How do you carry a global variable from one stack to a substack? I have this set of commands on every page of every stack in my project: on opencard --Create a global variable for the db global dbID global curUserID global UN put numtochar(39) into SQ I then proceed to login to the database (works fine normally) and then access whatever data I need. I just created a substack called CV with three pages - pgCV1, pgCV2 and pgCV3 I need the login credentials of the user to carry over so the fields on these pages can be populated with the existing data on the database. The important variable is dbID and curUserID What do I need to do, other than the global command, to make this happen? Mike From slava at lexiconbridge.com Sat Jul 30 17:34:58 2011 From: slava at lexiconbridge.com (Slava Paperno) Date: Sat, 30 Jul 2011 17:34:58 -0400 Subject: carrying a global variable from stack to sub stack In-Reply-To: <8632aaae54a6aecd62fdd741c3a1578a@mfelkerco.com> References: <1312060134258-3706864.post@n4.nabble.com> <8632aaae54a6aecd62fdd741c3a1578a@mfelkerco.com> Message-ID: <016101cc4f00$8b170e00$a1452a00$@com> > How do you carry a global variable from one stack to a substack? If you set a global variable in the main stack: on preOpenStack global gMyName put "Mike" into gMyName end preOpenStack then in any card in any substack you can retrieve the value like so: on openCard global gMyName answer gMyName --you will see "Mike" end open Card In other words, nothing needs to be done to "carry" a global variable from one stack to another. All you have to do is declare it with the word "global" in front. S. > > I have this set of commands on every page of every stack in > my project: > > on opencard > --Create a global variable for the db > global > dbID > global curUserID > global UN > > put numtochar(39) into SQ > > I then > proceed to login to the database (works fine normally) and then access > whatever data I need. > > I just created a substack called CV with three > pages - pgCV1, pgCV2 and pgCV3 > > I need the login credentials of the user > to carry over so the fields on these pages can be populated with the > existing data on the database. > > The important variable is dbID and > curUserID > > What do I need to do, other than the global command, to make > this happen? > > Mike From stephenREVOLUTION2 at barncard.com Sat Jul 30 19:37:44 2011 From: stephenREVOLUTION2 at barncard.com (stephen barncard) Date: Sat, 30 Jul 2011 16:37:44 -0700 Subject: Path to start using a stack in On-Rev? In-Reply-To: <1312050859.62628.YahooMailClassic@web161610.mail.bf1.yahoo.com> References: <85CC2950-EBF0-4083-9DE2-8C24F27D9033@comcast.net> <1312050859.62628.YahooMailClassic@web161610.mail.bf1.yahoo.com> Message-ID: I remember there was a setting in the client for that. Look in preferences (the gear icon upper right) sqb On 30 July 2011 11:34, Michael Kann wrote: > Bruce, > > I'm not sure why the on-rev app doesn't approve of .lc yet. There is > probably a line in a preferences file somewhere that hasn't been changed > yet. > > Mike > > --- On Sat, 7/30/11, Bruce Pokras wrote: > > From: Bruce Pokras > Subject: Re: Path to start using a stack in On-Rev? > To: "How to use LiveCode" > Date: Saturday, July 30, 2011, 12:22 PM > > Mike, thanks very much. Changing the HTML file's extension from .irev to > .lc worked. The web page is now displaying the result of the function that > is in the stack script of my test stack file. I had not seen anything about > the file extension being critical to functionality on On-Rev. > > However, here is another mystery. Now that my HTML file has the .lc > extension, the On-Rev app does not recognize it! I have to use a text editor > to edit it, or keep it as .irev and change it to .lc at deployment. Do you > know why that should be? > > Regards, > > Bruce > > On Jul 30, 2011, at 11:21 AM, Michael Kann wrote: > > > A little mistake in the last post: > > > > > > > > > > > --- On Sat, 7/30/11, Michael Kann wrote: > > > > From: Michael Kann > > Subject: Re: Path to start using a stack in On-Rev? > > To: "How to use LiveCode" > > Date: Saturday, July 30, 2011, 10:16 AM > > > > Bruce, > > > > Not sure about the path, but there is one other item to consider. If your > web page is good_page.irev then the old 3.5 engine will be used, so you > aren't going to connect to the stack no matter where it is. > > > > Try > > > > file: good_page.lc > > > > within that page you can use either > > > > > put "good stuff here" > > ?> > > > > or > > > > > > > put "good stuff here" > > > > ?> > > > > The file name seems to be the determining factor. > > > > Mike > > > > > > > > > > > > --- On Fri, 7/29/11, Bruce Pokras wrote: > > > > From: Bruce Pokras > > Subject: Path to start using a stack in On-Rev? > > To: "How to use LiveCode" > > Date: Friday, July 29, 2011, 8:41 PM > > > > If the stack and irev HTML page are in the same folder, what is the path > to the stack for the "start using" command? Is it simply: > > > > start using stack "Test_Stack.livecode" > > > > Or is it something more complicated than that? I am testing by putting a > simple function called "respond" in the stack file's stack script, but so > far no response in the .irev page when it is supposed to run the command > "put respond()". > > > > Am I missing something completely? > > > > Regards, > > > > Bruce Pokras > > Blazing Dawn Software > > _______________________________________________ > > use-livecode mailing list > > use-livecode at lists.runrev.com > > Please visit this url to subscribe, unsubscribe and manage your > subscription preferences: > > http://lists.runrev.com/mailman/listinfo/use-livecode > > _______________________________________________ > > use-livecode mailing list > > use-livecode at lists.runrev.com > > Please visit this url to subscribe, unsubscribe and manage your > subscription preferences: > > http://lists.runrev.com/mailman/listinfo/use-livecode > > _______________________________________________ > > use-livecode mailing list > > use-livecode at lists.runrev.com > > Please visit this url to subscribe, unsubscribe and manage your > subscription preferences: > > http://lists.runrev.com/mailman/listinfo/use-livecode > > > _______________________________________________ > use-livecode mailing list > use-livecode at lists.runrev.com > Please visit this url to subscribe, unsubscribe and manage your > subscription preferences: > http://lists.runrev.com/mailman/listinfo/use-livecode > _______________________________________________ > use-livecode mailing list > use-livecode at lists.runrev.com > Please visit this url to subscribe, unsubscribe and manage your > subscription preferences: > http://lists.runrev.com/mailman/listinfo/use-livecode > -- Stephen Barncard San Francisco Ca. USA more about sqb From mikekann at yahoo.com Sat Jul 30 19:55:45 2011 From: mikekann at yahoo.com (Michael Kann) Date: Sat, 30 Jul 2011 16:55:45 -0700 (PDT) Subject: using stacks with on-rev In-Reply-To: <1312060134258-3706864.post@n4.nabble.com> Message-ID: <1312070145.47937.YahooMailClassic@web161619.mail.bf1.yahoo.com> Martin, I couple posts after yours stephen barncard put the cherry on top of the "Re: Path to start using a stack in On-Rev"? thread. Check it out. Good luck. Mike --- On Sat, 7/30/11, Martin Koob wrote: From: Martin Koob Subject: using stacks with on-rev To: use-revolution at lists.runrev.com Date: Saturday, July 30, 2011, 4:08 PM Now that on-rev's engine is upgraded to 4.6.3 I thought I would try the 'start using stack' command. I created a simple stack with a couple of functions in it and I uploaded to the directory where my index.irev page is. I put the following command in my index.irev file start using stack "libstackmain.rev" When this page loads it stops at the above line,? it displays no error message. I also tried using a different extension for the stack file start using stack "libstackmain.livecode" I got the same result. Are stacks supported on on-rev? If so do I have wrong?? What is it relative to? Any other ideas how to get this working? Thanks Martin -- View this message in context: http://runtime-revolution.278305.n4.nabble.com/using-stacks-with-on-rev-tp3706864p3706864.html Sent from the Revolution - User mailing list archive at Nabble.com. _______________________________________________ use-livecode mailing list use-livecode at lists.runrev.com Please visit this url to subscribe, unsubscribe and manage your subscription preferences: http://lists.runrev.com/mailman/listinfo/use-livecode From stephenREVOLUTION2 at barncard.com Sat Jul 30 19:56:15 2011 From: stephenREVOLUTION2 at barncard.com (stephen barncard) Date: Sat, 30 Jul 2011 16:56:15 -0700 Subject: using stacks with on-rev In-Reply-To: <1312060134258-3706864.post@n4.nabble.com> References: <1312060134258-3706864.post@n4.nabble.com> Message-ID: stacks work, only with some limitations. These have been tested by me: Custom properties work. Create and name stacks works. Reading fields sometimes works. Can't write back to them, and no htmltext. Think of stacks for storage, not layout or graphics. I just tried out Mark Smith's excellent id3 mp3 tag library stack, installed with no changes and it works (on my custom livecode installation at Dreamhost) just like the desktop. The scripts had failed before, because they depend on custom properties. Be sure to use the .lc suffix if you want version 4.6.3 running the script , this is how I guess how On-rev determines which version of the server will be utilized. On 30 July 2011 14:08, Martin Koob wrote: > Now that on-rev's engine is upgraded to 4.6.3 I thought I would try the > 'start using stack' command. > > I created a simple stack with a couple of functions in it and I uploaded to > the directory where my index.irev page is. > > I put the following command in my index.irev file > > start using stack "libstackmain.rev" > > When this page loads it stops at the above line, it displays no error > message. > > I also tried using a different extension for the stack file > start using stack "libstackmain.livecode" > I got the same result. > > Are stacks supported on on-rev? > > If so do I have wrong? What is it relative to? > > Any other ideas how to get this working? > > Thanks > > Martin > > > > > > -- > View this message in context: > http://runtime-revolution.278305.n4.nabble.com/using-stacks-with-on-rev-tp3706864p3706864.html > Sent from the Revolution - User mailing list archive at Nabble.com. > > _______________________________________________ > use-livecode mailing list > use-livecode at lists.runrev.com > Please visit this url to subscribe, unsubscribe and manage your > subscription preferences: > http://lists.runrev.com/mailman/listinfo/use-livecode > -- Stephen Barncard San Francisco Ca. USA more about sqb From roger.e.eller at sealedair.com Sat Jul 30 20:16:44 2011 From: roger.e.eller at sealedair.com (Roger Eller) Date: Sat, 30 Jul 2011 20:16:44 -0400 Subject: Getting someone's age from dob and current year In-Reply-To: References: <1312060134258-3706864.post@n4.nabble.com> Message-ID: On Sat, Jul 30, 2011 at 5:20 PM, Admin wrote: > > > Hello all, > > I need to figure out someone's age. > > I have their date > of birth and I have today's date. How do I get the end result. My math > skills SUCK. > > Here's my code (loaded in from the database with a SELECT > statement): > > put item 7 of myLine into dobMonth > put item 8 of myLine > into dobDay > put item 9 of myLine into dobYear > put chartonum("dobYear") > into dobYearNum > put the long system date into field "TodayDate" > > I > figured I would subtract this year from the dobyear and get the age - am > I right? How would I do this if I am? > > Mike > Hey Mike, put this into a new button object. on mouseUp put the date into tToday ask "Enter the date of birth as dd/mm/yy" put it into tBorn convert tToday to seconds convert tBorn to seconds put (tToday - tBorn) into tAge -- in seconds -- now do the math in tAge -- to determine how many years that is answer "The age is" && tAge && "seconds old." & cr & cr \ & "You didn't expect me to do all of the math, now did you? :)" end mouseUp ?Roger From massung at gmail.com Sat Jul 30 20:17:39 2011 From: massung at gmail.com (Jeff Massung) Date: Sat, 30 Jul 2011 18:17:39 -0600 Subject: User Interface Question In-Reply-To: References: Message-ID: I never do mouse cursor changes in controls. I do all of them in the card. on mouseMove px,py if px,py is within the rect of ... then -- cursor A else if px,py is within the rect of ... then -- cursor B else -- put the cursor back to normal end if end mouseMove This makes it *much* easier to control what's shown when. Jeff M. On Sat, Jul 30, 2011 at 1:58 PM, Pete wrote: > Thanks Jeff. I'm trying to implement the cursor change and running into > some weirdness. > > In the script of the datagrid, I have these two handlers: > > *on* mouseEnter > > *lock* cursor > > *set* the cursor to hand > > *end* mouseEnter > > > *on* mouseLeave > > *unlock* cursor > > *end* mouseLeave > > > This changes the cursor as expected and most of the time, it returns to the > correct cursor when I move the mouse away from the datagrid. However, if I > move the cursor above the datagrid, the cursor stays as the hand and does > not return to any other image no matter where I move the mouse to. > > > This is the first time I've attempted to control the cursor so no doubt I'm > doing something wrong! > > > Pete > Molly's Revenge > > > > > On Sat, Jul 30, 2011 at 11:48 AM, Jeff Massung wrote: > > > For dragging, I'd go with "handles" on the left side of the datagrid that > > have hand icon when you hover over them. They've become pretty ubiquitous > > on > > webpages. For an example of what I mean: > > > > http://jqueryui.com/demos/sortable/ > > > > As for a context menu, you be okay with just a tool-tip. People generally > > try and right-click a lot for things they expect to be there. Otherwise, > > again, go with something common to show that actions are available. For > > example, and disclosure triangle on the far right of a UI widget is > > typically clickable and will bring up a menu. Left-click will bring up > your > > context menu, and r-click anywhere will bring it up as well. > > > > Jeff M. > > > > On Sat, Jul 30, 2011 at 12:42 PM, Pete wrote: > > > > > Wondering if anyone has any thoughts on a couple of user interface > > > questions. > > > > > > I'm providing a popup contextual menu when the user right-clicks on a > > > datagrid. I'm also providing the ability to re-order rows in a > datagrid > > by > > > dragging and dropping them. The common thread between these two > > functions > > > is that there is no on-screen indication that they exist. Popup menus > > are > > > invisible until they are invoked and drag and drop is equally invisible > > > until someone initiates it. > > > > > > Are there any common methods for indicating to users that features like > > > this > > > are available to them? > > > > > > Pete > > > Molly's Revenge > > > _______________________________________________ > > > use-livecode mailing list > > > use-livecode at lists.runrev.com > > > Please visit this url to subscribe, unsubscribe and manage your > > > subscription preferences: > > > http://lists.runrev.com/mailman/listinfo/use-livecode > > > > > _______________________________________________ > > use-livecode mailing list > > use-livecode at lists.runrev.com > > Please visit this url to subscribe, unsubscribe and manage your > > subscription preferences: > > http://lists.runrev.com/mailman/listinfo/use-livecode > > > > > _______________________________________________ > use-livecode mailing list > use-livecode at lists.runrev.com > Please visit this url to subscribe, unsubscribe and manage your > subscription preferences: > http://lists.runrev.com/mailman/listinfo/use-livecode > From roger.e.eller at sealedair.com Sat Jul 30 20:29:32 2011 From: roger.e.eller at sealedair.com (Roger Eller) Date: Sat, 30 Jul 2011 20:29:32 -0400 Subject: Getting someone's age from dob and current year In-Reply-To: References: <1312060134258-3706864.post@n4.nabble.com> Message-ID: On Sat, Jul 30, 2011 at 5:20 PM, Admin wrote: > > I figured I would subtract this year from the dobyear and get the age - am > I right? How would I do this if I am? > > Mike > I gotta start reading the whole message before I reply... If you already have two 4-digit year values in variables, you can simply put their difference into another variable (or container). put (largYear - smallYear) into tAge You can also say: subtract smallYear from largeYear answer largeYear -- which now becomes the age ?Roger From admin at mfelkerco.com Sat Jul 30 20:30:15 2011 From: admin at mfelkerco.com (Admin) Date: Sat, 30 Jul 2011 20:30:15 -0400 Subject: carrying a global variable from stack to sub stack In-Reply-To: <016101cc4f00$8b170e00$a1452a00$@com> References: <1312060134258-3706864.post@n4.nabble.com> <8632aaae54a6aecd62fdd741c3a1578a@mfelkerco.com> <016101cc4f00$8b170e00$a1452a00$@com> Message-ID: Thank you. The only place I did not have it was in my stack's script with the openstack handler. Duh! Mike On Sat, 30 Jul 2011 17:34:58 -0400, Slava Paperno wrote: >> How do you carry a global variable from one stack to a substack? > > If you set a global variable in the main stack: > > on preOpenStack > global gMyName > > put "Mike" into gMyName > end preOpenStack > > then in any card in any substack you can retrieve the value like so: > > on openCard > global gMyName > answer gMyName --you will see "Mike" > end open Card > > In other words, nothing needs to be done to "carry" a global variable from > one stack to another. All you have to do is declare it with the word > "global" in front. > > S. > >> I have this set of commands on every page of every stack in my project: on opencard --Create a global variable for the db global dbID global curUserID global UN put numtochar(39) into SQ I then proceed to login to the database (works fine normally) and then access whatever data I need. I just created a substack called CV with three pages - pgCV1, pgCV2 and pgCV3 I need the login credentials of the user to carry over so the fields on these pages can be populated with the existing data on the database. The important variable is dbID and curUserID What do I need to do, other than the global command, to make this happen? Mike > > _______________________________________________ > use-livecode mailing list > use-livecode at lists.runrev.com [1] > Please visit this url to subscribe, unsubscribe and manage your subscription preferences: > http://lists.runrev.com/mailman/listinfo/use-livecode [2] Links: ------ [1] mailto:use-livecode at lists.runrev.com [2] http://lists.runrev.com/mailman/listinfo/use-livecode From admin at mfelkerco.com Sat Jul 30 20:31:31 2011 From: admin at mfelkerco.com (Admin) Date: Sat, 30 Jul 2011 20:31:31 -0400 Subject: Getting someone's age from dob and current year In-Reply-To: References: <1312060134258-3706864.post@n4.nabble.com> Message-ID: <6b0e0364f1463b9075663dc224ddd737@mfelkerco.com> Thank you. I am sure that will do it! Mike On Sat, 30 Jul 2011 20:16:44 -0400, Roger Eller wrote: > On Sat, Jul 30, 2011 at 5:20 PM, Admin wrote: > >> Hello all, I need to figure out someone's age. I have their date of birth and I have today's date. How do I get the end result. My math skills SUCK. Here's my code (loaded in from the database with a SELECT statement): put item 7 of myLine into dobMonth put item 8 of myLine into dobDay put item 9 of myLine into dobYear put chartonum("dobYear") into dobYearNum put the long system date into field "TodayDate" I figured I would subtract this year from the dobyear and get the age - am I right? How would I do this if I am? Mike > > Hey Mike, put this into a new button object. > > on mouseUp > put the date into tToday > ask "Enter the date of birth as dd/mm/yy" > put it into tBorn > convert tToday to seconds > convert tBorn to seconds > put (tToday - tBorn) into tAge -- in seconds > -- now do the math in tAge > -- to determine how many years that is > answer "The age is" && tAge && "seconds old." & cr & cr > & "You didn't expect me to do all of the math, now did you? :)" > end mouseUp > > ?Roger > _______________________________________________ > use-livecode mailing list > use-livecode at lists.runrev.com [1] > Please visit this url to subscribe, unsubscribe and manage your subscription preferences: > http://lists.runrev.com/mailman/listinfo/use-livecode [2] Links: ------ [1] mailto:use-livecode at lists.runrev.com [2] http://lists.runrev.com/mailman/listinfo/use-livecode From admin at mfelkerco.com Sat Jul 30 20:34:56 2011 From: admin at mfelkerco.com (Admin) Date: Sat, 30 Jul 2011 20:34:56 -0400 Subject: running an exe from a web server In-Reply-To: References: <1312060134258-3706864.post@n4.nabble.com> Message-ID: Hi all, How to I run a .exe file or a mac executeable from a button - the file of which is on a web server (http://www.campstaffusa.com/newdb/upcc.exe)? I want to run a self-extracting zip file that opens up a live code program to allow the user to upload their picture to the database, since it will not allow you to do that in the revlet online player. I have the program all zipped up and it is now a self-extracting zip that will run the .exe file upon automatically extracting. All I need to do is have the person click the button and run the .exe file. I wish you could do that with a live code app - compress everything into one file that can be run - instead of needing the externals folder and any other .dll's. Mike From admin at mfelkerco.com Sat Jul 30 20:38:40 2011 From: admin at mfelkerco.com (Admin) Date: Sat, 30 Jul 2011 20:38:40 -0400 Subject: Getting someone's age from dob and current year In-Reply-To: <6b0e0364f1463b9075663dc224ddd737@mfelkerco.com> References: <1312060134258-3706864.post@n4.nabble.com> <6b0e0364f1463b9075663dc224ddd737@mfelkerco.com> Message-ID: <373366e9c8ad62faf8f60d437cb060dc@mfelkerco.com> Hi all - another one: I have a path stored in a MySQL database to an image file. I have a page with a image control I know how to SELECT the imagepath info and bring it into a variable, but how do I then load that image into the image control (with proper proportions)? I tried the obvious ways and nothing worked for me. The image control is named imgCandidatePic (just a small 140x140 square). A sample path would be http://www.server.com/userfiles/bear_harry/mypic.jpg for example I need to get that picture into the image control when the card loads (so in the card script using opencard). Thanks for your help everyone. Mike From mikekann at yahoo.com Sat Jul 30 21:04:28 2011 From: mikekann at yahoo.com (Michael Kann) Date: Sat, 30 Jul 2011 18:04:28 -0700 (PDT) Subject: Getting someone's age from dob and current year In-Reply-To: <373366e9c8ad62faf8f60d437cb060dc@mfelkerco.com> Message-ID: <1312074268.60741.YahooMailClassic@web161617.mail.bf1.yahoo.com> The following works: put "http://fake.com/fake.jpg" into k set the filename of img 1 to k To get the correct dimensions I guess you would have to find the dimensions of the online jpg, then make you img the same dimensions. Just a thought. Mike --- On Sat, 7/30/11, Admin wrote: From: Admin Subject: Re: Getting someone's age from dob and current year To: "How to use LiveCode" Date: Saturday, July 30, 2011, 7:38 PM ? Hi all - another one: I have a path stored in a MySQL database to an image file. I have a page with a image control I know how to SELECT the imagepath info and bring it into a variable, but how do I then load that image into the image control (with proper proportions)? I tried the obvious ways and nothing worked for me. The image control is named imgCandidatePic (just a small 140x140 square). A sample path would be http://www.server.com/userfiles/bear_harry/mypic.jpg for example I need to get that picture into the image control when the card loads (so in the card script using opencard). Thanks for your help everyone. Mike ? _______________________________________________ use-livecode mailing list use-livecode at lists.runrev.com Please visit this url to subscribe, unsubscribe and manage your subscription preferences: http://lists.runrev.com/mailman/listinfo/use-livecode From mkoob at rogers.com Sat Jul 30 21:18:12 2011 From: mkoob at rogers.com (Martin Koob) Date: Sat, 30 Jul 2011 18:18:12 -0700 (PDT) Subject: using stacks with on-rev In-Reply-To: <1312070145.47937.YahooMailClassic@web161619.mail.bf1.yahoo.com> References: <1312060134258-3706864.post@n4.nabble.com> <1312070145.47937.YahooMailClassic@web161619.mail.bf1.yahoo.com> Message-ID: <1312075092327-3707069.post@n4.nabble.com> Thanks Mike and Stephen So the answer is that the file has to have the .lc extension for the 4.6.3 engine to process it. Once I had the correct extension the line start using stack "libstackmain.rev" worked and I was able to call a function in the stack. That is interesting that .irev files are processed with the 3.5 engine .lc files are processed with the 4.6.3 engine I guess that keeps old sites from breaking. I was afraid with the transition to 4.6.3 that it may break an on-rev website that I have. It has been working fine though so I guess it is still running on the 3.5 engine. Martin -- View this message in context: http://runtime-revolution.278305.n4.nabble.com/using-stacks-with-on-rev-tp3706864p3707069.html Sent from the Revolution - User mailing list archive at Nabble.com. From mkoob at rogers.com Sat Jul 30 21:26:56 2011 From: mkoob at rogers.com (Martin Koob) Date: Sat, 30 Jul 2011 18:26:56 -0700 (PDT) Subject: Path to start using a stack in On-Rev? In-Reply-To: References: <4E188107.7070404@hyperactivesw.com> <70265F34-9EA0-4FF5-9E91-6F6DC68E5AAB@comcast.net> <1312039003.39634.YahooMailClassic@web161612.mail.bf1.yahoo.com> <1312039268.48913.YahooMailClassic@web161606.mail.bf1.yahoo.com> <85CC2950-EBF0-4083-9DE2-8C24F27D9033@comcast.net> <1312050859.62628.YahooMailClassic@web161610.mail.bf1.yahoo.com> Message-ID: <1312075616876-3707074.post@n4.nabble.com> Yes I found the setting to add lc as an irev type which allows me to edit it in the on-rev client but when I select a file with the .lc extension the 'View online' and 'Debug' buttons are disabled. It seems to treat the file as other plain text files except for the colourization. Is debugging supported for 4.6.3? Why would 'View online' not work? Martin -- View this message in context: http://runtime-revolution.278305.n4.nabble.com/Android-Why-so-quiet-Who-s-developing-Android-apps-tp3656291p3707074.html Sent from the Revolution - User mailing list archive at Nabble.com. From roger.e.eller at sealedair.com Sat Jul 30 21:54:11 2011 From: roger.e.eller at sealedair.com (Roger Eller) Date: Sat, 30 Jul 2011 21:54:11 -0400 Subject: Getting someone's age from dob and current year In-Reply-To: <373366e9c8ad62faf8f60d437cb060dc@mfelkerco.com> References: <1312060134258-3706864.post@n4.nabble.com> <6b0e0364f1463b9075663dc224ddd737@mfelkerco.com> <373366e9c8ad62faf8f60d437cb060dc@mfelkerco.com> Message-ID: On Sat, Jul 30, 2011 at 8:38 PM, Admin wrote: > > > Hi all - another one: > > I have a path stored in a MySQL database to > an image file. > > I have a page with a image control > > I know how to > SELECT the imagepath info and bring it into a variable, but how do I > then load that image into the image control (with proper proportions)? I > tried the obvious ways and > > nothing worked for me. > > The image control > is named imgCandidatePic (just a small 140x140 square). > > A sample path > would be http://www.server.com/userfiles/bear_harry/mypic.jpg for > example > > I need to get that picture into the image control when the > card loads (so in the card script using opencard). > > Thanks for your > help everyone. > > Mike > As long as your image size is not locked, the image container will adjust to the proportions of the original image. on mouseUp create img "bear_harry" set the filename of img "bear_harry" to " http://www.skcastle.com/wp-content/uploads/2007/08/bearharry.jpg" end mouseUp From stephenREVOLUTION2 at barncard.com Sat Jul 30 21:57:46 2011 From: stephenREVOLUTION2 at barncard.com (stephen barncard) Date: Sat, 30 Jul 2011 18:57:46 -0700 Subject: using stacks with on-rev In-Reply-To: <1312075092327-3707069.post@n4.nabble.com> References: <1312060134258-3706864.post@n4.nabble.com> <1312070145.47937.YahooMailClassic@web161619.mail.bf1.yahoo.com> <1312075092327-3707069.post@n4.nabble.com> Message-ID: Those guys in the engine room are pretty darn clever. I wondered how they were going to handle two versions. sqb On 30 July 2011 18:18, Martin Koob wrote: > Thanks Mike and Stephen > > So the answer is that the file has to have the .lc extension for the 4.6.3 > engine to process it. > > Once I had the correct extension the line > start using stack "libstackmain.rev" > worked and I was able to call a function in the stack. > > That is interesting that > .irev files are processed with the 3.5 engine > .lc files are processed with the 4.6.3 engine > > I guess that keeps old sites from breaking. I was afraid with the > transition to 4.6.3 that it may break an on-rev website that I have. It > has been working fine though so I guess it is still running on the 3.5 > engine. > > Martin > > > > > > -- > View this message in context: > http://runtime-revolution.278305.n4.nabble.com/using-stacks-with-on-rev-tp3706864p3707069.html > Sent from the Revolution - User mailing list archive at Nabble.com. > > _______________________________________________ > use-livecode mailing list > use-livecode at lists.runrev.com > Please visit this url to subscribe, unsubscribe and manage your > subscription preferences: > http://lists.runrev.com/mailman/listinfo/use-livecode > -- Stephen Barncard San Francisco Ca. USA more about sqb From stephenREVOLUTION2 at barncard.com Sat Jul 30 21:59:26 2011 From: stephenREVOLUTION2 at barncard.com (stephen barncard) Date: Sat, 30 Jul 2011 18:59:26 -0700 Subject: Path to start using a stack in On-Rev? In-Reply-To: <1312075616876-3707074.post@n4.nabble.com> References: <4E188107.7070404@hyperactivesw.com> <70265F34-9EA0-4FF5-9E91-6F6DC68E5AAB@comcast.net> <1312039003.39634.YahooMailClassic@web161612.mail.bf1.yahoo.com> <1312039268.48913.YahooMailClassic@web161606.mail.bf1.yahoo.com> <85CC2950-EBF0-4083-9DE2-8C24F27D9033@comcast.net> <1312050859.62628.YahooMailClassic@web161610.mail.bf1.yahoo.com> <1312075616876-3707074.post@n4.nabble.com> Message-ID: No the software hasn't been updated since the branding change. I guess the ability to change extensions was just to allow the viewing of various other text files. On 30 July 2011 18:26, Martin Koob wrote: > Yes I found the setting to add lc as an irev type which allows me to edit > it > in the on-rev client but when I select a file with the .lc extension the > 'View online' and 'Debug' buttons are disabled. It seems to treat the file > as other plain text files except for the colourization. > > Is debugging supported for 4.6.3? > Why would 'View online' not work? > > > Martin > > -- > View this message in context: > http://runtime-revolution.278305.n4.nabble.com/Android-Why-so-quiet-Who-s-developing-Android-apps-tp3656291p3707074.html > Sent from the Revolution - User mailing list archive at Nabble.com. > > _______________________________________________ > use-livecode mailing list > use-livecode at lists.runrev.com > Please visit this url to subscribe, unsubscribe and manage your > subscription preferences: > http://lists.runrev.com/mailman/listinfo/use-livecode > -- Stephen Barncard San Francisco Ca. USA more about sqb From slava at lexiconbridge.com Sat Jul 30 22:54:25 2011 From: slava at lexiconbridge.com (Slava Paperno) Date: Sat, 30 Jul 2011 22:54:25 -0400 Subject: lack of Mac OS X intuition: system menu Message-ID: <001201cc4f2d$2c2b7570$84826050$@lexiconbridge.com> I've been learning LC in Windows, and have now switched to Mac OS X, where everything is just so intuitive and easy... can someone please tell me how to remove my application menu from the system menu bar and restore the LC menu so I can do some work? My intuition fails on this point. I know I've seen this mentioned somewhere but can't find it now. Thanks! Slava From slava at lexiconbridge.com Sat Jul 30 23:07:39 2011 From: slava at lexiconbridge.com (Slava Paperno) Date: Sat, 30 Jul 2011 23:07:39 -0400 Subject: revBrowser and Unicode on Mac OS X Message-ID: <001501cc4f2f$04fb0a40$0ef11ec0$@lexiconbridge.com> Short version: revBrowser does not display Unicode characters in Mac OS X (PPC/Tiger and Intel/Snow Leopard). LC 4.6.2 and 4.6.3. Long version: I finished writing and testing my Russian dictionary application in Windows and started testing it in Mac OS X. My first surprise is that revBrowser does not display Cyrillic (Unicode) characters--neither on a PPC (Tiger) nor on an Intel (Snow Leopard) machine. The English portion is rendered fine, including styles and everything, but Cyrillic is rendered as if every double-byte character is interpreted as a sequence of two single bytes. The source (the HTML text that is assigned to revBrowser as its htmltext property) comes from a local html file or from a database record. The result is the same. When the same text is assigned to the unicodeText property of a field, everything looks fine. The usual character set meta tag in the html is present and indicates UTF-8. Works fine in Windows. Is there something about displaying UTF-8 in revBrowser in Mac OS X that I need to know? The browser that is used by LC is probably part of the Mac setup, right? All browsers on those two Macs display the same Web page fine. I'll be grateful for any solution. I hope it's something I'm doing wrong--or not doing. Thanks, Slava From warren at warrensweb.us Sun Jul 31 00:03:40 2011 From: warren at warrensweb.us (Warren Samples) Date: Sat, 30 Jul 2011 23:03:40 -0500 Subject: Getting someone's age from dob and current year In-Reply-To: References: <1312060134258-3706864.post@n4.nabble.com> Message-ID: <201107302303.40864.warren@warrensweb.us> On Saturday, July 30, 2011 07:16:44 PM Roger Eller wrote: > convert tToday to seconds > convert tBorn to seconds It's been talked about before, but this calls for a reminder: attempting to convert a date prior to Jan 1, 1970 to "seconds" returns "invalid date" instead of a number in Windows. Don't do this if you are going to need this to work in Windows! Under Windows, you need to subtract the birthyear from the current year and determine if the current month/day is later than the birth month/day and subtract one year if necessary. Conveniently, this methods does work on all platforms. Best, Warren From slava at lexiconbridge.com Sun Jul 31 00:16:48 2011 From: slava at lexiconbridge.com (Slava Paperno) Date: Sun, 31 Jul 2011 00:16:48 -0400 Subject: revBrowserCallScript() not working in Mac OS X Message-ID: <000001cc4f38$addce6c0$0996b440$@com> My application (LC 4.6.3) does this: revBrowserSet(sBrowserID, "htmltext", tMyHTML) tMyHTML is a fully-formed Web page that includes this function: function SetFontSize(FSizeStr) { document.body.style.fontSize = FSizeStr; } A button on the same card as the revBrowser instance does this: revBrowserCallScript(sBrowserID, "SetFontSize", 16 & "px") This works fine in Windows, but now that I started testing in Mac OS X (10.5.8. on PPC and 10.6.8 on Intel), the line above returns "execution error at line n/a (External handler: exception) near *** [WebUndefined cStringUsingEncoding:]: unrecognized selector sent to instance ...." This is my second major revBrowser surprise since I started testing in Mac OS X. Am I on the wrong track? Does revBrowser work the same in Windows and Mac OS X? I realize the underlying browser comes from the OS. Do I need to do anything differently for revBrowser in Mac OS? Thanks, Slava From coiin at verizon.net Sun Jul 31 00:41:15 2011 From: coiin at verizon.net (Colin Holgate) Date: Sun, 31 Jul 2011 00:41:15 -0400 Subject: lack of Mac OS X intuition: system menu In-Reply-To: <001201cc4f2d$2c2b7570$84826050$@lexiconbridge.com> References: <001201cc4f2d$2c2b7570$84826050$@lexiconbridge.com> Message-ID: <6B257A2D-77E5-4B90-ADF1-97A61BF7F3EF@verizon.net> What is the Application menu? What things are in the LC menu. On Jul 30, 2011, at 10:54 PM, Slava Paperno wrote: > can someone please tell me how > to remove my application menu from the system menu bar and restore the LC > menu so I can do some work? From slava at lexiconbridge.com Sun Jul 31 00:54:34 2011 From: slava at lexiconbridge.com (Slava Paperno) Date: Sun, 31 Jul 2011 00:54:34 -0400 Subject: lack of Mac OS X intuition: system menu In-Reply-To: <6B257A2D-77E5-4B90-ADF1-97A61BF7F3EF@verizon.net> References: <001201cc4f2d$2c2b7570$84826050$@lexiconbridge.com> <6B257A2D-77E5-4B90-ADF1-97A61BF7F3EF@verizon.net> Message-ID: <000101cc4f3d$f4c65620$de530260$@com> My stack has a menu bar built with the Menu Builder in the Tools menu. It is essentially a group of buttons use as a menu bar. When I open the stack in the IDE on Mac OS X, that menu replaces the standard LC IDE menu. No matter which mode I am in (Browse or Run), I cannot see the LC IDE menu in the Mac OS system menu bar. So nine of the IDE commands are available. I know this is a trivial situation in Mac OS, and there is a secret handshake that returns the IDE menu where it belongs. I even remember reading about this somewhere. But I can't find that now. Thanks, Colin, Slava > -----Original Message----- > From: use-livecode-bounces at lists.runrev.com [mailto:use-livecode- > bounces at lists.runrev.com] On Behalf Of Colin Holgate > Sent: Sunday, July 31, 2011 12:41 AM > To: How to use LiveCode > Subject: Re: lack of Mac OS X intuition: system menu > > What is the Application menu? What things are in the LC menu. > > > On Jul 30, 2011, at 10:54 PM, Slava Paperno wrote: > > > can someone please tell me how > > to remove my application menu from the system menu bar and restore > the LC > > menu so I can do some work? > From mazzapaolo at libero.it Sun Jul 31 02:10:40 2011 From: mazzapaolo at libero.it (paolo mazza) Date: Sun, 31 Jul 2011 08:10:40 +0200 Subject: lack of Mac OS X intuition: system menu In-Reply-To: <000101cc4f3d$f4c65620$de530260$@com> References: <001201cc4f2d$2c2b7570$84826050$@lexiconbridge.com> <6B257A2D-77E5-4B90-ADF1-97A61BF7F3EF@verizon.net> <000101cc4f3d$f4c65620$de530260$@com> Message-ID: Slava, using Mac OS, usually I have the application menu in the run mode and the LC IDE menu in the edit mode (unless I set the "Set as stack Menu Bar" option to false in the Menu Builder). If I select the "Preview in Menu Bar" button in the Menu Builder panel, for some reasons, I have the new application menu both in the run and in the edit mode. However, if I either close the Menu Bar panel, or I quit and restart the IDE, I get the IDE menu bar only when I set the edit mode. You could have a Manu Bar panel open somewhere. What about restarting the LC IDE ? All the best Paolo Mazza >> -----Original Message----- >> From: use-livecode-bounces at lists.runrev.com [mailto:use-livecode- >> bounces at lists.runrev.com] On Behalf Of Colin Holgate >> Sent: Sunday, July 31, 2011 12:41 AM >> To: How to use LiveCode >> Subject: Re: lack of Mac OS X intuition: system menu >> >> What is the Application menu? What things are in the LC menu. >> >> >> On Jul 30, 2011, at 10:54 PM, Slava Paperno wrote: >> >> > can someone please tell me how >> > to remove my application menu from the system menu bar and restore >> the LC >> > menu so I can do some work? >> > > > > > _______________________________________________ > use-livecode mailing list > use-livecode at lists.runrev.com > Please visit this url to subscribe, unsubscribe and manage your subscription preferences: > http://lists.runrev.com/mailman/listinfo/use-livecode > From gandalf at doctorTimothyMiller.com Sun Jul 31 05:22:31 2011 From: gandalf at doctorTimothyMiller.com (Timothy Miller) Date: Sun, 31 Jul 2011 02:22:31 -0700 Subject: Where does survive the inventive user ? In-Reply-To: <44857038-F42F-4374-8DA7-F6BCDF16629C@doctorTimothyMiller.com> References: <8CE1A70D9625EA6-2550-214DE@web-mmc-d04.sysops.aol.com> <44857038-F42F-4374-8DA7-F6BCDF16629C@doctorTimothyMiller.com> Message-ID: Meanwhile, a comprehensive LC tutorial, extremely user friendly, written in LC, starting at the most elementary level... Has it been envisioned? Tim From m.schonewille at economy-x-talk.com Sun Jul 31 05:43:08 2011 From: m.schonewille at economy-x-talk.com (Mark Schonewille) Date: Sun, 31 Jul 2011 11:43:08 +0200 Subject: lack of Mac OS X intuition: system menu In-Reply-To: <001201cc4f2d$2c2b7570$84826050$@lexiconbridge.com> References: <001201cc4f2d$2c2b7570$84826050$@lexiconbridge.com> Message-ID: <6E5B44D2-119D-4675-BC4F-5C66C61C296B@economy-x-talk.com> Hi Slava, This can indeed be a problem once in a while. If you have closed all palettes and windows and your own stack is the only stack visible, then you get may stuck. Just don't close the tools palette unless you're absolutely sure. Whenever you click in an IDE window or choose the pointer tool, you should see the IDE menu bar, unless the Preview In Menubar option is set to true. -- Best regards, Mark Schonewille Economy-x-Talk Consulting and Software Engineering Homepage: http://economy-x-talk.com Twitter: http://twitter.com/xtalkprogrammer KvK: 50277553 What does that error mean? Buy LiveCodeErrors for iPhone now http://qery.us/v4 A must-have for LiveCode programmers. On 31 jul 2011, at 04:54, Slava Paperno wrote: > I've been learning LC in Windows, and have now switched to Mac OS X, where > everything is just so intuitive and easy... can someone please tell me how > to remove my application menu from the system menu bar and restore the LC > menu so I can do some work? My intuition fails on this point. I know I've > seen this mentioned somewhere but can't find it now. Thanks! > > Slava From mikekann at yahoo.com Sun Jul 31 10:40:58 2011 From: mikekann at yahoo.com (Michael Kann) Date: Sun, 31 Jul 2011 07:40:58 -0700 (PDT) Subject: Blacklisted In-Reply-To: Message-ID: <1312123258.35835.YahooMailClassic@web161616.mail.bf1.yahoo.com> I sent an email request to support at runrev.com from my yahoo account and got the following failure notice: --------------------------------- Sorry, we were unable to deliver your message to the following address. : Remote host said: 550 5.7.1 Your IP 98.139.212.175 is blacklisted. Click delist.emailfiltering.com to delist [RCPT_TO] --------------------------------- Do you think this is legit or is it a scam to get you to click on the delist URL? More importantly, did my email get through? --------------------------------- Here's the question I was going to ask. I've installed LiveCode 4.6.3 on both XP and Vista and can't get past the dialog box where it wants you to associate the file endings .rev and .livecode with the program. Anywhere I click when I get that dialog box makes the program disappear. What I really want is to see the new dictionary so I can start playing around with the liveCode Server engine. Has anyone liberated the new dictionary so we can read it as a text file? Any help is appreciated as always, Mike From pmbrig at gmail.com Sun Jul 31 11:27:41 2011 From: pmbrig at gmail.com (Peter Brigham MD) Date: Sun, 31 Jul 2011 11:27:41 -0400 Subject: User Interface Question In-Reply-To: References: Message-ID: <3D40656D-2CDE-43AE-A7E2-A7FFF0BF63DA@gmail.com> And it's much better not to lock the cursor and then change it. Locked cursors have a way of sticking if the handler to unlock the cursor somehow misses. instead, don't lock anything, just set the defaultcursor to hand, then set it to empty when done. I usually have this in my stack script: on mousemove put the target into tTarg if the name of the target contains "button" then if the tool contains "browse" then set the defaultcursor to hand end if else set the defaultcursor to empty end if pass mouseMove end mousemove on mouseLeave set the defaultcursor to empty pass mouseLeave end mouseLeave -- Peter Peter M. Brigham pmbrig at gmail.com http://home.comcast.net/~pmbrig On Jul 30, 2011, at 8:17 PM, Jeff Massung wrote: > I never do mouse cursor changes in controls. I do all of them in the card. > > on mouseMove px,py > if px,py is within the rect of ... then > -- cursor A > else if px,py is within the rect of ... then > -- cursor B > else > -- put the cursor back to normal > end if > end mouseMove > > This makes it *much* easier to control what's shown when. > > Jeff M. > > On Sat, Jul 30, 2011 at 1:58 PM, Pete wrote: > >> Thanks Jeff. I'm trying to implement the cursor change and running into >> some weirdness. >> >> In the script of the datagrid, I have these two handlers: >> >> *on* mouseEnter >> >> *lock* cursor >> >> *set* the cursor to hand >> >> *end* mouseEnter >> >> >> *on* mouseLeave >> >> *unlock* cursor >> >> *end* mouseLeave >> >> >> This changes the cursor as expected and most of the time, it returns to the >> correct cursor when I move the mouse away from the datagrid. However, if I >> move the cursor above the datagrid, the cursor stays as the hand and does >> not return to any other image no matter where I move the mouse to. >> >> >> This is the first time I've attempted to control the cursor so no doubt I'm >> doing something wrong! >> >> >> Pete >> Molly's Revenge >> >> >> >> >> On Sat, Jul 30, 2011 at 11:48 AM, Jeff Massung wrote: >> >>> For dragging, I'd go with "handles" on the left side of the datagrid that >>> have hand icon when you hover over them. They've become pretty ubiquitous >>> on >>> webpages. For an example of what I mean: >>> >>> http://jqueryui.com/demos/sortable/ >>> >>> As for a context menu, you be okay with just a tool-tip. People generally >>> try and right-click a lot for things they expect to be there. Otherwise, >>> again, go with something common to show that actions are available. For >>> example, and disclosure triangle on the far right of a UI widget is >>> typically clickable and will bring up a menu. Left-click will bring up >> your >>> context menu, and r-click anywhere will bring it up as well. >>> >>> Jeff M. >>> >>> On Sat, Jul 30, 2011 at 12:42 PM, Pete wrote: >>> >>>> Wondering if anyone has any thoughts on a couple of user interface >>>> questions. >>>> >>>> I'm providing a popup contextual menu when the user right-clicks on a >>>> datagrid. I'm also providing the ability to re-order rows in a >> datagrid >>> by >>>> dragging and dropping them. The common thread between these two >>> functions >>>> is that there is no on-screen indication that they exist. Popup menus >>> are >>>> invisible until they are invoked and drag and drop is equally invisible >>>> until someone initiates it. >>>> >>>> Are there any common methods for indicating to users that features like >>>> this >>>> are available to them? >>>> >>>> Pete >>>> Molly's Revenge >>>> _______________________________________________ >>>> use-livecode mailing list >>>> use-livecode at lists.runrev.com >>>> Please visit this url to subscribe, unsubscribe and manage your >>>> subscription preferences: >>>> http://lists.runrev.com/mailman/listinfo/use-livecode >>>> >>> _______________________________________________ >>> use-livecode mailing list >>> use-livecode at lists.runrev.com >>> Please visit this url to subscribe, unsubscribe and manage your >>> subscription preferences: >>> http://lists.runrev.com/mailman/listinfo/use-livecode >>> >>> >> _______________________________________________ >> use-livecode mailing list >> use-livecode at lists.runrev.com >> Please visit this url to subscribe, unsubscribe and manage your >> subscription preferences: >> http://lists.runrev.com/mailman/listinfo/use-livecode >> > _______________________________________________ > use-livecode mailing list > use-livecode at lists.runrev.com > Please visit this url to subscribe, unsubscribe and manage your subscription preferences: > http://lists.runrev.com/mailman/listinfo/use-livecode From pmbrig at gmail.com Sun Jul 31 11:32:02 2011 From: pmbrig at gmail.com (Peter Brigham MD) Date: Sun, 31 Jul 2011 11:32:02 -0400 Subject: Getting someone's age from dob and current year In-Reply-To: <201107302303.40864.warren@warrensweb.us> References: <1312060134258-3706864.post@n4.nabble.com> <201107302303.40864.warren@warrensweb.us> Message-ID: <6AFFE4A9-7AC1-4A9E-8D1B-8B82041A00C3@gmail.com> On Jul 31, 2011, at 12:03 AM, Warren Samples wrote: > On Saturday, July 30, 2011 07:16:44 PM Roger Eller wrote: >> convert tToday to seconds >> convert tBorn to seconds > > > It's been talked about before, but this calls for a reminder: attempting to convert a date prior to Jan 1, > 1970 to "seconds" returns "invalid date" instead of a number in Windows. Don't do this if you are going to > need this to work in Windows! Under Windows, you need to subtract the birthyear from the current year and > determine if the current month/day is later than the birth month/day and subtract one year if necessary. > Conveniently, this methods does work on all platforms. function doAge tDOB -- assumes format of m/d/yy set the itemdelimiter to "/" put item 3 of tDOB into DOByr put item 2 of tDOB into DOBmo put item 1 of tDOB into DOBdy put item 3 of the short date into nowYr put item 2 of the short date into nowMo put item 1 of the short date into nowDy if DOByr > nowYr then -- prev century -- assumes no one with age > 100 put "19" before DOByr else put "20" before DOByr end if put "20" before nowYr put nowYr - DOByr into tAge if nowMo > DOBmo then return tAge if nowMo < DOBmo then return tAge -1 if nowDy < DOBdy then return tAge -1 return tAge end doAge -- Peter Peter M. Brigham pmbrig at gmail.com http://home.comcast.net/~pmbrig From rjearp at hotmail.com Sun Jul 31 11:37:59 2011 From: rjearp at hotmail.com (Bob Earp) Date: Sun, 31 Jul 2011 08:37:59 -0700 Subject: LC Summer Academy Message-ID: Thanks for this Mark, but is the actual material/lessons still available for viewing and if so where? I've been traveling a lot this year and have missed such valuable resources. best, Bob... Bob Earp White Rock, British Columbia. > Message: 6 > Date: Sat, 30 Jul 2011 10:56:55 -0500 > From: Mark Smith > To: How to use LiveCode > Subject: RE: LC Summer Academy > Message-ID: > <3F95480D5F0DA64EA0785711087864DEAA47E704DC at exch01.mchpe.cpe.umanitoba.ca> > > Content-Type: text/plain; charset="us-ascii" > > The Summer Academy was incredibly useful (and hats off to Elanor for being such a great presenter). A ton of information was presented in the 7 weeks, so much so that I had trouble keeping track of it all and definitely wanted a way to be able to go back and review it later. So I created the Summer Academy '11 Index... over 600 topical entries covering all of the material presented, including references to many livecode commands and tutorial links, and all keyword searchable. I've posted it to the RevOnline user samples (I think you can find it by searching for Index or Academy). > > Here are a couple of tips to help get you started: if you want to search for everything presented in a given week type Wk followed by the week number. If you want to know just what was presented in the Webinar portion, type Wk?w and replace ? with the week number you are interested in. This gives a kind of minute by minute overview of the webinar presentation. Interested in knowing what links where discussed? search for http. > > For those of you who attended, and for those of you interested in knowing what was covered I hope this tool is useful. > > Cheers, > > -- Mark From slava at lexiconbridge.com Sun Jul 31 12:14:55 2011 From: slava at lexiconbridge.com (Slava Paperno) Date: Sun, 31 Jul 2011 12:14:55 -0400 Subject: lack of Mac OS X intuition: system menu In-Reply-To: <6E5B44D2-119D-4675-BC4F-5C66C61C296B@economy-x-talk.com> References: <001201cc4f2d$2c2b7570$84826050$@lexiconbridge.com> <6E5B44D2-119D-4675-BC4F-5C66C61C296B@economy-x-talk.com> Message-ID: <003601cc4f9d$00074d00$0015e700$@lexiconbridge.com> Paolo, Mark--thanks for these tips. Combinations of "Set as stack Menu bar" and "Preview in Menu bar" now work as you say. I think I know what's going on, though. When the IDE is started by launching the *.livecode stack file, the stack starts in the Run mode. At this point the menu bar is my custom menu bar (a group), and by clicking the Browse tool, I cannot restore the IDE menu bar. Conversely, if I start the IDE first and then open that stack file, I have control, as you describe. I've noticed other strange things with 4.6.3 on this PPC G5. For example, if I build the PPC Mac OS standalone with the 4.6.2 in Windows and copy it to the G5 for testing, the menu bar in the standalone works as expected. But if I build the PPC standalone in the 4.6.3 on the G5, and then test it on the same machine, its menu bar consists only of the one item that the system puts in (with the About and Quit commands). My custom menu bar is not used. Weird. I'll have to experiment some more, or just give up on PPC and always author in Windows. The bigger problem at the moment is that the revBrowser on the Mac doesn't display Unicode Cyrillic. For each double-byte character it shows two single-byte characters, even though the same Web pages are displayed fine in revBrowser Windows, and also in Safari on the same Mac. The charset meta tag is correct, and I don't know what else to check. I'll go through the forums. Gratefully, Slava > -----Original Message----- > From: use-livecode-bounces at lists.runrev.com [mailto:use-livecode- > bounces at lists.runrev.com] On Behalf Of Mark Schonewille > Sent: Sunday, July 31, 2011 5:43 AM > To: How to use LiveCode > Subject: Re: lack of Mac OS X intuition: system menu > > Hi Slava, > > This can indeed be a problem once in a while. If you have closed all palettes and > windows and your own stack is the only stack visible, then you get may stuck. > Just don't close the tools palette unless you're absolutely sure. Whenever you > click in an IDE window or choose the pointer tool, you should see the IDE menu > bar, unless the Preview In Menubar option is set to true. > > -- > Best regards, > > Mark Schonewille > > Economy-x-Talk Consulting and Software Engineering > Homepage: http://economy-x-talk.com > Twitter: http://twitter.com/xtalkprogrammer > KvK: 50277553 > > What does that error mean? Buy LiveCodeErrors for iPhone now > http://qery.us/v4 A must-have for LiveCode programmers. > > On 31 jul 2011, at 04:54, Slava Paperno wrote: > > > I've been learning LC in Windows, and have now switched to Mac OS X, > > where everything is just so intuitive and easy... can someone please > > tell me how to remove my application menu from the system menu bar and > > restore the LC menu so I can do some work? My intuition fails on this > > point. I know I've seen this mentioned somewhere but can't find it now. > Thanks! > > > > Slava > > > _______________________________________________ > use-livecode mailing list > use-livecode at lists.runrev.com > Please visit this url to subscribe, unsubscribe and manage your subscription > preferences: > http://lists.runrev.com/mailman/listinfo/use-livecode From pete at mollysrevenge.com Sun Jul 31 12:47:04 2011 From: pete at mollysrevenge.com (Pete) Date: Sun, 31 Jul 2011 09:47:04 -0700 Subject: User Interface Question In-Reply-To: <3D40656D-2CDE-43AE-A7E2-A7FFF0BF63DA@gmail.com> References: <3D40656D-2CDE-43AE-A7E2-A7FFF0BF63DA@gmail.com> Message-ID: Thanks to all for the advice. I've tried all the suggestions but I still have the same problem. The cursor changes to the hand correctly when I hover over the datagrid. If I move the mouse off the datagrid in any direction except upwards, the cursor changes back to what it should be. If I move the mouse to any spot above the datagrid, the cursor stays as a hand and stays that way no matter where I move the mouse. It's as if the mouseLeave event never happens for mouse movement in the upward direction from the datagrid. While on the subject, can anyone recommend a good source for cursor icons (preferably free!)? Thanks, Pete Molly's Revenge On Sun, Jul 31, 2011 at 8:27 AM, Peter Brigham MD wrote: > And it's much better not to lock the cursor and then change it. Locked > cursors have a way of sticking if the handler to unlock the cursor somehow > misses. instead, don't lock anything, just set the defaultcursor to hand, > then set it to empty when done. I usually have this in my stack script: > > on mousemove > put the target into tTarg > if the name of the target contains "button" then > if the tool contains "browse" then > set the defaultcursor to hand > end if > else > set the defaultcursor to empty > end if > pass mouseMove > end mousemove > > on mouseLeave > set the defaultcursor to empty > pass mouseLeave > end mouseLeave > > > -- Peter > > Peter M. Brigham > pmbrig at gmail.com > http://home.comcast.net/~pmbrig > > > On Jul 30, 2011, at 8:17 PM, Jeff Massung wrote: > > > I never do mouse cursor changes in controls. I do all of them in the > card. > > > > on mouseMove px,py > > if px,py is within the rect of ... then > > -- cursor A > > else if px,py is within the rect of ... then > > -- cursor B > > else > > -- put the cursor back to normal > > end if > > end mouseMove > > > > This makes it *much* easier to control what's shown when. > > > > Jeff M. > > > > On Sat, Jul 30, 2011 at 1:58 PM, Pete wrote: > > > >> Thanks Jeff. I'm trying to implement the cursor change and running into > >> some weirdness. > >> > >> In the script of the datagrid, I have these two handlers: > >> > >> *on* mouseEnter > >> > >> *lock* cursor > >> > >> *set* the cursor to hand > >> > >> *end* mouseEnter > >> > >> > >> *on* mouseLeave > >> > >> *unlock* cursor > >> > >> *end* mouseLeave > >> > >> > >> This changes the cursor as expected and most of the time, it returns to > the > >> correct cursor when I move the mouse away from the datagrid. However, > if I > >> move the cursor above the datagrid, the cursor stays as the hand and > does > >> not return to any other image no matter where I move the mouse to. > >> > >> > >> This is the first time I've attempted to control the cursor so no doubt > I'm > >> doing something wrong! > >> > >> > >> Pete > >> Molly's Revenge > >> > >> > >> > >> > >> On Sat, Jul 30, 2011 at 11:48 AM, Jeff Massung > wrote: > >> > >>> For dragging, I'd go with "handles" on the left side of the datagrid > that > >>> have hand icon when you hover over them. They've become pretty > ubiquitous > >>> on > >>> webpages. For an example of what I mean: > >>> > >>> http://jqueryui.com/demos/sortable/ > >>> > >>> As for a context menu, you be okay with just a tool-tip. People > generally > >>> try and right-click a lot for things they expect to be there. > Otherwise, > >>> again, go with something common to show that actions are available. For > >>> example, and disclosure triangle on the far right of a UI widget is > >>> typically clickable and will bring up a menu. Left-click will bring up > >> your > >>> context menu, and r-click anywhere will bring it up as well. > >>> > >>> Jeff M. > >>> > >>> On Sat, Jul 30, 2011 at 12:42 PM, Pete wrote: > >>> > >>>> Wondering if anyone has any thoughts on a couple of user interface > >>>> questions. > >>>> > >>>> I'm providing a popup contextual menu when the user right-clicks on a > >>>> datagrid. I'm also providing the ability to re-order rows in a > >> datagrid > >>> by > >>>> dragging and dropping them. The common thread between these two > >>> functions > >>>> is that there is no on-screen indication that they exist. Popup menus > >>> are > >>>> invisible until they are invoked and drag and drop is equally > invisible > >>>> until someone initiates it. > >>>> > >>>> Are there any common methods for indicating to users that features > like > >>>> this > >>>> are available to them? > >>>> > >>>> Pete > >>>> Molly's Revenge > >>>> _______________________________________________ > >>>> use-livecode mailing list > >>>> use-livecode at lists.runrev.com > >>>> Please visit this url to subscribe, unsubscribe and manage your > >>>> subscription preferences: > >>>> http://lists.runrev.com/mailman/listinfo/use-livecode > >>>> > >>> _______________________________________________ > >>> use-livecode mailing list > >>> use-livecode at lists.runrev.com > >>> Please visit this url to subscribe, unsubscribe and manage your > >>> subscription preferences: > >>> http://lists.runrev.com/mailman/listinfo/use-livecode > >>> > >>> > >> _______________________________________________ > >> use-livecode mailing list > >> use-livecode at lists.runrev.com > >> Please visit this url to subscribe, unsubscribe and manage your > >> subscription preferences: > >> http://lists.runrev.com/mailman/listinfo/use-livecode > >> > > _______________________________________________ > > use-livecode mailing list > > use-livecode at lists.runrev.com > > Please visit this url to subscribe, unsubscribe and manage your > subscription preferences: > > http://lists.runrev.com/mailman/listinfo/use-livecode > > > _______________________________________________ > use-livecode mailing list > use-livecode at lists.runrev.com > Please visit this url to subscribe, unsubscribe and manage your > subscription preferences: > http://lists.runrev.com/mailman/listinfo/use-livecode > > From jacque at hyperactivesw.com Sun Jul 31 13:44:03 2011 From: jacque at hyperactivesw.com (J. Landman Gay) Date: Sun, 31 Jul 2011 12:44:03 -0500 Subject: lack of Mac OS X intuition: system menu In-Reply-To: <000101cc4f3d$f4c65620$de530260$@com> References: <001201cc4f2d$2c2b7570$84826050$@lexiconbridge.com> <6B257A2D-77E5-4B90-ADF1-97A61BF7F3EF@verizon.net> <000101cc4f3d$f4c65620$de530260$@com> Message-ID: <4E359463.9010505@hyperactivesw.com> On 7/30/11 11:54 PM, Slava Paperno wrote: > My stack has a menu bar built with the Menu Builder in the Tools menu. It is > essentially a group of buttons use as a menu bar. When I open the stack in > the IDE on Mac OS X, that menu replaces the standard LC IDE menu. No matter > which mode I am in (Browse or Run), I cannot see the LC IDE menu in the Mac > OS system menu bar. So nine of the IDE commands are available. I know this > is a trivial situation in Mac OS, and there is a secret handshake that > returns the IDE menu where it belongs. I even remember reading about this > somewhere. But I can't find that now. The IDE will replace the system menu with yours when your stack is frontmost. If you click on any IDE component (message box, tool palette, etc) the IDE menus will come back. It's dynamic. To keep your menus in the stack window while you are developing: set the editmenus of this stack to true That basically does the same thing as the checkbox in the menu builder. The editmenus property is not persistent across relaunches, so you'll need to set it each time you reopen the stack. I generally work with editmenus set to true until I'm ready to test, then I turn it off. -- Jacqueline Landman Gay | jacque at hyperactivesw.com HyperActive Software | http://www.hyperactivesw.com From shaosean at wehostmacs.com Sun Jul 31 14:40:49 2011 From: shaosean at wehostmacs.com (Shao Sean) Date: Sun, 31 Jul 2011 14:40:49 -0400 Subject: User Interface Question Message-ID: <6DB8F358-91F9-46F5-B09C-1CC3D900797F@wehostmacs.com> Is there a header on the datagrid? Does the same issue happen without a header? From slava at lexiconbridge.com Sun Jul 31 15:19:39 2011 From: slava at lexiconbridge.com (Slava Paperno) Date: Sun, 31 Jul 2011 15:19:39 -0400 Subject: lack of Mac OS X intuition: system menu In-Reply-To: <4E359463.9010505@hyperactivesw.com> References: <001201cc4f2d$2c2b7570$84826050$@lexiconbridge.com> <6B257A2D-77E5-4B90-ADF1-97A61BF7F3EF@verizon.net> <000101cc4f3d$f4c65620$de530260$@com> <4E359463.9010505@hyperactivesw.com> Message-ID: <000301cc4fb6$ce6269a0$6b273ce0$@com> Cool, thanks, Jacque! Since you know everything, do you also know why revBrowser won't display double-byte (Unicode Cyrillic) characters when my stack runs/is developed in Mac OS X (Tiger and Snow Leopard)? It displays two single-byte characters for each Cyrillic letter, which makes the text unreadable. The English text, including styles, displays fine, and this is not font-dependent: when I change the font in the HTML, Cyrillic is still garbled in the same way. The same stack works fine in Windows. The regular browsers on the Mac display the same Web pages correctly--but revBrowser doesn't. Does revBrowser work differently in Mac OS X than it does in Windows? Slava > -----Original Message----- > From: use-livecode-bounces at lists.runrev.com [mailto:use-livecode- > bounces at lists.runrev.com] On Behalf Of J. Landman Gay > Sent: Sunday, July 31, 2011 1:44 PM > To: How to use LiveCode > Subject: Re: lack of Mac OS X intuition: system menu >... > The IDE will replace the system menu with yours when your stack is > frontmost. If you click on any IDE component (message box, tool > palette, etc) the IDE menus will come back. It's dynamic. > > To keep your menus in the stack window while you are developing: > > set the editmenus of this stack to true > > That basically does the same thing as the checkbox in the menu builder. > The editmenus property is not persistent across relaunches, so you'll > need to set it each time you reopen the stack. > > I generally work with editmenus set to true until I'm ready to test, > then I turn it off. > > -- > Jacqueline Landman Gay | jacque at hyperactivesw.com > HyperActive Software | http://www.hyperactivesw.com From admin at mfelkerco.com Sun Jul 31 15:31:51 2011 From: admin at mfelkerco.com (Admin) Date: Sun, 31 Jul 2011 14:31:51 -0500 Subject: HELP - Major Emergency - substack not showing In-Reply-To: References: <1312060134258-3706864.post@n4.nabble.com> <6b0e0364f1463b9075663dc224ddd737@mfelkerco.com> <373366e9c8ad62faf8f60d437cb060dc@mfelkerco.com> Message-ID: I just finished creating a 3 card substack and now the cards won't open when clicked or coded to open. On top of that, Live Code is acting very sluggish ever since I started working on the substack. When you try to open one, the current stack window flashes and nothing happens. This is catastrophic for me right now. My project is due tomorrow and I was just about finished. How do I get them back???? I cannot afford to re-do it. My backups are also the same. Running newest version of Live Code. Same problem happens in the previous version. The substack worked fine and then all of a sudden, things got very sluggish and now the substack won't open. Mike From admin at mfelkerco.com Sun Jul 31 15:33:18 2011 From: admin at mfelkerco.com (Admin) Date: Sun, 31 Jul 2011 14:33:18 -0500 Subject: Getting someone's age from dob and current year In-Reply-To: <6AFFE4A9-7AC1-4A9E-8D1B-8B82041A00C3@gmail.com> References: <1312060134258-3706864.post@n4.nabble.com> <201107302303.40864.warren@warrensweb.us> <6AFFE4A9-7AC1-4A9E-8D1B-8B82041A00C3@gmail.com> Message-ID: <0006771a3fadd088389b4e593ad4910f@mfelkerco.com> Thanks everyone. I figured the dob problem out and it's now working. Now I just need to get my substack back. Have there been stability problems with substacks? Mike On Sun, 31 Jul 2011 11:32:02 -0400, Peter Brigham MD wrote: > On Jul 31, 2011, at 12:03 AM, Warren Samples wrote: > >> On Saturday, July 30, 2011 07:16:44 PM Roger Eller wrote: >> >>> convert tToday to seconds convert tBorn to seconds >> It's been talked about before, but this calls for a reminder: attempting to convert a date prior to Jan 1, 1970 to "seconds" returns "invalid date" instead of a number in Windows. Don't do this if you are going to need this to work in Windows! Under Windows, you need to subtract the birthyear from the current year and determine if the current month/day is later than the birth month/day and subtract one year if necessary. Conveniently, this methods does work on all platforms. > > function doAge tDOB > -- assumes format of m/d/yy > set the itemdelimiter to "/" > put item 3 of tDOB into DOByr > put item 2 of tDOB into DOBmo > put item 1 of tDOB into DOBdy > put item 3 of the short date into nowYr > put item 2 of the short date into nowMo > put item 1 of the short date into nowDy > if DOByr > nowYr then -- prev century > -- assumes no one with age > 100 > put "19" before DOByr > else > put "20" before DOByr > end if > put "20" before nowYr > put nowYr - DOByr into tAge > if nowMo > DOBmo then return tAge > if nowMo < DOBmo then return tAge -1 > if nowDy < DOBdy then return tAge -1 > return tAge > end doAge > > -- Peter > > Peter M. Brigham > pmbrig at gmail.com [1] > http://home.comcast.net/~pmbrig [2] > > _______________________________________________ > use-livecode mailing list > use-livecode at lists.runrev.com [3] > Please visit this url to subscribe, unsubscribe and manage your subscription preferences: > http://lists.runrev.com/mailman/listinfo/use-livecode [4] Links: ------ [1] mailto:pmbrig at gmail.com [2] http://home.comcast.net/~pmbrig [3] mailto:use-livecode at lists.runrev.com [4] http://lists.runrev.com/mailman/listinfo/use-livecode From admin at mfelkerco.com Sun Jul 31 16:32:16 2011 From: admin at mfelkerco.com (Admin) Date: Sun, 31 Jul 2011 15:32:16 -0500 Subject: HELP - Major Emergency - substack not showing In-Reply-To: References: <1312060134258-3706864.post@n4.nabble.com> <6b0e0364f1463b9075663dc224ddd737@mfelkerco.com> <373366e9c8ad62faf8f60d437cb060dc@mfelkerco.com> Message-ID: OK, I found out the substack is being either minimized or off the screen, but if I right click on the taskbar (the page shows up when I hover over the LiveCode icon) and choose "open maximized" and then move it into position, it shows. However, this is lost when I close the project and re-open it. What can I do? Mike On Sun, 31 Jul 2011 14:31:51 -0500, Admin wrote: > I just finished creating a 3 card substack and now the cards won't > open when clicked or coded to open. On top of that, Live Code is acting > very sluggish ever since I started working on the substack. When you try > to open one, the current stack window flashes and nothing happens. > > This > is catastrophic for me right now. My project is due tomorrow and I was > just about finished. How do I get them back???? I cannot afford to re-do > it. My backups are also the same. Running newest version of Live Code. > Same problem happens in the previous version. The substack worked fine > and then all of a sudden, things got very sluggish and now the substack > won't open. > > Mike > > _______________________________________________ > use-livecode mailing list > use-livecode at lists.runrev.com [1] > Please visit this url to subscribe, unsubscribe and manage your subscription preferences: > http://lists.runrev.com/mailman/listinfo/use-livecode [2] Links: ------ [1] mailto:use-livecode at lists.runrev.com [2] http://lists.runrev.com/mailman/listinfo/use-livecode From admin at mfelkerco.com Sun Jul 31 16:49:12 2011 From: admin at mfelkerco.com (Admin) Date: Sun, 31 Jul 2011 15:49:12 -0500 Subject: HELP - Major Emergency - substack not showing In-Reply-To: References: <1312060134258-3706864.post@n4.nabble.com> <6b0e0364f1463b9075663dc224ddd737@mfelkerco.com> <373366e9c8ad62faf8f60d437cb060dc@mfelkerco.com> Message-ID: <7b9bef42f5e84b50e0af6ba7dc1baf34@mfelkerco.com> OK. I fixed it. But something has made LiveCode totally twitchy as a result of all of this. Next question. How do you copy a substack. I need an identical copy of it, but need to make a few tiny changes. Thanks. Mike On Sun, 31 Jul 2011 15:32:16 -0500, Admin wrote: > OK, I found out the substack is being either minimized or off the > screen, but if I right click on the taskbar (the page shows up when I > hover over the LiveCode icon) and choose "open maximized" and then move > it into position, it shows. However, this is lost when I close the > project and re-open it. > > What can I do? > > Mike > > On Sun, 31 Jul 2011 > 14:31:51 -0500, Admin wrote: > >> I just finished creating a 3 card > > substack and now the cards won't > >> open when clicked or coded to open. > > On top of that, Live Code is acting > /pre> > dding-left:5px; border-left:#1010ff 2px solid; margin-left:5px; width:100%">it. My backups are > > also the same. Running newest version of Live Code. > blem > n the previous version. The substack worked fine and then all > >> g-left:5px; border > 2px solid; margin-left:5px; width:100%">won't > > open. > >> ckquote type > ="padding-left:5px; border-left:#1010ff 2px solid; margin-left:5px; width:100%">use-livecode at lists.runrev.com [1] [1] Please kquote type= > "padding-left:5px; border-left:#1010ff 2px solid; margin-left:5px; width:100%">http://lists.runrev.com/mailman/list > >> @list >> use-livecode at lists.runrev.com [2] http://lists.runrev.c > >> ist > lists.runrev.com Please visit this url to subscribe, unsubscribe and manage your subscription preferences: Links: ------ [1] mailto:use-livecode at lists.runrev.com From jacque at hyperactivesw.com Sun Jul 31 17:02:14 2011 From: jacque at hyperactivesw.com (J. Landman Gay) Date: Sun, 31 Jul 2011 16:02:14 -0500 Subject: HELP - Major Emergency - substack not showing In-Reply-To: <7b9bef42f5e84b50e0af6ba7dc1baf34@mfelkerco.com> References: <1312060134258-3706864.post@n4.nabble.com> <6b0e0364f1463b9075663dc224ddd737@mfelkerco.com> <373366e9c8ad62faf8f60d437cb060dc@mfelkerco.com> <7b9bef42f5e84b50e0af6ba7dc1baf34@mfelkerco.com> Message-ID: <4E35C2D6.6010106@hyperactivesw.com> On 7/31/11 3:49 PM, Admin wrote: > > > OK. I fixed it. But something has made LiveCode totally twitchy as > a result of all of this. It's probably something in a script somewhere. Substacks are behaving normally as far as I can see. > > Next question. How do you copy a substack. I > need an identical copy of it, but need to make a few tiny changes. "clone stack " This will make an independent copy that isn't a substack, it is its own mainstack. It's name will be "Copy of Whatever". If you are just making a copy in order to have a backup, it's usually easier to save the copy to disk and make the changes in the existing substack. -- Jacqueline Landman Gay | jacque at hyperactivesw.com HyperActive Software | http://www.hyperactivesw.com From pete at mollysrevenge.com Sun Jul 31 17:04:01 2011 From: pete at mollysrevenge.com (Pete) Date: Sun, 31 Jul 2011 14:04:01 -0700 Subject: User Interface Question In-Reply-To: <6DB8F358-91F9-46F5-B09C-1CC3D900797F@wehostmacs.com> References: <6DB8F358-91F9-46F5-B09C-1CC3D900797F@wehostmacs.com> Message-ID: Yes there is a header. I tried hiding the header by setting the show header property to false, but it still is displayed. Beginning to suspect that I have some sort of corruption in the datagrid. Pete Molly's Revenge On Sun, Jul 31, 2011 at 11:40 AM, Shao Sean wrote: > Is there a header on the datagrid? Does the same issue happen without a > header? > > > ______________________________**_________________ > use-livecode mailing list > use-livecode at lists.runrev.com > Please visit this url to subscribe, unsubscribe and manage your > subscription preferences: > http://lists.runrev.com/**mailman/listinfo/use-livecode > > From jacque at hyperactivesw.com Sun Jul 31 17:04:49 2011 From: jacque at hyperactivesw.com (J. Landman Gay) Date: Sun, 31 Jul 2011 16:04:49 -0500 Subject: lack of Mac OS X intuition: system menu In-Reply-To: <000301cc4fb6$ce6269a0$6b273ce0$@com> References: <001201cc4f2d$2c2b7570$84826050$@lexiconbridge.com> <6B257A2D-77E5-4B90-ADF1-97A61BF7F3EF@verizon.net> <000101cc4f3d$f4c65620$de530260$@com> <4E359463.9010505@hyperactivesw.com> <000301cc4fb6$ce6269a0$6b273ce0$@com> Message-ID: <4E35C371.8040306@hyperactivesw.com> On 7/31/11 2:19 PM, Slava Paperno wrote: > Cool, thanks, Jacque! Since you know everything, Hardly. As I prove here: > do you also know why > revBrowser won't display double-byte (Unicode Cyrillic) characters when my > stack runs/is developed in Mac OS X I wish I did. I've hardly used revBrowser and I've avoided unicode, so there are holes in my knowledge base about those. But you were right that revBrowser uses the built-in Safari webkit. How does your page look if you open it directly in Safari? -- Jacqueline Landman Gay | jacque at hyperactivesw.com HyperActive Software | http://www.hyperactivesw.com From littlejamesw at mac.com Sun Jul 31 18:33:11 2011 From: littlejamesw at mac.com (James Little) Date: Sun, 31 Jul 2011 15:33:11 -0700 Subject: Where does survive the inventive user ? In-Reply-To: <9F9F49D9-9AF5-45B6-AF72-B2B239E55388@gmail.com> References: <8CE1A70D9625EA6-2550-214DE@web-mmc-d04.sysops.aol.com> <44857038-F42F-4374-8DA7-F6BCDF16629C@doctorTimothyMiller.com> <9F9F49D9-9AF5-45B6-AF72-B2B239E55388@gmail.com> Message-ID: <82F8EE96-1FAD-406C-9D2B-EEEEACB1E7D9@mac.com> I too am not a professional programmer and did not come from a Hypercard background. As a physician and clinical researcher, I found RunRev 5 years ago because I wanted to test a theory that syringomyelia (a pathologic cyst in the spinal cord) would affect motor control, which might prove useful for diagnosis or clinical monitoring. Within 6 months of purchasing Runtime Revolution (now LiveCode) I had a desktop app that could test rapid alternating finger movements. It's a shame that the theory did not hold up, but after that experience, I became hooked as a RunRev hobbyist. Over the last 5 years, I tried other development environments, but I always came back to LiveCode; for me as primarily a content person and not a programmer, it was the learning curve that I could manage. Within the last year, I retired as a full-time physician and I'm able to devote more time to LiveCode. I've also begun to collaborate with my son on some LiveCode projects; at 24, he's picked it up much faster than I did. I still have much to learn (LiveCode Server, HTML & CSS integration with LiveCode, iOS & Android development). I am grateful to RunRev for its remarkable ongoing success at providing cross-platform tools that allow non-programmers to do some programming. I'm also grateful to the generosity of seasoned RunRev developers that contribute to this developer's list and to the forum. Those contributions are especially valuable to non-programmers, like myself. I concur with what Peter Brigham wrote, "I have no idea how you market to people like me." RunRev has continued to make available new tools that are valuable for non-programmers (e.g. Lessons, Summer Academy). RunRev has a strong website and annual conferences (live & streaming) that draw in potential users. There are wonderful 3rd party sites that assist non-programmers (LivecodeJournal.com, revolution.byu.edu, hyperactivesw.com ...). A possible suggestion for marketing, is that a mechanism / an exchange be established to assist a newer programmer in obtaining paid assistance from an experienced programmer. For example, I might have interest in contracting with an experienced LC programmer to beta test an app. Also, as a retired professor in my department at the University, I've thought about volunteering to assist faculty members undertake some LiveCode projects. Before I committed myself to such projects, I would like to know that there is available, at a reasonable cost, some advanced assistance. From the RunRev website (runrev dot com/support/consultants/), from this developer's list and from the forum, I certainly have many names of advanced LC programmers but 1) I don't know who might have interest in such short-term involvement, 2) I don't know who has experience in iOS or Android development, and 3) I don't know the ballpark the costs. Further elaborating a mechanism to link moderately-experienced to advanced LiveCode programmers may help promote LiveCode. Best, Jim L. On Jul 27, 2011, at 7:24 PM, Peter Brigham MD wrote: > ... I'm not a programmer, just picked it up on the side. I have no idea how you market to people like me, but I suspect there are lots of us scattered around. > > -- Peter > > On Jul 27, 2011, at 9:25 PM, Timothy Miller wrote: >> >> ... It's gradually dawning on me that "programmers" like me have become rather rare. Fewer and fewer non-professionals on this list, as far as I can tell. I don't understand why, seems like a shame. >> >> Many people have use for the kind of functionality an amateur and dabbler can get out of LiveCode, and it isn't that hard to do. >> >> Admittedly, HyperCard was easier, simply because it was less complex. I've wondered if LiveCode might be more approachable if it had some kind of "dumb mode," sort of like the old userLevel system in HyperCard. Probably won't happen though. >> >> FWIW... >> >> Tim >> From bvlahos at mac.com Sun Jul 31 19:50:28 2011 From: bvlahos at mac.com (Bill Vlahos) Date: Sun, 31 Jul 2011 16:50:28 -0700 Subject: Notification Manger or Bounce the Dock Icon Message-ID: <2CEFF3CB-7A83-465A-8807-DB0651C773EB@mac.com> How do I trigger the system (Mac, Windows, Linux) notification manager when a dialog box comes up? If my program is not the frontmost app and a dialog box comes up, the user won't notice that the application needs attention if other applications are covering the area of the screen showing the dialog box (either as sheet or modal). I expected a dialog box to trigger the system notification manager. On the Mac that would be to bounce the dock icon. I thought I read how LC can do this but I can't find it. Any help would be appreciated. Thanks, Bill Vlahos _________________ InfoWallet (http://www.infowallet.com) is about keeping your important life information with you, accessible, and secure. From slava at lexiconbridge.com Sun Jul 31 20:21:21 2011 From: slava at lexiconbridge.com (Slava Paperno) Date: Sun, 31 Jul 2011 20:21:21 -0400 Subject: revBrowser, Mac OS X, and Unicode In-Reply-To: <4E35C371.8040306@hyperactivesw.com> References: <001201cc4f2d$2c2b7570$84826050$@lexiconbridge.com> <6B257A2D-77E5-4B90-ADF1-97A61BF7F3EF@verizon.net> <000101cc4f3d$f4c65620$de530260$@com> <4E359463.9010505@hyperactivesw.com> <000301cc4fb6$ce6269a0$6b273ce0$@com> <4E35C371.8040306@hyperactivesw.com> Message-ID: <000a01cc4fe0$f4340420$dc9c0c60$@com> Jacqueline said: > But you were right that revBrowser uses the built-in Safari webkit. How does your page > look if you open it directly in Safari? It looks fine--just as I expect it to look, and just the way it looks in my Windows stack. Unicode Cyrillic is displayed as it is displayed in all browsers on the Web. If it turns out that revBrowser in Mac SO X cannot display Unicode, it's good-bye LiveCode for me. I've posted this in a forum. Perhaps someone knows what I am doing wrong, if anything. My one remaining secret weapon, yet to be deployed, is a support request that I am entitled to submit to runrev because I subscribed to the dev program. Slava From jacque at hyperactivesw.com Sun Jul 31 21:42:12 2011 From: jacque at hyperactivesw.com (J. Landman Gay) Date: Sun, 31 Jul 2011 20:42:12 -0500 Subject: revBrowser, Mac OS X, and Unicode In-Reply-To: <000a01cc4fe0$f4340420$dc9c0c60$@com> References: <001201cc4f2d$2c2b7570$84826050$@lexiconbridge.com> <6B257A2D-77E5-4B90-ADF1-97A61BF7F3EF@verizon.net> <000101cc4f3d$f4c65620$de530260$@com> <4E359463.9010505@hyperactivesw.com> <000301cc4fb6$ce6269a0$6b273ce0$@com> <4E35C371.8040306@hyperactivesw.com> <000a01cc4fe0$f4340420$dc9c0c60$@com> Message-ID: <4E360474.6030103@hyperactivesw.com> On 7/31/11 7:21 PM, Slava Paperno wrote: > My one remaining secret weapon, yet to be deployed, is a support > request that I am entitled to submit to runrev because I subscribed to the > dev program. You should be on the developer's list if you're in the dev program. The engineers read that list and will respond to you directly; it's one of the perks you've paid for. That would be the best place for you to ask, and you don't need to use a support ticket. I think you should have received sign-up info when you joined the developer program, but if you didn't or you don't have it any more, write to support at runrev.com and they will get you in. You're entitled. Monday is a bank holiday over there, so give them a day or two after that to get back to you. -- Jacqueline Landman Gay | jacque at hyperactivesw.com HyperActive Software | http://www.hyperactivesw.com From admin at mfelkerco.com Sun Jul 31 21:58:34 2011 From: admin at mfelkerco.com (Admin) Date: Sun, 31 Jul 2011 20:58:34 -0500 Subject: How do I remove part of a =?UTF-8?Q?path=3F?= In-Reply-To: <4E35C2D6.6010106@hyperactivesw.com> References: <1312060134258-3706864.post@n4.nabble.com> <6b0e0364f1463b9075663dc224ddd737@mfelkerco.com> <373366e9c8ad62faf8f60d437cb060dc@mfelkerco.com> <7b9bef42f5e84b50e0af6ba7dc1baf34@mfelkerco.com> <4E35C2D6.6010106@hyperactivesw.com> Message-ID: <4f672bf05eff586a0d7e41c89628ffc7@mfelkerco.com> Hello all, Say I have the following path: http://www.travelandworkusa.com/public_html/userfiles/Felker_Mike/robot_trans_noshadow.png How do I get rid of just /public_html ? Thanks. Mike From mikekann at yahoo.com Sun Jul 31 22:23:16 2011 From: mikekann at yahoo.com (Michael Kann) Date: Sun, 31 Jul 2011 19:23:16 -0700 (PDT) Subject: How do I remove part of a path? In-Reply-To: <4f672bf05eff586a0d7e41c89628ffc7@mfelkerco.com> Message-ID: <1312165396.64563.YahooMailClassic@web161606.mail.bf1.yahoo.com> I'll take this one. put" http://www.travelandworkusa.com/public_html/userfiles/Felker_Mike/robot_trans_noshadow.png" into k replace "/public_html" with empty in k -- put k back where you got it Good luck. You might want to take a little break and come back refreshed. Mike --- On Sun, 7/31/11, Admin wrote: From: Admin Subject: Re: How do I remove part of a path? To: "How to use LiveCode" Date: Sunday, July 31, 2011, 8:58 PM ? Hello all, Say I have the following path: http://www.travelandworkusa.com/public_html/userfiles/Felker_Mike/robot_trans_noshadow.png How do I get rid of just /public_html ? Thanks. Mike ? _______________________________________________ use-livecode mailing list use-livecode at lists.runrev.com Please visit this url to subscribe, unsubscribe and manage your subscription preferences: http://lists.runrev.com/mailman/listinfo/use-livecode From roger.e.eller at sealedair.com Sun Jul 31 22:23:38 2011 From: roger.e.eller at sealedair.com (Roger Eller) Date: Sun, 31 Jul 2011 22:23:38 -0400 Subject: How do I remove part of a path? In-Reply-To: <4f672bf05eff586a0d7e41c89628ffc7@mfelkerco.com> References: <1312060134258-3706864.post@n4.nabble.com> <6b0e0364f1463b9075663dc224ddd737@mfelkerco.com> <373366e9c8ad62faf8f60d437cb060dc@mfelkerco.com> <7b9bef42f5e84b50e0af6ba7dc1baf34@mfelkerco.com> <4E35C2D6.6010106@hyperactivesw.com> <4f672bf05eff586a0d7e41c89628ffc7@mfelkerco.com> Message-ID: On Sun, Jul 31, 2011 at 9:58 PM, Admin wrote: > > > Hello all, > > Say I have the following path: > > > > http://www.travelandworkusa.com/public_html/userfiles/Felker_Mike/robot_trans_noshadow.png > > > How do I get rid of just /public_html ? > > Thanks. > > Mike > put " http://www.travelandworkusa.com/public_html/userfiles/Felker_Mike/robot_trans_noshadow.png" into tURL replace "/public_html" with empty in tURL ?Roger From roger.e.eller at sealedair.com Sun Jul 31 22:25:38 2011 From: roger.e.eller at sealedair.com (Roger Eller) Date: Sun, 31 Jul 2011 22:25:38 -0400 Subject: How do I remove part of a path? In-Reply-To: <1312165396.64563.YahooMailClassic@web161606.mail.bf1.yahoo.com> References: <4f672bf05eff586a0d7e41c89628ffc7@mfelkerco.com> <1312165396.64563.YahooMailClassic@web161606.mail.bf1.yahoo.com> Message-ID: On Sun, Jul 31, 2011 at 10:23 PM, Michael Kann wrote: > I'll take this one. > > put" > http://www.travelandworkusa.com/public_html/userfiles/Felker_Mike/robot_trans_noshadow.png" > into k > replace "/public_html" with empty in k > > -- put k back where you got it > > Good luck. You might want to take a little break and come back refreshed. > > Mike > Way too slow Michael! ;-) ?Roger From slava at lexiconbridge.com Sun Jul 31 22:32:41 2011 From: slava at lexiconbridge.com (Slava Paperno) Date: Sun, 31 Jul 2011 22:32:41 -0400 Subject: lack of Mac OS X intuition: system menu In-Reply-To: <000301cc4fb6$ce6269a0$6b273ce0$@com> References: <001201cc4f2d$2c2b7570$84826050$@lexiconbridge.com> <6B257A2D-77E5-4B90-ADF1-97A61BF7F3EF@verizon.net> <000101cc4f3d$f4c65620$de530260$@com> <4E359463.9010505@hyperactivesw.com> <000301cc4fb6$ce6269a0$6b273ce0$@com> Message-ID: <000b01cc4ff3$4cc04ce0$e640e6a0$@com> Earlier today Jacque said: > > The IDE will replace the system menu with yours when your stack is > > frontmost. If you click on any IDE component (message box, tool > > palette, etc) the IDE menus will come back. It's dynamic. I have not been able to achieve this no matter what value of "editmenus of this stack" or the check boxes in Menu Builder. In my experiments with various permutations of those values, the system menu bar at the top of the Mac OS X screen show either the full IDE menu bar, or my custom menu bar regardless of which window is the active one and also regardless of whether I work in the Run or Browse mode. This is on Mac G5 (PPC) 10.5.8, LC 4.6.3. Here are some of these scenarios: Scenario 1: In Message Box put the editmenus of this stack --false In Menu Builder: Preview in Menu Bar [checked] Set as stack Menu Bar [clear] Mode: Browse No matter whether the frontmost window is my stack, the Tools p[palette, the Application Browser, the Message Box, or the Inspector, the menu at the top of the screen is my custom menu, and I can't restore the IDE menu bar by clicking on windows. Scenario 2: Everything the same as above except that it's Run mode. Same behavior: I see my custom menu bar, and can't restore the IDE menu bar by clicking on windows. No matter which of them is in front, the IDE menu bar does not come back. Scenario 3: In Message Box set the editmenus of this stack to true The behavior of the menu bar is exactly the same as above: my custom menu bar is up, both in Run and Browse modes, no matter which window is in front. It never becomes dynamic, if I understand your use of this term correctly. If I uncheck the Preview in Menu Bar box in Menu Builder, the menu bar always shows the IDE menus--again, it is not dynamic. The state of the Set as stack Menu Bar check box makes no difference. So I must be still doing something different/wrong. Slava PS. Thanks for your note about the dev list. I am on the list but have not yet posted the Unicode in revBrowser problem because I wanted first to see what can be gleaned on this list. > > > > To keep your menus in the stack window while you are developing: > > > > set the editmenus of this stack to true > > > > That basically does the same thing as the checkbox in the menu > builder. > > The editmenus property is not persistent across relaunches, so you'll > > need to set it each time you reopen the stack. > > > > I generally work with editmenus set to true until I'm ready to test, > > then I turn it off. > > > > -- > > Jacqueline Landman Gay | jacque at hyperactivesw.com > > HyperActive Software | http://www.hyperactivesw.com > > > > > _______________________________________________ > use-livecode mailing list > use-livecode at lists.runrev.com > Please visit this url to subscribe, unsubscribe and manage your > subscription preferences: > http://lists.runrev.com/mailman/listinfo/use-livecode From pete at mollysrevenge.com Sun Jul 31 22:36:27 2011 From: pete at mollysrevenge.com (Pete) Date: Sun, 31 Jul 2011 19:36:27 -0700 Subject: How do I remove part of a path? In-Reply-To: <4f672bf05eff586a0d7e41c89628ffc7@mfelkerco.com> References: <1312060134258-3706864.post@n4.nabble.com> <6b0e0364f1463b9075663dc224ddd737@mfelkerco.com> <373366e9c8ad62faf8f60d437cb060dc@mfelkerco.com> <7b9bef42f5e84b50e0af6ba7dc1baf34@mfelkerco.com> <4E35C2D6.6010106@hyperactivesw.com> <4f672bf05eff586a0d7e41c89628ffc7@mfelkerco.com> Message-ID: replace "/public.html" with empty in . That's assuming it's always the same string you want to get rid of. Alternatively: set the itemdelimiter to "/" replace item 4 of with empty Pete Molly's Revenge On Sun, Jul 31, 2011 at 6:58 PM, Admin wrote: > > > Hello all, > > Say I have the following path: > > > > http://www.travelandworkusa.com/public_html/userfiles/Felker_Mike/robot_trans_noshadow.png > > > How do I get rid of just /public_html ? > > Thanks. > > Mike > > _______________________________________________ > use-livecode mailing list > use-livecode at lists.runrev.com > Please visit this url to subscribe, unsubscribe and manage your > subscription preferences: > http://lists.runrev.com/mailman/listinfo/use-livecode > > From mikekann at yahoo.com Sun Jul 31 22:51:05 2011 From: mikekann at yahoo.com (Michael Kann) Date: Sun, 31 Jul 2011 19:51:05 -0700 (PDT) Subject: How do I remove part of a path? In-Reply-To: Message-ID: <1312167065.60222.YahooMailClassic@web161617.mail.bf1.yahoo.com> Pete, One little improvement is needed: replace item 4 of with empty in When I change the itemDelimiter to something exotic I almost always forget to change it back then wonder why those items don't obey. If you are going to change the itemDelimiter you might try something like: put the itemDel into old_delimiter set the itemDel to "/" -- gather some items set the itemDel to old_delimiter Mike --- On Sun, 7/31/11, Pete wrote: From: Pete Subject: Re: How do I remove part of a path? To: "How to use LiveCode" Date: Sunday, July 31, 2011, 9:36 PM replace "/public.html" with empty in .? That's assuming it's always the same string you want to get rid of.? Alternatively: set the itemdelimiter to "/" replace item 4 of with empty Pete Molly's Revenge On Sun, Jul 31, 2011 at 6:58 PM, Admin wrote: > > > Hello all, > > Say I have the following path: > > > > http://www.travelandworkusa.com/public_html/userfiles/Felker_Mike/robot_trans_noshadow.png > > > How do I get rid of just /public_html ? > > Thanks. > > Mike > > _______________________________________________ > use-livecode mailing list > use-livecode at lists.runrev.com > Please visit this url to subscribe, unsubscribe and manage your > subscription preferences: > http://lists.runrev.com/mailman/listinfo/use-livecode > > _______________________________________________ use-livecode mailing list use-livecode at lists.runrev.com Please visit this url to subscribe, unsubscribe and manage your subscription preferences: http://lists.runrev.com/mailman/listinfo/use-livecode From mikekann at yahoo.com Sun Jul 31 22:55:57 2011 From: mikekann at yahoo.com (Michael Kann) Date: Sun, 31 Jul 2011 19:55:57 -0700 (PDT) Subject: How do I remove part of a path? In-Reply-To: Message-ID: <1312167357.70982.YahooMailClassic@web161619.mail.bf1.yahoo.com> Pete, I checked to see if the itemDelimiter trick left behind a "/" and found that it does. http://www.travelandworkusa.com//userfiles/Felker_Mike/robot_trans_noshadow.png -- notice the 2 forward slashes where there should be uno. Mike --- On Sun, 7/31/11, Pete wrote: From: Pete Subject: Re: How do I remove part of a path? To: "How to use LiveCode" Date: Sunday, July 31, 2011, 9:36 PM replace "/public.html" with empty in .? That's assuming it's always the same string you want to get rid of.? Alternatively: set the itemdelimiter to "/" replace item 4 of with empty Pete Molly's Revenge On Sun, Jul 31, 2011 at 6:58 PM, Admin wrote: > > > Hello all, > > Say I have the following path: > > > > http://www.travelandworkusa.com/public_html/userfiles/Felker_Mike/robot_trans_noshadow.png > > > How do I get rid of just /public_html ? > > Thanks. > > Mike > > _______________________________________________ > use-livecode mailing list > use-livecode at lists.runrev.com > Please visit this url to subscribe, unsubscribe and manage your > subscription preferences: > http://lists.runrev.com/mailman/listinfo/use-livecode > > _______________________________________________ use-livecode mailing list use-livecode at lists.runrev.com Please visit this url to subscribe, unsubscribe and manage your subscription preferences: http://lists.runrev.com/mailman/listinfo/use-livecode From roger.e.eller at sealedair.com Sun Jul 31 22:58:45 2011 From: roger.e.eller at sealedair.com (Roger Eller) Date: Sun, 31 Jul 2011 22:58:45 -0400 Subject: How do I remove part of a path? In-Reply-To: References: <1312060134258-3706864.post@n4.nabble.com> <6b0e0364f1463b9075663dc224ddd737@mfelkerco.com> <373366e9c8ad62faf8f60d437cb060dc@mfelkerco.com> <7b9bef42f5e84b50e0af6ba7dc1baf34@mfelkerco.com> <4E35C2D6.6010106@hyperactivesw.com> <4f672bf05eff586a0d7e41c89628ffc7@mfelkerco.com> Message-ID: On Sun, Jul 31, 2011 at 10:36 PM, Pete wrote: > replace "/public.html" with empty in . That's > assuming > it's always the same string you want to get rid of. Alternatively: > > set the itemdelimiter to "/" > replace item 4 of with empty > > Pete > Molly's Revenge > > The problem with replacing an item, is that the URL will be left with an extra slash after .com// ?Roger From bvlahos at mac.com Sun Jul 31 22:59:48 2011 From: bvlahos at mac.com (Bill Vlahos) Date: Sun, 31 Jul 2011 19:59:48 -0700 Subject: State of an application status Message-ID: <93B45402-1148-48DA-9D14-9CCB664F40FF@mac.com> Using the on suspend and on resume handlers I can tell when a program becomes a background application for foreground application. There are times in my program that I want it to behave differently depending if it is foreground or not. How can I query the current state of the application? I suppose I could continually update some flag with the on suspend and on resume handlers but that seems pretty inelegant. Bill Vlahos _________________ InfoWallet (http://www.infowallet.com) is about keeping your important life information with you, accessible, and secure. From capellan2000 at gmail.com Sun Jul 31 23:24:19 2011 From: capellan2000 at gmail.com (Alejandro Tejada) Date: Sun, 31 Jul 2011 23:24:19 -0400 Subject: MD5 digest of arrays and controls Message-ID: Hi all, How could I get the MD5digest of an Array and all the properties of a control? For example, I created a control, by script, and I want to make sure that it's identical to another control created by a different script. How could I compare both controls using a MD5digest of their relevant properties? (obviously, their name should be different, so I do not want to include that property in the comparision). And How could I do this with two global Array variables? I collect data in these two different arrays using two different scripts and need to be sure that their data is identical. Thanks in advance! Al From jacque at hyperactivesw.com Sun Jul 31 23:41:40 2011 From: jacque at hyperactivesw.com (J. Landman Gay) Date: Sun, 31 Jul 2011 22:41:40 -0500 Subject: lack of Mac OS X intuition: system menu In-Reply-To: <000b01cc4ff3$4cc04ce0$e640e6a0$@com> References: <001201cc4f2d$2c2b7570$84826050$@lexiconbridge.com> <6B257A2D-77E5-4B90-ADF1-97A61BF7F3EF@verizon.net> <000101cc4f3d$f4c65620$de530260$@com> <4E359463.9010505@hyperactivesw.com> <000301cc4fb6$ce6269a0$6b273ce0$@com> <000b01cc4ff3$4cc04ce0$e640e6a0$@com> Message-ID: <4E362074.3000600@hyperactivesw.com> On 7/31/11 9:32 PM, Slava Paperno wrote: > Earlier today Jacque said: > >>> The IDE will replace the system menu with yours when your stack is >>> frontmost. If you click on any IDE component (message box, tool >>> palette, etc) the IDE menus will come back. It's dynamic. > > I have not been able to achieve this no matter what value of "editmenus of > this stack" or the check boxes in Menu Builder. Does your stack set the defaultMenuBar? The defaultMenuBar is handled by the IDE, and if your stack sets it, it will override the IDE's behavior. If you need to set it in a standalone, change your script to check the environment and only set the defaultMenuBar if the environment is not "development". That's all I can think of that would cause the IDE to fail to respond correctly. -- Jacqueline Landman Gay | jacque at hyperactivesw.com HyperActive Software | http://www.hyperactivesw.com From pete at mollysrevenge.com Sun Jul 31 23:45:24 2011 From: pete at mollysrevenge.com (Pete) Date: Sun, 31 Jul 2011 20:45:24 -0700 Subject: How do I remove part of a path? In-Reply-To: <1312167065.60222.YahooMailClassic@web161617.mail.bf1.yahoo.com> References: <1312167065.60222.YahooMailClassic@web161617.mail.bf1.yahoo.com> Message-ID: Yeah, messed up the syntax, but you get the idea! Pete Molly's Revenge On Sun, Jul 31, 2011 at 7:51 PM, Michael Kann wrote: > Pete, > > One little improvement is needed: > > replace item 4 of with empty in is> > > When I change the itemDelimiter to something exotic I almost always forget > to change it back then wonder why those items don't obey. If you are going > to change the itemDelimiter you might try something like: > > put the itemDel into old_delimiter > set the itemDel to "/" > -- gather some items > set the itemDel to old_delimiter > > Mike > > > --- On Sun, 7/31/11, Pete wrote: > > From: Pete > Subject: Re: How do I remove part of a path? > To: "How to use LiveCode" > Date: Sunday, July 31, 2011, 9:36 PM > > replace "/public.html" with empty in . That's > assuming > it's always the same string you want to get rid of. Alternatively: > > set the itemdelimiter to "/" > replace item 4 of with empty > > Pete > Molly's Revenge > > > > > On Sun, Jul 31, 2011 at 6:58 PM, Admin wrote: > > > > > > > Hello all, > > > > Say I have the following path: > > > > > > > > > http://www.travelandworkusa.com/public_html/userfiles/Felker_Mike/robot_trans_noshadow.png > > > > > > How do I get rid of just /public_html ? > > > > Thanks. > > > > Mike > > > > _______________________________________________ > > use-livecode mailing list > > use-livecode at lists.runrev.com > > Please visit this url to subscribe, unsubscribe and manage your > > subscription preferences: > > http://lists.runrev.com/mailman/listinfo/use-livecode > > > > > _______________________________________________ > use-livecode mailing list > use-livecode at lists.runrev.com > Please visit this url to subscribe, unsubscribe and manage your > subscription preferences: > http://lists.runrev.com/mailman/listinfo/use-livecode > _______________________________________________ > use-livecode mailing list > use-livecode at lists.runrev.com > Please visit this url to subscribe, unsubscribe and manage your > subscription preferences: > http://lists.runrev.com/mailman/listinfo/use-livecode > > From slava at lexiconbridge.com Sun Jul 31 23:57:37 2011 From: slava at lexiconbridge.com (Slava Paperno) Date: Sun, 31 Jul 2011 23:57:37 -0400 Subject: lack of Mac OS X intuition: system menu In-Reply-To: <4E362074.3000600@hyperactivesw.com> References: <001201cc4f2d$2c2b7570$84826050$@lexiconbridge.com> <6B257A2D-77E5-4B90-ADF1-97A61BF7F3EF@verizon.net> <000101cc4f3d$f4c65620$de530260$@com> <4E359463.9010505@hyperactivesw.com> <000301cc4fb6$ce6269a0$6b273ce0$@com> <000b01cc4ff3$4cc04ce0$e640e6a0$@com> <4E362074.3000600@hyperactivesw.com> Message-ID: <000c01cc4fff$29f18290$7dd487b0$@com> Then this has to remain a mystery: my stack doesn't set the defaultMenuBar, and when I test it in the Message box, I see that it is the rev file in Toolset. I made a working sample of my two revBrowser problems and posted it on the Mac OS forum in response to Bernd's kind interest. Maybe he'll spot something I've done wrong. If not, I'll post on the dev list. One of these problems doesn't seem to be related to Unicode: revBrowserCallScript() returns "WebUndefined cStringUsingEncoding: unrecognized selector sent to instance." That's a mouthful that tells me nothing, even though all the words make sense... Thanks again, Jacque! Slava > -----Original Message----- > From: use-livecode-bounces at lists.runrev.com [mailto:use-livecode- > bounces at lists.runrev.com] On Behalf Of J. Landman Gay > Sent: Sunday, July 31, 2011 11:42 PM > To: How to use LiveCode > Subject: Re: lack of Mac OS X intuition: system menu > > On 7/31/11 9:32 PM, Slava Paperno wrote: > > Earlier today Jacque said: > > > >>> The IDE will replace the system menu with yours when your stack is > >>> frontmost. If you click on any IDE component (message box, tool > >>> palette, etc) the IDE menus will come back. It's dynamic. > > > > I have not been able to achieve this no matter what value of > "editmenus of > > this stack" or the check boxes in Menu Builder. > > Does your stack set the defaultMenuBar? The defaultMenuBar is handled > by > the IDE, and if your stack sets it, it will override the IDE's > behavior. > > If you need to set it in a standalone, change your script to check the > environment and only set the defaultMenuBar if the environment is not > "development". > > That's all I can think of that would cause the IDE to fail to respond > correctly. > > -- > Jacqueline Landman Gay | jacque at hyperactivesw.com > HyperActive Software | http://www.hyperactivesw.com > > _______________________________________________ > use-livecode mailing list > use-livecode at lists.runrev.com > Please visit this url to subscribe, unsubscribe and manage your > subscription preferences: > http://lists.runrev.com/mailman/listinfo/use-livecode From tsj at unimelb.edu.au Sun Jul 31 23:57:26 2011 From: tsj at unimelb.edu.au (Terry Judd) Date: Mon, 1 Aug 2011 03:57:26 +0000 Subject: MD5 digest of arrays and controls In-Reply-To: References: Message-ID: <1B6C8F3A-9BC7-4F25-96DC-6D39C6EF13A1@unimelb.edu.au> On 01/08/2011, at 01:24 PM, Alejandro Tejada wrote: > Hi all, > > How could I get the MD5digest of an Array and > all the properties of a control? I guess you could use arrayEncode() first and then get an MD5digest of the data. > > For example, I created a control, by script, > and I want to make sure that it's identical > to another control created by a different script. How about copying the objects to the clipboard and then querying the clipboardData["objects"]. Terry... > > How could I compare both controls using a > MD5digest of their relevant properties? > (obviously, their name should be different, > so I do not want to include that property > in the comparision). > > And How could I do this with two global Array > variables? > > I collect data in these two different arrays > using two different scripts and need to be > sure that their data is identical. > > Thanks in advance! > > Al > > _______________________________________________ > use-livecode mailing list > use-livecode at lists.runrev.com > Please visit this url to subscribe, unsubscribe and manage your subscription preferences: > http://lists.runrev.com/mailman/listinfo/use-livecode >